+ All Categories
Home > Documents > JUDITH A. PENNA - · PDF fileSTUDENT’S SOLUTIONS MANUAL JUDITH A. PENNA Indiana...

JUDITH A. PENNA - · PDF fileSTUDENT’S SOLUTIONS MANUAL JUDITH A. PENNA Indiana...

Date post: 03-Feb-2018
Category:
Upload: dangthu
View: 221 times
Download: 0 times
Share this document with a friend
333
STUDENTS SOLUTIONS MANUAL JUDITH A. PENNA Indiana University Purdue University Indianapolis C OLLEGE A LGEBRA : G RAPHS AND M ODELS F IFTH E DITION Marvin L. Bittinger Indiana University Purdue University Indianapolis Judith A. Beecher Indiana University Purdue University Indianapolis David J. Ellenbogen Community College of Vermont Judith A. Penna Indiana University Purdue University Indianapolis Boston Columbus Indianapolis New York San Francisco Upper Saddle River Amsterdam Cape Town Dubai London Madrid Milan Munich Paris Montreal Toronto Delhi Mexico City Sao Paulo Sydney Hong Kong Seoul Singapore Taipei Tokyo
Transcript
Page 1: JUDITH A. PENNA -  · PDF fileSTUDENT’S SOLUTIONS MANUAL JUDITH A. PENNA Indiana University Purdue University Indianapolis COLLEGE ALGEBRA: GRAPHS AND MODELS FIFTH EDITION

STUDENT’S SOLUTIONS MANUAL

JUDITH A. PENNA Indiana University Purdue University Indianapolis

COLLEGE ALGEBRA: GRAPHS AND MODELS

FIFTH EDITION

Marvin L. Bittinger Indiana University Purdue University Indianapolis

Judith A. Beecher Indiana University Purdue University Indianapolis

David J. Ellenbogen Community College of Vermont

Judith A. Penna Indiana University Purdue University Indianapolis

Boston Columbus Indianapolis New York San Francisco Upper Saddle River

Amsterdam Cape Town Dubai London Madrid Milan Munich Paris Montreal Toronto Delhi Mexico City Sao Paulo Sydney Hong Kong Seoul Singapore Taipei Tokyo

Page 2: JUDITH A. PENNA -  · PDF fileSTUDENT’S SOLUTIONS MANUAL JUDITH A. PENNA Indiana University Purdue University Indianapolis COLLEGE ALGEBRA: GRAPHS AND MODELS FIFTH EDITION

The author and publisher of this book have used their best efforts in preparing this book. These efforts include the development, research, and testing of the theories and programs to determine their effectiveness. The author and publisher make no warranty of any kind, expressed or implied, with regard to these programs or the documentation contained in this book. The author and publisher shall not be liable in any event for incidental or consequential damages in connection with, or arising out of, the furnishing, performance, or use of these programs. Reproduced by Pearson from electronic files supplied by the author. Copyright © 2013, 2009, 2005 Pearson Education, Inc. Publishing as Pearson, 75 Arlington Street, Boston, MA 02116. All rights reserved. No part of this publication may be reproduced, stored in a retrieval system, or transmitted, in any form or by any means, electronic, mechanical, photocopying, recording, or otherwise, without the prior written permission of the publisher. Printed in the United States of America. ISBN-13: 978-0-321-79125-2 ISBN-10: 0-321-79125-8 1 2 3 4 5 6 BRR 15 14 13 12 11 www.pearsonhighered.com

Page 3: JUDITH A. PENNA -  · PDF fileSTUDENT’S SOLUTIONS MANUAL JUDITH A. PENNA Indiana University Purdue University Indianapolis COLLEGE ALGEBRA: GRAPHS AND MODELS FIFTH EDITION

Contents

Chapter R . . . . . . . . . . . . . . . . . . . . . . . . . . . 1

Chapter 1 . . . . . . . . . . . . . . . . . . . . . . . . . . 25

Chapter 2 . . . . . . . . . . . . . . . . . . . . . . . . . 63

Chapter 3 . . . . . . . . . . . . . . . . . . . . . . . . . 93

Chapter 4 . . . . . . . . . . . . . . . . . . . . . . . . . . 129

Chapter 5 . . . . . . . . . . . . . . . . . . . . . . . . . 181

Chapter 6 . . . . . . . . . . . . . . . . . . . . . . . . . 215

Chapter 7 . . . . . . . . . . . . . . . . . . . . . . . . . 275

Chapter 8 . . . . . . . . . . . . . . . . . . . . . . . . . 307

Copyright © 2013 Pearson Education, Inc.

Page 4: JUDITH A. PENNA -  · PDF fileSTUDENT’S SOLUTIONS MANUAL JUDITH A. PENNA Indiana University Purdue University Indianapolis COLLEGE ALGEBRA: GRAPHS AND MODELS FIFTH EDITION

0 5�5

0�3 �1

0�2

0 3.8

0 7

Chapter R

Basic Concepts of Algebra

Exercise Set R.1

1. Rational numbers:23, 6, −2.45, 18.4, −11, 3

√27, 5

16, −8

7,

0,√

16

3. Irrational numbers:√

3, 6√

26, 7.151551555 . . . , −√35, 5

√3

(Although there is a pattern in 7.151551555 . . . , there isno repeating block of digits.)

5. Whole numbers: 6, 3√

27, 0,√

16

7. Integers but not natural numbers: −11, 0

9. Rational numbers but not integers:23, −2.45, 18.4, 5

16,

−87

11. This is a closed interval, so we use brackets. Interval no-tation is [−5, 5].

13. This is a half-open interval. We use a parenthesis onthe left and a bracket on the right. Interval notation is(−3,−1].

15. This interval is of unlimited extent in the negative direc-tion, and the endpoint −2 is included. Interval notation is(−∞,−2].

17. This interval is of unlimited extent in the positive direc-tion, and the endpoint 3.8 is not included. Interval nota-tion is (3.8,∞).

19. {x|7 < x}, or {x|x > 7}.This interval is of unlimited extent in the positive directionand the endpoint 7 is not included. Interval notation is(7,∞).

21. The endpoints 0 and 5 are not included in the interval, sowe use parentheses. Interval notation is (0, 5).

23. The endpoint −9 is included in the interval, so we use abracket before the −9. The endpoint −4 is not included,so we use a parenthesis after the −4. Interval notation is[−9,−4).

25. Both endpoints are included in the interval, so we usebrackets. Interval notation is [x, x + h].

27. The endpoint p is not included in the interval, so we use aparenthesis before the p. The interval is of unlimited ex-tent in the positive direction, so we use the infinity symbol∞. Interval notation is (p,∞).

29. Since 6 is an element of the set of natural numbers, thestatement is true.

31. Since 3.2 is not an element of the set of integers, the state-ment is false.

33. Since −115

is an element of the set of rational numbers,the statement is true.

35. Since√

11 is an element of the set of real numbers, thestatement is false.

37. Since 24 is an element of the set of whole numbers, thestatement is false.

39. Since 1.089 is not an element of the set of irrational num-bers, the statement is true.

41. Since every whole number is an integer, the statement istrue.

43. Since every rational number is a real number, the state-ment is true.

45. Since there are real numbers that are not integers, thestatement is false.

47. The sentence 3 + y = y + 3 illustrates the commutativeproperty of addition.

49. The sentence −3 · 1 = −3 illustrates the multiplicativeidentity property.

51. The sentence 5 ·x = x ·5 illustrates the commutative prop-erty of multiplication.

53. The sentence 2(a+b) = (a+b)2 illustrates the commutativeproperty of multiplication.

55. The sentence −6(m+n) = −6(n+m) illustrates the com-mutative property of addition.

57. The sentence 8 · 18

= 1 illustrates the multiplicative inverseproperty.

Copyright © 2013 Pearson Education, Inc.

Page 5: JUDITH A. PENNA -  · PDF fileSTUDENT’S SOLUTIONS MANUAL JUDITH A. PENNA Indiana University Purdue University Indianapolis COLLEGE ALGEBRA: GRAPHS AND MODELS FIFTH EDITION

2 Chapter R: Basic Concepts of Algebra

59. The distance of −8.15 from 0 is 8.15, so | − 8.15| = 8.15.

61. The distance 295 from 0 is 295, so |295| = 295.

63. The distance of −√97 from 0 is

√97, so | − √

97| =√

97.

65. The distance of 0 from 0 is 0, so |0| = 0.

67. The distance of54

from 0 is54, so

∣∣∣54

∣∣∣ = 54.

69. |14 − (−8)| = |14 + 8| = |22| = 22, or

| − 8 − 14| = | − 22| = 22

71. | − 3 − (−9)| = | − 3 + 9| = |6| = 6, or

| − 9 − (−3)| = | − 9 + 3| = | − 6| = 6

73. |12.1 − 6.7| = |5.4| = 5.4, or

|6.7 − 12.1| = | − 5.4| = 5.4

75.∣∣∣∣− 3

4− 15

8

∣∣∣∣ =∣∣∣∣− 6

8− 15

8

∣∣∣∣ =∣∣∣∣− 21

8

∣∣∣∣ = 218, or∣∣∣∣15

8−(− 3

4

)∣∣∣∣ =∣∣∣∣15

8+

34

∣∣∣∣ =∣∣∣∣15

8+

68

∣∣∣∣ =∣∣∣∣21

8

∣∣∣∣ = 218

77. | − 7 − 0| = | − 7| = 7, or

|0 − (−7)| = |0 + 7| = |7| = 7

79. Answers may vary. One such number is0.124124412444 . . . .

81. Answers may vary. Since − 1101

= 0.0099 and

− 1100

= −0.01, one such number is −0.00999.

83. Since 12 +32 = 10, the hypotenuse of a right triangle withlegs of lengths 1 unit and 3 units has a length of

√10 units.

✏✏✏✏✏✏✏✏

3

1c c2 = 12 + 32

c2 = 10

c =√

10

Exercise Set R.2

1. 3−7 =137

(a−m =

1am

, a = 0)

3. Observe that each exponent is negative. We move eachfactor to the other side of the fraction bar and change thesign of each exponent.

x−5

y−4=

y4

x5

5. Observe that each exponent is negative. We move eachfactor to the other side of the fraction bar and change thesign of each exponent.

m−1n−12

t−6=

t6

m1n12, or

t6

mn12

7. 230 = 1 (For any nonzero real number, a0 = 1.)

9. z0 · z7 = z0+7 = z7, or

z0 · z7 = 1 · z7 = z7

11. 58 · 5−6 = 58+(−6) = 52, or 25

13. m−5 ·m5 = m−5+5 = m0 = 1

15. y3 · y−7 = y3+(−7) = y−4, or1y4

17. (x + 3)4(x + 3)−2 = (x + 3)4+(−2) = (x + 3)2

19. 3−3 · 38 · 3 = 3−3+8+1 = 36, or 729

21. 2x3 · 3x2 = 2 · 3 · x3+2 = 6x5

23. (−3a−5)(5a−7) = −3 · 5 · a−5+(−7) = −15a−12, or

− 15a12

25. (6x−3y5)(−7x2y−9) = 6(−7)x−3+2y5+(−9) =

−42x−1y−4, or − 42xy4

27. (2x)4(3x)3 = 24 · x4 · 33 · x3 = 16 · 27 · x4+3 = 432x7

29. (−2n)3(5n)2 = (−2)3n3 · 52n2 = −8 · 25 · n3+2 =

−200n5

31.y35

y31= y35−31 = y4

33.b−7

b12= b−7−12 = b−19, or

1b19

35.x2y−2

x−1y= x2−(−1)y−2−1 = x3y−3, or

x3

y3

37.32x−4y3

4x−5y8=

324x−4−(−5)y3−8 = 8xy−5, or

8xy5

39. (2x2y)4 = 24(x2)4y4 = 16x2·4y4 = 16x8y4

41. (−2x3)5 = (−2)5(x3)5 = (−2)5x3·5 = −32x15

43. (−5c−1d−2)−2 = (−5)−2c−1(−2)d−2(−2) =

c2d4

(−5)2=

c2d4

25

45. (3m4)3(2m−5)4 = 33m12 · 24m−20 =

27 · 16m12+(−20) = 432m−8, or432m8

47.(

2x−3y7

z−1

)3

=(2x−3y7)3

(z−1)3=

23x−9y21

z−3=

8x−9y21

z−3, or

8y21z3

x9

49.(

24a10b−8c7

12a6b−3c5

)−5

= (2a4b−5c2)−5 = 2−5a−20b25c−10,

orb25

32a20c10

Copyright © 2013 Pearson Education, Inc.

Page 6: JUDITH A. PENNA -  · PDF fileSTUDENT’S SOLUTIONS MANUAL JUDITH A. PENNA Indiana University Purdue University Indianapolis COLLEGE ALGEBRA: GRAPHS AND MODELS FIFTH EDITION

Exercise Set R.2 3

51. Convert 16,500,000 to scientific notation.

We want the decimal point to be positioned between the1 and the 6, so we move it 7 places to the left. Since16,500,000 is greater than 10, the exponent must be posi-tive.

16, 500, 000 = 1.65 × 107

53. Convert 0.000000437 to scientific notation.

We want the decimal point to be positioned between the4 and the 3, so we move it 7 places to the right. Since0.000000437 is a number between 0 and 1, the exponentmust be negative.

0.000000437 = 4.37 × 10−7

55. Convert 234,600,000,000 to scientific notation. We wantthe decimal point to be positioned between the 2 and the3, so we move it 11 places to the left. Since 234,600,000,000is greater than 10, the exponent must be positive.

234, 600, 000, 000 = 2.346 × 1011

57. Convert 0.00104 to scientific notation. We want the deci-mal point to be positioned between the 1 and the last 0, sowe move it 3 places to the right. Since 0.00104 is a numberbetween 0 and 1, the exponent must be negative.

0.00104 = 1.04 × 10−3

59. Convert 0.00000000000000000000000000167 to scientificnotation.

We want the decimal point to be positioned between the1 and the 6, so we move it 27 places to the right. Since0.00000000000000000000000000167 is a number between 0and 1, the exponent must be negative.

0.00000000000000000000000000167 = 1.67 × 10−27

61. Convert 7.6 × 105 to decimal notation.

The exponent is positive, so the number is greater than10. We move the decimal point 5 places to the right.

7.6 × 105 = 760, 000

63. Convert 1.09 × 10−7 to decimal notation.

The exponent is negative, so the number is between 0 and1. We move the decimal point 7 places to the left.

1.09 × 10−7 = 0.000000109

65. Convert 3.496 × 1010 to decimal notation.

The exponent is positive, so the number is greater than10. We move the decimal point 10 places to the right.

3.496 × 1010 = 34, 960, 000, 000

67. Convert 5.41 × 10−8 to decimal notation.

The exponent is negative, so the number is between 0 and1. We move the decimal point 8 places to the left.

5.41 × 10−8 = 0.0000000541

69. Convert 2.319 × 108 to decimal notation.

The exponent is positive, so the number is greater than10. We move the decimal point 8 places to the right.

2.319 × 108 = 231, 900, 000

71. (4.2 × 107)(3.2 × 10−2)

= (4.2 × 3.2) × (107 × 10−2)

= 13.44 × 105 This is not scientific notation.

= (1.344 × 10) × 105

= 1.344 × 106 Writing scientific notation

73. (2.6 × 10−18)(8.5 × 107)

= (2.6 × 8.5) × (10−18 × 107)

= 22.1 × 10−11 This is not scientific notation.

= (2.21 × 10) × 10−11

= 2.21 × 10−10

75. 6.4 × 10−7

8.0 × 106=

6.48.0

× 10−7

106

= 0.8 × 10−13 This is not scientificnotation.

= (8 × 10−1) × 10−13

= 8 × 10−14 Writing scientificnotation

77. 1.8 × 10−3

7.2 × 10−9

=1.87.2

× 10−3

10−9

= 0.25 × 106 This is not scientific notation.

= (2.5 × 10−1) × 106

= 2.5 × 105

79. The average number of pieces of trash per mile is the totalnumber of pieces of trash divided by the number of miles.

51.2 billion76 million

=5.12 × 1010

7.6 × 107

≈ 0.6737 × 103

≈ (6.737 × 10−1) × 103

≈ 6.737 × 102

On average, there are about 6.737× 102 pieces of trash oneach mile of roadway.

81. The number of people per square mile is the total numberof people divided by the number of square miles.

38, 0000.75

=3.8 × 104

7.5 × 10−1

≈ 0.50667 × 105

≈ (5.0667 × 10−1) × 105

≈ 5.0667 × 104

There are about 5.0667 × 104 people per square mile.

83. We multiply the number of light years by the number ofmiles in a light year.

4.22 × 5.88 × 1012 = 24.8136 × 1012

= (2.48136 × 10) × 1012

= 2.48136 × 1013

The distance from Earth to Alpha Centauri C is2.48136 × 1013 mi.

Copyright © 2013 Pearson Education, Inc.

Page 7: JUDITH A. PENNA -  · PDF fileSTUDENT’S SOLUTIONS MANUAL JUDITH A. PENNA Indiana University Purdue University Indianapolis COLLEGE ALGEBRA: GRAPHS AND MODELS FIFTH EDITION

4 Chapter R: Basic Concepts of Algebra

85. First find the number of seconds in 1 hour:

1 hour = 1 hr✧ × 60 min✦1 hr✧

× 60 sec1 min✦ = 3600 sec

The number of disintegrations produced in 1 hour is thenumber of disintegrations per second times the number ofseconds in 1 hour.

37 billion × 3600

= 37, 000, 000, 000 × 3600

= 3.7 × 1010 × 3.6 × 103 Writing scientificnotation

= (3.7 × 3.6) × (1010 × 103)

= 13.32 × 1013 Multiplying

= (1.332 × 10) × 1013

= 1.332 × 1014

One gram of radium produces 1.332× 1014 disintegrationsin 1 hour.

87. = 5 · 3 + 8 · 32 + 4(6 − 2)

= 5 · 3 + 8 · 32 + 4 · 4 Working inside parentheses

= 5 · 3 + 8 · 9 + 4 · 4 Evaluating 32

= 15 + 72 + 16 Multiplying

= 87 + 16 Adding in order

= 103 from left to right

89. 16 ÷ 4 · 4 ÷ 2 · 256

= 4 · 4 ÷ 2 · 256 Multiplying and dividingin order from left to right

= 16 ÷ 2 · 256

= 8 · 256

= 2048

91. 4(8 − 6)2 − 4 · 3 + 2 · 831 + 190

=4 · 22 − 4 · 3 + 2 · 8

3 + 1Calculating in thenumerator and inthe denominator

=4 · 4 − 4 · 3 + 2 · 8

4

=16 − 12 + 16

4

=4 + 16

4

=204

= 5

93. Since interest is compounded semiannually, n = 2. Substi-tute $3225 for P , 3.1% or 0.031 for i, 2 for n, and 4 for tin the compound interest formula.

A = P(1 +

i

n

)nt= $3225

(1 +

0.0312

)2·4Substituting

= $3225(1 + 0.0155)2·4 Dividing

= $3225(1.0155)2·4 Adding

= $3225(1.0155)8 Multiplying 2 and 4

≈ $3225(1.130939628) Evaluating theexponential expression

≈ $3647.2803 Multiplying

≈ $3647.28 Rounding to the nearest cent

95. Since interest is compounded quarterly, n = 4. Substitute$4100 for P , 2.3% or 0.023 for i, 4 for n, and 6 for t in thecompound interest formula.

A = P(1 +

i

n

)nt= $4100

(1 +

0.0234

)4·6Substituting

= $4100(1 + 0.00575)4·6 Dividing

= $4100(1.00575)4·6 Adding

= $4100(1.00575)24 Multiplying 4 and 6

≈ $4100(1.147521919) Evaluating theexponential expression

≈ $4704.839868 Multiplying

≈ $4704.84 Rounding to the nearest cent

97. Substitute $250 for P , 0.05 for r and 27 for t and performthe resulting computation.

S = P

(1 +

r

12

)12·t− 1

r

12

= $250

(1 +

0.0512

)12·27− 1

0.0512

≈ $170, 797.30

99. Substitute $120,000 for S, 0.03 for r, and 18 for t and solvefor P .

S = P

(1 +

r

12

)12·t− 1

r

12

$120, 000 = P

(1 +

0.0312

)12·18− 1

0.0312

$120, 000 = P

[(1.0025)216 − 1

0.0025

]$120, 000 ≈ P (285.94035)

$419.67 ≈ P

Copyright © 2013 Pearson Education, Inc.

Page 8: JUDITH A. PENNA -  · PDF fileSTUDENT’S SOLUTIONS MANUAL JUDITH A. PENNA Indiana University Purdue University Indianapolis COLLEGE ALGEBRA: GRAPHS AND MODELS FIFTH EDITION

Exercise Set R.3 5

101. (xt · x3t)2 = (x4t)2 = x4t·2 = x8t

103. (ta+x · tx−a)4 = (t2x)4 = t2x·4 = t8x

105.[ (3xayb)3

(−3xayb)2]2

=[27x3ay3b

9x2ay2b

]2=[3xayb

]2= 9x2ay2b

Exercise Set R.3

1. 7x3 − 4x2 + 8x + 5 = 7x3 + (−4x2) + 8x + 5

Terms: 7x3, −4x2, 8x, 5

The degree of the term of highest degree, 7x3, is 3. Thus,the degree of the polynomial is 3.

3. 3a4b− 7a3b3 + 5ab− 2 = 3a4b + (−7a3b3) + 5ab + (−2)

Terms: 3a4b, −7a3b3, 5ab, −2

The degrees of the terms are 5, 6, 2, and, 0, respectively,so the degree of the polynomial is 6.

5. (3ab2 − 4a2b− 2ab + 6)+

(−ab2 − 5a2b + 8ab + 4)

= (3 − 1)ab2 + (−4 − 5)a2b + (−2 + 8)ab + (6 + 4)

= 2ab2 − 9a2b + 6ab + 10

7. (2x + 3y + z − 7) + (4x− 2y − z + 8)+

(−3x + y − 2z − 4)

= (2 + 4 − 3)x + (3 − 2 + 1)y + (1 − 1 − 2)z+

(−7 + 8 − 4)

= 3x + 2y − 2z − 3

9. (3x2 − 2x− x3 + 2) − (5x2 − 8x− x3 + 4)

= (3x2 − 2x− x3 + 2) + (−5x2 + 8x + x3 − 4)

= (3 − 5)x2 + (−2 + 8)x + (−1 + 1)x3 + (2 − 4)

= −2x2 + 6x− 2

11. (x4 − 3x2 + 4x) − (3x3 + x2 − 5x + 3)

= (x4 − 3x2 + 4x) + (−3x3 − x2 + 5x− 3)

= x4 − 3x3 + (−3 − 1)x2 + (4 + 5)x− 3

= x4 − 3x3 − 4x2 + 9x− 3

13. (3a2)(−7a4) = [3(−7)](a2 · a4)

= −21a6

15. (6xy3)(9x4y2) = (6 · 9)(x · x4)(y3 · y2)

= 54x5y5

17. (a− b)(2a3 − ab + 3b2)

= (a− b)(2a3) + (a− b)(−ab) + (a− b)(3b2)

Using the distributive property

= 2a4 − 2a3b− a2b + ab2 + 3ab2 − 3b3

Using the distributive propertythree more times

= 2a4 − 2a3b− a2b + 4ab2 − 3b3 Collecting liketerms

19. (y − 3)(y + 5)

= y2 + 5y − 3y − 15 Using FOIL

= y2 + 2y − 15 Collecting like terms

21. (x + 6)(x + 3)

= x2 + 3x + 6x + 18 Using FOIL

= x2 + 9x + 18 Collecting like terms

23. (2a + 3)(a + 5)

= 2a2 + 10a + 3a + 15 Using FOIL

= 2a2 + 13a + 15 Collecting like terms

25. (2x + 3y)(2x + y)

= 4x2 + 2xy + 6xy + 3y2 Using FOIL

= 4x2 + 8xy + 3y2

27. (x + 3)2

= x2 + 2 · x · 3 + 32

[(A + B)2 = A2 + 2AB + B2]

= x2 + 6x + 9

29. (y − 5)2

= y2 − 2 · y · 5 + 52

[(A−B)2 = A2 − 2AB + B2]

= y2 − 10y + 25

31. (5x− 3)2

= (5x)2 − 2 · 5x · 3 + 32

[(A−B)2 = A2 − 2AB +B2]

= 25x2 − 30x + 9

33. (2x + 3y)2

= (2x)2 + 2(2x)(3y) + (3y)2

[(A+B)2 = A2+2AB+B2]

= 4x2 + 12xy + 9y2

35. (2x2 − 3y)2

= (2x2)2 − 2(2x2)(3y) + (3y)2

[(A−B)2 = A2 − 2AB + B2]= 4x4 − 12x2y + 9y2

37. (n + 6)(n− 6)

= n2 − 62 [(A + B)(A−B) = A2 −B2]

= n2 − 36

39. (3y + 4)(3y − 4)

= (3y)2 − 42 [(A + B)(A−B) = A2 −B2]

= 9y2 − 16

41. (3x− 2y)(3x + 2y)

= (3x)2 − (2y)2 [(A−B)(A+B) = A2−B2]

= 9x2 − 4y2

Copyright © 2013 Pearson Education, Inc.

Page 9: JUDITH A. PENNA -  · PDF fileSTUDENT’S SOLUTIONS MANUAL JUDITH A. PENNA Indiana University Purdue University Indianapolis COLLEGE ALGEBRA: GRAPHS AND MODELS FIFTH EDITION

6 Chapter R: Basic Concepts of Algebra

43. (2x + 3y + 4)(2x + 3y − 4)

= [(2x + 3y) + 4][(2x + 3y) − 4]

= (2x + 3y)2 − 42

= 4x2 + 12xy + 9y2 − 16

45. (x + 1)(x− 1)(x2 + 1)

= (x2 − 1)(x2 + 1)

= x4 − 1

47. (an + bn)(an − bn) = (an)2 − (bn)2

= a2n − b2n

49. (an + bn)2 = (an)2 + 2 · an · bn + (bn)2

= a2n + 2anbn + b2n

51. (x− 1)(x2 + x + 1)(x3 + 1)

= [(x− 1)x2 + (x− 1)x + (x− 1) · 1](x3 + 1)

= (x3 − x2 + x2 − x + x− 1)(x3 + 1)

= (x3 − 1)(x3 + 1)

= (x3)2 − 12

= x6 − 1

53. (xa−b)a+b

= x(a−b)(a+b)

= xa2−b2

55. (a + b + c)2

= (a + b + c)(a + b + c)

= (a + b + c)(a) + (a + b + c)(b) + (a + b + c)(c)

= a2 + ab + ac + ab + b2 + bc + ac + bc + c2

= a2 + b2 + c2 + 2ab + 2ac + 2bc

Exercise Set R.4

1. 3x + 18 = 3 · x + 3 · 6 = 3(x + 6)

3. 2z3 − 8z2 = 2z2 · z − 2z2 · 4 = 2z2(z − 4)

5. 4a2 − 12a + 16 = 4 · a2 − 4 · 3a + 4 · 4 = 4(a2 − 3a + 4)

7. a(b− 2) + c(b− 2) = (b− 2)(a + c)

9. 3x3 − x2 + 18x− 6

= x2(3x− 1) + 6(3x− 1)

= (3x− 1)(x2 + 6)

11. y3 − y2 + 2y − 2

= y2(y − 1) + 2(y − 1)

= (y − 1)(y2 + 2)

13. 24x3 − 36x2 + 72x− 108

= 12(2x3 − 3x2 + 6x− 9)

= 12[x2(2x− 3) + 3(2x− 3)]

= 12(2x− 3)(x2 + 3)

15. x3 − x2 − 5x + 5

= x2(x− 1) − 5(x− 1)

= (x− 1)(x2 − 5)

17. a3 − 3a2 − 2a + 6

= a2(a− 3) − 2(a− 3)

= (a− 3)(a2 − 2)

19. w2 − 7w + 10

We look for two numbers with a product of 10 and a sumof −7. By trial, we determine that they are −5 and −2.

w2 − 7w + 10 = (w − 5)(w − 2)

21. x2 + 6x + 5

We look for two numbers with a product of 5 and a sumof 6. By trial, we determine that they are 1 and 5.

x2 + 6x + 5 = (x + 1)(x + 5)

23. t2 + 8t + 15

We look for two numbers with a product of 15 and a sumof 8. By trial, we determine that they are 3 and 5.

t2 + 8t + 15 = (t + 3)(t + 5)

25. x2 − 6xy − 27y2

We look for two numbers with a product of −27 and a sumof −6. By trial, we determine that they are 3 and −9.

x2 − 6xy − 27y2 = (x + 3y)(x− 9y)

27. 2n2 − 20n− 48 = 2(n2 − 10n− 24)

Now factor n2−10n−24. We look for two numbers with aproduct of −24 and a sum of −10. By trial, we determinethat they are 2 and −12. Then n2 − 10n− 24 =(n + 2)(n − 12). We must include the common factor, 2,to have a factorization of the original trinomial.

2n2 − 20n− 48 = 2(n + 2)(n− 12)

29. y2 − 4y − 21

We look for two numbers with a product of −21 and a sumof −4. By trial, we determine that they are 3 and −7.

y2 − 4y − 21 = (y + 3)(y − 7)

31. y4 − 9y3 + 14y2 = y2(y2 − 9y + 14)

Now factor y2 − 9y + 14. Look for two numbers with aproduct of 14 and a sum of −9. The numbers are −2 and−7. Then y2 − 9y + 14 = (y − 2)(y − 7). We must includethe common factor, y2, in order to have a factorization ofthe original trinomial.

y4 − 9y3 + 14y2 = y2(y − 2)(y − 7)

33. 2x3 − 2x2y − 24xy2 = 2x(x2 − xy − 12y2)

Now factor x2 − xy − 12y2. Look for two numbers with aproduct of −12 and a sum of −1. The numbers are −4 and3. Then x2−xy−12y2 = (x−4y)(x+3y). We must includethe common factor, 2x, in order to have a factorization ofthe original trinomial.

2x3 − 2x2y − 24xy2 = 2x(x− 4y)(x + 3y)

Copyright © 2013 Pearson Education, Inc.

Page 10: JUDITH A. PENNA -  · PDF fileSTUDENT’S SOLUTIONS MANUAL JUDITH A. PENNA Indiana University Purdue University Indianapolis COLLEGE ALGEBRA: GRAPHS AND MODELS FIFTH EDITION

Exercise Set R.4 7

35. 2n2 + 9n− 56

We use the FOIL method.

1. There is no common factor other than 1 or −1.

2. The factorization must be of the form(2n+ )(n+ ).

3. Factor the constant term, −56. The possibilitiesare −1 ·56, 1(−56), −2 ·28, 2(−28), −4 ·16, 4(−16),−7 · 8, and 7(−8). The factors can be written inthe opposite order as well: 56(−1), −56 · 1, 28(−2),−28 · 2, 16(−4), −16 · 4, 8(−7), and −8 · 7.

4. Find a pair of factors for which the sum of the out-side and the inside products is the middle term,9n. By trial, we determine that the factorizationis (2n− 7)(n + 8).

37. 12x2 + 11x + 2

We use the grouping method.

1. There is no common factor other than 1 or −1.

2. Multiply the leading coefficient and the constant:12 · 2 = 24.

3. Try to factor 24 so that the sum of the factors is thecoefficient of the middle term, 11. The factors wewant are 3 and 8.

4. Split the middle term using the numbers found instep (3):

11x = 3x + 8x

5. Factor by grouping.

12x2 + 11x + 2 = 12x2 + 3x + 8x + 2

= 3x(4x + 1) + 2(4x + 1)

= (4x + 1)(3x + 2)

39. 4x2 + 15x + 9

We use the FOIL method.

1. There is no common factor other than 1 or −1.

2. The factorization must be of the form(4x+ )(x+ ) or (2x+ )(2x+ ).

3. Factor the constant term, 9. The possibilities are1 · 9, −1(−9), 3 · 3, and −3(−3). The first two pairsof factors can be written in the opposite order aswell: 9 · 1, −9(−1).

4. Find a pair of factors for which the sum of the out-side and the inside products is the middle term,15x. By trial, we determine that the factorizationis (4x + 3)(x + 3).

41. 2y2 + y − 6

We use the grouping method.

1. There is no common factor other than 1 or −1.

2. Multiply the leading coefficient and the constant:2(−6) = −12.

3. Try to factor −12 so that the sum of the factors isthe coefficient of the middle term, 1. The factors wewant are 4 and −3.

4. Split the middle term using the numbers found instep (3):

y = 4y − 3y

5. Factor by grouping.

2y2 + y − 6 = 2y2 + 4y − 3y − 6

= 2y(y + 2) − 3(y + 2)

= (y + 2)(2y − 3)

43. 6a2 − 29ab + 28b2

We use the FOIL method.

1. There is no common factor other than 1 or −1.

2. The factorization must be of the form(6x+ )(x+ ) or (3x+ )(2x+ ).

3. Factor the coefficient of the last term, 28. The pos-sibilities are 1 · 28, −1(−28), 2 · 14, −2(−14), 4 · 7,and −4(−7). The factors can be written in the op-posite order as well: 28·1, −28(−1), 14·2, −14(−2),7 · 4, and −7(−4).

4. Find a pair of factors for which the sum of the out-side and the inside products is the middle term,−29. Observe that the second term of each bino-mial factor will contain a factor of b. By trial, wedetermine that the factorization is (3a−4b)(2a−7b).

45. 12a2 − 4a− 16

We will use the grouping method.

1. Factor out the common factor, 4.

12a2 − 4a− 16 = 4(3a2 − a− 4)

2. Now consider 3a2 − a − 4. Multiply the leadingcoefficient and the constant: 3(−4) = −12.

3. Try to factor −12 so that the sum of the factors isthe coefficient of the middle term, −1. The factorswe want are −4 and 3.

4. Split the middle term using the numbers found instep (3):

−a = −4a + 3a

5. Factor by grouping.

3a2 − a− 4 = 3a2 − 4a + 3a− 4

= a(3a− 4) + (3a− 4)

= (3a− 4)(a + 1)

We must include the common factor to get a factor-ization of the original trinomial.

12a2 − 4a− 16 = 4(3a− 4)(a + 1)

47. z2 − 81 = z2 − 92 = (z + 9)(z − 9)

49. 16x2 − 9 = (4x)2 − 32 = (4x + 3)(4x− 3)

51. 6x2 − 6y2 = 6(x2 − y2) = 6(x + y)(x− y)

53. 4xy4 − 4xz2 = 4x(y4 − z2)

= 4x[(y2)2 − z2]

= 4x(y2 + z)(y2 − z)

Copyright © 2013 Pearson Education, Inc.

Page 11: JUDITH A. PENNA -  · PDF fileSTUDENT’S SOLUTIONS MANUAL JUDITH A. PENNA Indiana University Purdue University Indianapolis COLLEGE ALGEBRA: GRAPHS AND MODELS FIFTH EDITION

8 Chapter R: Basic Concepts of Algebra

55. 7pq4 − 7py4 = 7p(q4 − y4)

= 7p[(q2)2 − (y2)2]

= 7p(q2 + y2)(q2 − y2)

= 7p(q2 + y2)(q + y)(q − y)

57. x2 + 12x + 36 = x2 + 2 · x · 6 + 62

= (x + 6)2

59. 9z2 − 12z + 4 = (3z)2 − 2 · 3z · 2 + 22 = (3z − 2)2

61. 1 − 8x + 16x2 = 12 − 2 · 1 · 4x + (4x)2

= (1 − 4x)2

63. a3 + 24a2 + 144a

= a(a2 + 24a + 144)

= a(a2 + 2 · a · 12 + 122)

= a(a + 12)2

65. 4p2 − 8pq + 4q2

= 4(p2 − 2pq + q2)

= 4(p− q)2

67. x3 + 64 = x3 + 43

= (x + 4)(x2 − 4x + 16)

69. m3 − 216 = m3 − 63

= (m− 6)(m2 + 6m + 36)

71. 8t3 + 8 = 8(t3 + 1)

= 8(t3 + 13)

= 8(t + 1)(t2 − t + 1)

73. 3a5 − 24a2 = 3a2(a3 − 8)

= 3a2(a3 − 23)

= 3a2(a− 2)(a2 + 2a + 4)

75. t6 + 1 = (t2)3 + 13

= (t2 + 1)(t4 − t2 + 1)

77. 18a2b− 15ab2 = 3ab · 6a− 3ab · 5b= 3ab(6a− 5b)

79. x3 − 4x2 + 5x− 20 = x2(x− 4) + 5(x− 4)

= (x− 4)(x2 + 5)

81. 8x2 − 32 = 8(x2 − 4)

= 8(x + 2)(x− 2)

83. 4y2 − 5

There are no common factors. We might try to factorthis polynomial as a difference of squares, but there is nointeger which yields 5 when squared. Thus, the polynomialis prime.

85. m2 − 9n2 = m2 − (3n)2

= (m + 3n)(m− 3n)

87. x2 + 9x + 20

We look for two numbers with a product of 20 and a sumof 9. They are 4 and 5.

x2 + 9x + 20 = (x + 4)(x + 5)

89. y2 − 6y + 5

We look for two numbers with a product of 5 and a sumof −6. They are −5 and −1.

y2 − 6y + 5 = (y − 5)(y − 1)

91. 2a2 + 9a + 4

We use the FOIL method.

1. There is no common factor other than 1 or −1.

2. The factorization must be of the form(2a+ )(a+ ).

3. Factor the constant term, 4. The possibilities are1 ·4, −1(−4), and 2 ·2. The first two pairs of factorscan be written in the opposite order as well: 4 · 1,−4(−1).

4. Find a pair of factors for which the sum of the out-side and the inside products is the middle term,9a. By trial, we determine that the factorizationis (2a + 1)(a + 4).

93. 6x2 + 7x− 3

We use the grouping method.

1. There is no common factor other than 1 or −1.

2. Multiply the leading coefficient and the constant:6(−3) = −18.

3. Try to factor −18 so that the sum of the factors isthe coefficient of the middle term, 7. The factors wewant are 9 and −2.

4. Split the middle term using the numbers found instep (3):

7x = 9x− 2x

5. Factor by grouping.

6x2 + 7x− 3 = 6x2 + 9x− 2x− 3

= 3x(2x + 3) − (2x + 3)

= (2x + 3)(3x− 1)

95. y2 − 18y + 81 = y2 − 2 · y · 9 + 92

= (y − 9)2

97. 9z2 − 24z + 16 = (3z)2 − 2 · 3z · 4 + 42

= (3z − 4)2

99. x2y2 − 14xy + 49 = (xy)2 − 2 · xy · 7 + 72

= (xy − 7)2

101. 4ax2 + 20ax− 56a = 4a(x2 + 5x− 14)

= 4a(x + 7)(x− 2)

103. 3z3 − 24 = 3(z3 − 8)

= 3(z3 − 23)

= 3(z − 2)(z2 + 2z + 4)

Copyright © 2013 Pearson Education, Inc.

Page 12: JUDITH A. PENNA -  · PDF fileSTUDENT’S SOLUTIONS MANUAL JUDITH A. PENNA Indiana University Purdue University Indianapolis COLLEGE ALGEBRA: GRAPHS AND MODELS FIFTH EDITION

Exercise Set R.5 9

105. 16a7b + 54ab7

= 2ab(8a6 + 27b6)

= 2ab[(2a2)3 + (3b2)3]

= 2ab(2a2 + 3b2)(4a4 − 6a2b2 + 9b4)

107. y3 − 3y2 − 4y + 12

= y2(y − 3) − 4(y − 3)

= (y − 3)(y2 − 4)

= (y − 3)(y + 2)(y − 2)

109. x3 − x2 + x− 1

= x2(x− 1) + (x− 1)

= (x− 1)(x2 + 1)

111. 5m4 − 20 = 5(m4 − 4)

= 5(m2 + 2)(m2 − 2)

113. 2x3 + 6x2 − 8x− 24

= 2(x3 + 3x2 − 4x− 12)

= 2[x2(x + 3) − 4(x + 3)]

= 2(x + 3)(x2 − 4)

= 2(x + 3)(x + 2)(x− 2)

115. 4c2 − 4cd− d2 = (2c)2 − 2 · 2c · d− d2

= (2c− d)2

117. m6 + 8m3 − 20 = (m3)2 + 8m3 − 20

We look for two numbers with a product of −20 and a sumof 8. They are 10 and −2.

m6 + 8m3 − 20 = (m3 + 10)(m3 − 2)

119. p− 64p4 = p(1 − 64p3)

= p[13 − (4p)3]

= p(1 − 4p)(1 + 4p + 16p2)

121. y4 − 84 + 5y2

= y4 + 5y2 − 84

= u2 + 5u− 84 Substituting u for y2

= (u + 12)(u− 7)

= (y2 + 12)(y2 − 7) Substituting y2 for u

123. y2 − 849

+27y = y2 +

27y − 8

49

=(y +

47

)(y − 2

7

)

125. x2 + 3x +94

= x2 + 2 · x · 32

+(

32

)2

=(x +

32

)2

127. x2 − x +14

= x2 − 2 · x · 12

+(

12

)2

=(x− 1

2

)2

129. (x + h)3 − x3

= [(x + h) − x][(x + h)2 + x(x + h) + x2]

= (x + h− x)(x2 + 2xh + h2 + x2 + xh + x2)

= h(3x2 + 3xh + h2)

131. (y − 4)2 + 5(y − 4) − 24

= u2 + 5u− 24 Substituting u for y − 4

= (u + 8)(u− 3)

= (y − 4 + 8)(y − 4 − 3) Substituting y − 4for u

= (y + 4)(y − 7)

133. x2n + 5xn − 24 = (xn)2 + 5xn − 24

= (xn + 8)(xn − 3)

135. x2 + ax + bx + ab = x(x + a) + b(x + a)

= (x + a)(x + b)

137. 25y2m − (x2n − 2xn + 1)

= (5ym)2 − (xn − 1)2

= [5ym + (xn − 1)][5ym − (xn − 1)]

= (5ym + xn − 1)(5ym − xn + 1)

139. (y − 1)4 − (y − 1)2

= (y − 1)2[(y − 1)2 − 1]

= (y − 1)2[y2 − 2y + 1 − 1]

= (y − 1)2(y2 − 2y)

= y(y − 1)2(y − 2)

Exercise Set R.5

1. x− 5 = 7

x = 12 Adding 5The solution is 12.

3. 3x + 4 = −8

3x = −12 Subtracting 4

x = −4 Dividing by 3The solution is −4.

5. 5y − 12 = 3

5y = 15 Adding 12

y = 3 Dividing by 5The solution is 3.

7. 6x− 15 = 45

6x = 60 Adding 15

x = 10 Dividing by 6The solution is 10.

9. 5x− 10 = 45

5x = 55 Adding 10

x = 11 Dividing by 5The solution is 11.

Copyright © 2013 Pearson Education, Inc.

Page 13: JUDITH A. PENNA -  · PDF fileSTUDENT’S SOLUTIONS MANUAL JUDITH A. PENNA Indiana University Purdue University Indianapolis COLLEGE ALGEBRA: GRAPHS AND MODELS FIFTH EDITION

10 Chapter R: Basic Concepts of Algebra

11. 9t + 4 = −5

9t = −9 Subtracting 4

t = −1 Dividing by 9

The solution is −1.

13. 8x + 48 = 3x− 12

5x + 48 = −12 Subtracting 3x

5x = −60 Subtracting 48

x = −12 Dividing by 5

The solution is −12.

15. 7y − 1 = 23 − 5y

12y − 1 = 23 Adding 5y

12y = 24 Adding 1

y = 2 Dividing by 12

The solution is 2.

17. 3x− 4 = 5 + 12x

−9x− 4 = 5 Subtracting 12x

−9x = 9 Adding 4

x = −1 Dividing by −9

The solution is −1.

19. 5 − 4a = a− 13

5 − 5a = −13 Subtracting a

−5a = −18 Subtracting 5

a =185

Dividing by −5

The solution is185

.

21. 3m− 7 = −13 + m

2m− 7 = −13 Subtracting m

2m = −6 Adding 7

m = −3 Dividing by 2

The solution is −3.

23. 11 − 3x = 5x + 3

11 − 8x = 3 Subtracting 5x

−8x = −8 Subtracting 11

x = 1

The solution is 1.

25. 2(x + 7) = 5x + 14

2x + 14 = 5x + 14

−3x + 14 = 14 Subtracting 5x

−3x = 0 Subtracting 14

x = 0

The solution is 0.

27. 24 = 5(2t + 5)

24 = 10t + 25

−1 = 10t Subtracting 25

− 110

= t Dividing by 10

The solution is − 110

.

29. 5y − 4(2y − 10) = 25

5y − 8y + 40 = 25

−3y + 40 = 25 Collecting like terms

−3y = −15 Subtracting 40

y = 5 Dividing by −3

The solution is 5.

31. 7(3x + 6) = 11 − (x + 2)

21x + 42 = 11 − x− 2

21x + 42 = 9 − x Collecting like terms

22x + 42 = 9 Adding x

22x = −33 Subtracting 42

x = −32

Dividing by 22

The solution is −32.

33. 4(3y − 1) − 6 = 5(y + 2)

12y − 4 − 6 = 5y + 10

12y − 10 = 5y + 10 Collecting like terms

7y − 10 = 10 Subtracting 5y

7y = 20 Adding 10

y =207

Dividing by 7

The solution is207

.

35. x2 + 3x− 28 = 0

(x + 7)(x− 4) = 0 Factoring

x + 7 = 0 or x− 4 = 0 Principle of zero products

x = −7 or x = 4

The solutions are −7 and 4.

37. x2 + 5x = 0

x(x + 5) = 0 Factoring

x = 0 or x + 5 = 0 Principle of zero products

x = 0 or x = −5

The solutions are 0 and −5.

39. y2 + 6y + 9 = 0

(y + 3)(y + 3) = 0

y + 3 = 0 or y + 3 = 0

y = −3 or y = −3

The solution is −3.

Copyright © 2013 Pearson Education, Inc.

Page 14: JUDITH A. PENNA -  · PDF fileSTUDENT’S SOLUTIONS MANUAL JUDITH A. PENNA Indiana University Purdue University Indianapolis COLLEGE ALGEBRA: GRAPHS AND MODELS FIFTH EDITION

Exercise Set R.5 11

41. x2 + 100 = 20x

x2 − 20x + 100 = 0 Subtracting 20x

(x− 10)(x− 10) = 0

x− 10 = 0 or x− 10 = 0

x = 10 or x = 10

The solution is 10.

43. x2 − 4x− 32 = 0

(x− 8)(x + 4) = 0

x− 8 = 0 or x + 4 = 0

x = 8 or x = −4

The solutions are 8 and −4.

45. 3y2 + 8y + 4 = 0

(3y + 2)(y + 2) = 0

3y + 2 = 0 or y + 2 = 0

3y = −2 or y = −2

y = −23

or y = −2

The solutions are −23

and −2.

47. 12z2 + z = 6

12z2 + z − 6 = 0

(4z + 3)(3z − 2) = 0

4z + 3 = 0 or 3z − 2 = 0

4z = −3 or 3z = 2

z = −34

or z =23

The solutions are −34

and23.

49. 12a2 − 28 = 5a

12a2 − 5a− 28 = 0

(3a + 4)(4a− 7) = 0

3a + 4 = 0 or 4a− 7 = 0

3a = −4 or 4a = 7

a = −43

or a =74

The solutions are −43

and74.

51. 14 = x(x− 5)

14 = x2 − 5x

0 = x2 − 5x− 14

0 = (x− 7)(x + 2)

x− 7 = 0 or x + 2 = 0

x = 7 or x = −2

The solutions are 7 and −2.

53. x2 − 36 = 0

(x + 6)(x− 6) = 0

x + 6 = 0 or x− 6 = 0

x = −6 or x = 6

The solutions are −6 and 6.

55. z2 = 144

z2 − 144 = 0

(z + 12)(z − 12) = 0

z + 12 = 0 or z − 12 = 0

z = −12 or z = 12

The solutions are −12 and 12.

57. 2x2 − 20 = 0

2x2 = 20

x2 = 10

x =√

10 or x = −√10 Principle of square roots

The solutions are√

10 and −√10, or ±√

10.

59. 6z2 − 18 = 0

6z2 = 18

z2 = 3

z =√

3 or z = −√3

The solutions are√

3 and −√3, or ±√

3.

61. A =12bh

2A = bh Multiplying by 2 on both sides2Ah

= b Dividing by h on both sides

63. P = 2l + 2w

P − 2l = 2w Subtracting 2l on both sides

P − 2l2

= w Dividing by 2 on both sides

65. A =12h(b1 + b2)

2Ah

= b1 + b2 Multiplying by2h

on both sides

2Ah

− b1 = b2, or

2A− b1h

h= b2

67. V =43πr3

3V = 4πr3 Multiplying by 3 on both sides3V4r3

= π Dividing by 4r3 on both sides

Copyright © 2013 Pearson Education, Inc.

Page 15: JUDITH A. PENNA -  · PDF fileSTUDENT’S SOLUTIONS MANUAL JUDITH A. PENNA Indiana University Purdue University Indianapolis COLLEGE ALGEBRA: GRAPHS AND MODELS FIFTH EDITION

12 Chapter R: Basic Concepts of Algebra

69. F =95C + 32

F − 32 =95C Subtracting 32 on both sides

59(F − 32) = C Multiplying by

59

on both sides

71. Ax + By = C

Ax = C −By Subtracting By on both sides

A =C −By

xDividing by x on both sides

73. 2w + 2h + l = p

2h = p− 2w − l Subtracting 2w and l

h =p− 2w − l

2Dividing by 2

75. 2x− 3y = 6

−3y = 6 − 2x Subtracting 2x

y =6 − 2x−3

, or Dividing by −3

2x− 63

77. a = b + bcd

a = b(1 + cd) Factoringa

1 + cd= b Dividing by 1 + cd

79. z = xy − xy2

z = x(y − y2) Factoringz

y − y2= x Dividing by y − y2

81. 3[5 − 3(4 − t)] − 2 = 5[3(5t− 4) + 8] − 26

3[5 − 12 + 3t] − 2 = 5[15t− 12 + 8] − 26

3[−7 + 3t] − 2 = 5[15t− 4] − 26

−21 + 9t− 2 = 75t− 20 − 26

9t− 23 = 75t− 46

−66t− 23 = −46

−66t = −23

t =2366

The solution is2366

.

83. x− {3x− [2x− (5x− (7x− 1))]} = x + 7

x− {3x− [2x− (5x− 7x + 1)]} = x + 7

x− {3x− [2x− (−2x + 1)]} = x + 7

x− {3x− [2x + 2x− 1]} = x + 7

x− {3x− [4x− 1]} = x + 7

x− {3x− 4x + 1} = x + 7

x− {−x + 1} = x + 7

x + x− 1 = x + 7

2x− 1 = x + 7

x− 1 = 7

x = 8The solution is 8.

85. (5x2 + 6x)(12x2 − 5x− 2) = 0

x(5x + 6)(4x + 1)(3x− 2) = 0

x = 0 or 5x+6 = 0 or 4x+1 = 0 or 3x−2 = 0

x = 0 or 5x = −6 or 4x = −1 or 3x = 2

x = 0 or x = −65

or x = −14

or x =23

The solutions are 0, −65, −1

4, and

23.

87. 3x3 + 6x2 − 27x− 54 = 0

3(x3 + 2x2 − 9x− 18) = 0

3[x2(x + 2) − 9(x + 2)] = 0 Factoring by grouping

3(x + 2)(x2 − 9) = 0

3(x + 2)(x + 3)(x− 3) = 0

x + 2 = 0 or x + 3 = 0 or x− 3 = 0

x = −2 or x = −3 or x = 3The solutions are −2, −3, and 3.

Exercise Set R.6

1. Since −53

is defined for all real numbers, the domain is

{x|x is a real number}.

3.3x− 3x(x− 1)The denominator is 0 when the factor x = 0 andalso when x − 1 = 0, or x = 1. The domain is{x|x is a real number and x = 0 and x = 1}.

5.x + 5

x2 + 4x− 5=

x + 5(x + 5)(x− 1)

We see that x + 5 = 0 when x = −5 and x− 1 = 0when x = 1. Thus, the domain is{x|x is a real number and x = −5 and x = 1}.

7. We first factor the denominator completely.7x2 − 28x + 28

(x2−4)(x2+3x−10)=

7x2 − 28x + 28(x+2)(x−2)(x+5)(x−2)

We see that x + 2 = 0 when x = −2, x − 2 = 0 whenx = 2, and x + 5 = 0 when x = −5. Thus, the domain is{x|x is a real number and x =−2 and x =2 and x =−5}.

Copyright © 2013 Pearson Education, Inc.

Page 16: JUDITH A. PENNA -  · PDF fileSTUDENT’S SOLUTIONS MANUAL JUDITH A. PENNA Indiana University Purdue University Indianapolis COLLEGE ALGEBRA: GRAPHS AND MODELS FIFTH EDITION

Exercise Set R.6 13

9.x2 − 4

x2 − 4x + 4=

(x + 2)(x−2)✏(x− 2)(x−2)✏ =

x + 2x− 2

11. x3 − 6x2 + 9xx3 − 3x2

=x(x2 − 6x + 9)

x2(x− 3)

=x/(x−3)✏ (x− 3)

x/ · x(x−3)✏=

x− 3x

13. 6y2 + 12y − 483y2 − 9y + 6

=6(y2 + 2y − 8)3(y2 − 3y + 2)

=2 · 3/ · (y + 4)(y−2)✏

3/(y − 1)(y−2)✏=

2(y + 4)y − 1

15. 4 − x

x2 + 4x− 32=

−1(x−4)✏(x−4)✏ (x + 8)

=−1

x + 8, or − 1

x + 8

17. r − s

r + s· r2 − s2

(r − s)2=

(r − s)(r2 − s2)(r + s)(r − s)2

=(r−s)✏ (r−s)✏ (r+s)✏ · 1

(r+s)✏ (r−s)✏ (r−s)✏= 1

19. x2 + 2x− 353x3 − 2x2

· 9x3 − 4x7x + 49

=(x+7)✏ (x− 5)(x)✧ (3x + 2)(3x−2)✘

x/ · x(3x−2)✘ (7)(x+7)✏=

(x− 5)(3x + 2)7x

21. 4x2 + 9x + 2x2 + x− 2

· x2 − 13x2 + x− 2

=(4x + 1)(x+2)✏ (x+1)✏ (x−1)✏(x+2)✏ (x−1)✏ (3x− 2)(x+1)✏

=4x + 13x− 2

23. m2 − n2

r + s÷ m− n

r + s

=m2 − n2

r + s· r + s

m− n

=(m + n)(m−n)✏ (r+s)✏

(r+s)✏ (m−n)✏= m + n

25. 3x + 122x− 8

÷ (x + 4)2

(x− 4)2

=3x + 122x− 8

· (x− 4)2

(x + 4)2

=3(x+4)✏ (x−4)✏ (x− 4)2(x−4)✏ (x+4)✏ (x + 4)

=3(x− 4)2(x + 4)

27. x2 − y2

x3 − y3· x2 + xy + y2

x2 + 2xy + y2

=(x + y)(x− y)(x2 + xy + y2)

(x− y)(x2 + xy + y2)(x + y)(x + y)

=1

x + y· (x + y)(x− y)(x2 + xy + y2)(x + y)(x− y)(x2 + xy + y2)

=1

x + y· 1 Removing a factor of 1

=1

x + y

29. (x− y)2 − z2

(x + y)2 − z2÷ x− y + z

x + y − z

=(x− y)2 − z2

(x + y)2 − z2· x + y − z

x− y + z

=(x− y + z)(x− y − z)(x + y − z)(x + y + z)(x + y − z)(x− y + z)

=(x− y + z)(x + y − z)(x− y + z)(x + y − z)

· x− y − z

x + y + z

= 1 · x− y − z

x + y + zRemoving a factor of 1

=x− y − z

x + y + z

31. 75x

+35x

=7 + 35x

=105x

=5/ · 25/ · x

=2x

33. 43a + 4

+3a

3a + 4=

4 + 3a3a + 4

= 1 (4 + 3a = 3a + 4)

35. 54z

− 38z

, LCD is 8z

=54z

· 22− 3

8z

=108z

− 38z

=78z

37. 3x + 2

+2

x2 − 4

=3

x + 2+

2(x + 2)(x− 2)

, LCD is (x+2)(x−2)

=3

x + 2· x− 2x− 2

+2

(x + 2)(x− 2)

=3x− 6

(x + 2)(x− 2)+

2(x + 2)(x− 2)

=3x− 4

(x + 2)(x− 2)

Copyright © 2013 Pearson Education, Inc.

Page 17: JUDITH A. PENNA -  · PDF fileSTUDENT’S SOLUTIONS MANUAL JUDITH A. PENNA Indiana University Purdue University Indianapolis COLLEGE ALGEBRA: GRAPHS AND MODELS FIFTH EDITION

14 Chapter R: Basic Concepts of Algebra

39. y

y2 − y − 20− 2

y + 4

=y

(y + 4)(y − 5)− 2

y + 4, LCD is (y + 4)(y − 5)

=y

(y + 4)(y − 5)− 2

y + 4· y − 5y − 5

=y

(y + 4)(y − 5)− 2y − 10

(y + 4)(y − 5)

=y − (2y − 10)(y + 4)(y − 5)

=y − 2y + 10

(y + 4)(y − 5)

=−y + 10

(y + 4)(y − 5)

41. 3x + y

+x− 5yx2 − y2

=3

x + y+

x− 5y(x + y)(x− y)

, LCD is (x + y)(x− y)

=3

x + y· x− y

x− y+

x− 5y(x + y)(x− y)

=3x− 3y

(x + y)(x− y)+

x− 5y(x + y)(x− y)

=4x− 8y

(x + y)(x− y)

43. y

y − 1+

21 − y

=y

y − 1+

−1−1

· 21 − y

=y

y − 1+

−2y − 1

=y − 2y − 1

45.x

2x− 3y− y

3y − 2x

=x

2x− 3y− −1

−1· y

3y − 2x

=x

2x− 3y− −y

2x− 3y

=x + y

2x− 3y[x− (−y) = x + y]

47. 9x + 23x2 − 2x− 8

+7

3x2 + x− 4

=9x + 2

(3x + 4)(x− 2)+

7(3x + 4)(x− 1)

,

LCD is (3x + 4)(x− 2)(x− 1)

=9x + 2

(3x+4)(x−2)· x− 1x− 1

+7

(3x+4)(x−1)· x− 2x− 2

=9x2 − 7x− 2

(3x+4)(x−2)(x−1)+

7x− 14(3x+4)(x−1)(x−2)

=9x2 − 16

(3x + 4)(x− 2)(x− 1)

=(3x+4)✘ (3x− 4)

(3x+4)✘ (x− 2)(x− 1)

=3x− 4

(x− 2)(x− 1)

49. 5aa− b

+ab

a2 − b2+

4ba + b

=5a

a− b+

ab

(a + b)(a− b)+

4ba + b

,

LCD is (a + b)(a− b)

=5a

a− b· a + b

a + b+

ab

(a + b)(a− b)+

4ba + b

· a− b

a− b

=5a2 + 5ab

(a+b)(a−b)+

ab

(a+b)(a−b)+

4ab− 4b2

(a+b)(a−b)

=5a2 + 10ab− 4b2

(a + b)(a− b)

51. 7x + 2

− x + 84 − x2

+3x− 2

4 − 4x + x2

=7

x + 2− x + 8

(2 + x)(2 − x)+

3x− 2(2 − x)2

,

LCD is (2 + x)(2 − x)2

=7

2 + x· (2 − x)2

(2 − x)2− x + 8

(2 + x)(2 − x)· 2 − x

2 − x+

3x− 2(2 − x)2

· 2 + x

2 + x

=28−28x+7x2−(16−6x−x2)+3x2+4x−4

(2 + x)(2 − x)2

=28 − 28x + 7x2 − 16 + 6x + x2 + 3x2 + 4x− 4

(2 + x)(2 − x)2

=11x2 − 18x + 8(2 + x)(2 − x)2

, or11x2 − 18x + 8(x + 2)(x− 2)2

Copyright © 2013 Pearson Education, Inc.

Page 18: JUDITH A. PENNA -  · PDF fileSTUDENT’S SOLUTIONS MANUAL JUDITH A. PENNA Indiana University Purdue University Indianapolis COLLEGE ALGEBRA: GRAPHS AND MODELS FIFTH EDITION

Exercise Set R.6 15

53. 1x + 1

+x

2 − x+

x2 + 2x2 − x− 2

=1

x + 1+

x

2 − x+

x2 + 2(x + 1)(x− 2)

=1

x + 1+

−1−1

· x

2 − x+

x2 + 2(x + 1)(x− 2)

=1

x + 1+

−x

x− 2+

x2 + 2(x + 1)(x− 2)

,

LCD is (x + 1)(x− 2)

=1

x + 1· x− 2x− 2

+−x

x− 2· x + 1x + 1

+x2 + 2

(x + 1)(x− 2)

=x− 2

(x+1)(x−2)+

−x2 − x

(x+1)(x−2)+

x2 + 2(x+1)(x−2)

=x− 2 − x2 − x + x2 + 2

(x + 1)(x− 2)

=0

(x + 1)(x− 2)= 0

55.

a− b

ba2 − b2

ab

=a− b

b· ab

a2 − b2

=a− b

b· ab

(a + b)(a− b)

=a b✧(a−b)✏

b/ (a + b)(a−b)✏=

a

a + b

57.

x

y− y

x1y

+1x

=

x

y− y

x1y

+1x

· xyxy

, LCM is xy

=

(xy− y

x

)(xy)(1

y+

1x

)(xy)

=x2 − y2

x + y

=(x+y)✏ (x− y)

(x+y)✏ · 1= x− y

59.c +

8c2

1 +2c

=c · c

2

c2+

8c2

1 · cc

+2c

=

c3 + 8c2

c + 2c

=c3 + 8c2

· c

c + 2

=(c+2)✏ (c2 − 2c + 4)c/

c/ · c(c+2)✏=

c2 − 2c + 4c

61. x2 + xy + y2

x2

y− y2

x

=x2 + xy + y2

x2

y· xx− y2

x· yy

=x2 + xy + y2

x3 − y3

xy

= (x2 + xy + y2) · xy

x3 − y3

=(x2 + xy + y2)(xy)

(x− y)(x2 + xy + y2)

=x2 + xy + y2

x2 + xy + y2· xy

x− y

= 1 · xy

x− y

=xy

x− y

63. a− a−1

a + a−1=

a− 1a

a +1a

=a · a

a− 1

a

a · aa

+1a

=

a2 − 1a

a2 + 1a

=a2 − 1

a· a

a2 + 1

=a2 − 1a2 + 1

Copyright © 2013 Pearson Education, Inc.

Page 19: JUDITH A. PENNA -  · PDF fileSTUDENT’S SOLUTIONS MANUAL JUDITH A. PENNA Indiana University Purdue University Indianapolis COLLEGE ALGEBRA: GRAPHS AND MODELS FIFTH EDITION

16 Chapter R: Basic Concepts of Algebra

65.

1x− 3

+2

x + 33

x− 1− 4

x + 2

=

1x− 3

· x + 3x + 3

+2

x + 3· x− 3x− 3

3x− 1

· x + 2x + 2

− 4x + 2

· x− 1x− 1

=

x + 3 + 2(x− 3)(x− 3)(x + 3)

3(x + 2) − 4(x− 1)(x− 1)(x + 2)

=

x + 3 + 2x− 6(x− 3)(x + 3)

3x + 6 − 4x + 4(x− 1)(x + 2)

=

3x− 3(x− 3)(x + 3)

−x + 10(x− 1)(x + 2)

=3x− 3

(x− 3)(x + 3)· (x− 1)(x + 2)

−x + 10

=(3x− 3)(x− 1)(x + 2)

(x− 3)(x + 3)(−x + 10), or

3(x− 1)2(x + 2)(x− 3)(x + 3)(−x + 10)

67.

a

1 − a+

1 + a

a1 − a

a+

a

1 + a

=

a

1 − a· aa

+1 + a

a· 1 − a

1 − a1 − a

a· 1 + a

1 + a+

a

1 + a· aa

=

a2 + (1 − a2)a(1 − a)

(1 − a2) + a2

a(1 + a)

=1

a/(1 − a)· a/(1 + a)

1

=1 + a

1 − a

69.

1a2

+2ab

+1b2

1a2

− 1b2

=

1a2

+2ab

+1b2

1a2

− 1b2

· a2b2

a2b2,

LCM is a2b2

=b2 + 2ab + a2

b2 − a2

=(b+a)✏ (b + a)(b+a)✏ (b− a)

=b + a

b− a

71. (x + h)2 − x2

h=

x2 + 2xh + h2 − x2

h

=2xh + h2

h

=h/(2x + h)

h/ · 1= 2x + h

73. (x + h)3 − x3

h=

x3 + 3x2h + 3xh2 + h3 − x3

h

=3x2h + 3xh2 + h3

h

=h/(3x2 + 3xh + h2)

h/ · 1= 3x2 + 3xh + h2

75.

x + 1x− 1

+ 1

x + 1x− 1

− 1

5

=

(x + 1) + (x− 1)x− 1

(x + 1) − (x− 1)x− 1

5

=[

2xx− 1

· x− 12

]5

=[

2/x(x−1)✦1 · 2/(x−1)✦

]5= x5

77. n(n + 1)(n + 2)2 · 3 +

(n + 1)(n + 2)2

=n(n + 1)(n + 2)

2 · 3 +(n + 1)(n + 2)

2· 33,

LCD is 2 · 3

=n(n + 1)(n + 2) + 3(n + 1)(n + 2)

2 · 3

=(n + 1)(n + 2)(n + 3)

2 · 3 Factoring the num-

erator by grouping

79. x2 − 9x3 + 27

· 5x2 − 15x + 45x2 − 2x− 3

+x2 + x

4 + 2x

=(x + 3)(x− 3)(5)(x2 − 3x + 9)

(x + 3)(x2 − 3x + 9)(x− 3)(x + 1)+

x2 + x

4 + 2x

=(x + 3)(x− 3)(x2 − 3x + 9)(x + 3)(x− 3)(x2 − 3x + 9)

· 5x + 1

+x2 + x

4 + 2x

= 1 · 5x + 1

+x2 + x

4 + 2x

=5

x + 1+

x2 + x

2(2 + x)

=5 · 2(2 + x) + (x2 + x)(x + 1)

2(x + 1)(2 + x)

=20 + 10x + x3 + 2x2 + x

2(x + 1)(2 + x)

=x3 + 2x2 + 11x + 20

2(x + 1)(2 + x)

Exercise Set R.7

1.√

(−21)2 = | − 21| = 21

3.√

9y2 =√

(3y)2 = |3y| = 3|y|

5.√

(a− 2)2 = |a− 2|

Copyright © 2013 Pearson Education, Inc.

Page 20: JUDITH A. PENNA -  · PDF fileSTUDENT’S SOLUTIONS MANUAL JUDITH A. PENNA Indiana University Purdue University Indianapolis COLLEGE ALGEBRA: GRAPHS AND MODELS FIFTH EDITION

Exercise Set R.7 17

7. 3√−27x3 = 3

√(−3x)3 = −3x

9. 4√

81x8 = 4√

(3x2)4 = |3x2| = 3x2

11. 5√

32 = 5√

25 = 2

13.√

180 =√

36 · 5 =√

36 · √5 = 6√

5

15.√

72 =√

36 · 2 =√

36 · √2 = 6√

2

17. 3√

54 = 3√

27 · 2 = 3√

27 · 3√

2 = 3 3√

2

19.√

128c2d4 =√

64c2d4 · 2 = |8cd2|√2 = 8√

2 |c|d2

21. 4√

48x6y4 = 4√

16x4y4 · 3x2 = |2xy| 4√

3x2 =

2|x||y| 4√

3x2

23.√x2 − 4x + 4 =

√(x− 2)2 = |x− 2|

25.√

15√

35 =√

15 · 35 =√

3 · 5 · 5 · 7 =√

52 · 3 · 7 =√52 · √3 · 7 = 5

√21

27.√

8√

10 =√

8 · 10 =√

2 · 4 · 2 · 5 =√

22 · 4 · 5 =

2 · 2√5 = 4√

5

29.√

2x3y√

12xy =√

24x4y2 =√

4x4y2 · 6 = 2x2y√

6

31. 3√

3x2y 3√

36x = 3√

108x3y = 3√

27x3 · 4y = 3x 3√

4y

33. 3√

2(x + 4) 3√

4(x + 4)4 = 3√

8(x + 4)5

= 3√

8(x + 4)3 · (x + 4)2

= 2(x + 4) 3√

(x + 4)2

35. 8

√m16n24

28= 8

√(m2n3

2

)8

=m2n3

2

37.√

40xy√8x

=

√40xy8x

=√

5y

39.3√

3x2

3√

24x5= 3

√3x2

24x5= 3

√1

8x3=

12x

41. 3

√64a4

27b3= 3

√64 · a3 · a

27 · b3

=3√

64a3 3√a

3√

27b3

=4a 3

√a

3b

43.

√7x3

36y6=

√7 · x2 · x36 · y6

=

√x2

√7x√

36y6

=x√

7x6y3

45. 5√

2 + 3√

32 = 5√

2 + 3√

16 · 2= 5

√2 + 3 · 4√2

= 5√

2 + 12√

2

= (5 + 12)√

2

= 17√

2

47. 6√

20 − 4√

45 +√

80 = 6√

4 · 5 − 4√

9 · 5 +√

16 · 5= 6 · 2√5 − 4 · 3√5 + 4

√5

= 12√

5 − 12√

5 + 4√

5

= (12 − 12 + 4)√

5

= 4√

5

49. 8√

2x2 − 6√

20x− 5√

8x2

= 8x√

2 − 6√

4 · 5x− 5√

4x2 · 2= 8x

√2 − 6 · 2√5x− 5 · 2x√2

= 8x√

2 − 12√

5x− 10x√

2

= −2x√

2 − 12√

5x

51.(√

8 + 2√

5)(√

8 − 2√

5)

=(√

8)2 − (2√5

)2= 8 − 4 · 5= 8 − 20

= −12

53. (2√

3 +√

5)(√

3 − 3√

5)

= 2√

3 · √3 − 2√

3 · 3√5 +√

5 · √3 −√5 · 3√5

= 2 · 3 − 6√

15 +√

15 − 3 · 5= 6 − 6

√15 +

√15 − 15

= −9 − 5√

15

55. (√

2 − 5)2 = (√

2)2 − 2 · √2 · 5 + 52

= 2 − 10√

2 + 25

= 27 − 10√

2

57. (√

5 −√6)2 = (

√5)2 − 2

√5 · √6 + (

√6)2

= 5 − 2√

30 + 6

= 11 − 2√

30

59. We use the Pythagorean theorem. We have a = 47 andb = 25.

c2 = a2 + b2

c2 = 472 + 252

c2 = 2209 + 625

c2 = 2834

c ≈ 53.2

The distance across the pond is about 53.2 yd.

Copyright © 2013 Pearson Education, Inc.

Page 21: JUDITH A. PENNA -  · PDF fileSTUDENT’S SOLUTIONS MANUAL JUDITH A. PENNA Indiana University Purdue University Indianapolis COLLEGE ALGEBRA: GRAPHS AND MODELS FIFTH EDITION

18 Chapter R: Basic Concepts of Algebra

61. a) h2 +(a

2

)2

= a2 Pythagorean theorem

h2 +a2

4= a2

h2 =3a2

4

h =

√3a2

4

h =a

2

√3

b) Using the result of part (a) we have

A =12· base · height

A =12a · a

2

√3(a

2+

a

2= a

)

A =a2

4

√3

63.

x

x

x x8√

2

❅❅

❅❅

❅❅

x2 + x2 = (8√

2)2 Pythagorean theorem

2x2 = 128

x2 = 64

x = 8

65.

√37

=

√37· 77

=

√2149

=√

21√49

=√

217

67.3√

73√

25=

3√

73√

25·

3√

53√

5=

3√

353√

125=

3√

355

69. 3

√169

= 3

√169

· 33

= 3

√4827

=3√

483√

27=

3√

8 · 63

=2 3√

63

71.2√

3 − 1=

2√3 − 1

·√

3 + 1√3 + 1

=2(√

3 + 1)3 − 1

=2(√

3 + 1)2

=2/(√

3 + 1)2/ · 1

=√

3 + 1

73.1 −√

22√

3 −√6

=1 −√

22√

3 −√6· 2

√3 +

√6

2√

3 +√

6

=2√

3 +√

6 − 2√

6 −√12

4 · 3 − 6

=2√

3 +√

6 − 2√

6 − 2√

312 − 6

=−√

66

, or −√

66

75.6√

m−√n

=6√

m−√n·√m +

√n√

m +√n

=6(√m +

√n)

(√m)2 − (

√n)2

=6√m + 6

√n

m− n

77.

√503

=√

503

·√

2√2

=√

1003√

2=

103√

2

79. 3

√25

= 3

√25· 44

= 3

√820

=3√

83√

20=

23√

20

81.

√11√3

=√

11√3

·√

11√11

=√

121√33

=11√33

83.9 −√

53 −√

3=

9 −√5

3 −√3· 9 +

√5

9 +√

5

=92 − (

√5)2

27 + 3√

5 − 9√

3 −√15

=81 − 5

27 + 3√

5 − 9√

3 −√15

=76

27 + 3√

5 − 9√

3 −√15

85.√a +

√b

3a=

√a +

√b

3a·√a−√

b√a−√

b

=(√a)2 − (

√b)2

3a(√a−√

b)

=a− b

3a√a− 3a

√b

87. y5/6 = 6√

y5

89. 163/4 = (161/4)3 =(

4√

16)3 = 23 = 8

91. 125−1/3 =1

1251/3=

13√

125=

15

93. a5/4b−3/4 =a5/4

b3/4=

4√a5

4√b3

=a 4√a

4√b3

, or a 4

√a

b3

95. m5/3n7/3 = 3√m5 3

√n7 = 3

√m5n7 = mn2 3

√m2n

97. 5√

173 = 173/5

99.(

5√

12)4 = 124/5

Copyright © 2013 Pearson Education, Inc.

Page 22: JUDITH A. PENNA -  · PDF fileSTUDENT’S SOLUTIONS MANUAL JUDITH A. PENNA Indiana University Purdue University Indianapolis COLLEGE ALGEBRA: GRAPHS AND MODELS FIFTH EDITION

Chapter R Review Exercises 19

101. 3√√

11 =(√

11)1/3 = (111/2)1/3 = 111/6

103.√

5 3√

5 = 51/2 · 51/3 = 51/2+1/3 = 55/6

105. 5√

322 = 322/5 = (321/5)2 = 22 = 4

107. (2a3/2)(4a1/2) = 8a3/2+1/2 = 8a2

109.( x6

9b−4

)1/2

=( x6

32b−4

)1/2

=x3

3b−2, or

x3b2

3

111.x2/3y5/6

x−1/3y1/2= x2/3−(−1/3)y5/6−1/2 = xy1/3 = x 3

√y

113. (m1/2n5/2)2/3 = m12 · 23n

52 · 23 = m1/3n5/3 =

3√m

3√n5 = 3

√mn5 = n

3√mn2

115. a3/4(a2/3 + a4/3) = a3/4+2/3 + a3/4+4/3 =

a17/12 + a25/12 = 12√a17 + 12

√a25 =

a12√a5 + a2 12

√a

117. 3√

6√

2 = 61/321/2 = 62/623/6

= (6223)1/6

= 6√

36 · 8= 6

√288

119. 4√xy 3√

x2y = (xy)1/4(x2y)1/3 = (xy)3/12(x2y)4/12

=[(xy)3(x2y)4

]1/12=[x3y3x8y4

]1/12= 12√

x11y7

121. 3√

a4√a3 =

(a4√a3)1/3 = (a4a3/2)1/3

= (a11/2)1/3

= a11/6

= 6√a11

= a6√a5

123.√

(a + x)3 3√

(a + x)24√a + x

=(a + x)3/2(a + x)2/3

(a + x)1/4

=(a + x)26/12

(a + x)3/12

= (a + x)23/12

= 12√

(a + x)23

= (a + x) 12√

(a + x)11

125.√

1 + x2 +1√

1 + x2

=√

1 + x2 · 1 + x2

1 + x2+

1√1 + x2

·√

1 + x2

√1 + x2

=(1 + x2)

√1 + x2

1 + x2+

√1 + x2

1 + x2

=(2 + x2)

√1 + x2

1 + x2

127.

(√a√a

)√a

=(a√a/2)√a

= aa/2

Chapter R Review Exercises

1. True

3. True

5. Rational numbers: −7, 43, −49, 0, 3

√64, 4

34,

127

, 102

7. Integers: −7, 43, 0, 3√

64, 102

9. Natural numbers: 43, 3√

64, 102

11. (−4, 7]

13. The distance of −78

from 0 is78, so

∣∣∣∣− 78

∣∣∣∣ = 78.

15. 3 · 2 − 4 · 22 + 6(3 − 1)

= 3 · 2 − 4 · 22 + 6 · 2 Working insideparentheses

= 3 · 2 − 4 · 4 + 6 · 2 Evaluating 22

= 6 − 16 + 12 Multiplying

= −10 + 12 Adding in order

= 2 from left to right

17. Convert 8.3 × 10−5 to decimal notation.

The exponent is negative, so the number is between 0 and1. We move the decimal point 5 places to the left.

8.3 × 10−5 = 0.000083

19. Convert 405,000 to scientific notation.

We want the decimal point to be positioned between the4 and the first 0, so we move it 5 places to the left. Since405,000 is greater than 10, the exponent must be positive.

405, 000 = 4.05 × 105

21. (3.1 × 105)(4.5 × 10−3)

= (3.1 × 4.5) × (105 × 10−3)

= 13.95 × 102 This is not scientific notation.

= (1.395 × 10) × 102

= 1.395 × 103 Writing scientific notation

23. (−3x4y−5)(4x−2y) = −3(4)x4+(−2)y−5+1 =

−12x2y−4, or−12x2

y4, or − 12x2

y4

25. 4√

81 = 4√

34 = 3

Copyright © 2013 Pearson Education, Inc.

Page 23: JUDITH A. PENNA -  · PDF fileSTUDENT’S SOLUTIONS MANUAL JUDITH A. PENNA Indiana University Purdue University Indianapolis COLLEGE ALGEBRA: GRAPHS AND MODELS FIFTH EDITION

20 Chapter R: Basic Concepts of Algebra

27. b− a−1

a− b−1=

b− 1a

a− 1b

=b · a

a− 1

a

a · bb− 1

b

=

ab

a− 1

aab

b− 1

b

=

ab− 1a

ab− 1b

=ab− 1

a· b

ab− 1

=(ab−1✏ )ba(ab−1✏ )

=b

a

29. (√

3 −√7)(

√3 +

√7) = (

√3)2 − (

√7)2

= 3 − 7

= −4

31. 8√

5 +25√

5= 8

√5 +

25√5·√

5√5

= 8√

5 +25

√5

5= 8

√5 + 5

√5

= 13√

5

33. (5a + 4b)(2a− 3b)

= 10a2 − 15ab + 8ab− 12b2

= 10a2 − 7ab− 12b2

35. 32x4 − 40xy3 = 8x · 4x3 − 8x · 5y3 = 8x(4x3 − 5y3)

37. 24x + 144 + x2

= x2 + 24x + 144

= (x + 12)2

39. 9x2 − 30x + 25 = (3x− 5)2

41. 18x2 − 3x + 6 = 3(6x2 − x + 2)

43. 6x3 + 48

= 6(x3 + 8)

= 6(x + 2)(x2 − 2x + 4)

45. 2x2 + 5x− 3 = (2x− 1)(x + 3)

47. 5x− 7 = 3x− 9

2x− 7 = −9

2x = −2

x = −1The solution is −1.

49. 6(2x− 1) = 3 − (x + 10)

12x− 6 = 3 − x− 10

12x− 6 = −x− 7

13x− 6 = −7

13x = −1

x = − 113

The solution is − 113

.

51. x2 − x = 20

x2 − x− 20 = 0

(x− 5)(x + 4) = 0

x− 5 = 0 or x + 4 = 0

x = 5 or x = −4

The solutions are 5 and −4.

53. x(x− 2) = 3

x2 − 2x = 3

x2 − 2x− 3 = 0

(x + 1)(x− 3) = 0

x + 1 = 0 or x− 3 = 0

x = −1 or x = 3

The solutions are −1 and 3.

55. n2 − 7 = 0

n2 = 7

n =√

7 or n = −√7

The solutions are√

7 and −√7, or ±√

7.

57. xy = x + 3

xy − x = 3

x(y − 1) = 3

x =3

y − 1

59.x

x2 + 9x + 20− 4

x2 + 7x + 12

=x

(x + 5)(x + 4)− 4

(x + 4)(x + 3)

LCD is (x + 5)(x + 4)(x + 3)

=x

(x + 5)(x + 4)· x + 3x + 3

− 4(x + 4)(x + 3)

· x + 5x + 5

=x(x + 3) − 4(x + 5)

(x + 5)(x + 4)(x + 3)

=x2 + 3x− 4x− 20

(x + 5)(x + 4)(x + 3)

=x2 − x− 20

(x + 5)(x + 4)(x + 3)

=(x− 5)(x+4)✏

(x + 5)(x+4)✏ (x + 3)

=x− 5

(x + 5)(x + 3)

Copyright © 2013 Pearson Education, Inc.

Page 24: JUDITH A. PENNA -  · PDF fileSTUDENT’S SOLUTIONS MANUAL JUDITH A. PENNA Indiana University Purdue University Indianapolis COLLEGE ALGEBRA: GRAPHS AND MODELS FIFTH EDITION

Chapter R Test 21

61.√

(a + b)3 3√a + b

6√

(a + b)7

=(a + b)3/2(a + b)1/3

(a + b)7/6

= (a + b)3/2+1/3−7/6

= (a + b)9/6+2/6−7/6

= (a + b)2/3

= 3√

(a + b)2

63. 8

√m32n16

38=(m32n16

38

)1/8

=m4n2

3

65. a = 8 and b = 17. Find c.c2 = a2 + b2

c2 = 82 + 172

c2 = 64 + 289

c2 = 353

c ≈ 18.8

The guy wire is about 18.8 ft long.

67. 9x2 − 36y2 = 9(x2 − 4y2)

= 9[x2 − (2y)2]

= 9(x + 2y)(x− 2y)

Answer C is correct.

69. Substitute $124, 000 − $20, 000, or $104,000 for P , 0.0575for r, and 12 · 30, or 360, for n and perform the resultingcomputation.

M = P

r

12

(1 +

r

12

)n

(1 +

r

12

)n

− 1

= $104, 000

0.057512

(1 +

0.057512

)360

(1 +

0.057512

)360

− 1

≈ $606.92

71. (an−bn)3 = (an−bn)(an−bn)2

= (an − bn)(a2n − 2anbn + b2n)

= a3n−2a2nbn+anb2n−a2nbn+2anb2n−b3n

= a3n − 3a2nbn + 3anb2n − b3n

73. m6n −m3n = m3n(m3n − 1)

= m3n[(mn)3 − 13]

= m3n(mn − 1)(m2n + mn + 1)

Chapter R Test

1. a) Whole numbers: 0, 3√

8, 29

b) Irrational numbers:√

12

c) Integers but not natural numbers: 0, −5

d) Rational numbers but not integers: 667, −13

4, −1.2

2. | − 17.6| = 17.6

3.∣∣∣∣1511

∣∣∣∣ = 1511

4. |0| = 0

5. (−3, 6]

6. | − 9 − 6| = | − 15| = 15, or

|6 − (−9)| = |6 + 9| = |15| = 15

7. 32 ÷ 23 − 12 ÷ 4 · 3= 32 ÷ 8 − 12 ÷ 4 · 3= 4 − 12 ÷ 4 · 3= 4 − 3 · 3= 4 − 9

= −5

8. Position the decimal point 6 places to the left, between the4 and the 5. Since 4,509,000 is a number greater than 10,the exponent must be positive.

4, 509, 000 = 4.509 × 106

9. The exponent is negative, so the number is between 0 and1. We move the decimal point 5 places to the left.

8.6 × 10−5 = 0.000086

10. 2.7 × 104

3.6 × 10−3= 0.75 × 107

= (7.5 × 10−1) × 107

= 7.5 × 106

11. x−8 · x5 = x−8+5 = x−3, or1x3

12. (2y2)3(3y4)2 = 23y6 · 32y8 = 8 · 9 · y6+8 = 72y14

13. (−3a5b−4)(5a−1b3)

= −3 · 5 · a5+(−1) · b−4+3

= −15a4b−1, or −15a4

b

14. (5xy4− 7xy2+ 4x2−3)−(−3xy4+ 2xy2−2y+ 4)

= (5xy4−7xy2+ 4x2− 3)+(3xy4−2xy2+ 2y − 4)

= (5 + 3)xy4+ (−7 − 2)xy2+ 4x2+ 2y + (−3 − 4)

= 8xy4 − 9xy2 + 4x2 + 2y − 7

Copyright © 2013 Pearson Education, Inc.

Page 25: JUDITH A. PENNA -  · PDF fileSTUDENT’S SOLUTIONS MANUAL JUDITH A. PENNA Indiana University Purdue University Indianapolis COLLEGE ALGEBRA: GRAPHS AND MODELS FIFTH EDITION

22 Chapter R: Basic Concepts of Algebra

15. (y − 2)(3y + 4) = 3y2 + 4y − 6y − 8 = 3y2 − 2y − 8

16. (4x− 3)2 = (4x)2 − 2 · 4x · 3 + 32 = 16x2 − 24x + 9

17.

x

y− y

x

x + y=

x

y· xx− y

x· yy

x + y

=

x2

xy− y2

xy

x + y

=

x2 − y2

xy

x + y

=x2 − y2

xy· 1x + y

=(x+y)✏ (x− y)

xy(x+y)✏=

x− y

xy

18.√

45 =√

9 · 5 =√

9√

5 = 3√

5

19. 3√

56 = 3√

8 · 7 = 3√

8 3√

7 = 2 3√

7

20. 3√

75 + 2√

27 = 3√

25 · 3 + 2√

9 · 3= 3 · 5√3 + 2 · 3√3

= 15√

3 + 6√

3

= 21√

3

21.√

18√

10 =√

18 · 10 =√

2 · 3 · 3 · 2 · 5 =2 · 3√5 = 6

√5

22. (2 +√

3)(5 − 2√

3)

= 2 · 5 − 4√

3 + 5√

3 − 2 · 3= 10 − 4

√3 + 5

√3 − 6

= 4 +√

3

23. 8x2 − 18 = 2(4x2 − 9) = 2(2x + 3)(2x− 3)

24. y2 − 3y − 18 = (y + 3)(y − 6)

25. 2n2 + 5n− 12 = (2n− 3)(n + 4)

26. x3 + 10x2 + 25x = x(x2 + 10x + 25) = x(x + 5)2

27. m3 − 8 = (m− 2)(m2 + 2m + 4)

28. 7x− 4 = 24

7x = 28

x = 4

The solution is 4.

29. 3(y − 5) + 6 = 8 − (y + 2)

3y − 15 + 6 = 8 − y − 2

3y − 9 = −y + 6

4y − 9 = 6

4y = 15

y =154

The solution is154

.

30. 2x2 + 5x + 3 = 0

(2x + 3)(x + 1) = 0

2x + 3 = 0 or x + 1 = 0

2x = −3 or x = −1

x = −32

or x = −1

The solutions are −32

and −1.

31. z2 − 11 = 0

z2 = 11

z =√

11 or z = −√11

The solutions are√

11 and −√11, or ±√

11.

32. 8x + 3y = 24

3y = −8x + 24

y =−8x + 24

3, or − 8

3x + 8

33. x2 + x− 6x2 + 8x + 15

· x2 − 25x2 − 4x + 4

=(x2 + x− 6)(x2 − 25)

(x2 + 8x + 15)(x2 − 4x + 4)

=(x+3)✏ (x−2)✏ (x+5)✏ (x− 5)(x+3)✏ (x+5)✏ (x−2)✏ (x− 2)

=x− 5x− 2

34.x

x2 − 1− 3

x2 + 4x− 5

=x

(x + 1)(x− 1)− 3

(x− 1)(x + 5)

LCD is (x + 1)(x− 1)(x + 5)

=x

(x+ 1)(x−1)· x+ 5x + 5

− 3(x−1)(x + 5)

· x+ 1x+ 1

=x(x + 5) − 3(x + 1)

(x + 1)(x− 1)(x + 5)

=x2 + 5x− 3x− 3

(x + 1)(x− 1)(x + 5)

=x2 + 2x− 3

(x + 1)(x− 1)(x + 5)

=(x + 3)(x−1)✏

(x + 1)(x−1)✏ (x + 5)

=x + 3

(x + 1)(x + 5)

Copyright © 2013 Pearson Education, Inc.

Page 26: JUDITH A. PENNA -  · PDF fileSTUDENT’S SOLUTIONS MANUAL JUDITH A. PENNA Indiana University Purdue University Indianapolis COLLEGE ALGEBRA: GRAPHS AND MODELS FIFTH EDITION

Chapter R Test 23

35.5

7 −√3

=5

7 −√3· 7 +

√3

7 +√

3=

35 + 5√

349 − 3

=

35 + 5√

346

36. m3/8 = 8√m3

37. 6√

35 = 35/6

38. a = 5 and b = 12. Find c.c2 = a2 + b2

c2 = 52 + 122

c2 = 25 + 144

c2 = 169

c = 13

The guy wire is 13 ft long.

39. (x− y − 1)2

= [(x− y) − 1]2

= (x− y)2 − 2(x− y)(1) + 12

= x2 − 2xy + y2 − 2x + 2y + 1

Copyright © 2013 Pearson Education, Inc.

Page 27: JUDITH A. PENNA -  · PDF fileSTUDENT’S SOLUTIONS MANUAL JUDITH A. PENNA Indiana University Purdue University Indianapolis COLLEGE ALGEBRA: GRAPHS AND MODELS FIFTH EDITION

Copyright © 2013 Pearson Education, Inc.

Page 28: JUDITH A. PENNA -  · PDF fileSTUDENT’S SOLUTIONS MANUAL JUDITH A. PENNA Indiana University Purdue University Indianapolis COLLEGE ALGEBRA: GRAPHS AND MODELS FIFTH EDITION

4

2

�2

�4

42�2�4 x

y

(�1, 4)

(�3, �5)

(0, 2)(4, 0)

(2, �2)

4

2

�2

�4

4�2�4 x

y

(�5, 1)

(2, 3)

(0, 1) (5, 1)

(2, �1)

Chapter 1

Graphs, Functions, and Models

Exercise Set 1.1

1. Point A is located 5 units to the left of the y-axis and4 units up from the x-axis, so its coordinates are (−5, 4).

Point B is located 2 units to the right of the y-axis and2 units down from the x-axis, so its coordinates are (2,−2).

Point C is located 0 units to the right or left of the y-axisand 5 units down from the x-axis, so its coordinates are(0,−5).

Point D is located 3 units to the right of the y-axis and5 units up from the x-axis, so its coordinates are (3, 5).

Point E is located 5 units to the left of the y-axis and4 units down from the x-axis, so its coordinates are(−5,−4).

Point F is located 3 units to the right of the y-axis and0 units up or down from the x-axis, so its coordinates are(3, 0).

3. To graph (4, 0) we move from the origin 4 units to the rightof the y-axis. Since the second coordinate is 0, we do notmove up or down from the x-axis.

To graph (−3,−5) we move from the origin 3 units to theleft of the y-axis. Then we move 5 units down from thex-axis.

To graph (−1, 4) we move from the origin 1 unit to the leftof the y-axis. Then we move 4 units up from the x-axis.

To graph (0, 2) we do not move to the right or the left ofthe y-axis since the first coordinate is 0. From the originwe move 2 units up.

To graph (2,−2) we move from the origin 2 units to theright of the y-axis. Then we move 2 units down from thex-axis.

5. To graph (−5, 1) we move from the origin 5 units to theleft of the y-axis. Then we move 1 unit up from the x-axis.

To graph (5, 1) we move from the origin 5 units to the rightof the y-axis. Then we move 1 unit up from the x-axis.

To graph (2, 3) we move from the origin 2 units to the rightof the y-axis. Then we move 3 units up from the x-axis.

To graph (2,−1) we move from the origin 2 units to theright of the y-axis. Then we move 1 unit down from thex-axis.

To graph (0, 1) we do not move to the right or the left ofthe y-axis since the first coordinate is 0. From the originwe move 1 unit up.

7. The first coordinate represents the year and the cor-responding second coordinate represents the percentageof the U.S. population that is foreign-born. The or-dered pairs are (1970, 4.7%), (1980, 6.2%), (1990, 8.0%),(2000, 10.4%), (2010, 12.8%).

9. To determine whether (−1,−9) is a solution, substitute−1 for x and −9 for y.

y = 7x− 2

−9 ? 7(−1) − 2∣∣∣ −7 − 2−9

∣∣ −9 TRUE

The equation −9 = −9 is true, so (−1,−9) is a solution.

To determine whether (0, 2) is a solution, substitute 0 forx and 2 for y.

y = 7x− 2

2 ? 7 · 0 − 2∣∣∣ 0 − 22∣∣ −2 FALSE

The equation 2 = −2 is false, so (0, 2) is not a solution.

11. To determine whether(2

3,34

)is a solution, substitute

23

for x and34

for y.

6x− 4y = 1

6 · 23− 4 · 3

4? 1∣∣

4 − 3∣∣∣

1∣∣ 1 TRUE

The equation 1 = 1 is true, so(2

3,34

)is a solution.

Copyright © 2013 Pearson Education, Inc.

Page 29: JUDITH A. PENNA -  · PDF fileSTUDENT’S SOLUTIONS MANUAL JUDITH A. PENNA Indiana University Purdue University Indianapolis COLLEGE ALGEBRA: GRAPHS AND MODELS FIFTH EDITION

y

x�4 �2 2 4

�4

�2

2

4

5x � 3y � �15(�3, 0)

(0, 5)

(0, 4)

y

x�4 �2 2 4

�4

�2

2

42x � y � 4

(2, 0)

26 Chapter 1: Graphs, Functions, and Models

To determine whether(1,

32

)is a solution, substitute 1 for

x and32

for y.

6x− 4y = 1

6 · 1 − 4 · 32

? 1∣∣6 − 6

∣∣∣0∣∣ 1 FALSE

The equation 0 = 1 is false, so(1,

32

)is not a solution.

13. To determine whether(− 1

2,−4

5

)is a solution, substitute

−12

for a and −45

for b.

2a + 5b = 3

2(− 1

2

)+ 5(− 4

5

)? 3∣∣

−1 − 4∣∣∣

−5∣∣ 3 FALSE

The equation −5 = 3 is false, so(− 1

2,−4

5

)is not a solu-

tion.

To determine whether(0,

35

)is a solution, substitute 0 for

a and35

for b.

2a + 5b = 3

2 · 0 + 5 · 35

? 3∣∣0 + 3

∣∣∣3∣∣ 3 TRUE

The equation 3 = 3 is true, so(0,

35

)is a solution.

15. To determine whether (−0.75, 2.75) is a solution, substi-tute −0.75 for x and 2.75 for y.

x2 − y2 = 3

(−0.75)2 − (2.75)2 ? 30.5625 − 7.5625

∣∣∣−7

∣∣ 3 FALSE

The equation −7 = 3 is false, so (−0.75, 2.75) is not asolution.

To determine whether (2,−1) is a solution, substitute 2for x and −1 for y.

x2 − y2 = 3

22 − (−1)2 ? 34 − 1

∣∣∣3∣∣ 3 TRUE

The equation 3 = 3 is true, so (2,−1) is a solution.

17. Graph 5x− 3y = −15.

To find the x-intercept we replace y with 0 and solve forx.

5x− 3 · 0 = −15

5x = −15

x = −3The x-intercept is (−3, 0).To find the y-intercept we replace x with 0 and solve fory.

5 · 0 − 3y = −15

−3y = −15

y = 5The y-intercept is (0, 5).We plot the intercepts and draw the line that containsthem. We could find a third point as a check that theintercepts were found correctly.

19. Graph 2x + y = 4.To find the x-intercept we replace y with 0 and solve forx.

2x + 0 = 4

2x = 4

x = 2The x-intercept is (2, 0).To find the y-intercept we replace x with 0 and solve fory.

2 · 0 + y = 4

y = 4The y-intercept is (0, 4).We plot the intercepts and draw the line that containsthem. We could find a third point as a check that theintercepts were found correctly.

21. Graph 4y − 3x = 12.To find the x-intercept we replace y with 0 and solve forx.

4 · 0 − 3x = 12

−3x = 12

x = −4The x-intercept is (−4, 0).

Copyright © 2013 Pearson Education, Inc.

Page 30: JUDITH A. PENNA -  · PDF fileSTUDENT’S SOLUTIONS MANUAL JUDITH A. PENNA Indiana University Purdue University Indianapolis COLLEGE ALGEBRA: GRAPHS AND MODELS FIFTH EDITION

y

x�4 �2 2 4

�4

�2

2

44y � 3x � 12

(�4, 0)(0, 3)

6

2

�2

42�4 x

y

y � 3x � 5

4

2

�2

�4

42�2�4 x

y

x � y � 3

y

x�4 �2 2 4

�4

�2

2

4

34y � ��x � 3

y

x�4 �2 2 4

�4

�2

2

4

5x � 2y � 8

Exercise Set 1.1 27

To find the y-intercept we replace x with 0 and solve fory.

4y − 3 · 0 = 12

4y = 12

y = 3

The y-intercept is (0, 3).

We plot the intercepts and draw the line that containsthem. We could find a third point as a check that theintercepts were found correctly.

23. Graph y = 3x + 5.

We choose some values for x and find the correspondingy-values.

When x = −3, y = 3x + 5 = 3(−3) + 5 = −9 + 5 = −4.

When x = −1, y = 3x + 5 = 3(−1) + 5 = −3 + 5 = 2.

When x = 0, y = 3x + 5 = 3 · 0 + 5 = 0 + 5 = 5

We list these points in a table, plot them, and draw thegraph.

x y (x, y)

−3 −4 (−3,−4)

−1 2 (−1, 2)

0 5 (0, 5)

25. Graph x− y = 3.

Make a table of values, plot the points in the table, anddraw the graph.

x y (x, y)

−2 −5 (−2,−5)

0 −3 (0,−3)

3 0 (3, 0)

27. Graph y = −34x + 3.

By choosing multiples of 4 for x, we can avoid fractionvalues for y. Make a table of values, plot the points in thetable, and draw the graph.

x y (x, y)

−4 6 (−4, 6)

0 3 (0, 3)

4 0 (4, 0)

29. Graph 5x− 2y = 8.

We could solve for y first.

5x− 2y = 8

−2y = −5x + 8 Subtracting 5x on both sides

y =52x− 4 Multiplying by −1

2on both

sides

By choosing multiples of 2 for x we can avoid fractionvalues for y. Make a table of values, plot the points in thetable, and draw the graph.

x y (x, y)

0 −4 (0,−4)

2 1 (2, 1)

4 6 (4, 6)

Copyright © 2013 Pearson Education, Inc.

Page 31: JUDITH A. PENNA -  · PDF fileSTUDENT’S SOLUTIONS MANUAL JUDITH A. PENNA Indiana University Purdue University Indianapolis COLLEGE ALGEBRA: GRAPHS AND MODELS FIFTH EDITION

4

2

�2

�4

64�2 x

y

x � 4y � 5

4

2

�4

2�2�6 x

y

2x � 5y � �10

�2

�8

�6

�4

42�2�4 x

y

y � �x 2

6

4

2

�2

42�2�4 x

y

y � x 2 � 3

y

x�8 �4 4 8

�8

�12

�4

4

y � �x2 � 2x � 3

28 Chapter 1: Graphs, Functions, and Models

31. Graph x− 4y = 5.

Make a table of values, plot the points in the table, anddraw the graph.

x y (x, y)

−3 −2 (−3,−2)

1 −1 (1,−1)

5 0 (5, 0)

33. Graph 2x + 5y = −10.

In this case, it is convenient to find the intercepts alongwith a third point on the graph. Make a table of values,plot the points in the table, and draw the graph.

x y (x, y)

−5 0 (−5, 0)

0 −2 (0,−2)

5 −4 (5,−4)

35. Graph y = −x2.

Make a table of values, plot the points in the table, anddraw the graph.

x y (x, y)

−2 −4 (−2,−4)

−1 −1 (−1,−1)

0 0 (0, 0)

1 −1 (1,−1)

2 −4 (2,−4)

37. Graph y = x2 − 3.

Make a table of values, plot the points in the table, anddraw the graph.

x y (x, y)

−3 6 (−3, 6)

−1 −2 (−1,−2)

0 −3 (0,−3)

1 −2 (1,−2)

3 6 (3, 6)

39. Graph y = −x2 + 2x + 3.

Make a table of values, plot the points in the table, anddraw the graph.

x y (x, y)

−2 −5 (−2,−5)

−1 0 (−1, 0)

0 3 (0, 3)

1 4 (1, 4)

2 3 (2, 3)

3 0 (3, 0)

4 −5 (4,−5)

Copyright © 2013 Pearson Education, Inc.

Page 32: JUDITH A. PENNA -  · PDF fileSTUDENT’S SOLUTIONS MANUAL JUDITH A. PENNA Indiana University Purdue University Indianapolis COLLEGE ALGEBRA: GRAPHS AND MODELS FIFTH EDITION

�10 10

�10

10y � 2x � 1

�10 10

�10

104x � y � 7

�10 10

�10

10

y � ax � 2

�10 10

�10

102x � 3y � �5

�10 10

�10

10y � x2 � 6

�10 10

�10

10y � 2 � x2

�10 10

�10

10y � x2 � 4x � 2

Exercise Set 1.1 29

41. Graph (b) is the graph of y = 3 − x.

43. Graph (a) is the graph of y = x2 + 2x + 1.

45. Enter the equation, select the standard window, and graphthe equation as described in the Graphing Calculator Man-ual that accompanies the text.

47. First solve the equation for y: y = −4x + 7. Enter theequation in this form, select the standard window, andgraph the equation as described in the Graphing Calcula-tor Manual that accompanies the text.

49. Enter the equation, select the standard window, and graphthe equation as described in the Graphing Calculator Man-ual that accompanies the text.

51. First solve the equation for y.2x + 3y = −5

3y = −2x− 5

y =−2x− 5

3, or

13(−2x− 5)

Enter the equation in “y =” form, select the standard win-dow, and graph the equation as described in the GraphingCalculator Manual that accompanies the text.

53. Enter the equation, select the standard window, and graphthe equation as described in the Graphing Calculator Man-ual that accompanies the text.

55. Enter the equation, select the standard window, and graphthe equation as described in the Graphing Calculator Man-ual that accompanies the text.

57. Enter the equation, select the standard window, and graphthe equation as described in the Graphing Calculator Man-ual that accompanies the text.

59. Standard window:

[−4, 4,−4, 4]

We see that the standard window is a better choice for thisgraph.

Copyright © 2013 Pearson Education, Inc.

Page 33: JUDITH A. PENNA -  · PDF fileSTUDENT’S SOLUTIONS MANUAL JUDITH A. PENNA Indiana University Purdue University Indianapolis COLLEGE ALGEBRA: GRAPHS AND MODELS FIFTH EDITION

30 Chapter 1: Graphs, Functions, and Models

61. Standard window:

[−1, 1,−0.3, 0.3], Xscl = 0.1, Yscl = 0.1

We see that [−1, 1,−0.3, 0.3] is a better choice for thisgraph.

63. Either point can be considered as (x1, y1).

d =√

(4 − 5)2 + (6 − 9)2

=√

(−1)2 + (−3)2 =√

10 ≈ 3.162

65. Either point can be considered as (x1, y1).

d =√

(−13 − (−8))2 + (1 − (−11))2

=√

(−5)2 + 122 =√

169 = 13

67. Either point can be considered as (x1, y1).

d =√

(6 − 9)2 + (−1 − 5)2

=√

(−3)2 + (−6)2 =√

45 ≈ 6.708

69. Either point can be considered as (x1, y1).

d =

√(−8 − 8)2 +

(711

− 711

)2

=√

(−16)2 + 02 = 16

71. d =

√[− 3

5−(− 3

5

)]2+(− 4 − 2

3

)2

=

√02 +

(− 14

3

)2

=143

73. Either point can be considered as (x1, y1).

d =√

(−4.2 − 2.1)2 + [3 − (−6.4)]2

=√

(−6.3)2 + (9.4)2 =√

128.05 ≈ 11.316

75. Either point can be considered as (x1, y1).

d =√

(0 − a)2 + (0 − b)2 =√a2 + b2

77. First we find the length of the diameter:

d =√

(−3 − 9)2 + (−1 − 4)2

=√

(−12)2 + (−5)2 =√

169 = 13

The length of the radius is one-half the length of the di-

ameter, or12(13), or 6.5.

79. First we find the distance between each pair of points.

For (−4, 5) and (6, 1):

d =√

(−4 − 6)2 + (5 − 1)2

=√

(−10)2 + 42 =√

116

For (−4, 5) and (−8,−5):

d =√

(−4 − (−8))2 + (5 − (−5))2

=√

42 + 102 =√

116

For (6, 1) and (−8,−5):

d =√

(6 − (−8))2 + (1 − (−5))2

=√

142 + 62 =√

232

Since (√

116)2 + (√

116)2 = (√

232)2, the points could bethe vertices of a right triangle.

81. First we find the distance between each pair of points.

For (−4, 3) and (0, 5):

d =√

(−4 − 0)2 + (3 − 5)2

=√

(−4)2 + (−2)2 =√

20

For (−4, 3) and (3,−4):

d =√

(−4 − 3)2 + [3 − (−4)]2

=√

(−7)2 + 72 =√

98

For (0, 5) and (3,−4):

d =√

(0 − 3)2 + [5 − (−4)]2

=√

(−3)2 + 92 =√

90

The greatest distance is√

98, so if the points are the ver-tices of a right triangle, then it is the hypotenuse. But(√

20)2 + (√

90)2 �= (√

98)2, so the points are not the ver-tices of a right triangle.

83. We use the midpoint formula.(4 + (−12)

2,−9 + (−3)

2

)=(− 8

2,−12

2

)= (−4,−6)

85. We use the midpoint formula.

(0 +(− 2

5

)2

,

12− 0

2

)=

(−25

2,

122

)=(− 1

5,14

)

87. We use the midpoint formula.(6.1 + 3.8

2,−3.8 + (−6.1)

2

)=(

9.92

,−9.92

)=

(4.95,−4.95)

89. We use the midpoint formula.(−6 + (−6)2

,5 + 8

2

)=(− 12

2,132

)=(− 6,

132

)

91. We use the midpoint formula.(−16

+(− 2

3

)2

,−3

5+

54

2

)=

(−56

2,

13202

)=

(− 5

12,1340

)

Copyright © 2013 Pearson Education, Inc.

Page 34: JUDITH A. PENNA -  · PDF fileSTUDENT’S SOLUTIONS MANUAL JUDITH A. PENNA Indiana University Purdue University Indianapolis COLLEGE ALGEBRA: GRAPHS AND MODELS FIFTH EDITION

8

6

4

2

�2

4 6�2�4 x

y

y

x

2

4

�2

�4

�2�4 42

x2 � y2 � 4

y

x

2

4

�2

�2�4 42

6

8

x2 � (y � 3)2 � 16

Exercise Set 1.1 31

93.

For the side with vertices (−3, 4) and (2,−1):(−3 + 22

,4 + (−1)

2

)=(− 1

2,32

)For the side with vertices (2,−1) and (5, 2):(

2 + 52

,−1 + 2

2

)=(

72,12

)For the side with vertices (5, 2) and (0, 7):(

5 + 02

,2 + 7

2

)=(

52,92

)For the side with vertices (0, 7) and (−3, 4):(

0 + (−3)2

,7 + 4

2

)=(− 3

2,112

)For the quadrilateral whose vertices are the points foundabove, the diagonals have endpoints(− 1

2,32

),(

52,92

)and

(72,12

),(− 3

2,112

).

We find the length of each of these diagonals.

For(− 1

2,32

),(

52,92

):

d =

√(− 1

2− 5

2

)2

+(

32− 9

2

)2

=√

(−3)2 + (−3)2 =√

18

For(

72,12

),(− 3

2,112

):

d =

√(72−(− 3

2

))2

+(

12− 11

2

)2

=√

52 + (−5)2 =√

50

Since the diagonals do not have the same lengths, the mid-points are not vertices of a rectangle.

95. We use the midpoint formula.(√7 +

√2

2,−4 + 3

2

)=(√

7 +√

22

,−12

)

97. Square the viewing window. For the graph shown, onepossibility is [−12, 9,−4, 10].

99. (x− h)2 + (y − k)2 = r2

(x− 2)2 + (y − 3)2 =(

53

)2

Substituting

(x− 2)2 + (y − 3)2 =259

101. The length of a radius is the distance between (−1, 4) and(3, 7):

r =√

(−1 − 3)2 + (4 − 7)2

=√

(−4)2 + (−3)2 =√

25 = 5

(x− h)2 + (y − k)2 = r2

[x− (−1)]2 + (y − 4)2 = 52

(x + 1)2 + (y − 4)2 = 25

103. The center is the midpoint of the diameter:(7 + (−3)

2,13 + (−11)

2

)= (2, 1)

Use the center and either endpoint of the diameter to findthe length of a radius. We use the point (7, 13):

r =√

(7 − 2)2 + (13 − 1)2

=√

52 + 122 =√

169 = 13

(x− h)2 + (y − k)2 = r2

(x− 2)2 + (y − 1)2 = 132

(x− 2)2 + (y − 1)2 = 169

105. Since the center is 2 units to the left of the y-axis and thecircle is tangent to the y-axis, the length of a radius is 2.

(x− h)2 + (y − k)2 = r2

[x− (−2)]2 + (y − 3)2 = 22

(x + 2)2 + (y − 3)2 = 4

107. x2 + y2 = 4

(x− 0)2 + (y − 0)2 = 22

Center: (0, 0); radius: 2

109. x2 + (y − 3)2 = 16

(x− 0)2 + (y − 3)2 = 42

Center: (0, 3); radius: 4

Copyright © 2013 Pearson Education, Inc.

Page 35: JUDITH A. PENNA -  · PDF fileSTUDENT’S SOLUTIONS MANUAL JUDITH A. PENNA Indiana University Purdue University Indianapolis COLLEGE ALGEBRA: GRAPHS AND MODELS FIFTH EDITION

(x � 1)2 � (y � 5)2 � 36

y

x

4

8

�4

�4�8 84

12

(x � 4)2 � (y � 5)2 � 9

y

x

2

�2

�4

�2�4 2�6

�6

�8

32 Chapter 1: Graphs, Functions, and Models

111. (x− 1)2 + (y − 5)2 = 36

(x− 1)2 + (y − 5)2 = 62

Center: (1, 5); radius: 6

113. (x + 4)2 + (y + 5)2 = 9

[x− (−4)]2 + [y − (−5)]2 = 32

Center: (−4,−5); radius: 3

115. From the graph we see that the center of the circle is(−2, 1) and the radius is 3. The equation of the circleis [x− (−2)]2 + (y − 1)2 = 32, or (x+ 2)2 + (y − 1)2 = 32.

117. From the graph we see that the center of the circle is(5,−5) and the radius is 15. The equation of the circleis (x−5)2 +[y− (−5)]2 = 152, or (x−5)2 +(y+5)2 = 152.

119. If the point (p, q) is in the fourth quadrant, then p > 0and q < 0. If p > 0, then −p < 0 so both coordinates ofthe point (q,−p) are negative and (q,−p) is in the thirdquadrant.

121. Use the distance formula. Either point can be consideredas (x1, y1).

d =√

(a + h− a)2 + (√a + h−√

a)2

=√

h2 + a + h− 2√a2 + ah + a

=√

h2 + 2a + h− 2√a2 + ah

Next we use the midpoint formula.(a+a+h

2,

√a+

√a+h

2

)=(

2a+h

2,

√a+

√a+h

2

)

123. First use the formula for the area of a circle to find r2:A = πr2

36π = πr2

36 = r2

Then we have:(x− h)2 + (y − k)2 = r2

(x− 2)2 + [y − (−7)]2 = 36

(x− 2)2 + (y + 7)2 = 36

125. Let (0, y) be the required point. We set the distance from(−2, 0) to (0, y) equal to the distance from (4, 6) to (0, y)and solve for y.√

[0 − (−2)]2 + (y − 0)2 =√

(0 − 4)2 + (y − 6)2√4 + y2 =

√16 + y2 − 12y + 36

4 + y2 = 16 + y2 − 12y + 36Squaring both sides

−48 = −12y

4 = y

The point is (0, 4).

127. a) When the circle is positioned on a coordinate systemas shown in the text, the center lies on the y-axisand is equidistant from (−4, 0) and (0, 2).

Let (0, y) be the coordinates of the center.√(−4−0)2+(0−y)2 =

√(0−0)2+(2−y)2

42 + y2 = (2 − y)2

16 + y2 = 4 − 4y + y2

12 = −4y

−3 = y

The center of the circle is (0,−3).

b) Use the point (−4, 0) and the center (0,−3) to findthe radius.

(−4 − 0)2 + [0 − (−3)]2 = r2

25 = r2

5 = r

The radius is 5 ft.

129. x2 + y2 = 1(√3

2

)2

+(− 1

2

)2

? 1∣∣34

+14

∣∣∣∣∣1∣∣ 1 TRUE(√

32

,−12

)lies on the unit circle.

131. x2 + y2 = 1(−

√2

2

)2

+(√

22

)2

? 1∣∣24

+24

∣∣∣∣∣1∣∣ 1 TRUE(

−√

22

,

√2

2

)lies on the unit circle.

133. See the answer section in the text.

Copyright © 2013 Pearson Education, Inc.

Page 36: JUDITH A. PENNA -  · PDF fileSTUDENT’S SOLUTIONS MANUAL JUDITH A. PENNA Indiana University Purdue University Indianapolis COLLEGE ALGEBRA: GRAPHS AND MODELS FIFTH EDITION

Exercise Set 1.2 33

Exercise Set 1.2

1. This correspondence is a function, because each memberof the domain corresponds to exactly one member of therange.

3. This correspondence is a function, because each memberof the domain corresponds to exactly one member of therange.

5. This correspondence is not a function, because there is amember of the domain (m) that corresponds to more thanone member of the range (A and B).

7. This correspondence is a function, because each memberof the domain corresponds to exactly one member of therange.

9. This correspondence is a function, because each car hasexactly one license number.

11. This correspondence is a function, because each integerless than 9 corresponds to exactly one multiple of 5.

13. This correspondence is not a function, because at least onestudent will have more than one neighboring seat occupiedby another student.

15. The relation is a function, because no two ordered pairshave the same first coordinate and different second coor-dinates.

The domain is the set of all first coordinates:{2, 3, 4}.The range is the set of all second coordinates: {10, 15, 20}.

17. The relation is not a function, because the ordered pairs(−2, 1) and (−2, 4) have the same first coordinate and dif-ferent second coordinates.

The domain is the set of all first coordinates:{−7,−2, 0}.The range is the set of all second coordinates: {3, 1, 4, 7}.

19. The relation is a function, because no two ordered pairshave the same first coordinate and different second coor-dinates.

The domain is the set of all first coordinates:{−2, 0, 2, 4,−3}.The range is the set of all second coordinates: {1}.

21. g(x) = 3x2 − 2x + 1

a) g(0) = 3 · 02 − 2 · 0 + 1 = 1

b) g(−1) = 3(−1)2 − 2(−1) + 1 = 6

c) g(3) = 3 · 32 − 2 · 3 + 1 = 22

d) g(−x) = 3(−x)2 − 2(−x) + 1 = 3x2 + 2x + 1

e) g(1 − t) = 3(1 − t)2 − 2(1 − t) + 1 =

3(1−2t+t2)−2(1−t)+1 = 3−6t+3t2−2+2t+1 =

3t2 − 4t + 2

23. g(x) = x3

a) g(2) = 23 = 8

b) g(−2) = (−2)3 = −8

c) g(−x) = (−x)3 = −x3

d) g(3y) = (3y)3 = 27y3

e) g(2 + h) = (2 + h)3 = 8 + 12h + 6h2 + h3

25. g(x) =x− 4x + 3

a) g(5) =5 − 45 + 3

=18

b) g(4) =4 − 44 + 7

= 0

c) g(−3) =−3 − 4−3 + 3

=−70

Since division by 0 is not defined, g(−3) does notexist.

d) g(−16.25) =−16.25 − 4−16.25 + 3

=−20.25−13.25

=8153

≈ 1.5283

e) g(x + h) =x + h− 4x + h + 3

27. g(x) =x√

1 − x2

g(0) =0√

1 − 02=

0√1

=01

= 0

g(−1) =−1√

1 − (−1)2=

−1√1 − 1

=−1√

0=

−10

Since division by 0 is not defined, g(−1) does not exist.

g(5) =5√

1 − 52=

5√1 − 25

=5√−24

Since√−24 is not defined as a real number, g(5) does not

exist as a real number.

g

(12

)=

12√

1 −(

12

)2=

12√

1 − 14

=

12√34

=

12√3

2

=12· 2√

3=

1 · 22√

3=

1√3, or

√3

3

29.

Rounding to the nearest tenth, we see that g(−2.1) ≈−21.8, g(5.08) ≈ −130.4, and g(10.003) ≈ −468.3.

31. Graph f(x) =12x + 3.

We select values for x and find the corresponding valuesof f(x). Then we plot the points and connect them witha smooth curve.

Copyright © 2013 Pearson Education, Inc.

Page 37: JUDITH A. PENNA -  · PDF fileSTUDENT’S SOLUTIONS MANUAL JUDITH A. PENNA Indiana University Purdue University Indianapolis COLLEGE ALGEBRA: GRAPHS AND MODELS FIFTH EDITION

4

2

�2

�4

42�2�4 x

y

f(x) � x � 31 2

4

2

�2

�4

42�2�4 x

y

f(x) � �x2 � 4

4

2

�2

�4

42�2�4 x

y

f(x) � �x � 1

y

x

34 Chapter 1: Graphs, Functions, and Models

x f(x) (x, f(x))

−4 1 (−4, 1)

0 3 (0, 3)

2 4 (2, 4)

33. Graph f(x) = −x2 + 4.

We select values for x and find the corresponding valuesof f(x). Then we plot the points and connect them witha smooth curve.

x f(x) (x, f(x))

−3 −5 (−3,−5)

−2 0 (−2, 0)

−1 3 (−1, 3)

0 4 (0, 4)

1 3 (1, 3)

2 0 (2, 0)

3 −5 (3,−5)

35. Graph f(x) =√x− 1.

We select values for x and find the corresponding valuesof f(x). Then we plot the points and connect them witha smooth curve.

x f(x) (x, f(x))

1 0 (1, 0)

2 1 (2, 1)

4 1.7 (4, 1.7)

5 2 (5, 2)

37. From the graph we see that, when the input is 1, the outputis −2, so h(1) = −2. When the input is 3, the output is2, so h(3) = 2. When the input is 4, the output is 1, soh(4) = 1.

39. From the graph we see that, when the input is −4, theoutput is 3, so s(−4) = 3. When the input is −2, the

output is 0, so s(−2) = 0. When the input is 0, the outputis −3, so s(0) = −3.

41. From the graph we see that, when the input is −1, theoutput is 2, so f(−1) = 2. When the input is 0, the outputis 0, so f(0) = 0. When the input is 1, the output is −2,so f(1) = −2.

43. This is not the graph of a function, because we can find avertical line that crosses the graph more than once.

45. This is the graph of a function, because there is no verticalline that crosses the graph more than once.

47. This is the graph of a function, because there is no verticalline that crosses the graph more than once.

49. This is not the graph of a function, because we can find avertical line that crosses the graph more than once.

51. We can substitute any real number for x. Thus, the do-main is the set of all real numbers, or (−∞,∞).

53. We can substitute any real number for x. Thus, the do-main is the set of all real numbers, or (−∞,∞).

55. The input 0 results in a denominator of 0. Thus, the do-main is {x|x �= 0}, or (−∞, 0) ∪ (0,∞).

57. We can substitute any real number in the numerator, butwe must avoid inputs that make the denominator 0. Wefind these inputs.

2 − x = 0

2 = x

The domain is {x|x �= 2}, or (−∞, 2) ∪ (2,∞).

59. We find the inputs that make the denominator 0:

x2 − 4x− 5 = 0

(x− 5)(x + 1) = 0

x− 5 = 0 or x + 1 = 0

x = 5 or x = −1

The domain is {x|x �= 5 and x �= −1}, or(−∞,−1) ∪ (−1, 5) ∪ (5,∞).

61. We can substitute any real number for x. Thus, the do-main is the set of all real numbers, or (−∞,∞).

63. We can substitute any real number in the numerator, butwe must avoid inputs that make the denominator 0. Wefind these inputs.

Copyright © 2013 Pearson Education, Inc.

Page 38: JUDITH A. PENNA -  · PDF fileSTUDENT’S SOLUTIONS MANUAL JUDITH A. PENNA Indiana University Purdue University Indianapolis COLLEGE ALGEBRA: GRAPHS AND MODELS FIFTH EDITION

Exercise Set 1.2 35

x2 − 7x = 0

x(x− 7) = 0

x = 0 or x− 7 = 0

x = 0 or x = 7

The domain is {x|x �= 0 and x �= 7}, or (−∞, 0) ∪ (0, 7) ∪(7,∞).

65. We can substitute any real number for x. Thus, the do-main is the set of all real numbers, or (−∞,∞).

67. The inputs on the x-axis that correspond to points on thegraph extend from 0 to 5, inclusive. Thus, the domain is{x|0 ≤ x ≤ 5}, or [0, 5].

The outputs on the y-axis extend from 0 to 3, inclusive.Thus, the range is {y|0 ≤ y ≤ 3}, or [0, 3].

69. The inputs on the x-axis that correspond to points on thegraph extend from −2π to 2π inclusive. Thus, the domainis {x| − 2π ≤ x ≤ 2π}, or [−2π, 2π].

The outputs on the y-axis extend from −1 to 1, inclusive.Thus, the range is {y| − 1 ≤ y ≤ 1}, or [−1, 1].

71. The graph extends to the left and to the right withoutbound. Thus, the domain is the set of all real numbers, or(−∞,∞).

The only output is −3, so the range is {−3}.73. The inputs on the x-axis extend from −5 to 3, inclusive.

Thus, the domain is [−5, 3].

The outputs on the y-axis extend from −2 to 2, inclusive.Thus, the range is [−2, 2].

75.

To find the domain we look for the inputs on the x-axisthat correspond to a point on the graph. We see that eachpoint on the x-axis corresponds to a point on the graph sothe domain is the set of all real numbers, or (−∞,∞).

To find the range we look for outputs on the y-axis. Thenumber 0 is the smallest output, and every number greaterthan 0 is also an output. Thus, the range is [0,∞).

77.

We see that each point on the x-axis corresponds to a pointon the graph so the domain is the set of all real numbers,or (−∞,∞). We also see that each point on the y-axiscorresponds to an output so the range is the set of all realnumbers, or (−∞,∞).

79.

We see that each point on the x-axis except 3 correspondsto a point on the graph, so the domain is (−∞, 3)∪(3,∞).We also see that each point on the y-axis except 0 corre-sponds to an output, so the range is (−∞, 0) ∪ (0,∞).

81.

Each point on the x-axis corresponds to a point on thegraph, so the domain is the set of all real numbers, or(−∞,∞).

Each point on the y-axis also corresponds to a point on thegraph, so the range is the set of all real numbers, (−∞,∞).

83.

The largest input on the x-axis is 7 and every number lessthan 7 is also an input. Thus, the domain is (−∞, 7].

The number 0 is the smallest output, and every numbergreater than 0 is also an output. Thus, the range is [0,∞).

85.

Each point on the x-axis corresponds to a point on thegraph, so the domain is the set of all real numbers, or(−∞,∞).

The largest output is 3 and every number less than 3 isalso an output. Thus, the range is (−∞, 3].

87. a) In 2018, x = 2018 − 1985 = 33.

V (33) = 0.4652(33) + 10.87 ≈ $26.22

In 2025, x = 2025 − 1985 = 40.

V (40) = 0.4652(40) + 10.87 ≈ $29.48

Copyright © 2013 Pearson Education, Inc.

Page 39: JUDITH A. PENNA -  · PDF fileSTUDENT’S SOLUTIONS MANUAL JUDITH A. PENNA Indiana University Purdue University Indianapolis COLLEGE ALGEBRA: GRAPHS AND MODELS FIFTH EDITION

y

x

2

4

�2

�4

�2�4 42

y � (x �1)2

y

x

2

4

�2

�4

�2�4 42

�2x � 5y � 10

36 Chapter 1: Graphs, Functions, and Models

b)Substitute 40 for V (x) and solve for x.

40 = 0.4652x + 10.87

29.13 = 0.4652x

63 ≈ x

It will take approximately $40 to equal the value of$1 in 1913 about 63 yr after 1985, or in 2048.

89. E(t) = 1000(100 − t) + 580(100 − t)2

a) E(99.5) = 1000(100−99.5)+580(100−99.5)2

= 1000(0.5) + 580(0.5)2

= 500 + 580(0.25) = 500 + 145

= 645 m above sea level

b) E(100) = 1000(100 − 100) + 580(100 − 100)2

= 1000 · 0 + 580(0)2 = 0 + 0

= 0 m above sea level, or at sea level

91. To determine whether (0,−7) is a solution, substitute 0for x and −7 for y.

y = 0.5x + 7

−7 ? 0.5(0) + 7∣∣∣ 0 + 7−7

∣∣ 7 FALSE

The equation −7 = 7 is false, so (0,−7) is not a solution.

To determine whether (8, 11) is a solution, substitute 8 forx and 11 for y.

y = 0.5x + 7

11 ? 0.5(8) + 7∣∣∣ 4 + 711∣∣ 11 TRUE

The equation 11 = 11 is true, so (8, 11) is a solution.

93. Graph y = (x− 1)2.

Make a table of values, plot the points in the table, anddraw the graph.

x y (x, y)

−1 4 (−1, 4)

0 1 (0, 1)

1 0 (1, 0)

2 1 (2, 1)

3 4 (3, 4)

95. Graph −2x− 5y = 10.

Make a table of values, plot the points in the table, anddraw the graph.

x y (x, y)

−5 0 (−5, 0)

0 −2 (0,−2)

5 −4 (5,−4)

97. We find the inputs for which 2x + 5 is nonnegative.2x + 5 ≥ 0

2x ≥ −5

x ≥ −52

Thus, the domain is{x

∣∣∣∣x ≥ −52

}, or

[− 5

2,∞)

.

99.√x + 6 is not defined for values of x for which x + 6 is

negative. We find the inputs for which x+6 is nonnegative.x + 6 ≥ 0

x ≥ −6

We must also avoid inputs that make the denominator 0.

(x + 2)(x− 3) = 0

x + 2 = 0 or x− 3 = 0

x = −2 or x = 3

Then the domain is {x|x ≥ −6 and x �= −2 and x �= 3},or [−6,−2) ∪ (−2, 3) ∪ (3,∞).

101. Answers may vary. Two possibilities are f(x) = x, g(x) =x + 1 and f(x) = x2, g(x) = x2 − 4.

103. First find the value of x for which x + 3 = −1.x + 3 = −1

x = −4

Then we have:

g(x + 3) = 2x + 1

g(−1) = g(−4 + 3) = 2(−4) + 1 = −8 + 1 = −7

Exercise Set 1.3

1. a) Yes. Each input is 1 more than the one that pre-cedes it.

b) Yes. Each output is 3 more than the one that pre-cedes it.

c) Yes. Constant changes in inputs result in constantchanges in outputs.

3. a) Yes. Each input is 15 more than the one that pre-cedes it.

b) No. The change in the outputs varies.

c) No. Constant changes in inputs do not result inconstant changes in outputs.

Copyright © 2013 Pearson Education, Inc.

Page 40: JUDITH A. PENNA -  · PDF fileSTUDENT’S SOLUTIONS MANUAL JUDITH A. PENNA Indiana University Purdue University Indianapolis COLLEGE ALGEBRA: GRAPHS AND MODELS FIFTH EDITION

Exercise Set 1.3 37

5. Two points on the line are (−4,−2) and (1, 4).

m =y2 − y1

x2 − x1=

4 − (−2)1 − (−4)

=65

7. Two points on the line are (0, 3) and (5, 0).

m =y2 − y1

x2 − x1=

0 − 35 − 0

=−35

, or −35

9. m =y2 − y1

x2 − x1=

3 − 33 − 0

=03

= 0

11. m =y2 − y1

x2 − x1=

2 − 4−1 − 9

=−2−10

=15

13. m =y2 − y1

x2 − x1=

6 − (−9)4 − 4

=150

Since division by 0 is not defined, the slope is not defined.

15. m =y2 − y1

x2 − x1=

−0.4 − (−0.1)−0.3 − 0.7

=−0.3−1

= 0.3

17. m =y2 − y1

x2 − x1=

−2 − (−2)4 − 2

=02

= 0

19. m =y2 − y1

x2 − x1=

35−(− 3

5

)−1

2− 1

2

=

65−1

= −65

21. m =y2 − y1

x2 − x1=

−5 − (−13)−8 − 16

=8

−24= −1

3

23. m =7 − (−7)

−10 − (−10)=

140

Since division by 0 is not defined, the slope is not defined.

25. We have the points (4, 3) and (−2, 15).

m =y2 − y1

x2 − x1=

15 − 3−2 − 4

=12−6

= −2

27. We have the points(

15

,12

)and

(− 1,−11

2

).

m =y2 − y1

x2 − x1=

−112

− 12

−1 − 15

=−6

−65

= −6 ·(− 5

6

)= 5

29. We have the points(− 6,

45

)and

(0,

45

).

m =y2 − y1

x2 − x1=

45− 4

5−6 − 0

=0−6

= 0

31. y = 1.3x − 5 is in the form y = mx + b with m = 1.3, sothe slope is 1.3.

33. The graph of x = −2 is a vertical line, so the slope is notdefined.

35. f(x) = −12

x + 3 is in the form y = mx + b with m = −12,

so the slope is −12.

37. y = 9− x can be written as y = −x + 9, or y = −1 · x + 9.Now we have an equation in the form y = mx + b withm = −1, so the slope is −1.

39. The graph of y = 0.7 is a horizontal line, so the slope is0. (We also see this if we write the equation in the formy = 0x + 0.7).

41. We have the points (2009, 945) and (1998, 425). We findthe average rate of change, or slope.

m =945 − 425

2009 − 1998=

52011

≈ 47.3

The average rate of change in the revenue of the fireworksindustry in the U.S. from 1998 to 2009 was about $47.3 mil-lion per year.

43. We have the points (2000, 348, 194) and (2010, 319, 294).We find the average rate of change, or slope.

m =348, 194 − 319, 294

2000 − 2010=

28, 900−10

= −2890

The average rate of change in the population in St. Louis,Missouri, from 2000 to 2010 was −2890 people per year.

45. We have the points (2009, 21.0) and (2012, 25.0). We findthe average rate of change, or slope.

m =21.0 − 25.02009 − 2012

=−4.0−3

≈ 1.3

The average rate of change in the sales of electric bikes inChina from 2009 to 2012 was about 1.3 million bikes peryear.

47. We have the points (1990, 3.1) and (2008, 5.5). We findthe average rate of change, or slope.

m =5.5 − 3.1

2008 − 1990=

2.418

≈ 0.13

The average rate of change in the consumption of broccoliper capita from 1990 to 2008 was about 0.13 lb per year.

49. y =35

x − 7

The equation is in the form y = mx + b where m =35

and b = −7. Thus, the slope is35, and the y-intercept is

(0,−7).

51. x = −25

This is the equation of a vertical line25

unit to the leftof the y-axis. The slope is not defined, and there is noy-intercept.

53. f(x) = 5 − 12

x, or f(x) = −12

x + 5

The second equation is in the form y = mx + b where

m = −12

and b = 5. Thus, the slope is −12

and the y-

intercept is (0, 5).

55. Solve the equation for y.

3x + 2y = 10

2y = −3x + 10

y = −32

x + 5

Slope: −32; y-intercept: (0, 5)

Copyright © 2013 Pearson Education, Inc.

Page 41: JUDITH A. PENNA -  · PDF fileSTUDENT’S SOLUTIONS MANUAL JUDITH A. PENNA Indiana University Purdue University Indianapolis COLLEGE ALGEBRA: GRAPHS AND MODELS FIFTH EDITION

2 4

2

4

�4 �2

�4

�2

x

y

12y � ��x � 3

2 4

2

4

�4 �2

�4

�2

x

y

f(x) � 3x � 1

2 4

2

�4 �2

�4

�6

�2

x

y

3x � 4y � 20

2 4

2

4

6

�4 �2

�2

x

y

x � 3y � 18

133

y � �x � 1

00

8000

300

38 Chapter 1: Graphs, Functions, and Models

57. y = −6 = 0 · x − 6

Slope: 0; y-intercept: (0,−6)

59. Solve the equation for y.

5y − 4x = 8

5y = 4x + 8

y =45

x +85

Slope:45; y-intercept:

(0,

85

)

61. Solve the equation for y.

4y − x + 2 = 0

4y = x − 2

y =14

x − 12

Slope:14; y-intercept:

(0,−1

2

)

63. Graph y = −12

x − 3.

Plot the y-intercept, (0,−3). We can think of the slope

as−12

. Start at (0,−3) and find another point by moving

down 1 unit and right 2 units. We have the point (2,−4).

We could also think of the slope as1−2

. Then we can start

at (0,−3) and get another point by moving up 1 unit andleft 2 units. We have the point (−2,−2). Connect thethree points to draw the graph.

65. Graph f(x) = 3x − 1.

Plot the y-intercept, (0,−1). We can think of the slope

as31. Start at (0,−1) and find another point by moving

up 3 units and right 1 unit. We have the point (1, 2). Wecan move from the point (1, 2) in a similar manner to geta third point, (2, 5). Connect the three points to draw thegraph.

67. First solve the equation for y.3x − 4y = 20

−4y = −3x + 20

y =34

x − 5

Plot the y-intercept, (0,−5). Then using the slope,34,

start at (0,−5) and find another point by moving up3 units and right 4 units. We have the point (4,−2). Wecan move from the point (4,−2) in a similar manner to geta third point, (8, 1). Connect the three points to draw thegraph.

69. First solve the equation for y.x + 3y = 18

3y = −x + 18

y = −13

x + 6

Plot the y-intercept, (0, 6). We can think of the slope as−13

. Start at (0, 6) and find another point by moving down

1 unit and right 3 units. We have the point (3, 5). We canmove from the point (3, 5) in a similar manner to get athird point, (6, 4). Connect the three points and draw thegraph.

71. a)

b) P (0) =133

· 0 + 1 = 1 atm

P (33) =133

· 33 + 1 = 2 atm

P (1000) =133

· 1000 + 1 = 311033

atm

Copyright © 2013 Pearson Education, Inc.

Page 42: JUDITH A. PENNA -  · PDF fileSTUDENT’S SOLUTIONS MANUAL JUDITH A. PENNA Indiana University Purdue University Indianapolis COLLEGE ALGEBRA: GRAPHS AND MODELS FIFTH EDITION

1110

12

y � �x � �

00

100

100

2 4

2

4

�4 �2

�4

�2

y

3x � 6y � 6

x

Chapter 1 Mid-Chapter Mixed Review 39

P (5000) =133

· 5000 + 1 = 1521733

atm

P (7000) =133

· 7000 + 1 = 213433

atm

73. a) D(r) =1110

r +12

The slope is1110

.

For each mph faster the car travels, it takes1110

ftlonger to stop.

b)

c) D(5) =1110

· 5 +12

=112

+12

=122

= 6 ft

D(10) =1110

· 10 +12

= 11 +12

= 1112, or 11.5 ft

D(20) =1110

· 20 +12

= 22 +12

= 2212, or 22.5 ft

D(50) =1110

· 50 +12

= 55 +12

= 5512, or 55.5 ft

D(65) =1110

· 65 +12

=1432

+12

=1442

= 72 ft

d) The speed cannot be negative. D(0) =12

which

says that a stopped car travels12

ft before stop-ping. Thus, 0 is not in the domain. The speed canbe positive, so the domain is {r|r > 0}, or (0,∞).

75. C(t) = 89 + 114.99t

C(24) = 89 + 114.99(24) = $2848.76

77. C(x) = 750 + 15x

C(32) = 750 + 15 · 32 = $1230

79. f(x) = x2 − 3x

f

(12

)=(

12

)2

− 3 · 12

=14− 3

2= −5

4

81. f(x) = x2 − 3x

f(−5) = (−5)2 − 3(−5) = 25 + 15 = 40

83. f(x) = x2 − 3x

f(a + h) = (a + h)2 − 3(a + h) = a2 + 2ah + h2 − 3a− 3h

85. m =y2 − y1

x2 − x1=

(a+h)2−a2

a + h− a=

a2+2ah+h2−a2

h=

2ah + h2

h=

h(2a + h)h

= 2a + h

87. False. For example, let f(x) = x + 1. Then f(c − d) =c− d + 1, but f(c) − f(d) = c + 1 − (d + 1) = c− d.

89. f(x) = mx + b

f(x + 2) = f(x) + 2

m(x + 2) + b = mx + b + 2

mx + 2m + b = mx + b + 2

2m = 2

m = 1

Thus, f(x) = 1 · x + b, or f(x) = x + b.

Chapter 1 Mid-Chapter Mixed Review

1. The statement is false. The x-intercept of a line that passesthrough the origin is (0, 0).

3. The statement is false. The line parallel to the y-axis thatpasses through (−5, 25) is x = −5.

5. Distance:d =

√(−8 − 3)2 + (−15 − 7)2

=√

(−11)2 + (−22)2

=√

121 + 484

=√

605 ≈ 24.6

Midpoint:(−8 + 3

2,−15 + 7

2

)=(−5

2,−82

)=(

− 52,−4

)

7. (x− h)2 + (y − k)2 = r2

(x− (−5))2 + (y − 2)2 = 132

(x + 5)2 + (y − 2)2 = 169

9. Graph 3x− 6y = 6.

We will find the intercepts along with a third point on thegraph. Make a table of values, plot the points, and drawthe graph.

x y (x, y)

2 0 (2, 0)

0 −1 (0,−1)

4 1 (4, 1)

11. Graph y = 2 − x2.

We choose some values for x and find the correspondingy-values. We list these points in a table, plot them, anddraw the graph.

Copyright © 2013 Pearson Education, Inc.

Page 43: JUDITH A. PENNA -  · PDF fileSTUDENT’S SOLUTIONS MANUAL JUDITH A. PENNA Indiana University Purdue University Indianapolis COLLEGE ALGEBRA: GRAPHS AND MODELS FIFTH EDITION

2 4

2

4

�4 �2

�4

�2

y

y � 2 � x2

x

2 4

2

4

�4 �2

�4

�2

g(x) � x2 � 1

x

y

40 Chapter 1: Graphs, Functions, and Models

x y (x, y)

−2 −2 (−2,−2)

−1 1 (−1, 1)

0 2 (0, 2)

1 1 (1, 1)

2 −2 (2,−2)

13. f(x) = x − 2x2

f(−4) = −4 − 2(−4)2 = −4 − 2 · 16 = −4 − 32 = −36

f(0) = 0 − 2 · 02 = 0 − 0 = 0

f(1) = 1 − 2 · 12 = 1 − 2 · 1 = 1 − 2 = −1

15. We can substitute any real number for x. Thus, the do-main is the set of all real numbers, or (−∞,∞).

17. We find the inputs for which the denominator is 0.

x2 + 2x − 3 = 0

(x + 3)(x − 1) = 0

x + 3 = 0 or x − 1 = 0

x = −3 or x = 1

The domain is {x|x = −3 and x = 1}, or(−∞,−3) ∪ (−3, 1) ∪ (1,∞).

19. Graph g(x) = x2 − 1.

Make a table of values, plot the points in the table, anddraw the graph.

x g(x) (x, g(x))

−2 3 (−2, 3)

−1 0 (−1, 0)

0 −1 (0,−1)

1 0 (1, 0)

2 3 (2, 3)

21. m =y2 − y1

x2 − x1=

−5 − 13−2 − (−2)

=−180

Since division by 0 is not defined, the slope is not defined.

23. m =y2 − y1

x2 − x1=

13− 1

327− 5

7

=0

−37

= 0

25. We can write y = −6 as y = 0x − 6, so the slope is 0 andthe y-intercept is (0,−6).

27. 3x − 16y + 1 = 0

3x + 1 = 16y

316

x +116

= y

Slope:316

; y-intercept:(

0,116

)

29. A vertical line (x = a) crosses the graph more than once.

31. Let A = (a, b) and B = (c, d). The coordinates of a point

C one-half of the way from A to B are(

a + c

2,b + d

2

).

A point D that is one-half of the way from C to B is12

+12· 12, or

34

of the way from A to B. Its coordinates

are( a+c

2 + c

2,b+d2 + d

2

), or

(a + 3c

4,b + 3d

4

). Then a

point E that is one-half of the way from D to B is34

+12· 14, or

78

of the way from A to B. Its coordinates

are( a+3c

4 + c

2,b+3d

4 + d

2

), or

(a + 7c

8,b + 7d

8

).

Exercise Set 1.4

1. We see that the y-intercept is (0,−2). Another point onthe graph is (1, 2). Use these points to find the slope.

m =y2 − y1

x2 − x1=

2 − (−2)1 − 0

=41

= 4

We have m = 4 and b = −2, so the equation is y = 4x− 2.

3. We see that the y-intercept is (0, 0). Another point on thegraph is (3,−3). Use these points to find the slope.

m =y2 − y1

x2 − x1=

−3 − 03 − 0

=−33

= −1

We have m = −1 and b = 0, so the equation isy = −1 · x + 0, or y = −x.

5. We see that the y-intercept is (0,−3). This is a horizontalline, so the slope is 0. We have m = 0 and b = −3, so theequation is y = 0 · x − 3, or y = −3.

7. We substitute29

for m and 4 for b in the slope-interceptequation.

y = mx + b

y =29

x + 4

9. We substitute −4 for m and −7 for b in the slope-interceptequation.

y = mx + b

y = −4x − 7

11. We substitute −4.2 for m and34

for b in the slope-interceptequation.

y = mx + b

y = −4.2x +34

Copyright © 2013 Pearson Education, Inc.

Page 44: JUDITH A. PENNA -  · PDF fileSTUDENT’S SOLUTIONS MANUAL JUDITH A. PENNA Indiana University Purdue University Indianapolis COLLEGE ALGEBRA: GRAPHS AND MODELS FIFTH EDITION

Exercise Set 1.4 41

13. Using the point-slope equation:

y − y1 = m(x − x1)

y − 7 =29(x − 3) Substituting

y − 7 =29

x − 23

y =29

x +193

Slope-intercept equation

Using the slope-intercept equation:

Substitute29

for m, 3 for x, and 7 for y in the slope-intercept equation and solve for b.

y = mx + b

7 =29· 3 + b

7 =23

+ b

193

= b

Now substitute29

for m and193

for b in y = mx + b.

y =29

x +193

15. The slope is 0 and the second coordinate of the given pointis 8, so we have a horizontal line 8 units above the x-axis.Thus, the equation is y = 8.

We could also use the point-slope equation or the slope-intercept equation to find the equation of the line.

Using the point-slope equation:

y − y1 = m(x − x1)

y − 8 = 0(x − (−2)) Substituting

y − 8 = 0

y = 8

Using the slope-intercept equation:

y = mx + b

y = 0(−2) + 8

y = 8

17. Using the point-slope equation:

y − y1 = m(x − x1)

y − (−1) = −35(x − (−4))

y + 1 = −35(x + 4)

y + 1 = −35

x − 125

y = −35

x − 175

Slope-intercept

equation

Using the slope-intercept equation:

y = mx + b

−1 = −35(−4) + b

−1 =125

+ b

−175

= b

Then we have y = −35

x − 175

.

19. First we find the slope.

m =−4 − 5

2 − (−1)=

−93

= −3

Using the point-slope equation:

Using the point (−1, 5), we get

y − 5 = −3(x − (−1)), or y − 5 = −3(x + 1).

Using the point (2,−4), we get

y − (−4) = −3(x − 2), or y + 4 = −3(x − 2).

In either case, the slope-intercept equation isy = −3x + 2.

Using the slope-intercept equation and the point (−1, 5):

y = mx + b

5 = −3(−1) + b

5 = 3 + b

2 = b

Then we have y = −3x + 2.

21. First we find the slope.

m =4 − 0−1 − 7

=4−8

= −12

Using the point-slope equation:

Using the point (7, 0), we get

y − 0 = −12(x − 7).

Using the point (−1, 4), we get

y − 4 = −12(x − (−1)), or

y − 4 = −12(x + 1).

In either case, the slope-intercept equation is

y = −12

x +72.

Using the slope-intercept equation and the point (7, 0):

0 = −12· 7 + b

72

= b

Then we have y = −12

x +72.

23. First we find the slope.

m =−4 − (−6)

3 − 0=

23

Copyright © 2013 Pearson Education, Inc.

Page 45: JUDITH A. PENNA -  · PDF fileSTUDENT’S SOLUTIONS MANUAL JUDITH A. PENNA Indiana University Purdue University Indianapolis COLLEGE ALGEBRA: GRAPHS AND MODELS FIFTH EDITION

42 Chapter 1: Graphs, Functions, and Models

We know the y-intercept is (0,−6), so we substitute in theslope-intercept equation.

y = mx + b

y =23

x − 6

25. First we find the slope.

m =7.3 − 7.3−4 − 0

=0−4

= 0

We know the y-intercept is (0, 7.3), so we substitute inthe slope-intercept equation.

y = mx + b

y = 0 · x + 7.3

y = 7.3

27. The equation of the horizontal line through (0,−3) is ofthe form y = b where b is −3. We have y = −3.

The equation of the vertical line through (0,−3) is of theform x = a where a is 0. We have x = 0.

29. The equation of the horizontal line through(

211

,−1)

is

of the form y = b where b is −1. We have y = −1.

The equation of the vertical line through(

211

,−1)

is of

the form x = a where a is211

. We have x =211

.

31. We have the points (1, 4) and (−2, 13). First we find theslope.

m =13 − 4−2 − 1

=9−3

= −3

We will use the point-slope equation, choosing (1, 4) forthe given point.

y − 4 = −3(x − 1)

y − 4 = −3x + 3

y = −3x + 7, or

h(x) = −3x + 7

Then h(2) = −3 · 2 + 7 = −6 + 7 = 1.

33. We have the points (5, 1) and (−5,−3). First we find theslope.

m =−3 − 1−5 − 5

=−4−10

=25

We will use the slope-intercept equation, choosing (5, 1)for the given point.

y = mx + b

1 =25· 5 + b

1 = 2 + b

−1 = b

Then we have f(x) =25

x − 1.

Now we find f(0).

f(0) =25· 0 − 1 = −1.

35. The slopes are263

and − 326

. Their product is −1, so thelines are perpendicular.

37. The slopes are25

and −25. The slopes are not the same and

their product is not −1, so the lines are neither parallel norperpendicular.

39. We solve each equation for y.

x + 2y = 5 2x + 4y = 8

y = −12

x +52

y = −12

x + 2

We see that m1 = −12

and m2 = −12. Since the slopes are

the same and the y-intercepts,52

and 2, are different, thelines are parallel.

41. We solve each equation for y.

y = 4x − 5 4y = 8 − x

y = −14

x + 2

We see that m1 = 4 and m2 = −14. Since

m1m2 = 4(− 1

4

)= −1, the lines are perpendicular.

43. y =27

x + 1; m =27

The line parallel to the given line will have slope27. We

use the point-slope equation for a line with slope27

and

containing the point (3, 5):

y − y1 = m(x − x1)

y − 5 =27(x − 3)

y − 5 =27

x − 67

y =27

x +297

Slope-intercept form

The slope of the line perpendicular to the given line is the

opposite of the reciprocal of27, or −7

2. We use the point-

slope equation for a line with slope −72

and containing the

point (3, 5):

y − y1 = m(x − x1)

y − 5 = −72(x − 3)

y − 5 = −72

x +212

y = −72

x +312

Slope-intercept form

45. y = −0.3x + 4.3; m = −0.3

The line parallel to the given line will have slope −0.3. Weuse the point-slope equation for a line with slope −0.3 andcontaining the point (−7, 0):

Copyright © 2013 Pearson Education, Inc.

Page 46: JUDITH A. PENNA -  · PDF fileSTUDENT’S SOLUTIONS MANUAL JUDITH A. PENNA Indiana University Purdue University Indianapolis COLLEGE ALGEBRA: GRAPHS AND MODELS FIFTH EDITION

Exercise Set 1.4 43

y − y1 = m(x − x1)

y − 0 = −0.3(x − (−7))

y = −0.3x − 2.1 Slope-intercept form

The slope of the line perpendicular to the given line is the

opposite of the reciprocal of −0.3, or1

0.3=

103

.

We use the point-slope equation for a line with slope103

and containing the point (−7, 0):

y − y1 = m(x − x1)

y − 0 =103

(x − (−7))

y =103

x +703

Slope-intercept form

47. 3x + 4y = 5

4y = −3x + 5

y = −34

x +54; m = −3

4

The line parallel to the given line will have slope −34. We

use the point-slope equation for a line with slope −34

and

containing the point (3,−2):

y − y1 = m(x − x1)

y − (−2) = −34(x − 3)

y + 2 = −34

x +94

y = −34

x +14

Slope-intercept form

The slope of the line perpendicular to the given line is the

opposite of the reciprocal of −34, or

43. We use the point-

slope equation for a line with slope43

and containing the

point (3,−2):

y − y1 = m(x − x1)

y − (−2) =43(x − 3)

y + 2 =43

x − 4

y =43

x − 6 Slope-intercept form

49. x = −1 is the equation of a vertical line. The line parallelto the given line is a vertical line containing the point(3,−3), or x = 3.

The line perpendicular to the given line is a horizontal linecontaining the point (3,−3), or y = −3.

51. x = −3 is a vertical line and y = 5 is a horizontal line, soit is true that the lines are perpendicular.

53. The lines have the same slope,25, and different y-

intercepts, (0, 4) and (0,−4), so it is true that the lines areparallel.

55. x = −1 and x = 1 are both vertical lines, so it is false thatthey are perpendicular.

57. No. The data points fall faster from 0 to 2 than after 2(that is, the rate of change is not constant), so they cannotbe modeled by a linear function.

59. Yes. The rate of change seems to be constant, so the datapoints might be modeled by a linear function.

61. a) Answers may vary depending on the data pointsused. We will use (2, 785) and (8, 1858).

m =1858 − 785

8 − 2=

10736

≈ 178.83

We will use the point-slope equation, letting(x1, y1) = (2, 785).

y − 785 = 178.83(x − 2)

y − 785 = 178.83x − 357.66

y = 178.83x + 427.34,

where x is the number of years after 2001 and y isin millions.

b) In 2013, x = 2013 − 2001 = 12.

y = 178.83(12) + 427.34 ≈ 2573

We estimate the number of world Internet users in2013 to be about 2573 million.

In 2018, x = 2018 − 2001 = 17.

y = 178.83(17) + 427.34 ≈ 3467

We estimate the number of world Internet users in2018 to be about 3467 million.

63. Answers may vary depending on the data points used. Wewill use (2, 16.3) and (5, 19.0).

m =19.0 − 16.3

5 − 2=

2.73

= 0.9

We will use the slope-intercept equation with (5, 19.0).

19.0 = 0.9(5) + b

19.0 = 4.5 + b

14.5 = b

We have y = 0.9x + 14.5 where x is the number of yearsafter 2005 and y is in billions of dollars.

In 2015, x = 2015 − 2005 = 10. Then we have

y = 0.9(10) + 14.5 = 23.5.

We predict the net sales will be $23.5 billion in 2015.

65. Answers may vary depending on the data points used. Wewill use (3, 24.9) and (15, 56.9).

m =56.9 − 24.9

15 − 3=

3212

≈ 2.67

We will use the point-slope equation with (3, 24.9).

y − 24.9 = 2.67(x − 3)

y − 24.9 = 2.67x − 8.01

y = 2.67x + 16.89

We have y = 2.67x + 16.89 where x is the number of yearsafter 1991 and y is in billions of dollars.

Copyright © 2013 Pearson Education, Inc.

Page 47: JUDITH A. PENNA -  · PDF fileSTUDENT’S SOLUTIONS MANUAL JUDITH A. PENNA Indiana University Purdue University Indianapolis COLLEGE ALGEBRA: GRAPHS AND MODELS FIFTH EDITION

44 Chapter 1: Graphs, Functions, and Models

In 2005, x = 2005 − 1991 = 14. We have

y = 2.67(14) + 16.89 ≈ $54.27 billion

In 2015, x = 2015 − 1991 = 24. We have

y = 2.67(24) + 16.89 ≈ $80.97 billion

67. a) Using the linear regression feature on a graphingcalculator, we have y = 171.2606061x+430.5272727,where x is the number of years after 2001 and y isin millions.

b) In 2013, x = 2013 − 2001 = 12. We have

y = 171.2606061(12) + 430.5272727 ≈ 2486 million

This is 87 million less than the value found in Ex-ercise 61.

c) r ≈ 0.9890; the line fits the data fairly well.

69. a) Using the linear regression feature on a graphingcalculator, we have y = 2.628571429x+17.41428571,where x is the number of years after 1991 and y isin billions of dollars.

b) In 2015, x = 2015 − 1991 = 24. We havey = 2.628571429(24) + 17.41428571 ≈ $80.5 billion.This value is $0.47 billion less than the value foundin Exercise 65.

c) r ≈ 0.9950; the line fits the data well.

71. a) Using the linear regression feature on a graphingcalculator, we get M = 0.2H + 156.

b) For H = 40: M = 0.2(40) + 156 = 164 beats perminute

For H = 65: M = 0.2(65) + 156 = 169 beats perminute

For H = 76: M = 0.2(76) + 156 ≈ 171 beats perminute

For H = 84: M = 0.2(84) + 156 ≈ 173 beats perminute

c) r = 1; all the data points are on the regression lineso it should be a good predictor.

73. m =y2 − y1

x2 − x1

=−1 − (−8)−5 − 2

=−1 + 8−7

=7−7

= −1

75. (x − h)2 + (y − k)2 = r2

[x − (−7)]2 + [y − (−1)]2 =(

95

)2

(x + 7)2 + (y + 1)2 =8125

77. The slope of the line containing (−3, k) and (4, 8) is8 − k

4 − (−3)=

8 − k

7.

The slope of the line containing (5, 3) and (1,−6) is−6 − 31 − 5

=−9−4

=94.

The slopes must be equal in order for the lines to be par-allel:

8 − k

7=

94

32 − 4k = 63 Multiplying by 28

−4k = 31

k = −314

, or − 7.75

79. The slope of the line containing (−1, 3) and (2, 9) is9 − 3

2 − (−1)=

63

= 2.

Then the slope of the desired line is −12. We find the

equation of that line:

y − 5 = −12(x − 4)

y − 5 = −12

x + 2

y = −12

x + 7

Exercise Set 1.5

1. 4x + 5 = 21

4x = 16 Subtracting 5 on both sides

x = 4 Dividing by 4 on both sidesThe solution is 4.

3. 23 − 25

x = −25

x + 23

23 = 23 Adding25

x on both sides

We get an equation that is true for any value of x, so thesolution set is the set of real numbers,{x|x is a real number}, or (−∞,∞).

5. 4x + 3 = 0

4x = −3 Subtracting 3 on both sides

x = −34

Dividing by 4 on both sides

The solution is −34.

7. 3 − x = 12

−x = 9 Subtracting 3 on both sides

x = −9 Multiplying (or dividing) by −1on both sides

The solution is −9.

9. 3 − 14

x =32

The LCD is 4.

4(

3 − 14

x

)= 4 · 3

2Multiplying by the LCDto clear fractions

12 − x = 6

−x = −6 Subtracting 12 on both sides

x = 6 Multiplying (or dividing) by −1on both sides

The solution is 6.

Copyright © 2013 Pearson Education, Inc.

Page 48: JUDITH A. PENNA -  · PDF fileSTUDENT’S SOLUTIONS MANUAL JUDITH A. PENNA Indiana University Purdue University Indianapolis COLLEGE ALGEBRA: GRAPHS AND MODELS FIFTH EDITION

Exercise Set 1.5 45

11.211

− 4x = −4x +911

211

=911

Adding 4x on both sides

We get a false equation. Thus, the original equation hasno solution.

13. 8 = 5x − 3

11 = 5x Adding 3 on both sides115

= x Dividing by 5 on both sides

The solution is115

.

15.25

y − 2 =13

The LCD is 15.

15(

25

y − 2)

= 15 · 13

Multiplying by the LCDto clear fractions

6y − 30 = 5

6y = 35 Adding 30 on both sides

y =356

Dividing by 6 on both sides

The solution is356

.

17. y + 1 = 2y − 7

1 = y − 7 Subtracting y on both sides

8 = y Adding 7 on both sidesThe solution is 8.

19. 2x + 7 = x + 3

x + 7 = 3 Subtracting x on both sides

x = −4 Subtracting 7 on both sidesThe solution is −4.

21. 3x − 5 = 2x + 1

x − 5 = 1 Subtracting 2x on both sides

x = 6 Adding 5 on both sidesThe solution is 6.

23. 4x − 5 = 7x − 2

−5 = 3x − 2 Subtracting 4x on both sides

−3 = 3x Adding 2 on both sides

−1 = x Dividing by 3 on both sidesThe solution is −1.

25. 5x − 2 + 3x = 2x + 6 − 4x

8x − 2 = 6 − 2x Collecting like terms

8x + 2x = 6 + 2 Adding 2x and 2 onboth sides

10x = 8 Collecting like terms

x =810

Dividing by 10 on both

sides

x =45

Simplifying

The solution is45.

27. 7(3x + 6) = 11 − (x + 2)

21x + 42 = 11 − x − 2 Using the distributiveproperty

21x + 42 = 9 − x Collecting like terms

21x + x = 9 − 42 Adding x and subtract-ing 42 on both sides

22x = −33 Collecting like terms

x = −3322

Dividing by 22 on both

sides

x = −32

Simplifying

The solution is −32.

29. 3(x + 1) = 5 − 2(3x + 4)

3x + 3 = 5 − 6x − 8 Removing parentheses

3x + 3 = −6x − 3 Collecting like terms

9x + 3 = −3 Adding 6x

9x = −6 Subtracting 3

x = −23

Dividing by 9

The solution is −23.

31. 2(x − 4) = 3 − 5(2x + 1)

2x − 8 = 3 − 10x − 5 Using the distributiveproperty

2x − 8 = −10x − 2 Collecting like terms

12x = 6 Adding 10x and 8 on both sides

x =12

Dividing by 12 on both sides

The solution is12.

33. Familiarize. Let s = the number of foreign students fromIndia. Then 22% more than this number is s + 0.22s, or1.22s.

Translate.Number ofstudents

from China︸ ︷︷ ︸is

22% more thannumber of students

from India︸ ︷︷ ︸� � �128, 000 = 1.22s

Carry out. We solve the equation.

128, 000 = 1.22s

105, 000 ≈ s

Check. 105, 000 + 0.22(105, 000) = 128, 100 ≈ 128, 000.The answer checks. (Remember that we rounded the valueof s.)

State. There were about 105,000 students from India inthe U.S. in the 2009-2010 school year.

Copyright © 2013 Pearson Education, Inc.

Page 49: JUDITH A. PENNA -  · PDF fileSTUDENT’S SOLUTIONS MANUAL JUDITH A. PENNA Indiana University Purdue University Indianapolis COLLEGE ALGEBRA: GRAPHS AND MODELS FIFTH EDITION

46 Chapter 1: Graphs, Functions, and Models

35. Familiarize. Let x = the number of metric tons of oliveoil consumed in the U.S. in 2009/2010.

Translate.Consumption

in Italy︸ ︷︷ ︸was 2.5 timesconsumption

in U.S.︸ ︷︷ ︸ plus 60, 000

� � � � � � �710, 000 = 2.5 · x + 60, 000

Carry out. We solve the equation.

710, 000 = 2.5x + 60, 000

650, 000 = 2.5x

260, 000 = x

Check. 2.5(260, 000) = 650, 000 and 650, 000 + 60, 000 =710, 000. The answer checks.

State. 260,000 metric tons of olive oil were consumed inthe U.S. in 2009 - 2010.

37. Familiarize. Let d = the average depth of the Atlantic

Ocean, in feet. Then45

d − 272 = the average depth of theIndian Ocean.

Translate.Averagedepth ofPacificOcean︸ ︷︷ ︸

is

Averagedepth ofAtlanticOcean︸ ︷︷ ︸

plus

Averagedepth ofIndianOcean︸ ︷︷ ︸

less 8890 ft

� � � � � � �14, 040 = d +

45

d − 272 − 8890

Carry out. We solve the equation.

14, 040 = d +45

d − 272 − 8890

14, 040 =95

d − 9162

23, 202 =95

d

59· 23, 202 = d

12, 890 = d

If d = 12, 890, then the average depth of the Indian Ocean

is45· 12, 890 − 272 = 10, 040.

Check. 12, 890 + 10, 040 − 8890 = 14, 040, so the answerchecks.

State. The average depth of the Indian Ocean is 10,040 ft.

39. Familiarize. Let w = the weight of a Smart for Two car,in pounds. Then 2w − 5 = the weight of a Mustang and2w − 5 + 2135, or 2w + 2130 = the weight of a Tundra.

Translate.Weight

ofTundra︸ ︷︷ ︸

plusweight

ofMustang︸ ︷︷ ︸

plusweight

ofSmart Car︸ ︷︷ ︸

istotal

weight︸ ︷︷ ︸� � � � � � �2w + 2130 + 2w − 5 + w = 11, 150

Carry out. We solve the equation.

2w + 2130 + 2w − 5 + w = 11, 150

5w + 2125 = 11, 150

5w = 9025

w = 1805

If w = 1805, then 2w − 5 = 2 · 1805 − 5 = 3605 and2w + 2130 = 2 · 1805 + 2130 = 5740.

Check. 1805+3605+5740 = 11, 150, so the answer checks.

State. The Tundra weighs 5740 lb, the Mustang weighs3605 lb, and the Smart For Two weighs 1805 lb.

41. Familiarize. Let v = the number of ABC viewers, inmillions. Then v + 1.7 = the number of CBS viewers andv − 1.7 = the number of NBC viewers.

Translate.ABC

viewers︸ ︷︷ ︸ plusCBS

viewers︸ ︷︷ ︸ plusNBC

viewers︸ ︷︷ ︸ istotal

viewers.︸ ︷︷ ︸� � � � � � �v + (v + 1.7) + (v − 1.7) = 29.1

Carry out.

v + (v + 1.7) + (v − 1.7) = 29.1

3v = 29.1

v = 9.7

Then v+1.7 = 9.7+1.7 = 11.4 and v−1.7 = 9.7−1.7 = 8.0.

Check. 9.7 + 11.4 + 8.0 = 29.1, so the answer checks.

State. ABC had 9.7 million viewers, CBS had 11.4 millionviewers, and NBC had 8.0 million viewers.

43. Familiarize. Let P = the amount Tamisha borrowed. Wewill use the formula I = Prt to find the interest owed. Forr = 5%, or 0.05, and t = 1, we have I = P (0.05)(1), or0.05P .

Translate.Amount borrowed︸ ︷︷ ︸ plus interest is $1365.� � � � �

P + 0.05P = 1365

Carry out. We solve the equation.

P + 0.05P = 1365

1.05P = 1365 Adding

P = 1300 Dividing by 1.05

Check. The interest due on a loan of $1300 for 1 year ata rate of 5% is $1300(0.05)(1), or $65, and $1300 + $65 =$1365. The answer checks.

State. Tamisha borrowed $1300.

45. Familiarize. Let s = Ryan’s sales for the month. Thenhis commission is 8% of s, or 0.08s.

Translate.Base salary︸ ︷︷ ︸ plus commission is total pay.︸ ︷︷ ︸� � � � �

1500 + 0.08s = 2284

Copyright © 2013 Pearson Education, Inc.

Page 50: JUDITH A. PENNA -  · PDF fileSTUDENT’S SOLUTIONS MANUAL JUDITH A. PENNA Indiana University Purdue University Indianapolis COLLEGE ALGEBRA: GRAPHS AND MODELS FIFTH EDITION

Exercise Set 1.5 47

Carry out. We solve the equation.1500 + 0.08s = 2284

0.08s = 784 Subtracting 1500

s = 9800

Check. 8% of $9800, or 0.08($9800), is $784 and $1500 +$784 = $2284. The answer checks.

State. Ryan’s sales for the month were $9800.

47. Familiarize. Let w = Soledad’s regular hourly wage. Sheearned 40w for working the first 40 hr. She worked 48−40,or 8 hr, of overtime. She earned 8(1.5w) for working 8 hrof overtime.

Translate. The total earned was $442, so we write anequation.

40w + 8(1.5w) = 442

Carry out. We solve the equation.40w + 8(1.5)w = 442

40w + 12w = 442

52w = 442

w = 8.5

Check. 40($8.50) + 8[1.5($8.50)] = $340 + $102 = $442,so the answer checks.

State. Soledad’s regular hourly wage is $8.50.

49. Familiarize. We make a drawing.

✔✔✔✔✔✔✔

❝❝

❝❝

❝❝

❝❝

A

B

C

x

5x

x − 2

We let x = the measure of angle A. Then 5x = the measureof angle B, and x − 2 = the measure of angle C. The sumof the angle measures is 180◦.

Translate.Measure

ofangle A︸ ︷︷ ︸

+Measure

ofangle B︸ ︷︷ ︸

+Measure

ofangle C︸ ︷︷ ︸

= 180.

� � � � � � �x + 5x + x − 2 = 180

Carry out. We solve the equation.x + 5x + x − 2 = 180

7x − 2 = 180

7x = 182

x = 26

If x = 26, then 5x = 5 · 26, or 130, and x − 2 = 26 − 2, or24.

Check. The measure of angle B, 130◦, is five times themeasure of angle A, 26◦. The measure of angle C, 24◦, is2◦ less than the measure of angle A, 26◦. The sum of theangle measures is 26◦ + 130◦ + 24◦, or 180◦. The answerchecks.

State. The measure of angles A, B, and C are 26◦, 130◦,and 24◦, respectively.

51. Familiarize. Using the labels on the drawing in the text,we let w = the width of the test plot and w + 25 = thelength, in meters. Recall that for a rectangle, Perime-ter = 2 · length + 2 · width.

Translate.Perimeter︸ ︷︷ ︸ = 2 · length︸ ︷︷ ︸ + 2 · width︸ ︷︷ ︸� � � � �

322 = 2(w + 25) + 2 · w

Carry out. We solve the equation.

322 = 2(w + 25) + 2 · w

322 = 2w + 50 + 2w

322 = 4w + 50

272 = 4w

68 = w

When w = 68, then w + 25 = 68 + 25 = 93.

Check. The length is 25 m more than the width: 93 =68 + 25. The perimeter is 2 · 93 + 2 · 68, or 186 + 136, or322 m. The answer checks.

State. The length is 93 m; the width is 68 m.

53. Familiarize. Let l = the length of the soccer field andl − 35 = the width, in yards.

Translate. We use the formula for the perimeter of arectangle. We substitute 330 for P and l − 35 for w.

P = 2l + 2w

330 = 2l + 2(l − 35)

Carry out. We solve the equation.

330 = 2l + 2(l − 35)

330 = 2l + 2l − 70

330 = 4l − 70

400 = 4l

100 = l

If l = 100, then l − 35 = 100 − 35 = 65.

Check. The width, 65 yd, is 35 yd less than the length,100 yd. Also, the perimeter is

2 · 100 yd + 2 · 65 yd = 200 yd + 130 yd = 330 yd.

The answer checks.

State. The length of the field is 100 yd, and the width is65 yd.

55. Familiarize. Let w = the number of pounds of Lily’sbody weight that is water.

Translate.55% of body weight︸ ︷︷ ︸ is water.

↓ ↓ ↓ ↓ ↓0.55 × 135 = w

Carry out. We solve the equation.

0.55 × 135 = w

74.25 = w

Copyright © 2013 Pearson Education, Inc.

Page 51: JUDITH A. PENNA -  · PDF fileSTUDENT’S SOLUTIONS MANUAL JUDITH A. PENNA Indiana University Purdue University Indianapolis COLLEGE ALGEBRA: GRAPHS AND MODELS FIFTH EDITION

48 Chapter 1: Graphs, Functions, and Models

Check. Since 55% of 135 is 74.25, the answer checks.

State. 74.25 lb of Lily’s body weight is water.

57. Familiarize. We make a drawing. Let t = the numberof hours the passenger train travels before it overtakes thefreight train. Then t+1 = the number of hours the freighttrain travels before it is overtaken by the passenger train.Also let d = the distance the trains travel.

✲�

80 mph t hr dPassenger train

✲�

60 mph t + 1 hr dFreight train

We can also organize the information in a table.

d = r · t

Distance Rate Time

Freightd 60 t + 1

train

Passengerd 80 t

train

Translate. Using the formula d = rt in each row of thetable, we get two equations.

d = 60(t + 1) and d = 80t.

Since the distances are the same, we have the equation

60(t + 1) = 80t.

Carry out. We solve the equation.

60(t + 1) = 80t

60t + 60 = 80t

60 = 20t

3 = t

When t = 3, then t + 1 = 3 + 1 = 4.

Check. In 4 hr the freight train travels 60 · 4, or 240 mi.In 3 hr the passenger train travels 80 · 3, or 240 mi. Sincethe distances are the same, the answer checks.

State. It will take the passenger train 3 hr to overtake thefreight train.

59. Familiarize. Let t = the number of hours it takes thekayak to travel 36 mi upstream. The kayak travels up-stream at a rate of 12 − 4, or 8 mph.

Translate. We use the formula d = rt.

36 = 8 · t

Carry out. We solve the equation.

36 = 8 · t

4.5 = t

Check. At a rate of 8 mph, in 4.5 hr the kayak travels8(4.5), or 36 mi. The answer checks.

State. It takes the kayak 4.5 hr to travel 36 mi upstream.

61. Familiarize. Let t = the number of hours it will take theplane to travel 1050 mi into the wind. The speed into theheadwind is 450 − 30, or 420 mph.

Translate. We use the formula d = rt.

1050 = 420 · t

Carry out. We solve the equation.

1050 = 420 · t

2.5 = t

Check. At a rate of 420 mph, in 2.5 hr the plane travels420(2.5), or 1050 mi. The answer checks.

State. It will take the plane 2.5 hr to travel 1050 mi intothe wind.

63. Familiarize. Let x = the amount invested at 3% interest.Then 5000−x = the amount invested at 4%. We organizethe information in a table, keeping in mind the simpleinterest formula, I = Prt.

Amount Interest Amountinvested rate Time of interest

3% 3%, or x(0.03)(1),invest- x 1 yrment 0.03 or 0.03x4% 4%, or (5000−x)(0.04)(1),invest- 5000−x 1 yrment 0.04 or 0.04(5000−x)Total 5000 176

Translate.Interest on

3% investment︸ ︷︷ ︸ plusinterest on

4% investment︸ ︷︷ ︸ is $176.

� � � � �0.03x + 0.04(5000 − x) = 176

Carry out. We solve the equation.

0.03x + 0.04(5000 − x) = 176

0.03x + 200 − 0.04x = 176

−0.01x + 200 = 176

−0.01x = −24

x = 2400

If x = 2400, then 5000 − x = 5000 − 2400 = 2600.

Check. The interest on $2400 at 3% for 1 yr is$2400(0.03)(1) = $72. The interest on $2600 at 4% for1 yr is $2600(0.04)(1) = $104. Since $72 + $104 = $176,the answer checks.

State. $2400 was invested at 3%, and $2600 was investedat 4%.

65. Familiarize. Let a = the number of unique ama-zon.com visitors. Then a + 42, 826, 225 = the number ofyoutube.com visitors.

Translate.Number of

facebook.comvisitors︸ ︷︷ ︸

isnumber of

amazon.comvisitors︸ ︷︷ ︸

plus

� � � �134, 078, 221 = a +

Copyright © 2013 Pearson Education, Inc.

Page 52: JUDITH A. PENNA -  · PDF fileSTUDENT’S SOLUTIONS MANUAL JUDITH A. PENNA Indiana University Purdue University Indianapolis COLLEGE ALGEBRA: GRAPHS AND MODELS FIFTH EDITION

Exercise Set 1.5 49

number ofyoutube.com

visitors︸ ︷︷ ︸less 51, 604, 956

� � �a + 42, 826, 225 − 51, 604, 956

Carry out. We solve the equation.134, 078, 221 = a + a + 42, 826, 225 − 51, 604, 956

134, 078, 221 = 2a− 8, 778, 731

142, 856, 952 = 2a

71, 428, 476 = a

Then a + 42, 826, 225 = 71, 428, 476 + 42, 826, 225 =114, 254, 701.

Check. The total number of amazon and youtube vis-itors was 71, 428, 476 + 114, 254, 701, or 185,683,177 and185, 683, 177 − 51, 604, 956 = 134, 078, 221. The answerchecks.

State. In February, 2011, amazon.com had 71,428,476unique visitors, and youtube.com had 114,254,701 uniquevisitors.

67. Familiarize. Let l = the elevation of Lucas Oil Stadium,in feet.

Translate.

Elevation ofInvesco Field︸ ︷︷ ︸ is 247 ft︸ ︷︷ ︸ more

than︸ ︷︷ ︸ 7 timesElevation ofLucas OilStadium︸ ︷︷ ︸� � � � � � �

5210 = 247 + 7 · l

Carry out. We solve the equation.5210 = 247 + 7l

4963 = 7l

709 = l

Check. 247 more than 7 times 709 is 247 + 7 · 709 =247 + 4963 = 5210. The answer checks.

State. The elevation of Lucas Oil Stadium is 709 ft.

69. Familiarize. Let n = the number of people in the ur-ban population of the Dominican Republic for whom bot-tled water was the primary source of drinking water in2009. We are told that the urban population is 66.8% of9,650,054, or 0.668(9, 650, 054).

Translate.Number for whombottled water isprimary source︸ ︷︷ ︸

is 67% ofurban

population︸ ︷︷ ︸� � � � �n = 0.67 · 0.668(9, 650, 054)

Carry out. We carry out the calculation.

n = 0.67 · 0.668(9, 650, 054) ≈ 4, 318, 978

Check. We can repeat the calculation. The answerchecks.

State. Bottled water was the primary source of drinkingwater for about 4,318,978 people in the urban populationof the Dominican Republic in 2009.

71. x + 5 = 0 Setting f(x) = 0

x + 5 − 5 = 0 − 5 Subtracting 5 on both sides

x = −5The zero of the function is −5.

73. −2x + 11 = 0 Setting f(x) = 0

−2x + 11 − 11 = 0 − 11 Subtracting 11 on bothsides

−2x = −11

x =112

Dividing by −2 on both sides

The zero of the function is112

.

75. 16 − x = 0 Setting f(x) = 0

16 − x + x = 0 + x Adding x on both sides

16 = x

The zero of the function is 16.

77. x + 12 = 0 Setting f(x) = 0

x + 12 − 12 = 0 − 12 Subtracting 12 onboth sides

x = −12The zero of the function is −12.

79. −x + 6 = 0 Setting f(x) = 0

−x + 6 + x = 0 + x Adding x on both sides

6 = x

The zero of the function is 6.

81. 20 − x = 0 Setting f(x) = 0

20 − x + x = 0 + x Adding x on both sides

20 = x

The zero of the function is 20.

83.25x− 10 = 0 Setting f(x) = 0

25x = 10 Adding 10 on both sides

52· 25x =

52· 10 Multiplying by

52

on both

sidesx = 25

The zero of the function is 25.

85. −x + 15 = 0 Setting f(x) = 0

15 = x Adding x on both sidesThe zero of the function is 15.

87. a) The graph crosses the x-axis at (4, 0). This is thex-intercept.

b) The zero of the function is the first coordinate ofthe x-intercept. It is 4.

89. a) The graph crosses the x-axis at (−2, 0). This is thex-intercept.

b) The zero of the function is the first coordinate ofthe x-intercept. It is −2.

Copyright © 2013 Pearson Education, Inc.

Page 53: JUDITH A. PENNA -  · PDF fileSTUDENT’S SOLUTIONS MANUAL JUDITH A. PENNA Indiana University Purdue University Indianapolis COLLEGE ALGEBRA: GRAPHS AND MODELS FIFTH EDITION

0 5

50 Chapter 1: Graphs, Functions, and Models

91. a) The graph crosses the x-axis at (−4, 0). This is thex-intercept.

b) The zero of the function is the first coordinate ofthe x-intercept. It is −4.

93. First find the slope of the given line.3x + 4y = 7

4y = −3x + 7

y = −34x +

74

The slope is −34. Now write a slope-intercept equation of

the line containing (−1, 4) with slope −34.

y − 4 = −34[x− (−1)]

y − 4 = −34(x + 1)

y − 4 = −34x− 3

4

y = −34x +

134

95. d =√

(x2 − x1)2 + (y2 − y1)2

=√

(−10 − 2)2 + (−3 − 2)2

=√

144 + 25 =√

169 = 13

97. f(x) =x

x− 3

f(−3) =−3

−3 − 3=

−3−6

=12

f(0) =0

0 − 3=

0−3

= 0

f(3) =3

3 − 3=

30

Since division by 0 is not defined, f(3) does not exist.

99. f(x) = 7 − 32x = −3

2x + 7

The function can be written in the form y = mx+ b, so itis a linear function.

101. f(x) = x2+1 cannot be written in the form f(x) = mx+b,so it is not a linear function.

103. 2x− {x− [3x− (6x + 5)]} = 4x− 1

2x− {x− [3x− 6x− 5]} = 4x− 1

2x− {x− [−3x− 5]} = 4x− 1

2x− {x + 3x + 5} = 4x− 1

2x− {4x + 5} = 4x− 1

2x− 4x− 5 = 4x− 1

−2x− 5 = 4x− 1

−6x− 5 = −1

−6x = 4

x = −23

The solution is −23.

105. The size of the cup was reduced 8 oz − 6 oz, or 2 oz, and2 oz8 oz

= 0.25, so the size was reduced 25%. The price per

ounce of the 8 oz cup was89/c8 oz

, or 11.125/c/oz. The price

per ounce of the 6 oz cup is71/c6 oz

, or 11.83/c/oz. Since theprice per ounce was not reduced, it is clear that the priceper ounce was not reduced by the same percent as the sizeof the cup. The price was increased by 11.83− 11.125/c, or

0.7083/c per ounce. This is an increase of0.7083/c11.125/c

≈ 0.064,

or about 6.4% per ounce.

107. We use a proportion to determine the number of caloriesc burned running for 75 minutes, or 1.25 hr.

7201

=c

1.25720(1.25) = c

900 = c

Next we use a proportion to determine how long the personwould have to walk to use 900 calories. Let t represent thistime, in hours. We express 90 min as 1.5 hr.

1.5480

=t

900900(1.5)

480= t

2.8125 = t

Then, at a rate of 4 mph, the person would have to walk4(2.8125), or 11.25 mi.

Exercise Set 1.6

1. 4x− 3 > 2x + 7

2x− 3 > 7 Subtracting 2x

2x > 10 Adding 3

x > 5 Dividing by 2

The solution set is {x|x > 5}, or (5,∞). The graph isshown below.

3. x + 6 < 5x− 6

6 + 6 < 5x− x Subtracting x and adding 6on both sides

12 < 4x124

< x Dividing by 4 on both sides

3 < x

This inequality could also be solved as follows:x + 6 < 5x− 6

x− 5x < −6 − 6 Subtracting 5x and 6 onboth sides

−4x < −12

x >−12−4

Dividing by −4 on both sides and

reversing the inequality symbolx > 3

Copyright © 2013 Pearson Education, Inc.

Page 54: JUDITH A. PENNA -  · PDF fileSTUDENT’S SOLUTIONS MANUAL JUDITH A. PENNA Indiana University Purdue University Indianapolis COLLEGE ALGEBRA: GRAPHS AND MODELS FIFTH EDITION

0 3

0�3

0 2213��

0 6

0

512���

0

1534��

0

314���

0 1

Exercise Set 1.6 51

The solution set is {x|x > 3}, or (3,∞). The graph isshown below.

5. 4 − 2x ≤ 2x + 16

4 − 4x ≤ 16 Subtracting 2x

−4x ≤ 12 Subtracting 4

x ≥ −3 Dividing by −4 and reversingthe inequality symbol

The solution set is {x|x ≥ −3}, or [−3,∞). The graph isshown below.

7. 14 − 5y ≤ 8y − 8

14 + 8 ≤ 8y + 5y

22 ≤ 13y2213

≤ y

This inequality could also be solved as follows:14 − 5y ≤ 8y − 8

−5y − 8y ≤ −8 − 14

−13y ≤ −22

y ≥ 2213

Dividing by −13 onboth sides and reversingthe inequality symbol

The solution set is{y

∣∣∣∣y ≥ 2213

}, or

[2213

,∞)

. The graph

is shown below.

9. 7x− 7 > 5x + 5

2x− 7 > 5 Subtracting 5x

2x > 12 Adding 7

x > 6 Dividing by 2The solution set is {x|x > 6}, or (6,∞). The graph isshown below.

11. 3x− 3 + 2x ≥ 1 − 7x− 9

5x− 3 ≥ −7x− 8 Collecting like terms5x + 7x ≥ −8 + 3 Adding 7x and 3

on both sides12x ≥ −5

x ≥ − 512

Dividing by 12 on both sides

The solution set is{x∣∣∣x ≥ − 5

12

}, or

[− 5

12,∞)

. The

graph is shown below.

13. −34x ≥ −5

8+

23x

58≥ 3

4x +

23x

58≥ 9

12x +

812

x

58≥ 17

12x

1217

· 58≥ 12

17· 1712

x

1534

≥ x

The solution set is{x

∣∣∣∣x ≤ 1534

}, or

(−∞,

1534

]. The

graph is shown below.

15. 4x(x− 2) < 2(2x− 1)(x− 3)

4x(x− 2) < 2(2x2 − 7x + 3)

4x2 − 8x < 4x2 − 14x + 6

−8x < −14x + 6

−8x + 14x < 6

6x < 6

x <66

x < 1

The solution set is {x|x < 1}, or (−∞, 1). The graph isshown below.

17. The radicand must be nonnegative, so we solve the in-equality x− 7 ≥ 0.

x− 7 ≥ 0

x ≥ 7

The domain is {x|x ≥ 7}, or [7,∞).

19. The radicand must be nonnegative, so we solve the in-equality 1 − 5x ≥ 0.

1 − 5x ≥ 0

1 ≥ 5x15≥ x

The domain is{x

∣∣∣∣x ≤ 15

}, or

(−∞,

15

].

Copyright © 2013 Pearson Education, Inc.

Page 55: JUDITH A. PENNA -  · PDF fileSTUDENT’S SOLUTIONS MANUAL JUDITH A. PENNA Indiana University Purdue University Indianapolis COLLEGE ALGEBRA: GRAPHS AND MODELS FIFTH EDITION

0�3 3

8 100

0�7 �1

0 232��

0 1 5

0 133��

113���

0�2 1

0

72��

12�

10.49.60

0574���

554���

52 Chapter 1: Graphs, Functions, and Models

21. The radicand must be positive, so we solve the inequality4 + x > 0.

4 + x > 0

x > −4

The domain is {x|x > −4}, or (−4,∞).

23. −2 ≤ x + 1 < 4

−3 ≤ x < 3 Subtracting 1

The solution set is [−3, 3). The graph is shown below.

25. 5 ≤ x− 3 ≤ 7

8 ≤ x ≤ 10 Adding 3

The solution set is [8, 10]. The graph is shown below.

27. −3 ≤ x + 4 ≤ 3

−7 ≤ x ≤ −1 Subtracting 4

The solution set is [−7,−1]. The graph is shown below.

29. −2 < 2x + 1 < 5

−3 < 2x < 4 Adding −1

−32< x < 2 Multiplying by

12

The solution set is(− 3

2, 2)

. The graph is shown below.

31. −4 ≤ 6 − 2x < 4

−10 ≤ −2x < −2 Adding −6

5 ≥ x > 1 Multiplying by −12

or 1 < x ≤ 5

The solution set is (1, 5]. The graph is shown below.

33. −5 <12(3x + 1) < 7

−10 < 3x + 1 < 14 Multiplying by 2

−11 < 3x < 13 Adding −1

−113

< x <133

Multiplying by13

The solution set is(− 11

3,133

). The graph is shown be-

low.

35. 3x ≤ −6 or x− 1 > 0

x ≤ −2 or x > 1

The solution set is (−∞,−2]∪ (1,∞). The graph is shownbelow.

37. 2x + 3 ≤ −4 or 2x + 3 ≥ 4

2x ≤ −7 or 2x ≥ 1

x ≤ −72or x ≥ 1

2

The solution set is(−∞,−7

2

]∪[12,∞)

. The graph is

shown below.

39. 2x− 20 < −0.8 or 2x− 20 > 0.8

2x < 19.2 or 2x > 20.8

x < 9.6 or x > 10.4

The solution set is (−∞, 9.6) ∪ (10.4,∞). The graph isshown below.

41. x + 14 ≤ −14

or x + 14 ≥ 14

x ≤ −574

or x ≥ −554

The solution set is(−∞,−57

4

]∪[− 55

4,∞)

. The

graph is shown below.

43. Familiarize and Translate. Spending is given by theequation y = 31.7x+487. We want to know when spendingwill be more than $775 billion, so we have

31.7x + 487 > 775.

Carry out. We solve the inequality.

31.7x + 487 > 775

31.7x > 288

x > 9 Rounding

Check. When x = 9, spending is 31.7(9) + 487 =772.3 ≈ 775. (Remember that we rounded the an-swer.) As a partial check we try a value of x less than9 and a value greater than 9. When x = 8.9, we havey = 31.7(8.9) + 487 = 769.13 < 775; when x = 9.1,y = 31.7(9.1) + 487 = 775.47 > 775. Since y ≈ 775 whenx = 9 and y > 775 when x = 9.1 > 9, the answer isprobably correct.

State. Spending will be more than $775 billion more than9 yr after 2007, or in years after 2016.

Copyright © 2013 Pearson Education, Inc.

Page 56: JUDITH A. PENNA -  · PDF fileSTUDENT’S SOLUTIONS MANUAL JUDITH A. PENNA Indiana University Purdue University Indianapolis COLLEGE ALGEBRA: GRAPHS AND MODELS FIFTH EDITION

Exercise Set 1.6 53

45. Familiarize. Let t = the number of hours worked. ThenAcme Movers charge 100 + 30t and Hank’s Movers charge55t.

Translate.Hank’s charge︸ ︷︷ ︸ is less than︸ ︷︷ ︸ Acme’s charge.︸ ︷︷ ︸� � �

55t < 100 + 30t

Carry out. We solve the inequality.

55t < 100 + 30t

25t < 100

t < 4

Check. When t = 4, Hank’s Movers charge 55 · 4, or $220and Acme Movers charge 100 + 30 · 4 = 100 + 120 = $220,so the charges are the same. As a partial check, we findthe charges for a value of t < 4. When t = 3.5, Hank’sMovers charge 55(3.5) = $192.50 and Acme Movers charge100 + 30(3.5) = 100 + 105 = $205. Since Hank’s charge isless than Acme’s, the answer is probably correct.

State. For times less than 4 hr it costs less to hire Hank’sMovers.

47. Familiarize. Let x = the amount invested at 4%. Then7500− x = the amount invested at 5%. Using the simple-interest formula, I = Prt, we see that in one year the4% investment earns 0.04x and the 5% investment earns0.05(7500 − x).

Translate.Interest at 4%︸ ︷︷ ︸ plus interest at 5%︸ ︷︷ ︸ is at least︸ ︷︷ ︸ $325.� � � � �

0.04x + 0.05(7500 − x) ≥ 325

Carry out. We solve the inequality.

0.04x + 0.05(7500 − x) ≥ 325

0.04x + 375 − 0.05x ≥ 325

−0.01x + 375 ≥ 325

−0.01x ≥ −50

x ≤ 5000

Check. When $5000 is invested at 4%, then $7500−$5000,or $2500, is invested at 5%. In one year the 4% invest-ment earns 0.04($5000), or $200, in simple interest andthe 5% investment earns 0.05($2500), or $125, so the totalinterest is $200 + $125, or $325. As a partial check, wedetermine the total interest when an amount greater than$5000 is invested at 4%. Suppose $5001 is invested at 4%.Then $2499 is invested at 5%, and the total interest is0.04($5001) + 0.05($2499), or $324.99. Since this amountis less than $325, the answer is probably correct.

State. The most that can be invested at 4% is $5000.

49. Familiarize and Translate. Let x = the amount in-vested at 7%. Then 2x = the amount invested at 4%,and 50, 000 − x − 2x, or 50, 000 − 3x = the amount in-vested at 5.5%. The interest earned is 0.07x + 0.04 · 2x +0.055(50, 000 − 3x), or 0.07x + 0.08x + 2750 − 0.165x, or−0.015x + 2750. The foundation wants the interest to beat least $2660 so we have

−0.015x + 2750 ≥ 2660.

Carry out. We solve the inequality.

−0.015x + 2750 ≥ 2660

−0.015x ≥ −90

x ≤ 6000

If $6000 is invested at 7%, then 2 · $6000, or $12,000 isinvested at 4%.

Check. If $6000 is invested at 7% and $12,000 is investedat 4%, the amount invested at 5.5% is $50, 000 − $6000 −$12, 000, or $32,000. The interest earned is 0.07 · $6000 +0.04 · $12, 000+0.055 · $32, 000, or $420+$480+$1760, or$2660.

As a partial check, we determine the total interest whenmore than $12,000 is invested at 4%. Suppose $12,001 isinvested at 4%. Then $12, 001/2, or $6000.50 is investedat 7% and $50, 000− $6000.50− $12, 001, or $31,998.50, isinvested at 5.5%. The interest earned is 0.07($6000.50) +0.04($12, 001)+0.055($31, 998.50), or $2659.99. Since thisis less than $2660, the answer is probably correct.

State. The most that can be invested at 4% is $12,000.

51. Familiarize. Let s = the monthly sales. Then the amountof sales in excess of $8000 is s− 8000.

Translate.Income from

plan B︸ ︷︷ ︸is greater

than︸ ︷︷ ︸income from

plan A.︸ ︷︷ ︸� � �1200 + 0.15(s− 8000) > 900 + 0.1s

Carry out. We solve the inequality.

1200 + 0.15(s− 8000) > 900 + 0.1s

1200 + 0.15s− 1200 > 900 + 0.1s

0.15s > 900 + 0.1s

0.05s > 900

s > 18, 000

Check. For sales of $18,000 the income from plan A is$900+0.1($18, 000), or $2700, and the income from plan Bis 1200+0.15(18, 000− 8000), or $2700 so the incomes arethe same. As a partial check we can compare the incomesfor an amount of sales greater than $18,000. For sales of$18,001, for example, the income from plan A is $900 +0.1($18, 001), or $2700.10, and the income from plan B is$1200 + 0.15($18, 001 − $8000), or $2700.15. Since plan Bis better than plan A in this case, the answer is probablycorrect.

State. Plan B is better than plan A for monthly salesgreater than $18,000.

53. Function; domain; range; domain; exactly one; range

55. x-intercept

Copyright © 2013 Pearson Education, Inc.

Page 57: JUDITH A. PENNA -  · PDF fileSTUDENT’S SOLUTIONS MANUAL JUDITH A. PENNA Indiana University Purdue University Indianapolis COLLEGE ALGEBRA: GRAPHS AND MODELS FIFTH EDITION

(3, 0)

(0, �2)

4

2

�2

�4

42�2�4 x

y

2x � 3y � 6

y

x

2

4

�2

�4

�2�4 42

y �� x � 123

y

x�4 �2 2 4

�4

�2

2

4 y � 2 � x2

54 Chapter 1: Graphs, Functions, and Models

57. 2x ≤ 5 − 7x < 7 + x

2x ≤ 5 − 7x and 5 − 7x < 7 + x

9x ≤ 5 and −8x < 2

x ≤ 59

and x > −14

The solution set is(− 1

4,59

].

59. 3y < 4 − 5y < 5 + 3y

0 < 4 − 8y < 5 Subtracting 3y

−4 < −8y < 1 Subtracting 412> y > −1

8Dividing by −8 and reversingthe inequality symbols

The solution set is(− 1

8,12

).

Chapter 1 Review Exercises

1. First we solve each equation for y.

ax + y = c x− by = d

y = −ax + c −by = −x + d

y =1bx− d

bIf the lines are perpendicular, the product of their slopes is

−1, so we have −a · 1b

= −1, or −a

b= −1, or

a

b= 1. The

statement is true.

3. f(−3) =

√3 − (−3)−3

=√

6−3

, so −3 is in the domain of

f(x). Thus, the statement is false.

5. The statement is true. See page 133 in the text.

7. For(

3,249

): 2x− 9y = −18

2 · 3 − 9 · 249

? −18∣∣6 − 24

∣∣∣−18

∣∣ −18 TRUE(3,

249

)is a solution.

For (0,−9): 2x− 9y = −18

2(0) − 9(−9) ? −180 + 81

∣∣∣81∣∣ −18 FALSE

(0,−9) is not a solution.

9. 2x− 3y = 6

To find the x-intercept we replace y with 0 and solve forx.

2x− 3 · 0 = 6

2x = 6

x = 3

The x-intercept is (3, 0).

To find the y-intercept we replace x with 0 and solve fory.

2 · 0 − 3y = 6

−3y = 6

y = −2

The y-intercept is (0,−2).

We plot the intercepts and draw the line that containsthem. We could find a third point as a check that theintercepts were found correctly.

11.

13.

15. m =(x1 + x2

2,y1 + y2

2

)

=(

3 + (−2)2

,7 + 4

2

)

=(

12,112

)

Copyright © 2013 Pearson Education, Inc.

Page 58: JUDITH A. PENNA -  · PDF fileSTUDENT’S SOLUTIONS MANUAL JUDITH A. PENNA Indiana University Purdue University Indianapolis COLLEGE ALGEBRA: GRAPHS AND MODELS FIFTH EDITION

x

y

1-1-3 32 54-2-4-5-1

-2

-3

-4

1

2

3

4

5

-5

f (x ) = 16 – x 2

x

y

2–2–6 64 108–4–8–10–2

–4

–6

–8

2

4

6

8

10

–10

f (x ) = x 3 – 7

Chapter 1 Review Exercises 55

17. (x− h)2 + (y − k)2 = r2

(x− 0)2 + [y − (−4)]2 =(

32

)2

Substituting

x2 + (y + 4)2 =94

19. The center is the midpoint of the diameter:(−3 + 72

,5 + 3

2

)=(

42,82

)= (2, 4)

Use the center and either endpoint of the diameter to findthe radius. We use the point (7, 3).

r =√

(7 − 2)2 + (3 − 4)2 =√

52 + (−1)2 =√25 + 1 =

√26

The equation of the circle is (x−2)2 + (y−4)2 = (√

26)2,or (x− 2)2 + (y − 4)2 = 26.

21. The correspondence is a function because each memberof the domain corresponds to exactly one member of therange.

23. The relation is a function, because no two ordered pairshave the same first coordinate and different second co-ordinates. The domain is the set of first coordinates:{−2, 0, 1, 2, 7}. The range is the set of second coordinates:{−7,−4,−2, 2, 7}.

25. f(x) =x− 7x + 5

a) f(7) =7 − 77 + 5

=012

= 0

b) f(x + 1) =x + 1 − 7x + 1 + 5

=x− 6x + 6

c) f(−5) =−5 − 7−5 + 5

=−120

Since division by 0 is not defined, f(−5) does not exist.

d) f

(− 1

2

)=

−12− 7

−12

+ 5=

−152

92

= −152

· 29

=

−3/ · 5 · 2/2/ · 3/ · 3 = −5

3

27. This is not the graph of a function, because we can find avertical line that crosses the graph more than once.

29. This is not the graph of a function, because we can find avertical line that crosses the graph more than once.

31. We can substitute any real number for x. Thus, the do-main is the set of all real numbers, or (−∞,∞).

33. Find the inputs that make the denominator zero:

x2 − 6x + 5 = 0

(x− 1)(x− 5) = 0

x− 1 = 0 or x− 5 = 0

x = 1 or x = 5

The domain is {x|x = 1 and x = 5}, or(−∞, 1) ∪ (1, 5) ∪ (5,∞).

35.

The inputs on the x axis extend from −4 to 4, so thedomain is [−4, 4].

The outputs on the y-axis extend from 0 to 4, so the rangeis [0, 4].

37.

Every point on the x-axis corresponds to a point on thegraph, so the domain is the set of all real numbers, or(−∞,∞).

Each point on the y-axis also corresponds to a point onthe graph, so the range is the set of all real numbers, or(−∞,∞).

39. a) Yes. Each input is 1 more than the one that pre-cedes it.

b) No. The change in the output varies.

c) No. Constant changes in inputs do not result inconstant changes in outputs.

41. m =y2 − y1

x2 − x1

=−6 − (−11)

5 − 2=

53

43. m =y2 − y1

x2 − x1

=0 − 312− 1

2

=−30

The slope is not defined.

45. y = − 711

x− 6

The equation is in the form y = mx+b. The slope is − 711

,

and the y-intercept is (0,−6).

Copyright © 2013 Pearson Education, Inc.

Page 59: JUDITH A. PENNA -  · PDF fileSTUDENT’S SOLUTIONS MANUAL JUDITH A. PENNA Indiana University Purdue University Indianapolis COLLEGE ALGEBRA: GRAPHS AND MODELS FIFTH EDITION

y

x�4 �2 2 4

�4

�2

2

4

y � ��x � 314

56 Chapter 1: Graphs, Functions, and Models

47. Graph y = −14x + 3.

Plot the y-intercept, (0, 3). We can think of the slope as−14

. Start at (0, 3) and find another point by moving down

1 unit and right 4 units. We have the point (4, 2).

We could also think of the slope as1−4

. Then we can start

at (0, 3) and find another point by moving up 1 unit andleft 4 units. We have the point (−4, 4). Connect the threepoints and draw the graph.

49. a) T (d) = 10d + 20

T (5) = 10(5) + 20 = 70◦C

T (20) = 10(20) + 20 = 220◦C

T (1000) = 10(1000) + 20 = 10, 020◦C

b) 5600 km is the maximum depth. Domain: [0, 5600].

51. y − y1 = m(x− x1)

y − (−1) = 3(x− (−2))

y + 1 = 3(x + 2)

y + 1 = 3x + 6

y = 3x + 5

53. The horizontal line that passes through(− 4,

25

)is

25

unit

above the x-axis. An equation of the line is y =25.

The vertical line that passes through(− 4,

25

)is 4 units

to the left of the y-axis. An equation of the line is x = −4.

55. 3x− 2y = 8 6x− 4y = 2

y =32x− 4 y =

32x− 1

2

The lines have the same slope,32, and different

y-intercepts, (0,−4) and(

0,−12

), so they are parallel.

57. The slope of y =32x + 7 is

32

and the slope of y = −23x− 4

is −23. Since

32

(− 2

3

)= −1, the lines are perpendicular.

59. From Exercise 58 we know that the slope of the given line

is −23. The slope of a line perpendicular to this line is the

negative reciprocal of −23, or

32.

We use the slope-intercept equation to find the y-intercept.y = mx + b

−1 =32· 1 + b

−1 =32

+ b

−52

= b

Then the equation of the desired line is y =32x− 5

2.

61. 4y − 5 = 1

4y = 6

y =32

The solution is32.

63. 5(3x + 1) = 2(x− 4)

15x + 5 = 2x− 8

13x = −13

x = −1The solution is −1.

65.35y − 2 =

38

The LCD is 40

40(

35y−2

)= 40 · 3

8Multiplying to clear fractions

24y − 80 = 15

24y = 95

y =9524

The solution is9524

.

67. x− 13 = −13 + x

−13 = −13 Subtracting x

We have an equation that is true for any real num-ber, so the solution set is the set of all real numbers,{x|x is a real number}, or (−∞,∞).

69. Familiarize. Let a = the amount originally invested. Us-ing the simple interest formula, I = Prt, we see that theinterest earned at 5.2% interest for 1 year is a(0.052) · 1 =0.052a.

Translate.Amountinvested︸ ︷︷ ︸ plus

interestearned︸ ︷︷ ︸ is $2419.60

� � � � �a + 0.052a = 2419.60

Carry out. We solve the equation.a + 0.052a = 2419.60

1.052a = 2419.60

a = 2300

Check. 5.2% of $2300 is 0.052($2300), or $119.60, and$2300 + $119.60 = $2419.60. The answer checks.

State. $2300 was originally invested.

Copyright © 2013 Pearson Education, Inc.

Page 60: JUDITH A. PENNA -  · PDF fileSTUDENT’S SOLUTIONS MANUAL JUDITH A. PENNA Indiana University Purdue University Indianapolis COLLEGE ALGEBRA: GRAPHS AND MODELS FIFTH EDITION

0 12

0 3

1 2��

Chapter 1 Test 57

71. 6x− 18 = 0

6x = 18

x = 3

The zero of the function is 3.

73. 2 − 10x = 0

−10x = −2

x =15, or 0.2

The zero of the function is15, or 0.2.

75. 2x− 5 < x + 7

x < 12

The solution set is {x|x < 12}, or (−∞, 12).

77. −3 ≤ 3x + 1 ≤ 5

−4 ≤ 3x ≤ 4

−43≤ x ≤ 4

3[− 4

3,43

]

79. 2x < −1 or x− 3 > 0

x < −12

or x > 3

The solution set is{x

∣∣∣∣x < −12or x > 3

}, or(

−∞,−12

)∪ (3,∞).

81. Familiarize and Translate. The number of home-schooled children in the U.S., in millions, is estimated bythe equation y = 0.08x + 0.83, where x is the number ofyears after 1999. We want to know for what year thisnumber will exceed 2.0 million, so we have

0.08x + 0.83 > 2.

Carry out. We solve the inequality.0.08x + 0.83 > 2

0.08x > 1.17

x > 14.625

Check. When x = 14.625, y = 0.08(14.625) + 0.83 = 2.0.As a partial check, we could try a value less than 14.625and a value greater than 14.625. When x = 14, we havey = 0.08(14) + 0.83 = 1.95 < 2.0; when x = 15, we havey = 0.08(15) + 0.83 = 2.03 > 2.0. Since y = 2.0 whenx = 14.625 and y > 2.0 when x = 15 > 14, the answer isprobably correct.

State. In years more than about 14 years after 1999, orin years after 2013, the number of homeschooled childrenwill exceed 2.0 million.

83. f(x) =x + 38 − 4x

When x = 2, the denominator is 0, so 2 is not in thedomain of the function. Thus, the domain is(−∞, 2) ∪ (2,∞) and answer B is correct.

85. The graph of f(x) = −12x− 2 has slope −1

2, so it slants

down from left to right. The y-intercept is (0,−2). Thus,graph C is the graph of this function.

87. f(x) =√

1 − x

x− |x|We cannot find the square root of a negative number, sox ≤ 1. Division by zero is undefined, so x < 0.

Domain of f is {x|x < 0}, or (−∞, 0).

89. Think of the slopes as−3/5

1and

1/21

. The graph of

f(x) changes35

unit vertically for each unit of horizon-

tal change while the graph of g(x) changes12

unit ver-

tically for each unit of horizontal change. Since35>

12,

the graph of f(x) = −35x + 4 is steeper than the graph of

g(x) =12x− 6.

91. The solution set of a disjunction is a union of sets, so it isnot possible for a disjunction to have no solution.

93. By definition, the notation 3 < x < 4 indicates that3 < x and x < 4. The disjunction x < 3 or x > 4 cannotbe written 3 > x > 4, or 4 < x < 3, because it is notpossible for x to be greater than 4 and less than 3.

Chapter 1 Test

1. 5y − 4 = x

5 · 910

− 4 ?12∣∣9

2− 4

∣∣∣∣∣12

∣∣ 12

TRUE(12,

910

)is a solution.

2. 5x− 2y = −10

To find the x-intercept we replace y with 0 and solve forx.

5x− 2 · 0 = −10

5x = −10

x = −2

The x-intercept is (−2, 0).

Copyright © 2013 Pearson Education, Inc.

Page 61: JUDITH A. PENNA -  · PDF fileSTUDENT’S SOLUTIONS MANUAL JUDITH A. PENNA Indiana University Purdue University Indianapolis COLLEGE ALGEBRA: GRAPHS AND MODELS FIFTH EDITION

y

x

23

�2

�4

�2�4 42

(0, 5)

5x � 2y � �10

(�2, 0) y

x

2

4

�2

�4

�2�4 42

f(x) � �x � 2� � 3

58 Chapter 1: Graphs, Functions, and Models

To find the y-intercept we replace x with 0 and solve fory.

5 · 0 − 2y = −10

−2y = −10

y = 5

The y-intercept is (0, 5).

We plot the intercepts and draw the line that containsthem. We could find a third point as a check that theintercepts were found correctly.

3. d =√

(5 − (−1))2 + (8 − 5)2 =√

62 + 32 =√

36 + 9 =√

45 ≈ 6.708

4. m =(−2 + (−4)

2,6 + 3

2

)=(−6

2,92

)=(− 3,

92

)

5. (x + 4)2 + (y − 5)2 = 36

[x− (−4)]2 + (y − 5)2 = 62

Center: (−4, 5); radius: 6

6. [x− (−1)]2 + (y − 2)2 = (√

5)2

(x + 1)2 + (y − 2)2 = 5

7. a) The relation is a function, because no two orderedpairs have the same first coordinate and differentsecond coordinates.

b) The domain is the set of first coordinates:{−4, 0, 1, 3}.

c) The range is the set of second coordinates: {0, 5, 7}.8. f(x) = 2x2 − x + 5

a) f(−1) = 2(−1)2 − (−1) + 5 = 2 + 1 + 5 = 8

b) f(a + 2) = 2(a + 2)2 − (a + 2) + 5

= 2(a2 + 4a + 4) − (a + 2) + 5

= 2a2 + 8a + 8 − a− 2 + 5

= 2a2 + 7a + 11

9. f(x) =1 − x

x

a) f(0) =1 − 0

0=

10

Since the division by 0 is not defined, f(0) does notexist.

b) f(1) =1 − 1

1=

01

= 0

10. From the graph we see that when the input is −3, theoutput is 0, so f(−3) = 0.

11. a) This is not the graph of a function, because we canfind a vertical line that crosses the graph more thanonce.

b) This is the graph of a function, because there is novertical line that crosses the graph more than once.

12. The input 4 results in a denominator of 0. Thus the do-main is {x|x = 4}, or (−∞, 4) ∪ (4,∞).

13. We can substitute any real number for x. Thus the domainis the set of all real numbers, or (−∞,∞).

14. We cannot find the square root of a negative number. Thus25−x2 ≥ 0 and the domain is {x|−5 ≤ x ≤ 5}, or [−5, 5].

15. a)

b) Each point on the x-axis corresponds to a point onthe graph, so the domain is the set of all real num-bers, or (−∞,∞).

c) The number 3 is the smallest output on the y-axisand every number greater than 3 is also an output,so the range is [3,∞).

16. m =5 − 2

3−2 − (−2)

=

1330

The slope is not defined.

17. m =12 − (−10)−8 − 4

=22−12

= −116

18. m =6 − 6

34− (−5)

=0234

= 0

19. We have the points (1995,21.6) and (2008,11.4).

m =11.4 − 21.62008 − 1995

=−10.2

13≈ −0.8

The average rate of change in the percent of 12th graderswho smoke daily was about −0.8% per year from 1995 to2008.

20. −3x + 2y = 5

2y = 3x + 5

y =32x +

52

Slope:32; y-intercept:

(0,

52

)

21. C(t) = 80 + 49.95t

2 yr = 2 · 1 yr = 2 · 12 months = 24 months

C(24) = 80 + 49.95(24) = $1278.80

Copyright © 2013 Pearson Education, Inc.

Page 62: JUDITH A. PENNA -  · PDF fileSTUDENT’S SOLUTIONS MANUAL JUDITH A. PENNA Indiana University Purdue University Indianapolis COLLEGE ALGEBRA: GRAPHS AND MODELS FIFTH EDITION

Chapter 1 Test 59

22. y = mx + b

y = −58x− 5

23. First we find the slope:

m =−2 − 4

3 − (−5)=

−68

= −34

Use the point-slope equation.

Using (−5, 4): y − 4 = −34(x− (−5)), or

y − 4 = −34(x + 5)

Using (3,−2): y − (−2) = −34(x− 3), or

y + 2 = −34(x− 3)

In either case, we have y = −34x +

14.

24. The vertical line that passes through(− 3

8, 11)

is38

unit

to the left of the y-axis. An equation of the line is x = −38.

25. 2x + 3y = −12 2y − 3x = 8

y = −23x− 4 y =

32x + 4

m1 = −23, m2 =

32; m1m2 = −1.

The lines are perpendicular.

26. First find the slope of the given line.

x + 2y = −6

2y = −x− 6

y = −12x− 3; m = −1

2

A line parallel to the given line has slope −12. We use the

point-slope equation.

y − 3 = −12(x− (−1))

y − 3 = −12(x + 1)

y − 3 = −12x− 1

2

y = −12x +

52

27. First we find the slope of the given line.

x + 2y = −6

2y = −x− 6

y = −12x− 3, m = −1

2The slope of a line perpendicular to this line is the negative

reciprocal of −12, or 2. Now we find an equation of the line

with slope 2 and containing (−1, 3).

Using the slope-intercept equation:

y = mx + b

3 = 2(−1) + b

3 = −2 + b

5 = b

The equation is y = 2x + 5.

Using the point-slope equation.

y − y1 = m(x− x1)

y − 3 = 2(x− (−1))

y − 3 = 2(x + 1)

y − 3 = 2x + 2

y = 2x + 5

28. a) Answers may vary depending on the data pointsused. We will use (1, 12, 485) and (3, 11, 788).

m =11, 788 − 12, 485

3 − 1=

−6972

= −348.5

We will use the point-slope equation with(1, 12, 485).

y − 12, 485 = −348.5(x− 1)

y − 12, 485 = −348.5x + 348.5

y = −348.5x + 12, 833.5,

where x is the number of years after 2005.In 2010, x = 2010 − 2005 = 5.

y = −348.5(5) + 12, 833.5 = 11, 091 mi

In 2013, x = 2013 − 2005 = 8.

y = −348.5(8) + 12, 833.5 = 10, 045.5 mi

b) y = −234.7x + 12, 623.8, where x is the number ofyears after 2005.

In 2010, y ≈ 11, 450 mi.

In 2013, y ≈ 10, 746 mi.

r ≈ −0.9036

29. 6x + 7 = 1

6x = −6

x = −1

The solution is −1.

30. 2.5 − x = −x + 2.5

2.5 = 2.5 True equation

The solution set is {x|x is a real number}, or (−∞,∞).

31. 32y − 4 =

53y + 6 The LCD is 6.

6(

32y − 4

)= 6

(53y + 6

)9y − 24 = 10y + 36

−24 = y + 36

−60 = y

The solution is −60.

Copyright © 2013 Pearson Education, Inc.

Page 63: JUDITH A. PENNA -  · PDF fileSTUDENT’S SOLUTIONS MANUAL JUDITH A. PENNA Indiana University Purdue University Indianapolis COLLEGE ALGEBRA: GRAPHS AND MODELS FIFTH EDITION

�3 0

60 Chapter 1: Graphs, Functions, and Models

32. 2(4x + 1) = 8 − 3(x− 5)

8x + 2 = 8 − 3x + 15

8x + 2 = 23 − 3x

11x + 2 = 23

11x = 21

x =2111

The solution is2111

.

33. Familiarize. Let l = the length, in meters. Then34l =

the width. Recall that the formula for the perimeter P ofa rectangle with length l and width w is P = 2l + 2w.

Translate.The perimeter︸ ︷︷ ︸ is 210 m.︸ ︷︷ ︸� � �

2l + 2 · 34l = 210

Carry out. We solve the equation.

2l + 2 · 34l = 210

2l +32l = 210

72l = 210

l = 60

If l = 60, then34l =

34· 60 = 45.

Check. The width, 45 m, is three-fourths of the length,60 m. Also, 2 · 60 m + 2 · 45 m = 210 m, so the answerchecks.

State. The length is 60 m and the width is 45 m.

34. Familiarize. Let p = the wholesale price of the juice.

Translate. We express 25/c as $0.25.

Wholesaleprice plus

50% ofwholesale

priceplus $0.25 is $2.95.

� � � � � � �p + 0.5p + 0.25 = 2.95

Carry out. We solve the equation.

p + 0.5p + 0.25 = 2.95

1.5p + 0.25 = 2.95

1.5p = 2.7

p = 1.8

Check. 50% of $1.80 is $0.90 and $1.80 + $0.90 + $0.25 =$2.95, so the answer checks.

State. The wholesale price of a bottle of juice is $1.80.

35. 3x + 9 = 0 Setting f(x) = 0

3x = −9

x = −3

The zero of the function is −3.

36. 5 − x ≥ 4x + 20

5 − 5x ≥ 20

−5x ≥ 15

x ≤ −3 Dividing by −5 and reversingthe inequality symbol

The solution set is {x|x ≤ −3}, or (−∞,−3].

37. −7 < 2x + 3 < 9

−10 < 2x < 6 Subtracting 3

−5 < x < 3 Dividing by 2

The solution set is (−5, 3).

38. 2x− 1 ≤ 3 or 5x + 6 ≥ 26

2x ≤ 4 or 5x ≥ 20

x ≤ 2 or x ≥ 4

The solution set is (−∞, 2] ∪ [4,∞).

39. Familiarize. Let t = the number of hours a move requires.Then Morgan Movers charges 90+25t to make a move andMcKinley Movers charges 40t.

Translate.Morgan Movers’

charge︸ ︷︷ ︸ is less than︸ ︷︷ ︸ McKinley Movers’charge.︸ ︷︷ ︸� � �

90 + 25t < 40t

Carry out. We solve the inequality.

90 + 25t < 40t

90 < 15t

6 < t

Check. For t = 6, Morgan Movers charge 90 + 25 · 6,or $240, and McKinley Movers charge 40 · 6, or $240, sothe charge is the same for 6 hours. As a partial check, wecan find the charges for a value of t greater than 6. Forinstance, for 6.5 hr Morgan Movers charge 90 + 25(6.5),or $252.50, and McKinley Movers charge 40(6.5), or $260.Since Morgan Movers cost less for a value of t greater than6, the answer is probably correct.

State. It costs less to hire Morgan Movers when a movetakes more than 6 hr.

Copyright © 2013 Pearson Education, Inc.

Page 64: JUDITH A. PENNA -  · PDF fileSTUDENT’S SOLUTIONS MANUAL JUDITH A. PENNA Indiana University Purdue University Indianapolis COLLEGE ALGEBRA: GRAPHS AND MODELS FIFTH EDITION

Chapter 1 Test 61

40. The slope is −12, so the graph slants down from left to

right. The y-intercept is (0, 1). Thus, graph B is the graph

of g(x) = 1 − 12x.

41. First we find the value of x for which x + 2 = −2:x + 2 = −2

x = −4

Now we find h(−4 + 2), or h(−2).

h(−4 + 2) =12(−4) = −2

Copyright © 2013 Pearson Education, Inc.

Page 65: JUDITH A. PENNA -  · PDF fileSTUDENT’S SOLUTIONS MANUAL JUDITH A. PENNA Indiana University Purdue University Indianapolis COLLEGE ALGEBRA: GRAPHS AND MODELS FIFTH EDITION

Copyright © 2013 Pearson Education, Inc.

Page 66: JUDITH A. PENNA -  · PDF fileSTUDENT’S SOLUTIONS MANUAL JUDITH A. PENNA Indiana University Purdue University Indianapolis COLLEGE ALGEBRA: GRAPHS AND MODELS FIFTH EDITION

x

y

1–1–3 32 54–2–4–5–1

–2

–3

–4

1

2

3

4

5

–5

f (x ) = x 2

x

y

1–1–3 32 54–2–4–5–1

–2

–3

–4

1

2

3

4

5

–5

f (x ) = 5 — | x |

x

y

1–1–3 32 54–2–4–5–1

–2

–3

–4

1

2

3

4

5

–5

f (x ) = x 2 — 6x + 10

Chapter 2

More on Functions

Exercise Set 2.1

1. a) For x-values from −5 to 1, the y-values increase from−3 to 3. Thus the function is increasing on theinterval (−5, 1).

b) For x-values from 3 to 5, the y-values decrease from3 to 1. Thus the function is decreasing on the inter-val (3, 5).

c) For x-values from 1 to 3, y is 3. Thus the functionis constant on (1, 3).

3. a) For x-values from −3 to −1, the y-values increasefrom −4 to 4. Also, for x-values from 3 to 5, they-values increase from 2 to 6. Thus the function isincreasing on (−3,−1) and on (3, 5).

b) For x-values from 1 to 3, the y-values decrease from3 to 2. Thus the function is decreasing on the inter-val (1, 3).

c) For x-values from −5 to −3, y is 1. Thus the func-tion is constant on (−5,−3).

5. a) For x-values from −∞ to −8, the y-values increasefrom −∞ to 2. Also, for x-values from −3 to −2, they-values increase from −2 to 3. Thus the functionis increasing on (−∞,−8) and on (−3,−2).

b) For x-values from −8 to −6, the y-values decreasefrom 2 to −2. Thus the function is decreasing onthe interval (−8,−6).

c) For x-values from −6 to −3, y is −2. Also, for x-values from −2 to ∞, y is 3. Thus the function isconstant on (−6,−3) and on (−2,∞).

7. The x-values extend from −5 to 5, so the domain is [−5, 5].

The y-values extend from −3 to 3, so the range is [−3, 3].

9. The x-values extend from −5 to −1 and from 1 to 5, sothe domain is [−5,−1] ∪ [1, 5].

The y-values extend from −4 to 6, so the range is [−4, 6].

11. The x-values extend from −∞ to ∞, so the domain is(−∞,∞).

The y-values extend from −∞ to 3, so the range is (−∞, 3].

13. From the graph we see that a relative maximum value ofthe function is 3.25. It occurs at x = 2.5. There is norelative minimum value.

The graph starts rising, or increasing, from the left andstops increasing at the relative maximum. From this point,the graph decreases. Thus the function is increasing on(−∞, 2.5) and is decreasing on (2.5,∞).

15. From the graph we see that a relative maximum value ofthe function is 2.370. It occurs at x = −0.667. We alsosee that a relative minimum value of 0 occurs at x = 2.

The graph starts rising, or increasing, from the left andstops increasing at the relative maximum. From this pointit decreases to the relative minimum and then increasesagain. Thus the function is increasing on (−∞,−0.667)and on (2,∞). It is decreasing on (−0.667, 2).

17.

The function is increasing on (0,∞) and decreasing on(−∞, 0). We estimate that the minimum is 0 at x = 0.There are no maxima.

19.

The function is increasing on (−∞, 0) and decreasing on(0,∞). We estimate that the maximum is 5 at x = 0.There are no minima.

21.

The function is decreasing on (−∞, 3) and increasing on(3,∞). We estimate that the minimum is 1 at x = 3.There are no maxima.

Copyright © 2013 Pearson Education, Inc.

Page 67: JUDITH A. PENNA -  · PDF fileSTUDENT’S SOLUTIONS MANUAL JUDITH A. PENNA Indiana University Purdue University Indianapolis COLLEGE ALGEBRA: GRAPHS AND MODELS FIFTH EDITION

y � �x2 � 300x � 6

00

50,000

300

20 – w

w

64 Chapter 2: More on Functions

23.

Beginning at the left side of the window, the graph firstdrops as we move to the right. We see that the function isdecreasing on (−∞, 1). We then find that the function isincreasing on (1, 3) and decreasing again on (3,∞). TheMAXIMUM and MINIMUM features also show that therelative maximum is −4 at x = 3 and the relative minimumis −8 at x = 1.

25.

We find that the function is increasing on (−1.552, 0) andon (1.552,∞) and decreasing on (−∞,−1.552) and on(0, 1.552). The relative maximum is 4.07 at x = 0 andthe relative minima are −2.314 at x = −1.552 and −2.314at x = 1.552.

27. a)

b) 22, 506 at a = 150

c) The greatest number of baskets will be sold when$150 thousand is spent on advertising. For thatamount, 22,506 baskets will be sold.

29. Graph y =8x

x2 + 1.

Increasing: (−1, 1)

Decreasing: (−∞,−1), (1,∞)

31. Graph y = x√

4 − x2, for −2 ≤ x ≤ 2.

Increasing: (−1.414, 1.414)

Decreasing: (−2,−1.414), (1.414, 2)

33. If x = the length of the rectangle, in meters, then the

width is480 − 2x

2, or 240−x. We use the formula Area =

length × width:

A(x) = x(240 − x), or

A(x) = 240x− x2

35. After t minutes, the balloon has risen 120t ft. We use thePythagorean theorem.[d(t)]2 = (120t)2 + 4002

d(t) =√

(120t)2 + 4002

We considered only the positive square root since distancemust be nonnegative.

37. Let w = the width of the rectangle. Then the

length =40 − 2w

2, or 20 − w. Divide the rectangle into

quadrants as shown below.

In each quadrant there are two congruent triangles. Onetriangle is part of the rhombus and both are part of therectangle. Thus, in each quadrant the area of the rhombusis one-half the area of the rectangle. Then, in total, thearea of the rhombus is one-half the area of the rectangle.

A(w) =12(20 − w)(w)

A(w) = 10w − w2

239. We will use similar triangles, expressing all distances in

feet.(

6 in. =12

ft, s in. =s

12ft, and d yd = 3d ft

)We

have

3d7

=

12s

12s

12· 3d = 7 · 1

2sd

4=

72

d =4s· 72, so

d(s) =14s.

41. a) If the length = x feet, then the width = 30−x feet.

A(x) = x(30 − x)

A(x) = 30x− x2

b) The length of the rectangle must be positive andless than 30 ft, so the domain of the function is{x|0 < x < 30}, or (0, 30).

c) We see from the graph that the maximum value ofthe area function on the interval (0, 30) appears tobe 225 when x = 15. Then the dimensions that yieldthe maximum area are length = 15 ft and width= 30 − 15, or 15 ft.

Copyright © 2013 Pearson Education, Inc.

Page 68: JUDITH A. PENNA -  · PDF fileSTUDENT’S SOLUTIONS MANUAL JUDITH A. PENNA Indiana University Purdue University Indianapolis COLLEGE ALGEBRA: GRAPHS AND MODELS FIFTH EDITION

y � 112x � 16x2

00

250

7

0

150

0 16

y � x 256 � x2

4

2

�2

�4

42�2�4 x

y

4

�2

�4

42�2�4 x

y

Exercise Set 2.1 65

43. a) If the height of the file is x inches, then the widthis 14− 2x inches and the length is 8 in. We use theformula Volume = length × width × height to findthe volume of the file.

V (x) = 8(14 − 2x)x, or

V (x) = 112x− 16x2

b) The height of the file must be positive and less thanhalf of the measure of the long side of the piece of

plastic. Thus, the domain is{x

∣∣∣∣0 < x <142

}, or

{x|0 < x < 7}.c)

d) Using the MAXIMUM feature, we find that themaximum value of the volume function occurs whenx = 3.5, so the file should be 3.5 in. tall.

45. a) The length of a diameter of the circle (and a di-agonal of the rectangle) is 2 · 8, or 16 ft. Let l =the length of the rectangle. Use the Pythagoreantheorem to write l as a function of x.

x2 + l2 = 162

x2 + l2 = 256

l2 = 256 − x2

l =√

256 − x2

Since the length must be positive, we consideredonly the positive square root.

Use the formula Area = length × width to find thearea of the rectangle:

A(x) = x√

256 − x2

b) The width of the rectangle must be positive and lessthan the diameter of the circle. Thus, the domainof the function is {x|0 < x < 16}, or (0, 16).

c)

d) Using the MAXIMUM feature, we find that the max-imum area occurs when x is about 11.314. When x ≈11.314,

√256 − x2 ≈ √

256 − (11.314)2 ≈ 11.313.Thus, the dimensions that maximize the area areabout 11.314 ft by 11.313 ft. (Answers may varyslightly due to rounding differences.)

47. g(x) ={

x + 4, for x ≤ 1,8 − x, for x > 1

Since −4 ≤ 1, g(−4) = −4 + 4 = 0.

Since 0 ≤ 1, g(0) = 0 + 4 = 4.

Since 1 ≤ 1, g(1) = 1 + 4 = 5.

Since 3 > 1, g(3) = 8 − 3 = 5.

49. h(x) =

{−3x− 18, for x < −5,1, for −5 ≤ x < 1,x + 2, for x ≥ 1

Since −5 is in the interval [−5, 1), h(−5) = 1.

Since 0 is in the interval [−5, 1), h(0) = 1.

Since 1 ≥ 1, h(1) = 1 + 2 = 3.

Since 4 ≥ 1, h(4) = 4 + 2 = 6.

51. f(x) =

12x, for x < 0,

x + 3, for x ≥ 0

We create the graph in two parts. Graph f(x) =12x for

inputs x less than 0. Then graph f(x) = x + 3 for inputsx greater than or equal to 0.

53. f(x) =

−34x + 2, for x < 4,

−1, for x ≥ 4We create the graph in two parts. Graph

f(x) = −34x + 2 for inputs x less than 4. Then graph

f(x) = −1 for inputs x greater than or equal to 4.

Copyright © 2013 Pearson Education, Inc.

Page 69: JUDITH A. PENNA -  · PDF fileSTUDENT’S SOLUTIONS MANUAL JUDITH A. PENNA Indiana University Purdue University Indianapolis COLLEGE ALGEBRA: GRAPHS AND MODELS FIFTH EDITION

4

2

�2

�4

42�2�4 x

y

y

x�4 �2 2 4

�4

�2

2

4

8

2

�4

�8

84�4�8 x

y

4

2

�4

�2

42�4 x

y

g(x) � 1 � �x�

66 Chapter 2: More on Functions

55. f(x) =

x + 1, for x ≤ −3,

−1, for −3 < x < 4

12x, for x ≥ 4

We create the graph in three parts. Graph f(x) = x + 1for inputs x less than or equal to −3. Graph f(x) = −1for inputs greater than −3 and less than 4. Then graph

f(x) =12x for inputs greater than or equal to 4.

57. g(x) =

12x− 1, for x < 0,

3, for 0 ≤ x ≤ 1

−2x, for x > 1

We create the graph in three parts. Graph g(x) =12x− 1

for inputs less than 0. Graph g(x) = 3 for inputs greaterthan or equal to 0 and less than or equal to 1. Then graphg(x) = −2x for inputs greater than 1.

59. f(x) =

2, for x = 5,

x2 − 25x− 5

, for x �= 5

When x �= 5, the denominator of (x2 − 25)/(x − 5) isnonzero so we can simplify:

x2 − 25x− 5

=(x + 5)(x− 5)

x− 5= x + 5.

Thus, f(x) = x + 5, for x �= 5.

The graph of this part of the function consists of a linewith a “hole” at the point (5, 10), indicated by an opendot. At x = 5, we have f(5) = 2, so the point (5, 2) isplotted below the open dot.

61. f(x) = [[x]]

See Example 9.

63. f(x) = 1 + [[x]]

This function can be defined by a piecewise function withan infinite number of statements:

f(x) =

.

.

.−1, for −2 ≤ x < −1,0, for −1 ≤ x < 0,1, for 0 ≤ x < 1,2, for 1 ≤ x < 2,...

65. From the graph we see that the domain is (−∞,∞) andthe range is (−∞, 0) ∪ [3,∞).

67. From the graph we see that the domain is (−∞,∞) andthe range is [−1,∞).

69. From the graph we see that the domain is (−∞,∞) andthe range is {y|y ≤ −2 or y = −1 or y ≥ 2}.

71. From the graph we see that the domain is (−∞,∞) andthe range is {−5,−2, 4}. An equation for the function is:

f(x) =

{−2, for x < 2,−5, for x = 2,4, for x > 2

Copyright © 2013 Pearson Education, Inc.

Page 70: JUDITH A. PENNA -  · PDF fileSTUDENT’S SOLUTIONS MANUAL JUDITH A. PENNA Indiana University Purdue University Indianapolis COLLEGE ALGEBRA: GRAPHS AND MODELS FIFTH EDITION

8

6

4

2

42 t

C

E

D

CA B

h

r 6 – r

6

10

Exercise Set 2.1 67

73. From the graph we see that the domain is (−∞,∞) andthe range is (−∞,−1] ∪ [2,∞). Finding the slope of eachsegment and using the slope-intercept or point-slope for-mula, we find that an equation for the function is:

g(x) =

{x, for x ≤ −1,2, for −1 < x ≤ 2,x, for x > 2

This can also be expressed as follows:

g(x) =

{x, for x ≤ −1,2, for −1 < x < 2,x, for x ≥ 2

75. From the graph we see that the domain is [−5, 3] and therange is (−3, 5). Finding the slope of each segment andusing the slope-intercept or point-slope formula, we findthat an equation for the function is:

h(x) =

{x + 8, for −5 ≤ x < −3,3, for −3 ≤ x ≤ 1,3x− 6, for 1 < x ≤ 3

77. f(x) = 5x2 − 7

a) f(−3) = 5(−3)2 − 7 = 5 · 9 − 7 = 45 − 7 = 38

b) f(3) = 5 · 32 − 7 = 5 · 9 − 7 = 45 − 7 = 38

c) f(a) = 5a2 − 7

d) f(−a) = 5(−a)2 − 7 = 5a2 − 7

79. First find the slope of the given line.

8x− y = 10

8x = y + 10

8x− 10 = y

The slope of the given line is 8. The slope of a line per-pendicular to this line is the opposite of the reciprocal of

8, or −18.

y − y1 = m(x− x1)

y − 1 = −18[x− (−1)]

y − 1 = −18(x + 1)

y − 1 = −18x− 1

8

y = −18x +

78

81. Graph y = x4 + 4x3 − 36x2 − 160x + 400

Increasing: (−5,−2), (4,∞)

Decreasing: (−∞,−5), (−2, 4)

Relative maximum: 560 at x = −2

Relative minima: 425 at x = −5, −304 at x = 4

83. a) The function C(t) can be defined piecewise.

C(t) =

2, for 0 < t < 1,4, for 1 ≤ t < 2,6, for 2 ≤ t < 3,...

We graph this function.

b) From the definition of the function in part (a),we see that it can be written as

C(t) = 2[[t]] + 1, t > 0.

85. If [[x]]2 = 25, then [[x]] = −5 or [[x]] = 5. For−5 ≤ x < −4, [[x]] = −5. For 5 ≤ x < 6, [[x]] = 5.Thus, the possible inputs for x are{x| − 5 ≤ x < −4 or 5 ≤ x < 6}.

87. a) We add labels to the drawing in the text.

We write a proportion involving the lengths of thesides of the similar triangles BCD and ACE. Thenwe solve it for h.

h

6 − r=

106

h =106

(6 − r) =53(6 − r)

h =30 − 5r

3

Thus, h(r) =30 − 5r

3.

b) V = πr2h

V (r) = πr2

(30 − 5r

3

)Substituting for h

Copyright © 2013 Pearson Education, Inc.

Page 71: JUDITH A. PENNA -  · PDF fileSTUDENT’S SOLUTIONS MANUAL JUDITH A. PENNA Indiana University Purdue University Indianapolis COLLEGE ALGEBRA: GRAPHS AND MODELS FIFTH EDITION

68 Chapter 2: More on Functions

c) We first express r in terms of h.

h =30 − 5r

33h = 30 − 5r

5r = 30 − 3h

r =30 − 3h

5V = πr2h

V (h) = π

(30 − 3h

5

)2

h

Substituting for r

We can also write V (h) = πh

(30 − 3h

5

)2

.

Exercise Set 2.2

1. (f + g)(5) = f(5) + g(5)

= (52 − 3) + (2 · 5 + 1)

= 25 − 3 + 10 + 1

= 33

3. (f − g)(−1) = f(−1) − g(−1)

= ((−1)2 − 3) − (2(−1) + 1)

= −2 − (−1) = −2 + 1

= −1

5. (f/g)(− 1

2

)=

f(− 1

2

)g(− 1

2

)

=

(− 1

2

)2

− 3

2(− 1

2

)+ 1

=

14− 3

−1 + 1

=−11

40

Since division by 0 is not defined, (f/g)(− 1

2

)does not

exist.

7. (fg)(− 1

2

)= f

(− 1

2

)· g(− 1

2

)

=[(

− 12

)2

− 3][

2(− 1

2

)+ 1]

= −114

· 0 = 0

9. (g − f)(−1) = g(−1) − f(−1)

= [2(−1) + 1] − [(−1)2 − 3]

= (−2 + 1) − (1 − 3)

= −1 − (−2)

= −1 + 2

= 1

11. (h− g)(−4) = h(−4) − g(−4)

= (−4 + 4) −√−4 − 1

= 0 −√−5

Since√−5 is not a real number, (h−g)(−4) does not exist.

13. (g/h)(1) =g(1)h(1)

=√

1 − 11 + 4

=√

05

=05

= 0

15. (g + h)(1) = g(1) + h(1)

=√

1 − 1 + (1 + 4)

=√

0 + 5

= 0 + 5 = 5

17. f(x) = 2x + 3, g(x) = 3 − 5x

a) The domain of f and of g is the set of all real numbers,or (−∞,∞). Then the domain of f + g, f − g, ff ,

and fg is also (−∞,∞). For f/g we must exclude35

since g(3

5

)= 0. Then the domain of f/g is(

−∞,35

)∪(3

5,∞). For g/f we must exclude

−32

since f(− 3

2

)= 0. The domain of g/f is(

−∞,−32

)∪(− 3

2,∞).

b) (f + g)(x) = f(x) + g(x) = (2x + 3) + (3 − 5x) =−3x + 6

(f − g)(x) = f(x) − g(x) = (2x + 3) − (3 − 5x) =2x + 3 − 3 + 5x = 7x

(fg)(x) = f(x) · g(x) = (2x + 3)(3 − 5x) =6x− 10x2 + 9 − 15x = −10x2 − 9x + 9

(ff)(x) = f(x) · f(x) = (2x + 3)(2x + 3) =4x2 + 12x + 9

(f/g)(x) =f(x)g(x)

=2x + 33 − 5x

(g/f)(x) =g(x)f(x)

=3 − 5x2x + 3

Copyright © 2013 Pearson Education, Inc.

Page 72: JUDITH A. PENNA -  · PDF fileSTUDENT’S SOLUTIONS MANUAL JUDITH A. PENNA Indiana University Purdue University Indianapolis COLLEGE ALGEBRA: GRAPHS AND MODELS FIFTH EDITION

Exercise Set 2.2 69

19. f(x) = x− 3, g(x) =√x + 4

a) Any number can be an input in f , so the domain off is the set of all real numbers, or (−∞,∞).

The domain of g consists of all values of x for whichx+4 is nonnegative, so we have x+4 ≥ 0, or x ≥ −4.Thus, the domain of g is [−4,∞).

The domain of f + g, f − g, and fg is the set of allnumbers in the domains of both f and g. This is[−4,∞).

The domain of ff is the domain of f , or (−∞,∞).The domain of f/g is the set of all numbers inthe domains of f and g, excluding those for whichg(x) = 0. Since g(−4) = 0, the domain of f/g is(−4,∞).

The domain of g/f is the set of all numbers inthe domains of g and f , excluding those for whichf(x) = 0. Since f(3) = 0, the domain of g/f is[−4, 3) ∪ (3,∞).

b) (f + g)(x) = f(x) + g(x) = x− 3 +√x + 4

(f − g)(x) = f(x) − g(x) = x− 3 −√x + 4

(fg)(x) = f(x) · g(x) = (x− 3)√x + 4

(ff)(x) =[f(x)

]2 = (x− 3)2 = x2 − 6x + 9

(f/g)(x) =f(x)g(x)

=x− 3√x + 4

(g/f)(x) =g(x)f(x)

=√x + 4x− 3

21. f(x) = 2x− 1, g(x) = −2x2

a) The domain of f and of g is (−∞,∞). Then thedomain of f + g, f − g, fg, and ff is (−∞,∞).For f/g, we must exclude 0 since g(0) = 0. Thedomain of f/g is (−∞, 0) ∪ (0,∞). For g/f , we

must exclude12

since f(1

2

)= 0. The domain of

g/f is(−∞,

12

)∪(1

2,∞).

b) (f + g)(x) = f(x) + g(x) = (2x− 1) + (−2x2) =−2x2 + 2x− 1

(f − g)(x) = f(x) − g(x) = (2x− 1) − (−2x2) =2x2 + 2x− 1

(fg)(x) = f(x) · g(x) = (2x− 1)(−2x2) =−4x3 + 2x2

(ff)(x) = f(x) · f(x) = (2x− 1)(2x− 1) =4x2 − 4x + 1

(f/g)(x) =f(x)g(x)

=2x− 1−2x2

(g/f)(x) =g(x)f(x)

=−2x2

2x− 1

23. f(x) =√x− 3, g(x) =

√x + 3

a) Since f(x) is nonnegative for values of x in [3,∞),this is the domain of f . Since g(x) is nonnegativefor values of x in [−3,∞), this is the domain of g.The domain of f+g, f−g, and fg is the intersection

of the domains of f and g, or [3,∞). The domainof ff is the same as the domain of f , or [3,∞). Forf/g, we must exclude −3 since g(−3) = 0. This isnot in [3,∞), so the domain of f/g is [3,∞). Forg/f , we must exclude 3 since f(3) = 0. The domainof g/f is (3,∞).

b) (f + g)(x) = f(x) + g(x) =√x− 3 +

√x + 3

(f − g)(x) = f(x) − g(x) =√x− 3 −√

x + 3

(fg)(x) = f(x) · g(x) =√x−3 · √x + 3=

√x2−9

(ff)(x) = f(x) · f(x) =√x− 3 · √x− 3 = |x− 3|

(f/g)(x) =√x− 3√x + 3

(g/f)(x) =√x + 3√x− 3

25. f(x) = x + 1, g(x) = |x|a) The domain of f and of g is (−∞,∞). Then the

domain of f + g, f − g, fg, and ff is (−∞,∞).For f/g, we must exclude 0 since g(0) = 0. Thedomain of f/g is (−∞, 0) ∪ (0,∞). For g/f , wemust exclude −1 since f(−1) = 0. The domain ofg/f is (−∞,−1) ∪ (−1,∞).

b) (f + g)(x) = f(x) + g(x) = x + 1 + |x|(f − g)(x) = f(x) − g(x) = x + 1 − |x|(fg)(x) = f(x) · g(x) = (x + 1)|x|(ff)(x)=f(x)·f(x)=(x+1)(x+1)=x2 + 2x + 1

(f/g)(x) =x + 1|x|

(g/f)(x) =|x|

x + 1

27. f(x) = x3, g(x) = 2x2 + 5x− 3

a) Since any number can be an input for either f or g,the domain of f , g, f + g, f − g, fg, and ff is the setof all real numbers, or (−∞,∞).

Since g(−3) = 0 and g

(12

)= 0, the domain of f/g

is (−∞,−3) ∪(− 3,

12

)∪(1

2,∞).

Since f(0) = 0, the domain of g/f is(−∞, 0) ∪ (0,∞).

b) (f + g)(x) = f(x) + g(x) = x3 + 2x2 + 5x− 3

(f − g)(x) = f(x)−g(x) = x3−(2x2+5x−3) =

x3 − 2x2 − 5x + 3

(fg)(x) = f(x) · g(x) = x3(2x2 + 5x− 3) =

2x5 + 5x4 − 3x3

(ff)(x) = f(x) · f(x) = x3 · x3 = x6

(f/g)(x) =f(x)g(x)

=x3

2x2 + 5x− 3

(g/f)(x) =g(x)f(x)

=2x2 + 5x− 3

x3

Copyright © 2013 Pearson Education, Inc.

Page 73: JUDITH A. PENNA -  · PDF fileSTUDENT’S SOLUTIONS MANUAL JUDITH A. PENNA Indiana University Purdue University Indianapolis COLLEGE ALGEBRA: GRAPHS AND MODELS FIFTH EDITION

y

x2 4 6 8 10

�4

�2

2

4

6

G � F

70 Chapter 2: More on Functions

29. f(x) =4

x + 1, g(x) =

16 − x

a) Since x + 1 = 0 when x = −1, we must exclude−1 from the domain of f . It is (−∞,−1)∪ (−1,∞).Since 6−x = 0 when x = 6, we must exclude 6 fromthe domain of g. It is (−∞, 6)∪(6,∞). The domainof f + g, f − g, and fg is the intersection of thedomains of f and g, or (−∞,−1)∪ (−1, 6)∪ (6,∞).The domain of ff is the same as the domain of f ,or (−∞,−1) ∪ (−1,∞). Since there are no valuesof x for which g(x) = 0 or f(x) = 0, the domain off/g and g/f is (−∞,−1) ∪ (−1, 6) ∪ (6,∞).

b) (f + g)(x) = f(x) + g(x) =4

x + 1+

16 − x

(f − g)(x) = f(x) − g(x) =4

x + 1− 1

6 − x

(fg)(x)=f(x)·g(x)=4

x+1· 16−x

=4

(x+1)(6−x)

(ff)(x)=f(x)·f(x)=4

x + 1· 4x + 1

=16

(x + 1)2, or

16x2 + 2x + 1

(f/g)(x) =

4x + 1

16 − x

=4

x + 1· 6 − x

1=

4(6 − x)x + 1

(g/f)(x) =

16 − x

4x + 1

=1

6 − x· x + 1

4=

x + 14(6 − x)

31. f(x) =1x

, g(x) = x− 3

a) Since f(0) is not defined, the domain of f is(−∞, 0) ∪ (0,∞). The domain of g is (−∞,∞).Then the domain of f + g, f − g, fg, and ff is(−∞, 0) ∪ (0,∞). Since g(3) = 0, the domain off/g is (−∞, 0)∪ (0, 3)∪ (3,∞). There are no valuesof x for which f(x) = 0, so the domain of g/f is(−∞, 0) ∪ (0,∞).

b) (f + g)(x) = f(x) + g(x) =1x

+ x− 3

(f−g)(x) = f(x)−g(x) =1x−(x−3) =

1x−x + 3

(fg)(x)=f(x)·g(x)=1x·(x−3)=

x−3x

, or 1 − 3x

(ff)(x) = f(x) · f(x) =1x· 1x

=1x2

(f/g)(x) =f(x)g(x)

=

1x

x− 3=

1x· 1x− 3

=1

x(x− 3)

(g/f)(x)=g(x)f(x)

=x−3

1x

=(x−3) · x1

=x(x−3), or

x2 − 3x

33. f(x) =3

x− 2, g(x) =

√x− 1

a) Since f(2) is not defined, the domain of f is(−∞, 2)∪ (2,∞). Since g(x) is nonnegative for val-ues of x in [1,∞), this is the domain of g. Thedomain of f + g, f − g, and fg is the intersectionof the domains of f and g, or [1, 2) ∪ (2,∞). Thedomain of ff is the same as the domain of f , or(−∞, 2)∪ (2,∞). For f/g, we must exclude 1 sinceg(1) = 0, so the domain of f/g is (1, 2) ∪ (2,∞).There are no values of x for which f(x) = 0, so thedomain of g/f is [1, 2) ∪ (2,∞).

b) (f + g)(x) = f(x) + g(x) =3

x− 2+

√x− 1

(f − g)(x) = f(x) − g(x) =3

x− 2−√

x− 1

(fg)(x) = f(x) · g(x) =3

x− 2(√x− 1), or

3√x− 1

x− 2

(ff)(x) = f(x) · f(x) =3

x− 2· 3x− 2

· 9(x− 2)2

(f/g)(x) =f(x)g(x)

=

3x− 2√x− 1

=3

(x− 2)√x− 1

(g/f)(x) =g(x)f(x)

=√x− 13

x− 2

=(x− 2)

√x− 1

3

35. From the graph we see that the domain of F is [2, 11] andthe domain of G is [1, 9]. The domain of F + G is the setof numbers in the domains of both F and G. This is [2, 9].

37. The domain of G/F is the set of numbers in the domains ofboth F and G (See Exercise 33.), excluding those for whichF = 0. Since F (3) = 0, the domain of G/F is [2, 3)∪(3, 9].

39.

41. From the graph, we see that the domain of F is [0, 9] andthe domain of G is [3, 10]. The domain of F +G is the setof numbers in the domains of both F and G. This is [3, 9].

43. The domain of G/F is the set of numbers in the domainsof both F and G (See Exercise 39.), excluding those forwhich F = 0. Since F (6) = 0 and F (8) = 0, the domainof G/F is [3, 6) ∪ (6, 8) ∪ (8, 9].

Copyright © 2013 Pearson Education, Inc.

Page 74: JUDITH A. PENNA -  · PDF fileSTUDENT’S SOLUTIONS MANUAL JUDITH A. PENNA Indiana University Purdue University Indianapolis COLLEGE ALGEBRA: GRAPHS AND MODELS FIFTH EDITION

6

4

2

�2

108642 x

y

G � F

Exercise Set 2.2 71

45.

47. a) P (x) = R(x) − C(x) = 60x− 0.4x2 − (3x + 13) =

60x− 0.4x2 − 3x− 13 = −0.4x2 + 57x− 13

b) R(100)=60·100−0.4(100)2 =6000−0.4(10, 000) =

6000 − 4000 = 2000

C(100) = 3 · 100 + 13 = 300 + 13 = 313

P (100) = R(100) − C(100) = 2000 − 313 = 1687

49. f(x) = 3x− 5

f(x + h) = 3(x + h) − 5 = 3x + 3h− 5f(x + h) − f(x)

h=

3x + 3h− 5 − (3x− 5)h

=3x + 3h− 5 − 3x + 5

h

=3hh

= 3

51. f(x) = 6x + 2

f(x + h) = 6(x + h) + 2 = 6x + 6h + 2f(x + h) − f(x)

h=

6x + 6h + 2 − (6x + 2)h

=6x + 6h + 2 − 6x− 2

h

=6hh

= 6

53. f(x) =13x + 1

f(x + h) =13(x + h) + 1 =

13x +

13h + 1

f(x + h) − f(x)h

=

13x +

13h + 1 −

(13x + 1

)h

=

13x +

13h + 1 − 1

3x− 1

h

=

13h

h=

13

55. f(x) =13x

f(x + h) =1

3(x + h)

f(x + h) − f(x)h

=

13(x + h)

− 13x

h

=

13(x + h)

· xx− 1

3x· x + h

x + h

h

=

x

3x(x + h)− x + h

3x(x + h)h

=

x− (x + h)3x(x + h)

h=

x− x− h

3x(x + h)h

=

−h

3x(x + h)h

=−h

3x(x + h)· 1h

=−h

3x(x + h) · h =−1 · h/

3x(x + h) · h/=

−13x(x + h)

, or − 13x(x + h)

57. f(x) = − 14x

f(x + h) = − 14(x + h)

f(x + h) − f(x)h

=− 1

4(x + h)−(− 1

4x

)h

=− 1

4(x + h)· xx−(− 1

4x

)· x + h

x + h

h

=− x

4x(x + h)+

x + h

4x(x + h)h

=

−x + x + h

4x(x + h)h

=

h

4x(x + h)h

=h

4x(x+h)· 1h

=h/·1

4x(x+h)·h/ =1

4x(x+h)

Copyright © 2013 Pearson Education, Inc.

Page 75: JUDITH A. PENNA -  · PDF fileSTUDENT’S SOLUTIONS MANUAL JUDITH A. PENNA Indiana University Purdue University Indianapolis COLLEGE ALGEBRA: GRAPHS AND MODELS FIFTH EDITION

2 4

2

4

�4 �2

�4

�2

x

y

y � 3x � 1

72 Chapter 2: More on Functions

59. f(x) = x2 + 1

f(x + h) = (x + h)2 + 1 = x2 + 2xh + h2 + 1

f(x + h) − f(x)h

=x2 + 2xh + h2 + 1 − (x2 + 1)

h

=x2 + 2xh + h2 + 1 − x2 − 1

h

=2xh + h2

h

=h(2x + h)

h

=h

h· 2x + h

1= 2x + h

61. f(x) = 4 − x2

f(x + h) = 4 − (x + h)2 = 4 − (x2 + 2xh + h2) =

4 − x2 − 2xh− h2

f(x + h) − f(x)h

=4 − x2 − 2xh− h2 − (4 − x2)

h

=4 − x2 − 2xh− h2 − 4 + x2

h

=−2xh− h2

h=

h/(−2x− h)h/

= −2x− h

63. f(x) = 3x2 − 2x + 1

f(x + h) = 3(x + h)2 − 2(x + h) + 1 =

3(x2 + 2xh + h2) − 2(x + h) + 1 =

3x2 + 6xh + 3h2 − 2x− 2h + 1

f(x) = 3x2 − 2x + 1f(x + h) − f(x)

h=

(3x2 + 6xh + 3h2−2x−2h + 1)−(3x2 − 2x + 1)h

=

3x2 + 6xh + 3h2 − 2x− 2h + 1 − 3x2 + 2x− 1h

=

6xh + 3h2 − 2hh

=h(6x + 3h− 2)

h · 1 =

h

h· 6x + 3h− 2

1= 6x + 3h− 2

65. f(x) = 4 + 5|x|f(x + h) = 4 + 5|x + h|f(x + h) − f(x)

h=

4 + 5|x + h| − (4 + 5|x|)h

=4 + 5|x + h| − 4 − 5|x|

h

=5|x + h| − 5|x|

h

67. f(x) = x3

f(x + h) = (x + h)3 = x3 + 3x2h + 3xh2 + h3

f(x) = x3

f(x + h) − f(x)h

=x3 + 3x2h + 3xh2 + h3 − x3

h=

3x2h + 3xh2 + h3

h=

h(3x2 + 3xh + h2)h · 1 =

h

h· 3x2 + 3xh + h2

1= 3x2 + 3xh + h2

69. f(x) =x− 4x + 3

f(x + h) − f(x)h

=

x + h− 4x + h + 3

− x− 4x + 3

h=

x + h− 4x + h + 3

− x− 4x + 3

h· (x + h + 3)(x + 3)(x + h + 3)(x + 3)

=

(x + h− 4)(x + 3) − (x− 4)(x + h + 3)h(x + h + 3)(x + 3)

=

x2+hx−4x+3x+3h−12−(x2+hx+3x−4x−4h−12)h(x+h+3)(x+3)

=

x2 + hx− x + 3h− 12 − x2 − hx + x + 4h + 12h(x + h + 3)(x + 3)

=

7hh(x + h + 3)(x + 3)

=h

h· 7(x + h + 3)(x + 3)

=

7(x + h + 3)(x + 3)

71. Graph y = 3x− 1.

We find some ordered pairs that are solutions of the equa-tion, plot these points, and draw the graph.

When x = −1, y = 3(−1) − 1 = −3 − 1 = −4.

When x = 0, y = 3 · 0 − 1 = 0 − 1 = −1.

When x = 2, y = 3 · 2 − 1 = 6 − 1 = 5.

x y

−1 −4

0 −1

2 5

73. Graph x− 3y = 3.

First we find the x- and y-intercepts.x− 3 · 0 = 3

x = 3

The x-intercept is (3, 0).

0 − 3y = 3

−3y = 3

y = −1

The y-intercept is (0,−1).

Copyright © 2013 Pearson Education, Inc.

Page 76: JUDITH A. PENNA -  · PDF fileSTUDENT’S SOLUTIONS MANUAL JUDITH A. PENNA Indiana University Purdue University Indianapolis COLLEGE ALGEBRA: GRAPHS AND MODELS FIFTH EDITION

2 4

2

4

�4 �2

�4

�2

x

y

x � 3y � 3

Exercise Set 2.3 73

We find a third point as a check. We let x = −3 and solvefor y.

−3 − 3y = 3

−3y = 6

y = −2

Another point on the graph is (−3,−2). We plot the pointsand draw the graph.

75. Answers may vary; f(x) =1

x + 7, g(x) =

1x− 3

77. The domain of h(x) is{x∣∣∣x �= 7

3

}, and the domain of g(x)

is {x|x �= 3}, so73

and 3 are not in the domain of (h/g)(x).

We must also exclude the value of x for which g(x) = 0.

x4 − 15x− 15

= 0

x4 − 1 = 0 Multiplying by 5x− 15

x4 = 1

x = ±1

Then the domain of (h/g)(x) is{x∣∣∣x �= 7

3and x �= 3 and x �= −1 and x �= 1

}, or

(−∞,−1) ∪ (−1, 1) ∪(1,

73

)∪(7

3, 3)∪ (3,∞).

Exercise Set 2.3

1. (f ◦ g)(−1) = f(g(−1)) = f((−1)2 − 2(−1) − 6) =

f(1 + 2 − 6) = f(−3) = 3(−3) + 1 = −9 + 1 = −8

3. (h ◦ f)(1) = h(f(1)) = h(3 · 1 + 1) = h(3 + 1) =

h(4) = 43 = 64

5. (g ◦ f)(5) = g(f(5)) = g(3 · 5 + 1) = g(15 + 1) =

g(16) = 162 − 2 · 16 − 6 = 218

7. (f ◦ h)(−3) = f(h(−3)) = f((−3)3) = f(−27) =

3(−27) + 1 = −81 + 1 = −80

9. (g ◦ g)(−2) = g(g(−2)) = g((−2)2 − 2(−2) − 6) =

g(4 + 4 − 6) = g(2) = 22 − 2 · 2 − 6 = 4 − 4 − 6 = −6

11. (h ◦ h)(2) = h(h(2)) = h(23) = h(8) = 83 = 512

13. (f ◦ f)(−4)=f(f(−4))=f(3(−4) + 1)=f(−12 + 1) =

f(−11) = 3(−11) + 1 = −33 + 1 = −32

15. (h ◦ h)(x) = h(h(x)) = h(x3) = (x3)3 = x9

17. (f ◦ g)(x) = f(g(x)) = f(x− 3) = x− 3 + 3 = x

(g ◦ f)(x) = g(f(x)) = g(x + 3) = x + 3 − 3 = x

The domain of f and of g is (−∞,∞), so the domain off ◦ g and of g ◦ f is (−∞,∞).

19. (f ◦ g)(x)=f(g(x))=f(3x2−2x−1)=3x2−2x−1+1=3x2 − 2x(g ◦ f)(x)=g(f(x))=g(x+1)=3(x+1)2−2(x+1)−1=3(x2+2x+1)−2(x+1)−1 = 3x2+6x+3−2x−2−1=3x2 + 4x

The domain of f and of g is (−∞,∞), so the domain off ◦ g and of g ◦ f is (−∞,∞).

21. (f ◦ g)(x)=f(g(x))=f(4x−3)=(4x−3)2−3=16x2 − 24x + 9 − 3 = 16x2 − 24x + 6(g ◦ f)(x)=g(f(x))=g(x2−3)=4(x2−3)−3=4x2 − 12 − 3 = 4x2 − 15

The domain of f and of g is (−∞,∞), so the domain off ◦ g and of g ◦ f is (−∞,∞).

23. (f ◦ g)(x) = f(g(x)) = f( 1x

)=

4

1 − 5 · 1x

=4

1 − 5x

=

4x− 5

x

= 4 · x

x− 5=

4xx− 5

(g ◦ f)(x) = g(f(x)) = g( 4

1 − 5x

)=

14

1 − 5x

=

1 · 1 − 5x4

=1 − 5x

4

The domain of f is{x

∣∣∣∣x �= 15

}and the domain of g is

{x|x �= 0}. Consider the domain of f ◦ g. Since 0 is not in

the domain of g, 0 is not in the domain of f ◦ g. Since15

is not in the domain of f , we know that g(x) cannot be15.

We find the value(s) of x for which g(x) =15.

1x

=15

5 = x Multiplying by 5x

Thus 5 is also not in the domain of f ◦g. Then the domainof f ◦g is {x|x �= 0 and x �= 5}, or (−∞, 0)∪(0, 5)∪(5,∞).

Now consider the domain of g ◦ f . Recall that15

is not inthe domain of f , so it is not in the domain of g ◦ f . Now 0is not in the domain of g but f(x) is never 0, so the domain

of g ◦ f is{x∣∣∣x �= 1

5

}, or

(−∞,

15

)∪(1

5,∞).

25. (f ◦ g)(x) = f(g(x)) = f

(x + 7

3

)=

3(x + 7

3

)− 7 = x + 7 − 7 = x

(g ◦ f)(x) = g(f(x)) = g(3x− 7) =(3x− 7) + 7

3=

3x3

= x

Copyright © 2013 Pearson Education, Inc.

Page 77: JUDITH A. PENNA -  · PDF fileSTUDENT’S SOLUTIONS MANUAL JUDITH A. PENNA Indiana University Purdue University Indianapolis COLLEGE ALGEBRA: GRAPHS AND MODELS FIFTH EDITION

74 Chapter 2: More on Functions

The domain of f and of g is (−∞,∞), so the domain off ◦ g and of g ◦ f is (−∞,∞).

27. (f ◦ g)(x) = f(g(x)) = f(√x) = 2

√x + 1

(g ◦ f)(x) = g(f(x)) = g(2x + 1) =√

2x + 1

The domain of f is (−∞,∞) and the domain of g is{x|x ≥ 0}. Thus the domain of f ◦ g is {x|x ≥ 0}, or[0,∞).

Now consider the domain of g◦f . There are no restrictionson the domain of f , but the domain of g is {x|x ≥ 0}. Since

f(x) ≥ 0 for x ≥ −12, the domain of g ◦ f is

{x∣∣∣x ≥ −1

2

},

or[− 1

2,∞).

29. (f ◦ g)(x) = f(g(x)) = f(0.05) = 20

(g ◦ f)(x) = g(f(x)) = g(20) = 0.05

The domain of f and of g is (−∞,∞), so the domain off ◦ g and of g ◦ f is (−∞,∞).

31. (f ◦ g)(x) = f(g(x)) = f(x2 − 5) =√x2 − 5 + 5 =

√x2 = |x|

(g ◦ f)(x) = g(f(x)) = g(√x + 5) =

(√x + 5)2 − 5 = x + 5 − 5 = x

The domain of f is {x|x ≥ −5} and the domain of g is(−∞,∞). Since x2 ≥ 0 for all values of x, then x2−5 ≥ −5for all values of x and the domain of g ◦ f is (−∞,∞).

Now consider the domain of f ◦g. There are no restrictionson the domain of g, so the domain of f ◦ g is the same asthe domain of f , {x|x ≥ −5}, or [−5,∞).

33. (f ◦ g)(x) = f(g(x)) = f(√

3 − x) = (√

3 − x)2 + 2 =

3 − x + 2 = 5 − x

(g ◦ f)(x) = g(f(x)) = g(x2 + 2) =√

3 − (x2 + 2) =√3 − x2 − 2 =

√1 − x2

The domain of f is (−∞,∞) and the domain of g is{x|x ≤ 3}, so the domain of f ◦ g is {x|x ≤ 3}, or (−∞, 3].

Now consider the domain of g◦f . There are no restrictionson the domain of f and the domain of g is {x|x ≤ 3}, sowe find the values of x for which f(x) ≤ 3. We see thatx2 + 2 ≤ 3 for −1 ≤ x ≤ 1, so the domain of g ◦ f is{x| − 1 ≤ x ≤ 1}, or [−1, 1].

35. (f ◦ g)(x) = f(g(x)) = f

(1

1 + x

)=

1 −(

11 + x

)1

1 + x

=

1 + x− 11 + x

11 + x

=

x

1 + x· 1 + x

1= x

(g ◦ f)(x) = g(f(x)) = g

(1 − x

x

)=

1

1 +(

1 − x

x

) =1

x + 1 − x

x

=

11x

= 1 · x1

= x

The domain of f is {x|x �= 0} and the domain of g is{x|x �= −1}, so we know that −1 is not in the domainof f ◦ g. Since 0 is not in the domain of f , values of xfor which g(x) = 0 are not in the domain of f ◦ g. Butg(x) is never 0, so the domain of f ◦ g is {x|x �= −1}, or(−∞,−1) ∪ (−1,∞).

Now consider the domain of g ◦ f . Recall that 0 is not inthe domain of f . Since −1 is not in the domain of g, weknow that g(x) cannot be −1. We find the value(s) of xfor which f(x) = −1.

1 − x

x= −1

1 − x = −x Multiplying by x

1 = 0 False equation

We see that there are no values of x for which f(x) = −1,so the domain of g ◦ f is {x|x �= 0}, or (−∞, 0) ∪ (0,∞).

37. (f ◦ g)(x) = f(g(x)) = f(x + 1) =

(x + 1)3 − 5(x + 1)2 + 3(x + 1) + 7 =

x3 + 3x2 + 3x + 1 − 5x2 − 10x− 5 + 3x + 3 + 7 =

x3 − 2x2 − 4x + 6

(g ◦ f)(x) = g(f(x)) = g(x3 − 5x2 + 3x + 7) =

x3 − 5x2 + 3x + 7 + 1 = x3 − 5x2 + 3x + 8

The domain of f and of g is (−∞,∞), so the domain off ◦ g and of g ◦ f is (−∞,∞).

39. h(x) = (4 + 3x)5

This is 4 + 3x to the 5th power. The most obvious answeris f(x) = x5 and g(x) = 4 + 3x.

41. h(x) =1

(x− 2)4

This is 1 divided by (x−2) to the 4th power. One obvious

answer is f(x) =1x4

and g(x) = x−2. Another possibility

is f(x) =1x

and g(x) = (x− 2)4.

43. f(x) =x− 1x + 1

, g(x) = x3

45. f(x) = x6, g(x) =2 + x3

2 − x3

47. f(x) =√x, g(x) =

x− 5x + 2

49. f(x) = x3 − 5x2 + 3x− 1, g(x) = x + 2

Copyright © 2013 Pearson Education, Inc.

Page 78: JUDITH A. PENNA -  · PDF fileSTUDENT’S SOLUTIONS MANUAL JUDITH A. PENNA Indiana University Purdue University Indianapolis COLLEGE ALGEBRA: GRAPHS AND MODELS FIFTH EDITION

2 4

2

4

�4 �2

�4

�2

y

x

Chapter 2 Mid-Chapter Mixed Review 75

51. a) Use the distance formula, distance = rate ×time. Substitute 3 for the rate and t for time.

r(t) = 3t

b) Use the formula for the area of a circle.

A(r) = πr2

c) (A ◦ r)(t) = A(r(t)) = A(3t) = π(3t)2 = 9πt2

This function gives the area of the ripple in termsof time t.

53. The manufacturer charges m+2 per drill. The chain storesells each drill for 150%(m+2), or 1.5(m+2), or 1.5m+3.Thus, we have P (m) = 1.5m + 3.

55. Equations (a)− (f) are in the form y = mx+ b, so we canread the y-intercepts directly from the equations. Equa-

tions (g) and (h) can be written in this form as y =23x− 2

and y = −2x + 3, respectively. We see that only equa-tion (c) has y-intercept (0, 1).

57. If a line slopes down from left to right, its slope is negative.The equations y = mx+ b for which m is negative are (b),(d), (f), and (h). (See Exercise 55.)

59. The only equation that has (0, 0) as a solution is (a).

61. Only equations (c) and (g) have the same slope and differ-ent y-intercepts. They represent parallel lines

63. Only the composition (c ◦ p)(a) makes sense. It representsthe cost of the grass seed required to seed a lawn with areaa.

Chapter 2 Mid-Chapter Mixed Review

1. The statement is true. See page 162 in the text.

3. The statement is true. See Example 2 on page 185 in thetext, for instance.

5. From the graph we see that a relative maximum value of6.30 occurs at x = −1.29. We also see that a relativeminimum value of −2.30 occurs at x = 1.29.

The graph starts rising, or increasing, from the left andstops increasing at the relative maximum. From this pointit decreases to the relative minimum and then increasesagain. Thus the function is increasing on (−∞,−1.29)and on (1.29,∞). It is decreasing on (−1.29, 1.29).

7. A(h) =12(h− 2)h

A(h) =12h2 − h

9. g(x) ={

x + 2, for x < −4,−x, for x ≥ −4

We create the graph in two parts. Graph g(x) = x + 2for inputs less than −4. Then graph g(x) = −x for inputsgreater than or equal to −4.

11. (fg)(0) = f(0) · g(0)

= (3 · 0 − 1) · (02 + 4)

= −1 · 4= −4

13. (g/f)(

13

)=

g

(13

)

f

(13

)

=

(13

)2

+ 4

3 · 13− 1

=

19

+ 4

1 − 1

=

3790

Since division by 0 is not defined, (g/f)(

13

)does not exist.

15. f(x) = x− 1, g(x) =√x + 2

a) Any number can be an input for f , so the domainof f is the set of all real numbers, or (−∞,∞).

The domain of g consists of all values for which x+2is nonnegative, so we have x + 2 ≥ 0, or x ≥ −2, or[−2,∞). Then the domain of f + g, f − g, and fgis [−2,∞).The domain of ff is (−∞,∞).

Since g(−2) = 0, the domain of f/g is (−2,∞).

Since f(1) = 0, the domain of g/f is [−2, 1)∪(1,∞).

b) (f + g)(x) = f(x) + g(x) = x− 1 +√x + 2

(f − g)(x) = f(x) − g(x) = x− 1 −√x + 2

(fg)(x) = f(x) · g(x) = (x− 1)√x + 2

(ff)(x) = f(x) · f(x) = (x− 1)(x− 1) =x2 − x− x + 1 = x2 − 2x + 1

(f/g)(x) =f(x)g(x)

=x− 1√x + 2

(g/f)(x) =g(x)f(x)

=√x + 2x− 1

Copyright © 2013 Pearson Education, Inc.

Page 79: JUDITH A. PENNA -  · PDF fileSTUDENT’S SOLUTIONS MANUAL JUDITH A. PENNA Indiana University Purdue University Indianapolis COLLEGE ALGEBRA: GRAPHS AND MODELS FIFTH EDITION

x

y

1—1—3 32 54—2—4—5—1

—2

—3

—4

1

2

3

4

5

—5

y = |x | – 2

x

y

1—1—3 32 54—2—4—5—1

—2

—3

—4

1

2

3

4

5

—5

5y = 4x + 5

76 Chapter 2: More on Functions

17. f(x) = 6 − x2

f(x + h) − f(x)h

=6 − (x + h)2 − (6 − x2)

h=

6−(x2+2xh+h2)−6+x2

h=

6−x2−2xh−h2−6 + x2

h=

−2xh− h2

h=

h/(−2x− h)h/ · 1 = −2x− h

19. (g ◦ h)(2) = g(h(2)) = g(22 − 2 · 2 + 3) = g(4 − 4 + 3) =g(3) = 33 + 1 = 27 + 1 = 28

21. (h ◦ f)(−1) = h(f(−1)) = h(5(−1) − 4) = h(−5 − 4) =h(−9) = (−9)2 − 2(−9) + 3 = 81 + 18 + 3 = 102

23. (f ◦ g)(x) = f(g(x)) = f(√x) = 3

√x + 2

(g ◦ f)(x) = g(f(x)) = g(3x + 2) =√

3x + 2

The domain of f is (−∞,∞) and the domain of g is [0,∞).

Consider the domain of f ◦ g. Since any number can be aninput for f , the domain of f ◦ g is the same as the domainof g, [0,∞).

Now consider the domain of g ◦ f . Since the inputs of g

must be nonnegative, we must have 3x+2 ≥ 0, or x ≥ −23.

Thus the domain of g ◦ f is[− 2

3,∞)

.

25. Under the given conditions, (f + g)(x) and (f/g)(x) havedifferent domains if g(x) = 0 for one or more real numbersx.

27. This approach is not valid. Consider Exercise 23 on page

188 in the text, for example. Since (f ◦ g)(x) =4x

x− 5,

an examination of only this composed function would leadto the incorrect conclusion that the domain of f ◦ g is(−∞, 5)∪ (5,∞). However, we must also exclude from thedomain of f ◦g those values of x that are not in the domainof g. Thus, the domain of f ◦ g is (−∞, 0)∪ (0, 5)∪ (5,∞).

Exercise Set 2.4

1. If the graph were folded on the x-axis, the parts above andbelow the x-axis would not coincide, so the graph is notsymmetric with respect to the x-axis.

If the graph were folded on the y-axis, the parts to theleft and right of the y-axis would coincide, so the graph issymmetric with respect to the y-axis.

If the graph were rotated 180◦, the resulting graph wouldnot coincide with the original graph, so it is not symmetricwith respect to the origin.

3. If the graph were folded on the x-axis, the parts above andbelow the x-axis would coincide, so the graph is symmetricwith respect to the x-axis.

If the graph were folded on the y-axis, the parts to the leftand right of the y-axis would not coincide, so the graph isnot symmetric with respect to the y-axis.

If the graph were rotated 180◦, the resulting graph wouldnot coincide with the original graph, so it is not symmetricwith respect to the origin.

5. If the graph were folded on the x-axis, the parts above andbelow the x-axis would not coincide, so the graph is notsymmetric with respect to the x-axis.

If the graph were folded on the y-axis, the parts to the leftand right of the y-axis would not coincide, so the graph isnot symmetric with respect to the y-axis.

If the graph were rotated 180◦, the resulting graph wouldcoincide with the original graph, so it is symmetric withrespect to the origin.

7.

The graph is symmetric with respect to the y-axis. It isnot symmetric with respect to the x-axis or the origin.

Test algebraically for symmetry with respect to the x-axis:

y = |x| − 2 Original equation

−y = |x| − 2 Replacing y by −y

y = −|x| + 2 Simplifying

The last equation is not equivalent to the original equation,so the graph is not symmetric with respect to the x-axis.

Test algebraically for symmetry with respect to the y-axis:

y = |x| − 2 Original equation

y = | − x| − 2 Replacing x by −x

y = |x| − 2 Simplifying

The last equation is equivalent to the original equation, sothe graph is symmetric with respect to the y-axis.

Test algebraically for symmetry with respect to the origin:

y = |x| − 2 Original equation

−y = | − x| − 2 Replacing x by −x andy by −y

−y = |x| − 2 Simplifying

y = −|x| + 2

The last equation is not equivalent to the original equation,so the graph is not symmetric with respect to the origin.

9.

Copyright © 2013 Pearson Education, Inc.

Page 80: JUDITH A. PENNA -  · PDF fileSTUDENT’S SOLUTIONS MANUAL JUDITH A. PENNA Indiana University Purdue University Indianapolis COLLEGE ALGEBRA: GRAPHS AND MODELS FIFTH EDITION

x

y

1—1—3 32 54—2—4—5—1

—2

—3

—4

1

2

3

4

5

—5

5y = 2x 2 – 3

x

y

1—1—3 32 54—2—4—5—1

—2

—3

—4

1

2

3

4

5

—5

y = —1x

Exercise Set 2.4 77

The graph is not symmetric with respect to the x-axis, they-axis, or the origin.

Test algebraically for symmetry with respect to the x-axis:

5y = 4x + 5 Original equation

5(−y) = 4x + 5 Replacing y by −y

−5y = 4x + 5 Simplifying

5y = −4x− 5

The last equation is not equivalent to the original equation,so the graph is not symmetric with respect to the x-axis.

Test algebraically for symmetry with respect to the y-axis:

5y = 4x + 5 Original equation

5y = 4(−x) + 5 Replacing x by −x

5y = −4x + 5 Simplifying

The last equation is not equivalent to the original equation,so the graph is not symmetric with respect to the y-axis.

Test algebraically for symmetry with respect to the origin:

5y = 4x + 5 Original equation

5(−y) = 4(−x) + 5 Replacing x by −xandy by −y

−5y = −4x + 5 Simplifying

5y = 4x− 5

The last equation is not equivalent to the original equation,so the graph is not symmetric with respect to the origin.

11.

The graph is symmetric with respect to the y-axis. It isnot symmetric with respect to the x-axis or the origin.

Test algebraically for symmetry with respect to the x-axis:

5y = 2x2 − 3 Original equation

5(−y) = 2x2 − 3 Replacing y by −y

−5y = 2x2 − 3 Simplifying

5y = −2x2 + 3

The last equation is not equivalent to the original equation,so the graph is not symmetric with respect to the x-axis.

Test algebraically for symmetry with respect to the y-axis:

5y = 2x2 − 3 Original equation

5y = 2(−x)2 − 3 Replacing x by −x

5y = 2x2 − 3

The last equation is equivalent to the original equation, sothe graph is symmetric with respect to the y-axis.

Test algebraically for symmetry with respect to the origin:

5y = 2x2 − 3 Original equation

5(−y) = 2(−x)2 − 3 Replacing x by −x andy by −y

−5y = 2x2 − 3 Simplifying

5y = −2x2 + 3

The last equation is not equivalent to the original equation,so the graph is not symmetric with respect to the origin.

13.

The graph is not symmetric with respect to the x-axis orthe y-axis. It is symmetric with respect to the origin.

Test algebraically for symmetry with respect to the x-axis:

y =1x

Original equation

−y =1x

Replacing y by −y

y = − 1x

Simplifying

The last equation is not equivalent to the original equation,so the graph is not symmetric with respect to the x-axis.

Test algebraically for symmetry with respect to the y-axis:

y =1x

Original equation

y =1−x

Replacing x by −x

y = − 1x

Simplifying

The last equation is not equivalent to the original equation,so the graph is not symmetric with respect to the y-axis.

Test algebraically for symmetry with respect to the origin:

y =1x

Original equation

−y =1−x

Replacing x by −x and y by −y

y =1x

Simplifying

The last equation is equivalent to the original equation, sothe graph is symmetric with respect to the origin.

15. Test for symmetry with respect to the x-axis:

5x− 5y = 0 Original equation

5x− 5(−y) = 0 Replacing y by −y

5x + 5y = 0 Simplifying

Copyright © 2013 Pearson Education, Inc.

Page 81: JUDITH A. PENNA -  · PDF fileSTUDENT’S SOLUTIONS MANUAL JUDITH A. PENNA Indiana University Purdue University Indianapolis COLLEGE ALGEBRA: GRAPHS AND MODELS FIFTH EDITION

78 Chapter 2: More on Functions

The last equation is not equivalent to the original equation,so the graph is not symmetric with respect to the x-axis.

Test for symmetry with respect to the y-axis:

5x− 5y = 0 Original equation

5(−x) − 5y = 0 Replacing x by −x−5x− 5y = 0 Simplifying

5x+ 5y = 0

The last equation is not equivalent to the original equation,so the graph is not symmetric with respect to the y-axis.

Test for symmetry with respect to the origin:

5x− 5y = 0 Original equation

5(−x) − 5(−y) = 0 Replacing x by −x andy by −y

−5x+ 5y = 0 Simplifying

5x− 5y = 0

The last equation is equivalent to the original equation, sothe graph is symmetric with respect to the origin.

17. Test for symmetry with respect to the x-axis:

3x2 − 2y2 = 3 Original equation

3x2 − 2(−y)2 = 3 Replacing y by −y3x2 − 2y2 = 3 Simplifying

The last equation is equivalent to the original equation, sothe graph is symmetric with respect to the x-axis.

Test for symmetry with respect to the y-axis:

3x2 − 2y2 = 3 Original equation

3(−x)2 − 2y2 = 3 Replacing x by −x3x2 − 2y2 = 3 Simplifying

The last equation is equivalent to the original equation, sothe graph is symmetric with respect to the y-axis.

Test for symmetry with respect to the origin:

3x2 − 2y2 = 3 Original equation

3(−x)2 − 2(−y)2 = 3 Replacing x by −xand y by −y

3x2 − 2y2 = 3 Simplifying

The last equation is equivalent to the original equation, sothe graph is symmetric with respect to the origin.

19. Test for symmetry with respect to the x-axis:

y = |2x| Original equation

−y = |2x| Replacing y by −yy = −|2x| Simplifying

The last equation is not equivalent to the original equation,so the graph is not symmetric with respect to the x-axis.

Test for symmetry with respect to the y-axis:

y = |2x| Original equation

y = |2(−x)| Replacing x by −xy = | − 2x| Simplifying

y = |2x|The last equation is equivalent to the original equation, sothe graph is symmetric with respect to the y-axis.

Test for symmetry with respect to the origin:

y = |2x| Original equation

−y = |2(−x)| Replacing x by −x and y by −y−y = | − 2x| Simplifying

−y = |2x|y = −|2x|

The last equation is not equivalent to the original equation,so the graph is not symmetric with respect to the origin.

21. Test for symmetry with respect to the x-axis:

2x4 + 3 = y2 Original equation

2x4 + 3 = (−y)2 Replacing y by −y2x4 + 3 = y2 Simplifying

The last equation is equivalent to the original equation, sothe graph is symmetric with respect to the x-axis.

Test for symmetry with respect to the y-axis:

2x4 + 3 = y2 Original equation

2(−x)4 + 3 = y2 Replacing x by −x2x4 + 3 = y2 Simplifying

The last equation is equivalent to the original equation, sothe graph is symmetric with respect to the y-axis.

Test for symmetry with respect to the origin:

2x4 + 3 = y2 Original equation

2(−x)4 + 3 = (−y)2 Replacing x by −xand y by −y

2x4 + 3 = y2 Simplifying

The last equation is equivalent to the original equation, sothe graph is symmetric with respect to the origin.

23. Test for symmetry with respect to the x-axis:

3y3 = 4x3 + 2 Original equation

3(−y)3 = 4x3 + 2 Replacing y by −y−3y3 = 4x3 + 2 Simplifying

3y3 = −4x3 − 2

The last equation is not equivalent to the original equation,so the graph is not symmetric with respect to the x-axis.

Test for symmetry with respect to the y-axis:

3y3 = 4x3 + 2 Original equation

3y3 = 4(−x)3 + 2 Replacing x by −x3y3 = −4x3 + 2 Simplifying

The last equation is not equivalent to the original equation,so the graph is not symmetric with respect to the y-axis.

Test for symmetry with respect to the origin:

3y3 = 4x3 + 2 Original equation

3(−y)3 = 4(−x)3 + 2 Replacing x by −xand y by −y

−3y3 = −4x3 + 2 Simplifying

3y3 = 4x3 − 2

The last equation is not equivalent to the original equation,so the graph is not symmetric with respect to the origin.

Copyright © 2013 Pearson Education, Inc.

Page 82: JUDITH A. PENNA -  · PDF fileSTUDENT’S SOLUTIONS MANUAL JUDITH A. PENNA Indiana University Purdue University Indianapolis COLLEGE ALGEBRA: GRAPHS AND MODELS FIFTH EDITION

4

2

�2

�4

42�2�4 x

y

2 4

2

4

�4 �2

�4

�2

x

y

f(x) � (x � 3)2

Exercise Set 2.5 79

25. Test for symmetry with respect to the x-axis:

xy = 12 Original equation

x(−y) = 12 Replacing y by −y−xy = 12 Simplifying

xy = −12

The last equation is not equivalent to the original equation,so the graph is not symmetric with respect to the x-axis.

Test for symmetry with respect to the y-axis:

xy = 12 Original equation

−xy = 12 Replacing x by −xxy = −12 Simplifying

The last equation is not equivalent to the original equation,so the graph is not symmetric with respect to the y-axis.

Test for symmetry with respect to the origin:

xy = 12 Original equation

−x(−y) = 12 Replacing x by −x and y by −yxy = 12 Simplifying

The last equation is equivalent to the original equation, sothe graph is symmetric with respect to the origin.

27. x-axis: Replace y with −y; (−5,−6)

y-axis: Replace x with −x; (5, 6)

Origin: Replace x with −x and y with −y; (5,−6)

29. x-axis: Replace y with −y; (−10, 7)

y-axis: Replace x with −x; (10,−7)

Origin: Replace x with −x and y with −y; (10, 7)

31. x-axis: Replace y with −y; (0, 4)

y-axis: Replace x with −x; (0,−4)

Origin: Replace x with −x and y with −y; (0, 4)

33. The graph is symmetric with respect to the y-axis, so thefunction is even.

35. The graph is symmetric with respect to the origin, so thefunction is odd.

37. The graph is not symmetric with respect to either the y-axis or the origin, so the function is neither even nor odd.

39. f(x) = −3x3 + 2x

f(−x) = −3(−x)3 + 2(−x) = 3x3 − 2x

−f(x) = −(−3x3 + 2x) = 3x3 − 2x

f(−x) = −f(x), so f is odd.

41. f(x) = 5x2 + 2x4 − 1

f(−x) = 5(−x)2 + 2(−x)4 − 1 = 5x2 + 2x4 − 1

f(x) = f(−x), so f is even.

43. f(x) = x17

f(−x) = (−x)17 = −x17

−f(x) = −x17

f(−x) = −f(x), so f is odd.

45. f(x) = x− |x|f(−x) = (−x) − |(−x)| = −x− |x|−f(x) = −(x− |x|) = −x+ |x|f(x) �= f(−x), so f is not even.

f(−x) �= −f(x), so f is not odd.

Thus, f(x) = x− |x| is neither even nor odd.

47. f(x) = 8

f(−x) = 8

f(x) = f(−x), so f is even.

49.

51. f(x) = x√

10 − x2

f(−x) = −x√10 − (−x)2 = −x√10 − x2

−f(x) = −x√10 − x2

Since f(−x) = −f(x), f is odd.

53. If the graph were folded on the x-axis, the parts above andbelow the x-axis would coincide, so the graph is symmetricwith respect to the x-axis.

If the graph were folded on the y-axis, the parts to the leftand right of the y-axis would not coincide, so the graph isnot symmetric with respect to the y-axis.

If the graph were rotated 180◦, the resulting graph wouldnot coincide with the original graph, so it is not symmetricwith respect to the origin.

55. See the answer section in the text.

57. a), b) See the answer section in the text.

59. Let f(x) and g(x) be even functions. Then by definition,f(x) = f(−x) and g(x) = g(−x). Thus, (f + g)(x) =f(x) + g(x) = f(−x) + g(−x) = (f + g)(−x) and f + g iseven. The statement is true.

Exercise Set 2.5

1. Shift the graph of f(x) = x2 right 3 units.

Copyright © 2013 Pearson Education, Inc.

Page 83: JUDITH A. PENNA -  · PDF fileSTUDENT’S SOLUTIONS MANUAL JUDITH A. PENNA Indiana University Purdue University Indianapolis COLLEGE ALGEBRA: GRAPHS AND MODELS FIFTH EDITION

2 4

2

4

�4 �2

�2

x

y

g(x) � x � 3

2 4

2

4

�4 �2

�4

�2

x

y

h(x) � �√x

2 4

2

4

6

8

�4 �2

�2

x

y

h(x) � � � 41x

2 4

2

�4 �2

�4

�2

x

y

h(x) � �3x � 3

2 4

2

4

�4

�4

�2

x

y

h(x) � �|x| � 212

2 4

2

4

�4 �2

�4

�2

x

y

g(x) � �(x � 2)3

2 4

2

4

�4 �2

�4

�2

x

y

g(x) � (x � 1)2 � 1

2 4

2

4

�4

�4

�2

x

y

g(x) � �x3 � 2 13

80 Chapter 2: More on Functions

3. Shift the graph of g(x) = x down 3 units.

5. Reflect the graph of h(x) =√x across the x-axis.

7. Shift the graph of h(x) =1x

up 4 units.

9. First stretch the graph of h(x) = x vertically by multi-plying each y-coordinate by 3. Then reflect it across thex-axis and shift it up 3 units.

11. First shrink the graph of h(x) = |x| vertically by multiply-

ing each y-coordinate by12. Then shift it down 2 units.

13. Shift the graph of g(x) = x3 right 2 units and reflect itacross the x-axis.

15. Shift the graph of g(x) = x2 left 1 unit and down 1 unit.

17. First shrink the graph of g(x) = x3 vertically by multiply-

ing each y-coordinate by13. Then shift it up 2 units.

Copyright © 2013 Pearson Education, Inc.

Page 84: JUDITH A. PENNA -  · PDF fileSTUDENT’S SOLUTIONS MANUAL JUDITH A. PENNA Indiana University Purdue University Indianapolis COLLEGE ALGEBRA: GRAPHS AND MODELS FIFTH EDITION

2 4

2

4

�4 �2

�4

�2

x

y

f(x) � √x � 2

2

2

4

�4 �2

�4

�2

x

y

f(x) � √x � 23

Exercise Set 2.5 81

19. Shift the graph of f(x) =√x left 2 units.

21. Shift the graph of f(x) = 3√x down 2 units.

23. Think of the graph of f(x) = |x|. Sinceg(x) = f(3x), the graph of g(x) = |3x| is the graphof f(x) = |x| shrunk horizontally by dividing each x-

coordinate by 3(or multiplying each x-coordinate by

13

).

25. Think of the graph of f(x) =1x

. Since h(x) = 2f(x),

the graph of h(x) =2x

is the graph of f(x) =1x

stretchedvertically by multiplying each y-coordinate by 2.

27. Think of the graph of g(x) =√x. Since f(x) = 3g(x)− 5,

the graph of f(x) = 3√x − 5 is the graph of g(x) =

√x

stretched vertically by multiplying each y-coordinate by 3and then shifted down 5 units.

29. Think of the graph of f(x) = |x|. Since g(x) =

f

(13x

)− 4, the graph of g(x) =

∣∣∣∣13x∣∣∣∣− 4 is the graph of

f(x) = |x| stretched horizontally by multiplying each x-coordinate by 3 and then shifted down 4 units.

31. Think of the graph of g(x) = x2. Since f(x)=−14g(x− 5),

the graph of f(x) = −14(x− 5)2 is the graph of g(x) = x2

shifted right 5 units, shrunk vertically by multiplying each

y-coordinate by14, and reflected across the x-axis.

33. Think of the graph of g(x) =1x

. Since f(x) =

g(x + 3) + 2, the graph of f(x) =1

x+ 3+ 2 is the graph

of g(x) =1x

shifted left 3 units and up 2 units.

35. Think of the graph of f(x) = x2. Since h(x) = −f(x−3)+5, the graph of h(x) = −(x−3)2 +5 is the graph of f(x) =x2 shifted right 3 units, reflected across the x-axis, andshifted up 5 units.

37. The graph of y = g(x) is the graph of y = f(x) shrunk

vertically by a factor of12. Multiply the y-coordinate by

12: (−12, 2).

39. The graph of y = g(x) is the graph of y = f(x) reflectedacross the y-axis, so we reflect the point across the y-axis:(12, 4).

41. The graph of y = g(x) is the graph of y = f(x) shifteddown 2 units. Subtract 2 from the y-coordinate: (−12, 2).

43. The graph of y = g(x) is the graph of y = f(x) stretchedvertically by a factor of 4. Multiply the y-coordinate by 4:(−12, 16).

45. g(x) = x2 + 4 is the function f(x) = x2 + 3 shifted up1 unit, so g(x) = f(x) + 1. Answer B is correct.

47. If we substitute x − 2 for x in f , we get (x − 2)3 + 3, sog(x) = f(x− 2). Answer A is correct.

49. Shape: h(x) = x2

Turn h(x) upside-down (that is, reflect it across the x-axis): g(x) = −h(x) = −x2

Shift g(x) right 8 units: f(x) = g(x− 8) = −(x− 8)2

51. Shape: h(x) = |x|Shift h(x) left 7 units: g(x) = h(x+ 7) = |x+ 7|Shift g(x) up 2 units: f(x) = g(x) + 2 = |x+ 7| + 2

53. Shape: h(x) =1x

Shrink h(x) vertically by a factor of12

(that is,

multiply each function value by12

):

g(x) =12h(x) =

12· 1x

, or12x

Shift g(x) down 3 units: f(x) = g(x) − 3 =12x

− 3

55. Shape: m(x) = x2

Turn m(x) upside-down (that is, reflect it across the x-axis): h(x) = −m(x) = −x2

Shift h(x) right 3 units: g(x) = h(x− 3) = −(x− 3)2

Shift g(x) up 4 units: f(x) = g(x) + 4 = −(x− 3)2 + 4

57. Shape: m(x) =√x

Reflect m(x) across the y-axis: h(x) = m(−x) =√−x

Shift h(x) left 2 units: g(x) = h(x+ 2) =√−(x+ 2)

Shift g(x) down 1 unit: f(x) = g(x) − 1 =√−(x+ 2) − 1

Copyright © 2013 Pearson Education, Inc.

Page 85: JUDITH A. PENNA -  · PDF fileSTUDENT’S SOLUTIONS MANUAL JUDITH A. PENNA Indiana University Purdue University Indianapolis COLLEGE ALGEBRA: GRAPHS AND MODELS FIFTH EDITION

4

�2

�6

�4

642�2�4 x

y

(�4, 0)

(�3, 4)

(2, �6)

(5, 0)

(�1, 4)

g(x) � �2f(x)

4

2

�2

�4

8642�2�10 �8 �6 �4 x

y

(�10, 0)

(�4, 3)

(2, �2) (6, �2)

(8, 0)

g (x) � f ��qx�

6

2

�2

�4

642�2�4 x

y

(�3, 3)

(�2, 4) (0, 4)

(1, 3) (6, 3)

(3, 1.5)

g (x) � �qf (x � 1) � 3

y

x�4�6 �2 2 4 6

�4

�2

2

4 g(x) � f(�x)

(�5, 0)

(�2, 3)

(3, �2)

(1, �2)

(4, 0)

(�1, 0)

(�7, �3) (�4, �3)

4

y

x�4�6�8�10 �2 2 6

�4

�6

�2

2

4h(x) � �g(x � 2) � 1

(�9, 1) (�2, 1)(�3, 0)

(0, �3)

(5, �5)

(3, 3)

y

x�4 �2 2 4

�4

�2

2

4

6

8

(�1, 4)

52(�, �2)

12(�, 1)

72(�, 6)

(1, 4)

(0, 0)

(��, 0)72

(��, 4)52

(��, 1)12

h(x) � g(2x)

82 Chapter 2: More on Functions

59. Each y-coordinate is multiplied by −2. We plot and con-nect (−4, 0), (−3, 4), (−1, 4), (2,−6), and (5, 0).

61. The graph is reflected across the y-axis and stretched hor-izontally by a factor of 2. That is, each x-coordinate is

multiplied by −2(or divided by −1

2

). We plot and con-

nect (8, 0), (6,−2), (2,−2), (−4, 3), and (−10, 0).

63. The graph is shifted right 1 unit so each x-coordinate isincreased by 1. The graph is also reflected across the x-axis, shrunk vertically by a factor of 2, and shifted up 3

units. Thus, each y-coordinate is multiplied by −12

and

then increased by 3. We plot and connect (−3, 3), (−2, 4),(0, 4), (3, 1.5), and (6, 3).

65. The graph is reflected across the y-axis so eachx-coordinate is replaced by its opposite.

67. The graph is shifted left 2 units so each x-coordinate isdecreased by 2. It is also reflected across the x-axis so eachy-coordinate is replaced with its opposite. In addition, thegraph is shifted up 1 unit, so each y-coordinate is thenincreased by 1.

69. The graph is shrunk horizontally. The x-coordinates ofy = h(x) are one-half the corresponding x-coordinates ofy = g(x).

71. g(x) = f(−x) + 3

The graph of g(x) is the graph of f(x) reflected across they-axis and shifted up 3 units. This is graph (f).

73. g(x) = −f(x) + 3

The graph of g(x) is the graph of f(x) reflected across thex-axis and shifted up 3 units. This is graph (f).

75. g(x) =13f(x− 2)

The graph of g(x) is the graph of f(x) shrunk verticallyby a factor of 3

(that is, each y-coordinate is multiplied

by13

)and then shifted right 2 units. This is graph (d).

77. g(x) =13f(x+ 2)

The graph of g(x) is the graph of f(x) shrunk verticallyby a factor of 3

(that is, each y-coordinate is multiplied

by13

)and then shifted left 2 units. This is graph (c).

79. f(−x) = 2(−x)4 − 35(−x)3 + 3(−x) − 5 =2x4 + 35x3 − 3x− 5 = g(x)

81. The graph of f(x) = x3 − 3x2 is shifted up 2 units. Aformula for the transformed function is g(x) = f(x) + 2,or g(x) = x3 − 3x2 + 2.

83. The graph of f(x) = x3 − 3x2 is shifted left 1 unit. Aformula for the transformed function is k(x) = f(x + 1),or k(x) = (x+ 1)3 − 3(x+ 1)2.

85. Test for symmetry with respect to the x-axis.

y = 3x4 − 3 Original equation

−y = 3x4 − 3 Replacing y by −yy = −3x4 + 3 Simplifying

Copyright © 2013 Pearson Education, Inc.

Page 86: JUDITH A. PENNA -  · PDF fileSTUDENT’S SOLUTIONS MANUAL JUDITH A. PENNA Indiana University Purdue University Indianapolis COLLEGE ALGEBRA: GRAPHS AND MODELS FIFTH EDITION

y

x�4 �2 2 4

�4

�2

2

4

| f(x)|

(�1, 2)

(�4, 0)

(1, 2)(4, 2)

y

x�4 �2 2 4

�4

�2

2

4

g(|x|)

(�2, 1)(�4, 0)

(2, 1)(4, 0)

Exercise Set 2.5 83

The last equation is not equivalent to the original equation,so the graph is not symmetric with respect to the x-axis.

Test for symmetry with respect to the y-axis.

y = 3x4 − 3 Original equation

y = 3(−x)4 − 3 Replacing x by −xy = 3x4 − 3 Simplifying

The last equation is equivalent to the original equation, sothe graph is symmetric with respect to the y-axis.

Test for symmetry with respect to the origin:

y = 3x4 − 3

−y = 3(−x)4 − 3 Replacing x by −x andy by −y

−y = 3x4 − 3

y = −3x4 + 3 Simplifying

The last equation is not equivalent to the original equation,so the graph is not symmetric with respect to the origin.

87. Test for symmetry with respect to the x-axis:

2x− 5y = 0 Original equation

2x− 5(−y) = 0 Replacing y by −y2x+ 5y = 0 Simplifying

The last equation is not equivalent to the original equation,so the graph is not symmetric with respect to the x-axis.

Test for symmetry with respect to the y-axis:

2x− 5y = 0 Original equation

2(−x) − 5y = 0 Replacing x by −x−2x− 5y = 0 Simplifying

The last equation is not equivalent to the original equation,so the graph is not symmetric with respect to the y-axis.

Test for symmetry with respect to the origin:

2x− 5y = 0 Original equation

2(−x) − 5(−y) = 0 Replacing x by −x andy by −y

−2x+ 5y = 0

2x− 5y = 0 Simplifying

The last equation is equivalent to the original equation, sothe graph is symmetric with respect to the origin.

89. Familiarize. Let g = the total amount spent on gift cards,in billions of dollars.

Translate.$5 billion︸ ︷︷ ︸ is 6% of total amount spent︸ ︷︷ ︸� � � � �

5 = 0.06 · g

Carry out. We solve the equation.

5 = 0.06 · g5

0.06= g

83.3 ≈ g

Check. 6% of $83.3 billion is 0.06($83.3 billion) =$4.998 billion ≈ $5 billion. (Remember that we roundedthe value of g.) The answer checks.

State. About $83.3 billion was spent on gift cards.

91. Each point for which f(x) < 0 is reflected across the x-axis.

93. The graph of y = g(|x|) consists of the points of y = g(x)for which x ≥ 0 along with their reflections across they-axis.

95. Think of the graph of g(x) = int(x). Since

f(x) = g

(x− 1

2

), the graph of f(x) = int

(x− 1

2

)is the

graph of g(x) = int(x) shifted right12

unit. The domainis the set of all real numbers; the range is the set of allintegers.

97. On the graph of y = 2f(x) each y-coordinate of y = f(x) ismultiplied by 2, so (3, 4 · 2), or (3, 8) is on the transformedgraph.

On the graph of y = 2+f(x), each y-coordinate of y = f(x)is increased by 2 (shifted up 2 units), so (3, 4+2), or (3, 6)is on the transformed graph.

On the graph of y = f(2x), each x-coordinate of y =

f(x) is multiplied by12

(or divided by 2), so(

12· 3, 4

), or(

32, 4)

is on the transformed graph.

Copyright © 2013 Pearson Education, Inc.

Page 87: JUDITH A. PENNA -  · PDF fileSTUDENT’S SOLUTIONS MANUAL JUDITH A. PENNA Indiana University Purdue University Indianapolis COLLEGE ALGEBRA: GRAPHS AND MODELS FIFTH EDITION

84 Chapter 2: More on Functions

Exercise Set 2.6

1. y = kx

54 = k · 125412

= k, or k =92

The variation constant is92, or 4.5. The equation of vari-

ation is y =92x, or y = 4.5x.

3. y =k

x

3 =k

1236 = k

The variation constant is 36. The equation of variation is

y =36x

.

5. y = kx

1 = k · 14

4 = k

The variation constant is 4. The equation of variation isy = 4x.

7. y =k

x

32 =k18

18· 32 = k

4 = k

The variation constant is 4. The equation of variation is

y =4x

.

9. y = kx

34

= k · 212· 34

= k

38

= k

The variation constant is38. The equation of variation is

y =38x.

11. y =k

x

1.8 =k

0.30.54 = k

The variation constant is 0.54. The equation of variation

is y =0.54x

.

13. Let S = the sales tax and p = the purchase price.

S = kp S varies directly as p.

17.50 = k · 260 Substituting

0.067 ≈ k Variation constant

S = 0.067p Equation of variation

S = 0.067(21) Substituting

S ≈ 1.41

The sales tax is $1.41.

15. W =k

LW varies inversely as L.

1200 =k

8Substituting

9600 = k Variation constant

W =9600L

Equation of variation

W =960014

Substituting

W ≈ 686

A 14-m beam can support about 686 kg.

17. Let F = the number of grams of fat and w = the weight.

F = kw F varies directly as w.

60 = k · 120 Substituting60120

= k, or Solving for k

12

= k Variation constant

F =12w Equation of variation

F =12· 180 Substituting

F = 90

The maximum daily fat intake for a person weighing 180lb is 90 g.

19. T =k

PT varies inversely as P .

5 =k

7Substituting

35 = k Variation constant

T =35P

Equation of variation

T =3510

Substituting

T = 3.5

It will take 10 bricklayers 3.5 hr to complete the job.

Copyright © 2013 Pearson Education, Inc.

Page 88: JUDITH A. PENNA -  · PDF fileSTUDENT’S SOLUTIONS MANUAL JUDITH A. PENNA Indiana University Purdue University Indianapolis COLLEGE ALGEBRA: GRAPHS AND MODELS FIFTH EDITION

Exercise Set 2.6 85

21. d = km d varies directly as m.

40 = k · 3 Substituting403

= k Variation constant

d =403m Equation of variation

d =403

· 5 =2003

Substituting

d = 6623

A 5-kg mass will stretch the spring 6623

cm.

23. P =k

WP varies inversely as W .

330 =k

3.2Substituting

1056 = k Variation constant

P =1056W

Equation of variation

550 =1056W

Substituting

550W = 1056 Multiplying by W

W =1056550

Dividing by 550

W = 1.92 Simplifying

A tone with a pitch of 550 vibrations per second has awavelength of 1.92 ft.

25. y =k

x2

0.15 =k

(0.1)2Substituting

0.15 =k

0.010.15(0.01) = k

0.0015 = k

The equation of variation is y =0.0015x2

.

27. y = kx2

0.15 = k(0.1)2 Substituting

0.15 = 0.01k0.150.01

= k

15 = k

The equation of variation is y = 15x2.

29. y = kxz

56 = k · 7 · 8 Substituting

56 = 56k

1 = k

The equation of variation is y = xz.

31. y = kxz2

105 = k · 14 · 52 Substituting

105 = 350k105350

= k

310

= k

The equation of variation is y =310xz2.

33. y = kxz

wp

328

= k3 · 107 · 8 Substituting

328

= k · 3056

328

· 5630

= k

15

= k

The equation of variation is y =15xz

wp, or

xz

5wp.

35. I =k

d2

90 =k

52Substituting

90 =k

252250 = k

The equation of variation is I =2250d2

.

Substitute 40 for I and find d.

40 =2250d2

40d2 = 2250

d2 = 56.25

d = 7.5

The distance from 5 m to 7.5 m is 7.5−5, or 2.5 m, so it is2.5 m further to a point where the intensity is 40 W/m2.

37. d = kr2

200 = k · 602 Substituting

200 = 3600k2003600

= k

118

= k

The equation of variation is d =118r2.

Substitute 72 for d and find r.

72 =118r2

1296 = r2

36 = r

A car can travel 36 mph and still stop in 72 ft.

Copyright © 2013 Pearson Education, Inc.

Page 89: JUDITH A. PENNA -  · PDF fileSTUDENT’S SOLUTIONS MANUAL JUDITH A. PENNA Indiana University Purdue University Indianapolis COLLEGE ALGEBRA: GRAPHS AND MODELS FIFTH EDITION

x

y

1–1–3 32 54–2–4–5–1

–2

–3

–4

1

2

3

4

5

–5

f (x ) = x 2 – 1

86 Chapter 2: More on Functions

39. E =kR

IWe first find k.

3.89 =k · 93215.2

Substituting

3.89

(215.293

)= k Multiplying by

215.293

9 ≈ k

The equation of variation is E =9RI

.

Substitute 3.89 for E and 238 for I and solve for R.

3.89 =9R238

3.89(238)9

= R Multiplying by2389

103 ≈ R

Bronson Arroyo would have given up about 103 earnedruns if he had pitched 238 innings.

41. parallel

43. relative minimum

45. inverse variation

47. Let V represent the volume and p represent the price of ajar of peanut butter.

V = kp V varies directly as p.

π

(32

)2

(5) = k(2.89) Substituting

3.89π = k Variation constant

V = 3.89πp Equation of variation

π(1.625)2(5.5) = 3.89πp Substituting

3.73 ≈ p

If cost is directly proportional to volume, the larger jarshould cost $3.73.

Now let W represent the weight and p represent the priceof a jar of peanut butter.

W = kp

18 = k(2.89) Substituting

6.23 ≈ k Variation constant

W = 6.23p Equation of variation

28 = 6.23p Substituting

4.49 ≈ p

If cost is directly proportional to weight, the larger jarshould cost $4.49. (Answers may vary slightly due torounding differences.)

Chapter 2 Review Exercises

1. This statement is true by the definition of the greatestinteger function.

3. The graph of y = f(x− d) is the graph of y = f(x) shiftedright d units, so the statement is true.

5. a) For x-values from −4 to −2, the y-values increasefrom 1 to 4. Thus the function is increasing on theinterval (−4,−2).

b) For x-values from 2 to 5, the y-values decrease from4 to 3. Thus the function is decreasing on the inter-val (2, 5).

c) For x-values from −2 to 2, y is 4. Thus the functionis constant on the interval (−2, 2).

7.

The function is increasing on (0,∞) and decreasing on(−∞, 0). We estimate that the minimum value is −1 atx = 0. There are no maxima.

9.

We find that the function is increasing on (2,∞) and de-creasing on (−∞, 2). The relative minimum is −1 at x = 2.There are no maxima.

11.

We find that the function is increasing on (−∞,−1.155)and on (1.155,∞) and decreasing on (−1.155, 1.155). Therelative maximum is 3.079 at x = −1.155 and the relativeminimum is −3.079 at x = 1.155.

13. If l = the length of the tablecloth, then the width is20 − 2l

2, or 10 − l. We use the formula Area = length ×

width.A(l) = l(10 − l), or

A(l) = 10l − l2

15. a) If the length of the side parallel to the garage isx feet long, then the length of each of the other

two sides is66 − x

2, or 33 − x

2. We use the formula

Area = length × width.

Copyright © 2013 Pearson Education, Inc.

Page 90: JUDITH A. PENNA -  · PDF fileSTUDENT’S SOLUTIONS MANUAL JUDITH A. PENNA Indiana University Purdue University Indianapolis COLLEGE ALGEBRA: GRAPHS AND MODELS FIFTH EDITION

y1 � x �33 � ��x2

0

600

0 80

y

x�4 �2 2 4

�4

�2

2

4

4

2

�4

42�2�4 x

y

Chapter 2 Review Exercises 87

A(x) = x

(33 − x

2

), or

A(x) = 33x− x2

2b) The length of the side parallel to the garage must

be positive and less than 66 ft, so the domain of thefunction is {x|0 < x < 66}, or (0, 66).

c)

d) By observing the graph or using the MAXIMUMfeature, we see that the maximum value of the func-tion occurs when x = 33. When x = 33, then

33 − x

2= 33 − 33

2= 33 − 16.5 = 16.5. Thus the di-

mensions that yield the maximum area are 33 ft by16.5 ft.

17. f(x) =

−x, for x ≤ −4,12x+ 1, for x > −4

We create the graph in two parts. Graph f(x) = −x for

inputs less than or equal to −4. Then graph f(x) =12x+ 1

for inputs greater than −4.

19. f(x) =

x2 − 1x+ 1

, for x �= −1,

3, for x = −1

We create the graph in two parts. Graph f(x) =x2 − 1x+ 1

for all inputs except −1. Then graph f(x) = 3 for x = −1.

21. f(x) = [[x− 3]]

This function could be defined by a piecewise function withan infinite number of statements.

f(x) =

.

.

.−4, for −1 ≤ x < 0,−3, for 0 ≤ x < 1,−2, for 1 ≤ x < 2,−1, for 2 ≤ x < 3,...

23. f(x) =

x2 − 1x+ 1

, for x �= −1,

3, for x = −1

Since −2 �= −1, f(−2) =(−2)2 − 1−2 + 1

=4 − 1−1

=3−1

= −3.

Since x = −1, we have f(−1) = 3.

Since 0 �= −1, f(0) =02 − 10 + 1

=−11

= −1.

Since 4 �= −1, f(4) =42 − 14 + 1

=16 − 1

5=

155

= 3.

25. (fg)(2) = f(2) · g(2)

=√

2 − 2 · (22 − 1)

= 0 · (4 − 1)

= 0

27. f(x) =4x2

, g(x) = 3 − 2x

a) Division by zero is undefined, so the domain of f is{x|x �= 0}, or (−∞, 0) ∪ (0,∞). The domain of g isthe set of all real numbers, or (−∞,∞).

The domain of f + g, f − g and fg is {x|x �= 0},or (−∞, 0) ∪ (0,∞). Since g

(32

)= 0, the domain

of f/g is{x

∣∣∣∣x �= 0 and x �= 32

}, or

(−∞, 0) ∪(

0,32

)∪(

32,∞)

.

b) (f + g)(x) =(

4x2

)+ (3 − 2x) =

4x2

+ 3 − 2x

(f − g)(x) =(

4x2

)− (3 − 2x) =

4x2

− 3 + 2x

Copyright © 2013 Pearson Education, Inc.

Page 91: JUDITH A. PENNA -  · PDF fileSTUDENT’S SOLUTIONS MANUAL JUDITH A. PENNA Indiana University Purdue University Indianapolis COLLEGE ALGEBRA: GRAPHS AND MODELS FIFTH EDITION

x

y

1-1-3 32 54-2-4-5-1

-2

-3

-4

1

2

3

4

5

-5

x 2 + y 2 = 4

x

y

1–1–3 32 54–2–4–5–1

–2

–3

–4

1

2

3

4

5

–5

x + y = 3

88 Chapter 2: More on Functions

(fg)(x) =(

4x2

)(3 − 2x) =

12x2

− 8x

(f/g)(x) =

(4x2

)(3 − 2x)

=4

x2(3 − 2x)

29. P (x) = R(x) − C(x)

= (120x− 0.5x2) − (15x+ 6)

= 120x− 0.5x2 − 15x− 6

= −0.5x2 + 105x− 6

31. f(x) = 3 − x2

f(x+ h) = 3 − (x+ h)2 = 3 − (x2 + 2xh+ h2) =

3 − x2 − 2xh− h2

f(x+ h) − f(x)h

=3 − x2 − 2xh− h2 − (3 − x2)

h

=3 − x2 − 2xh− h2 − 3 + x2

h

=−2xh− h2

h=h(−2x− h)

h

=h

h· −2x− h

1= −2x− h

33. (f ◦ g)(1)=f(g(1))=f(12 + 4)=f(1 + 4)=f(5)=

2 · 5 − 1 = 10 − 1 = 9

35. (h ◦ f)(−2) = h(f(−2)) = h(2(−2) − 1) =

h(−4 − 1) = h(−5) = 3 − (−5)3 = 3 − (−125) =

3 + 125 = 128

37. (f ◦ h)(−1) = f(h(−1)) = f(3 − (−1)3) =

f(3 − (−1)) = f(3 + 1) = f(4) = 2 · 4 − 1 = 8 − 1 = 7

39. (f ◦ f)(x) = f(f(x)) = f(2x− 1) = 2(2x− 1) − 1 =4x− 2 − 1 = 4x− 3

41. a) f ◦ g(x) = f(3 − 2x) =4

(3 − 2x)2

g ◦ f(x) = g

(4x2

)= 3 − 2

(4x2

)= 3 − 8

x2

b) The domain of f is {x|x �= 0} and the domain of gis the set of all real numbers. To find the domainof f ◦ g, we find the values of x for which g(x) = 0.

Since 3 − 2x = 0 when x =32, the domain of f ◦ g

is{x

∣∣∣∣x �= 32

}, or

(−∞,

32

)∪(

32,∞)

. Since any

real number can be an input for g, the domain ofg ◦ f is the same as the domain of f , {x|x �= 0}, or(−∞, 0) ∪ (0,∞).

43. f(x) =√x, g(x) = 5x+ 2. Answers may vary.

45. x2 + y2 = 4

The graph is symmetric with respect to the x-axis, they-axis, and the origin.

Replace y with −y to test algebraically for symmetry withrespect to the x-axis.

x2 + (−y)2 = 4

x2 + y2 = 4

The resulting equation is equivalent to the original equa-tion, so the graph is symmetric with respect to the x-axis.

Replace x with −x to test algebraically for symmetry withrespect to the y-axis.

(−x)2 + y2 = 4

x2 + y2 = 4

The resulting equation is equivalent to the original equa-tion, so the graph is symmetric with respect to the y-axis.

Replace x and −x and y with −y to test for symmetrywith respect to the origin.

(−x)2 + (−y)2 = 4

x2 + y2 = 4

The resulting equation is equivalent to the original equa-tion, so the graph is symmetric with respect to the origin.

47. x+ y = 3

The graph is not symmetric with respect to the x-axis, they-axis, or the origin.

Replace y with −y to test algebraically for symmetry withrespect to the x-axis.

x− y = 3

The resulting equation is not equivalent to the originalequation, so the graph is not symmetric with respect tothe x-axis.

Copyright © 2013 Pearson Education, Inc.

Page 92: JUDITH A. PENNA -  · PDF fileSTUDENT’S SOLUTIONS MANUAL JUDITH A. PENNA Indiana University Purdue University Indianapolis COLLEGE ALGEBRA: GRAPHS AND MODELS FIFTH EDITION

x

y

1–1–3 32 54–2–4–5–1

–2

–3

–4

1

2

3

4

5

–5

y = x 3

Chapter 2 Review Exercises 89

Replace x with −x to test algebraically for symmetry withrespect to the y-axis.

−x+ y = 3

The resulting equation is not equivalent to the originalequation, so the graph is not symmetric with respect tothe y-axis.

Replace x and −x and y with −y to test for symmetrywith respect to the origin.

−x− y = 3

x+ y = −3

The resulting equation is not equivalent to the originalequation, so the graph is not symmetric with respect tothe origin.

49. y = x3

The graph is symmetric with respect to the origin. It isnot symmetric with respect to the x-axis or the y-axis.

Replace y with −y to test algebraically for symmetry withrespect to the x-axis.

−y = x3

y = −x3

The resulting equation is not equivalent to the originalequation, so the graph is not symmetric with respect tothe x-axis.

Replace x with −x to test algebraically for symmetry withrespect to the y-axis.

y = (−x)3

y = −x3

The resulting equation is not equivalent to the originalequation, so the graph is not symmetric with respect tothe y-axis.

Replace x and −x and y with −y to test for symmetrywith respect to the origin.

−y = (−x)3

−y = −x3

y = x3

The resulting equation is equivalent to the original equa-tion, so the graph is symmetric with respect to the origin.

51. The graph is symmetric with respect to the y-axis, so thefunction is even.

53. The graph is symmetric with respect to the origin, so thefunction is odd.

55. f(x) = 9 − x2

f(−x) = 9 − (−x2) = 9 − x2

f(x) = f(−x), so f is even.

57. f(x) = x7 − x5

f(−x) = (−x)7 − (−x)5 = −x7 + x5

f(x) �= f(−x), so f is not even.

−f(x) = −(x7 − x5) = −x7 + x5

f(−x) = −f(x), so f is odd.

59. f(x) =√

16 − x2

f(−x) =√

16 − (−x2) =√

16 − x2

f(x) = f(−x), so f is even.

61. Shape: g(x) = x2

Shift g(x) left 3 units: f(x) = g(x+ 3) = (x+ 3)2

63. Shape: h(x) = |x|Stretch h(x) vertically by a factor of 2 (that is, multiplyeach function value by 2): g(x) = 2h(x) = 2|x|.Shift g(x) right 3 units: f(x) = g(x− 3) = 2|x− 3|.

65. The graph is shrunk horizontally by a factor of 2. Thatis, each x-coordinate is divided by 2. We plot and connect(− 5

2, 3)

,(− 3

2, 0)

, (0, 1) and (2,−2).

67. Each y-coordinate is increased by 3. We plot and connect(−5, 6), (−3, 3), (0, 4) and (4, 1).

69. y = kx

6 = 9x23

= x Variation constant

Equation of variation: y =23x

Copyright © 2013 Pearson Education, Inc.

Page 93: JUDITH A. PENNA -  · PDF fileSTUDENT’S SOLUTIONS MANUAL JUDITH A. PENNA Indiana University Purdue University Indianapolis COLLEGE ALGEBRA: GRAPHS AND MODELS FIFTH EDITION

x

y

1–1–3 32 54–2–4–5–1

–2

–3

–4

1

2

3

4

5

–5

f (x ) = 2 – x 2

90 Chapter 2: More on Functions

71.y =

k

x

6 =k

954 = k Variation constant

Equation of variation: y =54x

73. y =kxz2

w

2 =k(16)

(12

)2

0.2

2 =k(16)

(14

)0.2

2 =4k0.2

2 = 20k110

= k

y =110

xz2

w

75. N = ka

87 = k · 29

3 = k

N = 3a

N = 3 · 25

N = 75

Ellen’s score would have been 75 if she had answered 25questions correctly.

77. f(x) = x+ 1, g(x) =√x

The domain of f is (−∞,∞), and the domain of g is [0,∞).To find the domain of (g ◦ f)(x), we find the values of xfor which f(x) ≥ 0.

x+ 1 ≥ 0

x ≥ −1

Thus the domain of (g ◦ f)(x) is [−1,∞). Answer A iscorrect.

79. The graph of g(x) = −12f(x) + 1 is the graph of y = f(x)

shrunk vertically by a factor of12, then reflected across the

x-axis, and shifted up 1 unit. The correct graph is B.

81. Reflect the graph of y = f(x) across the x-axis and thenacross the y-axis.

83. In the graph of y = f(cx), the constant c stretches orshrinks the graph of y = f(x) horizontally. The constantc in y = cf(x) stretches or shrinks the graph of y = f(x)vertically. For y = f(cx), the x-coordinates of y = f(x) aredivided by c; for y = cf(x), the y-coordinates of y = f(x)are multiplied by c.

85. If all of the exponents are even numbers, then f(x) is aneven function. If a0 = 0 and all of the exponents are oddnumbers, then f(x) is an odd function.

87. Let y = k1x and x =k2

z. Then y = k1 · k2

z, or y =

k1k2

z,

so y varies inversely as z.

Chapter 2 Test

1. a) For x-values from −5 to −2, the y-values increasefrom −4 to 3. Thus the function is increasing onthe interval (−5,−2).

b) For x-values from 2 to 5, the y-values decrease from2 to −1. Thus the function is decreasing on theinterval (2, 5).

c) For x-values from −2 to 2, y is 2. Thus the functionis constant on the interval (−2, 2).

2.

The function is increasing on (−∞, 0) and decreasing on(0,∞). The relative maximum is 2 at x = 0. There are nominima.

3.

We find that the function is increasing on (−∞,−2.667)and on (0,∞) and decreasing on (−2.667, 0). The relativemaximum is 9.481 at −2.667 and the relative minimum is0 at x = 0.

4. If b = the length of the base, in inches, then the height =4b − 6. We use the formula for the area of a triangle,

A =12bh.

A(b) =12b(4b− 6), or

A(b) = 2b2 − 3b

Copyright © 2013 Pearson Education, Inc.

Page 94: JUDITH A. PENNA -  · PDF fileSTUDENT’S SOLUTIONS MANUAL JUDITH A. PENNA Indiana University Purdue University Indianapolis COLLEGE ALGEBRA: GRAPHS AND MODELS FIFTH EDITION

y

x

2

4

�2

�4

�2�4 42

Chapter 2 Test 91

5. f(x) =

x2, for x < −1,|x|, for −1 ≤ x ≤ 1,√x− 1, for x > 1

6. Since −1 ≤ −78≤ 1, f

(− 7

8

)=∣∣∣∣− 7

8

∣∣∣∣ = 78.

Since 5 > 1, f(5) =√

5 − 1 =√

4 = 2.

Since −4 < −1, f(−4) = (−4)2 = 16.

7. (f + g)(−6) = f(−6) + g(−6) =

(−6)2 − 4(−6) + 3 +√

3 − (−6) =

36 + 24 + 3 +√

3 + 6 = 63 +√

9 = 63 + 3 = 66

8. (f − g)(−1) = f(−1) − g(−1) =

(−1)2 − 4(−1) + 3 −√3 − (−1) =

1 + 4 + 3 −√3 + 1 = 8 −√

4 = 8 − 2 = 6

9. (fg)(2) = f(2) · g(2) = (22 − 4 · 2 + 3)(√

3 − 2) =

(4 − 8 + 3)(√

1) = −1 · 1 = −1

10. (f/g)(1) =f(1)g(1)

=12 − 4 · 1 + 3√

3 − 1=

1 − 4 + 3√2

=0√2

= 0

11. Any real number can be an input for f(x) = x2, so thedomain is the set of real numbers, or (−∞,∞).

12. The domain of g(x) =√x− 3 is the set of real numbers for

which x− 3 ≥ 0, or x ≥ 3. Thus the domain is {x|x ≥ 3},or [3,∞).

13. The domain of f + g is the intersection of the domains off and g. This is {x|x ≥ 3}, or [3,∞).

14. The domain of f − g is the intersection of the domains off and g. This is {x|x ≥ 3}, or [3,∞).

15. The domain of fg is the intersection of the domains of fand g. This is {x|x ≥ 3}, or [3,∞).

16. The domain of f/g is the intersection of the domains of fand g, excluding those x-values for which g(x) = 0. Sincex− 3 = 0 when x = 3, the domain is (3,∞).

17. (f + g)(x) = f(x) + g(x) = x2 +√x− 3

18. (f − g)(x) = f(x) − g(x) = x2 −√x− 3

19. (fg)(x) = f(x) · g(x) = x2√x− 3

20. (f/g)(x) =f(x)g(x)

=x2

√x− 3

21. f(x) =12x+ 4

f(x+ h) =12(x+ h) + 4 =

12x+

12h+ 4

f(x+ h) − f(x)h

=

12x+

12h+ 4 −

(12x+ 4

)h

=

12x+

12h+ 4 − 1

2x− 4

h

=

12h

h=

12h · 1

h=

12· hh

=12

22. f(x) = 2x2 − x+ 3

f(x+h)=2(x+h)2−(x+h)+3=2(x2+2xh+h2)−x−h+3=

2x2 + 4xh+ 2h2 − x− h+ 3

f(x+h)−f(x)h

=2x2+4xh+2h2−x−h+ 3−(2x2−x+ 3)

h

=2x2+4xh+2h2−x−h+3−2x2+x−3

h

=4xh+ 2h2 − h

h

=h/(4x+ 2h− 1)

h/= 4x+ 2h− 1

23. (g ◦ h)(2) = g(h(2)) = g(3 · 22 + 2 · 2 + 4) =

g(3 · 4 + 4 + 4) = g(12 + 4 + 4) = g(20) = 4 · 20 + 3 =

80 + 3 = 83

24. (f ◦ g)(−1) = f(g(−1)) = f(4(−1) + 3) = f(−4 + 3) =

f(−1) = (−1)2 − 1 = 1 − 1 = 0

25. (h ◦ f)(1) = h(f(1)) = h(12 − 1) = h(1 − 1) = h(0) =

3 · 02 + 2 · 0 + 4 = 0 + 0 + 4 = 4

26. (g ◦ g)(x) = g(g(x)) = g(4x+ 3) = 4(4x+ 3) + 3 =

16x+ 12 + 3 = 16x+ 15

27. (f ◦ g)(x) = f(g(x)) = f(x2 + 1) =√x2 + 1 − 5 =√

x2 − 4

(g ◦ f)(x) = g(f(x)) = g(√x− 5) = (

√x− 5)2 + 1 =

x− 5 + 1 = x− 4

28. The inputs for f(x) must be such that x−5 ≥ 0, or x ≥ 5.Then for (f ◦g)(x) we must have g(x) ≥ 5, or x2+1 ≥ 5, orx2 ≥ 4. Then the domain of (f ◦g)(x) is (−∞,−2]∪[2,∞).

Since we can substitute any real number for x in g, thedomain of (g ◦ f)(x) is the same as the domain of f(x),[5,∞).

29. Answers may vary. f(x) = x4, g(x) = 2x− 7

Copyright © 2013 Pearson Education, Inc.

Page 95: JUDITH A. PENNA -  · PDF fileSTUDENT’S SOLUTIONS MANUAL JUDITH A. PENNA Indiana University Purdue University Indianapolis COLLEGE ALGEBRA: GRAPHS AND MODELS FIFTH EDITION

92 Chapter 2: More on Functions

30. y = x4 − 2x2

Replace y with −y to test for symmetry with respect tothe x-axis.

−y = x4 − 2x2

y = −x4 + 2x2

The resulting equation is not equivalent to the originalequation, so the graph is not symmetric with respect tothe x-axis.

Replace x with −x to test for symmetry with respect tothe y-axis.

y = (−x)4 − 2(−x)2

y = x4 − 2x2

The resulting equation is equivalent to the original equa-tion, so the graph is symmetric with respect to the y-axis.

Replace x with −x and y with −y to test for symmetrywith respect to the origin.

−y = (−x)4 − 2(−x)2

−y = x4 − 2x2

y = −x4 + 2x2

The resulting equation is not equivalent to the originalequation, so the graph is not symmetric with respect tothe origin.

31. f(x) =2x

x2 + 1

f(−x) =2(−x)

(−x)2 + 1= − 2x

x2 + 1f(x) �= f(−x), so f is not even.

−f(x) = − 2xx2 + 1

f(−x) = −f(x), so f is odd.

32. Shape: h(x) = x2

Shift h(x) right 2 units: g(x) = h(x− 2) = (x− 2)2

Shift g(x) down 1 unit: f(x) = (x− 2)2 − 1

33. Shape: h(x) = x2

Shift h(x) left 2 units: g(x) = h(x+ 2) = (x+ 2)2

Shift g(x) down 3 units: f(x) = (x+ 2)2 − 3

34. Each y-coordinate is multiplied by −12. We plot and con-

nect (−5, 1), (−3,−2), (1, 2) and (4,−1).

35. y =k

x

5 =k

630 = k Variation constant

Equation of variation: y =30x

36. y = kx

60 = k · 12

5 = k Variation constant

Equation of variation: y = 5x

37. y =kxz2

w

100 =k(0.1)(10)2

5100 = 2k

50 = k Variation constant

y =50xz2

wEquation of variation

38. d = kr2

200 = k · 602

118

= k Variation constant

d =118r2 Equation of variation

d =118

· 302

d = 50 ft

39. The graph of g(x) = 2f(x) − 1 is the graph of y = f(x)stretched vertically by a factor of 2 and shifted down 1 unit.The correct graph is C.

40. Each x-coordinate on the graph of y = f(x) is divided by

3 on the graph of y = f(3x). Thus the point(−3

3, 1)

, or

(−1, 1) is on the graph of f(3x).

Copyright © 2013 Pearson Education, Inc.

Page 96: JUDITH A. PENNA -  · PDF fileSTUDENT’S SOLUTIONS MANUAL JUDITH A. PENNA Indiana University Purdue University Indianapolis COLLEGE ALGEBRA: GRAPHS AND MODELS FIFTH EDITION

Chapter 3

Quadratic Functions and Equations;Inequalities

Exercise Set 3.1

1.√−3 =

√−1 · 3 =√−1 · √3 = i

√3, or

√3i

3.√−25 =

√−1 · 25 =√−1 · √25 = i · 5 = 5i

5. −√−33 = −√−1 · 33 = −√−1 · √33 = −i√33, or −√33i

7. −√−81 = −√−1 · 81 = −√−1 · √81 = −i · 9 = −9i

9.√−98 =

√−1 · 98 =√−1 · √98 = i

√49 · 2 =

i · 7√2 = 7i√

2, or 7√

2i

11. (−5 + 3i) + (7 + 8i)

= (−5 + 7) + (3i+ 8i) Collecting the real parts

and the imaginary parts

= 2 + (3 + 8)i

= 2 + 11i

13. (4 − 9i) + (1 − 3i)

= (4 + 1) + (−9i− 3i) Collecting the real parts

and the imaginary parts

= 5 + (−9 − 3)i

= 5 − 12i

15. (12 + 3i) + (−8 + 5i)

= (12 − 8) + (3i+ 5i)

= 4 + 8i

17. (−1 − i) + (−3 − i)= (−1 − 3) + (−i− i)= −4 − 2i

19. (3 +√−16) + (2 +

√−25) = (3 + 4i) + (2 + 5i)

= (3 + 2) + (4i+ 5i)

= 5 + 9i

21. (10 + 7i) − (5 + 3i)

= (10 − 5) + (7i− 3i) The 5 and the 3i areboth being subtracted.

= 5 + 4i

23. (13 + 9i) − (8 + 2i)

= (13 − 8) + (9i− 2i) The 8 and the 2i areboth being subtracted.

= 5 + 7i

25. (6 − 4i) − (−5 + i)

= [6 − (−5)] + (−4i− i)= (6 + 5) + (−4i− i)= 11 − 5i

27. (−5 + 2i) − (−4 − 3i)

= [−5 − (−4)] + [2i− (−3i)]

= (−5 + 4) + (2i+ 3i)

= −1 + 5i

29. (4 − 9i) − (2 + 3i)

= (4 − 2) + (−9i− 3i)

= 2 − 12i

31.√−4 · √−36 = 2i · 6i = 12i2 = 12(−1) = −12

33.√−81 · √−25 = 9i · 5i = 45i2 = 45(−1) = −45

35. 7i(2 − 5i)

= 14i− 35i2 Using the distributive law

= 14i+ 35 i2 = −1

= 35 + 14i Writing in the form a+ bi

37. −2i(−8 + 3i)

= 16i− 6i2 Using the distributive law

= 16i+ 6 i2 = −1

= 6 + 16i Writing in the form a+ bi

39. (1 + 3i)(1 − 4i)

= 1 − 4i+ 3i− 12i2 Using FOIL

= 1 − 4i+ 3i− 12(−1) i2 = −1

= 1 − i+ 12

= 13 − i41. (2 + 3i)(2 + 5i)

= 4 + 10i+ 6i+ 15i2 Using FOIL

= 4 + 10i+ 6i− 15 i2 = −1

= −11 + 16i

43. (−4 + i)(3 − 2i)

= −12 + 8i+ 3i− 2i2 Using FOIL

= −12 + 8i+ 3i+ 2 i2 = −1

= −10 + 11i

45. (8 − 3i)(−2 − 5i)

= −16 − 40i+ 6i+ 15i2

= −16 − 40i+ 6i− 15 i2 = −1

= −31 − 34i

Copyright © 2013 Pearson Education, Inc.

Page 97: JUDITH A. PENNA -  · PDF fileSTUDENT’S SOLUTIONS MANUAL JUDITH A. PENNA Indiana University Purdue University Indianapolis COLLEGE ALGEBRA: GRAPHS AND MODELS FIFTH EDITION

94 Chapter 3: Quadratic Functions and Equations; Inequalities

47. (3 +√−16)(2 +

√−25)

= (3 + 4i)(2 + 5i)

= 6 + 15i+ 8i+ 20i2

= 6 + 15i+ 8i− 20 i2 = −1

= −14 + 23i

49. (5 − 4i)(5 + 4i) = 52 − (4i)2

= 25 − 16i2

= 25 + 16 i2 = −1

= 41

51. (3 + 2i)(3 − 2i)

= 9 − 6i+ 6i− 4i2

= 9 − 6i+ 6i+ 4 i2 = −1

= 13

53. (7 − 5i)(7 + 5i)

= 49 + 35i− 35i− 25i2

= 49 + 35i− 35i+ 25 i2 = −1

= 74

55. (4 + 2i)2

= 16 + 2 · 4 · 2i+ (2i)2 Recall (A+B)2 =A2 + 2AB +B2

= 16 + 16i+ 4i2

= 16 + 16i− 4 i2 = −1

= 12 + 16i

57. (−2 + 7i)2

= (−2)2 + 2(−2)(7i) + (7i)2 Recall (A+B)2 =

A2 + 2AB +B2

= 4 − 28i+ 49i2

= 4 − 28i− 49 i2 = −1

= −45 − 28i

59. (1 − 3i)2

= 12 − 2 · 1 · (3i) + (3i)2

= 1 − 6i+ 9i2

= 1 − 6i− 9 i2 = −1

= −8 − 6i

61. (−1 − i)2= (−1)2 − 2(−1)(i) + i2

= 1 + 2i+ i2

= 1 + 2i− 1 i2 = −1

= 2i

63. (3 + 4i)2

= 9 + 2 · 3 · 4i+ (4i)2

= 9 + 24i+ 16i2

= 9 + 24i− 16 i2 = −1

= −7 + 24i

65. 35 − 11i

=3

5 − 11i· 5 + 11i5 + 11i

5 − 11i is the conjugateof 5 + 11i.

=3(5 + 11i)

(5 − 11i)(5 + 11i)

=15 + 33i

25 − 121i2

=15 + 33i25 + 121

i2 = −1

=15 + 33i

146

=15146

+33146

i Writing in the form a+ bi

67. 52 + 3i

=5

2 + 3i· 2 − 3i2 − 3i

2 − 3i is the conjugateof 2 + 3i.

=5(2 − 3i)

(2 + 3i)(2 − 3i)

=10 − 15i4 − 9i2

=10 − 15i

4 + 9i2 = −1

=10 − 15i

13

=1013

− 1513i Writing in the form a+ bi

69. 4 + i−3 − 2i

=4 + i

−3 − 2i· −3 + 2i−3 + 2i

−3 + 2i is the conjugateof the divisor.

=(4 + i)(−3 + 2i)

(−3 − 2i)(−3 + 2i)

=−12 + 5i+ 2i2

9 − 4i2

=−12 + 5i− 2

9 + 4i2 = −1

=−14 + 5i

13

= −1413

+513i Writing in the form a+ bi

Copyright © 2013 Pearson Education, Inc.

Page 98: JUDITH A. PENNA -  · PDF fileSTUDENT’S SOLUTIONS MANUAL JUDITH A. PENNA Indiana University Purdue University Indianapolis COLLEGE ALGEBRA: GRAPHS AND MODELS FIFTH EDITION

Exercise Set 3.1 95

71. 5 − 3i4 + 3i

=5 − 3i4 + 3i

· 4 − 3i4 − 3i

4 − 3i is the conjugateof 4 + 3i.

=(5 − 3i)(4 − 3i)(4 + 3i)(4 − 3i)

=20 − 27i+ 9i2

16 − 9i2

=20 − 27i− 9

16 + 9i2 = −1

=11 − 27i

25

=1125

− 2725i Writing in the form a+ bi

73.2 +

√3i

5 − 4i

=2 +

√3i

5 − 4i· 5 + 4i5 + 4i

5 + 4i is the conjugateof the divisor.

=(2 +

√3i)(5 + 4i)

(5 − 4i)(5 + 4i)

=10 + 8i+ 5

√3i+ 4

√3i2

25 − 16i2

=10 + 8i+ 5

√3i− 4

√3

25 + 16i2 = −1

=10 − 4

√3 + (8 + 5

√3)i

41

=10 − 4

√3

41+

8 + 5√

341

i Writing in theform a+ bi

75. 1 + i(1 − i)2

=1 + i

1 − 2i+ i2

=1 + i

1 − 2i− 1i2 = −1

=1 + i−2i

=1 + i−2i

· 2i2i

2i is the conjugateof −2i.

=(1 + i)(2i)(−2i)(2i)

=2i+ 2i2

−4i2

=2i− 2

4i2 = −1

= −24

+24i

= −12

+12i

77. 4 − 2i1 + i

+2 − 5i1 + i

=6 − 7i1 + i

Adding

=6 − 7i1 + i

· 1 − i1 − i 1 − i is the conjugate

of 1 + i.

=(6 − 7i)(1 − i)(1 + i)(1 − i)

=6 − 13i+ 7i2

1 − i2

=6 − 13i− 7

1 + 1i2 = −1

=−1 − 13i

2

= −12− 13

2i

79. i11 = i10 · i = (i2)5 · i = (−1)5 · i = −1 · i = −i81. i35 = i34 · i = (i2)17 · i = (−1)17 · i = −1 · i = −i83. i64 = (i2)32 = (−1)32 = 1

85. (−i)71 = (−1 · i)71 = (−1)71 · i71 = −i70 · i =

−(i2)35 · i = −(−1)35 · i = −(−1)i = i

87. (5i)4 = 54 · i4 = 625(i2)2 = 625(−1)2 = 625 · 1 = 625

89. First find the slope of the given line.

3x− 6y = 7

−6y = −3x+ 7

y =12x− 7

6

The slope is12. The slope of the desired line is the oppo-

site of the reciprocal of12, or −2. Write a slope-intercept

equation of the line containing (3,−5) with slope −2.

y − (−5) = −2(x− 3)

y + 5 = −2x+ 6

y = −2x+ 1

91. The domain of f is the set of all real numbers as is the

domain of g. When x = −53, g(x) = 0, so the domain of

f/g is(−∞,−5

3

)∪(− 5

3,∞).

93. (f/g)(2) =f(2)g(2)

=22 + 4

3 · 2 + 5=

4 + 46 + 5

=811

95. (a+bi)+(a−bi) = 2a, a real number. Thus, the statementis true.

97. (a+ bi)(c+ di) = (ac− bd) + (ad+ bc)i. The conjugate ofthe product is (ac− bd) − (ad+ bc)i =(a− bi)(c− di), the product of the conjugates of the indi-vidual complex numbers. Thus, the statement is true.

99. zz = (a+ bi)(a− bi) = a2 − b2i2 = a2 + b2

Copyright © 2013 Pearson Education, Inc.

Page 99: JUDITH A. PENNA -  · PDF fileSTUDENT’S SOLUTIONS MANUAL JUDITH A. PENNA Indiana University Purdue University Indianapolis COLLEGE ALGEBRA: GRAPHS AND MODELS FIFTH EDITION

96 Chapter 3: Quadratic Functions and Equations; Inequalities

101. [x− (3 + 4i)][x− (3 − 4i)]

= [x− 3 − 4i][x− 3 + 4i]

= [(x− 3) − 4i][(x− 3) + 4i]

= (x− 3)2 − (4i)2

= x2 − 6x+ 9 − 16i2

= x2 − 6x+ 9 + 16 i2 = −1

= x2 − 6x+ 25

Exercise Set 3.2

1. (2x− 3)(3x− 2) = 0

2x− 3 = 0 or 3x− 2 = 0 Using the principleof zero products

2x = 3 or 3x = 2

x =32or x =

23

The solutions are32

and23.

3. x2 − 8x− 20 = 0

(x− 10)(x+ 2) = 0 Factoring

x− 10 = 0 or x+ 2 = 0 Using the principleof zero products

x = 10 or x = −2

The solutions are 10 and −2.

5. 3x2 + x− 2 = 0

(3x− 2)(x+ 1) = 0 Factoring

3x− 2 = 0 or x+ 1 = 0 Using the principleof zero products

x =23or x = −1

The solutions are23

and −1.

7. 4x2 − 12 = 0

4x2 = 12

x2 = 3

x =√

3 or x = −√3 Using the principle

of square roots

The solutions are√

3 and −√3.

9. 3x2 = 21

x2 = 7

x =√

7 or x = −√7 Using the principle

of square roots

The solutions are√

7 and −√7.

11. 5x2 + 10 = 0

5x2 = −10

x2 = −2

x =√

2i or x = −√2i

The solutions are√

2i and −√2i.

13. x2 + 16 = 0

x2 = −16

x =√−16 or x = −√−16

x = 4i or x = −4iThe solutions are 4i and −4i.

15. 2x2 = 6x

2x2 − 6x = 0 Subtracting 6x on both sides

2x(x− 3) = 0

2x = 0 or x− 3 = 0

x = 0 or x = 3The solutions are 0 and 3.

17. 3y3 − 5y2 − 2y = 0

y(3y2 − 5y − 2) = 0

y(3y + 1)(y − 2) = 0

y = 0 or 3y + 1 = 0 or y − 2 = 0

y = 0 or y = −13or y = 2

The solutions are −13, 0 and 2.

19. 7x3 + x2 − 7x− 1 = 0

x2(7x+ 1) − (7x+ 1) = 0

(x2 − 1)(7x+ 1) = 0

(x+ 1)(x− 1)(7x+ 1) = 0

x+ 1 = 0 or x− 1 = 0 or 7x+ 1 = 0

x = −1 or x = 1 or x = −17

The solutions are −1, −17, and 1.

21. a) The graph crosses the x-axis at (−4, 0) and at (2, 0).These are the x-intercepts.

b) The zeros of the function are the first coordinates ofthe x-intercepts of the graph. They are −4 and 2.

23. a) The graph crosses the x-axis at (−1, 0) and at (3, 0).These are the x-intercepts.

b) The zeros of the function are the first coordinates ofthe x-intercepts of the graph. They are −1 and 3.

25. a) The graph crosses the x-axis at (−2, 0) and at (2, 0).These are the x-intercepts.

b) The zeros of the function are the first coordinates ofthe x-intercepts of the graph. They are −2 and 2.

27. a) The graph has only one x-intercept, (1, 0).

b) The zero of the function is the first coordinate ofthe x-intercept of the graph, 1.

29. x2 + 6x = 7

x2 + 6x+ 9 = 7 + 9 Completing the square:12 · 6 = 3 and 32 = 9

(x+ 3)2 = 16 Factoring

x+ 3 = ±4 Using the principleof square roots

x = −3 ± 4

Copyright © 2013 Pearson Education, Inc.

Page 100: JUDITH A. PENNA -  · PDF fileSTUDENT’S SOLUTIONS MANUAL JUDITH A. PENNA Indiana University Purdue University Indianapolis COLLEGE ALGEBRA: GRAPHS AND MODELS FIFTH EDITION

Exercise Set 3.2 97

x = −3 − 4 or x = −3 + 4

x = −7 or x = 1The solutions are −7 and 1.

31. x2 = 8x− 9

x2 − 8x = −9 Subtracting 8x

x2 − 8x+ 16 = −9 + 16 Completing the square:12 (−8) = −4 and (−4)2 = 16

(x− 4)2 = 7 Factoring

x− 4 = ±√7 Using the principle

of square rootsx = 4 ±√

7

The solutions are 4 −√7 and 4 +

√7, or 4 ±√

7.

33. x2 + 8x+ 25 = 0

x2 + 8x = −25 Subtracting 25

x2 + 8x+ 16 = −25 + 16 Completing thesquare:

12 · 8 = 4 and 42 = 16

(x+ 4)2 = −9 Factoring

x+ 4 = ±3i Using the principleof square roots

x = −4 ± 3iThe solutions are −4 − 3i and −4 + 3i, or −4 ± 3i.

35. 3x2 + 5x− 2 = 0

3x2 + 5x = 2 Adding 2

x2 +53x =

23

Dividing by 3

x2 +53x+

2536

=23

+2536

Completing the

square:12 · 5

3 = 56 and (5

6 )2 = 2536(

x+56

)2

=4936

Factoring andsimplifying

x+56

= ±76

Using the principleof square roots

x = −56± 7

6

x = −56− 7

6or x = −5

6+

76

x = −126

or x =26

x = −2 or x =13

The solutions are −2 and13.

37. x2 − 2x = 15

x2 − 2x− 15 = 0

(x− 5)(x+ 3) = 0 Factoring

x− 5 = 0 or x+ 3 = 0

x = 5 or x = −3The solutions are 5 and −3.

39. 5m2 + 3m = 2

5m2 + 3m− 2 = 0

(5m− 2)(m+ 1) = 0 Factoring

5m− 2 = 0 or m+ 1 = 0

m =25or m = −1

The solutions are25

and −1.

41. 3x2 + 6 = 10x

3x2 − 10x+ 6 = 0

We use the quadratic formula. Here a = 3, b = −10, andc = 6.

x =−b±√

b2 − 4ac2a

=−(−10)±√(−10)2−4·3·6

2 · 3 Substituting

=10 ±√

286

=10 ± 2

√7

6

=2(5 ±√

7)2 · 3 =

5 ±√7

3

The solutions are5 −√

73

and5 +

√7

3, or

5 ±√7

3.

43. x2 + x+ 2 = 0

We use the quadratic formula. Here a = 1, b = 1, andc = 2.

x =−b±√

b2 − 4ac2a

=−1 ±√

12 − 4 · 1 · 22 · 1 Substituting

=−1 ±√−7

2

=−1 ±√

7i2

= −12±

√7

2i

The solutions are −12−

√7

2i and −1

2+

√7

2i, or

−12±

√7

2i.

45. 5t2 − 8t = 3

5t2 − 8t− 3 = 0

We use the quadratic formula. Here a = 5, b = −8, andc = −3.

t =−b±√

b2 − 4ac2a

=−(−8) ±√(−8)2 − 4 · 5(−3)

2 · 5

=8 ±√

12410

=8 ± 2

√31

10

=2(4 ±√

31)2 · 5 =

4 ±√31

5

The solutions are4 −√

315

and4 +

√31

5, or

Copyright © 2013 Pearson Education, Inc.

Page 101: JUDITH A. PENNA -  · PDF fileSTUDENT’S SOLUTIONS MANUAL JUDITH A. PENNA Indiana University Purdue University Indianapolis COLLEGE ALGEBRA: GRAPHS AND MODELS FIFTH EDITION

98 Chapter 3: Quadratic Functions and Equations; Inequalities

4 ±√31

5.

47. 3x2 + 4 = 5x

3x2 − 5x+ 4 = 0We use the quadratic formula. Here a = 3, b = −5, andc = 4.

x =−b±√

b2 − 4ac2a

=−(−5) ±√(−5)2 − 4 · 3 · 4

2 · 3

=5 ±√−23

6=

5 ±√23i

6

=56±

√236i

The solutions are56−

√236i and

56

+√

236i, or

56±

√236i.

49. x2 − 8x+ 5 = 0We use the quadratic formula. Here a = 1, b = −8, andc = 5.

x =−b±√

b2 − 4ac2a

=−(−8) ±√(−8)2 − 4 · 1 · 5

2 · 1

=8 ±√

442

=8 ± 2

√11

2

=2(4 ±√

11)2

= 4 ±√

11

The solutions are 4 −√11 and 4 +

√11, or 4 ±√

11.

51. 3x2 + x = 5

3x2 + x− 5 = 0We use the quadratic formula. We have a = 3, b = 1, andc = −5.

x =−b±√

b2 − 4ac2a

=−1 ±√12 − 4 · 3 · (−5)

2 · 3

=−1 ±√

616

The solutions are−1 −√

616

and−1 +

√61

6, or

−1 ±√61

6.

53. 2x2 + 1 = 5x

2x2 − 5x+ 1 = 0We use the quadratic formula. We have a = 2, b = −5,and c = 1.

x =−b±√

b2 − 4ac2a

=−(−5) ±√(−5)2 − 4 · 2 · 1

2 · 2 =5 ±√

174

The solutions are5 −√

174

and5 +

√17

4, or

5 ±√17

4.

55. 5x2 + 2x = −2

5x2 + 2x+ 2 = 0

We use the quadratic formula. We have a = 5, b = 2, andc = 2.

x =−b±√

b2 − 4ac2a

=−2 ±√

22 − 4 · 5 · 22 · 5

=−2 ±√−36

10=

−2 ± 6i10

=2(−1 ± 3i)

2 · 5 =−1 ± 3i

5

= −15± 3

5i

The solutions are −15− 3

5i and −1

5+

35i, or −1

5± 3

5i.

57. 4x2 = 8x+ 5

4x2 − 8x− 5 = 0

a = 4, b = −8, c = −5

b2 − 4ac = (−8)2 − 4 · 4(−5) = 144

Since b2 − 4ac > 0, there are two different real-numbersolutions.

59. x2 + 3x+ 4 = 0

a = 1, b = 3, c = 4

b2 − 4ac = 32 − 4 · 1 · 4 = −7

Since b2 − 4ac < 0, there are two different imaginary-number solutions.

61. 5t2 − 7t = 0

a = 5, b = −7, c = 0

b2 − 4ac = (−7)2 − 4 · 5 · 0 = 49

Since b2 − 4ac = 49, there are two real-number solutions.

63. Graph y = x2 − 8x+ 12 and use the Zero feature twice.

The solutions are 2 and 6.

Copyright © 2013 Pearson Education, Inc.

Page 102: JUDITH A. PENNA -  · PDF fileSTUDENT’S SOLUTIONS MANUAL JUDITH A. PENNA Indiana University Purdue University Indianapolis COLLEGE ALGEBRA: GRAPHS AND MODELS FIFTH EDITION

Exercise Set 3.2 99

65. Graph y = 7x2 − 43x+ 6 and use the Zero feature twice.

One solution is approximately 0.143 and the other is 6.

67. Graph y1 = 6x + 1 and y2 = 4x2 and use the Intersectfeature twice.

The solutions are approximately −0.151 and 1.651.

69. Graph y = 2x2 − 5x− 4 and use the Zero feature twice.

The zeros are approximately −0.637 and 3.137.

71. x2 + 6x+ 5 = 0 Setting f(x) = 0

(x+ 5)(x+ 1) = 0 Factoring

x+ 5 = 0 or x+ 1 = 0

x = −5 or x = −1

The zeros of the function are −5 and −1.

73. x2 − 3x− 3 = 0

a = 1, b = −3, c = −3

x =−b±√

b2 − 4ac2a

=−(−3) ±√(−3)2 − 4 · 1 · (−3)

2 · 1=

3 ±√9 + 122

=3 ±√

212

The zeros of the function are3 −√

212

and3 +

√21

2, or

3 ±√21

2.

We use a calculator to find decimal approximations for thezeros:

3 +√

212

≈ 3.791 and3 −√

212

≈ −0.791.

75. x2 − 5x+ 1 = 0

a = 1, b = −5, c = 1

x =−b±√

b2 − 4ac2a

=−(−5) ±√(−5)2 − 4 · 1 · 1

2 · 1=

5 ±√25 − 42

=5 ±√

212

The zeros of the function are5 −√

212

and5 +

√21

2, or

5 ±√21

2.

We use a calculator to find decimal approximations for thezeros:

5 +√

212

≈ 4.791 and5 −√

212

≈ 0.209.

77. x2 + 2x− 5 = 0

a = 1, b = 2, c = −5

x =−b±√

b2 − 4ac2a

=−2 ±√22 − 4 · 1 · (−5)

2 · 1=

−2 ±√4 + 20

2=

−2 ±√24

2

=−2 ± 2

√6

2= −1 ±

√6

Copyright © 2013 Pearson Education, Inc.

Page 103: JUDITH A. PENNA -  · PDF fileSTUDENT’S SOLUTIONS MANUAL JUDITH A. PENNA Indiana University Purdue University Indianapolis COLLEGE ALGEBRA: GRAPHS AND MODELS FIFTH EDITION

100 Chapter 3: Quadratic Functions and Equations; Inequalities

The zeros of the function are −1 +√

6 and −1 −√6, or

−1 ±√6.

We use a calculator to find decimal approximations for thezeros:

−1 +√

6 ≈ 1.449 and −1 −√6 ≈ −3.449

79. 2x2 − x+ 4 = 0

a = 2, b = −1, c = 4

x =−b±√

b2 − 4ac2a

=−(−1) ±√(−1)2 − 4 · 2 · 4

2 · 2

=1 ±√−31

4=

1 ±√31i

4

=14±

√314i

The zeros of the function are14−

√314i and

14

+√

314i, or

14±

√314i.

81. 3x2 − x− 1 = 0

a = 3, b = −1, c = −1

x =−b±√

b2 − 4ac2a

=−(−1) ±√(−1)2 − 4 · 3 · (−1)

2 · 3

=1 ±√

136

The zeros of the function are1 −√

136

and1 +

√13

6, or

1 ±√13

6.

We use a calculator to find decimal approximations for thezeros:1 +

√13

6≈ 0.768 and

1 −√13

6≈ −0.434.

83. 5x2 − 2x− 1 = 0

a = 5, b = −2, c = −1

x =−b±√

b2 − 4ac2a

=−(−2) ±√(−2)2 − 4 · 5 · (−1)

2 · 5

=2 ±√

2410

=2 ± 2

√6

10

=2(1 ±√

6)2 · 5 =

1 ±√6

5

The zeros of the function are1 −√

65

and1 +

√6

5, or

1 ±√6

5.

We use a calculator to find decimal approximations for thezeros:

1 +√

65

≈ 0.690 and1 −√

65

≈ −0.290.

85. 4x2 + 3x− 3 = 0

a = 4, b = 3, c = −3

x =−b±√

b2 − 4ac2a

=−3 ±√32 − 4 · 4 · (−3)

2 · 4

=−3 ±√

578

The zeros of the function are−3 −√

578

and−3 +

√57

8,

or−3 ±√

578

.

We use a calculator to find decimal approximations for thezeros:

−3 +√

578

≈ 0.569 and−3 −√

578

≈ −1.319.

87. Graph y = 3x2 + 2x− 4 and use the Zero feature twice.

The zeros are approximately −1.535 and 0.869.

89. Graph y = 5.02x2−4.19x−2.057 and use the Zero featuretwice.

The zeros are approximately −0.347 and 1.181.

Copyright © 2013 Pearson Education, Inc.

Page 104: JUDITH A. PENNA -  · PDF fileSTUDENT’S SOLUTIONS MANUAL JUDITH A. PENNA Indiana University Purdue University Indianapolis COLLEGE ALGEBRA: GRAPHS AND MODELS FIFTH EDITION

Exercise Set 3.2 101

91. x4 − 3x2 + 2 = 0

Let u = x2.u2 − 3u+ 2 = 0 Substituting u for x2

(u− 1)(u− 2) = 0

u− 1 = 0 or u− 2 = 0

u = 1 or u = 2

Now substitute x2 for u and solve for x.x2 = 1 or x2 = 2

x = ±1 or x = ±√2

The solutions are −1, 1, −√2, and

√2.

93. x4 + 3x2 = 10

x4 + 3x2 − 10 = 0

Let u = x2.u2 + 3u− 10 = 0 Substituting u for x2

(u+ 5)(u− 2) = 0

u+ 5 = 0 or u− 2 = 0

u = −5 or u = 2

Now substitute x2 for u and solve for x.x2 = −5 or x2 = 2

x = ±√5i or x = ±√

2

The solutions are −√5i,

√5i, −√

2, and√

2.

95. y4 + 4y2 − 5 = 0

Let u = y2.

u2 + 4u− 5 = 0 Substituting u for y2

(u+ 5)(u− 1) = 0

u+ 5 = 0 or u− 1 = 0

u = −5 or u = 1

Now substitute y2 for u and solve for y.

y2 = −5 or y2 = 1

y = ±√5i or y = ±1

The solutions are −√5i,

√5i, −1, and 1.

97. x− 3√x− 4 = 0

Let u =√x.

u2 − 3u− 4 = 0 Substituting u for√x

(u+ 1)(u− 4) = 0

u+ 1 = 0 or u− 4 = 0

u = −1 or u = 4

Now substitute√x for u and solve for x.√

x = −1 or√x = 4

No solution x = 16

Note that√x must be nonnegative, so

√x = −1 has no

solution. The number 16 checks and is the solution. Thesolution is 16.

99. m2/3 − 2m1/3 − 8 = 0

Let u = m1/3.u2 − 2u− 8 = 0 Substituting u for m1/3

(u+ 2)(u− 4) = 0

u+ 2 = 0 or u− 4 = 0

u = −2 or u = 4

Now substitute m1/3 for u and solve for m.m1/3 = −2 or m1/3 = 4

(m1/3)3 = (−2)3 or (m1/3)3 = 43 Using theprinciple of powers

m = −8 or m = 64

The solutions are −8 and 64.

101. x1/2 − 3x1/4 + 2 = 0

Let u = x1/4.u2 − 3u+ 2 = 0 Substituting u for x1/4

(u− 1)(u− 2) = 0

u− 1 = 0 or u− 2 = 0

u = 1 or u = 2

Now substitute x1/4 for u and solve for x.x1/4 = 1 or x1/4 = 2

(x1/4)4 = 14 or (x1/4)4 = 24

x = 1 or x = 16

The solutions are 1 and 16.

103. (2x− 3)2 − 5(2x− 3) + 6 = 0

Let u = 2x− 3.u2 − 5u+ 6 = 0 Substituting u for 2x− 3

(u− 2)(u− 3) = 0

u− 2 = 0 or u− 3 = 0

u = 2 or u = 3

Now substitute 2x− 3 for u and solve for x.2x− 3 = 2 or 2x− 3 = 3

2x = 5 or 2x = 6

x =52or x = 3

The solutions are52

and 3.

105. (2t2 + t)2 − 4(2t2 + t) + 3 = 0

Let u = 2t2 + t.u2 − 4u+ 3 = 0 Substituting u for 2t2 + t

(u− 1)(u− 3) = 0

u− 1 = 0 or u− 3 = 0

u = 1 or u = 3

Now substitute 2t2 + t for u and solve for t.2t2 + t = 1 or 2t2 + t = 3

2t2 + t− 1 = 0 or 2t2 + t− 3 = 0

(2t− 1)(t+ 1) = 0 or (2t+ 3)(t− 1) = 0

Copyright © 2013 Pearson Education, Inc.

Page 105: JUDITH A. PENNA -  · PDF fileSTUDENT’S SOLUTIONS MANUAL JUDITH A. PENNA Indiana University Purdue University Indianapolis COLLEGE ALGEBRA: GRAPHS AND MODELS FIFTH EDITION

102 Chapter 3: Quadratic Functions and Equations; Inequalities

2t−1=0 or t+1=0 or 2t+3=0 or t−1 = 0

t=12or t=−1 or t=−3

2or t = 1

The solutions are12, −1, −3

2and 1.

107. Substitute 40 for h(x) and solve for x.

40 = 0.012x2 − 0.583x+ 35.727

0 = 0.012x2 − 0.583x− 4.273

a = 0.012, b = −0.583, c = −4.273

x =−b±√

b2 − 4ac2a

=−(−0.583) ±√(−0.583)2 − 4(0.012)(−4.273)

2(0.012)

=0.583 ±√

0.5449930.024

x ≈ −6.5 or x ≈ 55.0

Since we are looking for a year after 1940, we use thepositive solution. There were 40 million multigenerationalhouseholds about 55 yr after 1940, or in 1995.

109. Substitute 130 for t(x) and solve for x.

130 = 0.16x2 + 0.46x+ 21.36

0 = 0.16x2 + 0.46x− 108.64

a = 0.16, b = 0.46, c = −108.64

x =−b±√

b2 − 4ac2a

=−0.46 ±√(0.46)2 − 4(0.16)(−108.64)

2(0.16)

=−0.46 ±√

69.74120.32

x ≈ −28 or x ≈ 25

We use the positive solution because a negative number hasno meaning in this situation. The average U.S. householdreceived 130 TV channels about 25 yr after 1985, or in2010.

111. Familiarize and Translate. We will use the formulas = 16t2, substituting 1670 for s.

1670 = 16t2

Carry out. We solve the equation.

1670 = 16t2

104.375 = t2 Dividing by 16 on both sides

10.216 ≈ t Taking the square root onboth sides

Check. When t = 10.216, s = 16(10.216)2 ≈ 1670. Theanswer checks.

State. It would take an object about 10.216 sec to reachthe ground.

113. Familiarize. Let w = the width of the rug. Then w+1 =the length.

Translate. We use the Pythagorean equation.

w2 + (w + 1)2 = 52

Carry out. We solve the equation.

w2 + (w + 1)2 = 52

w2 + w2 + 2w + 1 = 25

2w2 + 2w + 1 = 25

2w2 + 2w − 24 = 0

2(w + 4)(w − 3) = 0

w + 4 = 0 or w − 3 = 0

w = −4 or w = 3

Since the width cannot be negative, we consider only 3.When w = 3, w + 1 = 3 + 1 = 4.

Check. The length, 4 ft, is 1 ft more than the width, 3 ft.The length of a diagonal of a rectangle with width 3 ft andlength 4 ft is

√32 + 42 =

√9 + 16 =

√25 = 5. The answer

checks.

State. The length is 4 ft, and the width is 3 ft.

115. Familiarize. Let n = the smaller number. Then n+ 5 =the larger number.

Translate.The product of the numbers︸ ︷︷ ︸ is 36.

↓ ↓ ↓n(n+ 5) = 36

Carry out.

n(n+ 5) = 36

n2 + 5n = 36

n2 + 5n− 36 = 0

(n+ 9)(n− 4) = 0

n+ 9 = 0 or n− 4 = 0

n = −9 or n = 4

If n = −9, then n + 5 = −9 + 5 = −4. If n = 4, thenn+ 5 = 4 + 5 = 9.

Check. The number −4 is 5 more than −9 and(−4)(−9) = 36, so the pair −9 and −4 check. The number9 is 5 more than 4 and 9 · 4 = 36, so the pair 4 and 9 alsocheck.

State. The numbers are −9 and −4 or 4 and 9.

117. Familiarize. We add labels to the drawing in the text.

We let x represent the length of a side of the square ineach corner. Then the length and width of the resultingbase are represented by 20− 2x and 10− 2x, respectively.Recall that for a rectangle, Area = length × width.

20 cm

10 cm20 − 2x

10 − 2xx

x

x

x

x

x

x

x

Copyright © 2013 Pearson Education, Inc.

Page 106: JUDITH A. PENNA -  · PDF fileSTUDENT’S SOLUTIONS MANUAL JUDITH A. PENNA Indiana University Purdue University Indianapolis COLLEGE ALGEBRA: GRAPHS AND MODELS FIFTH EDITION

Exercise Set 3.2 103

Translate.The area of the base︸ ︷︷ ︸ is 96 cm2.︸ ︷︷ ︸(20 − 2x)(10 − 2x) = 96

Carry out. We solve the equation.

200 − 60x+ 4x2 = 96

4x2 − 60x+ 104 = 0

x2 − 15x+ 26 = 0

(x− 13)(x− 2) = 0

x− 13 = 0 or x− 2 = 0

x = 13 or x = 2

Check. When x = 13, both 20 − 2x and 10 − 2x arenegative numbers, so we only consider x = 2. When x = 2,then 20− 2x = 20− 2 · 2 = 16 and 10− 2x = 10− 2 · 2 = 6,and the area of the base is 16 · 6, or 96 cm2. The answerchecks.

State. The length of the sides of the squares is 2 cm.

119. Familiarize. We have P = 2l + 2w, or 28 = 2l + 2w.Solving for w, we have

28 = 2l + 2w

14 = l + w Dividing by 2

14 − l = w.

Then we have l = the length of the rug and 14 − l = thewidth, in feet. Recall that the area of a rectangle is theproduct of the length and the width.

Translate.The area︸ ︷︷ ︸ is 48 ft2.︸ ︷︷ ︸� � �l(14 − l) = 48

Carry out. We solve the equation.

l(14 − l) = 48

14l − l2 = 48

0 = l2 − 14l + 48

0 = (l − 6)(l − 8)

l − 6 = 0 or l − 8 = 0

l = 6 or l = 8

If l = 6, then 14 − l = 14 − 6 = 8.

If l = 8, then 14 − l = 14 − 8 = 6.

In either case, the dimensions are 8 ft by 6 ft. Since weusually consider the length to be greater than the width,we let 8 ft = the length and 6 ft = the width.

Check. The perimeter is 2 ·8 ft+2 ·6 ft = 16 ft +12 ft =28 ft. The answer checks.

State. The length of the rug is 8 ft, and the width is 6 ft.

121. f(x) = 4 − 5x = −5x+ 4

The function can be written in the form y = mx+ b, so itis a linear function.

123. f(x) = 7x2

The function is in the form f(x) = ax2 + bx+ c, a �= 0, soit is a quadratic function.

125. f(x) = 1.2x− (3.6)2

The function is in the form f(x) = mx+ b, so it is a linearfunction.

127. In 2010, x = 2010 − 2004 = 6.

a(6) = 1.24(6) + 9.24 = 7.44 + 9.24 = 16.68

In 2010, $16.68 billion was spent on antipsychotic drugs.

129. Test for symmetry with respect to the x-axis:

3x2 + 4y2 = 5 Original equation

3x2 + 4(−y)2 = 5 Replacing y by −y3x2 + 4y2 = 5 Simplifying

The last equation is equivalent to the original equation, sothe graph is symmetric with respect to the x-axis.

Test for symmetry with respect to the y-axis:

3x2 + 4y2 = 5 Original equation

3(−x)2 + 4y2 = 5 Replacing x by −x3x2 + 4y2 = 5 Simplifying

The last equation is equivalent to the original equation, sothe equation is symmetric with respect to the y-axis.

Test for symmetry with respect to the origin:

3x2 + 4y2 = 5 Original equation

3(−x)2 + 4(−y)2 = 5 Replacing x by −xand y by −y

3x2 + 4y2 = 5 Simplifying

The last equation is equivalent to the original equation, sothe equation is symmetric with respect to the origin.

131. f(x) = 2x3 − xf(−x) = 2(−x)3 − (−x) = −2x3 + x

−f(x) = −2x3 + x

f(x) �= f(−x) so f is not even

f(−x) = −f(x), so f is odd.

133. a) kx2 − 17x+ 33 = 0

k(3)2 − 17(3) + 33 = 0 Substituting 3 for x

9k − 51 + 33 = 0

9k = 18

k = 2

b) 2x2 − 17x+ 33 = 0 Substituting 2 for k

(2x− 11)(x− 3) = 0

2x− 11 = 0 or x− 3 = 0

x =112

or x = 3

The other solution is112

.

135. a) (1 + i)2 − k(1 + i) + 2 = 0 Substituting1 + i for x

1 + 2i− 1 − k − ki+ 2 = 0

2 + 2i = k + ki

2(1 + i) = k(1 + i)

2 = k

Copyright © 2013 Pearson Education, Inc.

Page 107: JUDITH A. PENNA -  · PDF fileSTUDENT’S SOLUTIONS MANUAL JUDITH A. PENNA Indiana University Purdue University Indianapolis COLLEGE ALGEBRA: GRAPHS AND MODELS FIFTH EDITION

104 Chapter 3: Quadratic Functions and Equations; Inequalities

b) x2 − 2x+ 2 = 0 Substituting 2 for k

x =−(−2) ±√(−2)2 − 4 · 1 · 2

2 · 1

=2 ±√−4

2

=2 ± 2i

2= 1 ± i

The other solution is 1 − i.137. (x− 2)3 = x3 − 2

x3 − 6x2 + 12x− 8 = x3 − 2

0 = 6x2 − 12x+ 6

0 = 6(x2 − 2x+ 1)

0 = 6(x− 1)(x− 1)

x− 1 = 0 or x− 1 = 0

x = 1 or x = 1

The solution is 1.

139. (6x3 + 7x2 − 3x)(x2 − 7) = 0

x(6x2 + 7x− 3)(x2 − 7) = 0

x(3x− 1)(2x+ 3)(x2 − 7) = 0

x=0 or 3x− 1=0 or 2x+ 3=0 or x2 − 7 = 0

x=0 or x=13or x=−3

2or x =

√7 or

x = −√7

The exact solutions are −√7, −3

2, 0,

13, and

√7.

141. x2 + x−√2 = 0

x =−b±√

b2 − 4ac2a

=−1 ±

√12 − 4 · 1(−√

2)

2 · 1 =−1 ±

√1 + 4

√2

2

The solutions are−1 ±

√1 + 4

√2

2.

143. 2t2 + (t− 4)2 = 5t(t− 4) + 24

2t2 + t2 − 8t+ 16 = 5t2 − 20t+ 24

0 = 2t2 − 12t+ 8

0 = t2 − 6t+ 4 Dividing by 2

Use the quadratic formula.

t =−b±√

b2 − 4ac2a

=−(−6) ±√(−6)2 − 4 · 1 · 4

2 · 1

=6 ±√

202

=6 ± 2

√5

2

=2(3 ±√

5)2

= 3 ±√

5

The solutions are 3 ±√5.

145.√x− 3 − 4

√x− 3 = 2

Substitute u for 4√x− 3.

u2 − u− 2 = 0

(u− 2)(u+ 1) = 0

u− 2 = 0 or u+ 1 = 0

u = 2 or u = −1

Substitute 4√x− 3 for u and solve for x.

4√x− 3 = 2 or 4

√x− 3 = 1

x− 3 = 16 No solution

x = 19

The value checks. The solution is 19.

147.(y +

2y

)2

+ 3y +6y

= 4

(y +

2y

)2

+ 3(y +

2y

)− 4 = 0

Substitute u for y +2y.

u2 + 3u− 4 = 0

(u+ 4)(u− 1) = 0u = −4 or u = 1

Substitute y +2y

for u and solve for y.

y +2y

= −4 or y +2y

= 1

y2 + 2 = −4y or y2 + 2 = y

y2 + 4y + 2 = 0 or y2 − y + 2 = 0

y =−4±√

42−4·1·22 · 1 or

y =−(−1)±√(−1)2−4·1·2

2 · 1

y =−4 ±√

82

or y =1 ±√−7

2

y =−4 ± 2

√2

2or y =

1 ±√7i

2

y = −2 ±√2 or y =

12±

√7

2i

The solutions are −2 ±√

2 and12±

√7

2i.

Exercise Set 3.3

1. a) The minimum function value occurs at the vertex,

so the vertex is(− 1

2,−9

4

).

b) The axis of symmetry is a vertical line through the

vertex. It is x = −12.

c) The minimum value of the function is −94.

Copyright © 2013 Pearson Education, Inc.

Page 108: JUDITH A. PENNA -  · PDF fileSTUDENT’S SOLUTIONS MANUAL JUDITH A. PENNA Indiana University Purdue University Indianapolis COLLEGE ALGEBRA: GRAPHS AND MODELS FIFTH EDITION

x2 4

2

4

6 8�2

�4

�2

y

f (x) � x

2 � 8x � 12

x2 4 6

6

8

2

4

�2

y

f (x) � x

2 � 7x � 12

x2

12

16

4

8

�4�6 �2

y

f (x) � x

2 � 4x � 5

Exercise Set 3.3 105

3. f(x) = x2 − 8x+ 12 16 completes thesquare for x2 − 8x.

= x2 − 8x+ 16 − 16 + 12 Adding 16−16on the right side

= (x2 − 8x+ 16) − 16 + 12

= (x− 4)2 − 4 Factoring andsimplifying

= (x− 4)2 + (−4) Writing in the formf(x) = a(x− h)2 + k

a) Vertex: (4,−4)

b) Axis of symmetry: x = 4

c) Minimum value: −4

d) We plot the vertex and find several points on eitherside of it. Then we plot these points and connectthem with a smooth curve.

x f(x)

4 −4

2 0

1 5

5 −3

6 0

5. f(x) = x2 − 7x+ 12494

completes the

square for x2 − 7x.

= x2 − 7x+494

− 494

+ 12 Adding

494

− 494

on the right side

=(x2 − 7x+

494

)− 49

4+ 12

=(x− 7

2

)2

− 14

Factoring and

simplifying

=(x− 7

2

)2

+(− 1

4

)Writing in the

form f(x) = a(x− h)2 + k

a) Vertex:(

72,−1

4

)

b) Axis of symmetry: x =72

c) Minimum value: −14

d) We plot the vertex and find several points on eitherside of it. Then we plot these points and connectthem with a smooth curve.

x f(x)

72

−14

4 0

5 2

3 0

1 6

7. f(x) = x2 + 4x+ 5 4 completes thesquare for x2 + 4x

= x2 + 4x+ 4 − 4 + 5 Adding 4 − 4on the right side

= (x+ 2)2 + 1 Factoring and simplifying

= [x− (−2)]2 + 1 Writing in the formf(x) = a(x− h)2 + k

a) Vertex: (−2, 1)

b) Axis of symmetry: x = −2

c) Minimum value: 1

d) We plot the vertex and find several points on eitherside of it. Then we plot these points and connectthem with a smooth curve.

x f(x)

−2 1

−1 2

0 5

−3 2

−4 5

9. g(x) =x2

2+ 4x+ 6

=12(x2 + 8x) + 6 Factoring

12

out of the

first two terms

=12(x2+8x+16−16)+6 Adding 16 − 16 inside

the parentheses

=12(x2+8x+16)− 1

2·16+6 Removing −16 from

within the parentheses

=12(x+ 4)2 − 2 Factoring and simplifying

=12[x− (−4)]2 + (−2)

a) Vertex: (−4,−2)

b) Axis of symmetry: x = −4

c) Minimum value: −2

d) We plot the vertex and find several points on eitherside of it. Then we plot these points and connectthem with a smooth curve.

Copyright © 2013 Pearson Education, Inc.

Page 109: JUDITH A. PENNA -  · PDF fileSTUDENT’S SOLUTIONS MANUAL JUDITH A. PENNA Indiana University Purdue University Indianapolis COLLEGE ALGEBRA: GRAPHS AND MODELS FIFTH EDITION

x4 8

4

8

�8 �4

�8

�4

y

g(x) � � � 4x � 6x

2

2

x2 4

12

16

4

8

�4 �2

y

g(x) � 2x

2 � 6x � 8

x4 8

4

�8 �4

�8

�4

y

f (x) � �x

2 � 6x � 3

106 Chapter 3: Quadratic Functions and Equations; Inequalities

x g(x)

−4 −2

−2 0

0 6

−6 0

−8 6

11. g(x) = 2x2 + 6x+ 8

= 2(x2 + 3x) + 8 Factoring 2 out ofthe first two terms

= 2(x2 + 3x+

94− 9

4

)+ 8 Adding

94− 9

4inside the parentheses

= 2(x2 + 3x+

94

)− 2 · 9

4+ 8 Removing

−94

from within the parentheses

= 2(x+

32

)2

+72

Factoring and

simplifying

= 2[x−

(− 3

2

)]2+

72

a) Vertex:(− 3

2,72

)

b) Axis of symmetry: x = −32

c) Minimum value:72

d) We plot the vertex and find several points on eitherside of it. Then we plot these points and connectthem with a smooth curve.

x f(x)

−32

72

−1 4

0 8

−2 4

−3 8

13. f(x) = −x2 − 6x+ 3

= −(x2 + 6x) + 3 9 completes the squarefor x2 + 6x.

= −(x2 + 6x+ 9 − 9) + 3

= −(x+ 3)2 − (−9) + 3 Removing −9 fromthe parentheses

= −(x+ 3)2 + 9 + 3

= −[x− (−3)]2 + 12

a) Vertex: (−3, 12)

b) Axis of symmetry: x = −3

c) Maximum value: 12

d) We plot the vertex and find several points on eitherside of it. Then we plot these points and connectthem with a smooth curve.

x f(x)

−3 12

0 3

1 −4

−6 3

−7 −4

15. g(x) = −2x2 + 2x+ 1

= −2(x2 − x) + 1 Factoring −2 out of the

first two terms

= −2(x2−x+

14− 1

4

)+1 Adding

14− 1

4inside the parentheses

= −2(x2−x+

14

)−2(− 1

4

)+1

Removing −14

from within the parentheses

= −2(x− 1

2

)2

+32

a) Vertex:(

12,32

)

b) Axis of symmetry: x =12

c) Maximum value:32

Copyright © 2013 Pearson Education, Inc.

Page 110: JUDITH A. PENNA -  · PDF fileSTUDENT’S SOLUTIONS MANUAL JUDITH A. PENNA Indiana University Purdue University Indianapolis COLLEGE ALGEBRA: GRAPHS AND MODELS FIFTH EDITION

x2 4

2

4

�4 �2

�4

�2

y

g(x) � �2x

2 � 2x � 1

Exercise Set 3.3 107

d) We plot the vertex and find several points on eitherside of it. Then we plot these points and connectthem with a smooth curve.

x f(x)

12

32

1 1

2 −3

0 1

−1 −3

17. The graph of y = (x + 3)2 has vertex (−3, 0) and opensup. It is graph (f).

19. The graph of y = 2(x − 4)2 − 1 has vertex (4,−1) andopens up. It is graph (b).

21. The graph of y = −12(x+ 3)2 + 4 has vertex (−3, 4) and

opens down. It is graph (h).

23. The graph of y = −(x + 3)2 + 4 has vertex (−3, 4) andopens down. It is graph (c).

25. The function f(x) = −3x2 + 2x+ 5 is of the formf(x) = ax2 + bx+ c with a < 0, so it is true that it has amaximum value.

27. The statement is false. The graph of h(x) = (x+ 2)2 canbe obtained by translating the graph of h(x) = x2 twounits to the left.

29. The function f(x) = −(x+ 2)2 − 4 can be written asf(x) = −[x − (−2)]2 − 4, so it is true that the axis ofsymmetry is x = −2.

31. f(x) = x2 − 6x+ 5

a) The x-coordinate of the vertex is

− b

2a= − −6

2 · 1 = 3.

Since f(3) = 32−6·3+5 = −4, the vertex is (3,−4).

b) Since a = 1 > 0, the graph opens up so the secondcoordinate of the vertex, −4, is the minimum valueof the function.

c) The range is [−4,∞).

d) Since the graph opens up, function values decreaseto the left of the vertex and increase to the right ofthe vertex. Thus, f(x) is increasing on (3,∞) anddecreasing on (−∞, 3).

33. f(x) = 2x2 + 4x− 16

a) The x-coordinate of the vertex is

− b

2a= − 4

2 · 2 = −1.

Since f(−1) = 2(−1)2 + 4(−1) − 16 = −18, thevertex is (−1,−18).

b) Since a = 2 > 0, the graph opens up so the secondcoordinate of the vertex, −18, is the minimum valueof the function.

c) The range is [−18,∞).

d) Since the graph opens up, function values decreaseto the left of the vertex and increase to the right ofthe vertex. Thus, f(x) is increasing on (−1,∞) anddecreasing on (−∞,−1).

35. f(x) = −12x2 + 5x− 8

a) The x-coordinate of the vertex is

− b

2a= − 5

2(− 1

2

) = 5.

Since f(5) = −12· 52 + 5 · 5 − 8 =

92, the vertex is(

5,92

).

b) Since a = −12< 0, the graph opens down so the sec-

ond coordinate of the vertex,92, is the maximum

value of the function.

c) The range is(−∞, 9

2

].

d) Since the graph opens down, function values in-crease to the left of the vertex and decrease to theright of the vertex. Thus, f(x) is increasing on(−∞, 5) and decreasing on (5,∞).

37. f(x) = 3x2 + 6x+ 5

a) The x-coordinate of the vertex is

− b

2a= − 6

2 · 3 = −1.

Since f(−1) = 3(−1)2 + 6(−1) + 5 = 2, the vertexis (−1, 2).

b) Since a = 3 > 0, the graph opens up so the secondcoordinate of the vertex, 2, is the minimum value ofthe function.

c) The range is [2,∞).

d) Since the graph opens up, function values decreaseto the left of the vertex and increase to the right ofthe vertex. Thus, f(x) is increasing on (−1,∞) anddecreasing on (−∞,−1).

39. g(x) = −4x2 − 12x+ 9

a) The x-coordinate of the vertex is

− b

2a= − −12

2(−4)= −3

2.

Since g(− 3

2

)=−4

(− 3

2

)2

−12(− 3

2

)+ 9=18,

the vertex is(− 3

2, 18).

b) Since a = −4 < 0, the graph opens down so thesecond coordinate of the vertex, 18, is the maximumvalue of the function.

c) The range is (−∞, 18].

Copyright © 2013 Pearson Education, Inc.

Page 111: JUDITH A. PENNA -  · PDF fileSTUDENT’S SOLUTIONS MANUAL JUDITH A. PENNA Indiana University Purdue University Indianapolis COLLEGE ALGEBRA: GRAPHS AND MODELS FIFTH EDITION

108 Chapter 3: Quadratic Functions and Equations; Inequalities

d) Since the graph opens down, function values in-crease to the left of the vertex and decrease to theright of the vertex. Thus, g(x) is increasing on(−∞,−3

2

)and decreasing on

(− 3

2,∞).

41. Familiarize and Translate. The functions(t) = −16t2 + 20t + 6 is given in the statement of theproblem.

Carry out. The function s(t) is quadratic and the coef-ficient of t2 is negative, so s(t) has a maximum value. Itoccurs at the vertex of the graph of the function. We findthe first coordinate of the vertex. This is the time at whichthe ball reaches its maximum height.

t = − b

2a= − 20

2(−16)= 0.625

The second coordinate of the vertex gives the maximumheight.

s(0.625) = −16(0.625)2 + 20(0.625) + 6 = 12.25

Check. Completing the square, we write the function inthe form s(t) = −16(t− 0.625)2 + 12.25. We see that thecoordinates of the vertex are (0.625, 12.25), so the answerchecks.

State. The ball reaches its maximum height after0.625 seconds. The maximum height is 12.25 ft.

43. Familiarize and Translate. The functions(t) = −16t2 + 120t + 80 is given in the statement of theproblem.

Carry out. The function s(t) is quadratic and the coef-ficient of t2 is negative, so s(t) has a maximum value. Itoccurs at the vertex of the graph of the function. We findthe first coordinate of the vertex. This is the time at whichthe rocket reaches its maximum height.

t = − b

2a= − 120

2(−16)= 3.75

The second coordinate of the vertex gives the maximumheight.

s(3.75) = −16(3.75)2 + 120(3.75) + 80 = 305

Check. Completing the square, we write the function inthe form s(t) = −16(t − 3.75)2 + 305. We see that thecoordinates of the vertex are (3.75, 305), so the answerchecks.

State. The rocket reaches its maximum height after3.75 seconds. The maximum height is 305 ft.

45. Familiarize. Using the label in the text, we let x = theheight of the file. Then the length = 10 and the width =18 − 2x.

Translate. Since the volume of a rectangular solid islength × width × height we have

V (x) = 10(18 − 2x)x, or −20x2 + 180x.

Carry out. Since V (x) is a quadratic function with a =−20 < 0, the maximum function value occurs at the vertexof the graph of the function. The first coordinate of thevertex is

− b

2a= − 180

2(−20)= 4.5.

Check. When x = 4.5, then 18 − 2x = 9 and V (x) =10 ·9(4.5), or 405. As a partial check, we can find V (x) fora value of x less than 4.5 and for a value of x greater than4.5. For instance, V (4.4) = 404.8 and V (4.6) = 404.8.Since both of these values are less than 405, our resultappears to be correct.

State. The file should be 4.5 in. tall in order to maximizethe volume.

47. Familiarize. Let b = the length of the base of the triangle.Then the height = 20 − b.Translate. Since the area of a triangle is

12× base×height,

we have

A(b) =12b(20 − b), or −1

2b2 + 10b.

Carry out. Since A(b) is a quadratic function with

a = −12< 0, the maximum function value occurs at the

vertex of the graph of the function. The first coordinateof the vertex is

− b

2a= − 10

2(− 1

2

) = 10.

When b = 10, then 20 − b = 20 − 10 = 10, and the area is12· 10 · 10 = 50 cm2.

Check. As a partial check, we can find A(b) for a valueof b less than 10 and for a value of b greater than 10. Forinstance, V (9.9) = 49.995 and V (10.1) = 49.995. Sinceboth of these values are less than 50, our result appears tobe correct.

State. The area is a maximum when the base and theheight are both 10 cm.

49. C(x) = 0.1x2 − 0.7x+ 1.625

Since C(x) is a quadratic function with a = 0.1 > 0, aminimum function value occurs at the vertex of the graphof C(x). The first coordinate of the vertex is

− b

2a= − −0.7

2(0.1)= 3.5.

Thus, 3.5 hundred, or 350 chairs should be built to mini-mize the average cost per chair.

51. P (x) = R(x) − C(x)

P (x) = (50x− 0.5x2) − (10x+ 3)

P (x) = −0.5x2 + 40x− 3

Since P (x) is a quadratic function with a =−0.5 < 0, a maximum function value occurs at the vertexof the graph of the function. The first coordinate of thevertex is

− b

2a= − 40

2(−0.5)= 40.

P (40) = −0.5(40)2 + 40 · 40 − 3 = 797

Thus, the maximum profit is $797. It occurs when 40 unitsare sold.

Copyright © 2013 Pearson Education, Inc.

Page 112: JUDITH A. PENNA -  · PDF fileSTUDENT’S SOLUTIONS MANUAL JUDITH A. PENNA Indiana University Purdue University Indianapolis COLLEGE ALGEBRA: GRAPHS AND MODELS FIFTH EDITION

4

2

�2

42�2�4 x

y

g(x) � �2f(x)

(0, �4)(�3, �4)

(5, 0)(�5, 0)

(3, 4)

y

x

8

16

24

�8

�4�8 84

f(x) � (|x | � 5)2 � 3

Exercise Set 3.3 109

53. Familiarize. Using the labels on the drawing in the text,we let x = the width of each corral and 240 − 3x = thetotal length of the corrals.

Translate. Since the area of a rectangle is length×width,we have

A(x) = (240 − 3x)x = −3x2 + 240x.

Carry out. Since A(x) is a quadratic function with a =−3 < 0, the maximum function value occurs at the vertexof the graph of A(x). The first coordinate of the vertex is

− b

2a= − 240

2(−3)= 40.

A(40) = −3(40)2 + 240(40) = 4800

Check. As a partial check we can find A(x) for a valueof x less than 40 and for a value of x greater than 40. Forinstance, A(39.9) = 4799.97 and A(40.1) = 4799.97. Sinceboth of these values are less than 4800, our result appearsto be correct.

State. The largest total area that can be enclosed is4800 yd2.

55. Familiarize. We let s = the height of the elevator shaft,t1 = the time it takes the screwdriver to reach the bottomof the shaft, and t2 = the time it takes the sound to reachthe top of the shaft.

Translate. We know that t1 + t2 = 5. Using the informa-tion in Example 7 we also know that

s = 16t21, or t1 =√s

4and

s = 1100t2, or t2 =s

1100.

Then√s

4+

s

1100= 5.

Carry out. We solve the last equation above.√s

4+

s

1100= 5

275√s+ s = 5500 Multiplying by 1100

s+ 275√s− 5500 = 0

Let u =√s and substitute.

u2 + 275u− 5500 = 0

u =−b+

√b2 − 4ac

2aWe only want thepositive solution.

=−275 +

√2752 − 4 · 1(−5500)

2 · 1

=−275 +

√97, 625

2≈ 18.725

Since u ≈ 18.725, we have√s = 18.725, so s ≈ 350.6.

Check. If s ≈ 350.6, then t1 =√s

4=

√350.64

≈4.68 and t2 =

s

1100=

350.61100

≈ 0.32, so t1 + t2 = 4.68 +0.32 = 5.

The result checks.

State. The elevator shaft is about 350.6 ft tall.

57. f(x) = 3x− 7f(x+ h) − f(x)

h=

3(x+ h) − 7 − (3x− 7)h

=3x+ 3h− 7 − 3x+ 7

h

=3hh

= 3

59. The graph of f(x) is stretched vertically and reflectedacross the x-axis.

61. f(x) = −0.2x2 − 3x+ c

The x-coordinate of the vertex of f(x) is − b

2a=

− −32(−0.2)

= −7.5. Now we find c such that f(−7.5) =

−225.−0.2(−7.5)2 − 3(−7.5) + c = −225

−11.25 + 22.5 + c = −225

c = −236.25

63.

65. First we find the radius r of a circle with circumference x:2πr = x

r =x

2πThen we find the length s of a side of a square with perime-ter 24 − x:

4s = 24 − xs =

24 − x4

Then S = area of circle + area of square

S = πr2 + s2

S(x) = π

(x

)2

+(

24 − x4

)2

S(x) =(

14π

+116

)x2 − 3x+ 36

Copyright © 2013 Pearson Education, Inc.

Page 113: JUDITH A. PENNA -  · PDF fileSTUDENT’S SOLUTIONS MANUAL JUDITH A. PENNA Indiana University Purdue University Indianapolis COLLEGE ALGEBRA: GRAPHS AND MODELS FIFTH EDITION

110 Chapter 3: Quadratic Functions and Equations; Inequalities

Since S(x) is a quadratic function with

a =14π

+116> 0, the minimum function value occurs at

the vertex of the graph of S(x). The first coordinate ofthe vertex is

− b

2a= − −3

2(

14π

+116

) =24π

4 + π.

Then the string should be cut so that one piece is24π

4 + πin.,

or about 10.56 in. The other piece will be 24 − 24π4 + π

, or96

4 + πin., or about 13.44 in.

Chapter 3 Mid-Chapter Mixed Review

1. The statement is true. See page 240 in the text.

3. The statement is true. See page 251 in the text.

5.√−36 =

√−1 · 36 =√−1 · √36 = i · 6 = 6i

7. −√−16 = −√−1 · 16 = −√−1 · √16 = −i · 4 = −4i

9. (3 − 2i) + (−4 + 3i) = (3 − 4) + (−2i+ 3i) = −1 + i

11. (2 + 3i)(4 − 5i) = 8 − 10i+ 12i− 15i2

= 8 + 2i+ 15

= 23 + 2i

13. i13 = i12 · i = (i2)6 · i = (−1)6 · i = i

15. (−i)5 = (−1 · i)5 = (−1)5i5 = −i4 · i = −(i2)2 · i =

−(−1)2 · i = −i17. x2 + 3x− 4 = 0

(x+ 4)(x− 1) = 0

x+ 4 = 0 or x− 1 = 0

x = −4 or x = 1

The solutions are −4 and 1.

19. 4x2 = 24

x2 = 6

x =√

6 or x = −√6

The solutions are√

6 and −√6, or ±√

6.

21. 4x2 − 8x− 3 = 0

4x2 − 8x = 3

x2 − 2x =34

x2 − 2x+ 1 =34

+ 1 Completing the square:12(−2) = −1 and (−1)2 = 1

(x− 1)2 =74

x− 1 = ±√

72

x = 1 +√

72

x =2 ±√

72

The zeros are2 +

√7

2and

2 −√7

4, or

2 ±√7

2.

23. 4x2 − 12x+ 9 = 0

a) b2 − 4ac = (−12)2 − 4 · 4 · 9 = 144 − 144 = 0

There is one real-number solution.

b) 4x2 − 12x+ 9 = 0

(2x− 3)2 = 0

2x− 3 = 0

2x = 3

x =32

The solution is32.

25. x4 + 5x2 − 6 = 0

Let u = x2.u2 + 5u− 6 = 0 Substituting

(u+ 6)(u− 1) = 0

u+ 6 = 0 or u− 1 = 0

u = −6 or u = 1

x2 = −6 or x2 = 1

x = ±√6i or x = ±1

The solutions are ±√6i and ±1.

27. Familiarize. Let x = the smaller number. Then x+ 2 =the larger number.

Translate.The product of the numbers︸ ︷︷ ︸ is 35.

↓ ↓ ↓x(x+ 2) = 35

Carry out.

x(x+ 2) = 35

x2 + 2x = 35

x2 + 2x− 35 = 0

(x+ 7)(x− 5) = 0

Copyright © 2013 Pearson Education, Inc.

Page 114: JUDITH A. PENNA -  · PDF fileSTUDENT’S SOLUTIONS MANUAL JUDITH A. PENNA Indiana University Purdue University Indianapolis COLLEGE ALGEBRA: GRAPHS AND MODELS FIFTH EDITION

2

2

�6 �4 �2

�6

�2

�4

f(x) � �2x2 � 4x � 5

x

y

Exercise Set 3.4 111

x+ 7 = 0 or x− 5 = 0

x = −7 or x = 5

If x = −7, then x + 2 = −7 + 2 = −5; if x = 5, thenx+ 2 = 5 + 2 = 7.

Check. −5 is 2 more than −7, and (−7)(−5) = 35. Also,7 is 2 more than 5, and 5 · 7 = 35. The numbers check.

State. The numbers are 5 and 7 or −7 and −5.

29. f(x) = −2x2 − 4x− 5

= −2(x2 + 2x) − 5

= −2(x2 + 2x+ 1 − 1) − 5

= −2(x2 + 2x+ 1) − 2(−1) − 5

= −2(x+ 1)2 − 3

= −2[x− (−1)]2 + (−3)

a) Vertex: (−1,−3)

b) Axis of symmetry: x = −1

c) Maximum value: −3

d) Range: (−∞,−3]

e) Increasing: (−∞,−3); decreasing: (−3,∞)

f)x f(x)

−1 −3

−3 −11

−2 −5

0 −5

1 −11

31. Use the discriminant. If b2 − 4ac < 0, there are no x-intercepts. If b2 − 4ac = 0, there is one x-intercept. Ifb2 − 4ac > 0, there are two x-intercepts.

33. The x-intercepts of g(x) are also (x1, 0) and (x2, 0). This istrue because f(x) and g(x) have the same zeros. Considerg(x) = 0, or −ax2 − bx − c = 0. Multiplying by −1 onboth sides, we get an equivalent equation ax2 + bx+c = 0,or f(x) = 0.

Exercise Set 3.4

1. 14

+15

=1t, LCD is 20t

20t(1

4+

15

)= 20t · 1

t

20t · 14

+ 20t · 15

= 20t · 1t

5t+ 4t = 20

9t = 20

t =209

Check:14

+15

=1t

14

+15

?1209

∣∣520

+420

∣∣∣∣ 1 · 920

920

∣∣∣∣ 920

TRUE

The solution is209

.

3.x+ 2

4− x− 1

5= 15, LCD is 20

20(x+ 2

4− x− 1

5

)= 20 · 15

5(x+ 2) − 4(x− 1) = 300

5x+ 10 − 4x+ 4 = 300

x+ 14 = 300

x = 286The solution is 286.

5.12

+2x

=13

+3x, LCD is 6x

6x(1

2+

2x

)= 6x

(13

+3x

)3x+ 12 = 2x+ 18

3x− 2x = 18 − 12

x = 6Check:12

+2x

=13

+3x

12

+26

?13

+36|

12

+13

∣∣∣∣ 13

+12

TRUE

The solution is 6.

7.5

3x+ 2=

32x, LCD is 2x(3x+ 2)

2x(3x+ 2) · 53x+ 2

= 2x(3x+ 2) · 32x

2x · 5 = 3(3x+ 2)

10x = 9x+ 6

x = 66 checks, so the solution is 6.

9. y2

y + 4=

16y + 4

, LCD is y + 4

(y + 4) · y2

y + 4= (y + 4) · 16

y + 4y2 = 16

y = 4 or y = −4

Only 4 checks. It is the solution.

Copyright © 2013 Pearson Education, Inc.

Page 115: JUDITH A. PENNA -  · PDF fileSTUDENT’S SOLUTIONS MANUAL JUDITH A. PENNA Indiana University Purdue University Indianapolis COLLEGE ALGEBRA: GRAPHS AND MODELS FIFTH EDITION

112 Chapter 3: Quadratic Functions and Equations; Inequalities

11. x+6x

= 5, LCD is x

x

(x+

6x

)= x · 5

x2 + 6 = 5x

x2 − 5x+ 6 = 0

(x− 2)(x− 3) = 0

x− 2 = 0 or x− 3 = 0

x = 2 or x = 3

Both numbers check. The solutions are 2 and 3.

13. 6y + 3

+2y

=5y − 3y2 − 9

6y + 3

+2y

=5y − 3

(y + 3)(y − 3),

LCD is y(y+3)(y−3)

y(y+3)(y−3)(

6y+3

+2y

)= y(y+3)(y−3)· 5y−3

(y+3)(y−3)6y(y−3)+2(y+3)(y−3) = y(5y − 3)

6y2 − 18y + 2(y2 − 9) = 5y2 − 3y

6y2 − 18y + 2y2 − 18 = 5y2 − 3y

8y2 − 18y − 18 = 5y2 − 3y

3y2 − 15y − 18 = 0

y2 − 5y − 6 = 0

(y − 6)(y + 1) = 0

y − 6 = 0 or y + 1 = 0

y = 6 or y = −1

Both numbers check. The solutions are 6 and −1.

15. 2xx− 1

=5

x− 3, LCD is (x− 1)(x− 3)

(x− 1)(x− 3) · 2xx− 1

= (x− 1)(x− 3) · 5x− 3

2x(x− 3) = 5(x− 1)

2x2 − 6x = 5x− 5

2x2 − 11x+ 5 = 0

(2x− 1)(x− 5) = 0

2x− 1 = 0 or x− 5 = 0

2x = 1 or x = 5

x =12or x = 5

Both numbers check. The solutions are12

and 5.

17. 2x+ 5

+1

x− 5=

16x2 − 25

2x+ 5

+1

x− 5=

16(x+ 5)(x− 5)

,

LCD is (x+ 5)(x− 5)

(x+5)(x−5)(

2x+5

+1

x−5

)= (x+5)(x−5)· 16

(x+5)(x−5)2(x− 5) + x+ 5 = 16

2x− 10 + x+ 5 = 16

3x− 5 = 16

3x = 21

x = 7

7 checks, so the solution is 7.

19. 3xx+ 2

+6x

=12

x2 + 2x3xx+2

+6x

=12

x(x+2), LCD is x(x+2)

x(x+ 2)( 3xx+ 2

+6x

)= x(x+ 2) · 12

x(x+ 2)3x · x+ 6(x+ 2) = 12

3x2 + 6x+ 12 = 12

3x2 + 6x = 0

3x(x+ 2) = 0

3x = 0 or x+ 2 = 0

x = 0 or x = −2

Neither 0 nor −2 checks, so the equation has no solution.

21. 15x+ 20

− 1x2 − 16

=3

x− 41

5(x+ 4)− 1

(x+ 4)(x− 4)=

3x− 4

,

LCD is 5(x+ 4)(x− 4)

5(x+4)(x−4)(

15(x+4)

− 1(x+4)(x−4)

)= 5(x+4)(x−4)· 3

x−4x− 4 − 5 = 15(x+ 4)

x− 9 = 15x+ 60

−14x− 9 = 60

−14x = 69

x = −6914

−6914

checks, so the solution is −6914

.

Copyright © 2013 Pearson Education, Inc.

Page 116: JUDITH A. PENNA -  · PDF fileSTUDENT’S SOLUTIONS MANUAL JUDITH A. PENNA Indiana University Purdue University Indianapolis COLLEGE ALGEBRA: GRAPHS AND MODELS FIFTH EDITION

Exercise Set 3.4 113

23. 25x+ 5

− 3x2 − 1

=4

x− 12

5(x+ 1)− 3

(x+ 1)(x− 1)=

4x− 1

,

LCD is 5(x+ 1)(x− 1)

5(x+1)(x−1)(

25(x+1)

− 3(x+1)(x−1)

)= 5(x+1)(x−1)· 4

x−12(x− 1) − 5 · 3 = 20(x+ 1)

2x− 2 − 15 = 20x+ 20

2x− 17 = 20x+ 20

−18x− 17 = 20

−18x = 37

x = −3718

−3718

checks, so the solution is −3718

.

25. 8x2 − 2x+ 4

=x

x+ 2+

24x3 + 8

,

LCD is (x+ 2)(x2 − 2x+ 4)

(x+2)(x2−2x+4) · 8x2−2x+4

=

(x+2)(x2− 2x+4)(

x

x+2+

24(x+2)(x2−2x+4)

)8(x+ 2) = x(x2−2x+4)+24

8x+ 16 = x3−2x2+4x+24

0 = x3−2x2−4x+8

0 = x2(x−2) − 4(x−2)

0 = (x−2)(x2−4)

0 = (x−2)(x+2)(x−2)

x− 2 = 0 or x+ 2 = 0 or x− 2 = 0

x = 2 or x = −2 or x = 2

Only 2 checks. The solution is 2.

27. x

x− 4− 4x+ 4

=32

x2 − 16x

x− 4− 4x+ 4

=32

(x+4)(x−4),

LCD is (x+4)(x−4)

(x+4)(x−4)( xx−4

− 4x+4

)= (x+4)(x−4)· 32

(x+4)(x−4)x(x+ 4) − 4(x− 4) = 32

x2 + 4x− 4x+ 16 = 32

x2 + 16 = 32

x2 = 16

x = ±4

Neither 4 nor −4 checks, so the equation has no solution.

29.1

x− 6− 1x

=6

x2 − 6x1

x− 6− 1x

=6

x(x−6), LCD is x(x−6)

x(x−6)(

1x−6

− 1x

)= x(x−6) · 6

x(x−6)x− (x− 6) = 6

x− x+ 6 = 6

6 = 6

We get an equation that is true for all real numbers.Note, however, that when x = 6 or x = 0, divisionby 0 occurs in the original equation. Thus, the solutionset is {x|x is a real number and x �= 6 and x �= 0}, or(−∞, 0) ∪ (0, 6) ∪ (6,∞).

31.√

3x− 4 = 1

(√

3x− 4)2 = 12

3x− 4 = 1

3x = 5

x =53

Check:√3x− 4 = 1√

3 · 53− 4 ? 1∣∣√

5 − 4∣∣

√1∣∣∣

1∣∣ 1 TRUE

The solution is53.

33.√

2x− 5 = 2

(√

2x− 5)2 = 22

2x− 5 = 4

2x = 9

x =92

Check:√2x− 5 = 2√

2 · 92− 5 ? 2∣∣√

9 − 5∣∣

√4∣∣∣

2∣∣ 2 TRUE

The solution is92.

35.√

7 − x = 2

(√

7 − x)2 = 22

7 − x = 4

−x = −3

x = 3

Copyright © 2013 Pearson Education, Inc.

Page 117: JUDITH A. PENNA -  · PDF fileSTUDENT’S SOLUTIONS MANUAL JUDITH A. PENNA Indiana University Purdue University Indianapolis COLLEGE ALGEBRA: GRAPHS AND MODELS FIFTH EDITION

114 Chapter 3: Quadratic Functions and Equations; Inequalities

Check:√7 − x = 2

√7 − 3 ? 2√

4∣∣∣

2∣∣ 2 TRUE

The solution is 3.

37.√

1 − 2x = 3

(√

1 − 2x)2 = 32

1 − 2x = 9

−2x = 8

x = −4Check:√

1 − 2x = 3√1 − 2(−4) ? 3√

1 + 8∣∣∣√

9∣∣

3∣∣ 3 TRUE

The solution is −4.

39. 3√

5x− 2 = −3

( 3√

5x− 2)3 = (−3)3

5x− 2 = −27

5x = −25

x = −5Check:

3√

5x− 2 = −3

3√

5(−5) − 2 ? −33√−25 − 2

∣∣∣3√−27

∣∣−3

∣∣ −3 TRUEThe solution is −5.

41. 4√x2 − 1 = 1

( 4√x2 − 1)4 = 14

x2 − 1 = 1

x2 = 2

x = ±√2

Check:4√x2 − 1 = 1

4

√(±√

2)2 − 1 ? 14√

2 − 1∣∣∣

4√

1∣∣∣

1∣∣ 1 TRUE

The solutions are ±√2.

43.√y − 1 + 4 = 0√

y − 1 = −4The principal square root is never negative. Thus, there isno solution.

If we do not observe the above fact, we can continue andreach the same answer.

(√y − 1)2 = (−4)2

y − 1 = 16

y = 17

Check:√y − 1 + 4 = 0

√17 − 1 + 4 ? 0√

16 + 4∣∣∣

4 + 4∣∣

8∣∣ 0 FALSE

Since 17 does not check, there is no solution.

45.√b+ 3 − 2 = 1√

b+ 3 = 3

(√b+ 3)

2= 32

b+ 3 = 9

b = 6

Check:√b+ 3 − 2 = 1

√6 + 3 − 2 ? 1√

9 − 2∣∣∣

3 − 2∣∣

1∣∣ 1 TRUE

The solution is 6.

47.√z + 2 + 3 = 4√

z + 2 = 1

(√z + 2)2 = 12

z + 2 = 1

z = −1

Check:√z + 2 + 3 = 4

√−1 + 2 + 3 ? 4√1 + 3

∣∣∣1 + 3

∣∣4∣∣ 4 TRUE

The solution is −1.

49.√

2x+ 1 − 3 = 3√2x+ 1 = 6

(√

2x+ 1)2 = 62

2x+ 1 = 36

2x = 35

x =352

Copyright © 2013 Pearson Education, Inc.

Page 118: JUDITH A. PENNA -  · PDF fileSTUDENT’S SOLUTIONS MANUAL JUDITH A. PENNA Indiana University Purdue University Indianapolis COLLEGE ALGEBRA: GRAPHS AND MODELS FIFTH EDITION

Exercise Set 3.4 115

Check:√2x+ 1 − 3 = 3√

2 · 352

+ 1 − 3 ? 3∣∣√35 + 1 − 3

∣∣∣√36 − 3

∣∣6 − 3

∣∣∣3∣∣ 3 TRUE

The solution is352

.

51.√

2 − x− 4 = 6√2 − x = 10

(√

2 − x)2 = 102

2 − x = 100

−x = 98

x = −98

Check:√2 − x− 4 = 6√

2 − (−98) − 4 ? 6√100 − 4

∣∣∣10 − 4

∣∣6∣∣ 6 TRUE

The solution is −98.

53. 3√

6x+ 9 + 8 = 53√

6x+ 9 = −3

( 3√

6x+ 9)3 = (−3)3

6x+ 9 = −27

6x = −36

x = −6

Check:

3√

6x+ 9 + 8 = 5

3√

6(−6) + 9 + 8 ? 53√−27 + 8

∣∣∣−3 + 8

∣∣5∣∣ 5 TRUE

The solution is −6.

55.√x+ 4 + 2 = x√

x+ 4 = x− 2

(√x+ 4)2 = (x− 2)2

x+ 4 = x2 − 4x+ 4

0 = x2 − 5x

0 = x(x− 5)

x = 0 or x− 5 = 0

x = 0 or x = 5

Check:

For 0:√x+ 4 + 2 = x

√0 + 4 + 2 ? 0

2 + 2∣∣∣

4∣∣ 0 FALSE

For 5:√x+ 4 + 2 = x

√5 + 4 + 2 ? 5√

9 + 2∣∣∣

3 + 2∣∣

5∣∣ 5 TRUE

The number 5 checks but 0 does not. The solution is 5.

57.√x− 3 + 5 = x√

x− 3 = x− 5

(√x− 3)2 = (x− 5)2

x− 3 = x2 − 10x+ 25

0 = x2 − 11x+ 28

0 = (x− 4)(x− 7)

x− 4 = 0 or x− 7 = 0

x = 4 or x = 7

Check:

For 4:√x− 3 + 5 = x

√4 − 3 + 5 ? 4√

1 + 5∣∣∣

1 + 5∣∣

6∣∣ 4 FALSE

For 7:√x− 3 + 5 = x

√7 − 3 + 5 ? 7√

4 + 5∣∣∣

2 + 5∣∣

7∣∣ 7 TRUE

The number 7 checks but 4 does not. The solution is 7.

59.√x+ 7 = x+ 1

(√x+ 7)2 = (x+ 1)2

x+ 7 = x2 + 2x+ 1

0 = x2 + x− 6

0 = (x+ 3)(x− 2)

x+ 3 = 0 or x− 2 = 0

x = −3 or x = 2

Copyright © 2013 Pearson Education, Inc.

Page 119: JUDITH A. PENNA -  · PDF fileSTUDENT’S SOLUTIONS MANUAL JUDITH A. PENNA Indiana University Purdue University Indianapolis COLLEGE ALGEBRA: GRAPHS AND MODELS FIFTH EDITION

116 Chapter 3: Quadratic Functions and Equations; Inequalities

Check:

For −3:√x+ 7 = x+ 1

√−3 + 7 ? −3 + 1√4∣∣∣ −2

2∣∣ −2 FALSE

For 2:√x+ 7 = x+ 1

√2 + 7 ? 2 + 1√

9∣∣∣ 3

3∣∣ 3 TRUE

The number 2 checks but −3 does not. The solution is 2.

61.√

3x+ 3 = x+ 1

(√

3x+ 3)2 = (x+ 1)2

3x+ 3 = x2 + 2x+ 1

0 = x2 − x− 2

0 = (x− 2)(x+ 1)

x− 2 = 0 or x+ 1 = 0

x = 2 or x = −1

Check:

For 2:√3x+ 3 = x+ 1

√3 · 2 + 3 ? 2 + 1√

9∣∣∣ 3

3∣∣ 3 TRUE

For −1:√3x+ 3 = x+ 1√

3(−1) + 3 ? −1 + 1√0∣∣∣ 0

0∣∣ 0 TRUE

Both numbers check. The solutions are 2 and −1.

63.√

5x+ 1 = x− 1

(√

5x+ 1)2 = (x− 1)2

5x+ 1 = x2 − 2x+ 1

0 = x2 − 7x

0 = x(x− 7)

x = 0 or x− 7 = 0

x = 0 or x = 7

Check:

For 0:√5x+ 1 = x− 1

√5 · 0 + 1 ? 0 − 1√

1∣∣∣ −1

1∣∣ −1 FALSE

For 7:√5x+ 1 = x− 1

√5 · 7 + 1 ? 7 − 1√

36∣∣∣ 6

6∣∣ 6 TRUE

The number 7 checks but 0 does not. The solution is 7.

65.√x− 3 +

√x+ 2 = 5√x+ 2 = 5 −√

x− 3

(√x+ 2)2 = (5 −√

x− 3)2

x+ 2 = 25 − 10√x− 3 + (x− 3)

x+ 2 = 22 − 10√x− 3 + x

10√x− 3 = 20√x− 3 = 2

(√x− 3)2 = 22

x− 3 = 4

x = 7

Check:√x− 3 +

√x+ 2 = 5

√7 − 3 +

√7 + 2 ? 5

|√4 +

√9∣∣

2 + 3∣∣

5∣∣ 5 TRUE

The solution is 7.

67.√

3x− 5 +√

2x+ 3 + 1 = 0√3x− 5 +

√2x+ 3 = −1

The principal square root is never negative. Thus the sumof two principal square roots cannot equal −1. There is nosolution.

69.√x−√

3x− 3 = 1√x =

√3x− 3 + 1

(√x)2 = (

√3x− 3 + 1)2

x = (3x− 3) + 2√

3x− 3 + 1

2 − 2x = 2√

3x− 3

1 − x =√

3x− 3

(1 − x)2 = (√

3x− 3)2

1 − 2x+ x2 = 3x− 3

x2 − 5x+ 4 = 0

(x− 4)(x− 1) = 0x = 4 or x = 1

The number 4 does not check, but 1 does. The solution is1.

Copyright © 2013 Pearson Education, Inc.

Page 120: JUDITH A. PENNA -  · PDF fileSTUDENT’S SOLUTIONS MANUAL JUDITH A. PENNA Indiana University Purdue University Indianapolis COLLEGE ALGEBRA: GRAPHS AND MODELS FIFTH EDITION

Exercise Set 3.4 117

71.√

2y − 5 −√y − 3 = 1√

2y − 5 =√y − 3 + 1

(√

2y − 5)2 = (√y − 3 + 1)2

2y − 5 = (y − 3) + 2√y − 3 + 1

y − 3 = 2√y − 3

(y − 3)2 = (2√y − 3)2

y2 − 6y + 9 = 4(y − 3)

y2 − 6y + 9 = 4y − 12

y2 − 10y + 21 = 0

(y − 7)(y − 3) = 0y = 7 or y = 3

Both numbers check. The solutions are 7 and 3.

73.√y + 4 −√

y − 1 = 1√y + 4 =

√y − 1 + 1

(√y + 4)2 = (

√y − 1 + 1)2

y + 4 = y − 1 + 2√y − 1 + 1

4 = 2√y − 1

2 =√y − 1 Dividing by 2

22 = (√y − 1)2

4 = y − 1

5 = y

The answer checks. The solution is 5.

75.√x+ 5 +

√x+ 2 = 3√x+ 5 = 3 −√

x+ 2

(√x+ 5)2 = (3 −√

x+ 2)2

x+ 5 = 9 − 6√x+ 2 + x+ 2

−6 = −6√x+ 2

1 =√x+ 2 Dividing by −6

12 = (√x+ 2)2

1 = x+ 2

−1 = x

The answer checks. The solution is −1.

77. x1/3 = −2

(x1/3)3 = (−2)3 (x1/3 = 3√x)

x = −8

The value checks. The solution is −8.

79. t1/4 = 3

(t1/4)4 = 34 (t1/4 = 4√t)

t = 81

The value checks. The solution is 81.

81. P1V1

T1=P2V2

T2

P1V1T2 = P2V2T1 Multiplying by T1T2 onboth sides

P1V1T2

P2V2= T1 Dividing by P2V2 on

both sides

83. W =

√1LC

W 2 =(√

1LC

)2

Squaring both sides

W 2 =1LC

CW 2 =1L

Multiplying by C

C =1

LW 2Dividing by W 2

85. 1R

=1R1

+1R2

RR1R2 · 1R

= RR1R2

(1R1

+1R2

)Multiplying by RR1R2 on both sides

R1R2 = RR2 +RR1

R1R2 −RR2 = RR1 Subtracting RR2 onboth sides

R2(R1 −R) = RR1 Factoring

R2 =RR1

R1 −R Dividing by

R1 −R on both sides

87. I =

√A

P− 1

I + 1 =

√A

PAdding 1

(I + 1)2 =(√

A

P

)2

I2 + 2I + 1 =A

P

P (I2+2I+1) = A Multplying by P

P =A

I2+2I+1Dividing by I2+2I+1

We could also express this result as P =A

(I + 1)2.

89. 1F

=1m

+1p

Fmp · 1F

= Fmp( 1m

+1p

)Multiplying byFmp on both sides

mp = Fp+ Fm

mp− Fp = Fm Subtracting Fp onboth sides

p(m− F ) = Fm Factoring

p =Fm

m− F Dividing by m− F on

both sides

Copyright © 2013 Pearson Education, Inc.

Page 121: JUDITH A. PENNA -  · PDF fileSTUDENT’S SOLUTIONS MANUAL JUDITH A. PENNA Indiana University Purdue University Indianapolis COLLEGE ALGEBRA: GRAPHS AND MODELS FIFTH EDITION

118 Chapter 3: Quadratic Functions and Equations; Inequalities

91. 15 − 2x = 0 Setting f(x) = 0

15 = 2x152

= x, or

7.5 = x

The zero of the function is152

, or 7.5.

93. Familiarize. Let f = the number of highway fatalitiesinvolving distracted driving in 2004.

Translate.Fatalitiesin 2004︸ ︷︷ ︸ plus 18% more︸ ︷︷ ︸ is

fatalitiesin 2008︸ ︷︷ ︸� � � � �

f + 0.18f = 5870

Carry out. We solve the equation.

f + 0.18f = 5870

1.18f = 5870

f ≈ 4975

Check. 18% of 4975 is 0.18(4975) ≈ 896 and 4975+896 =5871 ≈ 5870. The answer checks. (Remember that werounded the value of f .)

State. About 4975 highway fatalities involved distracteddriving in 2004.

95. (x− 3)2/3 = 2

[(x− 3)2/3]3 = 23

(x− 3)2 = 8

x2 − 6x+ 9 = 8

x2 − 6x+ 1 = 0

a = 1, b = −6, c = 1

x =−b±√

b2 − 4ac2a

=−(−6) ±√(−6)2 − 4 · 1 · 1

2 · 1=

6 ±√32

2=

6 ± 4√

22

=2(3 ± 2

√2)

2= 3 ± 2

√2

Both values check. The solutions are 3 ± 2√

2.

97.√x+ 5 + 1 =

6√x+ 5

, LCD is√x+ 5

x+ 5 +√x+ 5 = 6 Multiplying by

√x+ 5√

x+ 5 = 1 − xx+ 5 = 1 − 2x+ x2

0 = x2 − 3x− 4

0 = (x− 4)(x+ 1)x = 4 or x = −1

Only −1 checks. The solution set is −1.

99. x2/3 = x

(x2/3)3 = x3

x2 = x3

0 = x3 − x2

0 = x2(x− 1)

x2 = 0 or x− 1 = 0

x = 0 or x = 1

Both numbers check. The solutions are 0 and 1.

Exercise Set 3.5

1. |x| = 7

The solutions are those numbers whose distance from 0 ona number line is 7. They are −7 and 7. That is,

x = −7 or x = 7.

The solutions are −7 and 7.

3. |x| = 0

The distance of 0 from 0 on a number line is 0. That is,

x = 0.

The solution is 0.

5. |x| =56

x = −56or x =

56

The solutions are −56

and56.

7. |x| = −10.7

The absolute value of a number is nonnegative. Thus, theequation has no solution.

9. |3x| = 1

3x = −1 or 3x = 1

x = −13or x =

13

The solutions are −13

and13.

11. |8x| = 24

8x = −24 or 8x = 24

x = −3 or x = 3

The solutions are −3 and 3.

13. |x− 1| = 4

x− 1 = −4 or x− 1 = 4

x = −3 or x = 5

The solutions are −3 and 5.

15. |x+ 2| = 6

x+ 2 = −6 or x+ 2 = 6

x = −8 or x = 4

The solutions are −8 and 4.

Copyright © 2013 Pearson Education, Inc.

Page 122: JUDITH A. PENNA -  · PDF fileSTUDENT’S SOLUTIONS MANUAL JUDITH A. PENNA Indiana University Purdue University Indianapolis COLLEGE ALGEBRA: GRAPHS AND MODELS FIFTH EDITION

)(�7 0 7

0�2 2

] [�4.5 0 4.5

0�3 3

013��

13�

Exercise Set 3.5 119

17. |3x+ 2| = 1

3x+ 2 = −1 or 3x+ 2 = 1

3x = −3 or 3x = −1

x = −1 or x = −13

The solutions are −1 and −13.

19.∣∣∣12x− 5

∣∣∣ = 17

12x− 5 = −17 or

12x− 5 = 17

12x = −12 or

12x = 22

x = −24 or x = 44

The solutions are −24 and 44.

21. |x− 1| + 3 = 6

|x− 1| = 3

x− 1 = −3 or x− 1 = 3

x = −2 or x = 4

The solutions are −2 and 4.

23. |x+ 3| − 2 = 8

|x+ 3| = 10

x+ 3 = −10 or x+ 3 = 10

x = −13 or x = 7

The solutions are −13 and 7.

25. |3x+ 1| − 4 = −1

|3x+ 1| = 3

3x+ 1 = −3 or 3x+ 1 = 3

3x = −4 or 3x = 2

x = −43or x =

23

The solutions are −43

and23.

27. |4x− 3| + 1 = 7

|4x− 3| = 6

4x− 3 = −6 or 4x− 3 = 6

4x = −3 or 4x = 9

x = −34or x =

94

The solutions are −34

and94.

29. 12 − |x+ 6| = 5

−|x+ 6| = −7

|x+ 6| = 7 Multiplying by −1

x+ 6 = −7 or x+ 6 = 7

x = −13 or x = 1

The solutions are −13 and 1.

31. 7 − |2x− 1| = 6

−|2x− 1| = −1

|2x− 1| = 1 Multiplying by −1

2x− 1 = −1 or 2x− 1 = 1

2x = 0 or 2x = 2

x = 0 or x = 1

The solutions are 0 and 1.

33. |x| < 7

To solve we look for all numbers x whose distance from0 is less than 7. These are the numbers between −7 and7. That is, −7 < x < 7. The solution set is (−7, 7). Thegraph is shown below.

35. |x| ≤ 2

−2 ≤ x ≤ 2

The solution set is [−2, 2]. The graph is shown below.

37. |x| ≥ 4.5

To solve we look for all numbers x whose distance from 0 isgreater than or equal to 4.5. That is, x ≤ −4.5 or x ≥ 4.5.The solution set and its graph are as follows.

{x|x ≤ −4.5 or x ≥ 4.5}, or (−∞,−4.5] ∪ [4.5,∞)

39. |x| > 3

x < −3 or x > 3

The solution set is (−∞,−3)∪ (3,∞). The graph is shownbelow.

41. |3x| < 1

−1 < 3x < 1

−13< x <

13

Dividing by 3

The solution set is(− 1

3,13

). The graph is shown below.

43. |2x| ≥ 6

2x ≤ −6 or 2x ≥ 6

x ≤ −3 or x ≥ 3

The solution set is (−∞,−3]∪ [3,∞). The graph is shownbelow.

Copyright © 2013 Pearson Education, Inc.

Page 123: JUDITH A. PENNA -  · PDF fileSTUDENT’S SOLUTIONS MANUAL JUDITH A. PENNA Indiana University Purdue University Indianapolis COLLEGE ALGEBRA: GRAPHS AND MODELS FIFTH EDITION

0�3 3

0 1�17

0 1�17

0 034�

14��

0�6 3

5.14.90

0

� 1 2

7 2

0 173��

0�8 7

120 Chapter 3: Quadratic Functions and Equations; Inequalities

45. |x+ 8| < 9

−9 < x+ 8 < 9

−17 < x < 1 Subtracting 8

The solution set is (−17, 1). The graph is shown below.

47. |x+ 8| ≥ 9

x+ 8 ≤ −9 or x+ 8 ≥ 9

x ≤ −17 or x ≥ 1 Subtracting 8

The solution set is (−∞,−17]∪[1,∞). The graph is shownbelow.

49.∣∣∣∣x− 1

4

∣∣∣∣ < 12

−12< x− 1

4<

12

−14< x <

34

Adding14

The solution set is(− 1

4,34

). The graph is shown below.

51. |2x+ 3| ≤ 9

−9 ≤ 2x+ 3 ≤ 9

−12 ≤ 2x ≤ 6 Subtracting 3

−6 ≤ x ≤ 3 Dividing by 2

The solution set is [−6, 3]. The graph is shown below.

53. |x− 5| > 0.1

x− 5 < −0.1 or x− 5 > 0.1

x < 4.9 or x > 5.1 Adding 5

The solution set is (−∞, 4.9) ∪ (5.1,∞). The graph isshown below.

55. |6 − 4x| ≥ 8

6 − 4x ≤ −8 or 6 − 4x ≥ 8

−4x ≤ −14 or −4x ≥ 2 Subtracting 6

x ≥ 144

or x ≤ −24

Dividing by −4 and

reversing the inequality symbols

x ≥ 72

or x ≤ −12

Simplifying

The solution set is(−∞,−1

2

]∪[72,∞)

. The graph is

shown below.

57.∣∣∣∣x+

23

∣∣∣∣ ≤ 53

−53≤ x+

23≤ 5

3

−73≤ x ≤ 1 Subtracting

23

The solution set is[− 7

3, 1]. The graph is shown below.

59.∣∣∣∣2x+ 1

3

∣∣∣∣ > 5

2x+ 13

< −5 or2x+ 1

3> 5

2x+ 1 < −15 or 2x+ 1 > 15 Multiplying by 3

2x < −16 or 2x > 14 Subtracting 1

x < −8 or x > 7 Dividing by 2

The solution set is {x|x < −8 or x > 7}, or(−∞,−8) ∪ (7,∞). The graph is shown below.

61. |2x− 4| < −5

Since |2x− 4| ≥ 0 for all x, there is no x such that |2x− 4|would be less than −5. There is no solution.

63. |7 − x| ≥ −4

Since absolute value is nonnegative, for any real-numbervalue of x we have |7−x| ≥ 0 ≥ −4. Thus the solution setis (−∞,∞). The graph is the entire number line.

65. y-intercept

67. relation

69. horizontal lines

71. decreasing

73. |3x− 1| > 5x− 2

3x− 1 < −(5x− 2) or 3x− 1 > 5x− 2

3x− 1 < −5x+ 2 or 1 > 2x

8x < 3 or12> x

x <38

or12> x

The solution set is(−∞, 3

8

)∪(−∞, 1

2

). This is equiv-

alent to(−∞, 1

2

).

Copyright © 2013 Pearson Education, Inc.

Page 124: JUDITH A. PENNA -  · PDF fileSTUDENT’S SOLUTIONS MANUAL JUDITH A. PENNA Indiana University Purdue University Indianapolis COLLEGE ALGEBRA: GRAPHS AND MODELS FIFTH EDITION

Chapter 3 Review Exercises 121

75. |p− 4| + |p+ 4| < 8

If p < −4, then |p− 4| = −(p− 4) and |p+ 4| = −(p+ 4).

Solve: −(p− 4) + [−(p+ 4)] < 8

−p+ 4 − p− 4 < 8

−2p < 8

p > −4

Since this is false for all values of p in the interval (−∞,−4)there is no solution in this interval.

If p ≥ −4, then |p+ 4| = p+ 4.

Solve: |p− 4| + p+ 4 < 8

|p− 4| < 4 − pp− 4 > −(4 − p) and p− 4 < 4 − pp− 4 > p− 4 and 2p < 8

−4 > −4 and p < 4

Since −4 > −4 is false for all values of p, there is nosolution in the interval [−4,∞).

Thus, |p− 4| + |p+ 4| < 8 has no solution.

77. |x− 3| + |2x+ 5| > 6

Divide the set of real numbers into three intervals:(−∞,−5

2

),[− 5

2, 3)

, and [3,∞).

Find the solution set of |x − 3| + |2x + 5| > 6 in eachinterval. Then find the union of the three solution sets.

If x < −52, then |x−3| = −(x−3) and |2x+5| = −(2x+5).

Solve: x < −52and −(x− 3) + [−(2x+ 5)] > 6

x < −52and −x+ 3 − 2x− 5 > 6

x < −52and −3x > 8

x < −52and x < −8

3

The solution set in this interval is(−∞,−8

3

).

If −52≤ x < 3, then |x−3| = −(x−3) and |2x+5| = 2x+5.

Solve: −52≤ x < 3 and −(x− 3) + 2x+ 5 > 6

−52≤ x < 3 and −x+ 3 + 2x+ 5 > 6

−52≤ x < 3 and x > −2

The solution set in this interval is (−2, 3).

If x ≥ 3, then |x− 3| = x− 3 and |2x+ 5| = 2x+ 5.

Solve: x ≥ 3 and x− 3 + 2x+ 5 > 6

x ≥ 3 and 3x > 4

x ≥ 3 and x >43

The solution set in this interval is [3,∞).

The union of the above solution sets is

(−∞,−8

3

)∪ (−2,∞). This is the solution set of

|x− 3| + |2x+ 5| > 6.

Chapter 3 Review Exercises

1. The statement is true. See page 248 in the text.

3. The statement is false. For example, 32 = (−3)2, but3 �= −3.

5. (2y + 5)(3y − 1) = 0

2y + 5 = 0 or 3y − 1 = 0

2y = −5 or 3y = 1

y = −52or y =

13

The solutions are −52

and13.

7. 3x2 + 2x = 8

3x2 + 2x− 8 = 0

(x+ 2)(3x− 4) = 0

x+ 2 = 0 or 3x− 4 = 0

x = −2 or 3x = 4

x = −2 or x =43

The solutions are −2 and43.

9. x2 + 10 = 0

x2 = −10

x = −√−10 or x =√−10

x = −√10i or x =

√10i

The solutions are −√10i and

√10i.

11. x2 + 2x− 15 = 0

(x+ 5)(x− 3) = 0

x+ 5 = 0 or x− 3 = 0

x = −5 or x = 3

The zeros of the function are −5 and 3.

13. 3x2 + 2x+ 3 = 0

a = 3, b = 2, c = 3

x =−b±√

b2 − 4ac2a

x =−2 ±√

22 − 4 · 3 · 32 · 3

=−2 ±√−32

2 · 3 =−2 ±√−16 · 2

2 · 3 =−2 ± 4i

√2

2 · 3

=2(−1 ± 2i

√2)

2 · 3 =−1 ± 2i

√2

3

The zeros of the function are−1 ± 2i

√2

3.

Copyright © 2013 Pearson Education, Inc.

Page 125: JUDITH A. PENNA -  · PDF fileSTUDENT’S SOLUTIONS MANUAL JUDITH A. PENNA Indiana University Purdue University Indianapolis COLLEGE ALGEBRA: GRAPHS AND MODELS FIFTH EDITION

0 2� 14

3

122 Chapter 3: Quadratic Functions and Equations; Inequalities

15. 38x+ 1

+8

2x+ 5= 1

LCD is (8x+1)(2x+5)

(8x+1)(2x+5)(

38x+1

+8

2x+5

)= (8x+1)(2x+5)·1

3(2x+ 5) + 8(8x+ 1) = (8x+1)(2x+5)

6x+ 15 + 64x+ 8 = 16x2 + 42x+ 5

70x+ 23 = 16x2 + 42x+ 5

0 = 16x2−28x−18

0 = 2(8x2−14x−9)

0 = 2(2x+1)(4x−9)

2x+ 1 = 0 or 4x− 9 = 0

2x = −1 or 4x = 9

x = −12or x =

94

Both numbers check. The solutions are −12

and94.

17.√x− 1 −√

x− 4 = 1√x− 1 =

√x− 4 + 1

(√x− 1)2 = (

√x− 4 + 1)2

x− 1 = x− 4 + 2√x− 4 + 1

x− 1 = x− 3 + 2√x− 4

2 = 2√x− 4

1 =√x− 4 Dividing by 2

12 = (√x− 4)2

1 = x− 4

5 = x

This number checks. The solution is 5.

19. |2y + 7| = 9

2y + 7 = −9 or 2y + 7 = 9

2y = −16 or 2y = 2

y = −8 or y = 1

The solutions are −8 and 1.

21. |3x+ 4| < 10

−10 < 3x+ 4 < 10

−14 < 3x < 6

−143< x < 2

The solution set is(− 14

3, 2)

. The graph is shown below.

23. |x+ 4| ≥ 2

x+ 4 ≤ −2 or x+ 4 ≥ 2

x ≤ −6 or x ≥ −2

The solution is (−∞,−6] ∪ [−2,∞).

25. −√−40 = −√−1 · √4 · √10 = −2√

10i

27.√−49−√−64

=7i−8i

= −78

29. (3 − 5i) − (2 − i) = (3 − 2) + [−5i− (−i)]= 1 − 4i

31. 2 − 3i1 − 3i

=2 − 3i1 − 3i

· 1 + 3i1 + 3i

=2 + 3i− 9i2

1 − 9i2

=2 + 3i+ 9

1 + 9

=11 + 3i

10

=1110

+310i

33. x2 − 3x = 18

x2−3x+94

= 18+94

(12(−3)=−3

2and

(− 3

2

)2

=94

)(x− 3

2

)2

=814

x− 32

= ±92

x =32± 9

2

x =32− 9

2or x =

32

+92

x = −3 or x = 6

The solutions are −3 and 6.

35. 3x2 + 10x = 8

3x2 + 10x− 8 = 0

(x+ 4)(3x− 2) = 0

x+ 4 = 0 or 3x− 2 = 0

x = −4 or 3x = 2

x = −4 or x =23

The solutions are −4 and23.

37. x2 = 10 + 3x

x2 − 3x− 10 = 0

(x+ 2)(x− 5) = 0

x+ 2 = 0 or x− 5 = 0

x = −2 or x = 5

The solutions are −2 and 5.

39. y4 − 3y2 + 1 = 0

Let u = y2.

u2 − 3u+ 1 = 0

u =−(−3) ±√(−3)2 − 4 · 1 · 1

2 · 1 =3 ±√

52

Copyright © 2013 Pearson Education, Inc.

Page 126: JUDITH A. PENNA -  · PDF fileSTUDENT’S SOLUTIONS MANUAL JUDITH A. PENNA Indiana University Purdue University Indianapolis COLLEGE ALGEBRA: GRAPHS AND MODELS FIFTH EDITION

x2 4

2

4

�4 �2

�4

�2

y

f (x) � �4x

2 � 3x � 1

Chapter 3 Review Exercises 123

Substitute y2 for u and solve for y.

y2 =3 ±√

52

y = ±√

3 ±√5

2

The solutions are ±√

3 ±√5

2.

41. (p− 3)(3p+ 2)(p+ 2) = 0

p− 3 = 0 or 3p+ 2 = 0 or p+ 2 = 0

p = 3 or 3p = −2 or p = −2

p = 3 or p = −23or p = −2

The solutions are −2, −23

and 3.

43. f(x) = −4x2 + 3x− 1

= −4(x2 − 3

4x

)− 1

= −4(x2 − 3

4x+

964

− 964

)− 1

= −4(x2 − 3

4x+

964

)− 4(− 9

64

)− 1

= −4(x2 − 3

4x+

964

)+

916

− 1

= −4(x− 3

8

)2

− 716

a) Vertex:(

38,− 7

16

)

b) Axis of symmetry: x =38

c) Maximum value: − 716

d) Range:(−∞,− 7

16

]e)

45. The graph of y = (x− 2)2 has vertex (2, 0) and opens up.It is graph (d).

47. The graph of y = −2(x + 3)2 + 4 has vertex (−3, 4) andopens down. It is graph (b).

49. Familiarize. Using the labels in the textbook, the legs ofthe right triangle are represented by x and x+ 10.

Translate. We use the Pythagorean theorem.

x2 + (x+ 10)2 = 502

Carry out. We solve the equation.x2 + (x+ 10)2 = 502

x2 + x2 + 20x+ 100 = 2500

2x2 + 20x− 2400 = 0

2(x2 + 10x− 1200) = 0

2(x+ 40)(x− 30) = 0

x+ 40 = 0 or x− 30 = 0

x = −40 or x = 30

Check. Since the length cannot be negative, we need tocheck only 30. If x = 30, then x+10 = 30+10 = 40. Since302 + 402 = 900 + 1600 = 2500 = 502, the answer checks.

State. The lengths of the legs are 30 ft and 40 ft.

51. Familiarize. Using the drawing in the textbook, let w =the width of the sidewalk, in ft. Then the length of thenew parking lot is 80 − 2w, and its width is 60 − 2w.

Translate. We use the formula for the area of a rectangle,A = lw.

New area︸ ︷︷ ︸ is23

of old area︸ ︷︷ ︸↓ ↓ ↓ ↓ ↓

(80 − 2w)(60 − 2w) =23

· 80 · 60

Carry out. We solve the equation.

(80 − 2w)(60 − 2w) =23· 80 · 60

4800 − 280w + 4w2 =23· 80 · 3 · 20

4w2 − 280w + 4800 = 3200

4w2 − 280w + 1600 = 0

w2 − 70w + 400 = 0 Dividing by 4

We use the quadratic formula.

w =−b±√

b2 − 4ac2a

=−(−70) ±√(−70)2 − 4 · 1 · 400

2 · 1

=70 ±√

33002

=70 ±√

33 · 1002

=70 ± 10

√33

2= 35 ± 5

√33

35 + 5√

33 ≈ 63.7 and 35 − 5√

33 ≈ 6.3

Check. The width of the sidewalk cannot be 63.7 ft be-cause this width exceeds the width of the original parkinglot, 60 ft. We check 35 − 5

√33 ≈ 6.3. If the width of the

sidewalk in about 6.3 ft, then the length of the new park-ing lot is 80− 2(6.3), or 67.4, and the width is 60− 2(6.3),or 47.4. The area of a parking lot with these dimensions is(67.4)(47.4) = 3194.76. Two-thirds of the area of the origi-

nal parking lot is23· 80 · 60 = 3200. Since 3194.76 ≈ 3200,

this answer checks.

Copyright © 2013 Pearson Education, Inc.

Page 127: JUDITH A. PENNA -  · PDF fileSTUDENT’S SOLUTIONS MANUAL JUDITH A. PENNA Indiana University Purdue University Indianapolis COLLEGE ALGEBRA: GRAPHS AND MODELS FIFTH EDITION

124 Chapter 3: Quadratic Functions and Equations; Inequalities

State. The width of the sidewalk is 35−5√

33 ft, or about6.3 ft.

53. Familiarize. Using the labels in the textbook, let x = thelength of the sides of the squares, in cm. Then the lengthof the base of the box is 20− 2x and the width of the baseis 10 − 2x.

Translate. We use the formula for the area of a rectangle,A = lw.

90 = (20 − 2x)(10 − 2x)

90 = 200 − 60x+ 4x2

0 = 4x2 − 60x+ 110

0 = 2x2 − 30x+ 55 Dividing by 2

We use the quadratic formula.

Carry out.

x =−b±√

b2 − 4ac2a

=−(−30) ±√(−30)2 − 4 · 2 · 55

2 · 2

=30 ±√

4604

=30 ±√

4 · 1154

=30 ± 2

√115

4

=15 ±√

1152

15 +√

1152

≈ 12.9 and15 −√

1152

≈ 2.1.

Check. The length of the sides of the squares cannotbe 12.9 cm because this length exceeds the width of thepiece of aluminum. We check 2.1 cm. If the sides of thesquares are 2.1 cm, then the length of the base of the box is20−2(2.1) = 15.8, and the width is 10−2(2.1) = 5.8. Thearea of the base is 15.8(5.8) = 91.64 ≈ 90. This answerchecks.

State. The length of the sides of the squares is

15 −√115

2cm, or about 2.1 cm.

55.√

4x+ 1 +√

2x = 1√4x+ 1 = 1 −√

2x

(√

4x+ 1)2 = (1 −√2x)2

4x+ 1 = 1 − 2√

2x+ 2x

2x = −2√

2x

x = −√2x

x2 = (−√2x)2

x2 = 2x

x2 − 2x = 0

x(x− 2) = 0x = 0 or x = 2

Only 0 checks, so answer B is correct.

57.√√√

x = 2(√√√x

)2

= 22

√√x = 4(√√

x)2 = 42

√x = 16

(√x)2 = 162

x = 256The answer checks. The solution is 256.

59. (x− 1)2/3 = 4

(x− 1)2 = 43

x− 1 = ±√64

x− 1 = ±8

x− 1 = −8 or x− 1 = 8

x = −7 or x = 9Both numbers check. The solutions are −7 and 9.

61.√x+ 2 + 4

√x+ 2 − 2 = 0

Let u = 4√x+ 2, so u2 = ( 4

√x+ 2)2 =

√x+ 2.

u2 + u− 2 = 0

(u+ 2)(u− 1) = 0u = −2 or u = 1

Substitute 4√x+ 2 for u and solve for x.

4√x+ 2 = −2 or 4

√x+ 2 = 1

No real solution x+ 2 = 1

x = −1This number checks. The solution is −1.

63. The maximum value occurs at the vertex. The first coordi-

nate of the vertex is − b

2a= − b

2(−3)=b

6and f

(b

6

)= 2.

−3(b

6

)2

+ b

(b

6

)− 1 = 2

− b2

12+b2

6− 1 = 2

−b2 + 2b2 − 12 = 24

b2 = 36

b = ±6

65. No; consider the quadratic formula

x =−b±√

b2 − 4ac2a

. If b2 − 4ac = 0, then x =−b2a

, so

there is one real zero. If b2 − 4ac > 0, then√b2 − 4ac is a

real number and there are two real zeros. If b2 − 4ac < 0,then

√b2 − 4ac is an imaginary number and there are two

imaginary zeros. Thus, a quadratic function cannot haveone real zero and one imaginary zero.

67. When both sides of an equation are multiplied by the LCD,the resulting equation might not be equivalent to the orig-inal equation. One or more of the possible solutions ofthe resulting equation might make a denominator of theoriginal equation 0.

Copyright © 2013 Pearson Education, Inc.

Page 128: JUDITH A. PENNA -  · PDF fileSTUDENT’S SOLUTIONS MANUAL JUDITH A. PENNA Indiana University Purdue University Indianapolis COLLEGE ALGEBRA: GRAPHS AND MODELS FIFTH EDITION

Chapter 3 Test 125

69. Absolute value is nonnegative.

Chapter 3 Test

1. (2x− 1)(x+ 5) = 0

2x− 1 = 0 or x+ 5 = 0

2x = 1 or x = −5

x =12or x = −5

The solutions are12

and −5.

2. 6x2 − 36 = 0

6x2 = 36

x2 = 6

x = −√6 or x =

√6

The solutions are −√6 and

√6.

3. x2 + 4 = 0

x2 = −4

x = ±√−4x = −2i or x = 2i

The solutions are −2i and 2i.

4. x2 − 2x− 3 = 0

(x+ 1)(x− 3) = 0

x+ 1 = 0 or x− 3 = 0

x = −1 or x = 3The solutions are −1 and 3.

5. x2 − 5x+ 3 = 0

a = 1, b = −5, c = 3

x =−b±√

b2 − 4ac2a

x =−(−5) ±√(−5)2 − 4 · 1 · 3

2 · 1

=5 ±√

132

The solutions are5 +

√13

2and

5 −√13

2.

6. 2t2 − 3t+ 4 = 0

a = 2, b = −3, c = 4

x =−b±√

b2 − 4ac2a

x =−(−3) ±√(−3)2 − 4 · 2 · 4

2 · 2

=3 ±√−23

4=

3 ± i√234

=34±

√234i

The solutions are34

+√

234i and

34−

√234i.

7. x+ 5√x− 36 = 0

Let u =√x.

u2 + 5u− 36 = 0

(u+ 9)(u− 4) = 0

u+ 9 = 0 or u− 4 = 0

u = −9 or u = 4

Substitute√x for u and solve for x.√

x = −9 or√x = 4

No solution x = 16

The number 16 checks. It is the solution.

8.3

3x+ 4+

2x−1

= 2, LCD is (3x+4)(x−1)

(3x+4)(x−1)(

33x+4

+2

x−1

)= (3x+4)(x−1)(2)

3(x− 1) + 2(3x+ 4) = 2(3x2 + x− 4)

3x− 3 + 6x+ 8 = 6x2 + 2x− 8

9x+ 5 = 6x2 + 2x− 8

0 = 6x2 − 7x− 13

0 = (x+ 1)(6x− 13)

x+ 1 = 0 or 6x− 13 = 0

x = −1 or 6x = 13

x = −1 or x =136

Both numbers check. The solutions are −1 and136

.

9.√x+ 4 − 2 = 1√

x+ 4 = 3

(√x+ 4)2 = 32

x+ 4 = 9

x = 5

This number checks. The solution is 5.

10.√x+ 4 −√

x− 4 = 2√x+ 4 =

√x− 4 + 2

(√x+ 4)2 = (

√x− 4 + 2)2

x+ 4 = x− 4 + 4√x− 4 + 4

4 = 4√x− 4

1 =√x− 4

12 = (√x− 4)2

1 = x− 4

5 = x

This number checks. The solution is 5.

11. |x+ 4| = 7

x+ 4 = −7 or x+ 4 = 7

x = −11 or x = 3

The solutions are −11 and 3.

Copyright © 2013 Pearson Education, Inc.

Page 129: JUDITH A. PENNA -  · PDF fileSTUDENT’S SOLUTIONS MANUAL JUDITH A. PENNA Indiana University Purdue University Indianapolis COLLEGE ALGEBRA: GRAPHS AND MODELS FIFTH EDITION

0�2 3

50�2

126 Chapter 3: Quadratic Functions and Equations; Inequalities

12. |4y − 3| = 5

4y − 3 = −5 or 4y − 3 = 5

4y = −2 or 4y = 8

y = −12or y = 2

The solutions are −12

and 2.

13. |x+ 3| ≤ 4

−4 ≤ x+ 3 ≤ 4

−7 ≤ x ≤ 1

The solution set is [−7, 1].

14. |2x− 1| < 5

−5 < 2x− 1 < 5

−4 < 2x < 6

−2 < x < 3

The solution set is (−2, 3).

15. |x+ 5| > 2

x+ 5 < −2 or x+ 5 > 2

x < −7 or x > −3

The solution set is (−∞,−7) ∪ (−3,∞).

16. |3 − 2x| ≥ 7

3 − 2x ≤ −7 or 3 − 2x ≥ 7

−2x ≤ −10 or −2x ≥ 4

x ≥ 5 or x ≤ −2

The solution set is (−∞,−2] ∪ [5,∞).

17. 1A

+1B

=1C

ABC

(1A

+1B

)= ABC · 1

C

BC +AC = AB

AC = AB −BCAC = B(A− C)

AC

A− C = B

18. R =√

3np

R2 = (√

3np)2

R2 = 3np

R2

3p= n

19. x2 + 4x = 1

x2 + 4x+ 4 = 1 + 4(

12(4) = 2 and 22 = 4

)(x+ 2)2 = 5

x+ 2 = ±√5

x = −2 ±√5

The solutions are −2 +√

5 and −2 −√5.

20. Familiarize and Translate. We will use the formulas = 16t2, substituting 2063 for s.

2063 = 16t2

Carry out. We solve the equation.

2063 = 16t2

206316

= t2

11.4 ≈ t

Check. When t = 11.4, s = 16(11.4)2 = 2079.36 ≈ 2063.The answer checks.

State. It would take an object about 11.4 sec to reach theground.

21.√−43 =

√−1 · √43 = i√

43, or√

43i

22. −√−25 = −√−1 · √25 = −5i

23. (5 − 2i) − (2 + 3i) = (5 − 2) + (−2i− 3i)

= 3 − 5i

24. (3 + 4i)(2 − i) = 6 − 3i+ 8i− 4i2

= 6 + 5i+ 4 (i2 = −1)

= 10 + 5i

25. 1 − i6 + 2i

=1 − i6 + 2i

· 6 − 2i6 − 2i

=6 − 2i− 6i+ 2i2

36 − 4i2

=6 − 8i− 2

36 + 4

=4 − 8i

40

=440

− 840i

=110

− 15i

26. i33 = (i2)16 · i = (−1)16 · i = 1 · i = i

27. 4x2 − 11x− 3 = 0

(4x+ 1)(x− 3) = 0

4x+ 1 = 0 or x− 3 = 0

4x = −1 or x = 3

x = −14or x = 3

The zeros of the functions are −14

and 3.

Copyright © 2013 Pearson Education, Inc.

Page 130: JUDITH A. PENNA -  · PDF fileSTUDENT’S SOLUTIONS MANUAL JUDITH A. PENNA Indiana University Purdue University Indianapolis COLLEGE ALGEBRA: GRAPHS AND MODELS FIFTH EDITION

2 4

2

4

6

8

�4 �2 x

y

f (x) � �x

2 � 2x � 8

Chapter 3 Test 127

28. 2x2 − x− 7 = 0

a = 2, b = −1, c = −7

x =−b±√

b2 − 4ac2a

x =−(−1) ±√(−1)2 − 4 · 2 · (−7)

2 · 2

=1 ±√

574

The solutions are1 +

√57

4and

1 −√57

4.

29. f(x) = −x2 + 2x+ 8

= −(x2 − 2x) + 8

= −(x2 − 2x+ 1 − 1) + 8

= −(x2 − 2x+ 1) − (−1) + 8

= −(x2 − 2x+ 1) + 1 + 8

= −(x− 1)2 + 9

a) Vertex: (1, 9)

b) Axis of symmetry: x = 1

c) Maximum value: 9

d) Range: (−∞, 9]

e)

30. Familiarize. We make a drawing, letting w = the widthof the rectangle, in ft. This leaves 80−w−w, or 80−2w ftof fencing for the length.

80 − 2w

House

w w

Translating. The area of a rectangle is given by lengthtimes width.

A(w) = (80 − 2w)w

= 80w − 2w2, or − 2w2 + 80wCarry out. This is a quadratic function with a < 0, soit has a maximum value that occurs at the vertex of thegraph of the function. The first coordinate of the vertex is

w = − b

2a= − 80

2(−2)= 20.

If w = 20, then 80 − 2w = 80 − 2 · 20 = 40.

Check. The area of a rectangle with length 40 ft andwidth 20 ft is 40 · 20, or 800 ft2. As a partial check, we

can find A(w) for a value of w less than 20 and for a valueof w greater than 20. For instance, A(19.9) = 799.98 andA(20.1) = 799.98. Since both of these values are less than800, the result appears to be correct.

State. The dimensions for which the area is a maximumare 20 ft by 40 ft.

31. f(x) = x2 − 2x− 1

= (x2 − 2x+ 1 − 1) − 1 Completing the square

= (x2 − 2x+ 1) − 1 − 1

= (x− 1)2 − 2

The graph of this function opens up and has vertex (1,−2).Thus the correct graph is C.

32. The maximum value occurs at the vertex. The first coordi-

nate of the vertex is − b

2a= − (−4)

2a=

2a

and f(

2a

)= 12.

Then we have:

a

(2a

)2

− 4(

2a

)+ 3 = 12

a · 4a2

− 8a

+ 3 = 12

4a− 8a

+ 3 = 12

−4a

+ 3 = 12

−4a

= 9

−4 = 9a

−49

= a

Copyright © 2013 Pearson Education, Inc.

Page 131: JUDITH A. PENNA -  · PDF fileSTUDENT’S SOLUTIONS MANUAL JUDITH A. PENNA Indiana University Purdue University Indianapolis COLLEGE ALGEBRA: GRAPHS AND MODELS FIFTH EDITION

Copyright © 2013 Pearson Education, Inc.

Page 132: JUDITH A. PENNA -  · PDF fileSTUDENT’S SOLUTIONS MANUAL JUDITH A. PENNA Indiana University Purdue University Indianapolis COLLEGE ALGEBRA: GRAPHS AND MODELS FIFTH EDITION

Chapter 4

Polynomial and Rational Functions

Exercise Set 4.1

1. g(x) =12x3 − 10x + 8

The leading term is12x3 and the leading coefficient is

12.

The degree of the polynomial is 3, so the polynomial iscubic.

3. h(x) = 0.9x− 0.13

The leading term is 0.9x and the leading coefficient is 0.9.The degree of the polynomial is 1, so the polynomial islinear.

5. g(x) = 305x4 + 4021

The leading term is 305x4 and the leading coefficient is305. The degree of the polynomial is 4, so the polynomialis quartic.

7. h(x) = −5x2 + 7x3 + x4 = x4 + 7x3 − 5x2

The leading term is x4 and the leading coefficient is 1 (x4 =1·x4). The degree of the polynomial is 4, so the polynomialis quartic.

9. g(x) = 4x3 − 12x2 + 8

The leading term is 4x3 and the leading coefficient is 4.The degree of the polynomial is 3, so the polynomial iscubic.

11. f(x) = −3x3 − x + 4

The leading term is −3x3. The degree, 3, is odd and theleading coefficient, −3, is negative. Thus the end behaviorof the graph is like that of (d).

13. f(x) = −x6 +34x4

The leading term is −x6. The degree, 6, is even and theleading coefficient, −1, is negative. Thus the end behaviorof the graph is like that of (b).

15. f(x) = −3.5x4 + x6 + 0.1x7 = 0.1x7 + x6 − 3.5x4

The leading term is 0.1x7. The degree, 7, is odd and theleading coefficient, 0.1, is positive. Thus the end behaviorof the graph is like that of (c).

17. f(x) = 10 +110

x4 − 25x3 =

110

x4 − 25x3 + 10

The leading term is110

x4. The degree, 4, is even and the

leading coefficient,110

, is positive. Thus the end behavior

of the graph is like that of (a).

19. f(x) = −x6 + 2x5 − 7x2

The leading term is −x6. The degree, 6, is even and theleading coefficient, −1, is negative. Thus, (c) is the correctgraph.

21. f(x) = x5 +110

x− 3

The leading term is x5. The degree, 5, is odd and theleading coefficient, 1, is positive. Thus, (d) is the correctgraph.

23. f(x) = x3 − 9x2 + 14x + 24

f(4) = 43 − 9 · 42 + 14 · 4 + 24 = 0

Since f(4) = 0, 4 is a zero of f(x).

f(5) = 53 − 9 · 52 + 14 · 5 + 24 = −6

Since f(5) �= 0, 5 is not a zero of f(x).

f(−2) = (−2)3 − 9(−2)2 + 14(−2) + 24 = −48

Since f(−2) �= 0, −2 is not a zero of f(x).

25. g(x) = x4 − 6x3 + 8x2 + 6x− 9

g(2) = 24 − 6 · 23 + 8 · 22 + 6 · 2 − 9 = 3

Since g(2) �= 0, 2 is not a zero of g(x).

g(3) = 34 − 6 · 33 + 8 · 32 + 6 · 3 − 9 = 0

Since g(3) = 0, 3 is a zero of g(x).

g(−1) = (−1)4 − 6(−1)3 + 8(−1)2 + 6(−1) − 9 = 0

Since g(−1) = 0, −1 is a zero of g(x).

27. f(x) = (x + 3)2(x− 1) = (x + 3)(x + 3)(x− 1)

To solve f(x) = 0 we use the principle of zero products,solving x+ 3 = 0 and x− 1 = 0. The zeros of f(x) are −3and 1.

The factor x + 3 occurs twice. Thus the zero −3 has amultiplicity of two.

The factor x−1 occurs only one time. Thus the zero 1 hasa multiplicity of one.

29. f(x) = −2(x− 4)(x− 4)(x− 4)(x+6) = −2(x− 4)3(x+6)

To solve f(x) = 0 we use the principle of zero products,solving x − 4 = 0 and x + 6 = 0. The zeros of f(x) are 4and −6.

The factor x − 4 occurs three times. Thus the zero 4 hasa multiplicity of 3.

The factor x + 6 occurs only one time. Thus the zero −6has a multiplicity of 1.

Copyright © 2013 Pearson Education, Inc.

Page 133: JUDITH A. PENNA -  · PDF fileSTUDENT’S SOLUTIONS MANUAL JUDITH A. PENNA Indiana University Purdue University Indianapolis COLLEGE ALGEBRA: GRAPHS AND MODELS FIFTH EDITION

130 Chapter 4: Polynomial and Rational Functions

31. f(x) = (x2 − 9)3 = [(x + 3)(x− 3)]3 = (x + 3)3(x− 3)3

To solve f(x) = 0 we use the principle of zero products,solving x+ 3 = 0 and x− 3 = 0. The zeros of f(x) are −3and 3.

The factors x+3 and x− 3 each occur three times so eachzero has a multiplicity of 3.

33. f(x) = x3(x− 1)2(x + 4)

To solve f(x) = 0 we use the principle of zero products,solving x = 0, x− 1 = 0, and x+ 4 = 0. The zeros of f(x)are 0, 1, and −4.

The factor x occurs three times. Thus the zero 0 has amultiplicity of three.

The factor x − 1 occurs twice. Thus the zero 1 has amultiplicity of two.

The factor x + 4 occurs only one time. Thus the zero −4has a multiplicity of one.

35. f(x) = −8(x− 3)2(x + 4)3x4

To solve f(x) = 0 we use the principle of zero products,solving x− 3 = 0, x+ 4 = 0, and x = 0. The zeros of f(x)are 3, −4, and 0.

The factor x − 3 occurs twice. Thus the zero 3 has amultiplicity of 2.

The factor x+4 occurs three times. Thus the zero −4 hasa multiplicity of 3.

The factor x occurs four times. Thus the zero 0 has amultiplicity of 4.

37. f(x) = x4 − 4x2 + 3

We factor as follows:f(x) = (x2 − 3)(x2 − 1)

= (x−√3)(x +

√3)(x− 1)(x + 1)

The zeros of the function are√

3, −√3, 1, and −1. Each

has a multiplicity of 1.

39. f(x) = x3 + 3x2 − x− 3

We factor by grouping:

f(x) = x2(x + 3) − (x + 3)

= (x2 − 1)(x + 3)

= (x− 1)(x + 1)(x + 3)

The zeros of the function are 1, −1, and −3. Each has amultiplicity of 1.

41. f(x) = 2x3 − x2 − 8x + 4

= x2(2x− 1) − 4(2x− 1)

= (2x− 1)(x2 − 4)

= (2x− 1)(x + 2)(x− 2)

The zeros of the function are12, −2, and 2. Each has a

multiplicity of 1.

43. Graph y = x3 − 3x − 1 and use the Zero feature threetimes. The real-number zeros are about −1.532, −0.347,and 1.879.

45. Graph y = x4 − 2x2 and use the Zero feature three times.The real-number zeros are about −1.414, 0, and 1.414.

47. Graph y = x3 − x and use the Zero feature three times.The real-number zeros are −1, 0, and 1.

49. Graph y = x8 + 8x7 − 28x6 − 56x5 + 70x4 + 56x3 − 28x2 −8x + 1 and use the Zero feature eight times. The real-number zeros are about −10.153, −1.871, −0.821, −0.303,0.098, 0.535, 1.219, and 3.297.

51. g(x) = x3 − 1.2x + 1

Graph the function and use the Zero, Maximum, and Min-imum features.

Zero: −1.386

Relative maximum: 1.506 when x ≈ −0.632

Relative minimum: 0.494 when x ≈ 0.632

Range: (−∞,∞)

53. f(x) = x6 − 3.8

Graph the function and use the Zero and Minimum fea-tures.

Zeros: −1.249, 1.249

There is no relative maximum.

Relative minimum: −3.8 when x = 0

Range: [−3.8,∞)

55. f(x) = x2 + 10x− x5

Graph the function and use the Zero, Maximum, and Min-imum features.

Zeros: −1.697, 0, 1.856

Relative maximum: 11.012 when x ≈ 1.258

Relative minimum: −8.183 when x ≈ −1.116

Range: (−∞,∞)

57. Graphing the function, we see that the graph touches thex-axis at (3, 0) but does not cross it, so the statement isfalse.

59. Graphing the function, we see that the statement is true.

61. For 1995, x = 1995 − 1990 = 5.

f(5) = −0.056316(5)4 − 19.500154(5)3 + 584.892054(5)2−1518.5717(5)+94, 299.1990≈98, 856 twin births

For 2005, x = 2005 − 1990 = 15.

f(15)=−0.056316(15)4−19.500154(15)3+584.892054(15)2−1518.5717(15)+94, 299.1990≈134, 457 twin births

63. d(3)=0.010255(3)3−0.340119(3)2+7.397499(3)+6.618361≈26 yr

d(12)=0.010255(12)3−0.340119(12)2+7.397499(12)+6.618361≈64 yr

d(16)=0.010255(16)3−0.340119(16)2+7.397499(16)+6.618361≈80 yr

Copyright © 2013 Pearson Education, Inc.

Page 134: JUDITH A. PENNA -  · PDF fileSTUDENT’S SOLUTIONS MANUAL JUDITH A. PENNA Indiana University Purdue University Indianapolis COLLEGE ALGEBRA: GRAPHS AND MODELS FIFTH EDITION

Exercise Set 4.1 131

65. We substitute 294 for s(t) and solve for t.

294 = 4.9t2 + 34.3t

0 = 4.9t2 + 34.3t− 294

t =−b±√

b2 − 4ac2a

=−34.3 ±√(34.3)2 − 4(4.9)(−294)

2(4.9)

=−34.3 ±√

6938.899.8

t = 5 or t = −12

Only the positive number has meaning in the situation. Itwill take the stone 5 sec to reach the ground.

67. For 2002, x = 2002 − 2000 = 2.

h(2) = 56.8328(2)4 − 1554.7494(2)3 + 10, 451.8211(2)2 −5655.7692(2) + 140, 589.1608 ≈ $159, 556

For 2005, x = 2005 − 2000 = 5.

h(5) = 56.8328(5)4 − 1554.7494(5)3 + 10, 451.8211(5)2 −5655.7692(5) + 140, 589.1608 ≈ $214, 783

For 2008, x = 2008 − 2000 = 8.

h(8) = 56.8328(8)4 − 1554.7494(8)3 + 10, 451.8211(8)2 −5655.7692(8) + 140, 589.1608 ≈ $201, 015

For 2009, x = 2009 − 2000 = 9.

h(9) = 56.8328(9)4 − 1554.7494(9)3 + 10, 451.8211(9)2 −5655.7692(9) + 140, 589.1608 ≈ $175, 752

69. A = P (1 + i)t

9039.75 = 8000(1 + i)2 Substituting9039.758000

= (1 + i)2

±1.063 ≈ 1 + i Taking the square rooton both sides−1 ± 1.063 ≈ i

−1 + 1.063 ≈ i or −1 − 1.063 ≈ i

0.063 ≈ i or −2.063 ≈ i

Only the positive result has meaning in this application.The interest rate is about 0.063, or 6.3%.

71. The sales drop and then rise. This suggests that aquadratic function that opens up might fit the data. Thus,we choose (b).

73. The sales rise and then drop. This suggests that aquadratic function that opens down might fit the data.Thus, we choose (c).

75. The sales fall steadily. Thus, we choose (a), a linear func-tion.

77. a) For each function, x represents the number of yearsafter 2000.

Quadratic: y = −342.2575758x2 + 2687.051515x +17, 133.10909; R2 ≈ 0.9388

Cubic: y = −31.88461538x3 + 88.18473193x2 +1217.170746x + 17, 936.6014; R2 ≈ 0.9783

Quartic: y = 3.968531469x4 − 103.3181818x3 +489.0064103x2 + 502.8350816x + 18, 108.04196;R2 ≈ 0.9815

Using the R2-values, we see that the quartic func-tion provides the best fit.

b) In 2010, x = 2010 − 2000 = 10. Evaluating thequartic function for x = 10, we estimate that therewere about 8404 U.S. foreign adoptions in 2010.

79. a) For each function x represents the number of yearsafter 1975 and y is in billions of dollars.

Cubic: y = −0.0029175919x3 + 0.1195127076x2 −0.5287827297x + 3.087289783; R2 ≈ 0.9023Quartic: y = −0.0001884254x4 + 0.0099636973x3 −0.1550252422x2 + 1.300895521x + 1.77274528;R2 ≈ 0.9734

Using the R2-values, we see that the quartic func-tion provides the better fit.

b) In 1988, x = 1988 − 1975 = 13. Evaluating thequartic function for x = 13, we estimate that therevenue was about $8.994 billion in 1988.

In 2002, x = 2002 − 1975 = 27. Evaluating thequartic function for x = 27, we estimate that therevenue was about $19.862 billion in 2002.

In 2010, x = 2010 − 1975 = 35. Evaluating thequartic function for x = 35, we estimate that therevenue was about $1.836 billion in 2010.

81. d =√

(x2 − x1)2 + (y2 − y1)2

=√

[−1 − (−5)]2 + (0 − 3)2

=√

42 + (−3)2 =√

16 + 9

=√

25 = 5

83. (x− 3)2 + (y + 5)2 = 49

(x− 3)2 + [y − (−5)]2 = 72

Center: (3,−5); radius: 7

85. 2y − 3 ≥ 1 − y + 5

2y − 3 ≥ 6 − y Collecting like terms

3y − 3 ≥ 6 Adding y

3y ≥ 9 Adding 3

y ≥ 3 Dividing by 3

The solution set is {y|y ≥ 3}, or [3,∞).

87. |x + 6| ≥ 7

x + 6 ≤ −7 or x + 6 ≥ 7

x ≤ −13 or x ≥ 1

The solution set is {x|x ≤ −13 or x ≥ 1}, or(−∞,−13] ∪ [1,∞).

89. f(x) = (x5 − 1)2(x2 + 2)3

The leading term of (x5 −1)2 is (x5)2, or x10. The leadingterm of (x2 + 2)3 is (x2)3, or x6. Then the leading term off(x) is x10 · x6, or x16, and the degree of f(x) is 16.

Copyright © 2013 Pearson Education, Inc.

Page 135: JUDITH A. PENNA -  · PDF fileSTUDENT’S SOLUTIONS MANUAL JUDITH A. PENNA Indiana University Purdue University Indianapolis COLLEGE ALGEBRA: GRAPHS AND MODELS FIFTH EDITION

y

x�4 �2 2 4

�4

�2

2

4f(x) � �x3 � 2x2

132 Chapter 4: Polynomial and Rational Functions

Exercise Set 4.2

1. f(x) = x5 − x2 + 6

a) This function has degree 5, so its graph can have atmost 5 real zeros.

b) This function has degree 5, so its graph can have atmost 5 x-intercepts.

c) This function has degree 5, so its graph can have atmost 5 − 1, or 4, turning points.

3. f(x) = x10 − 2x5 + 4x− 2

a) This function has degree 10, so its graph can haveat most 10 real zeros.

b) This function has degree 10, so its graph can haveat most 10 x-intercepts.

c) This function has degree 10, so its graph can haveat most 10 − 1, or 9, turning points.

5. f(x) = −x− x3 = −x3 − x

a) This function has degree 3, so its graph can have atmost 3 real zeros.

b) This function has degree 3, so its graph can have atmost 3 x-intercepts.

c) This function has degree 3, so its graph can have atmost 3 − 1, or 2, turning points.

7. f(x) =14x2 − 5

The leading term is14x2. The sign of the leading coef-

ficient,14, is positive and the degree, 2, is even, so we

would choose either graph (b) or graph (d). Note alsothat f(0) = −5, so the y-intercept is (0,−5). Thus, graph(d) is the graph of this function.

9. f(x) = x5 − x4 + x2 + 4

The leading term is x5. The sign of the leading coefficient,1, is positive and the degree, 5, is odd. Thus, graph (f) isthe graph of this function.

11. f(x) = x4 − 2x3 + 12x2 + x− 20

The leading term is x4. The sign of the leading coefficient,1, is positive and the degree, 4, is even, so we would chooseeither graph (b) or graph (d). Note also that f(0) = −20,so the y-intercept is (0,−20). Thus, graph (b) is the graphof this function.

13. f(x) = −x3 − 2x2

1. The leading term is −x3. The degree, 3, is odd andthe leading coefficient, −1, is negative so as x → ∞,f(x) → −∞ and as x → −∞, f(x) → ∞.

2. We solve f(x) = 0.

−x3 − 2x2 = 0

−x2(x + 2) = 0

−x2 = 0 or x + 2 = 0

x2 = 0 or x = −2

x = 0 or x = −2

The zeros of the function are 0 and −2, so the x-intercepts of the graph are (0, 0) and (−2, 0).

3. The zeros divide the x-axis into 3 intervals, (−∞,−2),(−2, 0), and (0,∞). We choose a value for x from eachinterval and find f(x). This tells us the sign of f(x)for all values of x in that interval.In (−∞,−2), test −3:

f(−3) = −(−3)3 − 2(−3)2 = 9 > 0

In (−2, 0), test −1:

f(−1) = −(−1)3 − 2(−1)2 = −1 < 0

In (0,∞), test 1:

f(1) = −13 − 2 · 12 = −3 < 0Thus the graph lies above the x-axis on (−∞,−2)and below the x-axis on (−2, 0) and (0,∞). We alsoknow the points (−3, 9), (−1,−1), and (1,−3) areon the graph.

4. From Step 2 we see that the y-intercept is (0, 0).

5. We find additional points on the graph and thendraw the graph.

x f(x)

−2.5 3.125

−1.5 −1.125

1.5 −7.875

6. Checking the graph as described on page 317 in thetext, we see that it appears to be correct.

15. h(x) = x2 + 2x− 3

1. The leading term is x2. The degree, 2, is evenand leading coefficient, 1, is positive so as x → ∞,h(x) → ∞ and as x → −∞, h(x) → ∞.

2. We solve h(x) = 0.

x2 + 2x− 3 = 0

(x + 3)(x− 1) = 0

x + 3 = 0 or x− 1 = 0

x = −3 or x = 1

The zeros of the function are −3 and 1, so the x-intercepts of the graph are (−3, 0) and (1, 0).

3. The zeros divide the x-axis into 3 intervals, (−∞,−3),(−3, 1), and (1,∞). We choose a value for x from eachinterval and find h(x). This tells us the sign of h(x)for all values of x in that interval.In (−∞,−3), test −4:

h(−4) = (−4)2 + 2(−4) − 3 = 5 > 0

In (−3, 1), test 0:

h(0) = 02 + 2 · 0 − 3 = −3 < 0

Copyright © 2013 Pearson Education, Inc.

Page 136: JUDITH A. PENNA -  · PDF fileSTUDENT’S SOLUTIONS MANUAL JUDITH A. PENNA Indiana University Purdue University Indianapolis COLLEGE ALGEBRA: GRAPHS AND MODELS FIFTH EDITION

h(x) � x2 � 2x � 3

y

x�4 �2 2 4

�4

�2

2

4

h(x) � x5 � 4x3

y

x�4 �2 2 4

�8

�4

4

8

y

x�4 �2 42 6

�8

�4

�12

4

8

h(x) � x(x � 4)(x � 1)(x � 2)

Exercise Set 4.2 133

In (1,∞), test 2:

h(2) = 22 + 2 · 2 − 3 = 5 > 0

Thus the graph lies above the x-axis on (−∞,−3)and on (1,∞). It lies below the x-axis on (−3, 1).We also know the points (−4, 5), (0,−3), and (2, 5)are on the graph.

4. From Step 3 we see that the y-intercept is (0,−3).

5. We find additional points on the graph and thendraw the graph.

x h(x)

−2 −3

−1 −4

3 12

6. Checking the graph as described on page 317 in thetext, we see that it appears to be correct.

17. h(x) = x5 − 4x3

1. The leading term is x5. The degree, 5, is odd andthe leading coefficient, 1, is positive so as x → ∞,h(x) → ∞ and as x → −∞, h(x) → −∞.

2. We solve h(x) = 0.

x5 − 4x3 = 0

x3(x2 − 4) = 0x3(x + 2)(x− 2) = 0

x3 = 0 or x + 2 = 0 or x− 2 = 0

x = 0 or x = −2 or x = 2

The zeros of the function are 0, −2, and 2 so the x-intercepts of the graph are (0, 0), (−2, 0), and (2, 0).

3. The zeros divide the x-axis into 4 intervals, (−∞,−2),(−2, 0), (0, 2), and (2,∞). We choose a value for xfrom each interval and find h(x). This tells us thesign of h(x) for all values of x in that interval.In (−∞,−2), test −3:

h(−3) = (−3)5 − 4(−3)3 = −135 < 0

In (−2, 0), test −1:

h(−1) = (−1)5 − 4(−1)3 = 3 > 0In (0, 2), test 1:

h(1) = 15 − 4 · 13 = −3 < 0

In (2,∞), test 3:

h(3) = 35 − 4 · 33 = 135 > 0Thus the graph lies below the x-axis on (−∞,−2)and on (0, 2). It lies above the x-axis on (−2, 0) andon (2,∞). We also know the points (−3,−135),(−1, 3), (1,−3), and (3, 135) are on the graph.

4. From Step 2 we see that the y-intercept is (0, 0).

5. We find additional points on the graph and thendraw the graph.

x h(x)

−2.5 −35.2

−1.5 5.9

1.5 −5.9

2.5 35.2

6. Checking the graph as described on page 317 in thetext, we see that it appears to be correct.

19. h(x) = x(x− 4)(x + 1)(x− 2)1. The leading term is x ·x ·x ·x, or x4. The degree, 4,

is even and the leading coefficient, 1, is positive soas x → ∞, h(x) → ∞ and as x → −∞, h(x) → ∞.

2. We see that the zeros of the function are 0, 4, −1,and 2 so the x-intercepts of the graph are (0, 0),(4, 0), (−1, 0), and (2, 0).

3. The zeros divide the x-axis into 5 intervals, (−∞,−1),(−1, 0), (0, 2), (2, 4), and (4,∞). We choose a valuefor x from each interval and find h(x). This tells usthe sign of h(x) for all values of x in that interval.In (−∞,−1), test −2:

h(−2) = −2(−2 − 4)(−2 + 1)(−2 − 2) = 48 > 0

In (−1, 0), test −0.5:

h(−0.5) = (−0.5)(−0.5 − 4)(−0.5 + 1)(−0.5 − 2) =−2.8125 < 0

In (0, 2), test 1:

h(1) = 1(1 − 4)(1 + 1)(1 − 2) = 6 > 0

In (2, 4), test 3:

h(3) = 3(3 − 4)(3 + 1)(3 − 2) = −12 < 0In (4,∞), test 5:

h(5) = 5(5 − 4)(5 + 1)(5 − 2) = 90 > 0Thus the graph lies above the x-axis on (−∞,−1),(0, 2), and (4,∞). It lies below the x-axis on (−1, 0)and on (2, 4). We also know the points (−2, 48),(−0.5,−2.8125), (1, 6), (3,−12), and (5, 90) are onthe graph.

4. From Step 2 we see that the y-intercept is (0, 0).5. We find additional points on the graph and then

draw the graph.

x h(x)

−1.5 14.4

1.5 4.7

2.5 −6.6

4.5 30.9

Copyright © 2013 Pearson Education, Inc.

Page 137: JUDITH A. PENNA -  · PDF fileSTUDENT’S SOLUTIONS MANUAL JUDITH A. PENNA Indiana University Purdue University Indianapolis COLLEGE ALGEBRA: GRAPHS AND MODELS FIFTH EDITION

y

x�4 �2 2 4

�4

�2

2

4

g(x) � ��x3 � �x214

34

y

2 4

2

4

�4 �2

�4

�2

x

g(x) � �x 4 � 2x 3

134 Chapter 4: Polynomial and Rational Functions

6. Checking the graph as described on page 317 in thetext, we see that it appears to be correct.

21. g(x) = −14x3 − 3

4x2

1. The leading term is −14x3. The degree, 3, is odd

and the leading coefficient, −14, is negative so as

x → ∞, g(x) → −∞ and as x → −∞, g(x) → ∞.

2. We solve g(x) = 0.

−14x3 − 3

4x2 = 0

−14x2(x + 3) = 0

−14x2 = 0 or x + 3 = 0

x2 = 0 or x = −3

x = 0 or x = −3

The zeros of the function are 0 and −3, so the x-intercepts of the graph are (0, 0) and (−3, 0).

3. The zeros divide the x-axis into 3 intervals, (−∞,−3),(−3, 0), and (0,∞). We choose a value for x from eachinterval and find g(x). This tells us the sign of g(x)for all values of x in that interval.In (−∞,−3), test −4:

g(−4) = −14(−4)3 − 3

4(−4)2 = 4 > 0

In (−3, 0), test −1:

g(−1) = −14(−1)3 − 3

4(−1)2 = −1

2< 0

In (0,∞), test 1:

g(1) = −14· 13 − 3

4· 12 = −1 < 0

Thus the graph lies above the x-axis on (−∞,−3)and below the x-axis on (−3, 0) and on (0,∞).

We also know the points (−4, 4),(− 1,−1

2

), and

(1,−1) are on the graph.

4. From Step 2 we see that the y-intercept is (0, 0).

5. We find additional points on the graph and thendraw the graph.

x g(x)

−2 −1

2 −5

2.5 −8.6

6. Checking the graph as described on page 317 in thetext, we see that it appears to be correct.

23. g(x) = −x4 − 2x3

1. The leading term is −x4. The degree, 4, is even andthe leading coefficient, −1, is negative so as x → ∞,g(x) → −∞ and as x → −∞, g(x) → −∞.

2. We solve f(x) = 0.

−x4 − 2x3 = 0

−x3(x + 2) = 0

−x3 = 0 or x + 2 = 0

x = 0 or x = −2

The zeros of the function are 0 and −2, so the x-intercepts of the graph are (0, 0) and (−2, 0).

3. The zeros divide the x-axis into 3 intervals, (−∞,−2),(−2, 0), and (0,∞). We choose a value for x from eachinterval and find g(x). This tells us the sign of g(x)for all values of x in that interval.In (−∞,−2), test −3:

g(−3) = −(−3)4 − 2(−3)3 = −27 < 0

In (−2, 0), test −1:

g(−1) = −(−1)4 − 2(−1)3 = 1 > 0

In (0,∞), test 1:

g(1) = −(1)4 − 2(1)3 = −3 < 0Thus the graph lies below the x-axis on (−∞,−2)and (0,∞) and above the x-axis on (−2, 0). We alsoknow the points (−3,−27), (−1, 1), and (1,−3) areon the graph.

4. From Step 2 we see that the y-intercept is (0, 0).

5. We find additional points on the graph and thendraw the graph.

x g(x)

−2.5 −7.8

−1.5 1.7

0.5 −0.3

1.5 −11.8

2 −32

6. Checking the graph as described on page 317 in thetext, we see that it appears to be correct.

25. f(x) = −12(x− 2)(x + 1)2(x− 1)

1. The leading term is −12· x · x · x · x, or −1

2x4. The

degree, 4, is even and the leading coefficient, −12, is

negative so as x → ∞, f(x) → −∞ and as x → −∞,f(x) → −∞.

2. We solve f(x) = 0.

−12(x− 2)(x + 1)2(x− 1) = 0

x− 2 = 0 or (x + 1)2 = 0 or x− 1 = 0

x = 2 or x + 1 = 0 or x = 1

x = 2 or x = −1 or x = 1

The zeros of the function are 2, −1, and 1, so the x-intercepts of the graph are (2, 0), (−1, 0), and (1, 0).

Copyright © 2013 Pearson Education, Inc.

Page 138: JUDITH A. PENNA -  · PDF fileSTUDENT’S SOLUTIONS MANUAL JUDITH A. PENNA Indiana University Purdue University Indianapolis COLLEGE ALGEBRA: GRAPHS AND MODELS FIFTH EDITION

2 4

2

4

�4 �2

�4

�2

x

y

12f(x) � ��(x � 2)(x � 1)2(x � 1)

�6 �4 �2 x

y

2 4

80

40

�40

�80

g(x) � �x(x � 1)2(x � 4)2

Exercise Set 4.2 135

3. The zeros divide the x-axis into 4 intervals, (−∞,−1),(−1, 1), (1, 2), and (2,∞). We choose a value for xfrom each interval and find f(x). This tells us thesign of f(x) for all values of x in that interval.In (−∞,−1), test −2:

f(−2) = −12(−2 − 2)(−2 + 1)2(−2 − 1) = −6 < 0

In (−1, 1), test 0:

f(0) = −12(0 − 2)(0 + 1)2(0 − 1) = −1 < 0

In (1, 2), test 1.5:

f(1.5) = −12(1.5 − 2)(1.5 + 1)2(1.5 − 1) = 0.78125>0

In (2,∞), test 3:

f(3) = −12(3 − 2)(3 + 1)2(3 − 1) = −16 < 0

Thus the graph lies below the x-axis on (−∞,−1),(−1, 1), and (2,∞) and above the x-axis on (1, 2). Wealso know the points (−2,−6), (0,−1), (1.5, 0.78125),and (3,−16) are on the graph.

4. From Step 2 we know that f(0) = −1 so the y-intercept is (0,−1).

5. We find additional points on the graph and then drawthe graph.

x f(x)

−3 −40

−0.5 −0.5

0.5 −0.8

1.5 0.8

6. Checking the graph as described on page 317 in thetext, we see that it appears to be correct.

27. g(x) = −x(x− 1)2(x + 4)2

1. The leading term is −x · x · x · x · x, or −x5. Thedegree, 5, is odd and the leading coefficient, −1, isnegative so as x → ∞, g(x) → −∞ and as x → −∞,g(x) → ∞.

2. We solve g(x) = 0.

−x(x− 1)2(x + 4)2 = 0

−x = 0 or (x− 1)2 = 0 or (x + 4)2 = 0

x = 0 or x− 1 = 0 or x + 4 = 0

x = 0 or x = 1 or x = −4

The zeros of the function are 0, 1, and −4, so thex-intercepts are (0, 0), (1, 0), and (−4, 0).

3. The zeros divide the x-axis into 4 intervals, (−∞,−4),(−4, 0), (0, 1), and (1,∞). We choose a value for xfrom each interval and find g(x). This tells us thesign of g(x) for all values of x in that interval.

In (−∞,−4), test −5:

g(−5) = −(−5)(−5 − 1)2(−5 + 4)2 = 180 > 0

In (−4, 0), test −1:

g(−1) = −(−1)(−1 − 1)2(−1 + 4)2 = 36 > 0

In (0, 1), test 0.5:

g(0.5) = −0.5(0.5 − 1)2(−0.5 + 4)2 = −2.53125 < 0In (1,∞), test 2:

g(2) = −2(2 − 1)2(2 + 4)2 = −72 < 0

Thus the graph lies above the x-axis on (−∞,−4)and on (−4, 0) and below the x-axis on (0, 1) and(1,∞). We also know the points (−5, 180), (−1, 36),(0.5,−2.53125), and (2,−72) are on the graph.

4. From Step 2 we see that the y-intercept is (0,0).

5. We find additional points on the graph and thendraw the graph.

x g(x)

−3 4.8

−2 72

1.5 −11.3

6. Checking the graph as described on page 317 in thetext, we see that it appears to be correct.

29. f(x) = (x− 2)2(x + 1)4

1. The leading term is x ·x ·x ·x ·x ·x, or x6. The degree,6, is even and the leading coefficient, 1, is positive soas x → ∞, f(x) → ∞ and as x → −∞, f(x) → ∞.

2. We see that the zeros of the function are 2 and −1 sothe x-intercepts of the graph are (2, 0) and (−1, 0).

3. The zeros divide the x-axis into 3 intervals, (−∞,−1),(−1, 2), and (2,∞). We choose a value for x from eachinterval and find f(x). This tells us the sign of f(x)for all values of x in that interval.In (−∞,−1), test −2:

f(−2) = (−2 − 2)2(−2 + 1)4 = 16 > 0

In (−1, 2), test 0:

f(0) = (0 − 2)2(0 + 1)4 = 4 > 0In (2,∞), test 3:

f(3) = (3 − 2)2(3 + 1)4 = 256 > 0Thus the graph lies above the x-axis on all 3 intervals.We also know the points (−2, 16), (0, 4), and (3, 256)are on the graph.

4. From Step 3 we know that f(0) = 4 so the y-interceptis (0, 4).

5. We find additional points on the graph and then drawthe graph.

Copyright © 2013 Pearson Education, Inc.

Page 139: JUDITH A. PENNA -  · PDF fileSTUDENT’S SOLUTIONS MANUAL JUDITH A. PENNA Indiana University Purdue University Indianapolis COLLEGE ALGEBRA: GRAPHS AND MODELS FIFTH EDITION

y

x�4 �2 2 4

�10

20

10

30

f(x) � (x � 2)2(x � 1)4

y

x�4 �2 2 4

�8

�12

�16

�4

4

g(x) � �(x � 1)4

y

x�4 �2 2 4

�4

�2

2

4

h(x) � x3 � 3x2 � x � 3

136 Chapter 4: Polynomial and Rational Functions

x f(x)

−1.5 0.8

−0.5 0.4

1 16

1.5 9.8

6. Checking the graph as described on page 317 in thetext, we see that it appears to be correct.

31. g(x) = −(x− 1)4

1. The leading term is −1 · x · x · x · x, or −x4. Thedegree, 4, is even and the leading coefficient, −1, isnegative so as x → ∞, g(x) → −∞ and as x → −∞,g(x) → −∞.

2. We see that the zero of the function is 1, so the x-intercept is (1, 0).

3. The zero divides the x-axis into 2 intervals, (−∞, 1)and (1,∞). We choose a value for x from each intervaland find g(x). This tells us the sign of g(x) for allvalues of x in that interval.In (−∞, 1), test 0:

g(0) = −(0 − 1)4 = −1 < 0

In (1,∞), test 2:

g(2) = −(2 − 1)4 = −1 < 0Thus the graph lies below the x-axis on both intervals.We also know the points (0,−1) and (2,−1) are onthe graph.

4. From Step 3 we know that g(0) = −1 so the y-intercept is (0,−1).

5. We find additional points on the graph and then drawthe graph.

x g(x)

−1 −16

−0.5 −5.1

1.5 0.1

3 −16

6. Checking the graph as described on page 317 in thetext, we see that it appears to be correct.

33. h(x) = x3 + 3x2 − x− 3

1. The leading term is x3. The degree, 3, is odd andthe leading coefficient, 1, is positive so as x → ∞,h(x) → ∞ and as x → −∞, h(x) → −∞.

2. We solve h(x) = 0.

x3 + 3x2 − x− 3 = 0

x2(x + 3) − (x + 3) = 0

(x + 3)(x2 − 1) = 0

(x + 3)(x + 1)(x− 1) = 0

x + 3 = 0 or x + 1 = 0 or x− 1 = 0

x = −3 or x = −1 or x = 1

The zeros of the function are −3, −1, and 1 so the x-intercepts of the graph are (−3, 0), (−1, 0), and (1, 0).

3. The zeros divide the x-axis into 4 intervals, (−∞,−3),(−3,−1), (−1, 1), and (1,∞). We choose a value forx from each interval and find h(x). This tells us thesign of h(x) for all values of x in that interval.In (−∞,−3), test −4:

h(−4) = (−4)3 + 3(−4)2 − (−4) − 3 = −15 < 0

In (−3,−1), test −2:

h(−2) = (−2)3 + 3(−2)2 − (−2) − 3 = 3 > 0

In (−1, 1), test 0:

h(0) = 03 + 3 · 02 − 0 − 3 = −3 < 0

In (1,∞), test 2:

h(2) = 23 + 3 · 22 − 2 − 3 = 15 > 0Thus the graph lies below the x-axis on (−∞,−3) andon (−1, 1) and above the x-axis on (−3,−1) and on(1,∞). We also know the points (−4,−15), (−2, 3),(0,−3), and (2, 15) are on the graph.

4. From Step 3 we know that h(0) = −3 so the y-intercept is (0,−3).

5. We find additional points on the graph and then drawthe graph.

x h(x)

−4.5 −28.9

−2.5 2.6

0.5 −2.6

2.5 28.9

6. Checking the graph as described on page 317 in thetext, we see that it appears to be correct.

35. f(x) = 6x3 − 8x2 − 54x + 72

1. The leading term is 6x3. The degree, 3, is odd andthe leading coefficient, 6, is positive so as x → ∞,f(x) → ∞ and as x → −∞, f(x) → −∞.

Copyright © 2013 Pearson Education, Inc.

Page 140: JUDITH A. PENNA -  · PDF fileSTUDENT’S SOLUTIONS MANUAL JUDITH A. PENNA Indiana University Purdue University Indianapolis COLLEGE ALGEBRA: GRAPHS AND MODELS FIFTH EDITION

y

x�4 �2 2 4

�100

�50

50

100

f(x) � 6x3 � 8x2 � 54x � 72

y

x�4 �2 2 4

�4

�2

2

4

�x � 3, for x � �2,4, for �2 � x � 1,

�x3 , for x � 112

g(x) �

Exercise Set 4.2 137

2. We solve f(x) = 0.

6x3 − 8x2 − 54x + 72 = 0

2(3x3 − 4x2 − 27x + 36) = 0

2[x2(3x− 4) − 9(3x− 4)] = 0

2(3x− 4)(x2 − 9) = 0

2(3x− 4)(x + 3)(x− 3) = 0

3x− 4 = 0 or x + 3 = 0 or x− 3 = 0

x =43

or x = −3 or x = 3

The zeros of the function are43, −3, and 3, so the x-

intercepts of the graph are(

43, 0)

, (−3, 0), and (3, 0).

3. The zeros divide the x-axis into 4 intervals, (−∞,−3),(− 3,

43

),(

43, 3)

, and (3,∞). We choose a value for

x from each interval and find f(x). This tells us thesign of f(x) for all values of x in that interval.In (−∞,−3), test −4:

f(−4) = 6(−4)3 − 8(−4)2 − 54(−4) + 72 = −224 < 0

In(− 3,

43

), test 0:

f(0) = 6 · 03 − 8 · 02 − 54 · 0 + 72 = 72 > 0

In(

43, 3)

, test 2:

f(2) = 6 · 23 − 8 · 22 − 54 · 2 + 72 = −20 < 0

In (3,∞), test 4:

f(4) = 6 · 43 − 8 · 42 − 54 · 4 + 72 = 112 > 0Thus the graph lies below the x-axis on (−∞,−3) and

on(

43, 3)

and above the x-axis on(− 3,

43

)and on

(3,∞). We also know the points (−4,−224), (0, 72),(2,−20), and (4, 112) are on the graph.

4. From Step 3 we know that f(0) = 72 so the y-intercept is (0, 72).

5. We find additional points on the graph and then drawthe graph.

x f(x)

−1 112

1 16

3.5 42.25

6. Checking the graph as described on page 317 in thetext, we see that it appears to be correct.

37. We graph g(x) = −x+3 for x ≤ −2, g(x)=4 for 2 < x < 1,

and g(x) =12x3 for x ≥ 1.

39. f(−5) = (−5)3 + 3(−5)2 − 9(−5) − 13 = −18

f(−4) = (−4)3 + 3(−4)2 − 9(−4) − 13 = 7

By the intermediate value theorem, since f(−5) and f(−4)have opposite signs then f(x) has a zero between −5 and−4.

41. f(−3) = 3(−3)2 − 2(−3) − 11 = 22

f(−2) = 3(−2)2 − 2(−2) − 11 = 5

Since both f(−3) and f(−2) are positive, we cannot usethe intermediate value theorem to determine if there is azero between −3 and −2.

43. f(2) = 24 − 2 · 22 − 6 = 2

f(3) = 34 − 2 · 32 − 6 = 57

Since both f(2) and f(3) are positive, we cannot use theintermediate value theorem to determine if there is a zerobetween 2 and 3.

45. f(4) = 43 − 5 · 42 + 4 = −12

f(5) = 53 − 5 · 52 + 4 = 4

By the intermediate value theorem, since f(4) and f(5)have opposite signs then f(x) has a zero between 4 and 5.

47. The graph of y = x, or y = x+0, has y-intercept (0, 0), so(d) is the correct answer.

49. The graph of y − 2x = 6, or y = 2x + 6, has y-intercept(0, 6), so (e) is the correct answer.

51. The graph of y = 1 − x, or y = −x + 1, has y-intercept(0, 1), so (b) is the correct answer.

53. 2x− 12

= 4 − 3x

5x− 12

= 4 Adding 3x

5x =92

Adding12

x =15· 92

Multiplying by15

x =910

The solution is910

.

Copyright © 2013 Pearson Education, Inc.

Page 141: JUDITH A. PENNA -  · PDF fileSTUDENT’S SOLUTIONS MANUAL JUDITH A. PENNA Indiana University Purdue University Indianapolis COLLEGE ALGEBRA: GRAPHS AND MODELS FIFTH EDITION

138 Chapter 4: Polynomial and Rational Functions

55. 6x2 − 23x− 55 = 0

(3x + 5)(2x− 11) = 0

3x + 5 = 0 or 2x− 11 = 0

3x = −5 or 2x = 11

x = −53

or x =112

The solutions are −53

and112

.

Exercise Set 4.3

1. a) x3 − 7x2 + 8x + 16x + 1 x4 − 6x3 + x2 + 24x− 20

x4 + x3

− 7x3 + x2

− 7x3 − 7x2

8x2 + 24x8x2 + 8x

16x− 2016x+ 16

− 4

Since the remainder is not 0, x+1 is not a factor off(x).

b) x3 − 4x2 − 7x + 10x− 2 x4 − 6x3 + x2 + 24x− 20

x4 − 2x3

− 4x3 + x2

− 4x3 + 8x2

− 7x2 + 24x− 7x2 + 14x

10x− 2010x− 20

0

Since the remainder is 0, x− 2 is a factor of f(x).

c) x3 − 11x2 + 56x − 256x + 5 x4 − 6x3 + x2 + 24x − 20

x4 + 5x3

− 11x3 + x2

− 11x3 − 55x2

56x2 + 24x56x2 + 280x

− 256x− 20− 256x− 1280

1260

Since the remainder is not 0, x+5 is not a factor off(x).

3. a) x2 + 2x − 3x− 4 x3 − 2x2 − 11x+ 12

x3 − 4x2

2x2 − 11x2x2 − 8x

− 3x + 12− 3x + 12

0

Since the remainder is 0, x− 4 is a factor of g(x).

b) x2 + x − 8x− 3 x3 − 2x2 − 11x+ 12

x3 − 3x2

x2 − 11xx2 − 3x

− 8x + 12− 8x + 24

− 12

Since the remainder is not 0, x− 3 is not a factor ofg(x).

c) x2 − x − 12x− 1 x3 − 2x2 − 11x+ 12

x3 − x2

− x2 − 11x− x2 + x

− 12x+ 12− 12x+ 12

0

Since the remainder is 0, x− 1 is a factor of g(x).

5. x2 − 2x + 4x + 2 x3 + 0x2 + 0x− 8

x3 + 2x2

− 2x2 + 0x− 2x2 − 4x

4x− 84x+ 8

−16

x3 − 8 = (x + 2)(x2 − 2x + 4) − 16

7. x2 − 3x + 2x + 9 x3 + 6x2 − 25x+ 18

x3 + 9x2

− 3x2 − 25x− 3x2 − 27x

2x + 182x + 18

0

x3 + 6x2 − 25x + 18 = (x + 9)(x2 − 3x + 2) + 0

9. x3 − 2x2 + 2x − 4x + 2 x4 + 0x3 − 2x2 + 0x+ 3

x4 + 2x3

− 2x3 − 2x2

− 2x3 − 4x2

2x2 + 0x2x2 + 4x

− 4x+ 3− 4x− 8

11

x4 − 2x2 + 3 = (x + 2)(x3 − 2x2 + 2x− 4) + 11

11. (2x4 + 7x3 + x− 12) ÷ (x + 3)

= (2x4 + 7x3 + 0x2 + x− 12) ÷ [x− (−3)]

−3∣∣ 2 7 0 1 −12

−6 −3 9 −302 1 −3 10 −42

The quotient is 2x3 +x2 − 3x+10. The remainder is −42.

Copyright © 2013 Pearson Education, Inc.

Page 142: JUDITH A. PENNA -  · PDF fileSTUDENT’S SOLUTIONS MANUAL JUDITH A. PENNA Indiana University Purdue University Indianapolis COLLEGE ALGEBRA: GRAPHS AND MODELS FIFTH EDITION

Exercise Set 4.3 139

13. (x3 − 2x2 − 8) ÷ (x + 2)

= (x3 − 2x2 + 0x− 8) ÷ [x− (−2)]

−2∣∣ 1 −2 0 −8

−2 8 −161 −4 8 −24

The quotient is x2 − 4x + 8. The remainder is −24.

15. (3x3 − x2 + 4x− 10) ÷ (x + 1)

= (3x3 − x2 + 4x− 10) ÷ [x− (−1)]

−1∣∣ 3 −1 4 −10

−3 4 −83 −4 8 −18

The quotient is 3x2 − 4x + 8. The remainder is −18.

17. (x5 + x3 − x) ÷ (x− 3)

= (x5 + 0x4 + x3 + 0x2 − x + 0) ÷ (x− 3)

3∣∣ 1 0 1 0 −1 0

3 9 30 90 2671 3 10 30 89 267

The quotient is x4 + 3x3 + 10x2 + 30x + 89.The remainder is 267.

19. (x4 − 1) ÷ (x− 1)

= (x4 + 0x3 + 0x2 + 0x− 1) ÷ (x− 1)

1∣∣ 1 0 0 0 −1

1 1 1 11 1 1 1 0

The quotient is x3 + x2 + x + 1. The remainder is 0.

21. (2x4 + 3x2 − 1) ÷(x− 1

2

)

(2x4 + 0x3 + 3x2 + 0x− 1) ÷(x− 1

2

)12

∣∣ 2 0 3 0 −11 1

274

78

2 1 72

74 − 1

8

The quotient is 2x3 + x2 +72x +

74. The remainder is −1

8.

23. f(x) = x3 − 6x2 + 11x− 6Find f(1).1∣∣ 1 −6 11 −6

1 −5 61 −5 6 0

f(1) = 0

Find f(−2).−2∣∣ 1 −6 11 −6

−2 16 −541 −8 27 −60

f(−2) = −60

Find f(3).3∣∣ 1 −6 11 −6

3 −9 61 −3 2 0

f(3) = 0

25. f(x) = x4 − 3x3 + 2x + 8

Find f(−1).

−1∣∣ 1 −3 0 2 8

−1 4 −4 21 −4 4 −2 10

f(−1) = 10

Find f(4).

4∣∣ 1 −3 0 2 8

4 4 16 721 1 4 18 80

f(4) = 80

Find f(−5).

−5∣∣ 1 −3 0 2 8

−5 40 −200 9901 −8 40 −198 998

f(−5) = 998

27. f(x) = 2x5 − 3x4 + 2x3 − x + 8

Find f(20).

20∣∣ 2 −3 2 0 −1 8

40 740 14, 840 296, 800 5, 935, 9802 37 742 14, 840 296, 799 5, 935, 988

f(20) = 5, 935, 988

Find f(−3).

−3∣∣ 2 −3 2 0 −1 8

−6 27 −87 261 −7802 −9 29 −87 260 −772

f(−3) = −772

29. f(x) = x4 − 16

Find f(2).

2∣∣ 1 0 0 0 −16

2 4 8 161 2 4 8 0

f(2) = 0

Find f(−2).

−2∣∣ 1 0 0 0 −16

−2 4 −8 161 −2 4 −8 0

f(−2) = 0

Find f(3).

3∣∣ 1 0 0 0 −16

3 9 27 811 3 9 27 65

f(3) = 65

Find f(1 −√2).

1−√2∣∣ 1 0 0 0 −16

1−√2 3−2

√2 7−5

√2 17−12

√2

1 1−√2 3−2

√2 7−5

√2 1−12

√2

f(1 −√2) = 1 − 12

√2

Copyright © 2013 Pearson Education, Inc.

Page 143: JUDITH A. PENNA -  · PDF fileSTUDENT’S SOLUTIONS MANUAL JUDITH A. PENNA Indiana University Purdue University Indianapolis COLLEGE ALGEBRA: GRAPHS AND MODELS FIFTH EDITION

140 Chapter 4: Polynomial and Rational Functions

31. f(x) = 3x3 + 5x2 − 6x + 18

If −3 is a zero of f(x), then f(−3) = 0. Find f(−3) usingsynthetic division.

−3∣∣ 3 5 −6 18

−9 12 −183 −4 6 0

Since f(−3) = 0, −3 is a zero of f(x).

If 2 is a zero of f(x), then f(2) = 0. Find f(2) usingsynthetic division.

2∣∣ 3 5 −6 18

6 22 323 11 16 50

Since f(2) �= 0, 2 is not a zero of f(x).

33. h(x) = x4 + 4x3 + 2x2 − 4x− 3

If −3 is a zero of h(x), then h(−3) = 0. Find h(−3) usingsynthetic division.

−3∣∣ 1 4 2 −4 −3

−3 −3 3 31 1 −1 −1 0

Since h(−3) = 0, −3 is a zero of h(x).

If 1 is a zero of h(x), then h(1) = 0. Find h(1) usingsynthetic division.

1∣∣ 1 4 2 −4 −3

1 5 7 31 5 7 3 0

Since h(1) = 0, 1 is a zero of h(x).

35. g(x) = x3 − 4x2 + 4x− 16

If i is a zero of g(x), then g(i) = 0. Find g(i) using syn-thetic division. Keep in mind that i2 = −1.

i∣∣ 1 −4 4 −16

i −4i− 1 3i + 41 −4 + i 3 − 4i −12 + 3i

Since g(i) �= 0, i is not a zero of g(x).

If −2i is a zero of g(x), then g(−2i) = 0. Find g(−2i)using synthetic division. Keep in mind that i2 = −1.

−2i∣∣ 1 −4 4 −16

−2i 8i− 4 161 −4 − 2i 8i 0

Since g(−2i) = 0, −2i is a zero of g(x).

37. f(x) = x3 − 72x2 + x− 3

2If −3 is a zero of f(x), then f(−3) = 0. Find f(−3) usingsynthetic division.

−3∣∣∣ 1 − 7

2 1 − 32

−3 392 − 123

2

1 − 132

412 −63

Since f(−3) �= 0, −3 is not a zero of f(x).

If12

is a zero of f(x), then f

(12

)= 0.

Find f

(12

)using synthetic division.

12

∣∣∣ 1 − 72 1 − 3

212 − 3

2 − 14

1 −3 −12 − 7

4

Since f

(12

)�= 0,

12

is not a zero of f(x).

39. f(x) = x3 + 4x2 + x− 6

Try x− 1. Use synthetic division to see whether f(1) = 0.

1∣∣ 1 4 1 −6

1 5 61 5 6 0

Since f(1) = 0, x − 1 is a factor of f(x). Thus f(x) =(x− 1)(x2 + 5x + 6).

Factoring the trinomial we get

f(x) = (x− 1)(x + 2)(x + 3).

To solve the equation f(x) = 0, use the principle of zeroproducts.

(x− 1)(x + 2)(x + 3) = 0

x− 1 = 0 or x + 2 = 0 or x + 3 = 0

x = 1 or x = −2 or x = −3

The solutions are 1, −2, and −3.

41. f(x) = x3 − 6x2 + 3x + 10

Try x− 1. Use synthetic division to see whether f(1) = 0.

1∣∣ 1 −6 3 10

1 −5 −21 −5 −2 8

Since f(1) �= 0, x− 1 is not a factor of P (x).

Try x+1. Use synthetic division to see whether f(−1) = 0.

−1∣∣ 1 −6 3 10

−1 7 −101 −7 10 0

Since f(−1) = 0, x + 1 is a factor of f(x).

Thus f(x) = (x + 1)(x2 − 7x + 10).

Factoring the trinomial we get

f(x) = (x + 1)(x− 2)(x− 5).

To solve the equation f(x) = 0, use the principle of zeroproducts.

(x + 1)(x− 2)(x− 5) = 0

x + 1 = 0 or x− 2 = 0 or x− 5 = 0

x = −1 or x = 2 or x = 5

The solutions are −1, 2, and 5.

43. f(x) = x3 − x2 − 14x + 24

Try x + 1, x − 1, and x + 2. Using synthetic division wefind that f(−1) �= 0, f(1) �= 0 and f(−2) �= 0. Thusx + 1, x− 1, and x + 2, are not factors of f(x).

Try x− 2. Use synthetic division to see whether f(2) = 0.

Copyright © 2013 Pearson Education, Inc.

Page 144: JUDITH A. PENNA -  · PDF fileSTUDENT’S SOLUTIONS MANUAL JUDITH A. PENNA Indiana University Purdue University Indianapolis COLLEGE ALGEBRA: GRAPHS AND MODELS FIFTH EDITION

Exercise Set 4.3 141

2∣∣ 1 −1 −14 24

2 2 −241 1 −12 0

Since f(2) = 0, x − 2 is a factor of f(x). Thus f(x) =(x− 2)(x2 + x− 12).

Factoring the trinomial we get

f(x) = (x− 2)(x + 4)(x− 3)

To solve the equation f(x) = 0, use the principle of zeroproducts.

(x− 2)(x + 4)(x− 3) = 0

x− 2 = 0 or x + 4 = 0 or x− 3 = 0

x = 2 or x = −4 or x = 3

The solutions are 2, −4, and 3.

45. f(x) = x4 − 7x3 + 9x2 + 27x− 54

Try x+ 1 and x− 1. Using synthetic division we find thatf(−1) �= 0 and f(1) �= 0. Thus x + 1 and x − 1 are notfactors of f(x). Try x + 2. Use synthetic division to seewhether f(−2) = 0.

−2∣∣ 1 −7 9 27 −54

−2 18 −54 541 −9 27 −27 0

Since f(−2) = 0, x + 2 is a factor of f(x). Thus f(x) =(x + 2)(x3 − 9x2 + 27x− 27).

We continue to use synthetic division to factor g(x) =x3 − 9x2 + 27x− 27. Trying x+ 2 again and x− 2 we findthat g(−2) �= 0 and g(2) �= 0. Thus x + 2 and x − 2 arenot factors of g(x). Try x− 3.

3∣∣ 1 −9 27 −27

3 −18 271 −6 9 0

Since g(3) = 0, x− 3 is a factor of x3 − 9x2 + 27x− 27.

Thus f(x) = (x + 2)(x− 3)(x2 − 6x + 9).

Factoring the trinomial we get

f(x) = (x + 2)(x− 3)(x− 3)2, or f(x) = (x + 2)(x− 3)3.

To solve the equation f(x) = 0, use the principle of zeroproducts.

(x + 2)(x− 3)(x− 3)(x− 3) = 0

x + 2 = 0 or x− 3 = 0 or x− 3 = 0 or x− 3 = 0

x = −2 or x = 3 or x = 3 or x = 3

The solutions are −2 and 3.

47. f(x) = x4 − x3 − 19x2 + 49x− 30

Try x− 1. Use synthetic division to see whether f(1) = 0.

1∣∣ 1 −1 −19 49 −30

1 0 −19 301 0 −19 30 0

Since f(1) = 0, x − 1 is a factor of f(x). Thus f(x) =(x− 1)(x3 − 19x + 30).

We continue to use synthetic division to factor g(x) =x3 − 19x+ 30. Trying x− 1, x+ 1, and x+ 2 we find thatg(1) �= 0, g(−1) �= 0, and g(−2) �= 0. Thus x − 1, x + 1,and x + 2 are not factors of x3 − 19x + 30. Try x− 2.

2∣∣ 1 0 −19 30

2 4 −301 2 −15 0

Since g(2) = 0, x− 2 is a factor of x3 − 19x + 30.

Thus f(x) = (x− 1)(x− 2)(x2 + 2x− 15).

Factoring the trinomial we get

f(x) = (x− 1)(x− 2)(x− 3)(x + 5).

To solve the equation f(x) = 0, use the principle of zeroproducts.

(x− 1)(x− 2)(x− 3)(x + 5) = 0

x−1 = 0 or x−2 = 0 or x−3 = 0 or x+5 = 0

x = 1 or x = 2 or x = 3 or x = −5

The solutions are 1, 2, 3, and −5.

49. f(x) = x4 − x3 − 7x2 + x + 6

1. The leading term is x4. The degree, 4, is even andthe leading coefficient, 1, is positive so as x → ∞,f(x) → ∞ and as x → −∞, f(x) → ∞.

2. Find the zeros of the function. We first use syntheticdivision to determine if f(1) = 0.

1∣∣ 1 −1 −7 1 6

1 0 −7 −61 0 −7 −6 0

1 is a zero of the function and we havef(x) = (x− 1)(x3 − 7x− 6).Synthetic division shows that −1 is a zero of g(x) =x3 − 7x− 6.

−1∣∣ 1 0 −7 −6

−1 1 61 −1 −6 0

Then we have f(x) = (x− 1)(x + 1)(x2 − x− 6).To find the other zeros we solve the following equa-tion:

x2 − x− 6 = 0

(x− 3)(x + 2) = 0

x− 3 = 0 or x + 2 = 0

x = 3 or x = −2

The zeros of the function are 1, −1, 3, and −2 so thex-intercepts of the graph are (1, 0), (−1, 0), (3, 0), and(−2, 0).

3. The zeros divide the x-axis into five intervals,(−∞,−2), (−2,−1), (−1, 1), (1, 3), and (3,∞). Wechoose a value for x from each interval and find f(x).This tells us the sign of f(x) for all values of x in theinterval.In (−∞,−2), test −3:

f(−3) = (−3)4 − (−3)3 − 7(−3)2 +(−3)+6 = 48 > 0

In (−2,−1), test −1.5:

f(−1.5) = (−1.5)4−(−1.5)3−7(−1.5)2+(−1.5)+6 =

−2.8125 < 0

In (−1, 1), test 0:

f(0) = 04 − 03 − 7 · 02 + 0 + 6 = 6 > 0

Copyright © 2013 Pearson Education, Inc.

Page 145: JUDITH A. PENNA -  · PDF fileSTUDENT’S SOLUTIONS MANUAL JUDITH A. PENNA Indiana University Purdue University Indianapolis COLLEGE ALGEBRA: GRAPHS AND MODELS FIFTH EDITION

y

x�4 �2 2 4

�8

�12

�4

4

8

f(x) � x4 � x3 � 7x2 � x � 6

f(x) � x3 � 7x � 6

y

x�4 �2 2 4

�20

�10

10

20

142 Chapter 4: Polynomial and Rational Functions

In (1, 3), test 2:

f(2) = 24 − 23 − 7 · 22 + 2 + 6 = −12 < 0

In (3,∞), test 4:

f(4) = 44 − 43 − 7 · 42 + 4 + 6 = 90 > 0

Thus the graph lies above the x-axis on (−∞,−2),on (−1, 1), and on (3,∞). It lies below the x-axison (−2,−1) and on (1, 3). We also know the points(−3, 48), (−1.5,−2.8125), (0, 6), (2,−12), and (4, 90)are on the graph.

4. From Step 3 we see that f(0) = 6 so the y-interceptis (0, 6).

5. We find additional points on the graph and draw thegraph.

x f(x)

−2.5 14.3

−0.5 3.9

0.5 4.7

2.5 −11.8

6. Checking the graph as described on page 317 in thetext, we see that it appears to be correct.

51. f(x) = x3 − 7x + 6

1. The leading term is x3. The degree, 3, is odd andthe leading coefficient, 1, is positive so as x → ∞,f(x) → ∞ and as x → −∞, f(x) → −∞.

2. Find the zeros of the function. We first use syntheticdivision to determine if f(1) = 0.

1∣∣ 1 0 −7 6

1 1 −61 1 −6 0

1 is a zero of the function and we havef(x) = (x − 1)(x2 + x − 6). To find the other zeroswe solve the following equation.

x2 + x− 6 = 0

(x + 3)(x− 2) = 0

x + 3 = 0 or x− 2 = 0

x = −3 or x = 2

The zeros of the function are 1, −3, and 2 so the x-intercepts of the graph are (1, 0), (−3, 0), and (2, 0).

3. The zeros divide the x-axis into four intervals,(−∞,−3), (−3, 1), (1, 2), and (2,∞). We choose avalue for x from each interval and find f(x). This tellsus the sign of f(x) for all values of x in the interval.In (−∞,−3), test −4:

f(−4) = (−4)3 − 7(−4) + 6 = −30 < 0

In (−3, 1), test 0:

f(0) = 03 − 7 · 0 + 6 = 6 > 0In (1, 2), test 1.5:

f(1.5) = (1.5)3 − 7(1.5) + 6 = −1.125 < 0

In (2,∞), test 3:

f(3) = 33 − 7 · 3 + 6 = 12 > 0Thus the graph lies below the x-axis on (−∞,−3)and on (1, 2). It lies above the x-axis on (−3, 1) andon (2,∞). We also know the points (−4,−30), (0, 6),(1.5,−1.125), and (3, 12) are on the graph.

4. From Step 3 we see that f(0) = 6 so the y-interceptis (0, 6).

5. We find additional points on the graph and draw thegraph.

x f(x)

−3.5 −12.4

−2 12

2.5 4.1

4 42

6. Checking the graph as described on page 317 in thetext, we see that it appears to be correct.

53. f(x) = −x3 + 3x2 + 6x− 8

1. The leading term is −x3. The degree, 3, is odd andthe leading coefficient, −1, is negative so as x → ∞,f(x) → −∞ and as x → −∞, f(x) → ∞.

2. Find the zeros of the function. We first use syntheticdivision to determine if f(1) = 0.

1∣∣ −1 3 6 −8

−1 2 8−1 2 8 0

1 is a zero of the function and we havef(x) = (x− 1)(−x2 +2x+8). To find the other zeroswe solve the following equation.

−x2 + 2x + 8 = 0

x2 − 2x− 8 = 0

(x− 4)(x + 2) = 0

x− 4 = 0 or x + 2 = 0

x = 4 or x = −2

The zeros of the function are 1, 4, and −2 so the x-intercepts of the graph are (1, 0), (4, 0), and (−2, 0).

3. The zeros divide the x-axis into four intervals,(−∞,−2), (−2, 1), (1, 4), and (4,∞). We choose avalue for x from each interval and find f(x). This tellsus the sign of f(x) for all values of x in the interval.In (−∞,−2), test −3:

f(−3) = −(−3)3 + 3(−3)2 + 6(−3) − 8 = 28 > 0

In (−2, 1), test 0:

f(0) = −03 + 3 · 02 + 6 · 0 − 8 = −8 < 0

In (1, 4), test 2:

f(2) = −23 + 3 · 22 + 6 · 2 − 8 = 8 > 0

In (4,∞), test 5:

f(5) = −53 + 3 · 52 + 6 · 5 − 8 = −28 < 0

Copyright © 2013 Pearson Education, Inc.

Page 146: JUDITH A. PENNA -  · PDF fileSTUDENT’S SOLUTIONS MANUAL JUDITH A. PENNA Indiana University Purdue University Indianapolis COLLEGE ALGEBRA: GRAPHS AND MODELS FIFTH EDITION

f(x) � �x3 � 3x2 � 6x � 8

y

x�4 �2 2 4

�8

�4

4

8

Exercise Set 4.3 143

Thus the graph lies above the x-axis on (−∞,−2)and on (1, 4). It lies below the x-axis on (−2, 1) andon (4,∞). We also know the points (−3, 28), (0,−8),(2, 8), and (5,−28) are on the graph.

4. From Step 3 we see that f(0) = −8 so the y-interceptis (0,−8).

5. We find additional points on the graph and draw thegraph.

x f(x)

−2.5 11.4

−1 −10

3 10

4.5 −11.4

6. Checking the graph as described on page 317 in thetext, we see that it appears to be correct.

55. 2x2 + 12 = 5x

2x2 − 5x + 12 = 0

a = 2, b = −5, c = 12

x =−b±√

b2 − 4ac2a

x =−(−5) ±√(−5)2 − 4 · 2 · 12

2 · 2=

5 ±√−714

=5 ± i

√71

4=

54±

√714

i

The solutions are54

+√

714

i and54−

√714

i, or54±

√714

i.

57. We substitute −14 for g(x) and solve for x.

−14 = x2 + 5x− 14

0 = x2 + 5x

0 = x(x + 5)

x = 0 or x + 5 = 0

x = 0 or x = −5

When the output is −14, the input is 0 or −5.

59. We substitute −20 for g(x) and solve for x.

−20 = x2 + 5x− 14

0 = x2 + 5x + 6

0 = (x + 3)(x + 2)

x + 3 = 0 or x + 2 = 0

x = −3 or x = −2

When the output is −20, the input is −3 or −2.

61. Let b and h represent the length of the base and the heightof the triangle, respectively.

b + h = 30, so b = 30 − h.

A =12bh =

12(30 − h)h = −1

2h2 + 15h

Find the value of h for which A is a maximum:

h =−15

2(−1/2)= 15

When h = 15, b = 30 − 15 = 15.

The area is a maximum when the base and the height areeach 15 in.

63. a) −4, −3, 2, and 5 are zeros of the function, so x+ 4,x + 3, x− 2, and x− 5 are factors.

b) We first write the product of the factors:

P (x) = (x + 4)(x + 3)(x− 2)(x− 5)

Note that P (0) = 4 · 3(−2)(−5) > 0 and the graphshows a positive y-intercept, so this function is acorrect one.

c) Yes; two examples are f(x) = c · P (x) for any non-zero constant c and g(x) = (x− a)P (x).

d) No; only the function in part (b) has the givengraph.

65. Divide x3 − kx2 + 3x + 7k by x + 2.−2∣∣ 1 −k 3 7k

−2 2k + 4 −4k − 141 −k − 2 2k + 7 3k − 14

Thus P (−2) = 3k − 14.

We know that if x+ 2 is a factor of f(x), then f(−2) = 0.

We solve 0 = 3k − 14 for k.0 = 3k − 14

143

= k

67. 2x2

x2 − 1+

4x + 3

=12x− 4

x3 + 3x2 − x− 3,

LCM is (x + 1)(x− 1)(x + 3)

(x + 1)(x− 1)(x + 3)[

2x2

(x + 1)(x− 1)+

4x + 3

]=

(x + 1)(x− 1)(x + 3) · 12x− 4(x + 1)(x− 1)(x + 3)

2x2(x + 3) + 4(x + 1)(x− 1) = 12x− 4

2x3 + 6x2 + 4x2 − 4 = 12x− 4

2x3 + 10x2 − 12x = 0

x3 + 5x2 − 6x = 0

x(x2 + 5x− 6) = 0

x(x + 6)(x− 1) = 0

x = 0 or x + 6 = 0 x− 1 = 0

x = 0 or x = −6 x = 1

Only 0 and −6 check. They are the solutions.

69. Answers may vary. One possibility is P (x) = x15 − x14.

71. −i∣∣ 1 3i −4i −2

−i 2 −4 − 2i1 2i 2 − 4i −6 − 2i

Q(x) = x2 + 2ix + (2 − 4i), R(x) = −6 − 2i

Copyright © 2013 Pearson Education, Inc.

Page 147: JUDITH A. PENNA -  · PDF fileSTUDENT’S SOLUTIONS MANUAL JUDITH A. PENNA Indiana University Purdue University Indianapolis COLLEGE ALGEBRA: GRAPHS AND MODELS FIFTH EDITION

144 Chapter 4: Polynomial and Rational Functions

73. i∣∣ 1 −3 7 (i2 = −1)

i −3i− 11 −3 + i 6 − 3i

The answer is x− 3 + i, R 6 − 3i.

Chapter 4 Mid-Chapter Mixed Review

1. P (0) = 5−2 ·03 = 5, so the y-intercept is (0, 5). The givenstatement is false.

3. f(8) = (8 + 7)(8 − 8) = 15 · 0 = 0

The given statement is true.

5. f(x) = (x2 − 10x + 25)3 = [(x− 5)2]3 = (x− 5)6

Solving (x− 5)6 = 0, we get x = 5.

The factor x − 5 occurs 6 times, so the zero has a multi-plicity of 6.

7. g(x) = x4−3x2+2 = (x2−1)(x2−2) = (x+1)(x−1)(x2−2)

Solving (x+1)(x−1)(x2−2) = 0, we get x = −1 or x = 1 orx = ±√

2.

Each factor occurs 1 time, so the multiplicity of each zerois 1.

9. f(x) = x4 − x3 − 6x2

The sign of the leading coefficient, 1, is positive and thedegree, 4, is even. Thus, graph (d) is the graph of thefunction.

11. f(x) = 6x3 + 8x2 − 6x− 8

The sign of the leading coefficient, 6, is positive and thedegree, 3, is odd. Thus, graph (b) is the graph of thefunction.

13. f(−2) = (−2)3 − 2(−2)2 + 3 = −13

f(0) = 03 − 2 · 02 + 3 = 3

By the intermediate value theorem, since f(−2) and f(0)have opposite signs, f(x) has a zero between −2 and 0.

15. x3 − 5x2 − 5x − 4x− 1 x4 − 6x3 + 0x2 + x − 2

x4 − x3

− 5x3 + 0x2

− 5x3 + 5x2

− 5x2 + x− 5x2 + 5x

− 4x− 2− 4x+ 4

− 6

P (x) = (x− 1)(x3 − 5x2 − 5x− 4) − 6x− 1

17. (x5 − 5) ÷ (x + 1) = (x5 − 5) ÷ [x− (−1)]

−1∣∣ 1 0 0 0 0 −5

−1 1 −1 1 −11 −1 1 −1 1 −6

Q(x) = x4 − x3 + x2 − x + 1, R(x) = −6

19. 12

∣∣∣ 20 −40 010 −15

20 −30 −15

f

(12

)= −15

21. f(x) = x3 − 4x2 + 9x− 36

If −3i is a zero of f(x), then f(−3i) = 0. We find f(−3i).

−3i∣∣ 1 −4 9 −36

−3i −9 + 12i 361 −4 − 3i 12i 0

Since f(−3i) = 0, −3i is a zero of f(x).

If 3 is a zero of f(x), then f(3) = 0. We find f(3).

3∣∣ 1 −4 9 −36

3 −3 181 −1 6 −18

Since f(3) �= 0, 3 is not a zero of f(x).

23. h(x) = x3 − 2x2 − 55x + 56

Try x− 1.

1∣∣ 1 −2 −55 56

1 −1 −561 −1 −56 0

Since h(1) = 0, x− 1 is a factor of h(x). Thenh(x) = (x − 1)(x2 − x − 56). Factoring the trinomial, weget h(x) = (x− 1)(x− 8)(x + 7).

Now we solve h(x) = 0.

(x− 1)(x− 8)(x + 7) = 0

x− 1 = 0 or x− 8 = 0 or x + 7 = 0

x = 1 or x = 8 or x = −7

The solutions are 1, 8, and −7.

25. The range of a polynomial function with an odd degreeis (−∞,∞). The range of a polynomial function with aneven degree is [s,∞) for some real number s if an > 0 andis (−∞, s] for some real number s if an < 0.

27. If function values change from positive to negative or fromnegative to positive in an interval, there would have to bea zero in the interval. Thus, between a pair of consecutivezeros, all the function values must have the same sign.

Exercise Set 4.4

1. Find a polynomial function of degree 3 with −2, 3, and 5as zeros.

Such a function has factors x + 2, x− 3, and x− 5, so wehave f(x) = an(x + 2)(x− 3)(x− 5).

The number an can be any nonzero number. The simplestpolynomial will be obtained if we let it be 1. Multiplyingthe factors, we obtain

f(x) = (x + 2)(x− 3)(x− 5)

= (x2 − x− 6)(x− 5)

= x3 − 6x2 − x + 30.

Copyright © 2013 Pearson Education, Inc.

Page 148: JUDITH A. PENNA -  · PDF fileSTUDENT’S SOLUTIONS MANUAL JUDITH A. PENNA Indiana University Purdue University Indianapolis COLLEGE ALGEBRA: GRAPHS AND MODELS FIFTH EDITION

Exercise Set 4.4 145

3. Find a polynomial function of degree 3 with −3, 2i, and−2i as zeros.

Such a function has factors x+3, x− 2i, and x+2i, so wehave f(x) = an(x + 3)(x− 2i)(x + 2i).

The number an can be any nonzero number. The simplestpolynomial will be obtained if we let it be 1. Multiplyingthe factors, we obtainf(x) = (x + 3)(x− 2i)(x + 2i)

= (x + 3)(x2 + 4)

= x3 + 3x2 + 4x + 12.

5. Find a polynomial function of degree 3 with√

2, −√2, and

3 as zeros.

Such a function has factors x−√2, x+

√2, and x− 3, so

we have f(x) = an(x−√2)(x +

√2)(x− 3).

The number an can be any nonzero number. The simplestpolynomial will be obtained if we let it be 1. Multiplyingthe factors, we obtain

f(x) = (x−√2)(x +

√2)(x− 3)

= (x2 − 2)(x− 3)

= x3 − 3x2 − 2x + 6.

7. Find a polynomial function of degree 3 with 1−√3, 1+

√3,

and −2 as zeros.

Such a function has factors x− (1−√3), x− (1+

√3), and

x + 2, so we have

f(x) = an[x− (1 −√3)][x− (1 +

√3)](x + 2).

The number an can be any nonzero number. The simplestpolynomial will be obtained if we let it be 1. Multiplyingthe factors, we obtain

f(x) = [x− (1 −√3)][x− (1 +

√3)](x + 2)

= [(x− 1) +√

3][(x− 1) −√3](x + 2)

= [(x− 1)2 − (√

3)2](x + 2)

= (x2 − 2x + 1 − 3)(x + 2)

= (x2 − 2x− 2)(x + 2)

= x3 − 2x2 − 2x + 2x2 − 4x− 4

= x3 − 6x− 4.

9. Find a polynomial function of degree 3 with 1 + 6i, 1− 6i,and −4 as zeros.

Such a function has factors x− (1 + 6i), x− (1 − 6i), andx + 4, so we have

f(x) = an[x− (1 + 6i)][x− (1 − 6i)](x + 4).

The number an can be any nonzero number. The simplestpolynomial will be obtained if we let it be 1. Multiplyingthe factors, we obtainf(x) = [x− (1 + 6i)][x− (1 − 6i)](x + 4)

= [(x− 1) − 6i][(x− 1) + 6i](x + 4)

= [(x− 1)2 − (6i)2](x + 4)

= (x2 − 2x + 1 + 36)(x + 4)

= (x2 − 2x + 37)(x + 4)

= x3 − 2x2 + 37x + 4x2 − 8x + 148

= x3 + 2x2 + 29x + 148.

11. Find a polynomial function of degree 3 with −13, 0, and 2

as zeros.

Such a function has factors x +13, x− 0 (or x), and x− 2

so we have

f(x) = an

(x +

13

)(x)(x− 2).

The number an can be any nonzero number. The simplestpolynomial will be obtained if we let it be 1. Multiplyingthe factors, we obtain

f(x) =(x +

13

)(x)(x− 2)

=(x2 +

13x

)(x− 2)

= x3 − 53x2 − 2

3x.

13. A polynomial function of degree 5 has at most 5 real zeros.Since 5 zeros are given, these are all of the zeros of thedesired function. We proceed as in Exercises 1-11, lettingan = 1.f(x) = (x + 1)3(x− 0)(x− 1)

= (x3 + 3x2 + 3x + 1)(x2 − x)

= x5 + 2x4 − 2x2 − x

15. A polynomial function of degree 4 has at most 4 real zeros.Since 4 zeros are given, these are all of the zeros of thedesired function. We proceed as in Exercises 1-11, lettingan = 1.f(x) = (x + 1)3(x− 0)

= (x3 + 3x2 + 3x + 1)(x)

= x4 + 3x3 + 3x2 + x

17. A polynomial function of degree 4 can have at most 4 zeros.Since f(x) has rational coefficients, in addition to the threezeros given, the other zero is the conjugate of

√3, or −√

3.

19. A polynomial function of degree 4 can have at most 4 zeros.Since f(x) has rational coefficients, the other zeros are theconjugates of the given zeros. They are i and 2 +

√5.

21. A polynomial function of degree 4 can have at most 4 zeros.Since f(x) has rational coefficients, in addition to the threezeros given, the other zero is the conjugate of 3i, or −3i.

23. A polynomial function of degree 4 can have at most 4 zeros.Since f(x) has rational coefficients, the other zeros are theconjugates of the given zeros. They are −4+3i and 2+

√3.

25. A polynomial function f(x) of degree 5 has at most 5 zeros.Since f(x) has rational coefficients, in addition to the 3given zeros, the other zeros are the conjugates of

√5 and

−4i, or −√5 and 4i.

27. A polynomial function f(x) of degree 5 has at most 5 zeros.Since f(x) has rational coefficients, the other zero is theconjugate of 2 − i, or 2 + i.

29. A polynomial function f(x) of degree 5 has at most 5 zeros.Since f(x) has rational coefficients, in addition to the 3given zeros, the other zeros are the conjugates of −3 + 4iand 4 −√

5, or −3 − 4i and 4 +√

5.

Copyright © 2013 Pearson Education, Inc.

Page 149: JUDITH A. PENNA -  · PDF fileSTUDENT’S SOLUTIONS MANUAL JUDITH A. PENNA Indiana University Purdue University Indianapolis COLLEGE ALGEBRA: GRAPHS AND MODELS FIFTH EDITION

146 Chapter 4: Polynomial and Rational Functions

31. A polynomial function f(x) of degree 5 has at most 5 zeros.Since f(x) has rational coefficients, the other zero is theconjugate of 4 − i, or 4 + i.

33. Find a polynomial function of lowest degree with rationalcoefficients that has 1 + i and 2 as some of its zeros. 1− iis also a zero.

Thus the polynomial function is

f(x) = an(x− 2)[x− (1 + i)][x− (1 − i)].

If we let an = 1, we obtain

f(x) = (x− 2)[(x− 1) − i][(x− 1) + i]

= (x− 2)[(x− 1)2 − i2]

= (x− 2)(x2 − 2x + 1 + 1)

= (x− 2)(x2 − 2x + 2)

= x3 − 4x2 + 6x− 4.

35. Find a polynomial function of lowest degree with rationalcoefficients that has 4i as one of its zeros. −4i is also azero.

Thus the polynomial function is

f(x) = an(x− 4i)(x + 4i).

If we let an = 1, we obtain

f(x) = (x− 4i)(x + 4i) = x2 + 16.

37. Find a polynomial function of lowest degree with rationalcoefficients that has −4i and 5 as some of its zeros.

4i is also a zero.

Thus the polynomial function is

f(x) = an(x− 5)(x + 4i)(x− 4i).

If we let an = 1, we obtain

f(x) = (x− 5)[x2 − (4i)2]

= (x− 5)(x2 + 16)

= x3 − 5x2 + 16x− 80

39. Find a polynomial function of lowest degree with rationalcoefficients that has 1 − i and −√

5 as some of its zeros.1 + i and

√5 are also zeros.

Thus the polynomial function is

f(x) = an[x− (1 − i)][x− (1 + i)](x +√

5)(x−√5).

If we let an = 1, we obtain

f(x) = [x− (1 − i)][x− (1 + i)](x +√

5)(x−√5)

= [(x− 1) + i][(x− 1) − i](x +√

5)(x−√5)

= (x2 − 2x + 1 + 1)(x2 − 5)

= (x2 − 2x + 2)(x2 − 5)

= x4 − 2x3 + 2x2 − 5x2 + 10x− 10

= x4 − 2x3 − 3x2 + 10x− 10

41. Find a polynomial function of lowest degree with rationalcoefficients that has

√5 and −3i as some of its zeros.

−√5 and 3i are also zeros.

Thus the polynomial function is

f(x) = an(x−√5)(x +

√5)(x + 3i)(x− 3i).

If we let an = 1, we obtain

f(x) = (x2 − 5)(x2 + 9)

= x4 + 4x2 − 45

43. f(x) = x3 + 5x2 − 2x− 10

Since −5 is a zero of f(x), we have f(x) = (x + 5) ·Q(x).We use synthetic division to find Q(x).

−5∣∣ 1 5 −2 −10

−5 0 101 0 −2 0

Then f(x) = (x + 5)(x2 − 2). To find the other zeros wesolve x2 − 2 = 0.

x2 − 2 = 0

x2 = 2

x = ±√2

The other zeros are −√2 and

√2.

45. If −i is a zero of f(x) = x4 − 5x3 + 7x2 − 5x + 6, i is alsoa zero. Thus x+ i and x− i are factors of the polynomial.Since (x + i)(x− i) = x2 + 1, we know that f(x) =(x2 + 1) ·Q(x). Divide x4 − 5x3 + 7x2 − 5x+ 6 by x2 + 1.

x2 − 5x + 6x2 + 1 x4 − 5x3 + 7x2 − 5x + 6

x4 + x2

−5x3 + 6x2 − 5x−5x3 − 5x

6x2 + 66x2 + 6

0

Thusx4−5x3+7x2−5x+6 = (x+i)(x−i)(x2−5x+6)

= (x+i)(x−i)(x−2)(x−3)

Using the principle of zero products we find the other zerosto be i, 2, and 3.

47. x3 − 6x2 + 13x− 20 = 0

If 4 is a zero, then x− 4 is a factor. Use synthetic divisionto find another factor.4∣∣ 1 −6 13 −20

4 −8 201 −2 5 0

(x− 4)(x2 − 2x + 5) = 0

x−4 = 0 or x2−2x+5 = 0 Principle ofzero products

x = 4 or x =2±√

4−202

Quadratic formula

x = 4 or x =2±4i

2= 1 ± 2i

The other zeros are 1 + 2i and 1 − 2i.

49. f(x) = x5 − 3x2 + 1

According to the rational zeros theorem, any rational zeroof f must be of the form p/q, where p is a factor of theconstant term, 1, and q is a factor of the coefficient of x5,1.

Copyright © 2013 Pearson Education, Inc.

Page 150: JUDITH A. PENNA -  · PDF fileSTUDENT’S SOLUTIONS MANUAL JUDITH A. PENNA Indiana University Purdue University Indianapolis COLLEGE ALGEBRA: GRAPHS AND MODELS FIFTH EDITION

Exercise Set 4.4 147

Possibilities for p

Possibilities for q:

±1±1

Possibilities for p/q: 1, −1

51. f(x) = 2x4 − 3x3 − x + 8

According to the rational zeros theorem, any rational zeroof f must be of the form p/q, where p is a factor of theconstant term, 8, and q is a factor of the coefficient of x4,2.Possibilities for p

Possibilities for q:

±1,±2,±4,±8±1,±2

Possibilities for p/q: 1, −1, 2, −2, 4, −4, 8, −8,12, −1

2

53. f(x) = 15x6 + 47x2 + 2

According to the rational zeros theorem, any rational zeroof f must be of the form p/q, where p is a factor of 2 andq is a factor of 15.Possibilities for p

Possibilities for q:

±1,±2±1,±3,±5,±15

Possibilities for p/q: 1,−1, 2,−2,13,−1

3,23,−2

3,15,

−15,25,−2

5,

115

,− 115

,215

,− 215

55. f(x) = x3 + 3x2 − 2x− 6

a) Possibilities for p

Possibilities for q:

±1,±2,±3,±6±1

Possibilities for p/q: 1,−1, 2,−2, 3,−3, 6,−6

We use synthetic division to find a zero. We findthat one zero is −3 as shown below.−3∣∣ 1 3 −2 −6

−3 0 61 0 −2 0

Then we have f(x) = (x + 3)(x2 − 2).

We find the other zeros:x2 − 2 = 0

x2 = 2

x = ±√2.

There is only one rational zero, −3. The otherzeros are ±√

2. (Note that we could have usedfactoring by grouping to find this result.)

b) f(x) = (x + 3)(x−√2)(x +

√2)

57. f(x) = 3x3 − x2 − 15x + 5

a) Possibilities for p

Possibilities for q:

±1,±5±1,±3

Possibilities for p/q: 1,−1, 5,−5,13,−1

3,53,−5

3We use synthetic division to find a zero. We find

that one zero is13

as shown below.

13

∣∣ 3 −1 −15 51 0 −5

3 0 −15 0

Then we have f(x) =(x− 1

3

)(3x2 − 15), or

3(x− 1

3

)(x2 − 5).

Now x2 − 5 = 0 for x = ±√5. Thus, there is

only one rational zero,13. The other zeros are

±√5. (Note that we could have used factoring

by grouping to find this result.)

b) f(x) = 3(x− 1

3

)(x +

√5)(x−

√5)

59. f(x) = x3 − 3x + 2

a) Possibilities for p

Possibilities for q:

±1,±2±1

Possibilities for p/q: 1,−1, 2,−2

We use synthetic division to find a zero. We findthat −2 is a zero as shown below.−2∣∣ 1 0 −3 2

−2 4 −21 −2 1 0

Then we have f(x) = (x + 2)(x2 − 2x + 1) =(x + 2)(x− 1)2.

Now (x − 1)2 = 0 for x = 1. Thus, the rationalzeros are −2 and 1. (The zero 1 has a multiplicityof 2.) These are the only zeros.

b) f(x) = (x + 2)(x− 1)2

61. f(x) = 2x3 + 3x2 + 18x + 27

a) Possibilities for p

Possibilities for q:

±1,±3,±9,±27±1,±2

Possibilities for p/q: 1,−1, 3,−3, 9, −9, 27, −27,12, −1

2,

32, −3

2,

92, −9

2,

272

, −272

We use synthetic division to find a zero. We find

that −32

is a zero as shown below.

− 32

∣∣ 2 3 18 27−3 0 −27

2 0 18 0

Then we have f(x) =(x +

32

)(2x2 + 18), or

2(x +

32

)(x2 + 9).

Now x2 + 9 = 0 for x = ±3i. Thus, the

only rational zero is −32. The other zeros are

±3i. (Note that we could have used factoring bygrouping to find this result.)

b) f(x) = 2(x +

32

)(x + 3i)(x− 3i)

Copyright © 2013 Pearson Education, Inc.

Page 151: JUDITH A. PENNA -  · PDF fileSTUDENT’S SOLUTIONS MANUAL JUDITH A. PENNA Indiana University Purdue University Indianapolis COLLEGE ALGEBRA: GRAPHS AND MODELS FIFTH EDITION

148 Chapter 4: Polynomial and Rational Functions

63. f(x) = 5x4 − 4x3 + 19x2 − 16x− 4

a) Possibilities for p

Possibilities for q:

±1,±2,±4±1,±5

Possibilities for p/q: 1,−1, 2,−2, 4,−4,15,−1

525,−2

5,45,−4

5We use synthetic division to find a zero. We findthat 1 is a zero as shown below.1∣∣ 5 −4 19 −16 −4

5 1 20 45 1 20 4 0

Then we havef(x) = (x− 1)(5x3 + x2 + 20x + 4)

= (x− 1)[x2(5x + 1) + 4(5x + 1)]

= (x− 1)(5x + 1)(x2 + 4).

We find the other zeros:5x + 1 = 0 or x2 + 4 = 0

5x = −1 or x2 = −4

x = −15

or x = ±2i

The rational zeros are −15

and 1. The other zerosare ±2i.

b) From part (a) we see that

f(x) = (5x + 1)(x − 1)(x + 2i)(x − 2i), or

5(x +

15

)(x− 1)(x + 2i)(x− 2i).

65. f(x) = x4 − 3x3 − 20x2 − 24x− 8

a) Possibilities for p

Possibilities for q:

±1,±2,±4,±8±1

Possibilities for p/q: 1,−1, 2,−2, 4,−4, 8,−8

We use synthetic division to find a zero. We findthat −2 is a zero as shown below.−2∣∣ 1 −3 −20 −24 −8

−2 10 20 81 −5 −10 −4 0

Now we determine whether −1 is a zero.−1∣∣ 1 −5 −10 −4

−1 6 41 −6 −4 0

Then we have f(x) = (x + 2)(x + 1)(x2 − 6x− 4).

Use the quadratic formula to find the other zeros.

x2 − 6x− 4 = 0

x =−(−6) ±√(−6)2 − 4 · 1 · (−4)

2 · 1

=6 ±√

522

=6 ± 2

√13

2= 3 ±√

13

The rational zeros are −2 and −1. The other zerosare 3 ±√

13.

b) f(x) = (x+2)(x+1)[x−(3+√

13)][x−(3−√13)]

= (x+2)(x+1)(x−3−√13)(x−3+

√13)

67. f(x) = x3 − 4x2 + 2x + 4

a) Possibilities for p

Possibilities for q:

±1,±2,±4±1

Possibilities for p/q: 1,−1, 2,−2, 4,−4

Synthetic division shows that neither −1 nor 1is a zero. Try 2.

2∣∣ 1 −4 2 4

2 −4 −41 −2 −2 0

Then we have f(x) = (x− 2)(x2 − 2x− 2). Usethe quadratic formula to find the other zeros.

x2 − 2x− 2 = 0

x =−(−2) ±√(−2)2 − 4 · 1 · (−2)

2 · 1

=2 ±√

122

=2 ± 2

√3

2= 1 ±√

3

The only rational zero is 2. The other zeros are1 ±√

3.

b) f(x) = (x− 2)[x− (1 +√

3)][x− (1 −√3)]

= (x− 2)(x− 1 −√3)(x− 1 +

√3)

69. f(x) = x3 + 8

a) Possibilities for p

Possibilities for q:

±1,±2,±4,±8±1

Possibilities for p/q: 1,−1, 2,−2, 4,−4, 8,−8

We use synthetic division to find a zero. We findthat −2 is a zero as shown below.−2∣∣ 1 0 0 8

−2 4 −81 −2 4 0

We have f(x) = (x + 2)(x2 − 2x + 4). Use thequadratic formula to find the other zeros.

x2 − 2x + 4 = 0

x =−(−2) ±√(−2)2 − 4 · 1 · 4

2 · 1

=2 ±√−12

2=

2 ± 2√

3i2

= 1 ±√3i

The only rational zero is −2. The other zerosare 1 ±√

3i.

b) f(x) = (x + 2)[x− (1 +√

3i)][x− (1 −√3i)]

= (x + 2)(x− 1 −√3i)(x− 1 +

√3i)

71. f(x) =13x3 − 1

2x2 − 1

6x +

16

=16(2x3 − 3x2 − x + 1)

Copyright © 2013 Pearson Education, Inc.

Page 152: JUDITH A. PENNA -  · PDF fileSTUDENT’S SOLUTIONS MANUAL JUDITH A. PENNA Indiana University Purdue University Indianapolis COLLEGE ALGEBRA: GRAPHS AND MODELS FIFTH EDITION

Exercise Set 4.4 149

a) The second form of the equation is equivalent tothe first and has the advantage of having integercoefficients. Thus, we can use the rational zerostheorem for g(x) = 2x3 − 3x2 −x+ 1. The zerosof g(x) are the same as the zeros of f(x). Wefind the zeros of g(x).Possibilities for p

Possibilities for q:

±1±1,±2

Possibilities for p/q: 1,−1,12,−1

2

Synthetic division shows that −12

is not a zero.

Try12.

12

∣∣∣ 2 −3 −1 11 −1 −1

2 −2 −2 0

We have g(x) =(x− 1

2

)(2x2 − 2x− 2) =(

x− 12

)(2)(x2 − x− 1). Use the quadratic for-

mula to find the other zeros.x2 − x− 1 = 0

x =−(−1) ±√(−1)2 − 4 · 1 · (−1)

2 · 1

=1 ±√

52

The only rational zero is12. The other zeros are

1 ±√5

2.

b) f(x) =16g(x)

=16

(x− 1

2

)(2)[x− 1+

√5

2

][x− 1−√

52

]

=13

(x− 1

2

)(x− 1+

√5

2

)(x− 1−√

52

)

73. f(x) = x4 + 2x3 − 5x2 − 4x + 6

According to the rational zeros theorem, the possible ra-tional zeros are ±1, ±2 , ±3, and ±6. Synthetic divisionshows that only 1 and −3 are zeros.

75. f(x) = x3 − x2 − 4x + 3

According to the rational zeros theorem, the possible ra-tional zeros are ±1 and ±3. Synthetic division shows thatnone of these is a zero. Thus, there are no rational zeros.

77. f(x) = x4 + 2x3 + 2x2 − 4x− 8

According to the rational zeros theorem, the possible ra-tional zeros are ±1, ±2, ±4, and ±8. Synthetic divisionshows that none of the possibilities is a zero. Thus, thereare no rational zeros.

79. f(x) = x5 − 5x4 + 5x3 + 15x2 − 36x + 20

According to the rational zeros theorem, the possible ra-tional zeros are ±1, ±2, ±4, ±5, ±10, and ±20. We try−2.−2∣∣ 1 −5 5 15 −36 20

−2 14 −38 46 −201 −7 19 −23 10 0

Thus, −2 is a zero. Now try 1.

1∣∣ 1 −7 19 −23 10

1 −6 13 −101 −6 13 −10 0

1 is also a zero. Try 2.

2∣∣ 1 −6 13 −10

2 −8 101 −4 5 0

2 is also a zero.

We have f(x) = (x + 2)(x − 1)(x − 2)(x2 − 4x + 5). Thediscriminant of x2 − 4x + 5 is (−4)2 − 4 · 1 · 5, or 4 < 0,so x2 − 4x + 5 has two nonreal zeros. Thus, the rationalzeros are −2, 1, and 2.

81. f(x) = 3x5 − 2x2 + x− 1

The number of variations in sign in f(x) is 3. Then thenumber of positive real zeros is either 3 or less than 3 by2, 4, 6, and so on. Thus, the number of positive real zerosis 3 or 1.f(−x) = 3(−x)5 − 2(−x)2 + (−x) − 1

= −3x5 − 2x2 − x− 1

There are no variations in sign in f(−x), so there are 0negative real zeros.

83. h(x) = 6x7 + 2x2 + 5x + 4

There are no variations in sign in h(x), so there are 0positive real zeros.

h(−x) = 6(−x)7 + 2(−x)2 + 5(−x) + 4

= −6x7 + 2x2 − 5x + 4

The number of variations in sign in h(−x) is 3. Thus, thereare 3 or 1 negative real zeros.

85. F (p) = 3p18 + 2p4 − 5p2 + p + 3

There are 2 variations in sign in F (p), so there are 2 or 0positive real zeros.

F (−p) = 3(−p)18 + 2(−p)4 − 5(−p)2 + (−p) + 3

= 3p18 + 2p4 − 5p2 − p + 3

There are 2 variations in sign in F (−p), so there are 2 or0 negative real zeros.

87. C(x) = 7x6 + 3x4 − x− 10

There is 1 variation in sign in C(x), so there is 1 positivereal zero.C(−x) = 7(−x)6 + 3(−x)4 − (−x) − 10

= 7x6 + 3x4 + x− 10

There is 1 variation in sign in C(−x), so there is 1 negativereal zero.

Copyright © 2013 Pearson Education, Inc.

Page 153: JUDITH A. PENNA -  · PDF fileSTUDENT’S SOLUTIONS MANUAL JUDITH A. PENNA Indiana University Purdue University Indianapolis COLLEGE ALGEBRA: GRAPHS AND MODELS FIFTH EDITION

f(x) � 4x3 � x2 � 8x � 2

y

�8

�4

4

8

x�4 �2 2 4

150 Chapter 4: Polynomial and Rational Functions

89. h(t) = −4t5 − t3 + 2t2 + 1

There is 1 variation in sign in h(t), so there is 1 positivereal zero.h(−t) = −4(−t)5 − (−t)3 + 2(−t)2 + 1

= 4t5 + t3 + 2t2 + 1

There are no variations in sign in h(−t), so there are 0negative real zeros.

91. f(y) = y4 + 13y3 − y + 5

There are 2 variations in sign in f(y), so there are 2 or 0positive real zeros.

f(−y) = (−y)4 + 13(−y)3 − (−y) + 5

= y4 − 13y3 + y + 5

There are 2 variations in sign in f(−y), so there are 2 or0 negative real zeros.

93. r(x) = x4 − 6x2 + 20x− 24

There are 3 variations in sign in r(x), so there are 3 or 1positive real zeros.

r(−x) = (−x)4 − 6(−x)2 + 20(−x) − 24

= x4 − 6x2 − 20x− 24

There is 1 variation in sign in r(−x), so there is 1 negativereal zero.

95. R(x) = 3x5 − 5x3 − 4x

There is 1 variation in sign in R(x), so there is 1 positivereal zero.R(−x) = 3(−x)5 − 5(−x)3 − 4(−x)

= −3x5 + 5x3 + 4x

There is 1 variation in sign in R(−x), so there is 1 negativereal zero.

97. f(x) = 4x3 + x2 − 8x− 2

1. The leading term is 4x3. The degree, 3, is odd andthe leading coefficient, 4, is positive so as x → ∞,f(x) → ∞ and x → −∞, f(x) → −∞.

2. We find the rational zeros p/q of f(x).Possibilities for p

Possibilities for q:

±1,±2±1,±2,±4

Possibilities for p/q: 1, −1 , 2, −2,12, −1

2,

14, −1

4

Synthetic division shows that −14

is a zero.

− 14

∣∣ 4 1 −8 −2−1 0 2

4 0 −8 0

We have f(x) =(x +

14

)(4x2 − 8) =

4(x +

14

)(x2 − 2). Solving x2 − 2 = 0 we get

x = ±√2. Thus the zeros of the function are −1

4,

−√2, and

√2 so the x-intercepts of the graph are(

− 14, 0)

, (−√2, 0), and (

√2, 0).

3. The zeros divide the x-axis into 4 intervals,

(−∞,−√2),(−

√2,−1

4

),(− 1

4,√

2)

, and

(√

2,∞). We choose a value for x from each intervaland find f(x). This tells us the sign of f(x) for allvalues of x in that interval.In (−∞,−√

2), test −2:

f(−2) = 4(−2)3 + (−2)2 − 8(−2) − 2 = −14 < 0

In(−

√2,−1

4

), test −1:

f(−1) = 4(−1)3 + (−1)2 − 8(−1) − 2 = 3 > 0

In(− 1

4,√

2)

, test 0:

f(0) = 4 · 03 + 02 − 8 · 0 − 2 = −2 < 0

In (√

2,∞), test 2:

f(2) = 4 · 23 + 22 − 8 · 2 − 2 = 18 > 0

Thus the graph lies below the x-axis on (−∞,−√2)

and on(− 1

4,√

2)

. It lies above the x-axis on(−

√2,−1

4

)and on (

√2,∞). We also know the

points (−2,−14), (−1, 3), (0,−2), and (2, 18) are onthe graph.

4. From Step 3 we see that f(0) = −2 so the y-interceptis (0,−2).

5. We find additional points on the graph and then drawthe graph.

x f(x)

−1.5 −1.25

−0.5 1.75

1 −5

1.5 1.75

6. Checking the graph as described on page 317 in thetext, we see that it appears to be correct.

99. f(x) = 2x4 − 3x3 − 2x2 + 3x

1. The leading term is 2x4. The degree, 4, is even andthe leading coefficient, 2, is positive so as x → ∞,f(x) → ∞ and as x → −∞, f(x) → ∞.

2. We find the rational zeros p/q of f(x). First note thatf(x) = x(2x3 − 3x2 − 2x + 3), so 0 is a zero. Nowconsider g(x) = 2x3 − 3x2 − 2x + 3.Possibilities for p

Possibilities for q:

±1,±3±1,±2

Possibilities for p/q: 1, −1 , 3, −3,12, −1

2,

32, −3

2

We try 1.

1∣∣ 2 −3 −2 3

2 −1 −32 −1 −3 0

Copyright © 2013 Pearson Education, Inc.

Page 154: JUDITH A. PENNA -  · PDF fileSTUDENT’S SOLUTIONS MANUAL JUDITH A. PENNA Indiana University Purdue University Indianapolis COLLEGE ALGEBRA: GRAPHS AND MODELS FIFTH EDITION

y

x�4 �2 2 4

�4

�2

2

4

f(x) � 2x4 � 3x3 � 2x2 � 3x

Exercise Set 4.4 151

Then f(x) = x(x−1)(2x2−x−3). Using the principle

of zero products to solve 2x2−x−3 = 0, we get x =32

or x = −1.

Thus the zeros of the function are 0, 1,32, and −1 so

the x-intercepts of the graph are (0, 0), (1, 0),(

32, 0)

,

and (−1, 0).

3. The zeros divide the x-axis into 5 intervals, (−∞,−1),

(−1, 0), (0, 1),(

1,32

), and

(32,∞)

. We choose a

value for x from each interval and find f(x). Thistells us the sign of f(x) for all values of x in thatinterval.In (−∞,−1), test −2:

f(−2) = 2(−2)4 − 3(−2)3 − 2(−2)2 + 3(−2) = 42 > 0In (−1, 0), test −0.5:

f(−0.5) = 2(−0.5)4−3(−0.5)3−2(−0.5)2+3(−0.5) =

−1.5 < 0In (0, 1), test 0.5:

f(0.5) = 2(0.5)4−3(0.5)3−2(0.5)2+3(0.5) = 0.75 > 0

In(

1,32

), test 1.25:

f(1.25) = 2(1.25)4 − 3(1.25)3 − 2(1.25)2 + 3(1.25) =

−0.3515625 < 0

In(

32,∞)

, test 2:

f(2) = 2 · 24 − 3 · 23 − 2 · 22 + 3 · 2 = 6 > 0Thus the graph lies above the x-axis on (−∞,−1),

on (0, 1), and on(

32,∞)

. It lies below the x-axis

on (−1, 0) and on(

1,32

). We also know the points

(−2, 42), (−0.5,−1.5), (0.5, 0.75), (1.25,−0.3515625),and (2, 6) are on the graph.

4. From Step 2 we know that f(0) = 0 so the y-interceptis (0, 0).

5. We find additional points on the graph and then drawthe graph.

x f(x)

−1.5 11.25

2.5 26.25

3 72

6. Checking the graph as described on page 317 in thetext, we see that it appears to be correct.

101. f(x) = x2 − 8x + 10

a) − b

2a= − −8

2 · 1 = −(−4) = 4

f(4) = 42 − 8 · 4 + 10 = −6

The vertex is (4,−6).

b) The axis of symmetry is x = 4.

c) Since the coefficient of x2 is positive, there is aminimum function value. It is the second coor-dinate of the vertex, −6. It occurs when x = 4.

103. −45x + 8 = 0

−45x = −8 Subtracting 8

−54

(− 4

5x

)= −5

4(−8) Multiplying by −5

4x = 10

The zero is 10.

105. g(x) = −x3 − 2x2

Leading term: −x3; leading coefficient: −1

The degree is 3, so the function is cubic.

Since the degree is odd and the leading coefficient is nega-tive, as x → ∞, g(x) → −∞ and as x → −∞, g(x) → ∞.

107. f(x) = −49

Leading term: −49; leading coefficient: −4

9;

for all x, f(x) = −49

The degree is 0, so this is a constant function.

109. g(x) = x4 − 2x3 + x2 − x + 2

Leading term: x4; leading coefficient: 1

The degree is 4, so the function is quartic.

Since the degree is even and the leading coefficient is pos-itive, as x → ∞, g(x) → ∞ and as x → −∞, g(x) → ∞.

111. f(x) = 2x3 − 5x2 − 4x + 3

a) 2x3 − 5x2 − 4x + 3 = 0Possibilities for p

Possibilities for q:±1,±3±1,±2

Possibilities for p/q: 1, −1, 3, −3,12, −1

2,

32, −3

2The first possibility that is a solution of f(x) = 0 is−1:−1∣∣ 2 −5 −4 3

−2 7 −32 −7 3 0

Thus, −1 is a solution.Then we have:

(x + 1)(2x2 − 7x + 3) = 0

(x + 1)(2x− 1)(x− 3) = 0

The other solutions are12

and 3.

Copyright © 2013 Pearson Education, Inc.

Page 155: JUDITH A. PENNA -  · PDF fileSTUDENT’S SOLUTIONS MANUAL JUDITH A. PENNA Indiana University Purdue University Indianapolis COLLEGE ALGEBRA: GRAPHS AND MODELS FIFTH EDITION

152 Chapter 4: Polynomial and Rational Functions

b) The graph of y = f(x− 1) is the graph of y = f(x)shifted 1 unit right. Thus, we add 1 to each solutionof f(x) = 0 to find the solutions of f(x − 1) = 0.

The solutions are −1 + 1, or 0;12

+ 1, or32; and

3 + 1, or 4.

c) The graph of y = f(x + 2) is the graph of y =f(x) shifted 2 units left. Thus, we subtract 2 fromeach solution of f(x) = 0 to find the solutions off(x + 2) = 0. The solutions are −1 − 2, or −3;12− 2, or −3

2; and 3 − 2, or 1.

d) The graph of y = f(2x) is a horizontal shrinking ofthe graph of y = f(x) by a factor of 2. We divideeach solution of f(x) = 0 by 2 to find the solutions

of f(2x) = 0. The solutions are−12

or −12;

1/22

, or14; and

32.

113. P (x) = 2x5 − 33x4 − 84x3 + 2203x2 − 3348x− 10, 080

2x5 − 33x4 − 84x3 + 2203x2 − 3348x− 10, 080 = 0

Trying some of the many possibilities for p/q, we find that4 is a zero.4∣∣ 2 −33 −84 2203 −3348 −10, 080

8 −100 −736 5868 10, 0802 −25 −184 1467 2520 0

Then we have:

(x− 4)(2x4 − 25x3 − 184x2 + 1467x + 2520) = 0

We now use the fourth degree polynomial above to findanother zero. Synthetic division shows that 4 is not adouble zero, but 7 is a zero.7∣∣ 2 −25 −184 1467 2520

14 −77 −1827 −25202 −11 −261 −360 0

Now we have:

(x− 4)(x− 7)(2x3 − 11x2 − 261x− 360) = 0

Use the third degree polynomial above to find a third zero.Synthetic division shows that 7 is not a double zero, but15 is a zero.15∣∣ 2 −11 −261 −360

30 285 3602 19 24 0

We have:P (x) = (x− 4)(x− 7)(x− 15)(2x2 + 19x + 24)

= (x− 4)(x− 7)(x− 15)(2x + 3)(x + 8)

The rational zeros are 4, 7, 15, −32, and −8.

Exercise Set 4.5

1. f(x) =x2

2 − x

We find the value(s) of x for which the denominator is 0.

2 − x = 0

2 = x

The domain is {x|x �= 2}, or (−∞, 2) ∪ (2,∞).

3. f(x) =x + 1

x2 − 6x + 5We find the value(s) of x for which the denominator is 0.

x2 − 6x + 5 = 0

(x− 1)(x− 5) = 0

x− 1 = 0 or x− 5 = 0

x = 1 or x = 5

The domain is {x|x �= 1 and x �= 5}, or (−∞, 1) ∪ (1, 5) ∪(5,∞).

5. f(x) =3x− 43x + 15

We find the value(s) of x for which the denominator is 0.

3x + 15 = 0

3x = −15

x = −5

The domain is {x|x �= −5}, or (−∞,−5) ∪ (−5,∞).

7. Graph (d) is the graph of f(x) =8

x2 − 4.

x2 − 4 = 0 when x = ±2, so x = −2 and x = 2 are verticalasymptotes.

The x-axis, y = 0, is the horizontal asymptote becausethe degree of the numerator is less than the degree of thedenominator.

There is no oblique asymptote.

9. Graph (e) is the graph of f(x) =8x

x2 − 4.

As in Exercise 7, x = −2 and x = 2 are vertical asymp-totes.

The x-axis, y = 0, is the horizontal asymptote becausethe degree of the numerator is less than the degree of thedenominator.

There is no oblique asymptote.

11. Graph (c) is the graph of f(x) =8x3

x2 − 4.

As in Exercise 7, x = −2 and x = 2 are vertical asymp-totes.

The degree of the numerator is greater than the degree ofthe denominator, so there is no horizontal asymptote butthere is an oblique asymptote. To find it we first divide tofind an equivalent expression.

8xx2 − 4 8x3

8x3−32x32x

8x3

x2 − 4= 8x +

32xx2 − 4

Now we multiply by 1, using (1/x2)/(1/x2).

32xx2 − 4

·1x2

1x2

=

32x

1 − 4x2

As |x| becomes very large, each expression with x in thedenominator tends toward zero.

Copyright © 2013 Pearson Education, Inc.

Page 156: JUDITH A. PENNA -  · PDF fileSTUDENT’S SOLUTIONS MANUAL JUDITH A. PENNA Indiana University Purdue University Indianapolis COLLEGE ALGEBRA: GRAPHS AND MODELS FIFTH EDITION

2 4

2

4

�4 �2

�4

�2

f(x) � 1x

x � 0

y � 0

x

y

Exercise Set 4.5 153

Then, as |x| → ∞, we have32x

1 − 4x2

→ 01 − 0

, or 0.

Thus, as |x| becomes very large, the graph of f(x) getsvery close to the graph of y = 8x, so y = 8x is the obliqueasymptote.

13. g(x) =1x2

The numerator and the denominator have no common fac-tors. The zero of the denominator is 0, so the verticalasymptote is x = 0.

15. h(x) =x + 72 − x

The numerator and the denominator have no common fac-tors. 2 − x = 0 when x = 2, so the vertical asymptote isx = 2.

17. f(x) =3 − x

(x− 4)(x + 6)The numerator and the denominator have no common fac-tors. The zeros of the denominator are 4 and −6, so thevertical asymptotes are x = 4 and x = −6.

19. g(x) =x2

2x2 − x− 3=

x2

(2x− 3)(x + 1)The numerator and the denominator have no common fac-tors. The zeros of the denominator are

32

and −1, so the

vertical asymptotes are x =32

and x = −1.

21. f(x) =3x2 + 54x2 − 3

The numerator and the denominator have the same degree

and the ratio of the leading coefficients is34, so y =

34

isthe horizontal asymptote.

23. h(x) =x2 − 42x4 + 3

The degree of the numerator is less than the degree of thedenominator, so y = 0 is the horizontal asymptote.

25. g(x) =x3 − 2x2 + x− 1

x2 − 16The degree of the numerator is greater than the degree ofthe denominator, so there is no horizontal asymptote.

27. g(x) =x2 + 4x− 1

x + 3

x + 1x + 3 x2 + 4x− 1

x2 + 3xx − 1x + 3

−4

Then g(x) = x + 1 +−4

x + 3. The oblique asymptote is

y = x + 1.

29. h(x) =x4 − 2x3 + 1

xx3 + 1 x4 + 0x3 + 0x2 + 0x

x4 + x− x

Then h(x) = x +−x

x3 + 1. The oblique asymptote is

y = x.

31. f(x) =x3 − x2 + x− 4x2 + 2x− 1

x − 3x2 + 2x− 1 x3 − x2 + x − 4

x3 + 2x2 − x− 3x2 + 2x− 4− 3x2 − 6x+ 3

8x− 7

Then f(x) = x− 3 +8x− 7

x2 + 2x− 1. The oblique asymptote

is y = x− 3.

33. f(x) =1x

1. The numerator and the denominator have no com-mon factors. 0 is the zero of the denominator, sothe domain excludes 0. It is (−∞, 0) ∪ (0,∞). Theline x = 0, or the y-axis, is the vertical asymptote.

2. Because the degree of the numerator is less thanthe degree of the denominator, the x-axis, or y = 0,is the horizontal asymptote. There are no obliqueasymptotes.

3. The numerator has no zeros, so there is no x-intercept.

4. Since 0 is not in the domain of the function, thereis no y-intercept.

5. Find other function values to determine the shapeof the graph and then draw it.

35. h(x) = − 4x2

1. The numerator and the denominator have no com-mon factors. 0 is the zero of the denominator, sothe domain excludes 0. It is (−∞, 0) ∪ (0,∞). Theline x = 0, or the y-axis, is the vertical asymptote.

2. Because the degree of the numerator is less thanthe degree of the denominator, the x-axis, or y = 0,is the horizontal asymptote. There is no obliqueasymptote.

Copyright © 2013 Pearson Education, Inc.

Page 157: JUDITH A. PENNA -  · PDF fileSTUDENT’S SOLUTIONS MANUAL JUDITH A. PENNA Indiana University Purdue University Indianapolis COLLEGE ALGEBRA: GRAPHS AND MODELS FIFTH EDITION

2 4

2

4

�4 �2

�4

�2

x

y

x � 0

h(x) � � 4x2

y � 0

4 8

4

�8 �4

�8

�12

�16

�4

g(x) � x2 � 4x � 3x � 1

x � �1

y � x � 5

x

y

8

4

6

2

�2

�4

4 62�2 x

y�2

x � 5f(x) �

y � 0

x � 5

6

4

42�2�4 x

y

2x � 1x

f(x) �

y � 2

x � 0��q, 0�

154 Chapter 4: Polynomial and Rational Functions

3. The numerator has no zeros, so there is no x-intercept.

4. Since 0 is not in the domain of the function, thereis no y-intercept.

5. Find other function values to determine the shapeof the graph and then draw it.

37. g(x) =x2 − 4x + 3

x + 1=

(x− 1)(x− 3)x + 1

1. The numerator and the denominator have no com-mon factors. The denominator, x+1, is 0 when x =−1, so the domain excludes −1. It is (−∞,−1) ∪(−1,∞). The line x = −1 is the vertical asymptote.

2. The degree of the numerator is 1 greater than thedegree of the denominator, so we divide to find theoblique asymptote.

x − 5x + 1 x2 − 4x+ 3

x2 + x− 5x+ 3− 5x− 5

8

The oblique asymptote is y = x − 5. There is nohorizontal asymptote.

3. The zeros of the numerator are 1 and 3. Thus thex-intercepts are (1, 0) and (3, 0).

4. g(0) =02 − 4 · 0 + 3

0 + 1= 3, so the y-intercept is (0, 3).

5. Find other function values to determine the shapeof the graph and then draw it.

39. f(x) =−2

x− 51. The numerator and the denominator have no com-

mon factors. 5 is the zero of the denominator, sothe domain excludes 5. It is (−∞, 5) ∪ (5,∞). Theline x = 5 is the vertical asymptote.

2. Because the degree of the numerator is less thanthe degree of the denominator, the x-axis, or y = 0,is the horizontal asymptote. There is no obliqueasymptote.

3. The numerator has no zeros, so there is no x-intercept.

4. f(0) =−2

0 − 5=

25, so

(0,

25

)is the y-intercept.

5. Find other function values to determine the shapeof the graph and then draw it.

41. f(x) =2x + 1

x1. The numerator and the denominator have no com-

mon factors. 0 is the zero of the denominator, sothe domain excludes 0. It is (−∞, 0) ∪ (0,∞). Theline x = 0, or the y-axis, is the vertical asymptote.

2. The numerator and denominator have the same de-gree, so the horizontal asymptote is determined bythe ratio of the leading coefficients, 2/1, or 2. Thus,y = 2 is the horizontal asymptote. There is nooblique asymptote.

3. The zero of the numerator is the solution of 2x+1 =

0, or −12. The x-intercept is

(− 1

2, 0)

.

4. Since 0 is not in the domain of the function, thereis no y-intercept.

5. Find other function values to determine the shapeof the graph and then draw it.

43. f(x) =x + 3x2 − 9

=x + 3

(x + 3)(x− 3)1. The domain of the function is (−∞,−3)∪ (−3, 3)∪

(3,∞). The numerator and denominator have thecommon factor x+3. The zeros of the denominatorare −3 and 3, and the zero of the numerator is −3.Since 3 is the only zero of the denominator thatis not a zero of the numerator, the only verticalasymptote is x = 3.

Copyright © 2013 Pearson Education, Inc.

Page 158: JUDITH A. PENNA -  · PDF fileSTUDENT’S SOLUTIONS MANUAL JUDITH A. PENNA Indiana University Purdue University Indianapolis COLLEGE ALGEBRA: GRAPHS AND MODELS FIFTH EDITION

y

x�4

(�3, 0)

�2 2 4

�4

�2

2

4

x � 3

y � 0f(x) � x � 3x2 � 9

4

2

�2

�4

2�2�4�6 x

x � �3

y � 0

yx

x2 � 3xf(x) �

)(0, 13

�1

�2

4

3

2

1

42 31 x

y

�1

1

(x � 2)2f(x) �

y � 0

x � 2

�0, ~�

(�3, 2)4

2

�2

�4

42�2�4�6 x

y

x2 � 2x � 3x2 � 4x � 3

x � �1

y � 1

f(x) �

Exercise Set 4.5 155

2. Because the degree of the numerator is less thanthe degree of the denominator, the x-axis, or y = 0,is the horizontal asymptote. There are no obliqueasymptotes.

3. The zero of the numerator, −3, is not in the domainof the function, so there is no x-intercept.

4. f(0) =0 + 302 − 9

= −13, so the y-intercept is

(0,−1

3

).

5. Find other function values to determine the shapeof the graph and then draw it.

45. f(x) =x

x2 + 3x=

x

x(x + 3)1. The zeros of the denominator are 0 and −3, so the

domain is (−∞,−3) ∪ (−3, 0) ∪ (0,∞). The zeroof the numerator is 0. Since −3 is the only zeroof the denominator that is not also a zero of thenumerator, the only vertical asymptote is x = −3.

2. Because the degree of the numerator is less thanthe degree of the denominator, the x-axis, or y = 0,is the horizontal asymptote. There is no obliqueasymptote.

3. The zero of the numerator is 0, but 0 is not in thedomain of the function, so there is no x-intercept.

4. Since 0 is not in the domain of the function, thereis no y-intercept.

5. Find other function values to determine the shapeof the graph and then draw it, indicating the “hole”when x = 0 with an open circle.

47. f(x) =1

(x− 2)2

1. The numerator and the denominator have no comm-non factors. 2 is the zero of the denominator, so thedomain excludes 2. It is (−∞, 2)∪ (2,∞). The linex = 2 is the vertical asymptote.

2. Because the degree of the numerator is less thanthe degree of the denominator, the x-axis, or y = 0,is the horizontal asymptote. There is no obliqueasymptote.

3. The numerator has no zeros, so there is no x-intercept.

4. f(0) =1

(0 − 2)2=

14, so

(0,

14

)is the y-

intercept.

5. Find other function values to determine the shapeof the graph and then draw it.

49. f(x) =x2 + 2x− 3x2 + 4x + 3

=(x + 3)(x− 1)(x + 3)(x + 1)

1. The zeros of the denominator are −3 and −1, sothe domain is (−∞,−3) ∪ (−3,−1) ∪ (−1,∞). Thezeros of the numerator are −3 and 1. Since −1 isthe only zero of the denominator that is not also azero of the numerator, the only vertical asymptoteis x = −1.

2. The numerator and the denominator have the samedegree, so the horizontal asymptote is determinedby the ratio of the leading coefficients, 1/1, or 1.Thus, y = 1 is the horizontal asymptote. There isno oblique asymptote.

3. The only zero of the numerator that is in the domainof the function is 1, so the only x-intercept is (1, 0).

4. f(0) =02 + 2 · 0 − 302 + 4 · 0 + 3

=−33

= −1, so the

y-intercept is (0,−1).

5. Find other function values to determine the shapeof the graph and then draw it, indicating the “hole”when x = −3 with an open circle.

51. f(x) =1

x2 + 31. The numerator and the denominator have no com-

mon factors. The denominator has no real-numberzeros, so the domain is (−∞,∞) and there is novertical asymptote.

Copyright © 2013 Pearson Education, Inc.

Page 159: JUDITH A. PENNA -  · PDF fileSTUDENT’S SOLUTIONS MANUAL JUDITH A. PENNA Indiana University Purdue University Indianapolis COLLEGE ALGEBRA: GRAPHS AND MODELS FIFTH EDITION

0.5

�0.5

0.5�0.5 x

y

1

x 2 � 3f(x) �

y � 0

�0, a�

4

�2

�4

4

(2, 4)

2�4 x

y

x 2 � 4

x � 2f(x) �

4

2

�4

42�4 x

y

x � 1

x � 2f(x) �

y � 1

x � �2

4

(0, �1)

2

1

�2�4 x

y

x 2 � 3x

2x 3 � 5x 2 � 3xf(x) �

y � 0

x � 3x � �q

156 Chapter 4: Polynomial and Rational Functions

2. Because the degree of the numerator is less thanthe degree of the denominator, the x-axis, or y = 0,is the horizontal asymptote. There is no obliqueasymptote.

3. The numerator has no zeros, so there is no x-intercept.

4. f(0) =1

02 + 3=

13, so

(0,

13

)is the y-intercept.

5. Find other function values to determine the shapeof the graph and then draw it.

53. f(x) =x2 − 4x− 2

=(x + 2)(x− 2)

x− 2= x + 2, x �= 2

The graph is the same as the graph of f(x) = x + 2 ex-cept at x = 2, where there is a hole. Thus the domain is(−∞, 2) ∪ (2,∞). The zero of f(x) = x + 2 is −2, so thex-intercept is (−2, 0); f(0) = 2, so the y-intercept is (0, 2).

55. f(x) =x− 1x + 2

1. The numerator and the denominator have no com-mon factors. −2 is the zero of the denominator, sothe domain excludes −2. It is (−∞,−2)∪ (−2,∞).The line x = −2 is the vertical asymptote.

2. The numerator and denominator have the same de-gree, so the horizontal asymptote is determined bythe ratio of the leading coefficients, 1/1, or 1. Thus,y = 1 is the horizontal asymptote. There is nooblique asymptote.

3. The zero of the numerator is 1, so the x-intercept is(1, 0).

4. f(0) =0 − 10 + 2

= −12, so

(0,−1

2

)is the y-

intercept.

5. Find other function values to determine the shapeof the graph and then draw it.

57. f(x)=x2+3x

2x3−5x2−3x=

x(x+3)x(2x2−5x−3)

=x(x+3)

x(2x+1)(x−3)

1. The zeros of the denominator are 0, −12, and 3, so

the domain is(−∞,−1

2

)∪(− 1

2, 0)∪(0, 3)∪(3,∞).

The zeros of the numerator are 0 and −3. Since−1

2and 3 are the only zeros of the denominator

that are not also zeros of the numerator, the vertical

asymptotes are x = −12

and x = 3.

2. Because the degree of the numerator is less thanthe degree of the denominator, the x-axis, or y = 0,is the horizontal asymptote. There is no obliqueasymptote.

3. The only zero of the numerator that is in the do-main of the function is −3 so the only x-intercept is(−3, 0).

4. 0 is not in the domain of the function, so there is noy-intercept.

5. Find other function values to determine the shapeof the graph and then draw it, indicating the “hole”when x = 0 with an open circle.

59. f(x) =x2 − 9x + 1

=(x + 3)(x− 3)

x + 11. The numerator and the denominator have no com-

mon factors. −1 is the zero of the denominator, sothe domain is (−∞,−1)∪(−1,∞). The line x = −1is the vertical asymptote.

2. Because the degree of the numerator is one greaterthan the degree of the denominator, there is anoblique asymptote. Using division, we find thatx2 − 9x + 1

= x− 1 +−8

x + 1. As |x| becomes very large,

the graph of f(x) gets close to the graph of y = x−1.Thus, the line y = x− 1 is the oblique asymptote.

3. The zeros of the numerator are −3 and 3. Thus, thex-intercepts are (−3, 0) and (3, 0).

Copyright © 2013 Pearson Education, Inc.

Page 160: JUDITH A. PENNA -  · PDF fileSTUDENT’S SOLUTIONS MANUAL JUDITH A. PENNA Indiana University Purdue University Indianapolis COLLEGE ALGEBRA: GRAPHS AND MODELS FIFTH EDITION

4

2

42�2�4 x

y

x 2 � 9

x � 1f(x) �

y � x � 1

x � �1

4

2

�4

42�2�4 x

y

x 2 � x � 2

2x 2 � 1f(x) �

y � q

4

2

�2

�4

4

(1, 5)

2�2�4 x

y

3x 2 � x � 2

x � 1g(x) �

4

2

4�4 x

y

x � 1

x 2 � 2x � 3f(x) �

y � 0x � �1

x � 3

Exercise Set 4.5 157

4. f(0) =02 − 90 + 1

= −9, so (0,−9) is the y-

intercept.

5. Find other function values to determine the shapeof the graph and then draw it.

61. f(x) =x2 + x− 22x2 + 1

=(x + 2)(x− 1)

2x2 + 11. The numerator and the denominator have no com-

mon factors. The denominator has no real-numberzeros, so the domain is (−∞,∞) and there is novertical asymptote.

2. The numerator and the denominator have the samedegree, so the horizontal asymptote is determinedby the ratio of the leading coefficients, 1/2. Thus,y = 1/2 is the horizontal asymptote. There is nooblique asymptote.

3. The zeros of the numerator are −2 and 1. Thus, thex-intercepts are (−2, 0) and (1, 0).

4. f(0) =02 + 0 − 22 · 02 + 1

= −2, so (0,−2) is the

y-intercept.

5. Find other function values to determine the shapeof the graph and then draw it.

63. g(x) =3x2 − x− 2

x− 1=

(3x + 2)(x− 1)x− 1

= 3x + 2, x �= 1

The graph is the same as the graph of g(x) = 3x + 2except at x = 1, where there is a hole. Thus the domainis (−∞, 1) ∪ (1,∞).

The zero of g(x) = 3x + 2 is −23, so the x-intercept is(

− 23, 0)

; g(0) = 2, so the y-intercept is (0, 2).

65. f(x) =x− 1

x2 − 2x− 3=

x− 1(x + 1)(x− 3)

1. The numerator and the denominator have no com-mon factors. The zeros of the denominator are −1and 3. Thus, the domain is (−∞,−1) ∪ (−1, 3) ∪(3,∞) and the lines x = −1 and x = 3 are thevertical asymptotes.

2. Because the degree of the numerator is less thanthe degree of the denominator, the x-axis, or y = 0,is the horizontal asymptote. There is no obliqueasymptote.

3. 1 is the zero of the numerator, so (1, 0) is the x-intercept.

4. f(0) =0 − 1

02 − 2 · 0 − 3=

13, so

(0,

13

)is the

y-intercept.

5. Find other function values to determine the shapeof the graph and then draw it.

67. f(x) =3x2 + 11x− 4x2 + 2x− 8

=(3x− 1)(x + 4)(x + 4)(x− 2)

1. The domain of the function is (−∞,−4)∪ (−4, 2)∪(2,∞). The numerator and the denominator havethe common factor x+ 4. The zeros of the denomi-nator are −4 and 2, and the zeros of the numerator

are13

and −4. Since 2 is the only zero of the de-nominator that is not a zero of the numerator, theonly vertical asymptote is x = 2.

2. The numerator and the denominator have the samedegree, so the horizontal asymptote is determinedby the ratio of the leading coefficients, 3/1, or 3.Thus, y = 3 is the horizontal asymptote. There isno oblique asymptote.

3. The only zero of the numerator that is in the domain

of the function is13, so the x-intercept is

(13, 0)

.

Copyright © 2013 Pearson Education, Inc.

Page 161: JUDITH A. PENNA -  · PDF fileSTUDENT’S SOLUTIONS MANUAL JUDITH A. PENNA Indiana University Purdue University Indianapolis COLLEGE ALGEBRA: GRAPHS AND MODELS FIFTH EDITION

y

x�8 �4��4, ��

4 8

�8

�4

4

8

f(x) � 3x2 � 11x � 4 x2 � 2x � 8 x � 2

y � 3

136

4

2

42�2�4 x

y

x � 3

(x � 1)3f(x) �

y � 0

x � �1

4

2

42�2�4 x

y

x3 � 1x

f(x) �

x � 0

50

20�20 x

y

x3 � 2x2 � 15x

x2 � 5x � 14f (x) �

y � x � 7

x � 7x � �2

158 Chapter 4: Polynomial and Rational Functions

4. f(0) =3 · 02 + 11 · 0 − 4

02 + 2 · 0 − 8=

−4−8

=12, so the

y-intercept is(

0,12

).

5. Find other function values to determine the shapeof the graph and then draw it.

69. f(x) =x− 3

(x + 1)3

1. The numerator and the denominator have no com-mon factors. −1 is the zero of the denominator, sothe domain excludes −1. It is (−∞,−1)∪ (−1,∞).The line x = −1 is the vertical asymptote.

2. Because the degree of the numerator is less thanthe degree of the denominator, the x-axis, or y = 0,is the horizontal asymptote. There is no obliqueasymptote.

3. 3 is the zero of the numerator, so (3, 0) is the x-intercept.

4. f(0) =0 − 3

(0 + 1)3= −3, so (0,−3) is the y-

intercept.

5. Find other function values to determine the shapeof the graph and then draw it.

71. f(x) =x3 + 1

x1. The numerator and the denominator have no com-

mon factors. 0 is the zero of the denominator, sothe domain excludes 0. It is (−∞, 0) ∪ (0,∞). Theline x = 0, or the y-axis, is the vertical asymptote.

2. Because the degree of the numerator is more thanone greater than the degree of the denominator,there is no horizontal or oblique asymptote.

3. The real-number zero of the numerator is −1, so thex-intercept is (−1, 0).

4. Since 0 is not in the domain of the function, thereis no y-intercept.

5. Find other function values to determine the shapeof the graph and then draw it.

73. f(x) =x3 + 2x2 − 15xx2 − 5x− 14

=x(x + 5)(x− 3)(x + 2)(x− 7)

1. The numerator and the denominator have no com-mon factors. The zeros of the denominator are −2and 7. Thus, the domain is (−∞,−2) ∪ (−2, 7) ∪(7,∞) and the lines x = −2 and x = 7 are thevertical asymptotes.

2. Because the degree of the numerator is one greaterthan the degree of the denominator, there is anoblique asymptote. Using division, we find thatx3 + 2x2 − 15xx2 − 5x− 14

= x + 7 +34x + 98

x2 − 5x− 14. As |x| be-

comes very large, the graph of f(x) gets close to thegraph of y = x + 7. Thus, the line y = x + 7 is theoblique asymptote.

3. The zeros of the numerator are 0, −5, and 3. Thus,the x-intercepts are (−5, 0), (0, 0), and (3, 0).

4. From part (3) we see that (0, 0) is the y-intercept.

5. Find other function values to determine the shapeof the graph and then draw it.

75. f(x) =5x4

x4 + 11. The numerator and the denominator have no com-

mon factors. The denominator has no real-numberzeros, so the domain is (−∞,∞) and there is novertical asymptote.

2. The numerator and denominator have the same de-gree, so the horizontal asymptote is determined bythe ratio of the leading coefficients, 5/1, or 5. Thus,y = 5 is the horizontal asymptote. There is nooblique asymptote.

3. The zero of the numerator is 0, so (0, 0) is the x-intercept.

4. From part (3) we see that (0, 0) is the y-intercept.

Copyright © 2013 Pearson Education, Inc.

Page 162: JUDITH A. PENNA -  · PDF fileSTUDENT’S SOLUTIONS MANUAL JUDITH A. PENNA Indiana University Purdue University Indianapolis COLLEGE ALGEBRA: GRAPHS AND MODELS FIFTH EDITION

4

2

42�2�4 x

y

5x 4

x 4 � 1f(x) �

y � 5

4

2

4�2�4 x

y

x 2

x 2 � x � 2f(x) �

x � �1 x � 2

y � 1

P(t) �500t

2t 2 � 9

0 5 10 15 20 25 30

10

20

30

40

50

60

t

P(t)

4

4�4 x

y

2x 3 � x 2 � 8x � 4

x 3 � x 2 � 9x � 9f(x) �

x � �3

x � �1

x � 3

y � 2

Exercise Set 4.5 159

5. Find other function values to determine the shapeof the graph and then draw it.

77. f(x) =x2

x2 − x− 2=

x2

(x + 1)(x− 2)1. The numerator and the denominator have no com-

mon factors. The zeros of the denominator are −1and 2. Thus, the domain is (−∞,−1) ∪ (−1, 2) ∪(2,∞) and the lines x = −1 and x = 2 are thevertical asymptotes.

2. The numerator and denominator have the same de-gree, so the horizontal asymptote is determined bythe ratio of the leading coefficients, 1/1, or 1. Thus,y = 1 is the horizontal asymptote. There is nooblique asymptote.

3. The zero of the numerator is 0, so the x-intercept is(0, 0).

4. From part (3) we see that (0, 0) is the y-intercept.

5. Find other function values to determine the shapeof the graph and then draw it.

79. Answers may vary. The numbers −4 and 5 must be zeros ofthe denominator. A function that satisfies these conditionsis

f(x) =1

(x + 4)(x− 5), or f(x) =

1x2 − x− 20

.

81. Answers may vary. The numbers −4 and 5 must be zeros ofthe denominator and −2 must be a zero of the numerator.In addition, the numerator and denominator must havethe same degree and the ratio of their leading coefficientsmust be 3/2. A function that satisfies these conditions is

f(x) =3x(x + 2)

2(x + 4)(x− 5), or f(x) =

3x2 + 6x2x2 − 2x− 40

.

Another function that satisfies these conditions is

g(x) =3(x + 2)2

2(x + 4)(x− 5), or g(x) =

3x2 + 12x + 122x2 − 2x− 40

.

83. a) The horizontal asymptote of N(t) is the ratio of theleading coefficients of the numerator and denomina-tor, 0.8/5, or 0.16. Thus, N(t) → 0.16 as t → ∞.

b) The medication never completely disappears fromthe body; a trace amount remains.

85. a)

b) P (0) = 0; P (1) = 45.455 thousand, or 45, 455;

P (3) = 55.556 thousand, or 55, 556;

P (8) = 29.197 thousand, or 29, 197

c) The degree of the numerator is less than the degreeof the denominator, so the x-axis is the horizontalasymptote. Thus, P (t) → 0 as t → ∞.

d) Eventually, no one will live in this community.

e) If we graph the function on a graphing calculatorand use the Maximum feature, we see that the max-imum population is 58.926 thousand, or 58,926, andit occurs when t ≈ 2.12 months.

87. domain, range, domain, range

89. slope-intercept equation

91. x-intercept

93. vertical lines

95. y-intercept

97. f(x) =x5 + 2x3 + 4x2

x2 + 2= x3 + 4 +

−8x2 + 2

As |x| → ∞,−8

x2 + 2→ 0 and the value of f(x) → x3 + 4.

Thus, the nonlinear asymptote is y = x3 + 4.

99.

Copyright © 2013 Pearson Education, Inc.

Page 163: JUDITH A. PENNA -  · PDF fileSTUDENT’S SOLUTIONS MANUAL JUDITH A. PENNA Indiana University Purdue University Indianapolis COLLEGE ALGEBRA: GRAPHS AND MODELS FIFTH EDITION

160 Chapter 4: Polynomial and Rational Functions

Exercise Set 4.6

1. x2 + 2x− 15 = 0

(x + 5)(x− 3) = 0

x + 5 = 0 or x− 3 = 0

x = −5 or x = 3

The solution set is {−5, 3}.3. Solve x2 + 2x− 15 ≤ 0.

From Exercise 2 we know the solution set of x2+2x−15 < 0is (−5, 3). The solution set of x2+2x−15 ≤ 0 includes theendpoints of this interval. Thus the solution set is [−5, 3].

5. Solve x2 + 2x− 15 ≥ 0.

From Exercise 4 we know the solution set of x2+2x−15 > 0is (−∞,−5)∪ (3,∞). The solution set of x2 +2x− 15 ≥ 0includes the endpoints −5 and 3. Thus the solution set is(−∞,−5] ∪ [3,∞).

7. Solvex− 2x + 4

> 0.

The denominator tells us that g(x) is not defined whenx = −4. From Exercise 6 we know that g(2) = 0. Thecritical values of −4 and 2 divide the x-axis into threeintervals (−∞,−4), (−4, 2), and (2,∞). We test a valuein each interval.

(−∞,−4): g(−5) = 7 > 0

(−4, 2): g(0) = −12< 0

(2,∞): g(3) =17> 0

Function values are positive on (−∞,−4) and on (2,∞).The solution set is (−∞,−4) ∪ (2,∞).

9. Solvex− 2x + 4

≥ 0.

From Exercise 7 we know that the solution set ofx− 2x + 4

> 0

is (−∞,−4)∪ (2,∞). We include the zero of the function,2, since the inequality symbol is ≥. The critical value−4 is not included because it is not in the domain of thefunction. The solution set is (−∞,−4) ∪ [2,∞).

11. 7x(x− 1)(x + 5)

= 0

7x = 0 Multiplying by (x− 1)(x + 5)

x = 0

The solution set is {0}.

13. Solve7x

(x− 1)(x + 5)≥ 0.

From our work in Exercise 12 we see that function valuesare positive on (−5, 0) and on (1,∞). We also includethe zero of the function, 0, in the solution set because theinequality symbol is ≥. The critical values −5 and 1 arenot included because they are not in the domain of thefunction. The solution set is (−5, 0] ∪ (1,∞).

15. Solve7x

(x− 1)(x + 5)< 0.

From our work in Exercise 12 we see that the solution setis (−∞,−5) ∪ (0, 1).

17. Solve x5 − 9x3 < 0.

From Exercise 16 we know the solutions of the relatedequation are −3, 0, and 3. These numbers divide the x-axisinto the intervals (−∞,−3), (−3, 0), (0, 3), and (3,∞). Wetest a value in each interval.

(−∞,−3): g(−4) = −448 < 0

(−3, 0): g(−1) = 8 > 0

(0, 3): g(1) = −8 < 0

(3,∞): g(4) = 448 > 0

Function values are negative on (−∞,−3) and on (0, 3).The solution set is (−∞,−3) ∪ (0, 3).

19. Solve x5 − 9x3 > 0.

From our work in Exercise 17 we see that the solution setis (−3, 0) ∪ (3,∞).

21. First we find an equivalent inequality with 0 on one side.

x3 + 6x2 < x + 30

x3 + 6x2 − x− 30 < 0

From the graph we see that the x-intercepts of the relatedfunction occur at x = −5, x = −3, and x = 2. They dividethe x-axis into the intervals (−∞,−5), (−5,−3), (−3, 2),and (2,∞). From the graph we see that the function hasnegative values only on (−∞,−5) and (−3, 2). Thus, thesolution set is (−∞,−5) ∪ (−3, 2).

23. By observing the graph or the denominator of the function,we see that the function is not defined for x = −2 or x = 2.We also see that 0 is a zero of the function. These num-bers divide the x-axis into the intervals (−∞,−2), (−2, 0),(0, 2), and (2,∞). From the graph we see that the functionhas positive values only on (−2, 0) and (2,∞). Since theinequality symbol is ≥, 0 must be included in the solutionset. It is (−2, 0] ∪ (2,∞).

25. (x− 1)(x + 4) < 0

The related equation is (x − 1)(x + 4) = 0. Using theprinciple of zero products, we find that the solutions ofthe related equation are 1 and −4. These numbers dividethe x-axis into the intervals (−∞,−4), (−4, 1), and (1,∞).We let f(x) = (x − 1)(x + 4) and test a value in eachinterval.

(−∞,−4): f(−5) = 6 > 0

(−4, 1): f(0) = −4 < 0

(1,∞): f(2) = 6 > 0

Function values are negative only in the interval (−4, 1).The solution set is (−4, 1).

27. x2 + x− 2 > 0 Polynomial inequality

x2 + x− 2 = 0 Related equation

(x + 2)(x− 1) = 0 Factoring

Copyright © 2013 Pearson Education, Inc.

Page 164: JUDITH A. PENNA -  · PDF fileSTUDENT’S SOLUTIONS MANUAL JUDITH A. PENNA Indiana University Purdue University Indianapolis COLLEGE ALGEBRA: GRAPHS AND MODELS FIFTH EDITION

Exercise Set 4.6 161

Using the principle of zero products, we find that the solu-tions of the related equation are −2 and 1. These numbersdivide the x-axis into the intervals (−∞,−2), (−2, 1), and(1,∞). We let f(x) = x2 + x− 2 and test a value in eachinterval.

(−∞,−2): f(−3) = 4 > 0

(−2, 1): f(0) = −2 < 0

(1,∞): f(2) = 4 > 0

Function values are positive on (−∞,−2) and (1,∞). Thesolution set is (−∞,−2) ∪ (1,∞).

29. x2 − x− 5 ≥ x− 2

x2 − 2x− 3 ≥ 0 Polynomial inequality

x2 − 2x− 3 = 0 Related equation

(x + 1)(x− 3) = 0 Factoring

Using the principle of zero products, we find that the solu-tions of the related equation are −1 and 3. The numbersdivide the x-axis into the intervals (−∞,−1), (−1, 3), and(3,∞). We let f(x) = x2 − 2x− 3 and test a value in eachinterval.

(−∞,−1): f(−2) = 5 > 0

(−1, 3): f(0) = −3 < 0

(3,∞): f(4) = 5 > 0

Function values are positive on (−∞,−1) and on (3,∞).Since the inequality symbol is ≥, the endpoints of theintervals must be included in the solution set. It is(−∞,−1] ∪ [3,∞).

31. x2 > 25 Polynomial inequality

x2 − 25 > 0 Equivalent inequality with0 on one side

x2 − 25 = 0 Related equation

(x + 5)(x− 5) = 0 Factoring

Using the principle of zero products, we find that the solu-tions of the related equation are −5 and 5. These numbersdivide the x-axis into the intervals (−∞,−5), (−5, 5), and(5,∞). We let f(x) = x2 − 25 and test a value in eachinterval.

(−∞,−5): f(−6) = 11 > 0

(−5, 5): f(0) = −25 < 0

(5,∞): f(6) = 11 > 0

Function values are positive on (−∞,−5) and (5,∞). Thesolution set is (−∞,−5) ∪ (5,∞).

33. 4 − x2 ≤ 0 Polynomial inequality

4 − x2 = 0 Related equation

(2 + x)(2 − x) = 0 Factoring

Using the principle of zero products, we find that the solu-tions of the related equation are −2 and 2. These numbersdivide the x-axis into the intervals (−∞,−2), (−2, 2), and(2,∞). We let f(x) = 4 − x2 and test a value in eachinterval.

(−∞,−2): f(−3) = −5 < 0

(−2, 2): f(0) = 4 > 0

(2,∞): f(3) = −5 < 0

Function values are negative on (−∞,−2) and(2,∞). Since the inequality symbol is ≤, the endpointsof the intervals must be included in the solution set. It is(−∞,−2] ∪ [2,∞).

35. 6x− 9 − x2 < 0 Polynomial inequality

6x− 9 − x2 = 0 Related equation

−(x2 − 6x + 9) = 0 Factoring out −1 andrearranging

−(x− 3)(x− 3) = 0 FactoringUsing the principle of zero products, we find that the so-lution of the related equation is 3. This number dividesthe x-axis into the intervals (−∞, 3) and (3,∞). We letf(x) = 6x− 9 − x2 and test a value in each interval.

(−∞, 3): f(−4) = −49 < 0

(3,∞): f(4) = −1 < 0

Function values are negative on both intervals. The solu-tion set is (−∞, 3) ∪ (3,∞).

37. x2 + 12 < 4x Polynomial inequality

x2 − 4x + 12 < 0 Equivalent inequality with 0on one side

x2 − 4x + 12 = 0 Related equationUsing the quadratic formula, we find that the relatedequation has no real-number solutions. The graph liesentirely above the x-axis, so the inequality has no so-lution. We could determine this algebraically by lettingf(x) = x2 − 4x + 12 and testing any real number (sincethere are no real-number solutions of f(x) = 0 to dividethe x-axis into intervals). For example, f(0) = 12 > 0,so we see algebraically that the inequality has no solution.The solution set is ∅.

39. 4x3 − 7x2 ≤ 15x Polynomialinequality

4x3 − 7x2 − 15x ≤ 0 Equivalentinequality with 0on one side

4x3 − 7x2 − 15x = 0 Related equation

x(4x + 5)(x− 3) = 0 FactoringUsing the principle of zero products, we find that the so-

lutions of the related equation are 0, −54, and 3. These

numbers divide the x-axis into the intervals(−∞,−5

4

),(

− 54, 0)

, (0, 3), and (3,∞). We let f(x) = 4x3 − 7x2 −15x and test a value in each interval.(−∞,−5

4

): f(−2) = −30 < 0(

− 54, 0)

: f(−1) = 4 > 0

(0, 3) f(1) = −18 < 0

(3,∞): f(4) = 84 > 0

Function values are negative on(−∞,−5

4

)and (0, 3).

Since the inequality symbol is ≤, the endpoints of the

Copyright © 2013 Pearson Education, Inc.

Page 165: JUDITH A. PENNA -  · PDF fileSTUDENT’S SOLUTIONS MANUAL JUDITH A. PENNA Indiana University Purdue University Indianapolis COLLEGE ALGEBRA: GRAPHS AND MODELS FIFTH EDITION

162 Chapter 4: Polynomial and Rational Functions

intervals must be included in the solution set. It is(−∞,−5

4

]∪ [0, 3].

41. x3 + 3x2 − x− 3 ≥ 0 Polynomialinequality

x3 + 3x2 − x− 3 = 0 Related equation

x2(x + 3) − (x + 3) = 0 Factoring

(x2 − 1)(x + 3) = 0

(x + 1)(x− 1)(x + 3) = 0

Using the principle of zero products, we find that the solu-tions of the related equation are −1, 1, and −3. Thesenumbers divide the x-axis into the intervals (−∞,−3),(−3,−1), (−1, 1), and (1,∞). We let f(x) = x3+3x2−x−3and test a value in each interval.

(−∞,−3): f(−4) = −15 < 0

(−3,−1): f(−2) = 3 > 0

(−1, 1): f(0) = −3 < 0

(1,∞): f(2) = 15 > 0

Function values are positive on (−3,−1) and (1,∞). Sincethe inequality symbol is ≥, the endpoints of the intervalsmust be included in the solution set. It is [−3,−1]∪[1,∞).

43. x3 − 2x2 < 5x− 6 Polynomialinequality

x3 − 2x2 − 5x + 6 < 0 Equivalentinequality with 0on one side

x3 − 2x2 − 5x + 6 = 0 Related equation

Using the techniques of Section 3.3, we find that the solu-tions of the related equation are −2, 1, and 3. They dividethe x-axis into the intervals (−∞,−2), (−2, 1), (1, 3), and(3,∞). Let f(x) = x3 − 2x2 − 5x + 6 and test a value ineach interval.

(−∞,−2): f(−3) = −24 < 0

(−2, 1): f(0) = 6 > 0

(1, 3): f(2) = −4 < 0

(3,∞): f(4) = 18 > 0

Function values are negative on (−∞,−2) and (1, 3). Thesolution set is (−∞,−2) ∪ (1, 3).

45. x5 + x2 ≥ 2x3 + 2 Polynomialinequality

x5 − 2x3 + x2 − 2 ≥ 0 Relatedinequality with0 on one side

x5 − 2x3 + x2 − 2 = 0 Relatedequation

x3(x2 − 2) + x2 − 2 = 0 Factoring

(x3 + 1)(x2 − 2) = 0

Using the principle of zero products, we find that the real-number solutions of the related equation are −1, −√

2, and√2. These numbers divide the x-axis into the intervals

(−∞,−√2), (−√

2,−1), (−1,√

2), and (√

2,∞). We letf(x) = x5 − 2x3 + x2 − 2 and test a value in each interval.

(−∞,−√2): f(−2) = −14 < 0

(−√2,−1): f(−1.3) ≈ 0.37107 > 0

(−1,√

2): f(0) = −2 < 0

(√

2,∞): f(2) = 18 > 0

Function values are positive on (−√2,−1) and

(√

2,∞). Since the inequality symbol is ≥, the endpointsof the intervals must be included in the solution set. It is[−√

2,−1] ∪ [√

2,∞).

47. 2x3 + 6 ≤ 5x2 + x Polynomialinequality

2x3 − 5x2 − x + 6 ≤ 0 Equivalentinequality with 0on one side

2x3 − 5x2 − x + 6 = 0 Related equation

Using the techniques of Section 3.3, we find that the solu-

tions of the related equation are −1,32, and 2. We can also

use the graph of y = 2x3 − 5x2 − x + 6 to find these solu-tions. They divide the x-axis into the intervals (−∞,−1),(− 1,

32

),(

32, 2)

, and (2,∞). Let f(x) = 2x3−5x2−x+6

and test a value in each interval.

(−∞,−1): f(−2) = −28 < 0(− 1,

32

): f(0) = 6 > 0(

32, 2)

: f(1.6) = −0.208 < 0

(2,∞): f(3) = 12 > 0

Function values are negative in (−∞,−1) and(32, 2)

. Since the inequality symbol is ≤, the endpoints

of the intervals must be included in the solution set. The

solution set is (−∞,−1] ∪[32, 2].

49. x3 + 5x2 − 25x ≤ 125 Polynomialinequality

x3 + 5x2 − 25x− 125 ≤ 0 Equivalentinequality with 0on one side

x3 + 5x2 − 25x− 125 = 0 Related equation

x2(x + 5) − 25(x + 5) = 0 Factoring

(x2 − 25)(x + 5) = 0

(x + 5)(x− 5)(x + 5) = 0

Using the principle of zero products, we find that the solu-tions of the related equation are −5 and 5. These numbersdivide the x-axis into the intervals (−∞,−5), (−5, 5), and(5,∞). We let f(x) = x3 + 5x2 − 25x − 125 and test avalue in each interval.

(−∞,−5): f(−6) = −11 < 0

(−5, 5): f(0) = −125 < 0

(5,∞): f(6) = 121 > 0

Function values are negative on (−∞,−5) and(−5, 5). Since the inequality symbol is ≤, the endpoints

Copyright © 2013 Pearson Education, Inc.

Page 166: JUDITH A. PENNA -  · PDF fileSTUDENT’S SOLUTIONS MANUAL JUDITH A. PENNA Indiana University Purdue University Indianapolis COLLEGE ALGEBRA: GRAPHS AND MODELS FIFTH EDITION

Exercise Set 4.6 163

of the intervals must be included in the solution set. It is(−∞,−5] ∪ [−5, 5] or (−∞, 5].

51. 0.1x3 − 0.6x2 − 0.1x + 2 < 0 Polynomialinequality

0.1x3 − 0.6x2 − 0.1x + 2 = 0 Related equation

After trying all the possibilities, we find that the re-lated equation has no rational zeros. Using the graph ofy = 0.1x3 − 0.6x2 − 0.1x + 2, we find that the only real-number solutions of the related equation are approximately−1.680, 2.154, and 5.526. These numbers divide thex-axis into the intervals (−∞,−1.680), (−1.680, 2.154),(2.154, 5.526), and (5.526,∞). We let f(x) = 0.1x3 −0.6x2 − 0.1x + 2 and test a value in each interval.

(−∞,−1.680): f(−2) = −1 < 0

(−1.680, 2.154): f(0) = 2 > 0

(2.154, 5.526): f(3) = −1 < 0

(5.526,∞): f(6) = 1.4 > 0

Function values are negative on (−∞,−1.680) and(2.154, 5.526). The graph can also be used to determinethis. The solution set is (−∞,−1.680) ∪ (2.154, 5.526).

53. 1x + 4

> 0 Rational inequality

1x + 4

= 0 Related equation

The denominator of f(x) =1

x + 4is 0 when x = −4, so the

function is not defined for x = −4. The related equationhas no solution. Thus, the only critical value is −4. Itdivides the x-axis into the intervals (−∞,−4) and (−4,∞).We test a value in each interval.

(−∞,−4): f(−5) = −1 < 0

(−4,∞): f(0) =14> 0

Function values are positive on (−4,∞). This can also be

determined from the graph of y =1

x + 4. The solution set

is (−4,∞).

55. −42x + 5

< 0 Rational inequality

−42x + 5

= 0 Related equation

The denominator of f(x) =−4

2x + 5is 0 when x = −5

2,

so the function is not defined for x = −52. The related

equation has no solution. Thus, the only critical value is

−52. It divides the x-axis into the intervals

(−∞,−5

2

)

and(− 5

2,∞)

. We test a value in each interval.(−∞,−5

2

): f(−3) = 4 > 0(

− 52,∞)

: f(0) = −45< 0

Function values are negative on(− 5

2,∞)

. The solution

set is(− 5

2,∞)

.

57. 2xx− 4

≥ 0 Rational inequality

2xx− 4

= 0 Related equation

The denominator of f(x) =2x

x− 4is 0 when x = 4, so the

function is not defined for x = 4.

We solve the related equation f(x) = 0.2x

x− 4= 0

2x = 0 Multiplying by x− 4

x = 0

The critical values are 0 and 4. They divide the x-axis intothe intervals (−∞, 0), (0, 4), and (4,∞). We test a valuein each interval.

(−∞, 0): f(−1) =25> 0

(0, 4): f(1) = −23< 0

(4,∞): f(5) = 10 > 0

Function values are positive on (−∞, 0) and (4,∞). Sincethe inequality symbol is ≥ and f(0) = 0, then 0 must beincluded in the solution set. And since 4 is not in thedomain of f(x), 4 is not included in the solution set. It is(−∞, 0] ∪ (4,∞).

59.x− 4x + 3

− x + 2x− 1

≤ 0

The denominator of f(x) =x− 4x + 3

− x + 2x− 1

is 0 when x =

−3 or x = 1, so the function is not defined for these valuesof x. We solve the related equation f(x) = 0.

x− 4x + 3

− x + 2x− 1

= 0

(x+3)(x−1)(x−4x+3

− x+2x−1

)= (x+3)(x−1)·0

(x− 1)(x− 4) − (x + 3)(x + 2) = 0

x2 − 5x + 4 − (x2 + 5x + 6) = 0

−10x− 2 = 0

−10x = 2

x = −15

The critical values are −3, −15, and 1. They divide the x-

axis into the intervals (−∞,−3),(− 3,−1

5

),(− 1

5, 1)

,

and (1,∞). We test a value in each interval.

(−∞,−3): f(−4) = 7.6 > 0(− 3,−1

5

): f(−1) = −2 < 0(

− 15, 1)

: f(0) =23> 0

Copyright © 2013 Pearson Education, Inc.

Page 167: JUDITH A. PENNA -  · PDF fileSTUDENT’S SOLUTIONS MANUAL JUDITH A. PENNA Indiana University Purdue University Indianapolis COLLEGE ALGEBRA: GRAPHS AND MODELS FIFTH EDITION

164 Chapter 4: Polynomial and Rational Functions

(1,∞): f(2) = −4.4 < 0

Function values are negative on(− 3,−1

5

)and (1,∞).

Note that since the inequality symbol is ≤ and

f

(− 1

5

)= 0, then −1

5must be included in the solution

set. Note also that since neither −3 nor 1 is in the domainof f(x), they are not included in the solution set. It is(− 3,−1

5

]∪ (1,∞).

61. x + 6x− 2

>x− 8x− 5

Rational inequality

x + 6x− 2

− x− 8x− 5

> 0 Equivalent inequalitywith 0 on one side

The denominator of f(x) =x + 6x− 2

− x− 8x− 5

is 0 when x = 2

or x = 5, so the function is not defined for these values ofx. We solve the related equation f(x) = 0.

x + 6x− 2

− x− 8x− 5

= 0

(x− 2)(x− 5)(x + 6x− 2

− x− 8x− 5

)= (x− 2)(x− 5) · 0

(x− 5)(x + 6) − (x− 2)(x− 8) = 0

x2 + x− 30 − (x2 − 10x + 16) = 0

x2 + x− 30 − x2 + 10x− 16 = 0

11x− 46 = 0

11x = 46

x =4611

The critical values are 2,4611

, and 5. They divide the x-axis

into the intervals (−∞, 2),(

2,4611

),(

4611

, 5)

, and (5,∞).

We test a value in each interval.

(−∞, 2): f(0) = −4.6 < 0(2,

4611

): f(4) = 1 > 0(

4611

, 5)

: f(4.5) = −2.8 < 0

(5,∞): f(6) = 5 > 0

Function values are positive on(

2,4611

)and (5,∞). The

solution set is(

2,4611

)∪ (5,∞).

63. x + 1x− 2

≥ 3 Rational inequality

x + 1x− 2

− 3 ≥ 0 Equivalent inequalitywith 0 on one side

The denominator of f(x) =x + 1x− 2

− 3 is 0 when x = 2, so

the function is not defined for this value of x. We solvethe related equation f(x) = 0.

x + 1x− 2

− 3 = 0

(x− 2)(x + 1x− 2

− 3)

= (x− 2) · 0x + 1 − 3(x− 2) = 0

x + 1 − 3x + 6 = 0

−2x + 7 = 0

−2x = −7

x =72

The critical values are 2 and72. They divide the x-axis

into the intervals (−∞, 2),(

2,72

), and

(72,∞)

. We test

a value in each interval.

(−∞, 2): f(0) = −3.5 < 0(2,

72

): f(3) = 1 > 0(

72,∞)

: f(4) = −0.5 < 0

Function values are positive on(

2,72

). Note that since

the inequality symbol is ≥ and f

(72

)= 0, then

72

must

be included in the solution set. Note also that since 2 isnot in the domain of f(x), it is not included in the solution

set. It is(

2,72

].

65. x− 2 >1x

Rational inequality

x− 2 − 1x

> 0 Equivalent inequalitywith 0 on one side

The denominator of f(x) = x− 2 − 1x

is 0 when x = 0, sothe function is not defined for this value of x. We solvethe related equation f(x) = 0.

x− 2 − 1x

= 0

x

(x− 2 − 1

x

)= x · 0

x2 − 2x− x · 1x

= 0

x2 − 2x− 1 = 0

Using the quadratic formula we find that x = 1±√2. The

critical values are 1 − √2, 0, and 1 +

√2. They divide

the x-axis into the intervals (−∞, 1 − √2), (1 − √

2, 0),(0, 1 +

√2), and (1 +

√2,∞). We test a value in each

interval.

(−∞, 1 −√2): f(−1) = −2 < 0

(1 −√2, 0): f(−0.1) = 7.9 > 0

(0, 1 +√

2): f(1) = −2 < 0

(1 +√

2,∞): f(3) =23> 0

Copyright © 2013 Pearson Education, Inc.

Page 168: JUDITH A. PENNA -  · PDF fileSTUDENT’S SOLUTIONS MANUAL JUDITH A. PENNA Indiana University Purdue University Indianapolis COLLEGE ALGEBRA: GRAPHS AND MODELS FIFTH EDITION

Exercise Set 4.6 165

Function values are positive on (1 −√2, 0) and

(1 +√

2,∞). The solution set is(1 −√

2, 0) ∪ (1 +√

2,∞).

67.2

x2 − 4x + 3≤ 5

x2 − 92

x2 − 4x + 3− 5

x2 − 9≤ 0

2(x− 1)(x− 3)

− 5(x + 3)(x− 3)

≤ 0

The denominator of f(x) =2

(x− 1)(x− 3)− 5

(x + 3)(x− 3)is 0 when x = 1, 3, or −3,

so the function is not defined for these values of x. Wesolve the related equation f(x) = 0.

2(x− 1)(x− 3)

− 5(x + 3)(x− 3)

= 0

(x−1)(x−3)(x+3)(

2(x−1)(x−3)

− 5(x+3)(x−3)

)= (x− 1)(x− 3)(x + 3) · 0

2(x + 3) − 5(x− 1) = 0

2x + 6 − 5x + 5 = 0

−3x + 11 = 0

−3x = −11

x =113

The critical values are −3, 1, 3, and113

. They divide the x-

axis into the intervals (−∞,−3), (−3, 1), (1, 3),(

3,113

),

and(

113,∞)

. We test a value in each interval.

(−∞,−3): f(−4) ≈ −0.6571 < 0

(−3, 1): f(0) ≈ 1.2222 > 0

(1, 3): f(2) = −1 < 0(3,

113

): f(3.5) ≈ 0.6154 > 0(

113,∞)

: f(4) ≈ −0.0476 < 0

Function values are negative on (−∞,−3), (1, 3), and(113,∞)

. Note that since the inequality symbol is ≤ and

f

(113

)= 0, then

113

must be included in the solution set.

Note also that since −3, 1, and 3 are not in the domainof f(x), they are not included in the solution set. It is

(−∞,−3) ∪ (1, 3) ∪[113,∞)

.

69.3

x2 + 1≥ 6

5x2 + 23

x2 + 1− 6

5x2 + 2≥ 0

The denominator of f(x) =3

x2 + 1− 6

5x2 + 2has no real-

number zeros. We solve the related equation f(x) = 0.

3x2 + 1

− 65x2 + 2

= 0

(x2 + 1)(5x2 + 2)(

3x2 + 1

− 65x2 + 2

)=

(x2 + 1)(5x2 + 2) · 03(5x2 + 2) − 6(x2 + 1) = 0

15x2 + 6 − 6x2 − 6 = 0

9x2 = 0

x2 = 0

x = 0

The only critical value is 0. It divides the x-axis into theintervals (−∞, 0) and (0,∞). We test a value in each in-terval.

(−∞, 0): f(−1) ≈ 0.64286 > 0

(0,∞): f(1) ≈ 0.64286 > 0

Function values are positive on both intervals. Note thatsince the inequality symbol is ≥ and f(0) = 0, then 0 mustbe included in the solution set. It is (−∞, 0] ∪ [0,∞), or(−∞,∞).

71. 5x2 + 3x

<3

2x + 15

x2 + 3x− 3

2x + 1< 0

5x(x + 3)

− 32x + 1

< 0

The denominator of f(x) =5

x(x + 3)− 3

2x + 1is 0 when

x = 0, −3, or −12, so the function is not defined for these

values of x. We solve the related equation f(x) = 0.5

x(x + 3)− 3

2x + 1= 0

x(x+3)(2x+1)(

5x(x+3)

− 32x+1

)=

x(x+3)(2x+1)·05(2x + 1) − 3x(x + 3) = 0

10x + 5 − 3x2 − 9x = 0

−3x2 + x + 5 = 0

Using the quadratic formula we find that

x =1 ±√

616

. The critical values are −3,1 −√

616

, −12,

0, and1 +

√61

6. They divide the x-axis into

the intervals (−∞,−3),(− 3,

1 −√61

6

),(

1 −√61

6,−1

2

),(− 1

2, 0)

,(

0,1 +

√61

6

), and(

1 +√

616

,∞)

.

We test a value in each interval.

(−∞,−3): f(−4) ≈ 1.6786 > 0(− 3,

1 −√61

6

): f(−2) = −1.5 < 0

Copyright © 2013 Pearson Education, Inc.

Page 169: JUDITH A. PENNA -  · PDF fileSTUDENT’S SOLUTIONS MANUAL JUDITH A. PENNA Indiana University Purdue University Indianapolis COLLEGE ALGEBRA: GRAPHS AND MODELS FIFTH EDITION

166 Chapter 4: Polynomial and Rational Functions

(1 −√

616

,−12

): f(−1) = 0.5 > 0(

− 12, 0)

: f(−0.1) ≈ −20.99 < 0

(0,

1 +√

616

): f(1) = 0.25 > 0

(1 +

√61

6,∞)

: f(2) = −0.1 < 0

Function values are negative on(− 3,

1 −√61

6

),

(− 1

2, 0)

and(

1 +√

616

,∞)

. The solution set is

(− 3,

1 −√61

6

)∪(− 1

2, 0)∪(

1 +√

616

,∞)

.

73. 5x7x− 2

>x

x + 15x

7x− 2− x

x + 1> 0

The denominator of f(x) =5x

7x− 2− x

x + 1is 0

when x =27

or x = −1, so the function is not defined for

these values of x. We solve the related equation f(x) = 0.5x

7x− 2− x

x + 1= 0

(7x−2)(x+1)(

5x7x−2

− x

x+1

)= (7x−2)(x+1) · 0

5x(x + 1) − x(7x− 2) = 0

5x2 + 5x− 7x2 + 2x = 0

−2x2 + 7x = 0

−x(2x− 7) = 0

x = 0 or x =72

The critical values are −1, 0,27, and

72. They divide the x-

axis into the intervals (−∞,−1), (−1, 0),(

0,27

),(

27,72

),

and(

72,∞)

. We test a value in each interval.

(−∞,−1): f(−2) = −1.375 < 0

(−1, 0): f(−0.5) ≈ 1.4545 > 0(0,

27

): f(0.1) ≈ −0.4755 < 0(

27,72

): f(1) = 0.5 > 0(

72,∞)

: f(4) ≈ −0.0308 < 0

Function values are positive on (−1, 0) and(

27,72

). The

solution set is (−1, 0) ∪(

27,72

).

75.x

x2+4x−5+

3x2−25

≤2x

x2−6x+5x

x2+4x−5+

3x2−25

− 2xx2−6x+5

≤ 0

x

(x+5)(x−1)+

3(x+5)(x−5)

− 2x(x−5)(x−1)

≤ 0

The denominator of

f(x)=x

(x+5)(x−1)+

3(x+5)(x−5)

− 2x(x−5)(x−1)

is 0 when x = −5, 1, or 5, so the function is not defined forthese values of x. We solve the related equation f(x) = 0.

x

(x+5)(x−1)+

3(x+5)(x−5)

− 2x(x−5)(x−1)

= 0

x(x− 5) + 3(x− 1) − 2x(x + 5) = 0

Multiplying by (x+5)(x− 1)(x− 5)

x2 − 5x + 3x− 3 − 2x2 − 10x = 0

−x2 − 12x− 3 = 0

x2 + 12x + 3 = 0

Using the quadratic formula, we find that x = −6 ±√33.

The critical values are −6−√33, −5, −6 +

√33, 1, and 5.

They divide the x-axis into the intervals (−∞,−6−√33),

(−6−√33,−5), (−5,−6+

√33), (−6+

√33, 1), (1, 5), and

(5,∞). We test a value in each interval.

(−∞,−6 −√33): f(−12) ≈ 0.00194 > 0

(−6 −√33,−5): f(−6) ≈ −0.4286 < 0

(−5,−6 +√

33): f(−1) ≈ 0.16667 > 0

(−6 +√

33, 1): f(0) = −0.12 < 0

(1, 5): f(2) ≈ 1.4762 > 0

(5,∞): f(6) ≈ −2.018 < 0

Function values are negative on (−6 −√33,−5),

(−6 +√

33, 1), and (5,∞). Note that since the inequalitysymbol is ≤ and f(−6 ± √

33) = 0, then −6 − √33 and

−6 +√

33 must be included in the solution set. Note alsothat since −5, 1, and 5 are not in the domain of f(x), theyare not included in the solution set. It is [−6−√

33,−5)∪[−6 +

√33, 1) ∪ (5,∞).

77. We write and solve a rational inequality.4t

t2 + 1+ 98.6 > 100

4tt2 + 1

− 1.4 > 0

The denominator of f(t) =4t

t2 + 1− 1.4 has no real-

number zeros. We solve the related equation f(t) = 0.4t

t2 + 1− 1.4 = 0

4t− 1.4(t2 + 1) = 0 Multiplying by t2 + 1

4t− 1.4t2 − 1.4 = 0

Using the quadratic formula, we find that

t =4 ±√

8.162.8

; that is, t ≈ 0.408 or t ≈ 2.449. These

Copyright © 2013 Pearson Education, Inc.

Page 170: JUDITH A. PENNA -  · PDF fileSTUDENT’S SOLUTIONS MANUAL JUDITH A. PENNA Indiana University Purdue University Indianapolis COLLEGE ALGEBRA: GRAPHS AND MODELS FIFTH EDITION

Exercise Set 4.6 167

numbers divide the t-axis into the intervals (−∞, 0.408),(0.408, 2.449), and (2.449,∞). We test a value in eachinterval.

(−∞, 0.408): f(0) = −1.4 < 0

(0.408, 2.449): f(1) = 0.6 > 0

(2.449,∞): f(3) = −0.2 < 0

Function values are positive on (0.408, 2.449). The solutionset is (0.408, 2.449).

79. a) We write and solve a polynomial inequality.

−3x2 + 630x− 6000 > 0 (x ≥ 0)

We first solve the related equation.

−3x2 + 630x− 6000 = 0

x2 − 210x + 2000 = 0 Dividing by −3

(x− 10)(x− 200) = 0 Factoring

Using the principle of zero products or by observ-ing the graph of y = −3x2 + 630 − 6000, we seethat the solutions of the related equation are 10 and200. These numbers divide the x-axis into the in-tervals (−∞, 10), (10, 200), and (200,∞). Since weare restricting our discussion to nonnegative valuesof x, we consider the intervals [0, 10), (10, 200), and(200,∞).

We let f(x) = −3x2 + 630x− 6000 and test a valuein each interval.

[0, 10): f(0) = −6000 < 0

(10, 200): f(11) = 567 > 0

(200,∞): f(201) = −573 < 0Function values are positive only on (10, 200). Thesolution set is {x|10 < x < 200}, or (10, 200).

b) From part (a), we see that function values are neg-ative on [0, 10) and (200,∞). Thus, the solution setis {x|0 < x < 10 or x > 200}, or (0, 10) ∪ (200,∞).

81. We write an inequality.

27 ≤ n(n− 3)2

≤ 230

54 ≤ n(n− 3) ≤ 460 Multiplying by 2

54 ≤ n2 − 3n ≤ 460

We write this as two inequalities.

54 ≤ n2 − 3n and n2 − 3n ≤ 460

Solve each inequality.

n2 − 3n ≥ 54

n2 − 3n− 54 ≥ 0

n2 − 3n− 54 = 0 Related equation

(n + 6)(n− 9) = 0n = −6 or n = 9

Since only positive values of n have meaning in this ap-plication, we consider the intervals (0, 9) and (9,∞). Letf(n) = n2 − 3n− 54 and test a value in each interval.

(0, 9): f(1) = −56 < 0

(9,∞): f(10) = 16 > 0

Function values are positive on (9,∞). Since the inequalitysymbol is ≥, 9 must also be included in the solution setfor this portion of the inequality. It is {n|n ≥ 9}.

Now solve the second inequality.

n2 − 3n ≤ 460

n2 − 3n− 460 ≤ 0

n2 − 3n− 460 = 0 Related equation

(n + 20)(n− 23) = 0n = −20 or n = 23

We consider only positive values of n as above. Thus,we consider the intervals (0, 23) and (23,∞). Let f(n) =n2 − 3n− 460 and test a value in each interval.

(0, 23): f(1) = −462 < 0

(23,∞): f(24) = 44 > 0

Function values are negative on (0, 23). Since the inequal-ity symbol is ≤, 23 must also be included in the solutionset for this portion of the inequality. It is {n|0 < n ≤ 23}.

The solution set of the original inequality is{n|n ≥ 9 and 0 < n ≤ 23}, or {n|9 ≤ n ≤ 23}.

83. (x− h)2 + (y − k)2 = r2

[x− (−2)]2 + (y − 4)2 = 32

(x + 2)2 + (y − 4)2 = 9

85. h(x) = −2x2 + 3x− 8

a) − b

2a= − 3

2(−2)=

34

h

(34

)= −2

(34

)2

+ 3 · 34− 8 = −55

8

The vertex is(

34,−55

8

).

b) The coefficient of x2 is negative, so there is a maxi-mum value. It is the second coordinate of

the vertex, −558

. It occurs at x =34.

c) The range is(−∞,−55

8

].

87. |x2 − 5| = |5 − x2| = 5 − x2 when 5 − x2 ≥ 0. Thus wesolve 5 − x2 ≥ 0.

5 − x2 ≥ 0

5 − x2 = 0 Related equation

5 = x2

±√5 = x

Let f(x) = 5− x2 and test a value in each of the intervalsdetermined by the solutions of the related equation.

(−∞,−√5): f(−3) = −4 < 0

(−√5,√

5): f(0) = 5 > 0

(√

5,∞): f(3) = −4 < 0

Function values are positive on (−√5,√

5). Since the in-equality symbol is ≥, the endpoints of the interval mustbe included in the solution set. It is

[−√5,√

5].

Copyright © 2013 Pearson Education, Inc.

Page 171: JUDITH A. PENNA -  · PDF fileSTUDENT’S SOLUTIONS MANUAL JUDITH A. PENNA Indiana University Purdue University Indianapolis COLLEGE ALGEBRA: GRAPHS AND MODELS FIFTH EDITION

168 Chapter 4: Polynomial and Rational Functions

89. 2|x|2 − |x| + 2 ≤ 5

2|x|2 − |x| − 3 ≤ 0

2|x|2 − |x| − 3 = 0 Related equation

(2|x| − 3)(|x| + 1) = 0 Factoring

2|x| − 3 = 0 or |x| + 1 = 0

|x| =32

or |x| = −1

The solution of the first equation is x = −32

or x =32.

The second equation has no solution. Let f(x) = 2|x|2 −|x| − 3 and test a value in each interval determined by thesolutions of the related equation.(−∞,−3

2

): f(−2) = 3 > 0(

− 32,32

): f(0) = −3 < 0(

32,∞)

: f(2) = 3 > 0

Function values are negative on(− 3

2,32

). Since the in-

equality symbol is ≤, the endpoints of the interval must

also be included in the solution set. It is[− 3

2,32

].

91.∣∣∣∣∣1 +

1x

∣∣∣∣∣ < 3

−3 < 1 +1x

< 3

−3 < 1 +1x

and 1 +1x< 3

First solve −3 < 1 +1x

.

0 < 4 +1x

, or1x

+ 4 > 0

The denominator of f(x) =1x

+ 4 is 0 when x = 0, so thefunction is not defined for this value of x. Now solve therelated equation.

1x

+ 4 = 0

1 + 4x = 0 Multiplying by x

x = −14

The critical values are −14

and 0. Test a value in each ofthe intervals determined by them.(−∞,−1

4

): f(−1) = 3 > 0(

− 14, 0)

: f(−0.1) = −6 < 0

(0,∞): f(1) = 5 > 0

The solution set for this portion of the inequality is(−∞,−1

4

)∪ (0,∞).

Next solve 1 +1x< 3, or

1x− 2 < 0. The denominator of

f(x) =1x− 2 is 0 when x = 0, so the function is not de-

fined for this value of x. Now solve the related equation.1x− 2 = 0

1 − 2x = 0 Multiplying by x

x =12

The critical values are 0 and12. Test a value in each of the

intervals determined by them.

(−∞, 0): f(−1) = −3 < 0(0,

12

): f(0.1) = 8 > 0(

12,∞)

: f(1) = −1 < 0

The solution set for this portion of the inequality is

(−∞, 0) ∪(

12,∞)

.

The solution set of the original inequality is((−∞,−1

4

)∪ (0,∞)

)and

((−∞, 0) ∪

(12,∞))

,

or(−∞,−1

4

)∪(

12,∞)

.

93. First find a quadratic equation with solutions −4 and 3.

(x + 4)(x− 3) = 0

x2 + x− 12 = 0

Test a point in each of the three intervals determined by−4 and 3.

(−∞,−4): (−5 + 4)(−5 − 3) = 8 > 0

(−4, 3): (0 + 4)(0 − 3) = −12 < 0

(3,∞): (4 + 4)(4 − 3) = 8 > 0

Then a quadratic inequality for which the solution set is(−4, 3) is x2 + x− 12 < 0. Answers may vary.

95. f(x) =

√72

x2 − 4x− 21The radicand must be nonnegative and the denominatormust be nonzero. Thus, the values of x for which x2−4x−21 > 0 comprise the domain. By inspecting the graph ofy=x2−4x−21 we see that the domain is {x|x<−3 or x>7},or (−∞,−3) ∪ (7,∞).

Chapter 4 Review Exercises

1. f(−b) = (−b+a)(−b+b)(−b−c) = (−b+a)·0·(−b−c) = 0,so the statement is true.

3. In addition to the given possibilities, 9 and −9 are alsopossible rational zeros. The statement is false.

Copyright © 2013 Pearson Education, Inc.

Page 172: JUDITH A. PENNA -  · PDF fileSTUDENT’S SOLUTIONS MANUAL JUDITH A. PENNA Indiana University Purdue University Indianapolis COLLEGE ALGEBRA: GRAPHS AND MODELS FIFTH EDITION

g(x) � (x � 1)3(x � 2)2

y

x�4 �2 2 4

�8

�12

�16

�4

4

Chapter 4 Review Exercises 169

5. The domain of the function is the set of all real numbersexcept −2 and 3, or {x|x �= −2 and x �= 3}. The statementis false.

7. f(x) = x3 + 3x2 − 2x− 6

Graph the function and use the Zero, Maximum and Min-imum features.

a) Zeros: −3, −1.414, 1.414

b) Relative maximum: 2.303 at x = −2.291

c) Relative minimum: −6.303 at x = 0.291

d) Domain: all real numbers; range: all real numbers

9. f(x) = 7x2 − 5 + 0.45x4 − 3x3

= 0.45x4 − 3x3 + 7x2 − 5

The leading term is 0.45x4 and the leading coefficient is0.45. The degree of the polynomial is 4, so the polynomialis quartic.

11. g(x) = 6 − 0.5x

= −0.5x + 6

The leading term is −0.5x and the leading coefficient is−0.5. The degree of the polynomial is 1, so the polynomialis linear.

13. f(x) = −12x4 + 3x2 + x− 6

The leading term is −12x4. The degree, 4, is even and the

leading coefficient, −12, is negative. As x → ∞,

f(x) → −∞, and as x → −∞, f(x) → −∞.

15. g(x) =(x− 2

3

)(x + 2)3(x− 5)2

23, multiplicity 1;

−2, multiplicity 3;

5, multiplicity 2

17. h(x) = x3 + 4x2 − 9x− 36

= x2(x + 4) − 9(x + 4)

= (x + 4)(x2 − 9)

= (x + 4)(x + 3)(x− 3)

−4, ±3; each has multiplicity 1

19. a) Linear: f(x) = 0.5408695652x− 30.30434783;quadratic: f(x) = 0.0030322581x2 −0.5764516129x + 57.53225806;cubic: f(x) = 0.0000247619x3 − 0.0112857143x2 +2.002380952x− 82.14285714

b) For the linear function, f(300) ≈ 132; for thequadratic function, f(300) ≈ 158; for the cubicfunction, f(300) ≈ 171. Since the cubic functionyields the result that is closest to the actual num-ber of 180, it makes the best prediction.

c) Using the cubic function, we have: f(350) ≈ 298and f(400) ≈ 498.

21. g(x) = (x− 1)3(x + 2)2

1. The leading term is x ·x ·x ·x ·x, or x5. The degree,5, is odd and the leading coefficient, 1, is positive soas x → ∞, g(x) → ∞ and as x → −∞, g(x) → −∞.

2. We see that the zeros of the function are 1 and−2, so the x-intercepts of the graph are (1, 0) and(−2, 0).

3. The zeros divide the x-axis into 3 intervals,(−∞,−2), (−2, 1), and (1,∞). We choose a valuefor x from each interval and find g(x). This tells usthe sign of g(x) for all values of x in that interval.In (−∞,−2), test −3:

g(−3) = (−3 − 1)3(−3 + 2)2 = −64 < 0

In (−2, 1), test 0:

g(0) = (0 − 1)3(0 + 2)2 = −4 < 0

In (1,∞), test 2:

g(2) = (2 − 1)3(2 + 2)2 = 16 > 0Thus the graph lies below the x-axis on (−∞,−2)and on (−2, 1) and above the x-axis on (1,∞). Wealso know that the points (−3,−64), (0,−4), and(2, 16) are on the graph.

4. From Step 3 we know that g(0) = −4, so the y-intercept is (0,−4).

5. We find additional points on the graph and thendraw the graph.

x g(x)

−2.5 −10.7

−1 −8

−0.5 −7.6

0.5 −0.8

6. Checking the graph as described on page 317 in thetext, we see that it appears to be correct.

23. f(x) = x4 − 5x3 + 6x2 + 4x− 8

1. The leading term is x4. The degree, 4, is even andthe leading coefficient, 1, is positive so as x → ∞,f(x) → ∞ and as x → −∞, f(x) → ∞.

2. We solve f(x) = 0, or x4 − 5x3 + 6x2 + 4x− 8 = 0.The possible rational zeros are ±1, ±2, ±4, and ±8.We try −1.

−1∣∣ 1 −5 6 4 −8

−1 6 −12 81 −6 12 −8 0

Now we have (x + 1)(x3 − 6x2 + 12x − 8) = 0. Weuse synthetic division to determine if 2 is a zero ofx3 − 6x2 + 12x− 8 = 0.2∣∣ 1 −6 12 −8

2 −8 81 −4 4 0

Copyright © 2013 Pearson Education, Inc.

Page 173: JUDITH A. PENNA -  · PDF fileSTUDENT’S SOLUTIONS MANUAL JUDITH A. PENNA Indiana University Purdue University Indianapolis COLLEGE ALGEBRA: GRAPHS AND MODELS FIFTH EDITION

f(x) � x4 � 5x3 � 6x2 � 4x � 8

y

x�4 �2 2 4

�8

�4

4

8

170 Chapter 4: Polynomial and Rational Functions

We have (x + 1)(x− 2)(x2 − 4x + 4) = 0, or(x + 1)(x − 2)(x − 2)2 = 0. Thus the zeros of f(x)are −1 and 2 and the x-intercepts of the graph are(−1, 0) and (2, 0).

3. The zeros divide the x-axis into 3 intervals,(−∞,−1), (−1, 2), and (2,∞). We choose a valuefor x from each interval and find f(x). This tells usthe sign of f(x) for all values of x in that interval.In (−∞,−1), test −2:

f(−2) = (−2)4 − 5(−2)3 + 6(−2)2 + 4(−2) − 8 =64 > 0

In (−1, 2), test 0:

f(0) = 04 − 5 · 03 + 6 · 02 + 4 · 0 − 8 = −8 < 0

In (2,∞), test 3:

f(3) = 34 − 5 · 33 + 6 · 32 + 4 · 3 − 8 = 4 > 0Thus the graph lies above the x-axis on (−∞,−1)and on (2,∞) and below the x-axis on (−1, 2). Wealso know that the points (−2, 64), (0,−8), and(3, 4) are on the graph.

4. From Step 3 we know that f(0) = −8, so the y-intercept is (0,−8).

5. We find additional points on the graph and thendraw the graph.

x f(x)

−1.5 21.4

−0.5 −7.8

1 −2

4 40

6. Checking the graph as described on page 317 in thetext, we see that it appears to be correct.

25. f(1) = 4 · 12 − 5 · 1 − 3 = −4

f(2) = 4 · 22 − 5 · 2 − 3 = 3

By the intermediate value theorem, since f(1) and f(2)have opposite signs, f(x) has a zero between 1 and 2.

27. 6x2 + 16x + 52x− 3 6x3 − 2x2 + 4x − 1

6x3 − 18x2

16x2 + 4x16x2 − 48x

52x− 152x− 156

155

Q(x) = 6x2 + 16x + 52; R(x) = 155;

P (x) = (x− 3)(6x2 + 16x + 52) + 155

29. 5∣∣ 1 2 −13 10

5 35 1101 7 22 120

The quotient is x2 + 7x + 22; the remainder is 120.

31. −1∣∣ 1 0 0 0 −2 0

−1 1 −1 1 11 −1 1 −1 −1 1

The quotient is x4 − x3 + x2 − x− 1; the remainder is 1.

33. −2∣∣ 1 0 0 0 −16

−2 4 −8 161 −2 4 −8 0

f(−2) = 0

35. −i∣∣ 1 −5 1 −5

−i −1 + 5i 51 −5 − i 5i 0

f(−i) = 0, so −i is a zero of f(x).

−5∣∣ 1 −5 1 −5

−5 50 −2551 −10 51 −260

f(−5) �= 0, so −5 is not a zero of f(x).

37.13

∣∣ 1 − 43 − 5

323

13 − 1

3 − 23

1 −1 −2 0

f

(13

)= 0, so

13

is a zero of f(x).

1∣∣ 1 − 4

3 − 53

23

1 − 13 −2

1 − 13 −2 − 4

3

f(1) �= 0, so 1 is not a zero of f(x).

39. f(x) = x3 + 2x2 − 7x + 4

Try x+ 1 and x+ 2. Using synthetic division we find thatf(−1) �= 0 and f(−2) �= 0. Thus x + 1 and x + 2 are notfactors of f(x). Try x + 4.

−4∣∣ 1 2 −7 4

−4 8 −41 −2 1 0

Since f(−4) = 0, x + 4 is a factor of f(x). Thus f(x) =(x + 4)(x2 − 2x + 1) = (x + 4)(x− 1)2.

Now we solve f(x) = 0.x + 4 = 0 or (x− 1)2 = 0

x = −4 or x− 1 = 0

x = −4 or x = 1

The solutions of f(x) = 0 are −4 and 1.

41. f(x) = x4 − 4x3 − 21x2 + 100x− 100

Using synthetic division we find that f(2) = 0:

2∣∣ 1 −4 −21 100 −100

2 −4 −50 1001 −2 −25 50 0

Then we have:f(x) = (x− 2)(x3 − 2x2 − 25x + 50)

= (x− 2)[x2(x− 2) − 25(x− 2)]

= (x− 2)(x− 2)(x2 − 25)

= (x− 2)2(x + 5)(x− 5)

Copyright © 2013 Pearson Education, Inc.

Page 174: JUDITH A. PENNA -  · PDF fileSTUDENT’S SOLUTIONS MANUAL JUDITH A. PENNA Indiana University Purdue University Indianapolis COLLEGE ALGEBRA: GRAPHS AND MODELS FIFTH EDITION

Chapter 4 Review Exercises 171

Now solve f(x) = 0.

(x− 2)2 = 0 or x + 5 = 0 or x− 5 = 0

x− 2 = 0 or x = −5 or x = 5

x = 2 or x = −5 or x = 5

The solutions of f(x) = 0 are 2, −5, and 5.

43. A polynomial function of degree 3 with −4, −1, and 2as zeros has factors x + 4, x + 1, and x − 2 so we havef(x) = an(x + 4)(x + 1)(x− 2).

The simplest polynomial is obtained if we let an = 1.

f(x) = (x + 4)(x + 1)(x− 2)

= (x2 + 5x + 4)(x− 2)

= x3 − 2x2 + 5x2 − 10x + 4x− 8

= x3 + 3x2 − 6x− 8

45. A polynomial function of degree 3 with12, 1 − √

2, and

1 +√

2 as zeros has factors x− 12, x− (1 −√

2), and

x− (1 +√

2) so we have

f(x) = an

(x− 1

2

)[x− (1 −

√2)][x− (1 +

√2)].

Let an = 1.

f(x) =(x− 1

2

)[x− (1 −

√2)][x− (1 +

√2)]

=(x− 1

2

)[(x− 1) +

√2][(x− 1) −

√2]

=(x− 1

2

)(x2 − 2x + 1 − 2)

=(x− 1

2

)(x2 − 2x− 1)

= x3 − 2x2 − x− 12x2 + x +

12

= x3 − 52x2 +

12

If we let an = 2, we obtain f(x) = 2x3 − 5x2 + 1.

47. A polynomial function of degree 5 has at most 5 real ze-ros. Since 5 zeros are given, these are all of the zeros ofthe desired function. We proceed as in Exercise 39 above,letting an = 1.

f(x) = (x + 3)2(x− 2)(x− 0)2

= (x2 + 6x + 9)(x3 − 2x2)

= x5 + 6x4 + 9x3 − 2x4 − 12x3 − 18x2

= x5 + 4x4 − 3x3 − 18x2

49. A polynomial function of degree 5 can have at most 5 zeros.Since f(x) has rational coefficients, in addition to the 3given zeros, the other zeros are the conjugates of 1 +

√3

and −√3, or 1 −√

3 and√

3.

51. −√11 is also a zero.

f(x) = (x−√11)(x +

√11)

= x2 − 11

53. 1 − i is also a zero.f(x) = (x + 1)(x− 4)[x− (1 + i)][x− (1 − i)]

= (x2 − 3x− 4)(x2 − 2x + 2)

= x4 − 2x3 + 2x2 − 3x3 + 6x2 − 6x− 4x2 + 8x− 8

= x4 − 5x3 + 4x2 + 2x− 8

55. f(x) =(x− 1

3

)(x− 0)(x + 3)

=(x2 − 1

3x

)(x + 3)

= x3 +83x2 − x

57. g(x) = 3x4 − x3 + 5x2 − x + 1Possibilities for p

Possibilities for q:

±1±1,±3

Possibilities for p/q: ±1,±13

59. f(x) = 3x5 + 2x4 − 25x3 − 28x2 + 12x

a) We know that 0 is a zero since

f(x) = x(3x4 + 2x3 − 25x2 − 28x + 12).

Now consider g(x) = 3x4 + 2x3 − 25x2 − 28x + 12.

Possibilities for p/q: ±1,±2,±3,±4,±6,±12,

±13,±2

3,±4

3From the graph of y = 3x4+2x3−25x2−28x+12, we

see that, of all the possibilities above, only −2,13,

23, and 3 might be zeros. We use synthetic division

to determine if −2 is a zero.−2∣∣ 3 2 −25 −28 12

−6 8 34 −123 −4 −17 6 0

Now try 3 in the quotient above.

3∣∣ 3 −4 −17 6

9 15 −63 5 −2 0

We have f(x) = (x + 2)(x− 3)(3x2 + 5x− 2).

We find the other zeros.3x3 + 5x− 2 = 0

(3x− 1)(x + 2) = 0

3x− 1 = 0 or x + 2 = 0

3x = 1 or x = −2

x =13

or x = −2

The rational zeros of g(x) = 3x4+2x3−25x2−28x+

12 are −2, 3, and13. Since 0 is also a zero of f(x),

the zeros of f(x) are −2, 3,13, and 0. (The zero −2

has multiplicity 2.) These are the only zeros.

b) From our work above we seef(x) = x(x + 2)(x− 3)(3x− 1)(x + 2), or

x(x + 2)2(x− 3)(3x− 1).

Copyright © 2013 Pearson Education, Inc.

Page 175: JUDITH A. PENNA -  · PDF fileSTUDENT’S SOLUTIONS MANUAL JUDITH A. PENNA Indiana University Purdue University Indianapolis COLLEGE ALGEBRA: GRAPHS AND MODELS FIFTH EDITION

4

8

6

10

2

42�2�4 x

y

5

(x � 2)2f(x) �

y � 0

x � 2

172 Chapter 4: Polynomial and Rational Functions

61. f(x) = x4 − 6x3 + 9x2 + 6x− 10

a) Possibilities for p/q: ±1,±2,±5,±10

From the graph of f(x), we see that −1 and 1 mightbe zeros.−1∣∣ 1 −6 9 6 −10

−1 7 −16 101 −7 16 −10 0

1∣∣ 1 −7 16 −10

1 −6 101 −6 10 0

f(x) = (x + 1)(x− 1)(x2 − 6x + 10)Using the quadratic formula, we find that the otherzeros are 3 ± i.

The rational zeros are −1 and 1. The other zerosare 3 ± i.

b) f(x) = (x + 1)(x− 1)[x− (3 + i)][x− (3 − i)]

= (x + 1)(x− 1)(x− 3 − i)(x− 3 + i)

63. f(x) = 3x3 − 8x2 + 7x− 2

a) Possibilities for p/q: ±1,±2,±13,±2

3

From the graph of f(x), we see that23

and 1 mightbe zeros.1∣∣ 3 −8 7 −2

3 −5 23 −5 2 0

We have f(x) = (x− 1)(3x2 − 5x + 2).We find the other zeros.

3x2 − 5x + 2 = 0

(3x− 2)(x− 1) = 0

3x− 2 = 0 or x− 1 = 0

x =23

or x = 1

The rational zeros are 1 and23. (The zero 1 has

multiplicity 2.) These are the only zeros.

b) f(x) = (x− 1)2(3x− 2)

65. f(x) = x6 + x5 − 28x4 − 16x3 + 192x2

a) We know that 0 is a zero since

f(x) = x2(x4 + x3 − 28x2 − 16x + 192).

Consider g(x) = x4 + x3 − 28x2 − 16x + 192.

Possibilities for p/q: ±1,±2,±3,±4,±6,±8,±12,

±16,±24,±32,±48,±64,±96,

±192

From the graph of y = g(x), we see that −4, 3 and4 might be zeros.

−4∣∣ 1 1 −28 −16 192

−4 12 64 −1921 −3 −16 48 0

We have f(x) = x2 · g(x) =x2(x + 4)(x3 − 3x2 − 16x + 48).

We find the other zeros.x3 − 3x2 − 16x + 48 = 0

x2(x− 3) − 16(x− 3) = 0

(x− 3)(x2 − 16) = 0

(x− 3)(x + 4)(x− 4) = 0

x− 3 = 0 or x + 4 = 0 or x− 4 = 0

x = 3 or x = −4 or x = 4

The rational zeros are 0, −4, 3, and 4. (The zeros0 and −4 each have multiplicity 2.) These are theonly zeros.

b) f(x) = x2(x + 4)2(x− 3)(x− 4)

67. f(x) = 2x6 − 7x3 + x2 − x

There are 3 variations in sign in f(x), so there are 3 or 1positive real zeros.

f(−x) = 2(−x)6 − 7(−x)3 + (−x)2 − (−x)

= 2x6 + 7x3 + x2 + x

There are no variations in sign in f(−x), so there are nonegative real zeros.

69. g(x) = 5x5 − 4x2 + x− 1

There are 3 variations in sign in g(x), so there are 3 or 1positive real zeros.

g(−x) = 5(−x)5 − 4(−x)2 + (−x) − 1

= −5x5 − 4x2 − x− 1

There is no variation in sign in g(−x), so there are 0 neg-ative real zeros.

71. f(x) =5

(x− 2)2

1. The numerator and the denominator have no com-mon factors. The denominator is zero when x = 2,so the domain excludes 2. It is (−∞, 2) ∪ (2,∞).The line x = 2 is the vertical asymptote.

2. Because the degree of the numerator is less thanthe degree of the denominator, the x-axis, or y = 0,is the horizontal asymptote. There is no obliqueasymptote.

3. The numerator has no zeros, so there is no x-intercept.

4. f(0) =5

(0 − 2)2=

54, so the y-intercept is

(0,

54

).

5. Find other function values to determine the shapeof the graph and then draw it.

Copyright © 2013 Pearson Education, Inc.

Page 176: JUDITH A. PENNA -  · PDF fileSTUDENT’S SOLUTIONS MANUAL JUDITH A. PENNA Indiana University Purdue University Indianapolis COLLEGE ALGEBRA: GRAPHS AND MODELS FIFTH EDITION

4

2

�2

�4

2�2 x

y

x � 2

x 2 � 2x � 15f(x) �

y � 0

x � �3 x � 5

Chapter 4 Review Exercises 173

73. f(x) =x− 2

x2 − 2x− 15=

x− 2(x + 3)(x− 5)

1. The numerator and the denominator have no com-mon factors. The denominator is zero when x = −3,or x = 5, so the domain excludes −3 and 5. It is(−∞,−3) ∪ (−3, 5) ∪ (5,∞). The lines x = −3 andx = 5 are vertical asymptotes.

2. Because the degree of the numerator is less thanthe degree of the denominator, the x-axis, or y = 0,is the horizontal asymptote. There is no obliqueasymptote.

3. The numerator is zero when x = 2, so the x-intercept is (2, 0).

4. f(0) =0 − 2

02 − 2 · 0 − 15=

215

, so the y-intercept is(0,

215

).

5. Find other function values to determine the shapeof the graph and then draw it.

75. Answers may vary. The numbers −2 and 3 must be zerosof the denominator, and −3 must be zero of the numerator.In addition, the numerator and denominator must have thesame degree and the ratio of the leading coefficients mustbe 4.

f(x) =4x(x + 3)

(x + 2)(x− 3), or f(x) =

4x2 + 12xx2 − x− 6

77. x2 − 9 < 0 Polynomial inequality

x2 − 9 = 0 Related equation

(x + 3)(x− 3) = 0 Factoring

The solutions of the related equation are −3 and 3. Thesenumbers divide the x-axis into the intervals (−∞,−3),(−3, 3), and (3,∞).

We let f(x) = (x + 3)(x − 3) and test a value in eachinterval.

(−∞,−3): f(−4) = 7 > 0

(−3, 3): f(0) = −9 < 0

(3,∞): f(4) = 7 > 0

Function values are negative only on (−3, 3). The solutionset is (−3, 3).

79. (1 − x)(x + 4)(x− 2) ≤ 0 Polynomial inequality

(1 − x)(x + 4)(x− 2) = 0 Related equation

The solutions of the related equation are 1, −4 and2. These numbers divide the x-axis into the intervals(−∞,−4), (−4, 1), (1, 2) and (2,∞).

We let f(x) = (1 − x)(x + 4)(x − 2) and test a value ineach interval.

(−∞,−4) : f(−5) = 42 > 0

(−4, 1) : f(0) = −8 < 0

(1, 2) : f

(32

)=

118

> 0

(2,∞) : f(3) = −14 < 0

Function values are negative on (−4, 1) and (2,∞). Sincethe inequality symbol is ≤, the endpoints of the intervalsmust be included in the solution set. It is [−4, 1] ∪ [2,∞).

81. a) We write and solve a polynomial equation.

−16t2 + 80t + 224 = 0

−16(t2 − 5t− 14) = 0

−16(t + 2)(t− 7) = 0

The solutions are t = −2 and t = 7. Only t = 7 hasmeaning in this application. The rocket reaches theground at t = 7 seconds.

b) We write and solve a polynomial inequality.

−16t2 + 80t + 224 > 320 Polynomial inequality

−16t2 + 80t− 96 > 0 Equivalent inequality

−16t2 + 80t− 96 = 0 Related equation

−16(t2 − 5t + 6) = 0

−16(t− 2)(t− 3) = 0

The solutions of the related equation are 2 and 3.These numbers divide the t-axis into the intervals(−∞, 2), (2, 3) and (3,∞). We restrict our discus-sion to values of t such that 0 ≤ t ≤ 7 since we knowfrom part (a) the rocket is in the air for 7 sec. Weconsider the intervals [0, 2), (2, 3) and (3, 7]. We letf(t) = −16t2 + 80t − 96 and test a value in eachinterval.

[0, 2): f(1) = −32 < 0

(2, 3): f

(52

)= 4 > 0

(3, 7]: f(4) = −32 < 0Function values are positive on (2, 3). The solutionset is (2, 3)

83. g(x) =x2 + 2x− 3x2 − 5x + 6

=x2 + 2x− 3

(x− 2)(x− 3)The values of x that make the denominator 0 are 2 and3, so the domain is (−∞, 2) ∪ (2, 3) ∪ (3,∞). Answer A iscorrect.

85. f(x) = −12x4 + x3 + 1

The degree of the function is even and the leading coef-ficient is negative, so as x → ∞, f(x) → −∞ and asx → −∞, f(x) → −∞. In addition, f(0) = 1, so they-intercept is (0, 1). Thus B is the correct graph.

Copyright © 2013 Pearson Education, Inc.

Page 177: JUDITH A. PENNA -  · PDF fileSTUDENT’S SOLUTIONS MANUAL JUDITH A. PENNA Indiana University Purdue University Indianapolis COLLEGE ALGEBRA: GRAPHS AND MODELS FIFTH EDITION

174 Chapter 4: Polynomial and Rational Functions

87.∣∣∣∣1 − 1

x2

∣∣∣∣ < 3

−3 < 1 − 1x2

< 3

−3 <x2 − 1x2

< 3

−3 <(x + 1)(x− 1)

x2< 3

−3 <(x + 1)(x− 1)

x2and

(x + 1)(x− 1)x2

< 3

First, solve

−3 <(x + 1)(x− 1)

x2

0 <(x + 1)(x− 1)

x2+ 3

The denominator of f(x) =(x + 1)(x− 1)

x2+3 is zero when

x = 0, so the function is not defined for this value of x.Solve the related equation.

(x + 1)(x− 1)x2

+ 3 = 0

(x + 1)(x− 1) + 3x2 = 0 Multiplying by x2

x2 − 1 + 3x2 = 0

4x2 = 1

x2 =14

x = ±12

The critical values are −12, 0 and

12. Test a value in each

of the intervals determined by them.(−∞,−1

2

): f(−1) = 3 > 0(

− 12, 0)

: f

(− 1

4

)= −12 < 0(

0,12

): f

(14

)= −12 < 0(

12,∞)

: f(1) = 3 > 0

The solution set for this portion of the inequality is(−∞,−1

2

)∪(

12,∞)

.

Next, solve(x + 1)(x− 1)

x2< 3

(x + 1)(x− 1)x2

− 3 < 0

The denominator of f(x) =(x + 1)(x− 1)

x2− 3 is zero

when x = 0, so the function is not defined for this value ofx. Now solve the related equation.

(x + 1)(x− 1)x2

− 3 = 0

(x + 1)(x− 1) − 3x2 = 0 Multiplying by x2

x2 − 1 − 3x2 = 0

2x2 = −1

x2 = −12

There are no real solutions for this portion of the inequal-ity. The solution set of the original inequality is(−∞,−1

2

)∪(

12,∞)

.

89. (x− 2)−3 < 01

(x− 2)3< 0

The denominator of f(x) =1

(x− 2)3is zero when x = 2,

so the function is not defined for this value of x. Therelated equation

1(x− 2)3

= 0 has no solution, so 2 is the

only critical point. Test a value in each of the intervalsdetermined by this critical point.(−∞, 2) : f(1) = −1 < 0

(2,∞) : f(3) = 1 > 0

Function values are negative on (−∞, 2). The solution setis (−∞, 2).

91. Divide x3 + kx2 + kx− 15 by x + 3.−3∣∣ 1 k k −15

−3 9 − 3k −27 + 6k1 −3 + k 9 − 2k −42 + 6k

Thus f(−3) = −42 + 6k.

We know that if x+ 3 is a factor of f(x), then f(−3) = 0.We solve −42 + 6k = 0 for k.

−42 + 6k = 0

6k = 42

k = 7

93. f(x) =√x2 + 3x− 10

Since we cannot take the square root of a negative number,then x2 + 3x− 10 ≥ 0.

x2 + 3x− 10 ≥ 0 Polynomial inequality

x2 + 3x− 10 = 0 Related equation

(x + 5)(x− 2) = 0 Factoring

The solutions of the related equation are −5 and 2. Thesenumbers divide the x-axis into the intervals (−∞,−5),(−5, 2) and (2,∞).

We let g(x) = (x + 5)(x − 2) and test a value in eachinterval.

(−∞,−5) : g(−6) = 8 > 0

(−5, 2) : g(0) = −10 < 0

(2,∞) : g(3) = 8 > 0

Functions values are positive on (−∞,−5) and (2,∞).Since the equality symbol is ≥, the endpoints of theintervals must be included in the solution set. It is(−∞,−5] ∪ [2,∞).

Copyright © 2013 Pearson Education, Inc.

Page 178: JUDITH A. PENNA -  · PDF fileSTUDENT’S SOLUTIONS MANUAL JUDITH A. PENNA Indiana University Purdue University Indianapolis COLLEGE ALGEBRA: GRAPHS AND MODELS FIFTH EDITION

y

x�4 �2 2 4

�8

�4

4

8

f(x) � x3 � 5x2 � 2x � 8

Chapter 4 Test 175

95. f(x) =1√

5 − |7x + 2|We cannot take the square root of a negative number;neither can the denominator be zero. Thus we have5 − |7x + 2| > 0.

5 − |7x + 2| > 0 Polynomial inequality

|7x + 2| < 5

−5 < 7x + 2 < 5

−7 < 7x < 3

−1 < x <37

The solution set is(− 1,

37

).

97. No; since imaginary zeros of polynomials with rational co-efficients occur in conjugate pairs, a third-degree polyno-mial with rational coefficients can have at most two imag-inary zeros. Thus, there must be at least one real zero.

99. If P (x) is an even function, then P (−x) = P (x) andthus P (−x) has the same number of sign changes as P (x).Hence, P (x) has one negative real zero also.

101. A quadratic inequality ax2+bx+c ≤ 0, a > 0, or ax2+bx+c ≥ 0, a < 0, has a solution set that is a closed interval.

Chapter 4 Test

1. f(x) = 2x3 + 6x2 − x4 + 11

= −x4 + 2x3 + 6x2 + 11

The leading term is −x4 and the leading coefficient is −1.The degree of the polynomial is 4, so the polynomial isquartic.

2. h(x) = −4.7x + 29

The leading term is −4.7x and the leading coefficient is−4.7. The degree of the polynomial is 1, so the polynomialis linear.

3. f(x) = x(3x− 5)(x− 3)2(x + 1)3

The zeros of the function are 0,53, 3, and −1.

The factors x and 3x − 5 each occur once, so the zeros 0

and53

have multiplicity 1.

The factor x− 3 occurs twice, so the zero 3 has multiplic-ity 2.

The factor x + 1 occurs three times, so the zero −1 hasmultiplicity 3.

4. In 1930, x = 1930 − 1900 = 30.

f(30) = −0.0000007623221(30)4 + 0.00021189064(30)3 −0.016314058(30)2 + 0.2440779643(30) + 13.59260684 ≈11.3%

In 1990, x = 1990 − 1900 = 90.

f(90) = −0.0000007623221(90)4 + 0.00021189064(90)3 −0.016314058(90)2 + 0.2440779643(90) + 13.59260684 ≈7.9%

In 2000, x = 2000 − 1900 = 100.

f(100) = −0.0000007623221(100)4

+ 0.00021189064(100)3 − 0.016314058(100)2

+ 0.2440779643(100) + 13.59260684 ≈ 10.5%

5. f(x) = x3 − 5x2 + 2x + 8

1. The leading term is x3. The degree, 3, is odd andthe leading coefficient, 1, is positive so as x → ∞,f(x) → ∞ and as x → −∞, f(x) → −∞.

2. We solve f(x) = 0. By the rational zeros theorem,we know that the possible rational zeros are 1, −1,2, −2, 4, −4, 8, and −8. Synthetic division showsthat 1 is not a zero. We try −1.

−1∣∣ 1 −5 2 8

−1 6 −81 −6 8 0

We have f(x) = (x + 1)(x2 − 6x + 8) =(x + 1)(x− 2)(x− 4).

Now we find the zeros of f(x).

x + 1 = 0 or x− 2 = 0 or x− 4 = 0

x = −1 or x = 2 or x = 4

The zeros of the function are −1, 2, and 4, so thex-intercepts are (−1, 0), (2, 0), and (4, 0).

3. The zeros divide the x-axis into 4 intervals,(−∞,−1), (−1, 2), (2, 4), and (4,∞). We choose avalue for x in each interval and find f(x). This tellsus the sign of f(x) for all values of x in that interval.In (−∞,−1), test −3:

f(−3) = (−3)3 − 5(−3)2 + 2(−3) + 8 = −70 < 0

In (−1, 2), test 0:

f(0) = 03 − 5(0)2 + 2(0) + 8 = 8 > 0

In (2, 4), test 3:

f(3) = 33 − 5(3)2 + 2(3) + 8 = −4 < 0

In (4,∞), test 5:

f(5) = 53 − 5(5)2 + 2(5) + 8 = 18 > 0Thus the graph lies below the x-axis on (−∞,−1)and on (2, 4) and above the x-axis on (−1, 2) and(4,∞). We also know the points (−3,−70), (0, 8),(3,−4), and (5, 18) are on the graph.

4. From Step 3 we know that f(0) = 8, so the y-intercept is (0, 8).

5. We find additional points on the graph and drawthe graph.

x f(x)

−0.5 5.625

0.5 7.875

2.5 −2.625

6 56

Copyright © 2013 Pearson Education, Inc.

Page 179: JUDITH A. PENNA -  · PDF fileSTUDENT’S SOLUTIONS MANUAL JUDITH A. PENNA Indiana University Purdue University Indianapolis COLLEGE ALGEBRA: GRAPHS AND MODELS FIFTH EDITION

f(x) � �2x4 � x3 � 11x2 � 4x � 12

4

�8

�4

�12

42�2�4 x

y

176 Chapter 4: Polynomial and Rational Functions

6. Checking the graph as described on page 317 in thetext, we see that it appears to be correct.

6. f(x) = −2x4 + x3 + 11x2 − 4x− 12

1. The leading term is −2x4. The degree, 4, is even andthe leading coefficient, −2, is negative so x → ∞,f(x) → −∞ and as x → −∞, f(x) → −∞.

2. We solve f(x) = 0.

The possible rational zeros are ±1, ±2, ±3, ±4, ±6,

±12, ±12, and ±3

2. We try −1.

−1∣∣ −2 1 11 −4 −12

2 −3 −8 12−2 3 8 −12 0

We have f(x) = (x+1)(−2x3 +3x2 +8x−12). Findthe other zeros.−2x3 + 3x2 + 8x− 12 = 0

2x3 − 3x2 − 8x + 12 = 0 Multiplying by −1

x2(2x− 3) − 4(2x− 3) = 0

(2x− 3)(x2 − 4) = 0

(2x− 3)(x + 2)(x− 2) = 0

2x− 3 = 0 or x + 2 = 0 or x− 2 = 0

2x = 3 or x = −2 or x = 2

x =32

or x = −2 or x = 2

The zeros of the function are −1,32, −2, and 2, so

the x-intercepts are (−2, 0), (−1, 0),(

32, 0)

, and

(2, 0).

3. The zeros divide the x-axis into 5 intervals,

(−∞,−2), (−2,−1),(− 1,

32

),(

32, 2)

, and

(2,∞). We choose a value for x in each interval andfind f(x). This tells us the sign of f(x) for all valuesof x in that interval.In (−∞,−2), test −3: f(−3) = −90 < 0

In (−2,−1), test −1.5: f(−1.5) = 5.25 > 0

In(− 1,

32

), test 0: f(0) = −12 < 0

In(

32, 2)

, test 1.75: f(1.75) ≈ 1.29 > 0

In (2,∞), test 3: f(3) = −60 < 0Thus the graph lies below the x-axis on (−∞,−2),

on(− 1,

32

), and on (2,∞) and above the x-axis on

(−2,−1) and on(

32, 2)

. We also know the points

(−3,−90), (−1.5, 5.25), (0,−12), (1.75, 1.29), and(3,−60) are on the graph.

4. From Step 3 we know that f(0) = −12, so the y-intercept is (0,−12).

5. We find additional points on the graph and drawthe graph.

x f(x)

−0.5 −7.5

0.5 −11.25

2.5 −15.75

6. Checking the graph as described on page 317 in thetext, we see that it appears to be correct.

7. f(0) = −5 · 02 + 3 = 3

f(2) = −5 · 22 + 3 = −17

By the intermediate value theorem, since f(0) and f(2)have opposite signs, f(x) has a zero between 0 and 2.

8. g(−2) = 2(−2)3 + 6(−2)2 − 3 = 5

g(−1) = 2(−1)3 + 6(−1)2 − 3 = 1

Since both g(−2) and g(−1) are positive, we cannot usethe intermediate value theorem to determine if there is azero between −2 and −1.

9. x3 + 4x2 + 4x + 6x− 1 x4 + 3x3 + 0x2 + 2x− 5

x4 − x3

4x3 + 0x2

4x3 − 4x2

4x2 + 2x4x2 − 4x

6x− 56x− 6

1

The quotient is x3 + 4x2 + 4x + 6; the remainder is 1.

P (x) = (x− 1)(x3 + 4x2 + 4x + 6) + 1

10. 5∣∣ 3 0 −12 7

15 75 3153 15 63 322

Q(x) = 3x2 + 15x + 63; R(x) = 322

11. −3∣∣ 2 −6 1 −4

−6 36 −1112 −12 37 −115

P (−3) = −115

12. −2∣∣ 1 4 1 −6

−2 −4 61 2 −3 0

f(−2) = 0, so −2 is a zero of f(x).

Copyright © 2013 Pearson Education, Inc.

Page 180: JUDITH A. PENNA -  · PDF fileSTUDENT’S SOLUTIONS MANUAL JUDITH A. PENNA Indiana University Purdue University Indianapolis COLLEGE ALGEBRA: GRAPHS AND MODELS FIFTH EDITION

Chapter 4 Test 177

13. The function can be written in the form

f(x) = an(x + 3)2(x)(x− 6).

The simplest polynomial is obtained if we let an = 1.

f(x) = (x + 3)2(x)(x− 6)

= (x2 + 6x + 9)(x2 − 6x)

= x4 + 6x3 + 9x2 − 6x3 − 36x2 − 54x

= x4 − 27x2 − 54x

14. A polynomial function of degree 5 can have at most 5 zeros.Since f(x) has rational coefficients, in addition to the 3given zeros, the other zeros are the conjugates of

√3 and

2 − i, or −√3 and 2 + i.

15. −3i is also a zero.f(x) = (x + 10)(x− 3i)(x + 3i)

= (x + 10)(x2 + 9)

= x3 + 10x2 + 9x + 90

16.√

3 and 1 + i are also zeros.

f(x) = (x− 0)(x +√

3)(x−√3)[x− (1 − i)][x− (1 + i)]

= x(x2 − 3)[(x− 1) + i][(x− 1) − i]

= (x3 − 3x)(x2 − 2x + 1 + 1)

= (x3 − 3x)(x2 − 2x + 2)

= x5 − 2x4 + 2x3 − 3x3 + 6x2 − 6x

= x5 − 2x4 − x3 + 6x2 − 6x

17. f(x) = 2x3 + x2 − 2x + 12Possibilities for p

Possibilities for q:

±1,±2,±3,±4,±6,±12±1,±2

Possibilities for p/q: ±1,±2,±3,±4,±6,±12,±12,±3

2

18. h(x) = 10x4 − x3 + 2x− 5Possibilities for p

Possibilities for q:

±1,±5±1,±2,±5,±10

Possibilities for p/q: ±1,±5,±12,±5

2,±1

5,± 1

10

19. f(x) = x3 + x2 − 5x− 5

a) Possibilities for p/q: ±1,±5

From the graph of y = f(x), we see that −1 mightbe a zero.−1∣∣ 1 1 −5 −5

−1 0 51 0 −5 0

We have f(x) = (x + 1)(x2 − 5). We find the otherzeros.

x2 − 5 = 0

x2 = 5

x = ±√5

The rational zero is −1. The other zeros are ±√5.

b) f(x) = (x + 1)(x−√5)(x +

√5)

20. g(x) = 2x4 − 11x3 + 16x2 − x− 6

a) Possibilities for p/q: ±1,±2,±3,±6,±12,±3

2

From the graph of y = g(x), we see that −12, 1, 2,

and 3 might be zeros. We try −12.

− 12

∣∣ 2 −11 16 −1 −6−1 6 −11 6

2 −12 22 −12 0

Now we try 1.

1∣∣ 2 −12 22 −12

2 −10 122 −10 12 0

We have g(x)=(x +

12

)(x−1)(2x2−10x + 12) =

2(x +

12

)(x− 1)(x2 − 5x + 6). We find the other

zeros.x2 − 5x + 6 = 0

(x− 2)(x− 3) = 0

x− 2 = 0 or x− 3 = 0

x = 2 or x = 3

The rational zeros are −12, 1, 2, and 3. These are

the only zeros.

b) g(x) = 2(x +

12

)(x− 1)(x− 2)(x− 3)

= (2x + 1)(x− 1)(x− 2)(x− 3)

21. h(x) = x3 + 4x2 + 4x + 16

a) Possibilities for p/q: ±1,±2,±4,±8,±16

From the graph of h(x), we see that −4 might be azero.−4∣∣ 1 4 4 16

−4 0 −161 0 4 0

We have h(x) = (x + 4)(x2 + 4). We find the otherzeros.

x2 + 4 = 0

x2 = −4

x = ±2i

The rational zero is −4. The other zeros are ±2i.

b) h(x) = (x + 4)(x + 2i)(x− 2i)

22. f(x) = 3x4 − 11x3 + 15x2 − 9x + 2

a) Possibilities for p/q: ±1,±2,±13,±2

3

From the graph of f(x), we see that23

and 1 might

be zeros. We try23.

23

∣∣ 3 −11 15 −9 22 −6 6 −2

3 −9 9 −3 0

Copyright © 2013 Pearson Education, Inc.

Page 181: JUDITH A. PENNA -  · PDF fileSTUDENT’S SOLUTIONS MANUAL JUDITH A. PENNA Indiana University Purdue University Indianapolis COLLEGE ALGEBRA: GRAPHS AND MODELS FIFTH EDITION

f (x) � 2

(x � 3)2

y � 0

x � 3

y

x

2

4

�2

�4

�2�4 42

y

x�4 �2 2 4 6

�4

�2

2

4

f(x) � ��������x � 3

x2 � 3x � 4

y � 0

x � �1 x � 4

178 Chapter 4: Polynomial and Rational Functions

Now we try 1.

1∣∣ 3 −9 9 −3

3 −6 33 −6 3 0

We have f(x)=(x− 2

3

)(x−1)(3x2−6x + 3) =

3(x− 2

3

)(x− 1)(x2 − 2x + 1). We find the other

zeros.x2 − 2x + 1 = 0

(x− 1)(x− 1) = 0

x− 1 = 0 or x− 1 = 0

x = 1 or x = 1

The rational zeros are23

and 1. (The zero 1 has

multiplicity 3.) These are the only zeros.

b) f(x) = 3(x− 2

3

)(x− 1)(x− 1)(x− 1)

= (3x− 2)(x− 1)3

23. g(x) = −x8 + 2x6 − 4x3 − 1

There are 2 variations in sign in g(x), so there are 2 or 0positive real zeros.

g(−x) = −(−x)8 + 2(−x)6 − 4(−x)3 − 1

= −x8 + 2x6 + 4x3 − 1

There are 2 variations in sign in g(−x), so there are 2 or0 negative real zeros.

24. f(x) =2

(x− 3)2

1. The numerator and the denominator have no com-mon factors. The denominator is zero when x = 3,so the domain excludes 3. it is (−∞, 3) ∪ (3,∞).The line x = 3 is the vertical asymptote.

2. Because the degree of the numerator is less than thedegree of the denominator, the x-axis, or y = 0, isthe horizontal asymptote.

3. The numerator has no zeros, so there is no x-intercept.

4. f(0) =2

(0 − 3)2=

29, so the y-intercept is

(0,

29

).

5. Find other function values to determine the shapeof the graph and then draw it.

25. f(x) =x + 3

x2 − 3x− 4=

x + 3(x + 1)(x− 4)

1. The numerator and the denominator have no com-mon factors. The denominator is zero when x = −1or x = 4, so the domain excludes −1 and 4. It is(−∞,−1) ∪ (−1, 4) ∪ (4,∞). The lines x = −1 andx = 4 are vertical asymptotes.

2. Because the degree of the numerator is less than thedegree of the denominator, the x-axis, or y = 0, isthe horizontal asymptote.

3. The numerator is zero at x = −3, so the x-interceptis (−3, 0).

4. f(0) =0 + 3

(0 + 1)(0 − 4)= −3

4, so the y-intercept is(

0,−34

).

5. Find other function values to determine the shapeof the graph and then draw it.

26. Answers may vary. The numbers −1 and 2 must be zeros ofthe denominator, and −4 must be a zero of the numerator.

f(x) =x + 4

(x + 1)(x− 2), or f(x) =

x + 4x2 − x− 2

27. 2x2 > 5x + 3 Polynomial inequality

2x2 − 5x− 3 > 0 Equivalent inequality

2x2 − 5x− 3 = 0 Related equation

(2x + 1)(x− 3) = 0 Factoring

The solutions of the related equation are −12

and 3. These

numbers divide the x-axis into the intervals(−∞,−1

2

),(

− 12, 3)

, and (3,∞).

We let f(x) = (2x + 1)(x − 3) and test a value in eachinterval.(

−∞,−12

): f(−1) = 4 > 0(

− 12, 3)

: f(0) = −3 < 0

(3,∞) : f(4) = 9 > 0

Function values are positive on(−∞,−1

2

)and (3,∞).

The solution set is(−∞,−1

2

)∪ (3,∞).

Copyright © 2013 Pearson Education, Inc.

Page 182: JUDITH A. PENNA -  · PDF fileSTUDENT’S SOLUTIONS MANUAL JUDITH A. PENNA Indiana University Purdue University Indianapolis COLLEGE ALGEBRA: GRAPHS AND MODELS FIFTH EDITION

Chapter 4 Test 179

28. x + 1x− 4

≤ 3 Rational inequality

x + 1x− 4

− 3 ≤ 0 Equivalent inequality

The denominator of f(x) =x + 1x− 4

− 3 is zero when x = 4,

so the function is not defined for this value of x. We solvethe related equation f(x) = 0.

x + 1x− 4

− 3 = 0

(x− 4)(x + 1x− 4

− 3)

= (x− 4) · 0x + 1 − 3(x− 4) = 0

x + 1 − 3x + 12 = 0

−2x + 13 = 0

2x = 13

x =132

The critical values are 4 and132

. They divide the x-axis

into the intervals (−∞, 4),(

4,132

)and

(132,∞)

. We

test a value in each interval.(−∞, 4) : f(3) = −7 < 0(

4,132

): f(5) = 3 > 0(

132,∞)

: f(9) = −1 < 0

Function values are negative for (−∞, 4) and(

132,∞)

.

Since the inequality symbol is ≤, the endpoint of the in-

terval(

132,∞)

must be included in the solution set. It is

(−∞, 4) ∪[132,∞)

.

29. a) We write and solve a polynomial equation.

−16t2 + 64t + 192 = 0

−16(t2 − 4t− 12) = 0

−16(t + 2)(t− 6) = 0

The solutions are t = −2 and t = 6. Only t = 6 hasmeaning in this application. The rocket reaches theground at t = 6 seconds.

b) We write and solve a polynomial inequality.

−16t2 + 64t + 192 > 240

−16t2 + 64t− 48 > 0

−16t2 + 64t− 48 = 0 Related equation

−16(t2 − 4t + 3) = 0

−16(t− 1)(t− 3) = 0

The solutions of the related equation are 1 and 3.These numbers divide the t-axis into the intervals(−∞, 1), (1, 3) and (3,∞). Because the rocket re-turns to the ground at t = 6, we restrict our discus-sion to values of t such that 0 ≤ t ≤ 6. We considerthe intervals [0, 1), (1, 3) and (3, 6]. We let

f(t) = −16t2 + 64t − 48 and test a value in eachinterval.

[0, 1) : f

(12

)= −20 < 0

(1, 3) : f(2) = 16 > 0

(3, 6] : f(4) = −48 < 0

Function values are positive on (1, 3). The solutionset is (1, 3).

30. f(x) = x3 − x2 − 2

The degree of the function is odd and the leading coeffi-cient is positive, so as x → ∞, f(x) → ∞ and as x → −∞,f(x) → −∞. In addition, f(0) = −2, so the y-intercept is(0,−2). Thus D is the correct graph.

31. f(x) =√x2 + x− 12

Since we cannot take the square root of a negative number,then x2 + x− 12 ≥ 0.

x2 + x− 12 ≥ 0 Polynomial inequality

x2 + x− 12 = 0 Related equation

(x + 4)(x− 3) = 0 Factoring

The solutions of the related equation are −4 and 3. Thesenumbers divide the x-axis into the intervals (−∞,−4),(−4, 3) and (3,∞).

We let f(x) = (x + 4)(x − 3) and test a value in eachinterval.

(−∞,−4) : g(−5) = 8 > 0

(−4, 3) : g(0) = −12 < 0

(3,∞) : g(4) = 8 > 0

Function values are positive on (−∞,−4) and (3,∞).Since the inequality symbol is ≥, the endpoints of theintervals must be included in the solution set. It is(−∞,−4] ∪ [3,∞).

Copyright © 2013 Pearson Education, Inc.

Page 183: JUDITH A. PENNA -  · PDF fileSTUDENT’S SOLUTIONS MANUAL JUDITH A. PENNA Indiana University Purdue University Indianapolis COLLEGE ALGEBRA: GRAPHS AND MODELS FIFTH EDITION

Copyright © 2013 Pearson Education, Inc.

Page 184: JUDITH A. PENNA -  · PDF fileSTUDENT’S SOLUTIONS MANUAL JUDITH A. PENNA Indiana University Purdue University Indianapolis COLLEGE ALGEBRA: GRAPHS AND MODELS FIFTH EDITION

4

�4

42�4 x

yy � x

x � y 2 � 3

y � x 2 � 3

y � 3x � 2

x � 3y � 2

y � x

y

2 4�4 �2

2

4

�4

�2x

4

2

�2

�4

42�2�4 x

y

y � x

y � �x �

x � �y�

Chapter 5

Exponential and Logarithmic Functions

Exercise Set 5.1

1. We interchange the first and second coordinates of eachordered pair to find the inverse of the relation. It is

{(8, 7), (8,−2), (−4, 3), (−8, 8)}.3. We interchange the first and second coordinates of each

ordered pair to find the inverse of the relation. It is

{(−1,−1), (4,−3)}.5. Interchange x and y.

y = 4x−5� �x = 4y−5

7. Interchange x and y.

x3y = −5��y3x = −5

9. Interchange x and y.

x = y2−2y� � �y = x2−2x

11. Graph x = y2 − 3. Some points on the graph are (−3, 0),(−2,−1), (−2, 1), (1,−2), and (1, 2). Plot these pointsand draw the curve. Then reflect the graph across the liney = x.

13. Graph y = 3x− 2. The intercepts are (0,−2) and(

23, 0)

.

Plot these points and draw the line. Then reflect the graphacross the line y = x.

15. Graph y = |x|. Some points on the graph are (0, 0),(−2, 2), (2, 2), (−5, 5), and (5, 5). Plot these points anddraw the graph. Then reflect the graph across the liney = x.

17. We show that if f(a) = f(b), then a = b.13a− 6 =

13b− 6

13a =

13b Adding 6

a = b Multiplying by 3

Thus f is one-to-one.

19. We show that if f(a) = f(b), then a = b.

a3 +12

= b3 +12

a3 = b3 Subtracting12

a = b Taking cube roots

Thus f is one-to-one.

21. g(−1) = 1 − (−1)2 = 1 − 1 = 0 andg(1) = 1 − 12 = 1 − 1 = 0, so g(−1) = g(1) but −1 �= 1.Thus the function is not one-to-one.

23. g(−2) = (−2)4 − (−2)2 = 16 − 4 = 12 andg(2) = 24 −22 = 16−4 = 12, so g(−2) = g(2) but −2 �= 2.Thus the function is not one-to-one.

25. The function is one-to-one, because no horizontal linecrosses the graph more than once.

27. The function is not one-to-one, because there are manyhorizontal lines that cross the graph more than once.

29. The function is not one-to-one, because there are manyhorizontal lines that cross the graph more than once.

31. The function is one-to-one, because no horizontal linecrosses the graph more than once.

Copyright © 2013 Pearson Education, Inc.

Page 185: JUDITH A. PENNA -  · PDF fileSTUDENT’S SOLUTIONS MANUAL JUDITH A. PENNA Indiana University Purdue University Indianapolis COLLEGE ALGEBRA: GRAPHS AND MODELS FIFTH EDITION

x

y

2–2–6 64 108–4–8–10–2

–4

–6

–8

2

4

6

8

10

–10

f (x ) = 5x — 8

x

y

1–1–3 32 54–2–4–5–1

–2

–3

–4

1

2

3

4

5

–5

f (x ) = 1 — x 2

x

y

1–1–3 32 54–2–4–5–1

–2

–3

–4

1

2

3

4

5

–5

f (x ) = | x + 2 |

x

y

1–1–3 32 54–2–4–5–1

–2

–3

–4

1

2

3

4

5

–5

f (x ) =4

x

x

y

1–1–3 32 54–2–4–5–1

–2

–3

–4

1

2

3

4

5

–5

f (x ) =2

3

x

y

1–1–3 32 54–2–4–5–1

–2

–3

–4

1

2

3

4

5

–5

f (x ) = 25 x 2

�5 25

�5

15

y1 � 0.8x � 1.7,

y2 �x � 1.7

0.8y2 y1

12

�15 15

�10

10

y1 � �x � 4,

y2 � 2x � 8

y2y1

182 Chapter 5: Exponential and Logarithmic Functions

33. The graph of f(x) = 5x− 8 is shown below.

Since there is no horizontal line that crosses the graphmore than once, the function is one-to-one.

35. The graph of f(x) = 1 − x2 is shown below.

Since there are many horizontal lines that cross the graphmore than once, the function is not one-to-one.

37. The graph of f(x) = |x + 2| is shown below.

Since there are many horizontal lines that cross the graphmore than once, the function is not one-to-one.

39. The graph of f(x) = − 4x

is shown below.

Since there is no horizontal line that crosses the graphmore than once, the function is one-to-one.

41. The graph of f(x) =23

is shown below.

Since the horizontal line y =23

crosses the graph more thanonce, the function is not one-to-one.

43. The graph of f(x) =√

25 − x2 is shown below.

Since there are many horizontal lines that cross the graphmore than once, the function is not one-to-one.

45.

Both the domain and the range of f are the set of all realnumbers. Then both the domain and the range of f−1 arealso the set of all real numbers.

47.

Copyright © 2013 Pearson Education, Inc.

Page 186: JUDITH A. PENNA -  · PDF fileSTUDENT’S SOLUTIONS MANUAL JUDITH A. PENNA Indiana University Purdue University Indianapolis COLLEGE ALGEBRA: GRAPHS AND MODELS FIFTH EDITION

x

y

1—1—3 32 54—2—4—5—1

—2

—3

—4

1

2

3

4

5

—5

f (x ) = x + 4

x

y

1—1—3 32 54—2—4—5—1

—2

—3

—4

1

2

3

4

5

—5

f (x ) = 2x – 1

x

y

2—2—6 64 108—4—8—10

—2

—4

—6

—8

2

4

6

8

10

—10

f (x ) = –––4x + 7

Exercise Set 5.1 183

Both the domain and the range of f are the set of all realnumbers. Then both the domain and the range of f−1 arealso the set of all real numbers.

49. y1 =√x− 3, y2 = x2 + 3, x ≥ 0

The domain of f is [3,∞) and the range of f is [0,∞).Then the domain of f−1 is [0,∞) and the range of f−1 is[3,∞).

51. y1 = x2 − 4, x ≥ 0; y2 =√

4 + x

Since it is specified that x ≥ 0, the domain of f is [0,∞).The range of f is [−4,∞). Then the domain of f−1 is[−4,∞) and the range of f−1 is [0,∞).

53. y1 = (3x− 9)3, y2 =3√x + 93

Both the domain and the range of f are the set of all realnumbers. Then both the domain and the range of f−1 arealso the set of all real numbers.

55. a) The graph of f(x) = x+4 is shown below. It passesthe horizontal-line test, so it is one-to-one.

b) Replace f(x) with y: y = x + 4

Interchange x and y: x = y + 4

Solve for y: x− 4 = y

Replace y with f−1(x): f−1(x) = x− 4

57. a) The graph of f(x) = 2x−1 is shown below. It passesthe horizontal-line test, so it is one-to-one.

b) Replace f(x) with y: y = 2x− 1

Interchange x and y: x = 2y − 1

Solve for y:x + 1

2= y

Replace y with f−1(x): f−1(x) =x + 1

2

59. a) The graph of f(x) =4

x + 7is shown below. It passes

the horizontal-line test, so the function is one-to-one.

b) Replace f(x) with y: y =4

x + 7

Interchange x and y: x =4

y + 7

Solve for y: x(y + 7) = 4

y + 7 =4x

y =4x− 7

Replace y with f−1(x): f−1(x) =4x− 7

Copyright © 2013 Pearson Education, Inc.

Page 187: JUDITH A. PENNA -  · PDF fileSTUDENT’S SOLUTIONS MANUAL JUDITH A. PENNA Indiana University Purdue University Indianapolis COLLEGE ALGEBRA: GRAPHS AND MODELS FIFTH EDITION

x

y

2—2—6 64 108—4—8—10

—2

—4

—6

—8

2

4

6

8

10

—10

f (x ) = –––x + 4x – 3

x

y

1—1—3 32 54—2—4—5—1

—2

—3

—4

1

2

3

4

5

—5

f (x ) = x 3 – 1

x

y

1—1—3 32 54—2—4—5—1

—2

—3

—4

1

2

3

4

5

—5

f (x ) = x √ 4 – x 2

x

y

1—1—3 32 54—2—4—5—1

—2

—3

—4

1

2

3

4

5

—5f (x ) = 5x 2 – 2, x > 0

x

y

1—1—3 32 54—2—4—5—1

—2

—3

—4

1

2

3

4

5

—5

f (x ) = √ x + 1

184 Chapter 5: Exponential and Logarithmic Functions

61. a) The graph of f(x) =x + 4x− 3

is shown below. It passes

the horizontal-line test, so the function is one-to-one.

b) Replace f(x) with y: y =x + 4x− 3

Interchange x and y: x =y + 4y − 3

Solve for y: (y − 3)x = y + 4

xy − 3x = y + 4

xy − y = 3x + 4

y(x− 1) = 3x + 4

y =3x + 4x− 1

Replace y with f−1(x): f−1(x) =3x + 4x− 1

63. a) The graph of f(x) = x3−1 is shown below. It passesthe horizontal-line test, so the function is one-to-one.

b) Replace f(x) with y: y = x3 − 1

Interchange x and y: x = y3 − 1

Solve for y: x + 1 = y3

3√x + 1 = y

Replace y with f−1(x): f−1(x) = 3√x + 1

65. a) The graph of f(x) = x√

4 − x2 is shown below.Since there are many horizontal lines that cross thegraph more than once, the function is not one-to-oneand thus does not have an inverse that is a function.

67. a) The graph of f(x) = 5x2 − 2, x ≥ 0 is shown below.It passes the horizontal-line test, so it is one-to-one.

b) Replace f(x) with y: y = 5x2 − 2

Interchange x and y: x = 5y2 − 2

Solve for y: x + 2 = 5y2

x + 25

= y2

√x + 2

5= y

(We take the principal square root, becausex ≥ 0 in the original equation.)

Replace y with f−1(x): f−1(x) =

√x + 2

5for

all x in the range of f(x), or f−1(x) =

√x + 2

5,

x ≥ −2

69. a) The graph of f(x) =√x + 1 is shown below. It

passes the horizontal-line test, so the function is one-to-one.

Copyright © 2013 Pearson Education, Inc.

Page 188: JUDITH A. PENNA -  · PDF fileSTUDENT’S SOLUTIONS MANUAL JUDITH A. PENNA Indiana University Purdue University Indianapolis COLLEGE ALGEBRA: GRAPHS AND MODELS FIFTH EDITION

y

2 4�4 �2

2

4

�4

�2x

(�3, 0)

(3, 5)

(1, 2)

(�5, �5)

2 4�4 �2

2

4

�4

�2x

(5.375, 1.5)

(2, 0)(�6, �2)

(1, �1)

y

2 4

2

4

�4 �2

�4

�2x

y

2 4�4 �2

2

4

�4

�2x

y

f �1

f

Exercise Set 5.1 185

b) Replace f(x) with y: y =√x + 1

Interchange x and y: x =√y + 1

Solve for y: x2 = y + 1

x2 − 1 = y

Replace y with f−1(x): f−1(x) = x2 − 1 for all xin the range of f(x), or f−1(x) = x2 − 1, x ≥ 0.

71. f(x) = 3x

The function f multiplies an input by 3. Then to reversethis procedure, f−1 would divide each of its inputs by 3.

Thus, f−1(x) =x

3, or f−1(x) =

13x.

73. f(x) = −x

The outputs of f are the opposites, or additive inverses, ofthe inputs. Then the outputs of f−1 are the opposites ofits inputs. Thus, f−1(x) = −x.

75. f(x) = 3√x− 5

The function f subtracts 5 from each input and then takesthe cube root of the result. To reverse this procedure, f−1

would raise each input to the third power and then add 5to the result. Thus, f−1(x) = x3 + 5.

77. We reflect the graph of f across the line y = x. The re-flections of the labeled points are (−5,−5), (−3, 0), (1, 2),and (3, 5).

79. We reflect the graph of f across the line y = x. The re-flections of the labeled points are (−6,−2), (1,−1), (2, 0),and (5.375, 1.5).

81. We reflect the graph of f across the line y = x.

83. We find (f−1◦f)(x) and (f ◦f−1)(x) and check to see thateach is x.

(f−1 ◦ f)(x) = f−1(f(x)) = f−1

(78x

)=

87

(78x

)= x

(f ◦ f−1)(x) = f(f−1(x)) = f

(87x

)=

78

(87x

)= x

85. We find (f−1◦f)(x) and (f ◦f−1)(x) and check to see thateach is x.

(f−1 ◦ f)(x) = f−1(f(x)) = f−1

(1 − x

x

)=

11 − x

x+ 1

=1

1 − x + x

x

=11x

= x

(f ◦ f−1)(x) = f(f−1(x)) = f

(1

x + 1

)=

1 − 1x + 11

x + 1

=

x + 1 − 1x + 1

1x + 1

=

x

x + 11

x + 1

= x

87. (f−1 ◦ f)(x) = f−1(f(x)) =5(

25x + 1

)− 5

2=

2x + 5 − 52

=2x2

= x

(f ◦ f−1)(x) = f(f−1(x)) =25

(5x− 52

)+ 1 =

x− 1 + 1 = x

89. Replace f(x) with y: y = 5x− 3

Interchange x and y: x = 5y − 3

Solve for y: x + 3 = 5yx + 3

5= y

Replace y with f−1(x): f−1(x) =x + 3

5, or

15x +

35

The domain and range of f are (−∞,∞), so the domainand range of f−1 are also (−∞,∞).

91. Replace f(x) with y: y =2x

Interchange x and y: x =2y

Solve for y: xy = 2

y =2x

Copyright © 2013 Pearson Education, Inc.

Page 189: JUDITH A. PENNA -  · PDF fileSTUDENT’S SOLUTIONS MANUAL JUDITH A. PENNA Indiana University Purdue University Indianapolis COLLEGE ALGEBRA: GRAPHS AND MODELS FIFTH EDITION

f � f �1

2 4�4 �2

2

4

�4

�2

x

y

2 4�4 �2

2

4

�4

�2x

y

f �1

f

2 4�4 �2

2

4

�4

�2x

y

f �1

f

186 Chapter 5: Exponential and Logarithmic Functions

Replace y with f−1(x): f−1(x) =2x

The domain and range of f are (−∞, 0) ∪ (0,∞), so thedomain and range of f−1 are also (−∞, 0) ∪ (0,∞).

93. Replace f(x) with y: y =13x3 − 2

Interchange x and y: x =13y3 − 2

Solve for y: x + 2 =13y3

3x + 6 = y3

3√

3x + 6 = y

Replace y with f−1(x): f−1(x) = 3√

3x + 6

The domain and range of f are (−∞,∞), so the domainand range of f−1 are also (−∞,∞).

95. Replace f(x) with y: y =x + 1x− 3

Interchange x and y: x =y + 1y − 3

Solve for y: xy − 3x = y + 1

xy − y = 3x + 1

y(x− 1) = 3x + 1

y =3x + 1x− 1

Replace y with f−1(x): f−1(x) =3x + 1x− 1

The domain of f is (−∞, 3)∪ (3,∞) and the range of f is(−∞, 1) ∪ (1,∞). Thus the domain of f−1 is(−∞, 1)∪ (1,∞) and the range of f−1 is (−∞, 3)∪ (3,∞).

97. Since f(f−1(x)) = f−1(f(x)) = x, then f(f−1(5)) = 5and f−1(f(a)) = a.

99. Replace C(x) with y: y =60 + 2x

x

Interchange x and y: x =60 + 2y

y

Solve for y: xy = 60 + 2y

xy − 2y = 60

y(x− 2) = 60

y =60

x− 2

Replace y with C−1(x): C−1(x) =60

x− 2C−1(x) represents the number of people in the group,where x is the cost per person in dollars.

101. a) T (−13◦) =59(−13◦ − 32◦) =

59(−45◦) = −25◦

T (86◦) =59(86◦ − 32◦) =

59(54◦) = 30◦

b) Replace T (x) with y: y =59(x− 32)

Interchange x and y: x =59(y − 32)

Solve for y:95x = y − 32

95x + 32 = y

Replace y with T−1(x): T−1(x) =95x + 32

T−1(x) represents the Fahrenheit temperature whenthe Celsius temperature is x.

103. The functions for which the coefficient of x2 is negativehave a maximum value. These are (b), (d), (f), and (h).

105. Since |2| > 1 the graph of f(x) = 2x2 can be obtainedby stretching the graph of f(x) = x2 vertically. Since

0 <∣∣∣14

∣∣∣ < 1, the graph of f(x) =14x2 can be obtained by

shrinking the graph of y = x2 vertically. Thus the graphof f(x) = 2x2, or (a) is narrower.

107. We can write (f) as f(x) = −2[x − (−3)]2 + 1. Thus thegraph of (f) has vertex (−3, 1).

109. Graph y1 = f(x) and y2 = g(x) and observe that thegraphs are reflections of each other across the line y = x.Thus, the functions are inverses of each other.

111. The graph of f(x) = x2 − 3 is a parabola with vertex(0,−3). If we consider x-values such that x ≥ 0, then thegraph is the right-hand side of the parabola and it passesthe horizontal line test. We find the inverse of f(x) =x2 − 3, x ≥ 0.

Replace f(x) with y: y = x2 − 3

Interchange x and y: x = y2 − 3

Copyright © 2013 Pearson Education, Inc.

Page 190: JUDITH A. PENNA -  · PDF fileSTUDENT’S SOLUTIONS MANUAL JUDITH A. PENNA Indiana University Purdue University Indianapolis COLLEGE ALGEBRA: GRAPHS AND MODELS FIFTH EDITION

8

6

4

2

�1 x

y

f (x) � 3x

1 2 3�1�2�3

x

y

f (x) � 6x

1 2 3�1�3 �2�1

2

3

4

5

6

f (x) � �~�x

x

y

1 2 3�1�3 �2�1

2

3

4

5

2 4�4 �2

2

4

�4

�2x

y

y � �2x

Exercise Set 5.2 187

Solve for y: x + 3 = y2

√x + 3 = y

(We take the principal square root, because x ≥ 0 in theoriginal equation.)

Replace y with f−1(x): f−1(x) =√x + 3 for all x in the

range of f(x), or f−1(x) =√x + 3, x ≥ −3.

Answers may vary. There are other restrictions that alsomake f(x) one-to-one.

113. Answers may vary. f(x) =3x

, f(x) = 1 − x, f(x) = x.

Exercise Set 5.2

1. e4 ≈ 54.5982

3. e−2.458 ≈ 0.0856

5. f(x) = −2x − 1

f(0) = −20 − 1 = −1 − 1 = −2

The only graph with y-intercept (0,−2) is (f).

7. f(x) = ex + 3

This is the graph of f(x) = ex shifted up 3 units. Then(e) is the correct choice.

9. f(x) = 3−x − 2

f(0) = 3−0 − 2 = 1 − 2 = −1

Since the y-intercept is (0,−1), the correct graph is (a) or(c). Check another point on the graph. f(−1) =3−(−1) − 2 = 3 − 2 = 1, so (−1, 1) is on the graph. Thus(a) is the correct choice.

11. Graph f(x) = 3x.

Compute some function values, plot the correspondingpoints, and connect them with a smooth curve.

x y = f(x) (x, y)

−3127

(− 3,

127

)

−219

(− 2,

19

)

−113

(− 1,

13

)0 1 (0, 1)

1 3 (1, 3)

2 9 (2, 9)

3 27 (3, 27)

13. Graph f(x) = 6x.

Compute some function values, plot the correspondingpoints, and connect them with a smooth curve.

x y = f(x) (x, y)

−31

216

(− 3,

1216

)

−2136

(− 2,

136

)

−116

(− 1,

16

)0 1 (0, 1)

1 6 (1, 6)

2 36 (2, 36)

3 216 (3, 216)

15. Graph f(x) =(

14

)x

.

Compute some function values, plot the correspondingpoints, and connect them with a smooth curve.

x y = f(x) (x, y)

−3 64 (−3, 64)

−2 16 (−2, 16)

−1 4 (−1, 4)

0 1 (0, 1)

114

(1,

14

)

2116

(2,

116

)

3164

(3,

164

)

17. Graph y = −2x.

x y (x, y)

−3 −18

(− 3,−1

8

)

−2 −14

(− 2,−1

4

)

−1 −12

(− 1,−1

2

)0 −1 (0,−1)

1 −2 (1,−2)

2 −4 (2,−4)

3 −8 (3,−8)

Copyright © 2013 Pearson Education, Inc.

Page 191: JUDITH A. PENNA -  · PDF fileSTUDENT’S SOLUTIONS MANUAL JUDITH A. PENNA Indiana University Purdue University Indianapolis COLLEGE ALGEBRA: GRAPHS AND MODELS FIFTH EDITION

2 4�4 �2

2

4

�4

�2x

y

f(x) � �0.25x � 4

2 4�4 �2

2

4

�4

�2x

y

f(x) � 1 � e�x

x

y

1 2 3�1�3 �2

y � ~e x

�1

1

2

3

4

5

6

1 x

y

f (x) � 1 � e�x

2 3�1�2�3

12

�2�3�4�5�6

y

x

2

4

�2

�2�4 42

6

8

f (x)� 2x � 1

y

x

2

4

�2

�2�4 42

�4

6

f (x) � 2x � 3

4

42�4 �2

2

�4

�2x

y

f(x) � 21�x � 2

y

x

2

4

�2

�2�4 42

�4

6 f (x)� 4 � 3�x

188 Chapter 5: Exponential and Logarithmic Functions

19. Graph f(x) = −0.25x + 4.

x y = f(x) (x, y)

−3 −60 (−3,−60)

−2 −12 (−2,−12)

−1 0 (−1, 0)

0 3 (0, 3)

1 3.75 (1, 3.75)

2 3.94 (2, 3.94)

3 3.98 (3, 3.98)

21. Graph f(x) = 1 + e−x.

x y = f(x) (x, y)

−3 21.1 (−3, 21.1)

−2 8.4 (−2, 8.4)

−1 3.7 (−1, 3.7)

0 2 (0, 2)

1 1.4 (1, 1.4)

2 1.1 (2, 1.1)

3 1.0 (3, 1.0)

23. Graph y =14ex.

Choose values for x and compute the corresponding y-values. Plot the points (x, y) and connect them with asmooth curve.

x y (x, y)

−3 0.0124 (−3, 0.0124)

−2 0.0338 (−2, 0.0338)

−1 0.0920 (−1, 0.0920)

0 0.25 (0, 0.25)

1 0.6796 (1, 0.6796)

2 1.8473 (2, 1.8473)

3 5.0214 (3, 5.0214)

25. Graph f(x) = 1 − e−x.

Compute some function values, plot the correspondingpoints, and connect them with a smooth curve.

x y (x, y)

−3 −19.0855 (−3,−19.0855)

−2 −6.3891 (−2,−6.3891)

−1 −1.7183 (−1,−1.7183)

0 0 (0, 0)

1 0.6321 (1, 0.6321)

2 0.8647 (2, 0.8647)

3 0.9502 (3, 0.9502)

27. Shift the graph of y = 2x left 1 unit.

29. Shift the graph of y = 2x down 3 units.

31. Shift the graph of y = 2x left 1 unit, reflect it across they-axis, and shift it up 2 units.

33. Reflect the graph of y = 3x across the y-axis, then acrossthe x-axis, and then shift it up 4 units.

Copyright © 2013 Pearson Education, Inc.

Page 192: JUDITH A. PENNA -  · PDF fileSTUDENT’S SOLUTIONS MANUAL JUDITH A. PENNA Indiana University Purdue University Indianapolis COLLEGE ALGEBRA: GRAPHS AND MODELS FIFTH EDITION

y

x

2

4

�2�4 42

6

8

f (x) � ( )x � 1 32

f (x)� 2x � 3 � 5

y

x

4

2

�2

�4

�6�8 �2�4

4

42�4 �2

2

�4

�2x

y

f(x) � 3.2

x�1 � 1

y

x

2

4

�2�4 42

6

8

f (x) � e2x

y

x

2

4

�2 4 62

6

8

y � e�x � 1

f (x)� 2(1 � e�x )

y

x

2

4

�2

�4

�2 4 62

4

42�4 �2

2

�4

�2x

y

e�x � 4, for x ��2,x � 3,x2,

for �2 � x � 1,for x � 1

f (x) �

y � 82,000(1.01125)4x

0

200,000

0 25

Exercise Set 5.2 189

35. Shift the graph of y =(

32

)x

right 1 unit.

37. Shift the graph of y = 2x left 3 units and down 5 units.

39. Shift the graph of y = 2x right 1 unit, stretch it vertically,and shift it up 1 unit.

41. Shrink the graph of y = ex horizontally.

43. Reflect the graph of y = ex across the x-axis, shift it up 1unit, and shrink it vertically. The graph is in the answersection in the text.

45. Shift the graph of y = ex left 1 unit and reflect it acrossthe y-axis.

47. Reflect the graph of y = ex across the y-axis and thenacross the x-axis; shift it up 1 unit and then stretch itvertically.

49. We graph f(x) = e−x − 4 for x < −2, f(x) = x + 3 for−2 ≤ x < 1, and f(x) = x2 for x ≥ 1.

51. a) We use the formula A = P

(1 +

r

n

)nt

and substi-

tute 82,000 for P , 0.045 for r, and 4 for n.

A(t) = 82, 000(

1 +0.045

4

)4t

= 82, 000(1.01125)4t

b)

c) A(0) = 82, 000(1.01125)4·0 = $82, 000

A(2) = 82, 000(1.01125)4·2 ≈ $89, 677.22

A(5) = 82, 000(1.01125)4·5 ≈ $102, 561.54

A(10) = 82, 000(1.01125)4·10 ≈ $128, 278.90

d) Using the Intersect feature, we find that the firstcoordinate of the point of intersection of y1 =82, 000(1.01125)4x and y2 = 100, 000 is about 4.43,so the amount in the account will reach $100,000 inabout 4.43 years. This is about 4 years, 5 months,and 5 days.

53. We use the formula A = P

(1 +

r

n

)nt

and substitute 3000

for P , 0.05 for r, and 4 for n.

A(t) = 3000(

1 +0.054

)4t

= 3000(1.0125)4t

Copyright © 2013 Pearson Education, Inc.

Page 193: JUDITH A. PENNA -  · PDF fileSTUDENT’S SOLUTIONS MANUAL JUDITH A. PENNA Indiana University Purdue University Indianapolis COLLEGE ALGEBRA: GRAPHS AND MODELS FIFTH EDITION

y � 56,395(0.85)x

0

60,000

0 8

190 Chapter 5: Exponential and Logarithmic Functions

On Willis’ sixteenth birthday, t = 16 − 6 = 10.

A(10) = 3000(1.0125)4·10 = 4930.86

When the CD matures $4930.86 will be available.

55. We use the formula A = P

(1 +

r

n

)nt

and substitute 3000

for P , 0.04 for r, 2 for n, and 2 for t.

A = 3000(

1 +0.042

)2·2≈ $3247.30

57. We use the formula A = P

(1 +

r

n

)nt

and substitute

120,000 for P , 0.025 for r, 1 for n, and 10 for t.

A = 120, 000(

1 +0.025

1

)1·10≈ $153, 610.15

59. We use the formula A = P

(1 +

r

n

)nt

and substitute

53,500 for P , 0.055 for r, 4 for n, and 6.5 for t.

A = 53, 500(

1 +0.055

4

)4(6.5)

≈ $76, 305.59

61. We use the formula A = P

(1 +

r

n

)nt

and substitute

17,400 for P , 0.081 for r, 365 for n, and 5 for t.

A = 17, 400(

1 +0.081365

)365·5≈ $26, 086.69

63. W (x) = 23, 672.16(1.112)x

In 2000, x = 2000 − 1998 = 2.

W (2) = 23, 672.16(1.112)2 ≈ 29, 272 service members

In 2008, x = 2008 − 1998 = 10.

W (10) = 23, 672.16(1.112)10 ≈ 68, 436 service members

In 2013, x = 2013 − 1998 = 15.

W (15) = 23, 672.16(1.112)15 ≈ 116, 362 service members

65. T (x) = 400(1.055)x

In 1950, x = 1950 − 1913 = 37.

T (37) = 400(1.055)37 ≈ 2900 pages

In 1990, x = 1990 − 1913 = 77.

T (77) = 400(1.055)77 ≈ 24, 689 pages

In 2000, x = 2000 − 1913 = 87.

T (87) = 400(1.055)87 ≈ 42, 172 pages

67. M(x) = (200, 000)(1.1802)x

In 2003, x = 2003 − 2001 = 2.

M(2) = (200, 000)(1.1802)2 ≈ $278, 574

In 2007, x = 2007 − 2001 = 6.

M(6) = (200, 000)(1.1802)6 ≈ $540, 460

In 2013, x = 2013 − 2001 = 12.

M(12) = (200, 000)(1.1802)12 ≈ $1, 460, 486

69. R(x) = 80, 000(1.1522)x

In 1999, x = 1999 − 1996 = 3.

R(3) = 80, 000(1.1522)3 ≈ 122, 370 tons

In 2007, x = 2007 − 1996 = 11.

R(11) = 80, 000(1.1522)11 ≈ 380, 099 tons

In 2012, x = 2012 − 1996 = 16.

R(16) = 80, 000(1.1522)16 ≈ 771, 855 tons

71. T (x) = 23.7624(1.0752)x

In 2007, x = 2007 − 2006 = 1.

T (1) = 23.7624(1.0752)1 ≈ 25.5 million people, or25,500,000 people

In 2014, x = 2014 − 2006 = 8.

T (8) = 23.7624(1.0752)8 ≈ 42.4 million people, or42,400,000 people

73. a)

b) V (t) = 56, 395(0.9)t

V (0) = 56, 395(0.9)0 = $56, 395

V (1) = 56, 395(0.9)1 ≈ $50, 756

V (3) = 56, 395(0.9)3 ≈ $41, 112

V (6) = 56, 395(0.9)6 ≈ $29, 971

V (10) = 56, 395(0.9)10 ≈ $19, 664

c) Using the Intersect feature, we find that the firstcoordinate of the point of intersection ofy1 = 56, 395(0.9)x and y2 = 30, 000 is approxi-mately 6, so the machine will be replaced after 6years.

75. a)

b) f(25) = 100(1 − e−0.04(25)) ≈ 63%.

c) Using the Intersect feature, we find the first coordi-nate of the point of intersection ofy1 = 100(1 − e−0.04x) and y2 = 90 is approximately58, so 90% of the product market will have boughtthe product after 58 days.

77. Graph (c) is the graph of y = 3x − 3−x.

79. Graph (a) is the graph of f(x) = −2.3x.

81. Graph (l) is the graph of y = 2−|x|.

83. Graph (g) is the graph of f(x) = (0.58)x − 1.

85. Graph (i) is the graph of g(x) = e|x|.

Copyright © 2013 Pearson Education, Inc.

Page 194: JUDITH A. PENNA -  · PDF fileSTUDENT’S SOLUTIONS MANUAL JUDITH A. PENNA Indiana University Purdue University Indianapolis COLLEGE ALGEBRA: GRAPHS AND MODELS FIFTH EDITION

8642 x

y

x � 3y

�1

�2

�3

1

2

3

Exercise Set 5.3 191

87. Graph (k) is the graph of y = 2−x2.

89. Graph (m) is the graph of g(x) =ex − e−x

2.

91. Graph y1 = |1 − 3x| and y2 = 4 + 3−x2. Use the Intersect

feature to find their point of intersection, (1.481, 4.090).

93. Graph y1 = 2ex−3 and y2 =ex

x. Use the Intersect feature

to find their points of intersection, (−0.402,−1.662) and(1.051, 2.722).

95. Graph y1 = 5.3x − 4.2x and y2 = 1073. Use the Inter-sect feature to find the first coordinate of their point ofintersection. The solution is 4.448.

97. Graph y1 = 2x and y2 = 1. Use the Intersect feature tofind their point of intersection, (0, 1). Note that the graphof y1 lies above the graph of y2 for all points to the rightof this point. Thus the solution set is (0,∞).

99. Graph y1 = 2x + 3x and y2 = x2 + x3. Use the Inter-sect feature to find the first coordinates of their points ofintersection. The solutions are 2.294 and 3.228.

101. (1 − 4i)(7 + 6i) = 7 + 6i− 28i− 24i2

= 7 + 6i− 28i + 24

= 31 − 22i

103. 2x2 − 13x− 7 = 0 Setting f(x) = 0

(2x + 1)(x− 7) = 0

2x + 1 = 0 or x− 7 = 0

2x = −1 or x = 7

x = −12

or x = 7

The zeros of the function are −12

and 7, and the x-

intercepts are(− 1

2, 0)

and (7, 0).

105. x4 − x2 = 0 Setting h(x) = 0

x2(x2 − 1) = 0

x2(x + 1)(x− 1) = 0

x2 = 0 or x + 1 = 0 or x− 1 = 0

x = 0 or x = −1 or x = 1

The zeros of the function are 0, −1, and 1, and the x-intercepts are (0, 0), (−1, 0), and (1, 0).

107. x3 + 6x2 − 16x = 0

x(x2 + 6x− 16) = 0

x(x + 8)(x− 2) = 0

x = 0 or x + 8 = 0 or x− 2 = 0

x = 0 or x = −8 or x = 2

The solutions are 0, −8, and 2.

109. 7π ≈ 451.8078726 and π7 ≈ 3020.293228, so π7 is larger.

7080 ≈ 4.054 × 10147 and 8070 ≈ 1.646 × 10133, so 7080 islarger.

111. a)

b) There are no x-intercepts, so the function has nozeros.

c) Relative minimum: none;

relative maximum: 1 at x = 0

113. f(x + h) − f(x)h

=(2ex+h − 3) − (2ex − 3)

h

=2ex+h − 3 − 2ex + 3

h

=2ex+h − 2ex

h

=2ex(eh − 1)

h(ex · eh = ex+h)

Exercise Set 5.3

1. Graph x = 3y.

Choose values for y and compute the corresponding x-values. Plot the points (x, y) and connect them with asmooth curve.

x y (x, y)

127

−3(

127

,−3)

19

−2(

19,−2

)13

−1(

13,−1

)1 0 (1, 0)

3 1 (3, 1)

9 2 (9, 2)

27 3 (27, 3)

3. Graph x =(

12

)y

.

Choose values for y and compute the corresponding x-values. Plot the points (x, y) and connect them with asmooth curve.

Copyright © 2013 Pearson Education, Inc.

Page 195: JUDITH A. PENNA -  · PDF fileSTUDENT’S SOLUTIONS MANUAL JUDITH A. PENNA Indiana University Purdue University Indianapolis COLLEGE ALGEBRA: GRAPHS AND MODELS FIFTH EDITION

x

y

�1

�2

�3

1

2

3

x � �q�y

2 3 4 5 6 7 8

�1 x

y

y � log3 x

2 4 6 8 10�1

�2

�3

1

2

3

�1 x

y

2 4 6 8 10�1

�2

�3

1

2

3f (x) � log x

192 Chapter 5: Exponential and Logarithmic Functions

x y (x, y)

8 −3 (8,−3)

4 −2 (4,−2)

2 −1 (2,−1)

1 0 (1, 0)12

1(

12, 1)

14

2(

14, 2)

18

3(

18, 3)

5. Graph y = log3 x.

The equation y = log3 x is equivalent to x = 3y. We canfind ordered pairs that are solutions by choosing values fory and computing the corresponding x-values.

For y = −2, x = 3−2 =19.

For y = −1, x = 3−1 =13.

For y = 0, x = 30 = 1.

For y = 1, x = 31 = 3.

For y = 2, x = 32 = 9.

x, or 3y y

19

−2

13

−1

1 0

3 1

9 2

7. Graph f(x) = log x.

Think of f(x) as y. The equation y = log x is equivalentto x = 10y. We can find ordered pairs that are solutionsby choosing values for y and computing the correspondingx-values.

For y = −2, x = 10−2 = 0.01.

For y = −1, x = 10−1 = 0.1.

For y = 0, x = 100 = 1.

For y = 1, x = 101 = 10.

For y = 2, x = 102 = 100.

x, or 10y y

0.01 −2

0.1 −1

1 0

10 1

100 2

9. log2 16 = 4 because the exponent to which we raise 2 toget 16 is 4.

11. log5 125 = 3, because the exponent to which we raise 5 toget 125 is 3.

13. log 0.001 = −3, because the exponent to which we raise 10to get 0.001 is −3.

15. log2

14

= −2, because the exponent to which we raise 2 to

get14

is −2

17. ln 1 = 0, because the exponent to which we raise e to get1 is 0.

19. log 10 = 1, because the exponent to which we raise 10 toget 10 is 1.

21. log5 54 = 4, because the exponent to which we raise 5 toget 54 is 4.

23. log34√

3 = log3 31/4 =14, because the exponent to which

we raise 3 to get 31/4 is14.

25. log 10−7 = −7, because the exponent to which we raise 10to get 10−7 is −7.

27. log49 7 =12, because the exponent to which we raise 49 to

get 7 is12. (491/2 =

√49 = 7)

29. ln e3/4 =34, because the exponent to which we raise e to

get e3/4 is34.

31. log4 1 = 0, because the exponent to which we raise 4 toget 1 is 0.

33. ln√e = ln e1/2 =

12, because the exponent to which we

raise e to get e1/2 is12.

35. The exponent is thelogarithm.↓ ↓

103= 1000⇒3 = log101000↑ ↑ The base remains the

same.

We could also say 3 = log 1000.

37. The exponent isthe logarithm.↓ ↓

81/3 = 2 ⇒ log8 2 =13↑ ↑ The base remains

the same.

39. e3 = t ⇒ loge t = 3, or ln t = 3

41. e2 = 7.3891 ⇒ loge 7.3891 = 2, or ln 7.3891 = 2

43. pk = 3 ⇒ logp 3 = k

Copyright © 2013 Pearson Education, Inc.

Page 196: JUDITH A. PENNA -  · PDF fileSTUDENT’S SOLUTIONS MANUAL JUDITH A. PENNA Indiana University Purdue University Indianapolis COLLEGE ALGEBRA: GRAPHS AND MODELS FIFTH EDITION

f(x) � 3x

f �1(x) � log3 x

y

x

2

4

�2

�4

�2�4 42

f (x) � log x

f �1(x) � 10x

y

x

2

4

�2

�4

�2�4 42

f(x) � log2 (x � 3)

y

x

2

4

�2

�4

�2 4 62

y � log3 x � 1

y

x

2

4

�2

�4

4 6 82

Exercise Set 5.3 193

45. The logarithm is theexponent.↓ ↓

log55 = 1⇒51 = 5↑ ↑ The base remains the same.

47. log 0.01 = −2 is equivalent to log10 0.01 = −2.

The logarithm isthe exponent.↓ ↓

log100.01 = −2⇒10−2 = 0.01↑ ↑ The base remains

the same.

49. ln 30 = 3.4012 ⇒ e3.4012 = 30

51. loga M = −x ⇒ a−x = M

53. loga T 3 = x ⇒ ax = T 3

55. log 3 ≈ 0.4771

57. log 532 ≈ 2.7259

59. log 0.57 ≈ −0.2441

61. log(−2) does not exist. (The calculator gives an error mes-sage.)

63. ln 2 ≈ 0.6931

65. ln 809.3 ≈ 6.6962

67. ln(−1.32) does not exist. (The calculator gives an errormessage.)

69. Let a = 10, b = 4, and M = 100 and substitute in thechange-of-base formula.

log4 100 =log10 100log10 4

≈ 3.3219

71. Let a = 10, b = 100, and M = 0.3 and substitute in thechange-of-base formula.

log100 0.3 =log10 0.3log10 100

≈ −0.2614

73. Let a = 10, b = 200, and M = 50 and substitute in thechange-of-base formula.

log200 50 =log10 50log10 200

≈ 0.7384

75. Let a = e, b = 3, and M = 12 and substitute in thechange-of-base formula.

log3 12 =ln 12ln 3

≈ 2.2619

77. Let a = e, b = 100, and M = 15 and substitute in thechange-of-base formula.

log100 15 =ln 15ln 100

≈ 0.5880

79. Graph y = 3x and then reflect this graph across the liney = x to get the graph of y = log3 x.

81. Graph y = log x and then reflect this graph across the liney = x to get the graph of y = 10x.

83. Shift the graph of y = log2 x left 3 units.

Domain: (−3,∞)

Vertical asymptote: x = −3

85. Shift the graph of y = log3 x down 1 unit.

Domain: (0,∞)

Vertical asymptote: x = 0

Copyright © 2013 Pearson Education, Inc.

Page 197: JUDITH A. PENNA -  · PDF fileSTUDENT’S SOLUTIONS MANUAL JUDITH A. PENNA Indiana University Purdue University Indianapolis COLLEGE ALGEBRA: GRAPHS AND MODELS FIFTH EDITION

y

x

2

4

8642

6

8

f(x) � 4 ln x

y � 2 � ln x

y

x

2

4

�2

�4

4 6 82

4

42�4 �2

2

�4

�2x

y

f(x) � log (x � 1) � 2

12

4

42�4 �2

2

�4

�2x

y

5,log x � 1, for x � 0

for x � 0,g (x) �

194 Chapter 5: Exponential and Logarithmic Functions

87. Stretch the graph of y = ln x vertically.

Domain: (0,∞)

Vertical asymptote: x = 0

89. Reflect the graph of y = ln x across the x-axis and thenshift it up 2 units.

Domain: (0,∞)

Vertical asymptote: x = 0

91. Shift the graph of y = log x right 1 unit, shrink it verti-cally, and shift it down 2 units.

93. Graph g(x) = 5 for x ≤ 0 and g(x) = log x + 1 for x > 0.

95. a) We substitute 598.541 for P , since P is in thousands.

w(598.541) = 0.37 ln 598.541 + 0.05

≈ 2.4 ft/sec

b) We substitute 3833.995 for P , since P is in thou-sands.w(3833.995) = 0.37 ln 3833.995 + 0.05

≈ 3.1 ft/sec

c) We substitute 433.746 for P , since P is in thousands.

w(433.746) = 0.37 ln 433.746 + 0.05

≈ 2.3 ft/sec

d) We substitute 2242.193 for P , since P is in thou-sands.w(2242.193) = 0.37 ln 2242.193 + 0.05

≈ 2.9 ft/sec

e) We substitute 669.651 for P , since P is in thousands.

w(669.651) = 0.37 ln 669.651 + 0.05

≈ 2.5 ft/sec

f) We substitute 340.882 for P , since P is in thousands.

w(340.882) = 0.37 ln 340.882 + 0.05

≈ 2.2 ft/sec

g) We substitute 798.382 for P , since P is in thousands.

w(798.382) = 0.37 ln 798.382 + 0.05

≈ 2.5 ft/sec

h) We substitute 279.243 for P , since P is in thousands.

w(279.243) = 0.37 ln 279.243 + 0.05

≈ 2.1 ft/sec

97. a) R = log107.7 · I0

I0= log 107.7 = 7.7

b) R = log109.5 · I0

I0= log 109.5 = 9.5

c) R = log106.6 · I0

I0= log 106.6 = 6.6

d) R = log107.4 · I0

I0= log 107.4 = 7.4

e) R = log108.0 · I0

I0= log 108.0 = 8.0

f) R = log107.9 · I0

I0= log 107.9 = 7.9

g) R = log109.1 · I0

I0= log 109.1 = 9.1

h) R = log109.3 · I0

I0= log 109.3 = 9.3

99. a) 7 = − log[H+]

−7 = log[H+]

H+ = 10−7 Using the definition oflogarithm

b) 5.4 = − log[H+]

−5.4 = log[H+]

H+ = 10−5.4 Using the definition of

logarithm

H+ ≈ 4.0 × 10−6

Copyright © 2013 Pearson Education, Inc.

Page 198: JUDITH A. PENNA -  · PDF fileSTUDENT’S SOLUTIONS MANUAL JUDITH A. PENNA Indiana University Purdue University Indianapolis COLLEGE ALGEBRA: GRAPHS AND MODELS FIFTH EDITION

x

y

1—1—3 32 54—2—4—5—1

—2

—3

—4

1

2

3

4

5

—5

f (x ) = 3 + x 2

Chapter 5 Mid-Chapter Mixed Review 195

c) 3.2 = − log[H+]

−3.2 = log[H+]

H+ = 10−3.2 Using the definition oflogarithm

H+ ≈ 6.3 × 10−4

d) 4.8 = − log[H+]

−4.8 = log[H+]

H+ = 10−4.8 Using the definition oflogarithm

H+ ≈ 1.6 × 10−5

101. a) L = 10 log1014 · I0

I0= 10 log 1014 = 10 · 14

≈ 140 decibels

b) L = 10 log1011.5 · I0

I0= 10 log 1011.5 = 10 · 11.5

≈ 115 decibels

c) L = 10 log109 · I0

I0= 10 log 109 = 10 · 9= 90 decibels

d) L = 10 log106.5 · I0

I0= 10 log 106.5 = 10 · 6.5= 65 decibels

e) L = 10 log1010 · I0

I0= 10 log 1010 · 10 · 10

= 100 decibels

f) L = 10 log1019.4 · I0

I0= 10 log 1019.4 · 10 · 19.4

= 194 decibels

103. y = 6 = 0 · x + 6

Slope: 0; y-intercept (0, 6)

105. −5∣∣ 1 −6 3 10

−5 55 −2901 −11 58 −280

The remainder is −280, so f(−5) = −280.

107. f(x) = (x−√7)(x +

√7)(x− 0)

= (x2 − 7)(x)

= x3 − 7x

109. Using the change-of-base formula, we getlog5 8log2 8

= log2 8 = 3.

111. f(x) = log5 x3

x3 must be positive. Since x3 > 0 for x > 0, the domainis (0,∞).

113. f(x) = ln |x||x| must be positive. Since |x| > 0 for x = 0, the domainis (−∞, 0) ∪ (0,∞).

115. Graph y = log2(2x + 5) =log(2x + 5)

log 2. Observe

that outputs are negative for inputs between −52

and −2.

Thus, the solution set is(− 5

2,−2

).

117. Graph (d) is the graph of f(x) = ln |x|.119. Graph (b) is the graph of f(x) = lnx2.

121. a)

b) Use the Zero feature. The zero is 1.c) Use the Minimum feature. The relative minimum is

−0.368 at x = 0.368. There is no relative maximum.

123. a)

b) Use the Zero feature. The zero is 1.c) Use the Maximum feature. There is no relative min-

imum. The relative maximum is 0.184 at x = 1.649.

125. Graph y1 = 4 lnx and y2 =4

ex + 1and use the Intersect

feature to find the point(s) of intersection of the graphs.The point of intersection is (1.250, 0.891).

Chapter 5 Mid-Chapter Mixed Review

1. The statement is false. The domain of y = log x, for in-stance, is (0,∞).

3. f(0) = e−0 = 1, so the y-intercept is (0, 1). The givenstatement is false.

5. The graph of f(x) = 3 + x2 is shown below. Since thereare many horizontal lines that cross the graph more thanonce, the function is not one-to-one and thus does not havean inverse that is a function.

Copyright © 2013 Pearson Education, Inc.

Page 199: JUDITH A. PENNA -  · PDF fileSTUDENT’S SOLUTIONS MANUAL JUDITH A. PENNA Indiana University Purdue University Indianapolis COLLEGE ALGEBRA: GRAPHS AND MODELS FIFTH EDITION

196 Chapter 5: Exponential and Logarithmic Functions

7. (f−1 ◦ f)(x) = f−1(f(x)) = (√x− 5)2 +5 = x− 5+5 = x

(f ◦ f−1)(x) = f(f−1(x)) =√x2 + 5 − 5 =

√x2 = x

(This assumes that the domain of f−1(x) is restricted to{x|x ≥ 0}.)Since (f−1 ◦ f)(x) = x = (f ◦ f−1)(x), we know thatf−1(x) = x2 + 5.

9. The graph of y = log2 x is (d).

11. The graph of f(x) = ex−1 is (c).

13. The graph of f(x) = ln (x− 2) is (b).

15. The graph of f(x) = | log x| is (e).

17. A = P

(1 +

r

n

)nt

A = 3200(

1 +0.045

4

)4·6≈ $4185.57

19. ln e−4/5 is −45

because the exponent to which we raise e

to get e−4/5 is −45.

21. ln e2 = 2 because the exponent to which we raise e to gete2 is 2.

23. log2

116

= −4 because the exponent to which we raise 2 to

get116

, or 2−4, is −4.

25. log3 27 = 3 because the exponent to which we raise 3 toget 27 is 3.

27. ln e = 1 because the exponent to which we raise e to gete is 1.

29. log T = r is equivalent to 10r = T .

31. logπ 10 =log 10log π

=1

log π≈ 2.0115

33. The most interest will be earned the eighth year, becausethe principle is greatest during that year.

35. If log b < 0, then b < 1.

Exercise Set 5.4

1. Use the product rule.

log3(81 · 27) = log3 81 + log3 27 = 4 + 3 = 7

3. Use the product rule.

log5(5 · 125) = log5 5 + log5 125 = 1 + 3 = 4

5. Use the product rule.

logt 8Y = logt 8 + logt Y

7. Use the product rule.

lnxy = lnx + ln y

9. Use the power rule.

logb t3 = 3 logb t

11. Use the power rule.

log y8 = 8 log y

13. Use the power rule.

logc K−6 = −6 logc K

15. Use the power rule.

ln 3√

4 = ln 41/3 =13

ln 4

17. Use the quotient rule.

logtM

8= logt M − logt 8

19. Use the quotient rule.

logx

y= log x− log y

21. Use the quotient rule.

lnr

s= ln r − ln s

23. loga 6xy5z4

= loga 6 + loga x + loga y5 + loga z4

Product rule

= loga 6 + loga x + 5 loga y + 4 loga z

Power rule

25. logbp2q5

m4b9

= logb p2q5 − logb m4b9 Quotient rule

= logb p2 + logb q5 − (logb m4 + logb b9)

Product rule

= logb p2 + logb q5 − logb m4 − logb b9

= logb p2 + logb q5 − logb m4 − 9 (logb b9 = 9)

= 2 logb p + 5 logb q − 4 logb m− 9 Power rule

27. ln2

3x3y= ln 2 − ln 3x3y Quotient rule

= ln 2 − (ln 3 + lnx3 + ln y) Product rule

= ln 2 − ln 3 − lnx3 − ln y

= ln 2 − ln 3 − 3 lnx− ln y Power rule

29. log√r3t

= log(r3t)1/2

=12

log r3t Power rule

=12(log r3 + log t) Product rule

=12(3 log r + log t) Power rule

=32

log r +12

log t

Copyright © 2013 Pearson Education, Inc.

Page 200: JUDITH A. PENNA -  · PDF fileSTUDENT’S SOLUTIONS MANUAL JUDITH A. PENNA Indiana University Purdue University Indianapolis COLLEGE ALGEBRA: GRAPHS AND MODELS FIFTH EDITION

Exercise Set 5.4 197

31. loga

√x6

p5q8

=12

logax6

p5q8

=12[loga x

6 − loga(p5q8)] Quotient rule

=12[loga x

6 − (loga p5 + loga q

8)] Product rule

=12(loga x

6 − loga p5 − loga q

8)

=12(6 loga x− 5 loga p− 8 loga q) Power rule

= 3 loga x− 52

loga p− 4 loga q

33. loga4

√m8n12

a3b5

=14

logam8n12

a3b5Power rule

=14(loga m

8n12 − loga a3b5) Quotient rule

=14[loga m

8 + loga n12 − (loga a

3 + loga b5)]

Product rule

=14(loga m

8 + loga n12 − loga a

3 − loga b5)

=14(loga m

8+loga n12−3−loga b

5)

(loga a3 =3)

=14(8 loga m + 12 loga n− 3 − 5 loga b)

Power rule

= 2 loga m + 3 loga n− 34− 5

4loga b

35. loga 75 + loga 2

= loga(75 · 2) Product rule

= loga 150

37. log 10, 000 − log 100

= log10, 000

100Quotient rule

= log 100

= 2

39. 12

logn + 3 logm

= logn1/2 + logm3 Power rule

= logn1/2m3, or Product rule

logm3√n n1/2 =

√n

41. 12

loga x + 4 loga y − 3 loga x

= loga x1/2 + loga y4 − loga x3 Power rule

= loga x1/2y4 − loga x3 Product rule

= logax1/2y4

x3Quotient rule

= loga x−5/2y4, or loga

y4

x5/2Simplifying

43. ln x2 − 2 ln√x

= ln x2 − ln (√x)2 Power rule

= ln x2 − ln x [(√x)2 = x]

= lnx2

xQuotient rule

= ln x

45. ln(x2 − 4) − ln(x + 2)

= lnx2 − 4x + 2

Quotient rule

= ln(x + 2)(x− 2)

x + 2Factoring

= ln(x− 2) Removing a factor of 1

47. log(x2 − 5x− 14) − log(x2 − 4)

= logx2 − 5x− 14

x2 − 4Quotient rule

= log(x + 2)(x− 7)(x + 2)(x− 2)

Factoring

= logx− 7x− 2

Removing a factor of 1

49. lnx− 3[ln(x− 5) + ln(x + 5)]

= lnx− 3 ln[(x− 5)(x + 5)] Product rule

= lnx− 3 ln(x2 − 25)

= lnx− ln(x2 − 25)3 Power rule

= lnx

(x2 − 25)3Quotient rule

51. 32

ln 4x6 − 45

ln 2y10

=32

ln 22x6 − 45

ln 2y10 Writing 4 as 22

= ln(22x6)3/2 − ln(2y10)4/5 Power rule

= ln(23x9) − ln(24/5y8)

= ln23x9

24/5y8Quotient rule

= ln211/5x9

y8

53. loga211

= loga 2 − loga 11 Quotient rule

≈ 0.301 − 1.041

≈ −0.74

Copyright © 2013 Pearson Education, Inc.

Page 201: JUDITH A. PENNA -  · PDF fileSTUDENT’S SOLUTIONS MANUAL JUDITH A. PENNA Indiana University Purdue University Indianapolis COLLEGE ALGEBRA: GRAPHS AND MODELS FIFTH EDITION

198 Chapter 5: Exponential and Logarithmic Functions

55. loga 98 = loga(72 · 2)

= loga 72 + loga 2 Product rule

= 2 loga 7 + loga 2 Power rule

≈ 2(0.845) + 0.301

≈ 1.991

57.loga 2loga 7

≈ 0.3010.845

≈ 0.356

59. logb 125 = logb 53

= 3 logb 5 Power rule

≈ 3(1.609)

≈ 4.827

61. logb16

= logb 1 − logb 6 Quotient rule

= logb 1 − logb(2 · 3)

= logb 1 − (logb 2 + logb 3) Product rule

= logb 1 − logb 2 − logb 3

≈ 0 − 0.693 − 1.099

≈ −1.792

63. logb3b

= logb 3 − logb b Quotient rule

≈ 1.099 − 1

≈ 0.099

65. logp p3 = 3 (loga ax = x)

67. loge e|x−4| = |x− 4| (loga ax = x)

69. 3log3 4x = 4x (aloga x = x)

71. 10logw = w (aloga x = x)

73. ln e8t = 8t (loga ax = x)

75. logb√b = logb b1/2

=12

logb b Power rule

=12· 1 (logb b = 1)

=12

77. The degree of f(x) = 5 − x2 + x4 is 4, so the function isquartic.

79. f(x) = −34

is of the form f(x) = mx+ b(with m = 0 and

b = −34

), so it is a linear function. In fact, it is a constant

function.

81. f(x) = − 3x

is of the form f(x) =p(x)q(x)

where p(x) and q(x)

are polynomials and q(x) is not the zero polynomial, sof(x) is a rational function.

83. The degree of f(x) = −13x3 − 4x2 + 6x + 42 is 3, so the

function is cubic.

85. f(x) =12x + 3 is of the form f(x) = mx + b, so it is a

linear function.

87. 5log5 8 = 2x

8 = 2x (aloga x = x)

4 = x

The solution is 4.

89. loga(x2 + xy + y2) + loga(x− y)

= loga[(x2 + xy + y2)(x− y)] Product rule

= loga(x3 − y3) Multiplying

91. logax− y√x2 − y2

= logax− y

(x2 − y2)1/2

= loga(x− y) − loga(x2 − y2)1/2 Quotient rule

= loga(x− y) − 12

loga(x2 − y2) Power rule

= loga(x− y) − 12

loga[(x + y)(x− y)]

= loga(x− y) − 12[loga(x + y) + loga(x− y)]

Product rule

= loga(x− y) − 12

loga(x + y) − 12

loga(x− y)

=12

loga(x− y) − 12

loga(x + y)

93. loga4√

y2z5

4√x3z−2

= loga4

√y2z5

x3z−2

= loga4

√y2z7

x3

= loga

(y2z7

x3

)1/4

=14

loga

(y2z7

x3

)Power rule

=14(loga y

2z7 − loga x3) Quotient rule

=14(loga y

2 + loga z7 − loga x

3) Product rule

=14(2 loga y + 7 loga z − 3 loga x) Power rule

=14(2 · 3 + 7 · 4 − 3 · 2)

=14· 28

= 7

95. loga M − loga N = logaM

NThis is the quotient rule, so it is true.

Copyright © 2013 Pearson Education, Inc.

Page 202: JUDITH A. PENNA -  · PDF fileSTUDENT’S SOLUTIONS MANUAL JUDITH A. PENNA Indiana University Purdue University Indianapolis COLLEGE ALGEBRA: GRAPHS AND MODELS FIFTH EDITION

Exercise Set 5.5 199

97.loga M

x=

1x

loga M = loga M1/x. The statement is true

by the power rule.

99. loga 8x = loga 8 + loga x = loga x + loga 8. The statementis true by the product rule and the commutative propertyof addition.

101. loga

(1x

)= loga x

−1 = −1 · loga x = −1 · 2 = −2

103. We use the change-of-base formula.

log10 11·log11 12·log12 13 · · ·log998 999·log999 1000

= log10 11· log10 12log10 11

· log10 13log10 12

· · ·log10 999log10 998

· log10 1000log10 999

=log10 11log10 11

· log10 12log10 12

· · · log10 999log10 999

· log10 1000

= log10 1000

= 3

105. ln a− ln b + xy = 0

ln a− ln b = −xy

lna

b= −xy

Then, using the definition of a logarithm, we havee−xy =

a

b.

107. loga

(x +

√x2 − 55

)

= loga

(x +

√x2 − 55

· x−√x2 − 5

x−√x2 − 5

)

= loga

(5

5(x−√x2 − 5

))

= loga

(1

x−√x2 − 5

)= loga 1 − loga(x−√

x2 − 5)

= − loga(x−√x2 − 5)

Exercise Set 5.5

1. 3x = 81

3x = 34

x = 4 The exponents are the same.

The solution is 4.

3. 22x = 8

22x = 23

2x = 3 The exponents are the same.

x =32

The solution is32.

5. 2x = 33

log 2x = log 33 Taking the commonlogarithm on both sides

x log 2 = log 33 Power rule

x =log 33log 2

x ≈ 1.51850.3010

x ≈ 5.044

The solution is 5.044.

7. 54x−7 = 125

54x−7 = 53

4x− 7 = 3

4x = 10

x =104

=52

The solution is52.

9. 27 = 35x · 9x2

33 = 35x · (32)x2

33 = 35x · 32x2

33 = 35x+2x2

3 = 5x + 2x2

0 = 2x2 + 5x− 3

0 = (2x− 1)(x + 3)

x =12

or x = −3

The solutions are −3 and12.

11. 84x = 70

log 84x = log 70

x log 84 = log 70

x =log 70log 84

x ≈ 1.84511.9243

x ≈ 0.959

The solution is 0.959.

13. 10−x = 52x

log 10−x = log 52x

−x = 2x log 5

0 = x + 2x log 5

0 = x(1 + 2 log 5)

0 = x Dividing by 1 + 2 log 5

The solution is 0.

Copyright © 2013 Pearson Education, Inc.

Page 203: JUDITH A. PENNA -  · PDF fileSTUDENT’S SOLUTIONS MANUAL JUDITH A. PENNA Indiana University Purdue University Indianapolis COLLEGE ALGEBRA: GRAPHS AND MODELS FIFTH EDITION

200 Chapter 5: Exponential and Logarithmic Functions

15. e−c = 52c

ln e−c = ln 52c

−c = 2c ln 5

0 = c + 2c ln 5

0 = c(1 + 2 ln 5)

0 = c Dividing by 1 + 2 ln 5

The solution is 0.

17. et = 1000

ln et = ln 1000

t = ln 1000 Using loga ax = x

t ≈ 6.908

The solution is 6.908.

19. e−0.03t = 0.08

ln e−0.03t = ln 0.08

−0.03t = ln 0.08

t =ln 0.08−0.03

t ≈ −2.5257−0.03

t ≈ 84.191

The solution is 84.191.

21. 3x = 2x−1

ln 3x = ln 2x−1

x ln 3 = (x− 1) ln 2

x ln 3 = x ln 2 − ln 2

ln 2 = x ln 2 − x ln 3

ln 2 = x(ln 2 − ln 3)ln 2

ln 2 − ln 3= x

0.69310.6931 − 1.0986

≈ x

−1.710 ≈ x

The solution is −1.710.

23. (3.9)x = 48

log(3.9)x = log 48

x log 3.9 = log 48

x =log 48log 3.9

x ≈ 1.68120.5911

x ≈ 2.844

The solution is 2.844.

25. ex + e−x = 5

e2x + 1 = 5ex Multiplying by ex

e2x − 5ex + 1 = 0 This equation is quadraticin ex.

ex =5 ±√

212

x = ln(

5 ±√21

2

)≈ ±1.567

The solutions are −1.567 and 1.567.

27. 32x−1 = 5x

log 32x−1 = log 5x

(2x− 1) log 3 = x log 5

2x log 3 − log 3 = x log 5

− log 3 = x log 5 − 2x log 3

− log 3 = x(log 5 − 2 log 3)− log 3

log 5 − 2 log 3= x

1.869 ≈ x

The solution is 1.869.

29. 2ex = 5 − e−x

2ex − 5 + e−x = 0

ex(2ex − 5 + e−x) = ex · 0 Multiplying by ex

2e2x − 5ex + 1 = 0

Let u = ex.

2u2 − 5u + 1 = 0 Substituting

a = 2, b = −5, c = 1

u =−b±√

b2 − 4ac2a

u =−(−5) ±√(−5)2 − 4 · 2 · 1

2 · 2

u =5 ±√

174

Replace u with ex.

ex =5 −√

174

or ex =5 +

√17

4

ln ex = ln(

5 −√17

4

)or ln ex = ln

(5 +

√17

4

)x ≈ −1.518 or x ≈ 0.825

The solutions are −1.518 and 0.825.

31. log5 x = 4

x = 54 Writing an equivalentexponential equation

x = 625

The solution is 625.

33. log x = −4 The base is 10.

x = 10−4, or 0.0001

The solution is 0.0001.

Copyright © 2013 Pearson Education, Inc.

Page 204: JUDITH A. PENNA -  · PDF fileSTUDENT’S SOLUTIONS MANUAL JUDITH A. PENNA Indiana University Purdue University Indianapolis COLLEGE ALGEBRA: GRAPHS AND MODELS FIFTH EDITION

Exercise Set 5.5 201

35. lnx = 1 The base is e.

x = e1 = e

The solution is e.

37. log64

14

= x

14

= 64x

14

= (43)x

4−1 = 43x

−1 = 3x

−13

= x

The solution is −13.

39. log2(10 + 3x) = 5

25 = 10 + 3x

32 = 10 + 3x

22 = 3x223

= x

The answer checks. The solution is223

.

41. log x + log(x− 9) = 1 The base is 10.

log10[x(x− 9)] = 1

x(x− 9) = 101

x2 − 9x = 10

x2 − 9x− 10 = 0

(x− 10)(x + 1) = 0x = 10 or x = −1

Check: For 10:log x + log(x− 9) = 1

log 10 + log(10 − 9) ? 1log 10 + log 1

∣∣1 + 0

∣∣1∣∣ 1 TRUE

For −1:log x + log(x− 9) = 1

log(−1) + log(−1 − 9) ? 1|

The number −1 does not check, because negative numbersdo not have logarithms. The solution is 10.

43. log2(x + 20) − log2(x + 2) = log2 x

log2

x + 20x + 2

= log2 x

x + 20x + 2

= x Using the property oflogarithmic equality

x + 20 = x2 + 2x Multiplying byx + 2

0 = x2 + x− 20

0 = (x + 5)(x− 4)

x + 5 = 0 or x− 4 = 0

x = −5 or x = 4

Check: For −5:log2(x + 20) − log2(x + 2) = log2 x

log2(−5 + 20) − log2(−5 + 2) ? log2(−5)|

The number −5 does not check, because negative numbersdo not have logarithms.

For 4:log2(x + 20) − log2(x + 2) = log2 x

log2(4 + 20) − log2(4 + 2) ? log2 4log2 24 − log2 6

∣∣∣log2

246

∣∣∣∣∣log2 4

∣∣ log2 4 TRUE

The solution is 4.

45. log8(x + 1) − log8 x = 2

log8

(x + 1x

)= 2 Quotient rule

x + 1x

= 82

x + 1x

= 64

x + 1 = 64x

1 = 63x163

= x

The answer checks. The solution is163

.

47. log x + log(x + 4) = log 12

log x(x + 4) = log 12

x(x + 4) = 12 Using the property oflogarithmic equality

x2 + 4x = 12

x2 + 4x− 12 = 0

(x + 6)(x− 2) = 0

x + 6 = 0 or x− 2 = 0

x = −6 or x = 2

Check: For −6:log x + log(x + 4) = log 12

log(−6) + log(−6 + 4) ? log 12|

The number −6 does not check, because negative numbersdo not have logarithms.

Copyright © 2013 Pearson Education, Inc.

Page 205: JUDITH A. PENNA -  · PDF fileSTUDENT’S SOLUTIONS MANUAL JUDITH A. PENNA Indiana University Purdue University Indianapolis COLLEGE ALGEBRA: GRAPHS AND MODELS FIFTH EDITION

202 Chapter 5: Exponential and Logarithmic Functions

For 2:log x + log(x + 4) = log 12

log 2 + log(2 + 4) ? log 12log 2 + log 6

∣∣∣log(2 · 6)

∣∣log 12

∣∣∣ log 12 TRUE

The solution is 2.

49. log(x+8)−log(x+1) = log 6

logx + 8x + 1

= log 6 Quotient rule

x + 8x + 1

= 6 Using the property oflogarithmic equality

x + 8 = 6x + 6 Multiplying by x+1

2 = 5x25

= x

The answer checks. The solution is25.

51. log4(x + 3) + log4(x− 3) = 2

log4[(x + 3)(x− 3)] = 2 Product rule

(x + 3)(x− 3) = 42

x2 − 9 = 16

x2 = 25

x = ±5

The number 5 checks, but −5 does not. The solution is 5.

53. log(2x + 1) − log(x− 2) = 1

log(

2x + 1x− 2

)= 1 Quotient rule

2x + 1x− 2

= 101 = 10

2x + 1 = 10x− 20

Multiplying by x− 2

21 = 8x218

= x

The answer checks. The solution is218

.

55. ln(x + 8) + ln(x− 1) = 2 lnx

ln(x + 8)(x− 1) = lnx2

(x + 8)(x− 1) = x2 Using the property oflogarithmic equality

x2 + 7x− 8 = x2

7x− 8 = 0

7x = 8

x =87

The answer checks. The solution is87.

57. log6 x = 1 − log6(x− 5)

log6 x + log6(x− 5) = 1

log6 x(x− 5) = 1

61 = x(x− 5)

6 = x2 − 5x

0 = x2 − 5x− 6

0 = (x + 1)(x− 6)

x + 1 = 0 or x− 6 = 0

x = −1 or x = 6

The number −1 does not check, but 6 does. The answeris 6.

59. 9x−1 = 100(3x)

(32)x−1 = 100(3x)

32x−2 = 100(3x)

32x−2

3x= 100

3x−2 = 100

log 3x−2 = log 100

(x− 2) log 3 = 2

x− 2 =2

log 3

x = 2 +2

log 3x ≈ 6.192

The solution is 6.192.

61. ex − 2 = −e−x

ex − 2 = − 1ex

e2x − 2ex = −1 Multiplying by ex

e2x − 2ex

+ 1 = 0

Let u = ex.u2 − 2u + 1 = 0

(u− 1)(u− 1) = 0

u− 1 = 0 or u− 1 = 0

u = 1 or u = 1

ex = 1 or ex = 1 Replacing u with ex

x = 0 or x = 0

The solution is 0.

63. 2x − 5 = 3x + 1

Graph y1 = 2x − 5 and y2 = 3x + 1 and find the firstcoordinates of the points of intersection using the Intersectfeature. The solutions are −1.911 and 4.222.

65. xe3x − 1 = 3

Graph y1 = xe3x − 1 and y2 = 3 and find the first coordi-nate of the point of intersection using the Intersect feature.The solution is 0.621.

Copyright © 2013 Pearson Education, Inc.

Page 206: JUDITH A. PENNA -  · PDF fileSTUDENT’S SOLUTIONS MANUAL JUDITH A. PENNA Indiana University Purdue University Indianapolis COLLEGE ALGEBRA: GRAPHS AND MODELS FIFTH EDITION

Exercise Set 5.5 203

67. 5e5x + 10 = 3x + 40

Graph y1 = 5e5x + 10 and y2 = 3x + 40 and find the firstcoordinates of the points of intersection using the Intersectfeature. The solutions are −10 and 0.366.

69. log8 x + log8(x + 2) = 2

Graph y1 =log xlog 8

+log(x + 2)

log 8and y2 = 2 and find the

first coordinate of the point of intersection using the inter-sect feature. The solution is 7.062.

71. log5(x + 7) − log5(2x− 3) = 1

Graph y1 =log(x + 7)

log 5− log(2x− 3)

log 5and y2 = 1 and find

the first coordinate of the point of intersection using theIntersect feature. The solution is 2.444.

73. Solving the first equation for y, we get

y =12.4 − 2.3x

3.8. Graph y1 =

12.4 − 2.3x3.8

and

y2 = 1.1 ln(x − 2.05) and use the Intersect feature to findthe point of intersection. It is (4.093, 0.786).

75. Graph y1 = 2.3 ln(x + 10.7) and y2 = 10e−0.007x2and use

the Intersect feature to find the point of intersection. It is(7.586, 6.684).

77. g(x) = x2 − 6

a) − b

2a= − 0

2 · 1 = 0

g(0) = 02 − 6 = −6

The vertex is (0,−6).

b) The axis of symmetry is x = 0.

c) Since the coefficient of the x2-term is positive, thefunction has a minimum value. It is the second coor-dinate of the vertex, −6, and it occurs when x = 0.

79. G(x) = −2x2 − 4x− 7

a) − b

2a= − −4

2(−2)= −1

G(−1) = −2(−1)2 − 4(−1) − 7 = −5

The vertex is (−1,−5).

b) The axis of symmetry is x = −1.

c) Since the coefficient of the x2-term is negative, thefunction has a maximum value. It is the secondcoordinate of the vertex, −5, and it occurs whenx = −1.

81. ex + e−x

ex − e−x= 3

ex + e−x = 3ex − 3e−x Multiplying by ex − e−x

4e−x = 2ex Subtracting ex and adding 3e−x

2e−x = ex

2 = e2x Multiplying by ex

ln 2 = ln e2x

ln 2 = 2xln 22

= x

0.347 ≈ x

The solution is 0.347.

83.√

lnx = ln√x

√lnx =

12

lnx Power rule

lnx =14(lnx)2 Squaring both sides

0 =14(lnx)2 − lnx

Let u = lnx and substitute.14u2 − u = 0

u

(14u− 1

)= 0

u = 0 or14u− 1 = 0

u = 0 or14u = 1

u = 0 or u = 4

lnx = 0 or lnx = 4

x = e0 = 1 or x = e4 ≈ 54.598

Both answers check. The solutions are 1 and e4, or 1 and54.598.

85. (log3 x)2 − log3 x2 = 3

(log3 x)2 − 2 log3 x− 3 = 0

Let u = log3 x and substitute:

u2 − 2u− 3 = 0

(u− 3)(u + 1) = 0

u = 3 or u = −1

log3 x = 3 or log3 x = −1

x = 33 or x = 3−1

x = 27 or x =13

Both answers check. The solutions are13

and 27.

Copyright © 2013 Pearson Education, Inc.

Page 207: JUDITH A. PENNA -  · PDF fileSTUDENT’S SOLUTIONS MANUAL JUDITH A. PENNA Indiana University Purdue University Indianapolis COLLEGE ALGEBRA: GRAPHS AND MODELS FIFTH EDITION

204 Chapter 5: Exponential and Logarithmic Functions

87. lnx2 = (lnx)2

2 lnx = (lnx)2

0 = (lnx)2 − 2 lnx

Let u = lnx and substitute.0 = u2 − 2u

0 = u(u− 2)

u = 0 or u = 2

lnx = 0 or lnx = 2

x = 1 or x = e2 ≈ 7.389

Both answers check. The solutions are 1 and e2, or 1 and7.389.

89. 52x − 3 · 5x + 2 = 0

(5x − 1)(5x − 2) = 0 This equation isquadratic in 5x.

5x = 1 or 5x = 2

log 5x = log 1 or log 5x = log 2

x log 5 = 0 or x log 5 = log 2

x = 0 or x =log 2log 5

≈ 0.431

The solutions are 0 and 0.431.

91. lnxln x = 4

lnx · lnx = 4

(lnx)2 = 4

lnx = ±2

lnx = −2 or lnx = 2

x = e−2 or x = e2

x ≈ 0.135 or x ≈ 7.389

Both answers check. The solutions are e−2 and e2, or 0.135and 7.389.

93.√

(e2x · e−5x)−4

ex ÷ e−x= e7

√e12x

ex−(−x)= e7

e6x

e2x= e7

e4x = e7

4x = 7

x =74

The solution is74.

95. a = log8 225, so 8a = 225 = 152.

b = log2 15, so 2b = 15.

Then 8a = (2b)2

(23)a = 22b

23a = 22b

3a = 2b

a =23b.

Exercise Set 5.6

1. a) Substitute 5.9 for P0 and 0.0176 for k in P (t) =P0e

kt. We have:

P (t) = 5.9e0.0176t, where P (t) is in millions and t isthe number of years after 2010.

b) In 2016, t = 2016 − 2010 = 6.

P (6) = 5.9e0.0176(6) ≈ 6.6 million

c) Substitute 7 for P (t) and solve for t.

7 = 5.9e0.0176t

75.9

= e0.0176t

ln7

5.9= ln e0.0176t

ln7

5.9= 0.0176t

ln7

5.90.0176

= t

9.7 ≈ t

The population will be 7 million about 9.7 yr after2010.

d) T =ln 2

0.0176≈ 39.4 yr

3. a) k =ln 270.7

≈ 0.98%

b) k =ln 245.9

≈ 1.51%

c) T =ln 2

0.0321≈ 21.6 yr

d) T =ln 2

0.012≈ 57.8 yr

e) k =ln 2248

≈ 0.28%

f) T =ln 2

0.0232≈ 29.9 per yr

g) T =ln 2

0.014≈ 49.5 yr

h) k =ln 2

105.0≈ 0.66%

i) k =ln 234.0

≈ 2.04%

j) T =ln 2

0.0184≈ 37.7 yr

Copyright © 2013 Pearson Education, Inc.

Page 208: JUDITH A. PENNA -  · PDF fileSTUDENT’S SOLUTIONS MANUAL JUDITH A. PENNA Indiana University Purdue University Indianapolis COLLEGE ALGEBRA: GRAPHS AND MODELS FIFTH EDITION

Exercise Set 5.6 205

5. First note that 32,961,561,600 square yards = 32,961,561.6thousand square yards.

P (t) = P0ekt

32, 961, 561.6 = 10, 032, 619e0.00787t

32, 961, 561.610, 032, 619

= e0.00787t

ln(

32, 961, 561.610, 032, 619

)= ln e0.00787t

ln(

32, 961, 561.610, 032, 619

)= 0.00787t

ln(

32, 961, 561.610, 032, 619

)0.00787

= t

151 ≈ t

There will be one person for every 1000 square yards ofland about 151 yr after 2010.

7. a) Substitute 10,000 for P0 and 5.4%, or 0.054 for k.

P (t) = 10, 000e0.054t

b) P (1) = 10, 000e0.054(1) ≈ $10, 554.85

P (2) = 10, 000e0.054(2) ≈ $11, 140.48

P (5) = 10, 000e0.054(5) ≈ $13, 099.64

P (10) = 10, 000e0.054(10) ≈ $17, 160.07

c) T =ln 2

0.054≈ 12.8 yr

9. We use the function found in Example 5. If the bones havelost 77.2% of their carbon-14 from an initial amount P0,then 22.8%P0, or 0.228P0 remains. We substitute in thefunction.

0.228P0 = P0e−0.00012t

0.228 = e−0.00012t

ln 0.228 = ln e−0.00012t

ln 0.228 = −0.00012tln 0.228−0.00012

= t

12, 320 ≈ t

The bones are about 12,320 years old.

11. a) K =ln 23.1

≈ 0.224, or 22.4% per min

b) k =ln 222.3

≈ 0.031, or 3.1% per yr

c) T =ln 2

0.0115≈ 60.3 days

d) T =ln 2

0.065≈ 10.7 yr

e) k =ln 229.1

≈ 0.024, or 2.4% per yr

f) k =ln 270.0

≈ 0.010, or 1.0% per yr

g) k =ln 2

24, 100≈ 0.000029, or 0.0029% per yr

13. a) C(t) = C0e−kt

2.92 = 4.85e−k·4

2.924.85

= e−4k

ln(

2.924.85

)= ln e−4k

ln(

2.924.85

)= −4k

ln(

2.924.85

)−4

= k

0.1268 ≈ k

Then we have C(t) = 4.85e−0.1268t, where C is indollars and t is the number of years since 2009.

b) In 2015, t = 2015 − 2009 = 6.

C(6) = 4.85e−0.1268(6) ≈ $2.27

In 2018, t = 2018 − 2009 = 9.

C(9) = 4.85e−0.1268(9) ≈ $1.55c) 1.85 = 4.85e−0.1268t

1.854.85

= e−0.1268t

ln(

1.854.85

)= ln e−0.1268t

ln(

1.854.85

)= −0.1268t

ln(

1.854.85

)−0.1268

= t

8 ≈ t

At the given rate of decay, the average cost per wattwill be $1.85 about 8 yr after 2009, or in 2017.

15. a) In 2010, t = 2010 − 1980 = 30.C(t) = C0e

kt

565 = 28ek·30

56528

= e30k

ln(

56528

)= ln e30k

ln(

56528

)= 30k

ln(

56528

)30

= k

0.1002 ≈ k

C(t) = 28e0.1002t, where t is the number of yearsafter 1980.

b) In 2013, t = 2013 − 1980 = 33.

C(33) = 28e0.1002(33) ≈ 764 cable TV networks

In 2017, t = 2017 − 1980 = 37.

C(37) = 28e0.1002(37) ≈ 1141 cable TV networks

Copyright © 2013 Pearson Education, Inc.

Page 209: JUDITH A. PENNA -  · PDF fileSTUDENT’S SOLUTIONS MANUAL JUDITH A. PENNA Indiana University Purdue University Indianapolis COLLEGE ALGEBRA: GRAPHS AND MODELS FIFTH EDITION

206 Chapter 5: Exponential and Logarithmic Functions

c) 1500 = 28e0.1002t

150028

= e0.1002t

ln(

150028

)= ln e0.1002t

ln(

150028

)= 0.1002t

ln(

150028

)0.1002

= t

40 ≈ t

There will be 1500 cable TV networks about 40 yrafter 1980, or in 2020.

17. a)

b) N(0) =3500

1 + 19.9e−0.6(0)≈ 167

c) N(2) =3500

1 + 19.9e−0.6(2)≈ 500

N(5) =3500

1 + 19.9e−0.6(5)≈ 1758

N(8) =3500

1 + 19.9e−0.6(8)≈ 3007

N(12) =3500

1 + 19.9e−0.6(12)≈ 3449

N(16) =3500

1 + 19.9e−0.6(16)≈ 3495

d) As t → ∞, N(t) → 3500; the number of peopleinfected approaches 3500 but never actually reachesit.

19. To find k we substitute 105 for T1, 0 for T0, 5 for t, and70 for T (t) and solve for k.

70 = 0 + (105 − 0)e−5k

70 = 105e−5k

70105

= e−5k

ln70105

= ln e−5k

ln70105

= −5k

ln70105−5

= k

0.081 ≈ k

The function is T (t) = 105e−0.081t.

Now we find T (10).

T (10) = 105e−0.081(10) ≈ 46.7 ◦F

21. To find k we substitute 43 for T1, 68 for T0, 12 for t, and55 for T (t) and solve for k.

55 = 68 + (43 − 68)e−12k

−13 = −25e−12k

0.52 = e−12k

ln 0.52 = ln e−12k

ln 0.52 = −12k

0.0545 ≈ k

The function is T (t) = 68 − 25e−0.0545t.

Now we find T (20).

T (20) = 68 − 25e−0.0545(20) ≈ 59.6◦F

23. The data have the pattern of a decreasing exponentialfunction, so function (d) might be used as a model.

25. The data have a parabolic pattern, so function (a) mightbe used as a model.

27. The data have a logarithmic pattern, so function (e) mightbe used as a model.

29. a) y = 0.1377082721(1.023820625)x, where x is thenumber of years after 1900.

r ≈ 0.9824, so the function is a good fit.

b)

c) In 2007, x = 2007 − 1900 = 107.

y = 0.1377082721(1.023820625)107 ≈ 1.7%

In 2015, x = 2015 − 1900 = 115.

y = 0.1377082721(1.023820625)115 ≈ 2.1%

In 2020, x = 2020 − 1900 = 120.

y = 0.1377082721(1.023820625)120 ≈ 2.3%

31. a) y = 5.600477432(1.030576807)x, where y is in mil-lions and x is the number of years after 1980.

b)

c) In 2020, x = 2020 − 1980 = 40.

y = 5.600477432(1.030576807)40 ≈ 18.7 million un-occupied homes

33. a) y = 2079.930585(1.079796876)x, where x is thenumber of years after 2000.

b) In 2012, x = 2012 − 2000 = 12.

y = 2079.930585(1.079796876)12 ≈ $5226

In 2015, x = 2015 − 2000 = 15.

y = 2079.930585(1.079796876)15 ≈ $6579

Copyright © 2013 Pearson Education, Inc.

Page 210: JUDITH A. PENNA -  · PDF fileSTUDENT’S SOLUTIONS MANUAL JUDITH A. PENNA Indiana University Purdue University Indianapolis COLLEGE ALGEBRA: GRAPHS AND MODELS FIFTH EDITION

x

y

1—1—3 32 54—2—4—5—1

—2

—3

—4

1

2

3

4

5

—5

f (x ) = – |x | + 3

x

y

1—1—3 32 54—2—4—5—1

—2

—3

—4

1

2

3

4

5

—5

f (x ) = 2x – –34

x

y

1—1—3 32 54—2—4—5—1

—2

—3

—4

1

2

3

4

5

—5

f (x ) = 2 – 3x

Chapter 5 Review Exercises 207

c) Graph y1 = 2079.930585(1.079796876)x and y2 =7300, and find the first coordinate of the point ofintersection of the graphs. We find that the priceper acre will exceed $7300 about 16 yr after 2000,or in 2016.

35. Multiplication principle for inequalities

37. Principle of zero products

39. Power rule

41. P (t) = P0ekt

50, 000 = P0e0.07(18)

50, 000e0.07(18)

= P0

$14, 182.70 ≈ P0

43. a) At 1 m: I = I0e−1.4(1) ≈ 0.247I0

24.7% of I0 remains.

At 3 m: I = I0e−1.4(3) ≈ 0.015I0

1.5% of I0 remains.

At 5 m: I = I0e−1.4(5) ≈ 0.0009I0

0.09% of I0 remains.

At 50 m: I = I0e−1.4(50) ≈ (3.98 × 10−31)I0

Now, 3.98 × 10−31 = (3.98 × 10−29) × 10−2, so

(3.98 × 10−29)% remains.

b) I = I0e−1.4(10) ≈ 0.0000008I0

Thus, 0.00008% remains.

45. y = aex

ln y = ln(aex)

ln y = ln a + ln ex

ln y = ln a + x

Y = x + ln a

This function is of the form y = mx + b, so it is linear.

Chapter 5 Review Exercises

1. The statement is true. See page 395 in the text.

3. The graph of f−1 is a reflection of the graph of f acrossy = x, so the statement is false.

5. The statement is false. The range of y = ax, for instance,is (0,∞).

7. We interchange the first and second coordinates of eachpair to find the inverse of the relation. It is{(−2.7, 1.3), (−3, 8), (3,−5), (−3, 6), (−5, 7)}.

9. The graph of f(x) = −|x|+3 is shown below. The functionis not one-to-one, because there are many horizontal linesthat cross the graph more than once.

11. The graph of f(x) = 2x− 34

is shown below. The functionis one-to-one, because no horizontal line crosses the graphmore than once.

13. a) The graph of f(x) = 2−3x is shown below. It passesthe horizontal-line test, so the function is one-to-one.

b) Replace f(x) with y: y = 2 − 3x

Interchange x and y: x = 2 − 3y

Solve for y: y =−x + 2

3

Replace y with f−1(x): f−1(x) =−x + 2

3

Copyright © 2013 Pearson Education, Inc.

Page 211: JUDITH A. PENNA -  · PDF fileSTUDENT’S SOLUTIONS MANUAL JUDITH A. PENNA Indiana University Purdue University Indianapolis COLLEGE ALGEBRA: GRAPHS AND MODELS FIFTH EDITION

x

y

2—2—6 64 108—4—8—10

—2

—4

—6

—8

2

4

6

8

10

—10

f (x ) = √ x – 6

x

y

1—1—3 32 54—2—4—5—1

—2

—3

—4

1

2

3

4

5

—5

f (x ) = 3x 2 + 2x – 1

y � xf �1

f

2 4�4 �2

2

4

�4

�2

x

y

2 4�4 �2

2

4

�4

�2x

y

f(x) � (�)x13

2 4�4 �2

2

4

�4

�2x

y

f(x) � �e�x

2 4�4 �2

2

4

�4

�2x

y

f(x) � � ln x12

208 Chapter 5: Exponential and Logarithmic Functions

15. a) The graph of f(x) =√x− 6 is shown below. It

passes the horizontal-line test, so the function is one-to-one.

b) Replace f(x) with y: y =√x− 6

Interchange x and y: x =√y − 6

Solve for y: x2 = y − 6

x2 + 6 = y

Replace y with f−1(x): f−1(x) = x2 + 6, for all xin the range of f(x), or f−1(x) = x2 + 6, x ≥ 0.

17. a) The graph of f(x) = 3x2 + 2x − 1 is shown below.It is not one-to-one since there are many horizon-tal lines that cross the graph more than once. Thefunction does not have an inverse that is a function.

19. We find (f−1◦f)(x) and (f ◦f−1)(x) and check to see thateach is x.(f−1 ◦ f)(x) = f−1(f(x)) = f−1(6x− 5) =

(6x− 5) + 56

=6x6

= x

(f ◦ f−1)(x) = f(f−1(x)) = f

(x + 5

6

)=

6(x + 5

6

)− 5 = x + 5 − 5 = x

21. Replace f(x) with y: y = 2 − 5x

Interchange x and y: x = 2 − 5y

Solve for y: y =2 − x

5

Replace y with f−1(x): f−1(x) =2 − x

5The domain and range of f are (−∞,∞), so the domainand range of f−1 are also (−∞,∞).

23. Since f(f−1(x)) = x, then f(f−1(657)) = 657.

25.

27.

29.

31. f(x) = ex−3

This is the graph of f(x) = ex shifted right 3 units. Thecorrect choice is (c).

33. f(x) = − log3(x + 1)

This is the graph of log3 x shifted left 1 unit and reflectedacross the y-axis. The correct choice is (b).

35. f(x) = 3(1 − e−x), x ≥ 0

This is the graph of f(x) = ex reflected across the y-axis,reflected across the x-axis, shifted up 1 unit, and stretchedby a factor of 3. The correct choice is (e).

37. log5 125 = 3 because the exponent to which we raise 5 toget 125 is 3.

39. ln e = 1 because the exponent to which we raise e to get eis 1.

41. log 101/4 =14

because the exponent to which we raise 10

to get 101/4 is14.

Copyright © 2013 Pearson Education, Inc.

Page 212: JUDITH A. PENNA -  · PDF fileSTUDENT’S SOLUTIONS MANUAL JUDITH A. PENNA Indiana University Purdue University Indianapolis COLLEGE ALGEBRA: GRAPHS AND MODELS FIFTH EDITION

Chapter 5 Review Exercises 209

43. log 1 = 0 because the exponent to which we raise 10 to get1 is 0.

45. log23√

2 = log2 21/3 =13

because the exponent to which we

raise 2 to get 21/3 is13.

47. log4 x = 2 ⇒ 42 = x

49. 4−3 =164

⇒ log4

164

= −3

51. log 11 ≈ 1.0414

53. ln 3 ≈ 1.0986

55. log(−3) does not exist. (The calculator gives an error mes-sage.)

57. log5 24 =log 24log 5

≈ 1.9746

59. 3 logb x− 4 logb y +12

logb z

= logb x3 − logb y4 + logb z1/2

= logbx3z1/2

y4, or logb

x3√z

y4

61. ln 4√wr2 = ln(wr2)1/4

=14

lnwr2

=14(lnw + ln r2)

=14(lnw + 2 ln r)

=14

lnw +12

ln r

63. loga 3 = loga

(62

)= loga 6 − loga 2

≈ 0.778 − 0.301

≈ 0.477

65. loga15

= loga 5−1

= − loga 5

≈ −0.699

67. ln e−5k = −5k (loga ax = x)

69. log4 x = 2

x = 42 = 16

The solution is 16.

71. ex = 80

ln ex = ln 80

x = ln 80

x ≈ 4.382

The solution is 4.382.

73. log16 4 = x

16x = 4

(42)x = 41

42x = 41

2x = 1

x =12

The solution is12.

75. log2 x + log2(x− 2) = 3

log2 x(x− 2) = 3

x(x− 2) = 23

x2 − 2x = 8

x2 − 2x− 8 = 0

(x + 2)(x− 4) = 0

x + 2 = 0 or x− 4 = 0

x = −2 or x = 4

The number 4 checks, but −2 does not. The solution is 4.

77. log x2 = log x

x2 = x

x2 − x = 0

x(x− 1) = 0

x = 0 or x− 1 = 0

x = 0 or x = 1

The number 1 checks, but 0 does not. The solution is 1.

79. a) A(t) = P

(1 +

r

n

)nt

A(t) = 30, 000(

1 +0.042

4

)4t

Substituting

A(t) = 30, 000(1.0105)4t

b) A(0) = 30, 000(1.0105)4·0 = $30, 000

A(6) = 30, 000(1.0105)4·6 ≈ $38, 547.20

A(12) = 30, 000(1.0105)4·12 ≈ $49, 529.56

A(18) = 30, 000(1.0105)4·18 ≈ $63, 640.87

81. T =ln 2

0.086≈ 8.1 years

83. P (t) = P0ekt

0.73P0 = P0e−0.00012t

0.73 = e−0.00012t

ln 0.73 = ln e−0.00012t

ln 0.73 = −0.00012tln 0.73

−0.00012= t

2623 ≈ t

The skeleton is about 2623 years old.

Copyright © 2013 Pearson Education, Inc.

Page 213: JUDITH A. PENNA -  · PDF fileSTUDENT’S SOLUTIONS MANUAL JUDITH A. PENNA Indiana University Purdue University Indianapolis COLLEGE ALGEBRA: GRAPHS AND MODELS FIFTH EDITION

x

y

1—1—3 32 54—2—4—5—1

—2

—3

—4

1

2

3

4

5

—5

f (x ) = x 3 + 1

210 Chapter 5: Exponential and Logarithmic Functions

85. R = log106.3 · I0

I0= log 106.3 = 6.3

87. a) We substitute 353.823 for P , since P is in thousands.

W (353.823) = 0.37 ln 353.823 + 0.05

≈ 2.2 ft/sec

b) We substitute 3.4 for W and solve for P .3.4 = 3.7 lnP + 0.05

3.35 = 0.37 lnP

3.350.37

= lnP

e3.35/0.37 = P

P ≈ 8553.143

The population is about 8553.143 thousand, or8,553,143. (Answers may vary due to rounding dif-ferences.)

89. a) P (t) = 6.188e0.01985t, where P (t) is in millions andt is the number of years after 2011.

b) In 2013, t = 2013 − 2011 = 2.

P (2) = 6.188e0.01985(2) ≈ 6.439 million

In 2015, t = 2015 − 2011 = 4.

P (4) = 6.188e0.01985(4) ≈ 6.699 million

c) 10 = 6.188e0.01985t

106.188

= e0.01985t

ln(

106.188

)= ln e0.01985t

ln(

106.188

)= 0.01985t

ln(

106.188

)0.01985

= t

24 ≈ t

The population will be 10 million about 24 yr after2011.

d) T =ln 2

0.01985≈ 34.9 yr

91. We graph y1 =4 + 3xx− 2

, y2 =x + 4x− 3

, and y3 = x and ob-

serve that the graphs of y1 and y2 are not reflections ofeach other across the third line, y = x. Thus f(x) andg(x) are not inverses.

93. The graph of f(x) = ex−3 +2 is a translation of the graphof y = ex right three units and up 2 units. The horizontalasymptote of y = ex is y = 0, so the horizontal asymptoteof f(x) = ex−3 + 2 is translated up 2 units from y = 0.Thus, it is y = 2, and answer D is correct.

95. The graph of f(x) = 2x−2 is the graph of g(x) = 2x shifted2 units to the right. Thus D is the correct graph.

97. | log4 x| = 3

log4 x = −3 or log4 x = 3

x = 4−3 or x = 43

x =164

or x = 64

Both answers check. The solutions are164

and 64.

99. 5√x = 625

5√x = 54

√x = 4

x = 16

101. Reflect the graph of f(x) = ln x across the line y = x toobtain the graph of h(x) = ex. Then shift this graph 2units right to obtain the graph of g(x) = ex−2.

103. loga ab3 �= (loga a)(loga b3). If the first step had been cor-rect, then so would the second step. The correct procedurefollows.

loga ab3 = loga a + loga b3 = 1 + 3 loga b

Chapter 5 Test

1. We interchange the first and second coordinates of eachpair to find the inverse of the relation. It is{(5,−2), (3, 4)(−1, 0), (−3,−6)}.

2. The function is not one-to-one, because there are manyhorizontal lines that cross the graph more than once.

3. The function is one-to-one, because no horizontal linecrosses the graph more than once.

4. a) The graph of f(x) = x3+1 is shown below. It passesthe horizontal-line test, so the function is one-to-one.

b) Replace f(x) with y: y = x3 + 1

Interchange x and y: x = y3 + 1

Solve for y: y3 = x− 1

y = 3√x− 1

Replace y with f−1(x): f−1(x) = 3√x− 1

Copyright © 2013 Pearson Education, Inc.

Page 214: JUDITH A. PENNA -  · PDF fileSTUDENT’S SOLUTIONS MANUAL JUDITH A. PENNA Indiana University Purdue University Indianapolis COLLEGE ALGEBRA: GRAPHS AND MODELS FIFTH EDITION

x

y

1—1—3 32 54—2—4—5—1

—2

—3

—4

1

2

3

4

5

—5

f (x ) = 1 – x

x

y

1—1—3 32 54—2—4—5—1

—2

—3

—4

1

2

3

4

5

—5

f (x ) = –––x2 – x

x

y

1—1—3 32 54—2—4—5—1

—2

—3

—4

1

2

3

4

5

—5f (x ) = x 2 + x – 3

f �1

f

2 4 6�4 �2

2

4

6

�4

�2

x

y y � x

2 4�4 �2

2

4

�4

�2x

y

f (x) � 4�x

Chapter 5 Test 211

5. a) The graph of f(x) = 1−x is shown below. It passesthe horizontal-line test, so the function is one-to-one.

b) Replace f(x) with y: y = 1 − x

Interchange x and y: x = 1 − y

Solve for y: y = 1 − x

Replace y with f−1(x): f−1(x) = 1 − x

6. a) The graph of f(x) =x

2 − xis shown below. It passes

the horizontal-line test, so the function is one-to-one.

b) Replace f(x) with y: y =x

2 − x

Interchange x and y: x =y

2 − y

Solve for y: (2 − y)x = y

2x− xy = y

xy + y = 2x

y(x + 1) = 2x

y =2x

x + 1

Replace y with f−1(x): f−1(x) =2x

x + 1

7. a) The graph of f(x) = x2 +x−3 is shown below. It isnot one-to-one since there are many horizontal linesthat cross the graph more than once. The functiondoes not have an inverse that is a function.

8. We find (f−1◦f)(x) and (f ◦f−1)(x) and check to see thateach is x.

(f−1 ◦ f)(x) = f−1(f(x)) = f−1(−4x + 3) =3 − (−4x + 3)

4=

3 + 4x− 34

=4x4

= x

(f ◦ f−1)(x) = f(f−1(x)) = f

(3 − x

4

)=

−4(

3 − x

4

)+ 3 = −3 + x + 3 = x

9. Replace f(x) with y: y =1

x− 4

Interchange x and y: x =1

y − 4

Solve for y: x(y − 4) = 1

xy − 4x = 1

xy = 4x + 1

y =4x + 1

x

Replace y with f−1(x): f−1(x) =4x + 1

x

The domain of f(x) is (−∞, 4) ∪ (4,∞) and the rangeof f(x) is (−∞, 0) ∪ (0,∞). Thus, the domain of f−1 is(−∞, 0)∪ (0,∞) and the range of f−1 is (−∞, 4)∪ (4,∞).

10.

Copyright © 2013 Pearson Education, Inc.

Page 215: JUDITH A. PENNA -  · PDF fileSTUDENT’S SOLUTIONS MANUAL JUDITH A. PENNA Indiana University Purdue University Indianapolis COLLEGE ALGEBRA: GRAPHS AND MODELS FIFTH EDITION

2 4�4 �2

2

4

�4

�2x

y

f(x) � log x

f (x) � e x � 3

y

x

2

4

�2

�4

�2�4 42

f (x) � ln(x � 2)

y

x

2

4

�2

�4

�2�4 42

212 Chapter 5: Exponential and Logarithmic Functions

11.

12.

13.

14. log 0.00001 = −5 because the exponent to which we raise10 to get 0.00001 is −5.

15. ln e = 1 because the exponent to which we raise e to get eis 1.

16. ln 1 = 0 because the exponent to which we raise e to get 1is 0.

17. log45√

4 = log4 41/5 =15

because the exponent to which we

raise 4 to get 41/5 is15.

18. lnx = 4 ⇒ x = e4

19. 3x = 5.4 ⇒ x = log3 5.4

20. ln 16 ≈ 2.7726

21. log 0.293 ≈ −0.5331

22. log6 10 =log 10log 6

≈ 1.2851

23. 2 loga x− loga y +12

loga z

= loga x2 − loga y + loga z1/2

= logax2z1/2

y, or loga

x2√z

y

24. ln 5√

x2y = ln(x2y)1/5

=15

lnx2y

=15(lnx2 + ln y)

=15(2 lnx + ln y)

=25

lnx +15

ln y

25. loga 4 = loga

(82

)= loga 8 − loga 2

≈ 0.984 − 0.328

≈ 0.656

26. ln e−4t = −4t (loga ax = x)

27. log25 5 = x

25x = 5

(52)x = 51

52x = 51

2x = 1

x =12

The solution is12.

28. log3 x + log3(x + 8) = 2

log3 x(x + 8) = 2

x(x + 8) = 32

x2 + 8x = 9

x2 + 8x− 9 = 0

(x + 9)(x− 1) = 0x = −9 or x = 1

The number 1 checks, but −9 does not. The solution is 1.

29. 34−x = 27x

34−x = (33)x

34−x = 33x

4 − x = 3x

4 = 4x

x = 1

The solution is 1.

30. ex = 65

ln ex = ln 65

x = ln 65

x ≈ 4.174

The solution is 4.174.

31. R = log106.6 · I0

I0= log 106.6 = 6.6

Copyright © 2013 Pearson Education, Inc.

Page 216: JUDITH A. PENNA -  · PDF fileSTUDENT’S SOLUTIONS MANUAL JUDITH A. PENNA Indiana University Purdue University Indianapolis COLLEGE ALGEBRA: GRAPHS AND MODELS FIFTH EDITION

Chapter 5 Test 213

32. k =ln 245

≈ 0.0154 ≈ 1.54%

33. a) 1144.54 = 1000e3k

1.14454 = e3k

ln 1.14454 = ln e3k

ln 1.14454 = 3kln 1.14454

3= k

0.045 ≈ k

The interest rate is about 4.5%.

b) P (t) = 1000e0.045t

c) P (8) = 1000e0.045·8 ≈ $1433.33

d) T =ln 2

0.045≈ 15.4 yr

34. The graph of f(x) = 2x−1 + 1 is the graph of g(x) = 2x

shifted right 1 unit and up 1 unit. Thus C is the correctgraph.

35. 43√x = 8

(22)3√x = 23

22 3√x = 23

2 3√x = 3

3√x =

32

x =(

32

)3

x =278

The solution is278

.

Copyright © 2013 Pearson Education, Inc.

Page 217: JUDITH A. PENNA -  · PDF fileSTUDENT’S SOLUTIONS MANUAL JUDITH A. PENNA Indiana University Purdue University Indianapolis COLLEGE ALGEBRA: GRAPHS AND MODELS FIFTH EDITION

Copyright © 2013 Pearson Education, Inc.

Page 218: JUDITH A. PENNA -  · PDF fileSTUDENT’S SOLUTIONS MANUAL JUDITH A. PENNA Indiana University Purdue University Indianapolis COLLEGE ALGEBRA: GRAPHS AND MODELS FIFTH EDITION

x

y

1–1–3 32 54–2–4–5–1

–2

–3

–4

1

2

3

4

5

–5

x + y = 2

3x + y = 0

(-1, 3)

x

y

1–1–3 32 54–2–4–5–1

–2

–3

–4

1

2

3

4

5

–5

x + 4y = 3

x + 2y = 1

(-1, 1)

x

y

1–1–3 32 54–2–4–5–1

–2

–3

–4

1

2

3

4

5

–5

y + 1 = 2x

y — 1 = 2x

x

y

1–1–3 32 54–2–4–5–1

–2

–3

–4

1

2

3

4

5

–5

x — y = — 6

y = —2x

(-2, 4)

x

y

1–1–3 32 54–2–4–5–1

–2

–3

–4

1

2

3

4

5

–5

2y = x — 1

3x = 6y + 3

Chapter 6

Systems of Equations and Matrices

Exercise Set 6.1

1. Graph (c) is the graph of this system

3. Graph (f) is the graph of this system.

5. Graph (b) is the graph of this system.

7. Graph x + y = 2 and 3x + y = 0 and find the coordinatesof the point of intersection.

The solution is (−1, 3).

9. Graph x+2y = 1 and x+4y = 3 and find the coordinatesof the point of intersection.

The solution is (−1, 1).

11. Graph y+1 = 2x and y− 1 = 2x and find the coordinatesof the point of intersection.

The graphs do not intersect, so there is no solution.

13. Graph x− y = −6 and y = −2x and find the coordinatesof the point of intersection.

The solution is (−2, 4).

15. Graph 2y = x−1 and 3x = 6y+3 and find the coordinatesof the point of intersection.

The graphs coincide so there are infinitely many solutions.

Solving either equation for y, we get y =x− 1

2, so the so-

lutions can be expressed as(x,

x− 12

). Similarly, solving

either equation for x, we get x = 2y + 1, so the solutionscan also be expressed as (2y + 1, y).

17. x + y = 9, (1)

2x − 3y = −2 (2)

Solve equation (1) for either x or y. We choose to solve fory.

y = 9 − x

Then substitute 9 − x for y in equation (2) and solve theresulting equation.2x− 3(9 − x) = −2

2x− 27 + 3x = −2

5x− 27 = −2

5x = 25

x = 5

Now substitute 5 for x in either equation (1) or (2) andsolve for y.

Copyright © 2013 Pearson Education, Inc.

Page 219: JUDITH A. PENNA -  · PDF fileSTUDENT’S SOLUTIONS MANUAL JUDITH A. PENNA Indiana University Purdue University Indianapolis COLLEGE ALGEBRA: GRAPHS AND MODELS FIFTH EDITION

216 Chapter 6: Systems of Equations and Matrices

5 + y = 9 Using equation (1).

y = 4

The solution is (5, 4).

19. x− 2y = 7, (1)

x = y + 4 (2)

Use equation (2) and substitute y+4 for x in equation (1).Then solve for y.y + 4 − 2y = 7

−y + 4 = 7

−y = 3

y = −3

Substitute −3 for y in equation (2) to find x.

x = −3 + 4 = 1

The solution is (1,−3).

21. y = 2x− 6, (1)

5x− 3y = 16 (2)

Use equation (1) and substitute 2x − 6 for y in equation(2). Then solve for x.

5x− 3(2x− 6) = 16

5x− 6x + 18 = 16

−x + 18 = 16

−x = −2

x = 2

Substitute 2 for x in equation (1) to find y.

y = 2 · 2 − 6 = 4 − 6 = −2

The solution is (2,−2).

23. x + y = 3, (1)

y = 4 − x (2)

Use equation (2) and substitute 4−x for y in equation (1).

x + 4 − x = 3

4 = 3There are no values of x and y for which 4 = 3, so thesystem of equations has no solution.

25. x− 5y = 4, (1)

y = 7 − 2x (2)

Use equation (2) and substitute 7 − 2x for y in equation(1). Then solve for x.

x− 5(7 − 2x) = 4

x− 35 + 10x = 4

11x− 35 = 4

11x = 39

x =3911

Substitute3911

for x in equation (2) to find y.

y = 7 − 2 · 3911

= 7 − 7811

= − 111

The solution is(

3911

,− 111

).

27. x + 2y = 2, (1)

4x + 4y = 5 (2)Solve one equation for either x or y. We choose to solveequation (1) for x since x has a coefficient of 1 in thatequation.

x + 2y = 2

x = −2y + 2

Substitute −2y + 2 for x in equation (2) and solve for y.

4(−2y + 2) + 4y = 5

−8y + 8 + 4y = 5

−4y + 8 = 5

−4y = −3

y =34

Substitute34

for y in either equation (1) or equation (2)and solve for x.

x + 2y = 2 Using equation (1)

x + 2 · 34

= 2

x +32

= 2

x =12

The solution is(

12,34

).

29. 3x− y = 5, (1)

3y = 9x− 15 (2)

Solve one equation for x or y. We will solve equation (1)for y.

3x− y = 5

3x = y + 5

3x− 5 = y

Substitute 3x− 5 for y in equation (2) and solve for x.

3(3x− 5) = 9x− 15

9x− 15 = 9x− 15

−15 = −15The equation −15 = −15 is true for all values of x and y,so the system of equations has infinitely many solutions.We know that y = 3x− 5, so we can write the solutions inthe form (x, 3x− 5).

If we solve either equation for x, we get x =13y +

53, so we

can also write the solutions in the form(

13y +

53, y

).

31. x + 2y = 7, (1)

x − 2y = −5 (2)We add the equations to eliminate y.

x + 2y = 7

x − 2y = −52x = 2 Adding

x = 1

Copyright © 2013 Pearson Education, Inc.

Page 220: JUDITH A. PENNA -  · PDF fileSTUDENT’S SOLUTIONS MANUAL JUDITH A. PENNA Indiana University Purdue University Indianapolis COLLEGE ALGEBRA: GRAPHS AND MODELS FIFTH EDITION

Exercise Set 6.1 217

Back-substitute in either equation and solve for y.

1 + 2y = 7 Using equation (1)

2y = 6

y = 3

The solution is (1, 3). Since the system of equations hasexactly one solution it is consistent and the equations areindependent.

33. x − 3y = 2, (1)

6x + 5y = −34 (2)

Multiply equation (1) by −6 and add it to equation (2) toeliminate x.−6x + 18y = −12

6x + 5y = −3423y = −46

y = −2

Back-substitute to find x.x− 3(−2) = 2 Using equation (1)

x + 6 = 2

x = −4

The solution is (−4,−2). Since the system of equationshas exactly one solution it is consistent and the equationsare independent.

35. 3x − 12y = 6, (1)

2x − 8y = 4 (2)

Multiply equation (1) by 2 and equation (2) by −3 andadd.

6x − 24y = 12

−6x + 24y = −120 = 0

The equation 0 = 0 is true for all values of x and y. Thus,the system of equations has infinitely many solutions.

Solving either equation for y, we can write y =14x− 1

2so

the solutions are ordered pairs of the form(x,

14x− 1

2

).

Equivalently, if we solve either equation for x we getx = 4y + 2 so the solutions can also be expressed as(4y + 2, y). Since there are infinitely many solutions, thesystem of equations is consistent and the equations aredependent.

37. 4x − 2y = 3, (1)

2x − y = 4 (2)

Multiply equation (2) by −2 and add.

4x − 2y = 3

−4x + 2y = −80 = −5

We get a false equation so there is no solution. Since thereis no solution the system of equations is inconsistent andthe equations are independent.

39. 2x = 5 − 3y, (1)

4x = 11 − 7y (2)

We rewrite the equations.

2x + 3y = 5, (1a)

4x + 7y = 11 (2a)

Multiply equation (2a) by −2 and add to eliminate x.

−4x − 6y = −10

4x + 7y = 11y = 1

Back-substitute to find x.2x = 5 − 3 · 1 Using equation (1)

2x = 2

x = 1

The solution is (1, 1). Since the system of equations hasexactly one solution it is consistent and the equations areindependent.

41. 0.3x − 0.2y = −0.9,

0.2x − 0.3y = −0.6

First, multiply each equation by 10 to clear the decimals.

3x− 2y = −9 (1)

2x− 3y = −6 (2)

Now multiply equation (1) by 3 and equation (2) by −2and add to eliminate y.

9x − 6y = −27

−4x + 6y = 125x = −15

x = −3

Back-substitute to find y.

3(−3) − 2y = −9 Using equation (1)

−9 − 2y = −9

−2y = 0

y = 0

The solution is (−3, 0). Since the system of equations hasexactly one solution it is consistent and the equations areindependent.

43. 15x +

12y = 6, (1)

35x − 1

2y = 2 (2)

We could multiply both equations by 10 to clear fractions,but since the y-coefficients differ only by sign we will justadd to eliminate y.15x +

12y = 6

35x − 1

2y = 2

45x = 8

x = 10

Back-substitute to find y.

Copyright © 2013 Pearson Education, Inc.

Page 221: JUDITH A. PENNA -  · PDF fileSTUDENT’S SOLUTIONS MANUAL JUDITH A. PENNA Indiana University Purdue University Indianapolis COLLEGE ALGEBRA: GRAPHS AND MODELS FIFTH EDITION

218 Chapter 6: Systems of Equations and Matrices

15· 10 +

12y = 6 Using equation (1)

2 +12y = 6

12y = 4

y = 8

The solution is (10, 8). Since the system of equations hasexactly one solution it is consistent and the equations areindependent.

45. The statement is true. See page 485 of the text.

47. False; a consistent system of equations can have exactlyone solution or infinitely many solutions. See page 485 ofthe text.

49. True; a system of equations that has infinitely many so-lutions is consistent and dependent. See page 485 of thetext.

51. Familiarize. Let k = the number of knee replacements,in millions, and h = the number of hip replacements, inmillions, in 2030.

Translate. The total number of knee and hip replace-ments will be 4.072 million so we have one equation.

k + h = 4.072

There will be 2.982 million more knee replacements thanhip replacements, so we have a second equation.

k = h + 2.928

Carry out. We solve the system of equationsk + h = 4.072, (1)

k = h + 2.928. (2)

Substitute h + 2.928 for k in equation (1) and solve for h.

h + 2.928 + h = 4.072

2h + 2.928 = 4.072

2h = 1.144

h = 0.572

Back-substitute in equation (2) to find k.

k = 0.572 + 2.928 = 3.5

Check. 3.5 + 0.572 = 4.072 replacements and 3.5 is 2.928more than 0.572. The answer checks.

State. In 2030 there will be 3.5 million knee replacementsand 0.572 million, or 572,000, hip replacements.

53. Familiarize. Let x = the number of complaints aboutcell-phone providers and y = the number of complaintsabout cable/satellite TV providers.

Translate. The total number of complaints was 70,093 sowe have one equation.

x + y = 70, 093

Cable/satellite TV providers received 4861 fewer com-plaints than cell-phne providers so we have a second equa-tion.

x− 4861 = y

Carry out. We solve the system of equationsx + y = 70, 093, (1)

x− 4861 = y. (2)

Substitute x− 4861 for y in equation (1) and solve for x.

x + (x− 4861) = 70, 093

2x− 4861 = 70, 093

2x = 74, 954

x = 37, 477

Back-substitute in equation (2) to find y.

37, 477 − 4861 = y

32, 616 = y

Check. 37, 477 + 32, 616 = 70, 093 complaints and 32,616is 4861 less than 37,477. The answer checks.

State. There were 37,477 complaints about cell-phoneproviders and 32,616 complaints about cable/satellite TVproviders.

55. Familiarize. Let x = the number of Amish living in Wis-consin and y = the number living in Ohio.

Translate. The total number of Amish living in Wiscon-sin and Ohio is 73,950, so we have one equation:

x + y = 73, 950

The number of Amish living in Ohio is 12,510 more thanthree times the number living in Wisconsin, so we have asecond equation:

y = 3x + 12, 510

Carry out. We solve the system of equationsx + y = 73, 950, (1)

y = 3x + 12, 510. (2)

Substitute 3x + 12, 510 for y in equation (1) and solve forx.

x + (3x + 12, 510) = 73, 950

4x + 12, 510 = 73, 950

4x = 61, 440

x = 15, 360

Back-substitute in equation (2) to find y.

y = 3(15, 360) + 12, 510 = 46, 080 + 12, 510 = 58, 590

Check. 15, 360 + 58, 590 = 73, 950 and 58,590 is 12,510more than 3 times 15,360. The answer checks.

State. There are 15,360 Amish living in Wisconsin, and58,590 Amish live in Ohio.

57. Familiarize. Let p = the number of calories in a pieceof pecan pie and l = the number of calories in a piece oflemon meringue pie.

Translate. The number of calories in a piece of pecan pieis 221 less than twice the number of calories in a piece oflemon meringue pie, so we have one equation:

p = 2l − 221

The total number of calories in a piece of each kind of pieis 865, so we have a second equation:

p + l = 865

Copyright © 2013 Pearson Education, Inc.

Page 222: JUDITH A. PENNA -  · PDF fileSTUDENT’S SOLUTIONS MANUAL JUDITH A. PENNA Indiana University Purdue University Indianapolis COLLEGE ALGEBRA: GRAPHS AND MODELS FIFTH EDITION

Exercise Set 6.1 219

Carry out. We solve the system of equations:

p = 2l − 221, (1)

p + l = 865. (2)

Substitute 2l − 221 for p in equation (2) and solve for l.

(2l − 221) + l = 865

3l − 221 = 865

3l = 1086

l = 362

Back-substitute in equation (1) to find p.

p = 2 · 362 − 221 = 724 − 221 = 503

Check. 503 is 221 less than 2 times 362, and 503 + 362 =865. The answer checks.

State. A piece of pecan pie has 503 calories, and a pieceof lemon meringue pie has 362 calories.

59. Familiarize. Let x = the number of standard-deliverypackages and y = the number of express-delivery packages.

Translate. A total of 120 packages were shipped, so wehave one equation.

x + y = 120

Total shipping charges were $596, so we have a secondequation.

3.50x + 7.50y = 596

Carry out. We solve the system of equations:

x + y = 120,

3.50x + 7.50y = 596.

First we multiply both sides of the second equation by 10to clear the decimals. This gives us the system of equations

x + y = 120, (1)

35x + 75y = 5960. (2)

Now multiply equation (1) by −35 and add.

−35x − 35y = −4200

35x + 75y = 596040y = 1760

y = 44

Back-substitute to find x.x + 44 = 120 Using equation (1)

x = 76

Check. 76 + 44 = 120 packages. Total shipping chargesare $3.50(76) + $7.50(44) = $266 + $330 = $596. Theanswer checks.

State. The business shipped 76 standard-delivery pack-ages and 44 express-delivery packages.

61. Familiarize. Let x = the amount invested at 4% and y =the amount invested at 5%. Then the interest from theinvestments is 4%x and 5%y, or 0.04x and 0.05y.

Translate.

The total investment is $15,000.

x + y = 15, 000

The total interest is $690.

0.04x + 0.05y = 690

We have a system of equations:

x + y = 15, 000,

0.04x + 0.05y = 690

Multiplying the second equation by 100 to clear the deci-mals, we have:

x + y = 15, 000, (1)

4x + 5y = 69, 000. (2)

Carry out. We begin by multiplying equation (1) by −4and adding.

−4x − 4y = −60, 000

4x + 5y = 69, 000y = 9000

Back-substitute to find x.x + 9000 = 15, 000 Using equation (1)

x = 6000

Check. The total investment is $6000+$9000, or $15,000.The total interest is 0.04($6000) + 0.05($9000), or $240 +$450, or $690. The solution checks.

State. $6000 was invested at 4% and $9000 was investedat 5%.

63. Familiarize. Let x = the number of pounds of Frenchroast coffee used and y = the number of pounds of Kenyancoffee. We organize the information in a table.

Frenchroast

Kenyan Mixture

Amount x y 10 lbPriceperpound

$9.00 $7.50 $8.40

Totalcost

$9x $7.50y$8.40(10), or

$84

Translate. The first and third rows of the table give us asystem of equations.

x + y = 10,

9x + 7.5y = 84

Multiply the second equation by 10 to clear the decimals.

x + y = 10, (1)

90x + 75y = 840 (2)

Carry out. Begin by multiplying equation (1) by −75and adding.

−75x − 75y = −750

90x + 75y = 84015x = 90

x = 6

Back-substitute to find y.

6 + y = 10 Using equation (1)

y = 4

Copyright © 2013 Pearson Education, Inc.

Page 223: JUDITH A. PENNA -  · PDF fileSTUDENT’S SOLUTIONS MANUAL JUDITH A. PENNA Indiana University Purdue University Indianapolis COLLEGE ALGEBRA: GRAPHS AND MODELS FIFTH EDITION

220 Chapter 6: Systems of Equations and Matrices

Check. The total amount of coffee in the mixture is 6+4,or 10 lb. The total value of the mixture is 6($9)+4($7.50),or $54 + $30, or $84. The solution checks.

State. 6 lb of French roast coffee and 4 lb of Kenyan coffeeshould be used.

65. Familiarize. Let x = the number of servings of spaghettiand meatballs required and y = the number of servingsof iceberg lettuce required. Then x servings of spaghetticontain 260x Cal and 32x g of carbohydrates; y servingsof lettuce contain 5y Cal and 1 ·y or y, g of carbohydrates.

Translate. One equation comes from the fact that 400 Calare desired:

260x + 5y = 400.

A second equation comes from the fact that 50g of carbo-hydrates are required:

32x + y = 50.

Carry out. We solve the system

260x + 5y = 400, (1)

32x + y = 50. (2)

Multiply equation (2) by −5 and add.

260x + 5y = 400

−160x − 5y = −250100x = 150

x = 1.5

Back-substitute to find y.

32(1.5) + y = 50 Using equation (2)

48 + y = 50

y = 2

Check. 1.5 servings of spaghetti contain 260(1.5), or 390Cal and 32(1.5), or 48 g of carbohydrates; 2 servings oflettuce contain 5 · 2, or 10 Cal and 1 · 2, or 2 g of carbo-hydrates. Together they contain 390 + 10, or 400 Cal and48 + 2, or 50 g of carbohydrates. The solution checks.

State. 1.5 servings of spaghetti and meatballs and 2 serv-ings of iceberg lettuce are required.

67. Familiarize. It helps to make a drawing. Then organizethe information in a table. Let x = the speed of the boatand y = the speed of the stream. The speed upstream isx− y. The speed downstream is x + y.

46 km 2 hr (x + y) km/h� ✲Downstream

51 km 3 hr (x− y) km/h�✛

Upstream

Distance Speed Time

Downstream 46 x + y 2

Upstream 51 x− y 3

Translate. Using d = rt in each row of the table, we geta system of equations.

46 = (x + y)2 x + y = 23, (1)or

51 = (x− y)3 x− y = 17 (2)

Carry out. We begin by adding equations (1) and (2).

x + y = 23

x − y = 172x = 40

x = 20

Back-substitute to find y.

20 + y = 23 Using equation (1)

y = 3

Check. The speed downstream is 20+3, or 23 km/h. Thedistance traveled downstream in 2 hr is 23 · 2, or 46 km.The speed upstream is 20 − 3, or 17 km/h. The distancetraveled upstream in 3 hr is 17 · 3, or 51 km. The solutionchecks.

State. The speed of the boat is 20 km/h. The speed ofthe stream is 3 km/h.

69. Familiarize. Let d = the distance the slower plane trav-els, in km. Then 780 − d = the distance the faster planetravels. Let t = the number of hours each plane travels.We organize the information in a table.

Distance Speed Time

Slower plane d 190 t

Faster plane 780 − d 200 t

Translate. Using d = rt in each row of the table, we geta system of equations.

d = 190t, (1)

780 − d = 200t (2)

Carry out. We begin by adding the equations.

d = 190t

780 − d = 200t780 = 390t

2 = t

Check. In 2 hr, the slower plane travels 190 ·2, or 380 km,and the faster plane travels 200 · 2, or 400 km. The totaldistance traveled is 380 km + 400 km, or 780 km, so theanswer checks.

State. The planes will meet in 2 hr.

71. Familiarize and Translate. We use the system of equa-tions given in the problem.

y = 70 + 2x (1)

y = 175 − 5x, (2)

Carry out. Substitute 175− 5x for y in equation (1) andsolve for x.175 − 5x = 70 + 2x

105 = 7x Adding 5x and subtracting 70

15 = x

Back-substitute in either equation to find y. We chooseequation (1).

Copyright © 2013 Pearson Education, Inc.

Page 224: JUDITH A. PENNA -  · PDF fileSTUDENT’S SOLUTIONS MANUAL JUDITH A. PENNA Indiana University Purdue University Indianapolis COLLEGE ALGEBRA: GRAPHS AND MODELS FIFTH EDITION

Exercise Set 6.1 221

y = 70 + 2 · 15 = 70 + 30 = 100

Check. Substituting 15 for x and 100 for y in both of theoriginal equations yields true equations, so the solutionchecks.

State. The equilibrium point is (15, $100).

73. Familiarize and Translate. We find the value of x forwhich C = R, where

C = 14x + 350,

R = 16.5x.

Carry out. When C = R we have:

14x + 350 = 16.5x

350 = 2.5x

140 = x

Check. When x = 140, C = 14 · 140 + 350, or 2310 andR = 16.5(140), or 2310. Since C = R, the solution checks.

State. 140 units must be produced and sold in order tobreak even.

75. Familiarize and Translate. We find the value of x forwhich C = R, where

C = 15x + 12, 000,

R = 18x− 6000.

Carry out. When C = R we have:

15x + 12, 000 = 18x− 6000

18, 000 = 3x Subtracting 15x andadding 6000

6000 = x

Check. When x = 6000, C = 15·6000+12, 000, or 102,000and R = 18 · 6000 − 6000, or 102,000. Since C = R, thesolution checks.

State. 6000 units must be produced and sold in order tobreak even.

77. a) r(x) = 0.2308826185x + 44.61085102,

p(x) = 0.6288961625x + 26.81925282

b) Graph y1 = r(x) and y2 = p(x) and find the firstcoordinate of the point of intersection of the graphs.It is approximately 45, so we estimate that poultryconsumption will equal red meat consumption about45 yr after 1995.

79. Familiarize. Let t = the number of air travels in 2004,in millions. Then a decrease of 8.2% from this numberis t − 8.2%t, or t − 0.082t, or 0.918t. This represents thenumber of air travelers in 2009.

Translate.Number of air

travelers in 2009︸ ︷︷ ︸ is8.2% less thannumber in 2004︸ ︷︷ ︸� � �

769.6 = 0.918t

Carry out. We solve the equation.

769.6 = 0.918t

838.3 ≈ t

Check. A decrease of 8.2% from 838.3 million is0.918(838.3) = 769.5594 ≈ 769.6 million. The answerchecks.

State. There were about 838.3 million air travelers in2004.

81. Substituting 15 for f(x), we solve the following equation.

15 = x2 − 4x + 3

0 = x2 − 4x− 12

0 = (x− 6)(x + 2)

x− 6 = 0 or x + 2 = 0

x = 6 or x = −2

If the output is 15, the input is 6 or −2.

83. f(−9) = (−9)2 − 4(−9) + 3 = 120

If the input is −9, the output is 120.

85. Familiarize. Let x = the time spent jogging and y = thetime spent walking. Then Nancy jogs 8x km and walks4y km.

Translate.

The total time is 1 hr.

x + y = 1

The total distance is 6 km.

8x + 4y = 6

Carry out. Solve the system

x + y = 1, (1)

8x + 4y = 6. (2)

Multiply equation (1) by −4 and add.

−4x − 4y = −4

8x + 4y = 64x = 2

x =12

This is the time we need to find the distance spent jogging,so we could stop here. However, we will not be able tocheck the solution unless we find y also so we continue.We back-substitute.

12

+ y = 1 Using equation (1)

y =12

Then the distance jogged is 8 · 12, or 4 km, and the distance

walked is 4 · 12, or 2 km.

Check. The total time is12

hr +12

hr, or 1 hr. The totaldistance is 4 km + 2 km, or 6 km. The solution checks.

State. Nancy jogged 4 km on each trip.

87. Familiarize and Translate. We let x and y representthe speeds of the trains. Organize the information in atable. Using d = rt, we let 3x, 2y, 1.5x, and 3y representthe distances the trains travel.

Copyright © 2013 Pearson Education, Inc.

Page 225: JUDITH A. PENNA -  · PDF fileSTUDENT’S SOLUTIONS MANUAL JUDITH A. PENNA Indiana University Purdue University Indianapolis COLLEGE ALGEBRA: GRAPHS AND MODELS FIFTH EDITION

222 Chapter 6: Systems of Equations and Matrices

First situation:

3 hours x km/h y km/h 2 hours�

Union Central

216 km

Second situation:

1.5 hours x km/h y km/h 3 hours�

Union Central

216 km

Distance Distancetraveled traveledin first in second

situation situationTrain1 (fromUnion to Central)

3x 1.5x

Train2 (fromCentral to Union)

2y 3y

Total 216 216

The total distance in each situation is 216 km. Thus, wehave a system of equations.

3x + 2y = 216, (1)

1.5x + 3y = 216 (2)

Carry out. Multiply equation (2) by −2 and add.

3x + 2y = 216

−3x − 6y = −432−4y = −216

y = 54

Back-substitute to find x.3x + 2 · 54 = 216 Using equation (1)

3x + 108 = 216

3x = 108

x = 36

Check. If x = 36 and y = 54, the total distance the trainstravel in the first situation is 3 · 36 + 2 · 54, or 216 km.The total distance they travel in the second situation is1.5 · 36 + 3 · 54, or 216 km. The solution checks.

State. The speed of the first train is 36 km/h. The speedof the second train is 54 km/h.

89. Substitute the given solutions in the equationAx + By = 1 to get a system of equations.

3A − B = 1, (1)

−4A − 2B = 1 (2)

Multiply equation (1) by −2 and add.

−6A + 2B = −2

−4A − 2B = 1−10A = −1

A =110

Back-substitute to find B.

3(

110

)−B = 1 Using equation (1)

310

−B = 1

−B =710

B = − 710

We have A =110

and B = − 710

.

91. Familiarize. Let x and y represent the number of gallonsof gasoline used in city driving and in highway driving,respectively. Then 49x and 51y represent the number ofmiles driven in the city and on the highway, respectively.

Translate. The fact that 9 gal of gasoline were used givesus one equation:

x + y = 9.

A second equation comes from the fact that the car isdriven 447 mile:

49x + 51y = 447.

Carry out. We solve the system of equationsx + y = 9, (1)

49x + 51y = 447. (2)

Multiply equation (1) by −49 and add.

−49x − 49y = −441

49x + 51y = 4472y = 6

y = 3

Back-substitute to find x.x + 3 = 9

x = 6

Then in the city the car is driven 49(6), or 294 mi; on thehighway it is driven 51(3), or 153 mi.

Check. The number of gallons of gasoline used is 6 + 3,or 9. The number of miles driven is 294 + 153 = 447. Theanswer checks.

State. The car was driven 294 mi in the city and 153 mion the highway.

Exercise Set 6.2

1. x + y + z = 2, (1)

6x − 4y + 5z = 31, (2)

5x + 2y + 2z = 13 (3)

Multiply equation (1) by −6 and add it to equation (2).We also multiply equation (1) by −5 and add it to equation(3).

x + y + z = 2 (1)

− 10y − z = 19 (4)

− 3y − 3z = 3 (5)

Multiply the last equation by 10 to make the y-coefficienta multiple of the y-coefficient in equation (4).

Copyright © 2013 Pearson Education, Inc.

Page 226: JUDITH A. PENNA -  · PDF fileSTUDENT’S SOLUTIONS MANUAL JUDITH A. PENNA Indiana University Purdue University Indianapolis COLLEGE ALGEBRA: GRAPHS AND MODELS FIFTH EDITION

Exercise Set 6.2 223

x + y + z = 2 (1)

− 10y − z = 19 (4)

− 30y − 30z = 30 (6)

Multiply equation (4) by −3 and add it to equation (6).

x + y + z = 2 (1)

− 10y − z = 19 (4)

− 27z = −27 (7)

Solve equation (7) for z.

−27z = −27

z = 1

Back-substitute 1 for z in equation (4) and solve for y.

−10y − 1 = 19

−10y = 20

y = −2

Back-substitute 1 for z for −2 and y in equation (1) andsolve for x.x + (−2) + 1 = 2

x− 1 = 2

x = 3

The solution is (3,−2, 1).

3. x − y + 2z = −3 (1)

x + 2y + 3z = 4 (2)

2x + y + z = −3 (3)

Multiply equation (1) by −1 and add it to equation (2).We also multiply equation (1) by −2 and add it to equation(3).

x − y + 2z = −3 (1)

3y + z = 7 (4)

3y − 3z = 3 (5)

Multiply equation (4) by −1 and add it to equation (5).

x − y + 2z = −3 (1)

3y + z = 7 (4)

− 4z = −4 (6)

Solve equation (6) for z.

−4z = −4

z = 1

Back-substitute 1 for z in equation (4) and solve for y.

3y + 1 = 7

3y = 6

y = 2

Back-substitute 1 for z and 2 for y in equation (1) andsolve for x.x− 2 + 2 · 1 = −3

x = −3

The solution is (−3, 2, 1).

5. x + 2y − z = 5, (1)

2x − 4y + z = 0, (2)

3x + 2y + 2z = 3 (3)

Multiply equation (1) by −2 and add it to equation (2).Also, multiply equation (1) by −3 and add it to equa-tion (3).

x + 2y − z = 5, (1)

− 8y + 3z = −10, (4)

− 4y + 5z = −12 (5)

Multiply equation (5) by 2 to make the y-coefficient a mul-tiple of the y-coefficient of equation (4).

x + 2y − z = 5, (1)

− 8y + 3z = −10, (4)

− 8y + 10z = −24 (6)

Multiply equation (4) by −1 and add it to equation (6).

x + 2y − z = 5, (1)

− 8y + 3z = −10, (4)

7z = −14 (7)

Solve equation (7) for z.

7z = −14

z = −2

Back-substitute −2 for z in equation (4) and solve for y.

−8y + 3(−2) = −10

−8y − 6 = −10

−8y = −4

y =12

Back-substitute12

for y and −2 for z in equation (1) andsolve for x.

x + 2 · 12− (−2) = 5

x + 1 + 2 = 5

x = 2

The solution is(

2,12,−2

).

7. x + 2y − z = −8, (1)

2x − y + z = 4, (2)

8x + y + z = 2 (3)

Multiply equation (1) by −2 and add it to equation (2).Also, multiply equation (1) by −8 and add it to equa-tion (3).

x + 2y − z = −8, (1)

− 5y + 3z = 20, (4)

− 15y + 9z = 66 (5)

Multiply equation (4) by −3 and add it to equation (5).

x + 2y − z = −8, (1)

− 5y + 3z = 20, (4)

0 = 6 (6)

Copyright © 2013 Pearson Education, Inc.

Page 227: JUDITH A. PENNA -  · PDF fileSTUDENT’S SOLUTIONS MANUAL JUDITH A. PENNA Indiana University Purdue University Indianapolis COLLEGE ALGEBRA: GRAPHS AND MODELS FIFTH EDITION

224 Chapter 6: Systems of Equations and Matrices

Equation (6) is false, so the system of equations has nosolution.

9. 2x + y − 3z = 1, (1)

x − 4y + z = 6, (2)

4x − 7y − z = 13 (3)

Interchange equations (1) and (2).

x − 4y + z = 6, (2)

2x + y − 3z = 1, (1)

4x − 7y − z = 13 (3)

Multiply equation (2) by −2 and add it to equation (1).Also, multiply equation (2) by −4 and add it to equa-tion (3).

x − 4y + z = 6, (2)

9y − 5z = −11, (4)

9y − 5z = −11 (5)

Multiply equation (4) by −1 and add it to equation (5).

x − 4y + z = 6, (1)

9y − 5z = −11, (4)

0 = 0 (6)

The equation 0 = 0 tells us that equation (3) of the originalsystem is dependent on the first two equations. The systemof equations has infinitely many solutions and is equivalentto2x + y − 3z = 1, (1)

x − 4y + z = 6. (2)

To find an expression for the solutions, we first solve equa-tion (4) for either y or z. We choose to solve for z.

9y − 5z = −11

−5z = −9y − 11

z =9y + 11

5Back-substitute in equation (2) to find an expression for xin terms of y.

x− 4y +9y + 11

5= 6

x− 4y +95y +

115

= 6

x =115y +

195

=11y + 19

5

The solutions are given by(

11y + 195

, y,9y + 11

5

), where

y is any real number.

11. 4a + 9b = 8, (1)

8a + 6c = −1, (2)

6b + 6c = −1 (3)

Multiply equation (1) by −2 and add it to equation (2).

4a + 9b = 8, (1)

− 18b + 6c = −17, (4)

6b + 6c = −1 (3)

Multiply equation (3) by 3 to make the b-coefficient a mul-tiple of the b-coefficient in equation (4).

4a + 9b = 8, (1)

− 18b + 6c = −17, (4)

18b + 18c = −3 (5)

Add equation (4) to equation (5).

4a + 9b = 8, (1)

− 18b + 6c = −17, (4)

24c = −20 (6)

Solve equation (6) for c.

24c = −20

c = −2024

= −56

Back-substitute −56

for c in equation (4) and solve for b.

−18b + 6c = −17

−18b + 6(− 5

6

)= −17

−18b− 5 = −17

−18b = −12

b =1218

=23

Back-substitute23

for b in equation (1) and solve for a.

4a + 9b = 8

4a + 9 · 23

= 8

4a + 6 = 8

4a = 2

a =12

The solution is(

12,23,−5

6

).

13. 2x + z = 1, (1)

3y − 2z = 6, (2)

x − 2y = −9 (3)

Interchange equations (1) and (3).

x − 2y = −9, (3)

3y − 2z = 6, (2)

2x + z = 1 (1)

Multiply equation (3) by −2 and add it to equation (1).

x − 2y = −9, (3)

3y − 2z = 6, (2)

4y + z = 19 (4)

Multiply equation (4) by 3 to make the y-coefficient a mul-tiple of the y-coefficient in equation (2).

x − 2y = −9, (3)

3y − 2z = 6, (2)

12y + 3z = 57 (5)

Copyright © 2013 Pearson Education, Inc.

Page 228: JUDITH A. PENNA -  · PDF fileSTUDENT’S SOLUTIONS MANUAL JUDITH A. PENNA Indiana University Purdue University Indianapolis COLLEGE ALGEBRA: GRAPHS AND MODELS FIFTH EDITION

Exercise Set 6.2 225

Multiply equation (2) by −4 and add it to equation (5).

x − 2y = −9, (3)

3y − 2z = 6, (2)

11z = 33 (6)

Solve equation (6) for z.

11z = 33

z = 3

Back-substitute 3 for z in equation (2) and solve for y.

3y − 2z = 6

3y − 2 · 3 = 6

3y − 6 = 6

3y = 12

y = 4

Back-substitute 4 for y in equation (3) and solve for x.

x− 2y = −9

x− 2 · 4 = −9

x− 8 = −9

x = −1

The solution is (−1, 4, 3).

15. w + x + y + z = 2 (1)

w + 2x + 2y + 4z = 1 (2)

−w + x − y − z = −6 (3)

−w + 3x + y − z = −2 (4)

Multiply equation (1) by −1 and add to equation (2). Addequation (1) to equation (3) and to equation (4).

w + x + y + z = 2 (1)

x + y + 3z = −1 (5)

2x = −4 (6)

4x + 2y = 0 (7)

Solve equation (6) for x.

2x = −4

x = −2

Back-substitute −2 for x in equation (7) and solve for y.

4(−2) + 2y = 0

−8 + 2y = 0

2y = 8

y = 4

Back-substitute −2 for x and 4 for y in equation (5) andsolve for z.−2 + 4 + 3z = −1

3z = −3

z = −1

Back-substitute −2 for x, 4 for y, and −1 for z in equa-tion (1) and solve for w.

w − 2 + 4 − 1 = 2

w = 1

The solution is (1,−2, 4,−1).

17. Familiarize. Let x, y, and z represent the number of Win-ter Olympics sites in North America, Europe, and Asia,respectively.

Translate. The total number of sites is 21.

x + y + z = 21

The number of European sites is 5 more than the totalnumber of sites in North America and Asia.

y = x + z + 5

There are 4 more sites in North America than in Asia.

x = z + 4

We havex + y + z = 21,

y = x + z + 5,

x = z + 4or

x + y+ z = 21,

−x + y− z = 5,

x − z = 4.

Carry out. Solving the system of equations, we get(6, 13, 2).

Check. The total number of sites is 6+13+2, or 21. Thetotal number of sites in North America and Asia is 6+2, or8, and 5 more than this is 8+5, or 13, the number of sitesin Europe. Also, the number of sites in North America, 6,is 4 more than 2, the number of sites in Asia. The answerchecks.

State. The Winter Olympics have been held in 6 NorthAmerican sites, 13 European sites, and 2 Asian sites.

19. Familiarize. Let x, y, and z represent the number ofbillions of pounds of apples prduced in China, the UnitedStates, and Turkey, respectively.

Translate. A total of 74 billion lb of apples were grownin three countries.

x + y + z = 74

China produced 44 billion lb more than the combined pro-ductions of the United States and Turkey.

x = y + z + 44

The United States produced twice as many pounds asTurkey.

y = 2z

We havex + y + z = 74,

x = y + z + 44,

y = 2z.orx + y + z = 74,

x− y − z = 44,

y − 2z = 0.

Carry out. Solving the system of equations, we get(59, 10, 5).

Copyright © 2013 Pearson Education, Inc.

Page 229: JUDITH A. PENNA -  · PDF fileSTUDENT’S SOLUTIONS MANUAL JUDITH A. PENNA Indiana University Purdue University Indianapolis COLLEGE ALGEBRA: GRAPHS AND MODELS FIFTH EDITION

226 Chapter 6: Systems of Equations and Matrices

Check. The total productions is 59+10+5, or 74 billion lb.China’s production, 59 billion lb, is 44 billion lb more thanthe combined production of the United States and Turkey,10 + 5, or 15 billion lb. The production in the UnitedStates, 10 billion lb, is twice the production in Turkey, 5billion lb. The answer checks.

State. The apple production in China, the United States,and Turkey was 59 billion lb, 10 billion lb, and 5 billionlb, respectively.

21. Familiarize. Let x y, and z represent the number of mil-ligrams of caffeine in an 8-oz serving of brewed coffee, RedBull energy drink, and Mountain Dew, respectively.

Translate. The total amount of caffeine in one serving ofeach beverage is 197 mg.

x + y + z = 197

One serving of brewed coffee has 6 mg more caffeine thantwo servings of Mountain Dew.

x = 2z + 6

One serving of Red Bull contains 37 mg less caffeine thanone serving each of brewed coffee and Mountain Dew.

y = x + z − 37

We have a system of equations

x + y + z = 197, x+ y + z = 197,

x = 2z + 6, or x − 2z = 6,

y = x + z − 37 −x+ y− z =−37

Carry out. Solving the system of equations, we get(80, 80, 37).

Check. The total amount of caffeine is 80 + 80 + 37, or197 mg. Also, 80 mg is 6 mg more than twice 37 mg, and80 mg is 37 mg less than the total of 80 mg and 37 mg, or117 mg. The answer checks.

State. One serving each of brewed coffee, Red Bull energydrink, and Mountain Dew contains 80 mg, 80 mg, and37 mg of caffeine, respectively.

23. Familiarize. Let x, y, and z represent the number of fish,cats, and dogs owned by Americans, in millions, respec-tively.

Translate. The total number of fish, cats, and dogs ownedis 355 million.

x + y + z = 355

The number of fish owned is 11 million more than the totalnumber of cats and dogs owned.

x = y + z + 11

There are 16 million more cats than dogs.

y = z + 16

We havex + y + z = 355,

x = y + z + 11,

y = z + 16or

x + y+ z = 355,

x − y− z = 11,

y− z = 16.

Carry out. Solving the system of equations, we get(183, 94, 78).

Check. The total number of fish, cats, and dogs is183+94+78, or 355 million. The total number of cats anddogs owned is 94 + 78, or 172 million, and 172 million +11 million is 183 million, the number of fish owned. Thenumber of cats owned, 94 million, is 16 million more than78 million, the number of dogs owned. The solution checks.

State. Americans own 183 million fish, 94 million cats,and 78 million dogs.

25. Familiarize. Let x, y, and z represent the amounts earnedby The Dark Knight, Spider-Man 3, and The TwilightSaga: New Moon, respectively, in millions of dollars.

Translate. Total earnings are $452 million.

x + y + z = 452

Together, Spider-Man 3 and New Moon earned $136 mil-lion more than The Dark Knight.

y + z = x + 136

New Moon earned $15 million less than The Dark Knight.

z = x− 15

We havex + y + z = 452,

y + z = x + 136,

z = x− 15or

x + y+ z = 452,

−x + y+ z = 136,

−x + z = −15.

Carry out. Solving the system of equations, we get(158, 151, 143).

Check. The earnings total $158+$151+$143, or $452 mil-lion. Together, Spider-Man 3 and New Moon earned$151 + $143, or $294 million. This is $136 million morethan $158 million, the earnings of The Dark Knight. Also,$15 million less than $158 million, the earnings of TheDark Knight is $158− $15 or $143 million, the earnings ofNew Moon. The answer checks.

State. The Dark Knight, Spider-Man 3, and New Moongrossed $158 million, $151 million, and $143 million, re-spectively, in a weekend.

27. Familiarize. Let x, y, and z represent the number ofservings of ground beef, baked potato, and strawberriesrequired, respectively. One serving of ground beef con-tains 245x Cal, 0x or 0 g of carbohydrates, and 9x mgof calcium. One baked potato contains 145y Cal, 34y gof carbohydrates, and 8y mg of calcium. One serving ofstrawberries contains 45z Cal, 10z g of carbohydrates, and21z mg of calcium.

Translate.

The total number of calories is 485.

Copyright © 2013 Pearson Education, Inc.

Page 230: JUDITH A. PENNA -  · PDF fileSTUDENT’S SOLUTIONS MANUAL JUDITH A. PENNA Indiana University Purdue University Indianapolis COLLEGE ALGEBRA: GRAPHS AND MODELS FIFTH EDITION

Exercise Set 6.2 227

245x + 145y + 45z = 485

A total of 41.5 g of carbohydrates is required.

34y + 10z = 41.5

A total of 35 mg of calcium is required.

9x + 8y + 21z = 35

We have a system of equations.

245x + 145y + 45z = 485,

34y + 10z = 41.5,

9x + 8y + 21z = 35

Carry out. Solving the system of equations, we get(1.25, 1, 0.75).

Check. 1.25 servings of ground beef contains 306.25 Cal,no carbohydrates, and 11.25 mg of calcium; 1 baked potatocontains 145 Cal, 34 g of carbohydrates, and 8 mg of cal-cium; 0.75 servings of strawberries contains 33.75 Cal, 7.5 gof carbohydrates, and 15.75 mg of calcium. Thus, thereare a total of 306.25 + 145 + 33.75, or 485 Cal, 34 + 7.5, or41.5 g of carbohydrates, and 11.25 + 8 + 15.75, or 35 mgof calcium. The solution checks.

State. 1.25 servings of ground beef, 1 baked potato, and0.75 serving of strawberries are required.

29. Familiarize. Let x, y, and z represent the amounts in-vested at 3%, 4%, and 6%, respectively. Then the annualinterest from the investments is 3%x, 4%y, and 6%z, or0.03x, 0.04y, and 0.06z.

Translate.

A total of $5000 was invested.

x + y + z = 5000

The total interest is $243.

0.03x + 0.04y + 0.06z = 243

The amount invested at 6% is $1500 more than the amountinvested at 3%.

z = x + 1500

We have a system of equations.

x + y + z = 5000,

0.03x + 0.04y + 0.06z = 243,

z = x + 1500or

x+ y + z = 5000,

3x+ 4y + 6z = 24, 300,

−x + z = 1500

Carry out. Solving the system of equations, we get(1300, 900, 2800).

Check. The total investment was $1300+$900+$2800, or$5000. The total interest was 0.03($1300) + 0.04($900) +0.06($2800) = $39 + $36 + $168, or $243. The amountinvested at 6%, $2800, is $1500 more than the amountinvested at 3%, $1300. The solution checks.

State. $1300 was invested at 3%, $900 at 4%, and $2800at 6%.

31. Familiarize. Let x, y, and z represent the prices of orangejuice, a raisin bagel, and a cup of coffee, respectively. Thenew price for orange juice is x + 25%x, or x + 0.25x, or1.25x; the new price of a bagel is y+20%y, or y+0.2y, or1.2y.

Translate.

Orange juice, a raisin bagel, and a cup of coffee cost $8.15.

x + y + z = 8.15

After the price increase, orange juice, a raisin bagel, anda cup of coffee will cost $9.30.

1.25x + 1.2y + z = 9.30

After the price increases, a raisin bagel will cost 30/c (or$0.30) more than coffee.

1.2y = z + 0.30

We have a system of equations.x + y + z = 8.15, 100x+ 100y + 100z = 815,

1.25x+1.2y+z=9.30, or 125x+ 120y + 100z = 930,

1.2y = z + 0.30 12y− 10z = 3Carry out. Solving the system of equations, we get(2.4, 2.75, 3).

Check. If orange juice costs $2.40, a bagel costs $2.75,and a cup of coffee costs $3.00, then together they cost$2.40 + $2.75 + $3.00, or $8.15. After the price increasesorange juice will cost 1.25($2.40), or $3.00 and a bagel willcost 1.2($2.75) or $3.30. Then orange juice, a bagel, andcoffee will cost $3.00 + $3.30 + $3.00, or $9.30. After theprice increase the price of a raisin bagel, $3.30, will be 30/cmore than the price of coffee, $3.00. The solution checks.

State. Before the increase orange juice cost $2.40, a raisinbagel cost $2.75, and a cup of coffee cost $3.00.

33. a) Substitute the data points (0, 16), (7, 9), and(20, 21) in the function f(x) = ax2 + bx + c.

16 = a · 02 + b · 0 + c

9 = a · 72 + b · 7 + c

21 = a · 202 + b · 20 + c

We have a system of equations.

c = 16,

49a + 7b + c = 9,

400a + 20b + c = 21.

Solving the system of equations, we get(552

,−8752

, 16)

so f(x) =552

x2 − 8752

x + 16,

where x is the number of years after 1990 and f(x) isa percent.

b) In 2003, x = 2003 − 1990, or 13.

f(13) =552

· 132 − 8752

· 13 + 16 = 10.5%

35. a) Substitute the data points (0, 431), (5, 441), and(12, 418) in the function f(x) = ax2 + bx + c.

431 = a · 02 + b · 0 + c

441 = a · 52 + b · 5 + c

418 = a · 122 + b · 12 + c

Copyright © 2013 Pearson Education, Inc.

Page 231: JUDITH A. PENNA -  · PDF fileSTUDENT’S SOLUTIONS MANUAL JUDITH A. PENNA Indiana University Purdue University Indianapolis COLLEGE ALGEBRA: GRAPHS AND MODELS FIFTH EDITION

228 Chapter 6: Systems of Equations and Matrices

We have a system of equations.

c = 431,

25a + 5b + c = 441,

144a + 12b + c = 418

Solving the system of equations, we get(− 37

84,35384

, 431)

, so

f(x) = −3784

x2 +35384

x + 431,

where x is the number of years after 1997.

b) In 2000, x = 2000 − 1997, or 3.

f(3) = −3784

· 32 +35384

· 3 + 431 ≈ 440 acres

In 2010, x = 2010 − 1997 = 13.

f(13) = −3784

· 132 +35384

· 13 + 431 ≈ 411 acres

37. a) f(x)=0.143707483x2−0.6921768707x + 5.882482993,where x is the number of years after 2002.

b) In 2003, x = 2003 − 2002 = 1.

f(1) ≈ 5.3%

In 2007, x = 2007 − 2002 = 5.

f(5) ≈ 6.0%

In 2009, x = 2009 − 2002 = 7.

f(7) ≈ 8.1%

39. Perpendicular

41. A vertical line

43. A rational function

45. A vertical asymptote

47. 2x

+2y− 3

z= 3,

1x− 2

y− 3

z= 9,

7x− 2

y+

9z

= −39

Substitute u for1x

, v for1y, and w for

1z

and solve for u,

v, and w.

2u + 2v − 3w = 3,

u − 2v − 3w = 9,

7u − 2v + 9w = −39

Solving this system we get (−2,−1,−3).

Then1x

= −2, or x = −12;

1y

= −1, or y = −1; and

1z

= −3, or z = −13. The solution of the original system is(

− 12,−1,−1

3

).

49. Substituting, we get

A +34B + 3C = 12,

43A + B + 2C = 12,

2A + B + C = 12, or

4A + 3B + 12C = 48,

4A + 3B + 6C = 36, Clearing fractions

2A + B + C = 12.

Solving the system of equations, we get (3, 4, 2). The equa-tion is 3x + 4y + 2z = 12.

51. Substituting, we get59 = a(−2)3 + b(−2)2 + c(−2) + d,

13 = a(−1)3 + b(−1)2 + c(−1) + d,

−1 = a · 13 + b · 12 + c · 1 + d,

−17 = a · 23 + b · 22 + c · 2 + d, or

−8a + 4b − 2c + d = 59,

−a + b − c + d = 13,

a + b + c + d = −1,

8a + 4b + 2c + d = −17.Solving the system of equations, we get

(−4, 5,−3, 1), so y = −4x3 + 5x2 − 3x + 1.

Exercise Set 6.3

1. The matrix has 3 rows and 2 columns, so its order is 3×2.

3. The matrix has 1 row and 4 columns, so its order is 1× 4.

5. The matrix has 3 rows and 3 columns, so its order is 3×3.

7. We omit the variables and replace the equals signs with avertical line. 2 −1 7

1 4 −5

9. We omit the variables, writing zeros for the missing terms,and replace the equals signs with a vertical line.

1 −2 3 12

2 0 −4 8

0 3 1 7

11. Insert variables and replace the vertical line with equalssigns.

3x− 5y = 1,

x + 4y = −2

13. Insert variables and replace the vertical line with equalssigns.2x + y − 4z = 12,

3x + 5z = −1,

x − y + z = 2

Copyright © 2013 Pearson Education, Inc.

Page 232: JUDITH A. PENNA -  · PDF fileSTUDENT’S SOLUTIONS MANUAL JUDITH A. PENNA Indiana University Purdue University Indianapolis COLLEGE ALGEBRA: GRAPHS AND MODELS FIFTH EDITION

Exercise Set 6.3 229

15. 4x + 2y = 11,

3x − y = 2

Write the augmented matrix. We will use Gaussian elimi-nation. 4 2 11

3 −1 2

Multiply row 2 by 4 to make the first number in row 2 amultiple of 4. 4 2 11

12 −4 8

Multiply row 1 by −3 and add it to row 2. 4 2 11

0 −10 −25

Multiply row 1 by14

and row 2 by − 110

.

112

114

0 152

Write the system of equations that corresponds to the lastmatrix.

x +12y =

114, (1)

y =52

(2)

Back-substitute in equation (1) and solve for x.

x +12· 52

=114

x +54

=114

x =64

=32

The solution is(

32,52

).

17. 5x − 2y = −3,

2x + 5y = −24

Write the augmented matrix. We will use Gaussian elimi-nation. 5 −2 −3

2 5 −24

Multiply row 2 by 5 to make the first number in row 2 amultiple of 5. 5 −2 −3

10 25 −120

Multiply row 1 by −2 and add it to row 2. 5 −2 −3

0 29 −114

Multiply row 1 by15

and row 2 by129

.

1 −25

−35

0 1 −11429

Write the system of equations that corresponds to the lastmatrix.

x− 25y = −3

5, (1)

y = −11429

(2)

Back-substitute in equation (1) and solve for x.

x− 25

(− 114

29

)= −3

5

x +228145

= −35

x = −315145

= −6329

The solution is(− 63

29,−114

29

).

19. 3x + 4y = 7,

−5x + 2y = 10

Write the augmented matrix. We will use Gaussian elimi-nation. 3 4 7

−5 2 10

Multiply row 2 by 3 to make the first number in row 2 amultiple of 3. 3 4 7

−15 6 30

Multiply row 1 by 5 and add it to row 2. 3 4 7

0 26 65

Multiply row 1 by13

and row 2 by126

.

143

73

0 152

Write the system of equations that corresponds to the lastmatrix.

Copyright © 2013 Pearson Education, Inc.

Page 233: JUDITH A. PENNA -  · PDF fileSTUDENT’S SOLUTIONS MANUAL JUDITH A. PENNA Indiana University Purdue University Indianapolis COLLEGE ALGEBRA: GRAPHS AND MODELS FIFTH EDITION

230 Chapter 6: Systems of Equations and Matrices

x +43y =

73, (1)

y =52

(2)

Back-substitute in equation (1) and solve for x.

x +43· 52

=73

x +103

=73

x = −33

= −1

The solution is(− 1,

52

).

21. 3x + 2y = 6,

2x − 3y = −9

Write the augmented matrix. We will use Gauss-Jordanelimination. 3 2 6

2 −3 −9

Multiply row 2 by 3 to make the first number in row 2 amultiple of 3. 3 2 6

6 −9 −27

Multiply row 1 by −2 and add it to row 2. 3 2 6

0 −13 −39

Multiply row 2 by − 113

.

3 2 6

0 1 3

Multiply row 2 by −2 and add it to row 1. 3 0 0

0 1 3

Multiply row 1 by13.

1 0 0

0 1 3

We have x = 0, y = 3. The solution is (0, 3).

23. x − 3y = 8,

2x − 6y = 3

Write the augmented matrix.

1 −3 8

2 −6 3

Multiply row 1 by −2 and add it to row 2. 1 −3 8

0 0 −13

The last row corresponds to the false equation 0 = −13,so there is no solution.

25. −2x + 6y = 4,

3x − 9y = −6

Write the augmented matrix. −2 6 4

3 −9 −6

Multiply row 1 by −12.

1 −3 −2

3 −9 −6

Multiply row 1 by −3 and add it to row 2. 1 −3 −2

0 0 0

The last row corresponds to the equation 0 = 0 which istrue for all values of x and y. Thus, the system of equationsis dependent and is equivalent to the first equation −2x+6y = 4, or x− 3y = −2. Solving for x, we get x = 3y − 2.Then the solutions are of the form (3y − 2, y), where y isany real number.

27. x + 2y − 3z = 9,

2x − y + 2z = −8,

3x − y − 4z = 3

Write the augmented matrix. We will use Gauss-Jordanelimination.

1 2 −3 9

2 −1 2 −8

3 −1 −4 3

Multiply row 1 by −2 and add it to row 2. Also, multiplyrow 1 by −3 and add it to row 3.

1 2 −3 9

0 −5 8 −26

0 −7 5 −24

Multiply row 2 by −15

to get a 1 in the second row, secondcolumn.

Copyright © 2013 Pearson Education, Inc.

Page 234: JUDITH A. PENNA -  · PDF fileSTUDENT’S SOLUTIONS MANUAL JUDITH A. PENNA Indiana University Purdue University Indianapolis COLLEGE ALGEBRA: GRAPHS AND MODELS FIFTH EDITION

Exercise Set 6.3 231

1 2 −3 9

0 1 −85

265

0 −7 5 −24

Multiply row 2 by −2 and add it to row 1. Also, multiplyrow 2 by 7 and add it to row 3.

1 015

−75

0 1 −85

265

0 0 −315

625

Multiply row 3 by − 531

to get a 1 in the third row, thirdcolumn.

1 015

−75

0 1 −85

265

0 0 1 −2

Multiply row 3 by −15

and add it to row 1. Also, multiply

row 3 by85

and add it to row 2.

1 0 0 −1

0 1 0 2

0 0 1 −2

We have x = −1, y = 2, z = −2. The solution is(−1, 2,−2).

29. 4x − y − 3z = 1,

8x + y − z = 5,

2x + y + 2z = 5

Write the augmented matrix. We will use Gauss-Jordanelimination.

4 −1 −3 1

8 1 −1 5

2 1 2 5

First interchange rows 1 and 3 so that each number be-low the first number in the first row is a multiple of thatnumber.

2 1 2 5

8 1 −1 5

4 −1 −3 1

Multiply row 1 by −4 and add it to row 2. Also, multiplyrow 1 by −2 and add it to row 3.

2 1 2 5

0 −3 −9 −15

0 −3 −7 −9

Multiply row 2 by −1 and add it to row 3.

2 1 2 5

0 −3 −9 −15

0 0 2 6

Multiply row 2 by −13

to get a 1 in the second row, secondcolumn.

2 1 2 5

0 1 3 5

0 0 2 6

Multiply row 2 by −1 and add it to row 1.

2 0 −1 0

0 1 3 5

0 0 2 6

Multiply row 3 by12

to get a 1 in the third row, thirdcolumn.

2 0 −1 0

0 1 3 5

0 0 1 3

Add row 3 to row 1. Also multiply row 3 by −3 and addit to row 2.

2 0 0 3

0 1 0 −4

0 0 1 3

Finally, multiply row 1 by12.

1 0 032

0 1 0 −4

0 0 1 3

We have x =32, y = −4, z = 3. The solution is(

32,−4, 3

).

31. x − 2y + 3z = 4,

3x + y − z = 0,

2x + 3y − 5z = 1

Write the augmented matrix. We will use Gaussian elimi-nation.

1 −2 3 −4

3 1 −1 0

2 3 −5 1

Multiply row 1 by −3 and add it to row 2. Also, multiplyrow 1 by −2 and add it to row 3.

Copyright © 2013 Pearson Education, Inc.

Page 235: JUDITH A. PENNA -  · PDF fileSTUDENT’S SOLUTIONS MANUAL JUDITH A. PENNA Indiana University Purdue University Indianapolis COLLEGE ALGEBRA: GRAPHS AND MODELS FIFTH EDITION

232 Chapter 6: Systems of Equations and Matrices

1 −2 3 −4

0 7 −10 12

0 7 −11 9

Multiply row 2 by −1 and add it to row 3.

1 −2 3 −4

0 7 −10 12

0 0 −1 −3

Multiply row 2 by17

and multiply row 3 by −1.

1 −2 3 −4

0 1 −107

127

0 0 1 3

Now write the system of equations that corresponds to thelast matrix.x − 2y + 3z = −4, (1)

y − 107z =

127, (2)

z = 3 (3)

Back-substitute 3 for z in equation (2) and solve for y.

y − 107

· 3 =127

y − 307

=127

y =427

= 6

Back-substitute 6 for y and 3 for z in equation (1) andsolve for x.

x− 2 · 6 + 3 · 3 = −4

x− 3 = −4

x = −1

The solution is (−1, 6, 3).

33. 2x − 4y − 3z = 3,

x + 3y + z = −1,

5x + y − 2z = 2

Write the augmented matrix.

2 −4 −3 3

1 3 1 −1

5 1 −2 2

Interchange the first two rows to get a 1 in the first row,first column.

1 3 1 −1

2 −4 −3 3

5 1 −2 2

Multiply row 1 by −2 and add it to row 2. Also, multiplyrow 1 by −5 and add it to row 3.

1 3 1 −1

0 −10 −5 5

0 −14 −7 7

Multiply row 2 by − 110

to get a 1 in the second row, secondcolumn.

1 3 1 −1

0 112

−12

0 −14 −7 7

Multiply row 2 by 14 and add it to row 3.

1 3 1 −1

0 112

−12

0 0 0 0

The last row corresponds to the equation 0 = 0. Thisindicates that the system of equations is dependent. It isequivalent to

x + 3y + z = −1,

y +12z = −1

2We solve the second equation for y.

y = −12z − 1

2Substitute for y in the first equation and solve for x.

x + 3(− 1

2z − 1

2

)+ z = −1

x− 32z − 3

2+ z = −1

x =12z +

12

The solution is(

12z +

12,−1

2z − 1

2, z

), where z is any real

number.

35. p + q + r = 1,

p + 2q + 3r = 4,

4p + 5q + 6r = 7

Write the augmented matrix.

1 1 1 1

1 2 3 4

4 5 6 7

Multiply row 1 by −1 and add it to row 2. Also, multiplyrow 1 by −4 and add it to row 3.

1 1 1 1

0 1 2 3

0 1 2 3

Multiply row 2 by −1 and add it to row 3.

Copyright © 2013 Pearson Education, Inc.

Page 236: JUDITH A. PENNA -  · PDF fileSTUDENT’S SOLUTIONS MANUAL JUDITH A. PENNA Indiana University Purdue University Indianapolis COLLEGE ALGEBRA: GRAPHS AND MODELS FIFTH EDITION

Exercise Set 6.3 233

1 1 1 1

0 1 2 3

0 0 0 0

The last row corresponds to the equation 0 = 0. Thisindicates that the system of equations is dependent. It isequivalent to

p + q + r = 1,

q + 2r = 3.

We solve the second equation for q.

q = −2r + 3

Substitute for y in the first equation and solve for p.

p− 2r + 3 + r = 1

p− r + 3 = 1

p = r − 2

The solution is (r − 2,−2r + 3, r), where r is any realnumber.

37. a + b − c = 7,

a − b + c = 5,

3a + b − c = −1

Write the augmented matrix.

1 1 −1 7

1 −1 1 5

3 1 −1 −1

Multiply row 1 by −1 and add it to row 2. Also, multiplyrow 1 by −3 and add it to row 3.

1 1 −1 7

0 −2 2 −2

0 −2 2 −22

Multiply row 2 by −1 and add it to row 3.

1 1 −1 7

0 −2 2 −2

0 0 0 −20

The last row corresponds to the false equation 0 = −20.Thus, the system of equations has no solution.

39. −2w + 2x + 2y − 2z = −10,

w + x + y + z = −5,

3w + x − y + 4z = −2,

w + 3x − 2y + 2z = −6

Write the augmented matrix. We will use Gaussian elimi-nation.

−2 2 2 −2 −10

1 1 1 1 −5

3 1 −1 4 −2

1 3 −2 2 −6

Interchange rows 1 and 2.

1 1 1 1 −5

−2 2 2 −2 −10

3 1 −1 4 −2

1 3 −2 2 −6

Multiply row 1 by 2 and add it to row 2. Multiply row 1by −3 and add it to row 3. Multiply row 1 by −1 and addit to row 4.

1 1 1 1 −5

0 4 4 0 −20

0 −2 −4 1 13

0 2 −3 1 −1

Interchange rows 2 and 3.

1 1 1 1 −5

0 −2 −4 1 13

0 4 4 0 −20

0 2 −3 1 −1

Multiply row 2 by 2 and add it to row 3. Add row 2 torow 4.

1 1 1 1 −5

0 −2 −4 1 13

0 0 −4 2 6

0 0 −7 2 12

Multiply row 4 by 4.

1 1 1 1 −5

0 −2 −4 1 13

0 0 −4 2 6

0 0 −28 8 48

Multiply row 3 by −7 and add it to row 4.

1 1 1 1 −5

0 −2 −4 1 13

0 0 −4 2 6

0 0 0 −6 6

Multiply row 2 by −12, row 3 by −1

4, and row 6 by −1

6.

1 1 1 1 −5

0 1 2 −12

−132

0 0 1 −12

−32

0 0 0 1 −1

Write the system of equations that corresponds to the lastmatrix.

Copyright © 2013 Pearson Education, Inc.

Page 237: JUDITH A. PENNA -  · PDF fileSTUDENT’S SOLUTIONS MANUAL JUDITH A. PENNA Indiana University Purdue University Indianapolis COLLEGE ALGEBRA: GRAPHS AND MODELS FIFTH EDITION

234 Chapter 6: Systems of Equations and Matrices

w + x + y + z = −5, (1)

x + 2y − 12z = −13

2, (2)

y − 12z = −3

2, (3)

z = −1 (4)

Back-substitute in equation (3) and solve for y.

y − 12(−1) = −3

2

y +12

= −32

y = −2

Back-substitute in equation (2) and solve for x.

x + 2(−2) − 12(−1) = −13

2

x− 4 +12

= −132

x = −3

Back-substitute in equation (1) and solve for w.

w − 3 − 2 − 1 = −5

w = 1

The solution is (1,−3,−2,−1).

41. Familiarize. Let x and y represent the number of 44/c and17/c stamps purchased, respectively. Then Otto spent0.44x on 44/c stamps and 0.17y on 17/c stamps.

Translate. Otto spent a total of $22.35 on stamps.

0.44x + 0.17y = 22.35

He purchased a total of 60 stamps.

x + y = 60

We have a system of equations.

0.44x + 0.17y = 22.35, 44x + 17y = 2235,or

x + y = 60, x + y = 60

Carry out. Using Gaussian elimination or Gauss-Jordanelimination, we find that the solution is (45, 15).

Check. The total purchase price is $0.44(45) + $0.17(15),or $22.35. The number of stamps purchased is 45 + 15, or60. The answer checks.

State. Otto bought 45 44/c stamps and 15 17/c stamps.

43. Familiarize. Let x, y, and z represent the amount spenton advertising in fiscal years 2010, 2011, and 2012, respec-tively, in millions of dollars.

Translate. A total of $11 million was spent on advertis-ing.

x + y + z = 11

The amount spent in 2012 was three times the amountspent in 2010.

z = 3x

The amount spent in 2011 was $3 million less than theamount spent in 2012.

y = z − 3

We have a system of equations.x + y + z = 11, x + y + z = 11,z = 3x, or −3x + z = 0,y = z − 3, y − z = −3

Carry out. Using Gaussian elimination or Gauss-Jordanelimination we find that the solution is (2, 3, 6).

Check. The total spending is 2 + 3 + 6, or $11 million.The amount spent in 2012, $6 million, is three times theamount spent in 2010, $2 million. The amount spent in2011, $3 million, is $3 million less than the amount spentin 2012, $6 million. The answer checks.

State. The amounts spent on advertising in 2010, 2011,and 2012 were $2 million, $3 million, and $6 million, re-spectively.

45. The function has a variable in the exponent, so it is anexponential function.

47. The function is the quotient of two polynomials, so it is arational function.

49. The function is of the form f(x) = loga x, so it is logarith-mic.

51. The function is of the form f(x) = mx + b, so it is linear.

53. Substitute to find three equations.12 = a(−3)2 + b(−3) + c

−7 = a(−1)2 + b(−1) + c

−2 = a · 12 + b · 1 + c

We have a system of equations.9a − 3b + c = 12,

a − b + c = −7,

a + b + c = −2

Write the augmented matrix. We will use Gaussian elimi-nation.

9 −3 1 12

1 −1 1 −7

1 1 1 −2

Interchange the first two rows.

1 −1 1 −7

9 −3 1 12

1 1 1 −2

Multiply row 1 by −9 and add it to row 2. Also, multiplyrow 1 by −1 and add it to row 3.

1 −1 1 −7

0 6 −8 75

0 2 0 5

Interchange row 2 and row 3.

1 −1 1 −7

0 2 0 5

0 6 −8 75

Copyright © 2013 Pearson Education, Inc.

Page 238: JUDITH A. PENNA -  · PDF fileSTUDENT’S SOLUTIONS MANUAL JUDITH A. PENNA Indiana University Purdue University Indianapolis COLLEGE ALGEBRA: GRAPHS AND MODELS FIFTH EDITION

Exercise Set 6.3 235

Multiply row 2 by −3 and add it to row 3.

1 −1 1 −7

0 2 0 5

0 0 −8 60

Multiply row 2 by12

and row 3 by −18.

1 −1 1 −7

0 1 052

0 0 1 −152

Write the system of equations that corresponds to the lastmatrix.

x − y + z = −7,

y =52,

z = −152

Back-substitute52

for y and −152

for z in the first equationand solve for x.

x− 52− 15

2= −7

x− 10 = −7

x = 3

The solution is(

3,52,−15

2

), so the equation is

y = 3x2 +52x− 15

2.

55.

1 5

3 2

Multiply row 1 by −3 and add it to row 2. 1 5

0 −13

Multiply row 2 by − 113

.

1 5

0 1

Row-echelon form

Multiply row 2 by −5 and add it to row 1. 1 0

0 1

Reduced row-echelon form

57. y = x + z,

3y + 5z = 4,

x + 4 = y + 3z, or

x − y + z = 0,

3y + 5z = 4,

x − y − 3z = −4

Write the augmented matrix. We will use Gauss-Jordanelimination.

1 −1 1 0

0 3 5 4

1 −1 −3 −4

Multiply row 1 by −1 and add it to row 3.

1 −1 1 0

0 3 5 4

0 0 −4 −4

Multiply row 3 by −14.

1 −1 1 0

0 3 5 4

0 0 1 1

Multiply row 3 by −1 and add it to row 1. Also, multiplyrow 3 by −5 and add it to row 2.

1 −1 0 −1

0 3 0 −1

0 0 1 1

Multiply row 2 by13.

1 −1 0 −1

0 1 0 −13

0 0 1 1

Add row 2 to row 1.

1 0 0 −43

0 1 0 −13

0 0 1 1

Read the solution from the last matrix. It is(− 4

3,−1

3, 1)

.

59. x − 4y + 2z = 7,

3x + y + 3z = −5

Write the augmented matrix. 1 −4 2 7

3 1 3 −5

Copyright © 2013 Pearson Education, Inc.

Page 239: JUDITH A. PENNA -  · PDF fileSTUDENT’S SOLUTIONS MANUAL JUDITH A. PENNA Indiana University Purdue University Indianapolis COLLEGE ALGEBRA: GRAPHS AND MODELS FIFTH EDITION

236 Chapter 6: Systems of Equations and Matrices

Multiply row 1 by −3 and add it to row 2. 1 −4 2 7

0 13 −3 −26

Multiply row 2 by113

.

1 −4 2 7

0 1 − 313

−2

Write the system of equations that corresponds to the lastmatrix.

x− 4y + 2z = 7,

y − 313

z = −2

Solve the second equation for y.

y =313

z − 2

Substitute in the first equation and solve for x.

x− 4(

313

z − 2)

+ 2z = 7

x− 1213

z + 8 + 2z = 7

x = −1413

z − 1

The solution is(− 14

13z − 1,

313

z − 2, z)

, where z is any

real number.

61. 4x + 5y = 3,

−2x + y = 9,

3x − 2y = −15Write the augmented matrix.

4 5 3

−2 1 9

3 −2 −15

Multiply row 2 by 2 and row 3 by 4.

4 5 3

−4 2 18

12 −8 −60

Add row 1 to row 2. Also, multiply row 1 by −3 and addit to row 3.

4 5 3

0 7 21

0 −23 −69

Multiply row 2 by17

and row 3 by − 123

.

4 5 3

0 1 3

0 1 3

Multiply row 2 by −1 and add it to row 3.

4 5 3

0 1 3

0 0 0

The last row corresponds to the equation 0 = 0. Thus wehave a dependent system that is equivalent to

4x + 5y = 3, (1)

y = 3. (2)

Back-substitute in equation (1) to find x.

4x + 5 · 3 = 3

4x + 15 = 3

4x = −12

x = −3

The solution is (−3, 3).

Exercise Set 6.4

1.[

5 x]

=[y −3

]Corresponding entries of the two matrices must be equal.Thus we have 5 = y and x = −3.

3.[

3 2xy −8

]=[

3 −21 −8

]Corresponding entries of the two matrices must be equal.Thus, we have:

2x = −2 and y = 1

x = −1 and y = 1

5. A + B =[

1 24 3

]+[ −3 5

2 −1

]

=[

1 + (−3) 2 + 54 + 2 3 + (−1)

]

=[ −2 7

6 2

]

7. E + 0 =[

1 32 6

]+[

0 00 0

]

=[

1 + 0 3 + 02 + 0 6 + 0

]

=[

1 32 6

]

9. 3F = 3[

3 3−1 −1

]

=[

3 · 3 3 · 33 · (−1) 3 · (−1)

]

=[

9 9−3 −3

]

Copyright © 2013 Pearson Education, Inc.

Page 240: JUDITH A. PENNA -  · PDF fileSTUDENT’S SOLUTIONS MANUAL JUDITH A. PENNA Indiana University Purdue University Indianapolis COLLEGE ALGEBRA: GRAPHS AND MODELS FIFTH EDITION

Exercise Set 6.4 237

11. 3F = 3[

3 3−1 −1

]=[

9 9−3 −3

],

2A = 2[

1 24 3

]=[

2 48 6

]

3F + 2A =[

9 9−3 −3

]+[

2 48 6

]

=[

9 + 2 9 + 4−3 + 8 −3 + 6

]

=[

11 135 3

]

13. B − A =[ −3 5

2 −1

]−[

1 24 3

]

=[ −3 5

2 −1

]+[ −1 −2

−4 −3

][B − A = B + (−A)]

=[ −3 + (−1) 5 + (−2)

2 + (−4) −1 + (−3)

]

=[ −4 3

−2 −4

]

15. BA =[ −3 5

2 −1

] [1 24 3

]

=[ −3 · 1 + 5 · 4 −3 · 2 + 5 · 3

2 · 1 + (−1)4 2 · 2 + (−1)3

]

=[

17 9−2 1

]

17. CD =[

1 −1−1 1

] [1 11 1

]

=[

1 · 1 + (−1) · 1 1 · 1 + (−1) · 1−1 · 1 + 1 · 1 −1 · 1 + 1 · 1

]

=[

0 00 0

]

19. AI =[

1 24 3

] [1 00 1

]

=[

1 · 1 + 2 · 0 1 · 0 + 2 · 14 · 1 + 3 · 0 4 · 0 + 3 · 1

]

=[

1 24 3

]

21.[ −1 0 7

3 −5 2

] 6−4

1

=[ −1 · 6 + 0(−4) + 7 · 1

3 · 6 + (−5)(−4) + 2 · 1]

=[

140

]

23.

−2 4

5 1−1 −3

[ 3 −6

−1 4

]

=

−2 · 3 + 4(−1) −2(−6) + 4 · 4

5 · 3 + 1(−1) 5(−6) + 1 · 4−1 · 3 + (−3)(−1) −1(−6) + (−3) · 4

=

−10 28

14 −260 −6

25.

1

−53

[ −6 5 8

0 4 −1

]

This product is not defined because the number of columnsof the first matrix, 1, is not equal to the number of rowsof the second matrix, 2.

27.

1 −4 3

0 8 0−2 −1 5

3 0 0

0 −4 00 0 1

=

3 + 0 + 0 0 + 16 + 0 0 + 0 + 3

0 + 0 + 0 0 − 32 + 0 0 + 0 + 0−6 + 0 + 0 0 + 4 + 0 0 + 0 + 5

=

3 16 3

0 −32 0−6 4 5

29. a) A =[

40 20 30]

b) 40 + 10% · 40 = 1.1(40) = 44

20 + 10% · 20 = 1.1(20) = 22

30 + 10% · 30 = 1.1(30) = 33

We write the matrix that corresponds to theseamounts.

B =[

44 22 33]

c) A + B =[

40 20 30]+[

44 22 33]

=[

84 42 63]

The entries represent the total amount of each typeof produce ordered for both weeks.

31. a) C =[

140 27 3 13 64]

P =[

180 4 11 24 662]

B =[

50 5 1 82 20]

b) C + 2P + 3B

=[

140 27 3 13 64]+[

360 8 22 48 1324]+[

150 15 3 246 60]

=[

650 50 28 307 1448]

The entries represent the total nutritional value ofone serving of chicken, 1 cup of potato salad, and 3broccoli spears.

33. a) M =

1.50 0.15 0.26 0.23 0.641.55 0.14 0.24 0.21 0.751.62 0.22 0.31 0.28 0.531.70 0.20 0.29 0.33 0.68

Copyright © 2013 Pearson Education, Inc.

Page 241: JUDITH A. PENNA -  · PDF fileSTUDENT’S SOLUTIONS MANUAL JUDITH A. PENNA Indiana University Purdue University Indianapolis COLLEGE ALGEBRA: GRAPHS AND MODELS FIFTH EDITION

2 4

2

�4 �2

�4

�6

�2

x

y

f(x) � x2 � x � 6

238 Chapter 6: Systems of Equations and Matrices

b) N =[

65 48 93 57]

c) NM =[

419.46 48.33 73.78 69.88 165.65]

d) The entries of NM represent the total cost, in dol-lars, of each item for the day’s meals.

35. a) M =

900 500

450 1000600 700

b) P =[

5 8 4]

c) PM =[

10, 500 13, 300]

d) The entries of PM represent the total profit fromeach distributor.

37. a) C =[

20 25 15]

b) CM =[

20 25 15] 900 500

450 1000600 700

=[

38, 250 45, 500]

The total production costs for the products shippedto Distributors 1 and 2 are $38,250 and $45,500,respectively.

39. 2x − 3y = 7,

x + 5y = −6

Write the coefficients on the left in a matrix. Then writethe product of that matrix and the column matrix contain-ing the variables, and set the result equal to the columnmatrix containing the constants on the right.[

2 −31 5

] [xy

]=[

7−6

]

41. x + y − 2z = 6,

3x − y + z = 7,

2x + 5y − 3z = 8

Write the coefficients on the left in a matrix. Then writethe product of that matrix and the column matrix contain-ing the variables, and set the result equal to the columnmatrix containing the constants on the right. 1 1 −2

3 −1 12 5 −3

x

yz

=

6

78

43. 3x − 2y + 4z = 17,

2x + y − 5z = 13

Write the coefficients on the left in a matrix. Then writethe product of that matrix and the column matrix contain-ing the variables, and set the result equal to the columnmatrix containing the constants on the right.[

3 −2 42 1 −5

] xyz

=

[1713

]

45. −4w + x − y + 2z = 12,

w + 2x − y − z = 0,

−w + x + 4y − 3z = 1,

2w + 3x + 5y − 7z = 9

Write the coefficients on the left in a matrix. Then writethe product of that matrix and the column matrix contain-ing the variables, and set the result equal to the columnmatrix containing the constants on the right.

−4 1 −1 21 2 −1 −1

−1 1 4 −32 3 5 −7

wxyz

=

12019

47. f(x) = x2 − x− 6

a) − b

2a= − −1

2 · 1 =12

f

(12

)=(

12

)2

− 12− 6 = −25

4

The vertex is(

12,−25

4

).

b) The axis of symmetry is x =12.

c) Since the coefficient of x2 is positive, the functionhas a minimum value. It is the second coordinateof the vertex, −25

4.

d) Plot some points and draw the graph of the function.

49. f(x) = −x2 − 3x + 2

a) − b

2a= − −3

2(−1)= −3

2

f

(− 3

2

)= −

(− 3

2

)2

− 3(− 3

2

)+ 2 =

174

The vertex is(− 3

2,174

).

b) The axis of symmetry is x = −32.

c) Since the coefficient of x2 is negative, the functionhas a maximum value. It is the second coordinateof the vertex,

174

.

d) Plot some points and draw the graph of the function.

Copyright © 2013 Pearson Education, Inc.

Page 242: JUDITH A. PENNA -  · PDF fileSTUDENT’S SOLUTIONS MANUAL JUDITH A. PENNA Indiana University Purdue University Indianapolis COLLEGE ALGEBRA: GRAPHS AND MODELS FIFTH EDITION

2 4

2

4

�4 �2

�4

�2

x

y

f(x) � �x2 � 3x � 2

Chapter 6 Mid-Chapter Mixed Review 239

51. A =[ −1 0

2 1

], B =

[1 −10 2

]

(A + B)(A − B) =[

0 −12 3

] [ −2 12 −1

]

=[ −2 1

2 −1

]A2 − B2

=[ −1 0

2 1

] [ −1 02 1

]−[

1 −10 2

] [1 −10 2

]

=[

1 00 1

]−[

1 −30 4

]

=[

0 30 −3

]Thus (A + B)(A − B) �= A2 − B2.

53. In Exercise 51 we found that (A + B)(A − B) =[ −2 12 −1

]and we also found A2 and B2.

BA =[

1 −10 2

] [ −1 02 1

]=[ −3 −1

4 2

]

AB =[ −1 0

2 1

] [1 −10 2

]=[ −1 1

2 0

]A2 + BA − AB − B2

=[

1 00 1

]+[ −3 −1

4 2

]−[ −1 1

2 0

]−

[1 −30 4

]

=[ −2 1

2 −1

]Thus (A + B)(A − B) = A2 + BA − AB − B2.

Chapter 6 Mid-Chapter Mixed Review

1. False; see page 485 in the text.

3. True; see pages 522 and 523 in the text.

5. 2x + y = −4, (1)

x = y − 5 (2)

Substitute y − 5 for x in equation (1) and solve for y.

2(y − 5) + y = −4

2y − 10 + y = −4

3y − 10 = −4

3y = 6

y = 2

Back-substitute in equation (2) to find x.

x = 2 − 5 = −3

The solution is (−3, 2).

7. 2x − 3y = 8, (1)

3x + 2y = −1 (2)

Multiply equation (1) by 2 and equation (2) by 3 and add.

4x − 6y = 16

9x + 6y = −313x = 13

x = 1

Back-substitute and solve for y.

3 · 1 + 2y = −1 Using equation (2)

3 + 2y = −1

2y = −4

y = −2

The solution is (1,−2).

9. x + 2y + 3z = 4, (1)

x − 2y + z = 2, (2)

2x − 6y + 4z = 7 (3)

Multiply equation (1) by −1 and add it to equation (2).

Multiply equation (1) by −2 and add it to equation (3).

x + 2y + 3z = 4 (1)

− 4y − 2z = −2 (4)

− 10y − 2z = −1 (5)

Multiply equation (5) by 2.

x + 2y + 3z = 4 (1)

− 4y − 2z = −2 (4)

− 20y − 4z = −2 (6)

Multiply equation (4) by −5 and add it to equation (6).

x + 2y + 3z = 4 (1)

− 4y − 2z = −2 (4)

6z = 8 (7)

Solve equation (7) for z.

6z = 8

z =43

Copyright © 2013 Pearson Education, Inc.

Page 243: JUDITH A. PENNA -  · PDF fileSTUDENT’S SOLUTIONS MANUAL JUDITH A. PENNA Indiana University Purdue University Indianapolis COLLEGE ALGEBRA: GRAPHS AND MODELS FIFTH EDITION

240 Chapter 6: Systems of Equations and Matrices

Back-substitute43

for z in equation (4) and solve for y.

−4y − 2(

43

)= −2

−4y − 83

= −2

−4y =23

y = −16

Back-substitute −16

for y and43

for z in equation (1) andsolve for x.

x + 2(− 1

6

)+ 3 · 4

3= 4

x− 13

+ 4 = 4

x +113

= 4

x =13

The solution is(

13,−1

6,43

).

11. 2x + y = 5,

3x + 2y = 6Write the augmented matrix. We will use Gauss-Jordanelimination. 2 1 5

3 2 6

Multiply row 2 by 2. 2 1 5

6 4 12

Multiply row 1 by −3 and add it to row 2. 2 1 5

0 1 −3

Multiply row 2 by −1 and add it to row 1. 2 0 8

0 1 −3

Multiply row 1 by12.

1 0 4

0 1 −3

The solution is (4,−3).

13. A + B =[

3 −15 4

]+[ −2 6

1 −3

]

=[

3 + (−2) −1 + 65 + 1 4 + (−3)

]

=[

1 56 1

]

15. 4D = 4

−2 3 0

1 −1 2−3 4 1

=

−8 12 0

4 −4 8−12 16 4

17. AB =[

3 −15 4

] [ −2 61 −3

]

=[

3(−2) + 9 + (−1) · 1 3 · 6 + (−1)(−3)5(−2) + 4 · 1 5 · 6 + 4(−3)

]

=[ −7 21

−6 18

]

19. BC

=[ −2 6

1 −3

] [ −4 1 −12 3 −2

]

=[ −2(−4)+6 ·2 −2·1 + 6·3 −2(−1) + 6(−2)

1(−4)+(−3)·2 1·1+(−3) · 3 1(−1)+(−3)(−2)

]

=[

20 16 −10−10 −8 5

]

21. A matrix equation equivalent to the given system is 2 −1 3

1 2 −13 −4 2

x

yz

=

7

35

.

23. Add a non-zero multiple of one equation to a non-zeromultiple of the other equation, where the multiples arenot opposites.

25. No; see Exercise 17 on page 526 in the text, for example.

Exercise Set 6.5

1. BA =[

7 32 1

] [1 −3

−2 7

]=[

1 00 1

]

AB =[

1 −3−2 7

] [7 32 1

]=[

1 00 1

]Since BA = I = AB, B is the inverse of A.

3. BA =

2 3 2

3 3 41 1 1

−1 −1 6

1 0 −21 0 −3

=

3 −2 0

4 −3 01 −1 1

Since BA �= I, B is not the inverse of A.

5. A =[

3 25 3

]Write the augmented matrix.[

3 2 1 05 3 0 1

]Multiply row 2 by 3.[

3 2 1 015 9 0 3

]Multiply row 1 by −5 and add it to row 2.

Copyright © 2013 Pearson Education, Inc.

Page 244: JUDITH A. PENNA -  · PDF fileSTUDENT’S SOLUTIONS MANUAL JUDITH A. PENNA Indiana University Purdue University Indianapolis COLLEGE ALGEBRA: GRAPHS AND MODELS FIFTH EDITION

Exercise Set 6.5 241

[3 2 1 00 −1 −5 3

]Multiply row 2 by 2 and add it to row 1.[

3 0 −9 60 −1 −5 3

]

Multiply row 1 by13

and row 2 by −1.[1 0 −3 20 1 5 −3

]

Then A−1 =[ −3 2

5 −3

].

7. A =[

6 94 6

]Write the augmented matrix.[

6 9 1 04 6 0 1

]Multiply row 2 by 3.[

6 9 1 012 18 0 3

]Multiply row 1 by −2 and add it to row 2.[

6 9 1 00 0 −2 3

]We cannot obtain the identity matrix on the left since thesecond row contains only zeros to the left of the verticalline. Thus, A−1 does not exist.

9. A =

3 1 0

1 1 11 −1 2

Write the augmented matrix. 3 1 0 1 0 0

1 1 1 0 1 01 −1 2 0 0 1

Interchange the first two rows. 1 1 1 0 1 0

3 1 0 1 0 01 −1 2 0 0 1

Multiply row 1 by −3 and add it to row 2. Also, multiplyrow 1 by −1 and add it to row 3. 1 1 1 0 1 0

0 −2 −3 1 −3 00 −2 1 0 −1 1

Multiply row 2 by −12.

1 1 1 0 1 0

0 132

−12

32

0

0 −2 1 0 −1 1

Multiply row 2 by −1 and add it to row 1. Also, multiplyrow 2 by 2 and add it to row 3.

1 0 −12

12

−12

0

0 132

−12

32

0

0 0 4 −1 2 1

Multiply row 3 by14.

1 0 −1

212

−12

0

0 132

−12

32

0

0 0 1 −14

12

14

Multiply row 3 by12

and add it to row 1. Also, multiply

row 3 by −32

and add it to row 2.

1 0 038

−14

18

0 1 0 −18

34

−38

0 0 1 −14

12

14

Then A−1 =

38

−14

18

−18

34

−38

−14

12

14

.

11. A =

1 −4 8

1 −3 22 −7 10

Write the augmented matrix. 1 −4 8 1 0 0

1 −3 2 0 1 02 −7 10 0 0 1

Multiply row 1 by −1 and add it to row 2. Also, multiplyrow 1 by −2 and add it to row 3. 1 −4 8 1 0 0

0 1 −6 −1 1 00 1 −6 −2 0 1

Since the second and third rows are identical left of thevertical line, it will not be possible to obtain the identitymatrix on the left side. Thus, A−1 does not exist.

13. A =[

4 −31 −2

]Write the augmented matrix.[

4 −3 1 01 −2 0 1

]Interchange the rows.[

1 −2 0 14 −3 1 0

]Multiply row 1 by −4 and add it to row 2.[

1 −2 0 10 5 1 −4

]

Multiply row 2 by15.

1 −2 0 1

0 115

−45

Multiply row 2 by 2 and add it to row 1.

Copyright © 2013 Pearson Education, Inc.

Page 245: JUDITH A. PENNA -  · PDF fileSTUDENT’S SOLUTIONS MANUAL JUDITH A. PENNA Indiana University Purdue University Indianapolis COLLEGE ALGEBRA: GRAPHS AND MODELS FIFTH EDITION

242 Chapter 6: Systems of Equations and Matrices

1 0

25

−35

0 115

−45

Then A−1 =

25

−35

15

−45

, or

[0.4 −0.60.2 −0.8

].

The x−1 key on a graphing calculator can also be usedto find A−1.

15. A =

2 3 2

3 3 4−1 −1 −1

Write the augmented matrix. 2 3 2 1 0 0

3 3 4 0 1 0−1 −1 −1 0 0 1

Interchange rows 1 and 3. −1 −1 −1 0 0 1

3 3 4 0 1 02 3 2 1 0 0

Multiply row 1 by 3 and add it to row 2. Also, multiplyrow 1 by 2 and add it to row 3. −1 −1 −1 0 0 1

0 0 1 0 1 30 1 0 1 0 2

Multiply row 1 by −1. 1 1 1 0 0 −1

0 0 1 0 1 30 1 0 1 0 2

Interchange rows 2 and 3. 1 1 1 0 0 −1

0 1 0 1 0 20 0 1 0 1 3

Multiply row 2 by −1 and add it to row 1. 1 0 1 −1 0 −3

0 1 0 1 0 20 0 1 0 1 3

Multiply row 3 by −1 and add it to row 1. 1 0 0 −1 −1 −6

0 1 0 1 0 20 0 1 0 1 3

Then A−1 =

−1 −1 −6

1 0 20 1 3

.

The x−1 key on a graphing calculator can also be usedto find A−1.

17. A =

1 2 −1

−2 0 11 −1 0

Write the augmented matrix. 1 2 −1 1 0 0

−2 0 1 0 1 01 −1 0 0 0 1

Multiply row 1 by 2 and add it to row 2. Also, multiplyrow 1 by −1 and add it to row 3. 1 2 −1 1 0 0

0 4 −1 2 1 00 −3 1 −1 0 1

Add row 3 to row 1 and also to row 2. 1 −1 0 0 0 1

0 1 0 1 1 10 −3 1 −1 0 1

Add row 2 to row 1. Also, multiply row 2 by 3 and add itto row 3. 1 0 0 1 1 2

0 1 0 1 1 10 0 1 2 3 4

Then A−1 =

1 1 2

1 1 12 3 4

.

The x−1 key on a graphing calculator can also be usedto find A−1.

19. A =

1 3 −1

0 2 −11 1 0

Write the augmented matrix. 1 3 −1 1 0 0

0 2 −1 0 1 01 1 0 0 0 1

Multiply row 1 by −1 and add it to row 3. 1 3 −1 1 0 0

0 2 −1 0 1 00 −2 1 −1 0 1

Add row 3 to row 1 and also to row 2. 1 1 0 0 0 0

0 0 0 −1 1 10 −2 1 −1 0 1

Since the second row consists only of zeros to the left of thevertical line, it will not be possible to obtain the identitymatrix on the left side. Thus, A−1 does not exist. Agraphing calculator will return an error message when wetry to find A−1.

21. A =

1 2 3 40 1 3 −50 0 1 −20 0 0 −1

Write the augmented matrix.

1 2 3 4 1 0 0 00 1 3 −5 0 1 0 00 0 1 −2 0 0 1 00 0 0 −1 0 0 0 1

Multiply row 4 by −1.

1 2 3 4 1 0 0 00 1 3 −5 0 1 0 00 0 1 −2 0 0 1 00 0 0 1 0 0 0 −1

Copyright © 2013 Pearson Education, Inc.

Page 246: JUDITH A. PENNA -  · PDF fileSTUDENT’S SOLUTIONS MANUAL JUDITH A. PENNA Indiana University Purdue University Indianapolis COLLEGE ALGEBRA: GRAPHS AND MODELS FIFTH EDITION

Exercise Set 6.5 243

Multiply row 4 by −4 and add it to row 1. Multiply row4 by 5 and add it to row 2. Also, multiply row 4 by 2 andadd it to row 3.

1 2 3 0 1 0 0 40 1 3 0 0 1 0 −50 0 1 0 0 0 1 −20 0 0 1 0 0 0 −1

Multiply row 3 by −3 and add it to row 1 and to row 2.

1 2 0 0 1 0 −3 100 1 0 0 0 1 −3 10 0 1 0 0 0 1 −20 0 0 1 0 0 0 −1

Multiply row 2 by −2 and add it to row 1.

1 0 0 0 1 −2 3 80 1 0 0 0 1 −3 10 0 1 0 0 0 1 −20 0 0 1 0 0 0 −1

Then A−1 =

1 −2 3 80 1 −3 10 0 1 −20 0 0 −1

.

The x−1 key on a graphing calculator can also be usedto find A−1.

23. A =

1 −14 7 38−1 2 1 −2

1 2 −1 −61 −2 3 6

Write the augmented matrix.

1 −14 7 38 1 0 0 0−1 2 1 −2 0 1 0 0

1 2 −1 −6 0 0 1 01 −2 3 6 0 0 0 1

Add row 1 to row 2. Also, multiply row 1 by −1 and addit to row 3 and to row 4.

1 −14 7 38 1 0 0 00 −12 8 36 1 1 0 00 16 −8 −44 −1 0 1 00 12 −4 −32 −1 0 0 1

Add row 2 to row 4.

1 −14 7 38 1 0 0 00 −12 8 36 1 1 0 00 16 −8 −44 −1 0 1 00 0 4 4 0 1 0 1

Multiply row 4 by14.

1 −14 7 38 1 0 0 00 −12 8 36 1 1 0 00 16 −8 −44 −1 0 1 0

0 0 1 1 014

014

Multiply row 4 by −38 and add it to row 1. Multiply row4 by −36 and add it to row 2. Also, multiply row 4 by 44and add it to row 3.

1 −14 −31 0 1 −192

0 −192

0 −12 −28 0 1 −8 0 −90 16 36 0 −1 11 1 11

0 0 1 1 014

014

Multiply row 3 by136

.

1 −14 −31 0 1 −192

0 −192

0 −12 −28 0 1 −8 0 −9

049

1 0 − 136

1136

136

1136

0 0 1 1 014

014

Multiply row 3 by 31 and add it to row 1. Multiply row3 by 28 and add it to row 2. Also, multiply row 3 by −1and add it to row 4.

1 −29

0 0536

− 136

3136

− 136

049

0 029

59

79

−49

049

1 0 − 136

1136

136

1136

0 −49

0 1136

− 118

− 136

− 118

Multiply row 2 by12

and add it to row 1. Also, multiplyrow 2 by −1 and add it to row 3. Add row 2 to row 4.

1 0 0 014

14

54

−14

049

0 029

59

79

−49

0 0 1 0 −14

−14

−34

34

0 0 0 114

12

34

−12

Multiply row 2 by94.

1 0 0 014

14

54

−14

0 1 0 012

54

74

−1

0 0 1 0 −14

−14

−34

34

0 0 0 114

12

34

−12

Then A−1 =

14

14

54

−14

12

54

74

−1

−14

−14

−34

34

14

12

34

−12

, or

Copyright © 2013 Pearson Education, Inc.

Page 247: JUDITH A. PENNA -  · PDF fileSTUDENT’S SOLUTIONS MANUAL JUDITH A. PENNA Indiana University Purdue University Indianapolis COLLEGE ALGEBRA: GRAPHS AND MODELS FIFTH EDITION

244 Chapter 6: Systems of Equations and Matrices

0.25 0.25 1.25 −0.250.5 1.25 1.75 −1

−0.25 −0.25 −0.75 0.750.25 0.5 0.75 −0.5

.

The x−1 key on a graphing calculator can also be usedto find A−1.

25. Write an equivalent matrix equation, AX = B.[11 37 2

] [xy

]=[ −4

5

]Then we have X = A−1B.[

xy

]=[

2 −3−7 11

] [ −45

]=[ −23

83

]The solution is (−23, 83).

27. Write an equivalent matrix equation, AX = B. 3 1 0

2 −1 21 1 1

x

yz

=

2

−55

Then we have X = A−1B. x

yz

=

19

3 1 −2

0 −3 6−3 2 5

2

−55

=

19

−9

459

=

−1

51

The solution is (−1, 5, 1).

29. 4x + 3y = 2,

x − 2y = 6

Write an equivalent matrix equation, AX = B.[4 31 −2

] [xy

]=[

26

]

Then X = A−1B =

211

311

111

− 411

[ 2

6

]=[

2−2

].

The solution is (2,−2).

31. 5x + y = 2,

3x − 2y = −4

Write an equivalent matrix equation, AX = B.

[5 13 −2

] [xy

]=

213

113

313

− 513

[ 2

−4

]=

[2

−4

]

Then X = A−1B =[

02

].

The solution is (0, 2).

33. x + z = 1,

2x + y = 3,

x − y + z = 4

Write an equivalent matrix equation, AX = B. 1 0 1

2 1 01 −1 1

x

yz

=

1

34

Then X = A−1B=

−1

212

12

1 0 −132−1

2−1

2

1

34

=

3−3−2

.

The solution is (3,−3,−2).

35. 2x + 3y + 4z = 2,

x − 4y + 3z = 2,

5x + y + z = −4

Write an equivalent matrix equation, AX = B. 2 3 4

1 −4 35 1 1

x

yz

=

2

2−4

Then X=A−1B=

− 116

1112

25112

18− 9

56− 1

56316

13112

− 11112

2

2−4

=

−1

01

.

The solution is (−1, 0, 1).

37. 2w − 3x + 4y − 5z = 0,

3w − 2x + 7y − 3z = 2,

w + x − y + z = 1,

−w − 3x − 6y + 4z = 6

Write an equivalent matrix equation, AX = B.

2 −3 4 −53 −2 7 −31 1 −1 1

−1 −3 −6 4

wxyz

=

0216

Then X = A−1B =1

203

26 11 127 9−8 −19 39 −34−37 39 −48 −5−55 47 −11 20

0216

=

1−1

01

.

The solution is (1,−1, 0, 1).

39. Familiarize. Let x = the number of hot dogs sold andy = the number of sausages.

Translate.

The total number of items sold was 145.

x + y = 145

Copyright © 2013 Pearson Education, Inc.

Page 248: JUDITH A. PENNA -  · PDF fileSTUDENT’S SOLUTIONS MANUAL JUDITH A. PENNA Indiana University Purdue University Indianapolis COLLEGE ALGEBRA: GRAPHS AND MODELS FIFTH EDITION

Exercise Set 6.5 245

The number of hot dogs sold is 45 more than the numberof sausages.

x = y + 45

We have a system of equations:

x + y = 145, x + y = 145,or

x = y + 45, x − y = 45.

Carry out. Write an equivalent matrix equation,AX = B.[

1 11 −1

] [xy

]=[

14545

]

Then X = A−1B=

12

12

12

−12

[

14545

]=[

9550

], so the

solution is (95, 50).

Check. The total number of items is 95+50, or 145. Thenumber of hot dogs, 95, is 45 more than the number ofsausages. The solution checks.

State. Kayla sold 95 hot dogs and 50 Italian sausages.

41. Familiarize. Let x, y, and z represent the prices of oneton of topsoil, mulch, and pea gravel, respectively.

Translate.

Four tons of topsoil, 3 tons of mulch, and 6 tons of peagravel costs $2825.

4x + 3y + 6z = 2825

Five tons of topsoil, 2 tons of mulch, and 5 tons of peagravel costs $2663.

5x + 2y + 5z = 2663

Pea gravel costs $17 less per ton than topsoil.

z = x− 17

We have a system of equations.

4x + 3y + 6z = 2825,

5x + 2y + 5z = 2663,

z = x− 17, or

4x + 3y + 6z = 2825,

5x + 2y + 5z = 2663,

x − z = 17

Carry out. Write an equivalent matrix equation,AX = B. 4 3 6

5 2 51 0 −1

x

yz

=

2825

266317

Then X = A−1B =

−1

5310

310

1 −1 1

−15

310

− 710

2825

266317

=

239

179222

, so the solution is (239, 179, 222).

Check. Four tons of topsoil, 3 tons of mulch, and 6 tonsof pea gravel costs 4 · $239 + 3 · $179 + 6 · $222, or $956 +$537 + $1332, or $2825. Five tons of topsoil, 2 tons ofmulch, and 5 tons of pea gravel costs 5 · $239 + 2 · $179 +5 · $222, or $1195 + $358 + $1110, or $2663. The price ofpea gravel, $222, is $17 less than the price of topsoil, $239.The solution checks.

State. The price of topsoil is $239 per ton, of mulch is$179 per ton, and of pea gravel is $222 per ton.

43. −2∣∣ 1 −6 4 −8

−2 16 −401 −8 20 −48

f(−2) = −48

45. 2x2 + x = 7

2x2 + x− 7 = 0

a = 2, b = 1, c = −7

x =−b±√

b2 − 4ac2a

=−1 ±√12 − 4 · 2 · (−7)

2 · 2 =−1 ±√

1 + 564

=−1 ±√

574

The solutions are−1 +

√57

4and

−1 −√57

4, or

−1 ±√57

4.

47.√

2x + 1 − 1 =√

2x− 4

(√

2x + 1 − 1)2 = (√

2x− 4)2 Squaringboth sides

2x + 1 − 2√

2x + 1 + 1 = 2x− 4

2x + 2 − 2√

2x + 1 = 2x− 4

2 − 2√

2x + 1 = −4 Subtracting 2x

−2√

2x + 1 = −6 Subtracting 2√2x + 1 = 3 Dividing by −2

(√

2x + 1)2 = 32 Squaring bothsides

2x + 1 = 9

2x = 8

x = 4

The number 4 checks. It is the solution.

49. f(x) = x3 − 3x2 − 6x + 8

We use synthetic division to find one factor. We first tryx− 1.1∣∣ 1 −3 −6 8

1 −2 −81 −2 −8 0

Since f(1) = 0, x− 1 is a factor of f(x). We have f(x) =(x−1)(x2−2x−8). Factoring the trinomial we get f(x) =(x− 1)(x− 4)(x + 2).

Copyright © 2013 Pearson Education, Inc.

Page 249: JUDITH A. PENNA -  · PDF fileSTUDENT’S SOLUTIONS MANUAL JUDITH A. PENNA Indiana University Purdue University Indianapolis COLLEGE ALGEBRA: GRAPHS AND MODELS FIFTH EDITION

246 Chapter 6: Systems of Equations and Matrices

51. A = [x]

Write the augmented matrix.[x 1

]Multiply by

1x

.[1

1x

]

Then A−1 exists if and only if x �= 0. A−1 =[

1x

]

53. A =

0 0 x

0 y 0z 0 0

Write the augmented matrix. 0 0 x 1 0 0

0 y 0 0 1 0z 0 0 0 0 1

Interchange row 1 and row 3. z 0 0 0 0 1

0 y 0 0 1 00 0 x 1 0 0

Multiply row 1 by1z, row 2 by

1y, and row 3 by

1x

.

1 0 0 0 01z

0 1 0 01y

0

0 0 11x

0 0

Then A−1 exists if and only if x �= 0 and y �= 0 and z �= 0,or if and only if xyz �= 0.

A−1 =

0 01z

01y

0

1x

0 0

Exercise Set 6.6

1.∣∣∣∣ 5 3−2 −4

∣∣∣∣ = 5(−4) − (−2) · 3 = −20 + 6 = −14

3.∣∣∣∣ 4 −7−2 3

∣∣∣∣ = 4 · 3 − (−2)(−7) = 12 − 14 = −2

5.∣∣∣∣ −2 −√

5−√

5 3

∣∣∣∣=−2 · 3 − (−√5)(−√

5)=−6 − 5=−11

7.∣∣∣∣x 4x x2

∣∣∣∣=x · x2 − x · 4 = x3 − 4x

9. A =

7 −4 −6

2 0 −31 2 −5

M11 is the determinant of the matrix formed by deletingthe first row and first column of A:

M11 =∣∣∣∣ 0 −32 −5

∣∣∣∣=0(−5) − 2(−3) = 0 + 6 = 6

M32 is the determinant of the matrix formed by deletingthe third row and second column of A:

M32 =∣∣∣∣ 7 −62 −3

∣∣∣∣=7(−3) − 2(−6) = −21 + 12 = −9

M22 is the determinant of the matrix formed by deletingthe second row and second column of A:

M22 =∣∣∣∣ 7 −61 −5

∣∣∣∣=7(−5) − 1(−6) = −35 + 6 = −29

11. In Exercise 9 we found that M11 = 6.

A11 = (−1)1+1M11 = 1 · 6 = 6

In Exercise 9 we found that M32 = −9.

A32 = (−1)3+2M32 = −1(−9) = 9

In Exercise 9 we found that M22 = −29.

A22 = (−1)2+2(−29) = 1(−29) = −29

13. A =

7 −4 −6

2 0 −31 2 −5

|A|= 2A21 + 0A22 + (−3)A23

= 2(−1)2+1

∣∣∣∣−4 −62 −5

∣∣∣∣+0+(−3)(−1)2+3

∣∣∣∣ 7 −41 2

∣∣∣∣= 2(−1)[−4(−5) − 2(−6)] + 0+

(−3)(−1)[7 · 2 − 1(−4)]

= −2(32) + 0 + 3(18) = −64 + 0 + 54

= −10

15. A =

7 −4 −6

2 0 −31 2 −5

|A|= −6A13 + (−3)A23 + (−5)A33

= −6(−1)1+3

∣∣∣∣ 2 01 2

∣∣∣∣+ (−3)(−1)2+3

∣∣∣∣ 7 −41 2

∣∣∣∣+(−5)(−1)3+3

∣∣∣∣ 7 −42 0

∣∣∣∣= −6 · 1(2 · 2 − 1 · 0) + (−3)(−1)[7 · 2 − 1(−4)]+

−5 · 1(7 · 0 − 2(−4))

= −6(4) + 3(18) − 5(8) = −24 + 54 − 40

= −10

17. Enter A and then use the determinant operation, det, fromthe MATRIX MATH menu to evaluate |A|. We find that|A| = −10.

19. A =

1 0 0 −24 1 0 05 6 7 8

−2 −3 −1 0

M41 is the determinant of the matrix formed by deletingthe fourth row and the first column of A.

Copyright © 2013 Pearson Education, Inc.

Page 250: JUDITH A. PENNA -  · PDF fileSTUDENT’S SOLUTIONS MANUAL JUDITH A. PENNA Indiana University Purdue University Indianapolis COLLEGE ALGEBRA: GRAPHS AND MODELS FIFTH EDITION

Exercise Set 6.6 247

M41 =

0 0 −2

1 0 06 7 8

We will expand M41 across the first row.

M41 = 0(−1)1+1

∣∣∣∣ 0 07 8

∣∣∣∣+ 0(−1)1+2

∣∣∣∣ 1 06 8

∣∣∣∣+(−2)(−1)3+1

∣∣∣∣ 1 06 7

∣∣∣∣= 0 + 0 + (−2)(1)(1 · 7 − 6 · 0)

= 0 + 0 − 2(7) = −14

M33 is the determinant of the matrix formed by deletingthe third row and the third column of A.

M33 =

1 0 −2

4 1 0−2 −3 0

We will expand M33 down the third column.

M33 = −2(−1)1+3

∣∣∣∣ 4 1−2 −3

∣∣∣∣+0(−1)2+3

∣∣∣∣ 1 0−2 −3

∣∣∣∣+0(−1)3+3

∣∣∣∣ 1 04 1

∣∣∣∣= −2(1)[4(−3) − (−2)(1)] + 0 + 0

= −2(−10) + 0 + 0 = 20

21. A =

1 0 0 −24 1 0 05 6 7 8

−2 −3 −1 0

A24 = (−1)2+4M24 = 1 ·M24 = M24

=

∣∣∣∣∣∣1 0 05 6 7−2 −3 −1

∣∣∣∣∣∣We will expand across the first row.∣∣∣∣∣∣

1 0 05 6 7−2 −3 −1

∣∣∣∣∣∣= 1(−1)1+1

∣∣∣∣ 6 7−3 −1

∣∣∣∣+ 0(−1)1+2

∣∣∣∣ 5 7−2 −1

∣∣∣∣+0(−1)1+3

∣∣∣∣ 5 6−2 −3

∣∣∣∣= 1 · 1[6(−1) − (−3)(7)] + 0 + 0

= 1(15) = 15

A43 = (−1)4+3M43 = −1 ·M43

= −1 ·∣∣∣∣∣∣1 0 −24 1 05 6 8

∣∣∣∣∣∣We will expand across the first row.

−1 ·∣∣∣∣∣∣1 0 −24 1 05 6 8

∣∣∣∣∣∣= −1

[1(−1)1+1

∣∣∣∣1 06 8

∣∣∣∣+ 0(−1)1+2

∣∣∣∣4 05 8

∣∣∣∣+(−2)(−1)1+3

∣∣∣∣ 4 15 6

∣∣∣∣]

= −1[1·1(1·8−6 · 0) + 0 + (−2)·1(4·6−5·1)]

= −1[1(8) + 0 − 2(19)]

= −1(8 − 38) = −1(−30)

= 30

23. A =

1 0 0 −24 1 0 05 6 7 8

−2 −3 −1 0

|A| = 0 ·A13 + 0 ·A23 + 7A33 + (−1)A43

= 7A33 −A43

= 7(−1)3+3

∣∣∣∣∣∣1 0 −24 1 0−2 −3 0

∣∣∣∣∣∣−

(−1)4+3

∣∣∣∣∣∣1 0 −24 1 05 6 8

∣∣∣∣∣∣We will expand each determinant down the third column.We have:

7[− 2(−1)1+3

∣∣∣∣ 4 1−2 3

∣∣∣∣+0 + 0]+

[− 2(−1)1+3

∣∣∣∣ 4 15 6

∣∣∣∣+ 0 + 8(−1)3+3

∣∣∣∣ 1 04 1

∣∣∣∣]

= 7[−2(4(−3) − (−2) · 1)] + [−2(4 · 6 − 5 · 1)+

8(1 · 1 − 4 · 0)]

= 7[−2(−10)] + [−2(19) + 8(1)]

= 7(20) + (−30) = 140 − 30

= 110

25. We will expand across the first row. We could have chosenany other row or column just as well.∣∣∣ 3 1 2

∣∣∣∣∣∣ −2 3 1∣∣∣∣∣∣ 3 4 −6∣∣∣

= 3(−1)1+1

∣∣∣∣ 3 14 −6

∣∣∣∣+ 1 · (−1)1+2

∣∣∣∣−2 13 −6

∣∣∣∣+2(−1)1+3

∣∣∣∣−2 33 4

∣∣∣∣= 3·1[3(−6)−4 · 1]+1(−1)[−2(−6) − 3 · 1]+

2 · 1(−2 · 4−3 · 3)

= 3(−22) − (9) + 2(−17)

= −109

Copyright © 2013 Pearson Education, Inc.

Page 251: JUDITH A. PENNA -  · PDF fileSTUDENT’S SOLUTIONS MANUAL JUDITH A. PENNA Indiana University Purdue University Indianapolis COLLEGE ALGEBRA: GRAPHS AND MODELS FIFTH EDITION

248 Chapter 6: Systems of Equations and Matrices

27. We will expand down the second column. We could havechosen any other row or column just as well.∣∣∣ x 0 −1

∣∣∣∣∣∣ 2 x x2∣∣∣∣∣∣ −3 x 1∣∣∣

= 0(−1)1+2

∣∣∣∣ 2 x2

−3 1

∣∣∣∣+ x(−1)2+2

∣∣∣∣ x −1−3 1

∣∣∣∣+x(−1)3+2

∣∣∣∣x −12 x2

∣∣∣∣= 0(−1)[2·1−(−3)x2]+ x · 1[x · 1−(−3)(−1)]+

x(−1)[x · x2 − 2(−1)]

= 0 + x(x− 3) − x(x3 + 2)

= x2 − 3x− x4 − 2x = −x4 + x2 − 5x

29. −2x + 4y = 3,

3x − 7y = 1

D =∣∣∣∣−2 4

3 −7

∣∣∣∣ = −2(−7) − 3 · 4 = 14 − 12 = 2

Dx =∣∣∣∣ 3 41 −7

∣∣∣∣ = 3(−7) − 1 · 4 = −21 − 4 = −25

Dy =∣∣∣∣−2 3

3 1

∣∣∣∣ = −2 · 1 − 3 · 3 = −2 − 9 = −11

x =Dx

D=

−252

= −252

y =Dy

D=

−112

= −112

The solution is(− 25

2,−11

2

).

31. 2x − y = 5,

x − 2y = 1

D =∣∣∣∣ 2 −11 −2

∣∣∣∣ = 2(−2) − 1(−1) = −4 + 1 = −3

Dx =∣∣∣∣ 5 −11 −2

∣∣∣∣ = 5(−2) − 1(−1) = −10 + 1 = −9

Dy =∣∣∣∣ 2 51 1

∣∣∣∣ = 2 · 1 − 1 · 5 = 2 − 5 = −3

x =Dx

D=

−9−3

= 3

y =Dy

D=

−3−3

= 1

The solution is (3, 1).

33. 2x + 9y = −2,

4x − 3y = 3

D =∣∣∣∣ 2 94 −3

∣∣∣∣ = 2(−3) − 4 · 9 = −6 − 36 = −42

Dx =∣∣∣∣−2 9

3 −3

∣∣∣∣ = −2(−3) − 3 · 9 = 6 − 27 = −21

Dy =∣∣∣∣ 2 −24 3

∣∣∣∣ = 2 · 3 − 4(−2) = 6 + 8 = 14

x =Dx

D=

−21−42

=12

y =Dy

D=

14−42

= −13

The solution is(

12,−1

3

).

35. 2x + 5y = 7,

3x − 2y = 1

D =∣∣∣∣ 2 53 −2

∣∣∣∣ = 2(−2) − 3 · 5 = −4 − 15 = −19

Dx =∣∣∣∣ 7 51 −2

∣∣∣∣ = 7(−2) − 1 · 5 = −14 − 5 = −19

Dy =∣∣∣∣ 2 73 1

∣∣∣∣ = 2 · 1 − 3 · 7 = 2 − 21 = −19

x =Dx

D=

−19−19

= 1

y =Dy

D=

−19−19

= 1

The solution is (1, 1).

37. 3x + 2y − z = 4,

3x − 2y + z = 5,

4x − 5y − z = −1

D =

∣∣∣∣∣∣3 2 −13 −2 14 −5 −1

∣∣∣∣∣∣ = 42

Dx =

∣∣∣∣∣∣4 2 −15 −2 1−1 −5 −1

∣∣∣∣∣∣ = 63

Dy =

∣∣∣∣∣∣3 4 −13 5 14 −1 −1

∣∣∣∣∣∣ = 39

Dz =

∣∣∣∣∣∣3 2 43 −2 54 −5 −1

∣∣∣∣∣∣ = 99

x =Dx

D=

6342

=32

y =Dy

D=

3942

=1314

z =Dz

D=

9942

=3314

The solution is(

32,1314

,3314

).

(Note that we could have used Cramer’s rule to find onlytwo of the values and then used substitution to find theremaining value.)

Copyright © 2013 Pearson Education, Inc.

Page 252: JUDITH A. PENNA -  · PDF fileSTUDENT’S SOLUTIONS MANUAL JUDITH A. PENNA Indiana University Purdue University Indianapolis COLLEGE ALGEBRA: GRAPHS AND MODELS FIFTH EDITION

x

y

1–1–3 32 54–2–4–5–1

–2

–3

–4

1

2

3

4

5

–5

f (x ) = 3 x + 2

Exercise Set 6.6 249

39. 3x + 5y − z = −2,

x − 4y + 2z = 13,

2x + 4y + 3z = 1

D =

∣∣∣∣∣∣3 5 −11 −4 22 4 3

∣∣∣∣∣∣ = −67

Dx =

∣∣∣∣∣∣−2 5 −113 −4 21 4 3

∣∣∣∣∣∣ = −201

Dy =

∣∣∣∣∣∣3 −2 −11 13 22 1 3

∣∣∣∣∣∣ = 134

Dz =

∣∣∣∣∣∣3 5 −21 −4 132 4 1

∣∣∣∣∣∣ = −67

x =Dx

D=

−201−67

= 3

y =Dy

D=

134−67

= −2

z =Dz

D=

−67−67

= 1

The solution is (3,−2, 1).

(Note that we could have used Cramer’s rule to find onlytwo of the values and then used substitution to find theremaining value.)

41. x − 3y − 7z = 6,

2x + 3y + z = 9,

4x + y = 7

D =

∣∣∣∣∣∣1 −3 −72 3 14 1 0

∣∣∣∣∣∣ = 57

Dx =

∣∣∣∣∣∣6 −3 −79 3 17 1 0

∣∣∣∣∣∣ = 57

Dy =

∣∣∣∣∣∣1 6 −72 9 14 7 0

∣∣∣∣∣∣ = 171

Dz =

∣∣∣∣∣∣1 −3 62 3 94 1 7

∣∣∣∣∣∣ = −114

x =Dx

D=

5757

= 1

y =Dy

D=

17157

= 3

z =Dz

D=

−11457

= −2

The solution is (1, 3,−2).

(Note that we could have used Cramer’s rule to find onlytwo of the values and then used substitution to find theremaining value.)

43. 6y + 6z = −1,

8x + 6z = −1,

4x + 9y = 8

D =

∣∣∣∣∣∣0 6 68 0 64 9 0

∣∣∣∣∣∣ = 576

Dx =

∣∣∣∣∣∣−1 6 6−1 0 68 9 0

∣∣∣∣∣∣ = 288

Dy =

∣∣∣∣∣∣0 −1 68 −1 64 8 0

∣∣∣∣∣∣ = 384

Dz =

∣∣∣∣∣∣0 6 −18 0 −14 9 8

∣∣∣∣∣∣ = −480

x =Dx

D=

288576

=12

y =Dy

D=

384576

=23

z =Dz

D=

−480576

= −56

The solution is(

12,23,−5

6

).

(Note that we could have used Cramer’s rule to find onlytwo of the values and then used substitution to find theremaining value.)

45. The graph of f(x) = 3x+2 is shown below. Since it passesthe horizontal-line test, the function is one-to-one.

We find a formula for f−1(x).

Replace f(x) with y: y = 3x + 2

Interchange x and y: x = 3y + 2

Solve for y: y =x− 2

3

Replace y with f−1(x): f−1(x) =x− 2

3

Copyright © 2013 Pearson Education, Inc.

Page 253: JUDITH A. PENNA -  · PDF fileSTUDENT’S SOLUTIONS MANUAL JUDITH A. PENNA Indiana University Purdue University Indianapolis COLLEGE ALGEBRA: GRAPHS AND MODELS FIFTH EDITION

x

y

1–1–3 32 54–2–4–5–1

–2

–3

–4

1

2

3

4

5

–5

f (x ) = | x | + 3

4

2

�2

�4

42�2�4 x

y

y � 2x

4

2

�2

�4

42�2�4 x

y

y � x � 0

250 Chapter 6: Systems of Equations and Matrices

47. The graph of f(x) = |x| + 3 is shown below. It fails thehorizontal-line test, so it is not one-to-one.

49. (3 − 4i) − (−2 − i) = 3 − 4i + 2 + i =

(3 + 2) + (−4 + 1)i = 5 − 3i

51. (1 − 2i)(6 + 2i) = 6 + 2i− 12i− 4i2 =

6 + 2i− 12i + 4 = 10 − 10i

53.∣∣∣∣ y 23 y

∣∣∣∣ = y

y2 − 6 = y

y2 − y − 6 = 0

(y − 3)(y + 2) = 0

y − 3 = 0 or y + 2 = 0

y = 3 or y = −2

The solutions are 3 and −2.

55.∣∣∣∣∣∣2 x 11 2 −13 4 −2

∣∣∣∣∣∣ = −6

−x− 2 = −6 Evaluating thedeterminant

−x = −4

x = 4

The solution is 4.

57. Answers may vary.∣∣∣∣ a b−b a

∣∣∣∣59. Answers may vary.∣∣∣∣ 2πr 2πr

−h r

∣∣∣∣Exercise Set 6.7

1. Graph (f) is the graph of y > x.

3. Graph (h) is the graph of y ≤ x− 3.

5. Graph (g) is the graph of 2x + y < 4.

7. Graph (b) is the graph of 2x− 5y > 10.

9. Graph: y > 2x

1. We first graph the related equation y = 2x. Wedraw the line dashed since the inequality symbol is>.

2. To determine which half-plane to shade, test a pointnot on the line. We try (1, 1) and substitute:

y > 2x

1 ? 2 · 11∣∣∣ 2 FALSE

Since 1 > 2 is false, (1, 1) is not a solution, nor areany points in the half-plane containing (1, 1). Thepoints in the opposite half-plane are solutions, so weshade that half-plane and obtain the graph.

11. Graph: y + x ≥ 0

1. First graph the related equation y + x = 0. Drawthe line solid since the inequality is ≥.

2. Next determine which half-plane to shade by testinga point not on the line. Here we use (2, 2) as a check.

y + x ≥ 0

2 + 2 ? 04∣∣∣ 0 TRUE

Since 4 ≥ 0 is true, (2, 2) is a solution. Thus shadethe half-plane containing (2, 2).

13. Graph: y > x− 3

1. We first graph the related equation y = x−3. Drawthe line dashed since the inequality symbol is >.

2. To determine which half-plane to shade, test a pointnot on the line. We try (0, 0).

y > x− 3

0 ? 0 − 30∣∣∣ −3 TRUE

Since 0 > −3 is true, (0, 0) is a solution. Thus weshade the half-plane containing (0, 0).

Copyright © 2013 Pearson Education, Inc.

Page 254: JUDITH A. PENNA -  · PDF fileSTUDENT’S SOLUTIONS MANUAL JUDITH A. PENNA Indiana University Purdue University Indianapolis COLLEGE ALGEBRA: GRAPHS AND MODELS FIFTH EDITION

4

2

�2

4�2�4 x

y

y � x � 3

2

�2

�4

42�2�4 x

y

x � y � 4

4

2

�2

4�2�4 x

y

3x � 2y � 6

4

�2

�4

2�2�4 x

y

3y � 2x � 6

4

2

�4

�2

42�2�4 x

y

3x � 2 � 5x � y

Exercise Set 6.7 251

15. Graph: x + y < 4

1. First graph the related equation x + y = 4. Drawthe line dashed since the inequality is <.

2. To determine which half-plane to shade, test a pointnot on the line. We try (0, 0).

x + y < 4

0 + 0 ? 40∣∣∣ 4 TRUE

Since 0 < 4 is true, (0, 0) is a solution. Thus shadethe half-plane containing (0, 0).

17. Graph: 3x− 2y ≤ 6

1. First graph the related equation 3x− 2y = 6. Drawthe line solid since the inequality is ≤.

2. To determine which half-plane to shade, test a pointnot on the line. We try (0, 0).

3x− 2y ≤ 6

3(0) − 2(0) ? 60∣∣∣ 6 TRUE

Since 0 ≤ 6 is true, (0, 0) is a solution. Thus shadethe half-plane containing (0, 0).

19. Graph: 3y + 2x ≥ 6

1. First graph the related equation 3y + 2x = 6. Drawthe line solid since the inequality is ≥.

2. To determine which half-plane to shade, test a pointnot on the line. We try (0, 0).

3y + 2x ≥ 6

3 · 0 + 2 · 0 ? 60∣∣∣ 6 FALSE

Since 0 ≥ 6 is false, (0, 0) is not a solution. Weshade the half-plane which does not contain (0, 0).

21. Graph: 3x− 2 ≤ 5x + y

−2 ≤ 2x + y Adding −3x1. First graph the related equation 2x+y = −2. Draw

the line solid since the inequality is ≤.

2. To determine which half-plane to shade, test a pointnot on the line. We try (0, 0).

2x + y ≥ −2

2(0) + 0 ? −20∣∣∣ −2 TRUE

Since 0 ≥ −2 is true, (0, 0) is a solution. Thus shadethe half-plane containing the origin.

23. Graph: x < −4

1. We first graph the related equation x = −4. Drawthe line dashed since the inequality is <.

2. To determine which half-plane to shade, test a pointnot on the line. We try (0, 0).

x < −4

0 ? −4 FALSE

Since 0 < −4 is false, (0, 0) is not a solution. Thus,we shade the half-plane which does not contain theorigin.

Copyright © 2013 Pearson Education, Inc.

Page 255: JUDITH A. PENNA -  · PDF fileSTUDENT’S SOLUTIONS MANUAL JUDITH A. PENNA Indiana University Purdue University Indianapolis COLLEGE ALGEBRA: GRAPHS AND MODELS FIFTH EDITION

4

2

�2

�4

42�2 x

y

x � �4

y � 58

4

�4

�8

84�4�8 x

y

x

y

1–1–3 32 5 64–2–4–5–6–1

–2

–3

–4

1

2

3

4

5

6

–5

–6

—4 < y

x

y

1–1–3 32 5 64–2–4–5–6–1

–2

–3

–4

1

2

3

4

5

6

–5

–6

y < —1

4

2

�2

42�2�4 x

y

�4 � y � �1

4

2

�2

�4

42�2�4 x

y

y � �x�

252 Chapter 6: Systems of Equations and Matrices

25. Graph: y ≥ 5

1. First we graph the related equation y = 5. Drawthe line solid since the inequality is ≥.

2. To determine which half-plane to shade we test apoint not on the line. We try (0, 0).

y ≥ 5

0 ? 5 FALSE

Since 0 ≥ 5 is false, (0, 0) is not a solution. Weshade the half-plane that does not contain (0, 0).

27. Graph: −4 < y < −1

This is a conjunction of two inequalities

−4 < y and y < −1.

We can graph −4 < y and y < −1 separately and thengraph the intersection, or region in both solution sets.

29. Graph: y ≥ |x|1. Graph the related equation y = |x|. Draw the line

solid since the inequality symbol is ≥.

2. To determine the region to shade, observe that thesolution set consists of all ordered pairs (x, y) wherethe second coordinate is greater than or equal to theabsolute value of the first coordinate. We see thatthe solutions are the points on or above the graphof y = |x|.

31. Graph (f) is the correct graph.

33. Graph (a) is the correct graph.

35. Graph (b) is the correct graph.

37. First we find the related equations. One line goes through(0, 4) and (4, 0). We find its slope:

m =0 − 44 − 0

=−44

= −1

This line has slope −1 and y-intercept (0, 4), so its equationis y = −x + 4.

The other line goes through (0, 0) and (1, 3). We find theslope.

m =3 − 01 − 0

= 3

This line has slope 3 and y-intercept (0, 0), so its equationis y = 3x + 0, or y = 3x.

Observing the shading on the graph and the fact that thelines are solid, we can write the system of inqualities as

y ≤ −x + 4,

y ≤ 3x.Answers may vary.

39. The equation of the vertical line is x = 2 and the equationof the horizontal line is y = −1. The lines are dashed andthe shaded area is to the left of the vertical line and abovethe horizontal line, so the system of inequalities can bewritten

x < 2,

y > −1.

Copyright © 2013 Pearson Education, Inc.

Page 256: JUDITH A. PENNA -  · PDF fileSTUDENT’S SOLUTIONS MANUAL JUDITH A. PENNA Indiana University Purdue University Indianapolis COLLEGE ALGEBRA: GRAPHS AND MODELS FIFTH EDITION

4

2

�2

�4

42�2�4 x

y

w, w��

(2, 2)

2 4

2

4

�4 �2

�4

�2

x

y

4

2

�2

�4

42�2�4 x

y

(1, �3)

4

2

�2

�8

�6

�4

42�2�4 x

y

(3, �7)

4

2

�2

�4

4�2�4 x

y

w, �q��

Exercise Set 6.7 253

41. First we find the related equations. One line goes through(0, 3) and (3, 0). We find its slope:

m =0 − 33 − 0

=−33

= −1

This line has slope −1 and y-intercept (0, 3), so its equationis y = −x + 3.

The other line goes through (0, 1) and (1, 2). We find itsslope:

m =2 − 11 − 0

=11

= 1

This line has slope 1 and y-intercept (0, 1), so its equationis y = x + 1.

Observe that both lines are solid and that the shading liesbelow both lines, to the right of the y-axis, and above thex-axis. We can write this system of inequalities as

y ≤ −x + 3,

y ≤ x + 1,

x ≥ 0,

y ≥ 0.

43. Graph: y ≤ x,

y ≥ 3 − x

We graph the related equations y = x and y = 3 − xusing solid lines and determine the solution set for eachinequality. Then we shade the region common to bothsolution sets.

We find the vertex(

32,32

)by solving the system

y = x,

y = 3 − x.

45. Graph: y ≥ x,

y ≤ 4 − x

We graph the related equations y = x and y = 4 − xusing solid lines and determine the solution set for eachinequality. Then we shade the region common to bothsolution sets.

47. Graph: y ≥ −3,

x ≥ 1

We graph the related equations y = −3 and x = 1 usingsolid lines and determine the solution set for each inequal-ity. Then we shade the region common to both solutionsets.

We find the vertex (1,−3) by solving the system

y = −3,

x = 1.

49. Graph: x ≤ 3,

y ≥ 2 − 3x

We graph the related equations x = 3 and y = 2 − 3xusing solid lines and determine the half-plane containingthe solution set for each inequality. Then we shade theregion common to both solution sets.

We find the vertex (3,−7) by solving the system

x = 3,

y = 2 − 3x.

51. Graph: x + y ≤ 1,

x− y ≤ 2

We graph the related equations x + y = 1 and x − y = 2using solid lines and determine the half-plane containingthe solution set for each inequality. Then we shade theregion common to both solution sets.

We find the vertex(

32,−1

2

)by solving the system

Copyright © 2013 Pearson Education, Inc.

Page 257: JUDITH A. PENNA -  · PDF fileSTUDENT’S SOLUTIONS MANUAL JUDITH A. PENNA Indiana University Purdue University Indianapolis COLLEGE ALGEBRA: GRAPHS AND MODELS FIFTH EDITION

4

2

�2

�4

42�2�4 x

y

�¢, ∞��

64

2

�4

2�2 x

y

(0, 4)

(6, 4)(4, 2)

(0, 0)

(�12, 0)

(�4, �6)

(0, �3)

8

4

84�4�8 x

y

44 6

4

2

�2

�4

42�2 x

y

3, e1, #

3, !

�1, 256 ���

����

254 Chapter 6: Systems of Equations and Matrices

x + y = 1,

x− y = 2.

53. Graph: 2y − x ≤ 2,

y + 3x ≥ −1

We graph the related equations 2y−x = 2 and y+3x = −1using solid lines and determine the half-plane containingthe solution set for each inequality. Then we shade theregion common to both solution sets.

We find the vertex(− 4

7,57

)by solving the system

2y − x = 2,

y + 3x = −1.

55. Graph: x− y ≤ 2,

x + 2y ≥ 8,

y − 4 ≤ 0

We graph the related equations x − y = 2, x + 2y = 8,and y = 4 using solid lines and determine the half-planecontaining the solution set for each inequality. Then weshade the region common to all three solution sets.

We find the vertex (0, 4) by solving the system

x + 2y = 8,

y = 4.

We find the vertex (6, 4) by solving the system

x− y = 2,

y = 4.

We find the vertex (4, 2) by solving the system

x− y = 2,

x + 2y = 8.

57. Graph: 4x− 3y ≥ −12,

4x + 3y ≥ −36,

y ≤ 0,

x ≤ 0

Shade the intersection of the graphs of the four inequali-ties.

We find the vertex (−12, 0) by solving the system

4y + 3x = −36,

y = 0.

We find the vertex (0, 0) by solving the system

y = 0,

x = 0.

We find the vertex (0,−3) by solving the system

4y − 3x = −12,

x = 0.

We find the vertex (−4,−6) by solving the system

4y − 3x = −12,

4y + 3x = −36.

59. Graph: 3x + 4y ≥ 12,

5x + 6y ≤ 30,

1 ≤ x ≤ 3

Shade the intersection of the graphs of the given inequali-ties.

We find the vertex(

1,256

)by solving the system

5x + 6y = 30,

x = 1.

We find the vertex(

3,52

)by solving the system

5x + 6y = 30,

x = 3.

We find the vertex(

3,34

)by solving the system

3x + 4y = 12,

x = 3.

We find the vertex(

1,94

)by solving the system

3x + 4y = 12,

x = 1.

Copyright © 2013 Pearson Education, Inc.

Page 258: JUDITH A. PENNA -  · PDF fileSTUDENT’S SOLUTIONS MANUAL JUDITH A. PENNA Indiana University Purdue University Indianapolis COLLEGE ALGEBRA: GRAPHS AND MODELS FIFTH EDITION

B

x

y

A E

C

D

x

y

A

B

D

C

(0, 0)

(100, 100)

(150, 0)

(0, 200)

100

100

200

300

200 300 x

y

Exercise Set 6.7 255

61. Find the maximum and minimum values of

P = 17x− 3y + 60, subject to

6x + 8y ≤ 48,

0 ≤ y ≤ 4,

0 ≤ x ≤ 7.

Graph the system of inequalities and determine the ver-tices.

Vertex A: (0, 0)

Vertex B:We solve the system x = 0 and y = 4. The coordi-nates of point B are (0, 4).

Vertex C:We solve the system 6x + 8y = 48 and y = 4. The

coordinates of point C are(

83, 4)

.

Vertex D:We solve the system 6x + 8y = 48 and x = 7. The

coordinates of point D are(

7,34

).

Vertex E:We solve the system x = 7 and y = 0. The coordi-nates of point E are (7, 0).

Evaluate the objective function P at each vertex.

Vertex P = 17x− 3y + 60A(0, 0) 17 · 0 − 3 · 0 + 60 = 60

B(0, 4) 17 · 0 − 3 · 4 + 60 = 48

C

(83, 4)

17 · 83− 3 · 4 + 60 = 66

23

D

(7,

34

)17 · 7 − 3 · 3

4+ 60 = 176

34

E(7, 0) 17 · 7 − 3 · 0 + 60 = 179

The maximum value of P is 179 when x = 7 and y = 0.

The minimum value of P is 48 when x = 0 and y = 4.

63. Find the maximum and minimum values of

F = 5x + 36y, subject to

5x + 3y ≤ 34,

3x + 5y ≤ 30,

x ≥ 0,

y ≥ 0.

Graph the system of inequalities and find the vertices.

Vertex A: (0, 0)

Vertex B:We solve the system 3x + 5y = 30 and x = 0. Thecoordinates of point B are (0, 6).

Vertex C:We solve the system 5x+3y = 34 and 3x+5y = 30.The coordinates of point C are (5, 3).

Vertex D:We solve the system 5x + 3y = 34 and y = 0. The

coordinates of point D are(

345, 0)

.

Evaluate the objective function F at each vertex.

Vertex F = 5x + 36yA(0, 0) 5 · 0 + 36 · 0+ = 0

B(0, 6) 5 · 0 + 36 · 6 = 216

C(5, 3) 5 · 5 + 39 · 3 = 133

D

(345, 0)

5 · 345

+ 36 · 0 = 34

The maximum value of F is 216 when x = 0 and y = 6.

The minimum value of F is 0 when x = 0 and y = 0.

65. Let x = the number of jumbo biscuits and y = the numberof regular biscuits to be made per day. The income I isgiven by

I = 0.80x + 0.50y

subject to the constraints

x + y ≤ 200,

2x + y ≤ 300,

x ≥ 0,

y ≥ 0.

We graph the system of inequalities, determine the ver-tices, and find the value if I at each vertex.

Copyright © 2013 Pearson Education, Inc.

Page 259: JUDITH A. PENNA -  · PDF fileSTUDENT’S SOLUTIONS MANUAL JUDITH A. PENNA Indiana University Purdue University Indianapolis COLLEGE ALGEBRA: GRAPHS AND MODELS FIFTH EDITION

x

y

100

100 200 300 400

200

300

400

(100, 150)

(250, 150)

(100, 300)

1

2

4

6

8

10

12

2 3 x

y

D

C

A

B

10,000

10,000

20,000

30,000

40,000

20,000 30,000 40,000 x

y

(6000, 0)

(6000, 30,000)

(10,000, 30,000)

(22,000, 18,000)

(22,000, 0)

256 Chapter 6: Systems of Equations and Matrices

Vertex I = 0.10x + 0.08y(0, 0) 0.80(0) + 0.50(0) = 0

(0, 200) 0.80(0) + 0.50(200) = 100

(100, 100) 0.80(100) + 0.50(100) = 130

(150, 0) 0.80(150) + 0.50(0) = 120

The company will have a maximum income of $130 when100 of each type of biscuit are made.

67. Let x = the number of units of lumber and y = the numberof units of plywood produced per week. The profit P isgiven by

P = 20x + 30y

subject to the constraints

x + y ≤ 400,

x ≥ 100,

y ≥ 150.

We graph the system of inequalities, determine the verticesand find the value of P at each vertex.

Vertex P = 20x + 30y(100, 150) 20 · 100 + 30 · 150 = 6500

(100, 300) 20 · 100 + 30 · 300 = 11, 000

(250, 150) 20 · 250 + 30 · 150 = 9500

The maximum profit of $11,000 is achieved by producing100 units of lumber and 300 units of plywood.

69. Let x = the number of sacks of soybean meal to be usedand y = the number of sacks of oats. The minimum costis given by

C = 15x + 5y

subject to the constraints

50x + 15y ≥ 120,

8x + 5y ≥ 24,

5x + y ≥ 10,

x ≥ 0,

y ≥ 0.

Graph the system of inequalities, determine the vertices,and find the value of C at each vertex.

Vertex C = 15x + 5yA(0, 10) 15 · 0 + 5 · 10 = 50

B

(65, 4)

15 · 65

+ 5 · 4 = 38

C

(2413

,2413

)15 · 24

13+ 5 · 24

13= 36

1213

D(3, 0) 15 · 3 + 5 · 0 = 45

The minimum cost of $361213

is achieved by using2413

, or

11113

sacks of soybean meal and2413

, or 11113

sacks of oats.

71. Let x = the amount invested in corporate bonds and y =the amount invested in municipal bonds. The income I isgiven by

I = 0.08x + 0.075y

subject to the constraintsx + y ≤ 40, 000,

6000 ≤ x ≤ 22, 000,

0 ≤ y ≤ 30, 000.We graph the system of inequalities, determine the ver-tices, and find the value of I at each vertex.

Vertex I = 0.08x + 0.075y(6000, 0) 480

(6000, 30, 000) 2730

(10, 000, 30, 000) 3050

(22, 000, 18, 000) 3110

(22, 000, 0) 1760

The maximum income of $3110 occurs when $22,000 isinvested in corporate bonds and $18,000 is invested in mu-nicipal bonds.

73. Let x = the number of P1 airplanes and y = the num-ber of P2 airplanes to be used. The operating cost C, inthousands of dollars, is given by

Copyright © 2013 Pearson Education, Inc.

Page 260: JUDITH A. PENNA -  · PDF fileSTUDENT’S SOLUTIONS MANUAL JUDITH A. PENNA Indiana University Purdue University Indianapolis COLLEGE ALGEBRA: GRAPHS AND MODELS FIFTH EDITION

y

40

50

60

30

20

10

x30 40 502010

(50, 0)

(30, 10)

(6, 42)

(0, 60)

x

y

(0, 0)

(0, 5)

(4, 0)

(2, 4)

x5 63 421

y

5

6

4

3

2

1

(1.5, 3)

(0, 6)

(6, 0)

250

1000

2000

3000

500 750 1000 x

y

(0, 525)

(550, 250)

(600, 0)

(0, 0)

Exercise Set 6.7 257

C = 12x + 10y

subject to the constraints

40x + 80y ≥ 2000,

40x + 30y ≥ 1500,

120x + 40y ≥ 2400,

x ≥ 0,

y ≥ 0.

Graph the system of inequalities, determine the vertices,and find the value of C at each vertex.

Vertex C = 12x + 10y(0, 60) 12 · 0 + 10 · 60 = 600

(6, 42) 12 · 6 + 10 · 42 = 492

(30, 10) 12 · 30 + 10 · 10 = 460

(50, 0) 12 · 50 + 10 · 0 = 600

The minimum cost of $460 thousand is achieved using 30P1’s and 10 P2’s.

75. Let x = the number of knit suits and y = the number ofworsted suits made. The profit is given by

P = 34x + 31y

subject to

2x + 4y ≤ 20,

4x + 2y ≤ 16,

x ≥ 0,

y ≥ 0.

Graph the system of inequalities, determine the vertices,and find the value of P at each vertex.

Vertex P = 34x + 31y(0, 0) 34 · 0 + 31 · 0 = 0

(0, 5) 34 · 0 + 31 · 5 = 155

(2, 4) 34 · 2 + 31 · 4 = 192

(4, 0) 34 · 4 + 31 · 0 = 136

The maximum profit per day is $192 when 2 knit suits and4 worsted suits are made.

77. Let x = the number of pounds of meat and y = the numberof pounds of cheese in the diet in a week. The cost is givenby

C = 3.50x + 4.60y

subject to

2x + 3y ≥ 12,

2x + y ≥ 6,

x ≥ 0,

y ≥ 0.

Graph the system of inequalities, determine the vertices,and find the value of C at each vertex.

Vertex C = 3.50x + 4.60y(0, 6) 3.50(0) + 4.60(6) = 27.60

(1.5, 3) 3.50(1.5) + 4.60(3) = 19.05

(6, 0) 3.50(6) + 4.60(0) = 21.00

The minimum weekly cost of $19.05 is achieved when 1.5 lbof meat and 3 lb of cheese are used.

79. Let x = the number of animal A and y = the number ofanimal B. The total number of animals is given by

T = x + y

subject to

x + 0.2y ≤ 600,

0.5x + y ≤ 525,

x ≥ 0,

y ≥ 0.

Graph the system of inequalities, determine the vertices,and find the value of T at each vertex.

Copyright © 2013 Pearson Education, Inc.

Page 261: JUDITH A. PENNA -  · PDF fileSTUDENT’S SOLUTIONS MANUAL JUDITH A. PENNA Indiana University Purdue University Indianapolis COLLEGE ALGEBRA: GRAPHS AND MODELS FIFTH EDITION

4

�4

42�2�4 x

y

4

2

�2

�4

42�2�4 x

y

�x � y� � 1

4

2

�2

�4

42�2�4 x

y

�x� � �y�

x

y

100 200 300 400 500

5

10

15

20

(0, 0)

(95, 0)

(70, 5)

(25, 9.5)

(0, 10)

258 Chapter 6: Systems of Equations and Matrices

Vertex T = x + y(0, 0) 0 + 0 = 0

(0, 525) 0 + 525 = 525

(550, 250) 550 + 250 = 800

(600, 0) 600 + 0 = 600

The maximum total number of 800 is achieved when thereare 550 of A and 250 of B.

81. −5 ≤ x + 2 < 4

−7 ≤ x < 2 Subtracting 2

The solution set is {x| − 7 ≤ x < 2}, or [−7, 2).

83. x2 − 2x ≤ 3 Polynomial inequality

x2 − 2x− 3 ≤ 0

x2 − 2x− 3 = 0 Related equation

(x + 1)(x− 3) = 0 Factoring

Using the principle of zero products or by observing thegraph of y = x2 − 2x − 3, we see that the solutions ofthe related equation are −1 and 3. These numbers dividethe x-axis into the intervals (−∞,−1), (−1, 3), and (3,∞).We let f(x) = x2−2x−3 and test a value in each interval.

(−∞,−1): f(−2) = 5 > 0

(−1, 3): f(0) = −3 < 0

(3,∞): f(4) = 5 > 0

Function values are negative on (−1, 3). This can also bedetermined from the graph of y = x2 − 2x − 3. Since theinequality symbol is ≤, the endpoints of the interval mustbe included in the solution set. It is {x| − 1 ≤ x ≤ 3} or[−1, 3].

85. Graph: y ≥ x2 − 2,

y ≤ 2 − x2

First graph the related equations y = x2−2 and y = 2−x2

using solid lines. The solution set consists of the regionabove the graph of y = x2 − 2 and below the graph ofy = 2 − x2.

87.

89.

91. Let x = the number of chairs and y = the number of sofasproduced. Find the maximum value of

I = 120x + 575y

subject to

20x + 100y ≤ 1900,

x + 50y ≤ 500,

2x + 20y ≤ 240,

x ≥ 0,

y ≥ 0.

Graph the system of inequalities, determine the vertices,and find the value of I at each vertex.

Vertex I = 120x + 575y(0, 0) 120 · 0 + 575 · 0 = 0

(0, 10) 120 · 0 + 575 · 10 = 5750

(25, 9.5) 120 · 25 + 575(9.5) = 8462.50

(70, 5) 120 · 70 + 575 · 5 = 11, 275

(95, 0) 120 · 95 + 575 · 0 = 11, 400

The maximum income of $11,400 is achieved by making 95chairs and 0 sofas.

Exercise Set 6.8

1. x + 7(x− 3)(x + 2)

=A

x− 3+

B

x + 2x + 7

(x− 3)(x + 2)=

A(x + 2) + B(x− 3)(x− 3)(x + 2)

Adding

Equate the numerators:

x + 7 = A(x + 2) + B(x− 3)

Let x + 2 = 0, or x = −2. Then we get

Copyright © 2013 Pearson Education, Inc.

Page 262: JUDITH A. PENNA -  · PDF fileSTUDENT’S SOLUTIONS MANUAL JUDITH A. PENNA Indiana University Purdue University Indianapolis COLLEGE ALGEBRA: GRAPHS AND MODELS FIFTH EDITION

Exercise Set 6.8 259

−2 + 7 = 0 + B(−2 − 3)

5 = −5B

−1 = B

Next let x− 3 = 0, or x = 3. Then we get

3 + 7 = A(3 + 2) + 0

10 = 5A

2 = A

The decomposition is as follows:2

x− 3− 1

x + 2

3. 7x− 16x2 − 5x + 1

=7x− 1

(3x− 1)(2x− 1)Factoring the denominator

=A

3x− 1+

B

2x− 1

=A(2x− 1) + B(3x− 1)

(3x− 1)(2x− 1)Adding

Equate the numerators:

7x− 1 = A(2x− 1) + B(3x− 1)

Let 2x− 1 = 0, or x =12. Then we get

7(

12

)− 1 = 0 + B

(3 · 1

2− 1)

52

=12B

5 = B

Next let 3x− 1 = 0, or x =13. We get

7(

13

)− 1 = A

(2 · 1

3− 1)

+ 0

73− 1 = A

(23− 1)

43

= −13A

−4 = A

The decomposition is as follows:

− 43x− 1

+5

2x− 1

5. 3x2 − 11x− 26(x2 − 4)(x + 1)

=3x2 − 11x− 26

(x + 2)(x− 2)(x + 1)Factoring thedenominator

=A

x + 2+

B

x− 2+

C

x + 1

=A(x−2)(x+1)+B(x+2)(x+1)+C(x+2)(x−2)

(x+2)(x−2)(x+1)Adding

Equate the numerators:

3x2−11x−26 = A(x− 2)(x + 1)+B(x + 2)(x + 1)+C(x + 2)(x− 2)

Let x + 2 = 0 or x = −2. Then we get

3(−2)2−11(−2)−26 = A(−2−2)(−2+1)+0+0

12 + 22 − 26 = A(−4)(−1)

8 = 4A

2 = A

Next let x− 2 = 0, or x = 2. Then, we get

3 · 22 − 11 · 2 − 26 = 0 + B(2 + 2)(2 + 1) + 0

12 − 22 − 26 = B · 4 · 3−36 = 12B

−3 = B

Finally let x + 1 = 0, or x = −1. We get

3(−1)2−11(−1)−26 = 0 + 0 + C(−1 + 2)(−1 − 2)

3 + 11 − 26 = C(1)(−3)

−12 = −3C

4 = C

The decomposition is as follows:2

x + 2− 3

x− 2+

4x + 1

7. 9(x + 2)2(x− 1)

=A

x + 2+

B

(x + 2)2+

C

x− 1

=A(x + 2)(x− 1) + B(x− 1) + C(x + 2)2

(x + 2)2(x− 1)Adding

Equate the numerators:

9 = A(x + 2)(x− 1) + B(x− 1) + C(x + 2)2 (1)

Let x− 1 = 0, or x = 1. Then, we get

9 = 0 + 0 + C(1 + 2)2

9 = 9C

1 = C

Next let x + 2 = 0, or x = −2. Then, we get

9 = 0 + B(−2 − 1) + 0

9 = −3B

−3 = B

To find A we first simplify equation (1).

9 = A(x2 + x− 2) + B(x− 1) + C(x2 + 4x + 4)

= Ax2 + Ax− 2A + Bx−B + Cx2 + 4Cx + 4C

= (A + C)x2+(A + B + 4C)x+(−2A−B + 4C)

Then we equate the coefficients of x2.

0 = A + C

0 = A + 1 Substituting 1 for C

−1 = A

The decomposition is as follows:

− 1x + 2

− 3(x + 2)2

+1

x− 1

Copyright © 2013 Pearson Education, Inc.

Page 263: JUDITH A. PENNA -  · PDF fileSTUDENT’S SOLUTIONS MANUAL JUDITH A. PENNA Indiana University Purdue University Indianapolis COLLEGE ALGEBRA: GRAPHS AND MODELS FIFTH EDITION

260 Chapter 6: Systems of Equations and Matrices

9. 2x2 + 3x + 1(x2 − 1)(2x− 1)

=2x2 + 3x + 1

(x + 1)(x− 1)(2x− 1)Factoring thedenominator

=A

x + 1+

B

x− 1+

C

2x− 1

=A(x−1)(2x−1)+B(x+1)(2x−1)+C(x+1)(x−1)

(x + 1)(x− 1)(2x− 1)Adding

Equate the numerators:

2x2 + 3x + 1 = A(x− 1)(2x− 1)+B(x + 1)(2x− 1)+C(x + 1)(x− 1)

Let x + 1 = 0, or x = −1. Then, we get

2(−1)2 + 3(−1) + 1 = A(−1 − 1)[2(−1) − 1]+0+0

2 − 3 + 1 = A(−2)(−3)

0 = 6A

0 = A

Next let x− 1 = 0, or x = 1. Then, we get

2 · 12 + 3 · 1 + 1 = 0 + B(1 + 1)(2 · 1 − 1) + 0

2 + 3 + 1 = B · 2 · 16 = 2B

3 = B

Finally we let 2x− 1 = 0, or x =12. We get

2(

12

)2

+3(

12

)+ 1 = 0 + 0 + C

(12

+ 1)(

12− 1)

12

+32

+ 1 = C · 32·(− 1

2

)

3 = −34C

−4 = C

The decomposition is as follows:3

x− 1− 4

2x− 1

11. x4 − 3x3 − 3x2 + 10(x + 1)2(x− 3)

=x4 − 3x3 − 3x2 + 10x3 − x2 − 5x− 3

Multiplying thedenominator

Since the degree of the numerator is greater than the de-gree of the denominator, we divide.

x− 2x3 − x2 − 5x− 3 x4−3x3−3x2+ 0x+10

x4− x3−5x2− 3x−2x3+2x2+ 3x+10−2x3+2x2+10x+ 6

− 7x+ 4

The original expression is thus equivalent to the following:

x− 2 +−7x + 4

x3 − x2 − 5x− 3We proceed to decompose the fraction.

−7x + 4(x + 1)2(x− 3)

=A

x + 1+

B

(x + 1)2+

C

x− 3

=A(x + 1)(x− 3) + B(x− 3) + C(x + 1)2

(x + 1)2(x− 3)Adding

Equate the numerators:

−7x + 4 = A(x + 1)(x− 3) + B(x− 3)+

C(x + 1)2 (1)

Let x− 3 = 0, or x = 3. Then, we get

−7 · 3 + 4 = 0 + 0 + C(3 + 1)2

−17 = 16C

−1716

= C

Let x + 1 = 0, or x = −1. Then, we get

−7(−1) + 4 = 0 + B(−1 − 3) + 0

11 = −4B

−114

= B

To find A we first simplify equation (1).

−7x + 4

= A(x2 − 2x− 3) + B(x− 3) + C(x2 + 2x + 1)

= Ax2 − 2Ax− 3A + Bx− 3B + Cx2 − 2Cx + C

= (A+C)x2 + (−2A+B−2C)x + (−3A−3B+C)

Then equate the coefficients of x2.

0 = A + C

Substituting −1716

for C, we get A =1716

.

The decomposition is as follows:17/16x + 1

− 11/4(x + 1)2

− 17/16x− 3

The original expression is equivalent to the following:

x− 2 +17/16x + 1

− 11/4(x + 1)2

− 17/16x− 3

13. −x2 + 2x− 13(x2 + 2)(x− 1)

=Ax + B

x2 + 2+

C

x− 1

=(Ax + B)(x− 1) + C(x2 + 2)

(x2 + 2)(x− 1)Adding

Equate the numerators:

−x2 + 2x− 13 = (Ax + B)(x− 1) + C(x2 + 2) (1)

Let x− 1 = 0, or x = 1. Then we get

−12 + 2 · 1 − 13 = 0 + C(12 + 2)

−1 + 2 − 13 = C(1 + 2)

−12 = 3C

−4 = C

To find A and B we first simplify equation (1).

Copyright © 2013 Pearson Education, Inc.

Page 264: JUDITH A. PENNA -  · PDF fileSTUDENT’S SOLUTIONS MANUAL JUDITH A. PENNA Indiana University Purdue University Indianapolis COLLEGE ALGEBRA: GRAPHS AND MODELS FIFTH EDITION

Exercise Set 6.8 261

−x2 + 2x− 13

= Ax2 −Ax + Bx−B + Cx2 + 2C

= (A + C)x2 + (−A + B)x + (−B + 2C)

Equate the coefficients of x2:

−1 = A + C

Substituting −4 for C, we get A = 3.

Equate the constant terms:

−13 = −B + 2C

Substituting −4 for C, we get B = 5.

The decomposition is as follows:3x + 5x2 + 2

− 4x− 1

15. 6 + 26x− x2

(2x− 1)(x + 2)2

=A

2x− 1+

B

x + 2+

C

(x + 2)2

=A(x + 2)2 + B(2x− 1)(x + 2) + C(2x− 1)

(2x− 1)(x + 2)2

Adding

Equate the numerators:

6 + 26x− x2 = A(x + 2)2 + B(2x− 1)(x + 2)+C(2x− 1) (1)

Let 2x− 1 = 0, or x =12. Then, we get

6 + 26 · 12−(

12

)2

= A

(12

+ 2)2

+ 0 + 0

6 + 13 − 14

= A

(52

)2

754

=254A

3 = A

Let x + 2 = 0, or x = −2. We get

6 + 26(−2) − (−2)2 = 0 + 0 + C[2(−2) − 1]

6 − 52 − 4 = −5C

−50 = −5C

10 = C

To find B we first simplify equation (1).

6 + 26x− x2

= A(x2 + 4x + 4) + B(2x2 + 3x− 2) + C(2x− 1)

= Ax2+4Ax+4A+2Bx2+3Bx−2B+2Cx−C

= (A+2B)x2 + (4A+3B+2C)x + (4A−2B−C)

Equate the coefficients of x2:

−1 = A + 2B

Substituting 3 for A, we obtain B = −2.

The decomposition is as follows:3

2x− 1− 2

x + 2+

10(x + 2)2

17.6x3 + 5x2 + 6x− 2

2x2 + x− 1Since the degree of the numerator is greater than the de-gree of the denominator, we divide.

3x+ 12x2 + x− 1 6x3+5x2+6x−2

6x3+3x2−3x2x2+9x−22x2+ x−1

8x−1

The original expression is equivalent to

3x + 1 +8x− 1

2x2 + x− 1We proceed to decompose the fraction.

8x− 12x2 + x− 1

=8x− 1

(2x− 1)(x + 1)Factoring thedenominator

=A

2x− 1+

B

x + 1

=A(x + 1) + B(2x− 1)

(2x− 1)(x + 1)Adding

Equate the numerators:

8x− 1 = A(x + 1) + B(2x− 1)

Let x + 1 = 0, or x = −1. Then we get

8(−1) − 1 = 0 + B[2(−1) − 1]

−8 − 1 = B(−2 − 1)

−9 = −3B

3 = B

Next let 2x− 1 = 0, or x =12. We get

8(

12

)− 1 = A

(12

+ 1)

+ 0

4 − 1 = A

(32

)

3 =32A

2 = A

The decomposition is2

2x− 1+

3x + 1

.

The original expression is equivalent to

3x + 1 +2

2x− 1+

3x + 1

.

19. 2x2 − 11x + 5(x− 3)(x2 + 2x− 5)

=A

x− 3+

Bx + C

x2 + 2x− 5

=A(x2+2x−5)+(Bx+C)(x−3)

(x− 3)(x2 + 2x− 5)Adding

Equate the numerators:

2x2 − 11x + 5 = A(x2 + 2x− 5)+

(Bx + C)(x− 3) (1)

Let x− 3 = 0, or x = 3. Then, we get

Copyright © 2013 Pearson Education, Inc.

Page 265: JUDITH A. PENNA -  · PDF fileSTUDENT’S SOLUTIONS MANUAL JUDITH A. PENNA Indiana University Purdue University Indianapolis COLLEGE ALGEBRA: GRAPHS AND MODELS FIFTH EDITION

262 Chapter 6: Systems of Equations and Matrices

2 · 32 − 11 · 3 + 5 = A(32 + 2 · 3 − 5) + 0

18 − 33 + 5 = A(9 + 6 − 5)

−10 = 10A

−1 = A

To find B and C, we first simplify equation (1).

2x2 − 11x + 5 = Ax2 + 2Ax− 5A + Bx2 − 3Bx+

Cx− 3C

= (A + B)x2 + (2A− 3B + C)x+

(−5A− 3C)

Equate the coefficients of x2:

2 = A + B

Substituting −1 for A, we get B = 3.

Equate the constant terms:

5 = −5A− 3C

Substituting −1 for A, we get C = 0.

The decomposition is as follows:

− 1x− 3

+3x

x2 + 2x− 5

21.−4x2 − 2x + 10(3x + 5)(x + 1)2

The decomposition looks likeA

3x + 5+

B

x + 1+

C

(x + 1)2.

Add and equate the numerators.

−4x2 − 2x + 10

= A(x + 1)2 + B(3x + 5)(x + 1) + C(3x + 5)

= A(x2 + 2x + 1) + B(3x2 + 8x + 5) + C(3x + 5)or

−4x2 − 2x + 10

= (A + 3B)x2+(2A + 8B + 3C)x+(A + 5B + 5C)

Then equate corresponding coefficients.

−4 = A + 3B Coefficients of x2-terms

−2 = 2A + 8B + 3C Coefficients of x-terms

10 = A + 5B + 5C Constant terms

We solve this system of three equations and find A = 5,B = −3, C = 4.

The decomposition is5

3x + 5− 3

x + 1+

4(x + 1)2

.

23.36x + 1

12x2 − 7x− 10=

36x + 1(4x− 5)(3x + 2)

The decomposition looks likeA

4x− 5+

B

3x + 2.

Add and equate the numerators.

36x + 1 = A(3x + 2) + B(4x− 5)

or 36x + 1 = (3A + 4B)x + (2A− 5B)

Then equate corresponding coefficients.

36 = 3A + 4B Coefficients of x-terms

1 = 2A− 5B Constant terms

We solve this system of equations and find

A = 8 and B = 3.

The decomposition is8

4x− 5+

33x + 2

.

25.−4x2 − 9x + 8

(3x2 + 1)(x− 2)The decomposition looks likeAx + B

3x2 + 1+

C

x− 2.

Add and equate the numerators.

−4x2 − 9x + 8

= (Ax + B)(x− 2) + C(3x2 + 1)

= Ax2−2Ax+Bx−2B+3Cx2+C

or

−4x2 − 9x + 8

= (A + 3C)x2 + (−2A + B)x + (−2B + C)

Then equate corresponding coefficients.

−4 = A + 3C Coefficients of x2-terms

−9 = −2A + B Coefficients of x-terms

8 = −2B + C Constant terms

We solve this system of equations and find

A = 2, B = −5, C = −2.

The decomposition is2x− 53x2 + 1

− 2x− 2

.

27. x3 + x2 + 9x + 9 = 0

x2(x + 1) + 9(x + 1) = 0

(x + 1)(x2 + 9) = 0

x + 1 = 0 or x2 + 9 = 0

x = −1 or x2 = −9

x = −1 or x = ±3i

The solutions are −1, 3i, and −3i.

29. f(x) = x3 + x2 − 3x− 2

We use synthetic division to factor the polynomial. Usingthe possibilities found by the rational zeros theorem wefind that x + 2 is a factor:−2∣∣ 1 1 −3 −2

−2 2 21 −1 −1 0

We have x3 + x2 − 3x− 2 = (x + 2)(x2 − x− 1).

x3 + x2 − 3x− 2 = 0

(x + 2)(x2 − x− 1) = 0

x + 2 = 0 or x2 − x− 1 = 0

The solution of the first equation is −2. We use thequadratic formula to solve the second equation.

Copyright © 2013 Pearson Education, Inc.

Page 266: JUDITH A. PENNA -  · PDF fileSTUDENT’S SOLUTIONS MANUAL JUDITH A. PENNA Indiana University Purdue University Indianapolis COLLEGE ALGEBRA: GRAPHS AND MODELS FIFTH EDITION

Exercise Set 6.8 263

x =−b±√

b2 − 4ac2a

=−(−1) ±√(−1)2 − 4 · 1 · (−1)

2 · 1=

1 ±√5

2

The solutions are −2,1 +

√5

2and

1 −√5

2.

31. f(x) = x3 + 5x2 + 5x− 3

−3∣∣ 1 5 5 −3

−3 −6 31 2 −1 0

x3 + 5x2 + 5x− 3 = 0

(x + 3)(x2 + 2x− 1) = 0

x + 3 = 0 or x2 + 2x− 1 = 0

The solution of the first equation is −3. We use thequadratic formula to solve the second equation.

x =−b±√

b2 − 4ac2a

=−2 ±√22 − 4 · 1 · (−1)

2 · 1 =−2 ±√

82

=−2 ± 2

√2

2=

2(−1 ±√2)

2= −1 ±√

2

The solutions are −3, −1 +√

2, and −1 −√2.

33.x

x4 − a4

=x

(x2 + a2)(x + a)(x− a)Factoring thedenominator

=Ax + B

x2 + a2+

C

x + a+

D

x− a

=[(Ax + B)(x + a)(x− a) + C(x2 + a2)(x− a)+

D(x2 + a2)(x + a)]/[(x2 + a2)(x + a)(x− a)]

Equate the numerators:

x = (Ax + B)(x + a)(x− a) + C(x2 + a2)(x− a)+

D(x2 + a2)(x + a)

Let x− a = 0, or x = a. Then, we get

a = 0 + 0 + D(a2 + a2)(a + a)

a = D(2a2)(2a)

a = 4a3D

14a2

= D

Let x + a = 0, or x = −a. We get

−a = 0 + C[(−a)2 + a2](−a− a) + 0

−a = C(2a2)(−2a)

−a = −4a3C

14a2

= C

Equate the coefficients of x3:

0 = A + C + D

Substituting1

4a2for C and for D, we get

A = − 12a2

.

Equate the constant terms:

0 = −Ba2 − Ca3 + Da3

Substitute1

4a2for C and for D. Then solve for B.

0 = −Ba2 − 14a2

· a3 +1

4a2· a3

0 = −Ba2

0 = B

The decomposition is as follows:

−1

2a2x

x2 + a2+

14a2

x + a+

14a2

x− a

35.1 + lnx2

(lnx + 2)(lnx− 3)2=

1 + 2 lnx

(lnx + 2)(lnx− 3)2

Let u = lnx. Then we have:1 + 2u

(u + 2)(u− 3)2

=A

u + 2+

B

u− 3+

C

(u− 3)2

=A(u− 3)2 + B(u + 2)(u− 3) + C(u + 2)

(u + 2)(u− 3)2

Equate the numerators:

1 + 2u = A(u− 3)2 + B(u + 2)(u− 3) + C(u + 2)

Let u− 3 = 0, or u = 3.

1 + 2 · 3 = 0 + 0 + C(5)

7 = 5C75

= C

Let u + 2 = 0, or u = −2.

1 + 2(−2) = A(−2 − 3)2 + 0 + 0

−3 = 25A

− 325

= A

To find B, we equate the coefficients of u2:

0 = A + B

Substituting − 325

for A and solving for B, we get B =325

.

The decomposition of1 + 2u

(u + 2)(u− 3)2is as follows:

− 325(u + 2)

+3

25(u− 3)+

75(u− 3)2

Substituting lnx for u we get

− 325(lnx + 2)

+3

25(lnx− 3)+

75(lnx− 3)2

.

Copyright © 2013 Pearson Education, Inc.

Page 267: JUDITH A. PENNA -  · PDF fileSTUDENT’S SOLUTIONS MANUAL JUDITH A. PENNA Indiana University Purdue University Indianapolis COLLEGE ALGEBRA: GRAPHS AND MODELS FIFTH EDITION

264 Chapter 6: Systems of Equations and Matrices

Chapter 6 Review Exercises

1. The statement is true. See page 485 in the text.

3. The statement is true. See page 521 in the text.

5. (a)

7. (h)

9. (b)

11. (c)

13. 5x − 3y = −4, (1)

3x − y = −4 (2)

Multiply equation (2) by −3 and add.

5x − 3y = −4

−9x + 3y = 12−4x = 8

x = −2

Back-substitute to find y.

3(−2) − y = −4 Using equation (2)

−6 − y = −4

−y = 2y = −2

The solution is (−2,−2).

15. x + 5y = 12, (1)

5x + 25y = 12 (2)

Solve equation (1) for x.

x = −5y + 12

Substitute in equation (2) and solve for y.

5(−5y + 12) + 25y = 12

−25y + 60 + 25y = 12

60 = 12

We get a false equation, so there is no solution.

17. x + 5y − 3z = 4, (1)

3x − 2y + 4z = 3, (2)

2x + 3y − z = 5 (3)

Multiply equation (1) by −3 and add it to equation (2).

Multiply equation (1) by −2 and add it to equation (3).

x + 5y + 3z = 4 (1)

− 17y + 13z = −9 (4)

− 7y + 5z = −3 (5)

Multiply equation (5) by 17.

x + 5y + 3z = 4 (1)

− 17y + 13z = −9 (4)

− 119y + 85z = −51 (6)

Multiply equation (4) by −7 and add it to equation (6).

x + 5y + 3z = 4 (1)

− 17y + 13z = −9 (4)

− 6z = 12 (7)

Now we solve equation (7) for z.

−6z = 12

z = −2

Back-substitute −2 for z in equation (4) and solve for y.

−17y + 13(−2) = −9

−17y − 26 = −9

−17y = 17

y = −1

Finally, we back-substitute −1 for y and −2 for z in equa-tion (1) and solve for x.

x + 5(−1) − 3(−2) = 4

x− 5 + 6 = 4

x + 1 = 4

x = 3

The solution is (3,−1,−2).

19. x − y = 5, (1)

y − z = 6, (2)

z − w = 7, (3)

x + w = 8 (4)

Multiply equation (1) by −1 and add it to equation (4).

x − y = 5 (1)

y − z = 6 (2)

z − w = 7 (3)

y + w = 3 (5)

Multiply equation (2) by −1 and add it to equation (5).

x − y = 5 (1)

y − z = 6 (2)

z − w = 7 (3)

z + w = −3 (6)

Multiply equation (3) by −1 and add it to equation (6).

x − y = 5 (1)

y − z = 6 (2)

z − w = 7 (3)

2w = −10 (7)

Solve equation (7) for w.

2w = −10

w = −5

Back-substitute −5 for w in equation (3) and solve for z.

z − w = 7

z − (−5) = 7

z + 5 = 7

z = 2

Copyright © 2013 Pearson Education, Inc.

Page 268: JUDITH A. PENNA -  · PDF fileSTUDENT’S SOLUTIONS MANUAL JUDITH A. PENNA Indiana University Purdue University Indianapolis COLLEGE ALGEBRA: GRAPHS AND MODELS FIFTH EDITION

Chapter 6 Review Exercises 265

Back-substitute 2 for z in equation (2) and solve for y.

y − z = 6

y − 2 = 6

y = 8

Back-substitute 8 for y in equation (1) and solve for x.

x− y = 5

x− 8 = 5

x = 13

Writing the solution as (w, x, y, z), we have (−5, 13, 8, 2).

21. Systems 13, 14, 15, 17, 18, and 19 each have either nosolution or exactly one solution, so the equations in thosesystems are independent. System 16 has infinitely manysolutions, so the equations in that system are dependent.

23. 3x + 4y + 2z = 3

5x − 2y − 13z = 3

4x + 3y − 3z = 6

Write the augmented matrix. We will use Gaussian elimi-nation.

3 4 2 3

5 −2 −13 3

4 3 −3 6

Multiply row 2 and row 3 by 3.

3 4 2 3

15 −6 −39 9

12 9 −9 18

Multiply row 1 by −5 and add it to row 2.

Multiply row 1 by −4 and add it to row 3.

3 4 2 3

0 −26 −49 −6

0 −7 −17 6

Multiply row 3 by 26.

3 4 2 3

0 −26 −49 −6

0 −182 −442 156

Multiply row 2 by −7 and add it to row 3.

3 4 2 3

0 −26 −49 −6

0 0 −99 198

Multiply row 1 by13, row 2 by − 1

26, and row 3 by − 1

99.

143

23

1

0 14926

313

0 0 1 −2

x +43y +

23z = 1 (1)

y +4926

z =313

(2)

z = −2

Back-substitute in equation (2) and solve for y.

y +4926

(−2) =313

y − 4913

=313

y =5213

= 4

Back-substitute in equation (1) and solve for x.

x +43(4) +

23(−2) = 1

x +163

− 43

= 1

x = −3

The solution is (−3, 4,−2).

25. w + x + y + z = −2,

−3w − 2x + 3y + 2z = 10,

2w + 3x + 2y − z = −12,

2w + 4x − y + z = 1

Write the augmented matrix. We will use Gauss-Jordanelimination.

1 1 1 1 −2

−3 −2 3 2 10

2 3 2 −1 −12

2 4 −1 1 1

Multiply row 1 by 3 and add it to row 2.

Multiply row 1 by −2 and add it to row 3.

Multiply row 1 by −2 and add it to row 4.

1 1 1 1 −2

0 1 6 5 4

0 1 0 −3 −8

0 2 −3 −1 5

Multiply row 2 by −1 and add it to row 1.

Multiply row 2 by −1 and add it to row 3.

Multiply row 2 by −2 and add it to row 4.

1 0 −5 −4 −6

0 1 6 5 4

0 0 −6 −8 −12

0 0 −15 −11 −3

Multiply row 1 by 3.

Multiply row 3 by −12.

Copyright © 2013 Pearson Education, Inc.

Page 269: JUDITH A. PENNA -  · PDF fileSTUDENT’S SOLUTIONS MANUAL JUDITH A. PENNA Indiana University Purdue University Indianapolis COLLEGE ALGEBRA: GRAPHS AND MODELS FIFTH EDITION

266 Chapter 6: Systems of Equations and Matrices

3 0 −15 −12 −18

0 1 6 5 4

0 0 3 4 6

0 0 −15 −11 −3

Multiply row 3 by 5 and add it to row 1.

Multiply row 3 by −2 and add it to row 2.

Multiply row 3 by 5 and add it to row 4.

3 0 0 8 12

0 1 0 −3 −8

0 0 3 4 6

0 0 0 9 27

Multiply row 4 by19.

3 0 0 8 12

0 1 0 −3 −8

0 0 3 4 6

0 0 0 1 3

Multiply row 4 by −8 and add it to row 1.

Multiply row 4 by 3 and add it to row 2.

Multiply row 4 by −4 and add it to row 3.

3 0 0 0 −12

0 1 0 0 1

0 0 3 0 −6

0 0 0 1 3

Multiply rows 1 and 3 by13.

1 0 0 0 −4

0 1 0 0 1

0 0 1 0 −2

0 0 0 1 3

The solution is (−4, 1,−2, 3).

27. Familiarize. Let x = the amount invested at 3% and y =the amount invested at 3.5%. Then the interest from theinvestments is 3%x and 3.5%y, or 0.03x and 0.035y.

Translate.

The total investment is $5000.

x + y = 5000

The total interest is $167.

0.03x + 0.035y = 167

We have a system of equations.x + y = 5000,

0.03x + 0.035y = 167Multiplying the second equation by 1000 to clear the dec-imals, we have:

x + y = 5000, (1)

30x + 35y = 167, 000. (2)

Carry out. We begin by multiplying equation (1) by −30and adding.

−30x − 30y = −150, 000

30x + 35y = 167, 0005y = 17, 000

y = 3400

Back-substitute to find x.x + 3400 = 5000 Using equation (1)

x = 1600

Check. The total investment is $1600 + $3400, or $5000.The total interest is 0.03($1600) + 0.035($3400), or $48 +$119, or $167. The solution checks.

State. $1600 was invested at 3% and $3400 was investedat 3.5%.

29. Familiarize. Let x, y, and z represent the scores on thefirst, second, and third tests, respectively.

Translate.

The total score on the three tests is 226.

x + y + z = 226

The sum of the scores on the first and second tests exceedsthe score on the third test by 62.

x + y = z + 62

The first score exceeds the second by 6.

x = y + 6

We have a system of equations.

x + y + z = 226,

x + y = z + 62,

x = y + 6

or x + y + z = 226,

x + y − z = 62,

x − y = 6

Carry out. Solving the system of equations, we get(75, 69, 82).

Check. The sum of the scores is 75+69+82, or 226. Thesum of the scores on the first two tests is 75 + 69, or 144.This exceeds the score on the third test, 82, by 62. Thescore on the first test, 75, exceeds the score on the secondtest, 69, by 6. The solution checks.

State. The scores on the first, second, and third tests were75, 69, and 82, respectively.

31. A + B =

1 −1 0

2 3 −2−2 0 1

+

−1 0 6

1 −2 00 1 −3

=

1 + (−1) −1 + 0 0 + 6

2 + 1 3 + (−2) −2 + 0−2 + 0 0 + 1 1 + (−3)

=

0 −1 6

3 1 −2−2 1 −2

Copyright © 2013 Pearson Education, Inc.

Page 270: JUDITH A. PENNA -  · PDF fileSTUDENT’S SOLUTIONS MANUAL JUDITH A. PENNA Indiana University Purdue University Indianapolis COLLEGE ALGEBRA: GRAPHS AND MODELS FIFTH EDITION

Chapter 6 Review Exercises 267

33. −A = −1

1 −1 0

2 3 −2−2 0 1

=

−1 1 0

−2 −3 22 0 −1

35. B and C do not have the same order, so it is not possibleto find B + C.

37. BA =

−1 0 6

1 −2 00 1 −3

· 1 −1 0

2 3 −2−2 0 1

=

−1 + 0 − 12 1 + 0 + 0 0 + 0 + 6

1 − 4 + 0 −1 − 6 + 0 0 + 4 + 00 + 2 + 6 0 + 3 + 0 0 − 2 − 3

=

−13 1 6

−3 −7 48 3 −5

39. a) M =

0.98 0.23 0.30 0.28 0.451.03 0.19 0.27 0.34 0.411.01 0.21 0.35 0.31 0.390.99 0.25 0.29 0.33 0.42

b) N =[32 19 43 38

]c) NM =

[131.98 29.50 40.80 41.29 54.92

]d) The entries of NM represent the total cost, in dol-

lars, for each item for the day’s meal.

41. A =

0 0 3

0 −2 04 0 0

Write the augmented matrix. 0 0 3 1 0 0

0 −2 0 0 1 04 0 0 0 0 1

Interchange rows 1 and 3. 4 0 0 0 0 1

0 −2 0 0 1 00 0 3 1 0 0

Multiply row 1 by14, row 2 by −1

2, and row 3 by

13.

1 0 0 0 014

0 1 0 0 −12

0

0 0 113

0 0

A−1 =

0 014

0 −12

0

13

0 0

43. 3x − 2y + 4z = 13,

x + 5y − 3z = 7,

2x − 3y + 7z = −8

Write the coefficients on the left in a matrix. Then writethe product of that matrix and the column matrix contain-ing the variables, and set the result equal to the columnmatrix containing the constants on the right. 3 −2 4

1 5 −32 −3 7

x

yz

=

13

7−8

45. 5x − y + 2z = 17,

3x + 2y − 3z = −16,

4x − 3y − z = 5

Write an equivalent matrix equation, AX = B. 5 −1 2

3 2 −34 −3 −1

x

yz

=

17

−165

Then,

X = A−1B =

1180

780

180

980

1380

−2180

1780

−1180

−1380

17

−165

=

1

−25

The solution is (1,−2, 5).

47.∣∣∣∣ 1 −23 4

∣∣∣∣ = 1 · 4 − 3(−2) = 4 + 6 = 10

49. We will expand across the first row.∣∣∣∣∣∣2 −1 11 2 −13 4 −3

∣∣∣∣∣∣= 2(−1)1+1

∣∣∣∣ 2 −14 −3

∣∣∣∣+ (−1)(−1)1+2

∣∣∣∣ 1 −13 −3

∣∣∣∣+1(−1)1+3

∣∣∣∣ 1 23 4

∣∣∣∣= 2 · 1[2(−3) − 4(−1)] + (−1)(−1)[1(−3) − 3(−1)]+

1 · 1[1(4) − 3(2)]

= 2(−2) + 1(0) + 1(−2)

= −6

51. 5x − 2y = 19,

7x + 3y = 15

D =∣∣∣∣ 5 −27 3

∣∣∣∣ = 5(3) − 7(−2) = 29

Dx =∣∣∣∣ 19 −215 3

∣∣∣∣ = 19(3) − 15(−2) = 87

Dy =∣∣∣∣ 5 197 15

∣∣∣∣ = 5(15) − 7(19) = −58

Copyright © 2013 Pearson Education, Inc.

Page 271: JUDITH A. PENNA -  · PDF fileSTUDENT’S SOLUTIONS MANUAL JUDITH A. PENNA Indiana University Purdue University Indianapolis COLLEGE ALGEBRA: GRAPHS AND MODELS FIFTH EDITION

4

2

�2

42�4 x

y

y � 3x � 6

4

�4

4�4 x

y

(0, 9)

(2, 5)(5, 1)

(8, 0)

y

5

4

3

2

7

6

8

9

10

11

1

x7653 98 1110421

(0, 8)

(10, 0)(8, 0)

268 Chapter 6: Systems of Equations and Matrices

x =Dx

D=

8729

= 3

y =Dy

D=

−5829

= −2

The solution is (3,−2).

53. 3x − 2y + z = 5,

4x − 5y − z = −1,

3x + 2y − z = 4

D =

∣∣∣∣∣∣3 −2 14 −5 −13 2 −1

∣∣∣∣∣∣ = 42

Dx =

∣∣∣∣∣∣5 −2 1−1 −5 −14 2 −1

∣∣∣∣∣∣ = 63

Dy =

∣∣∣∣∣∣3 5 14 −1 −13 4 −1

∣∣∣∣∣∣ = 39

Dz =

∣∣∣∣∣∣3 −2 54 −5 −13 2 4

∣∣∣∣∣∣ = 99

x =Dx

D=

6342

=32

y =Dy

D=

3942

=1314

z =Dz

D=

9942

=3314

The solution is(

32,1314

,3314

).

55.

57. Graph: 2x + y ≥ 9,

4x + 3y ≥ 23,

x + 3y ≥ 8,

x ≥ 0,

y ≥ 0

Shade the intersection of the graphs of the given inequali-ties.

We find the vertex (0, 9) by solving the system

2x + y = 9,

x = 0.

We find the vertex (2, 5) by solving the system

2x + y = 9,

4x + 3y = 23.

We find the vertex (5, 1) by solving the system

4x + 3y = 23,

x + 3y = 8.

We find the vertex (8, 0) by solving the system

x + 3y = 8,

y = 0.

59. Let x = the number of questions answered from group Aand y = the number of questions answered from group B.Find the maximum value of S = 7x + 12y subject to

x + y ≥ 8,

8x + 10y ≤ 80,

x ≥ 0,

y ≥ 0

Graph the system of inequalities, determine the vertices,and find the value of T at each vertex.

Vertex S = 7x + 12y(0, 8) 7 · 0 + 12 · 8 = 96

(8, 0) 7 · 8 + 12 · 0 = 56

(10, 0) 7 · 10 + 12 · 0 = 70

The maximum score of 96 occurs when 0 questions fromgroup A and 8 questions from group B are answered cor-rectly.

61. −8x + 232x2 + 5x− 12

=−8x + 23

(2x− 3)(x + 4)

=A

2x− 3+

B

x + 4

=A(x + 4) + B(2x− 3)

(2x− 3)(x + 4)Equate the numerators.

−8x + 23 = A(x + 4) + B(2x− 3)

Copyright © 2013 Pearson Education, Inc.

Page 272: JUDITH A. PENNA -  · PDF fileSTUDENT’S SOLUTIONS MANUAL JUDITH A. PENNA Indiana University Purdue University Indianapolis COLLEGE ALGEBRA: GRAPHS AND MODELS FIFTH EDITION

x

y

�xy� � 1

Chapter 6 Review Exercises 269

Let x =32

: −8(

32

)+ 23 = A

(32

+ 4)

+ 0

−12 + 23 =112A

11 =112A

2 = A

Let x = −4 : −8(−4) + 23 = 0 + B[2(−4) − 3]

32 + 23 = −11B

55 = −11B

−5 = B

The decomposition is2

2x− 3− 5

x + 4.

63. Interchanging columns of a matrix is not a row-equivalentoperation, so answer A is correct. (See page 511 in thetext.)

65. Let x, y, and z represent the amounts invested at 4%, 5%,

and 512%, respectively.

Solve: x + y + z = 40, 000,

0.04x + 0.05y + 0.055z = 1990,

0.055z = 0.04x + 590

x = $10, 000, y = $12, 000, z = $18, 000

67. 3x

− 4y

+1z

= −2, (1)

5x

+1y− 2

z= 1, (2)

7x

+3y

+2z

= 19 (3)

Multiply equations (2) and (3) by 3.3x

− 4y

+1z

= −2 (1)

15x

+3y− 6

z= 3 (4)

21x

+9y

+6z

= 57 (5)

Multiply equation (1) by −5 and add it to equation (2).

Multiply equation (1) by −7 and add it to equation (3).3x

− 4y

+1z

= −2 (1)

23y

− 11z

= 13 (6)

37y

− 1z

= 71 (7)

Multiply equation (7) by 23.3x

− 4y

+1z

= −2 (1)

23y

− 11z

= 13 (6)

851y

− 23z

= 1633 (8)

Multiply equation (6) by −37 and add it to equation (8).3x

− 4y

+1z

= −2 (1)

23y

− 11z

= 13 (6)

384z

= 1152 (9)

Complete the solution.384z

= 1152

13

= z

23y

− 111/3

= 13

23y

− 33 = 13

23y

= 46

12

= y

3x− 4

1/2+

11/3

= −2

3x− 8 + 3 = −2

3x− 5 = −2

3x

= 3

1 = x

The solution is(

1,12,13

).

(We could also have solved this system of equations by first

substituting a for1x

, b for1y, and c for

1z

and proceeding

as we did in Exercise 66 above.)

69.

71. In general, (AB)2 �= A2B2. (AB)2 = ABAB andA2B2 = AABB. Since matrix multiplication is not com-mutative, BA �= AB, so (AB)2 �= A2B2.

73. If a1x + b1y = c1 and a2x + b2y = c2 are parallel lines,then a1 = ka2, b1 = kb2, and c1 �= kc2, for some number

k. Then∣∣∣∣ a1 b1a2 b2

∣∣∣∣ = 0,∣∣∣∣ c1 b1c2 b2

∣∣∣∣ �= 0, and∣∣∣∣ a1 c1a2 c2

∣∣∣∣ �= 0.

75. The denominator of the second fraction, x2 − 5x + 6, canbe factored into linear factors with real coefficients:

Copyright © 2013 Pearson Education, Inc.

Page 273: JUDITH A. PENNA -  · PDF fileSTUDENT’S SOLUTIONS MANUAL JUDITH A. PENNA Indiana University Purdue University Indianapolis COLLEGE ALGEBRA: GRAPHS AND MODELS FIFTH EDITION

270 Chapter 6: Systems of Equations and Matrices

(x− 3)(x− 2). Thus, the given expression is not a partialfraction decomposition.

Chapter 6 Test

1. 3x + 2y = 1, (1)

2x − y = −11 (2)

Multiply equation (2) by 2 and add.

3x + 2y = 1

4x − 2y = −227x = −21

x = −3

Back-substitute to find y.

2(−3) − y = −11 Using equation (2).

−6 − y = −11

−y = −5y = 5

The solution is (−3, 5). Since the system of equations hasexactly one solution, it is consistent and the equations areindependent.

2. 2x − y = 3, (1)

2y = 4x− 6 (2)

Solve equation (1) for y.

y = 2x− 3

Subsitute in equation (2) and solve for x.

2(2x− 3) = 4x− 6

4x− 6 = 4x− 6

0 = 0

The equation 0 = 0 is true for all values of x and y.Thus the system of equations has infinitely many solutions.Solving either equation for y, we get y = 2x − 3, so thesolutions are ordered pairs of the form (x, 2x−3). Equiva-

lently, if we solve either equation for x, we get x =y + 3

2,

so the solutions can also be expressed as(y + 3

2, y

). Since

there are infinitely many solutions, the system of equationsis consistent and the equations are dependent.

3. x − y = 4, (1)

3y = 3x− 8 (2)

Solve equation (1) for x.

x = y + 4

Subsitute in equation (2) and solve for y.

3y = 3(y + 4) − 8

3y = 3y + 12 − 8

0 = 4

We get a false equation so there is no solution. Since thereis no solution the system of equations is inconsistent andthe equations are independent.

4. 2x − 3y = 8, (1)

5x − 2y = 9 (2)

Multiply equation (1) by 5 and equation (2) by −2 andadd.

10x − 15y = 40

−10x + 4y = −18− 11y = 22

y = −2

Back-substitute to find x.2x− 3(−2) = 8

2x + 6 = 8

2x = 2x = 1

The solution is (1,−2). Since the system of equations hasexactly one solution, it is consistent and the equations areindependent.

5. 4x + 2y + z = 4, (1)

3x − y + 5z = 4, (2)

5x + 3y − 3z = −2 (3)

Multiply equations (2) and (3) by 4.

4x + 2y + z = 4 (1)

12x − 4y + 20z = 16 (4)

20x + 12y − 12z = −8 (5)

Multiply equation (1) by −3 and add it to equation (4).

Multiply equation (1) by −5 and add it to equation (5).

4x + 2y + z = 4 (1)

−10y + 17z = 4 (6)

2y − 17z = −28 (7)

Interchange equations (6) and (7).

4x + 2y + z = 4 (1)

2y − 17z = −28 (7)

−10y + 17z = 4 (6)

Multiply equation (7) by 5 and add it to equation (6).

4x + 2y + z = 4 (1)

2y − 17z = −28 (7)

−68z = −136 (8)

Solve equation (8) for z.

−68z = −136

z = 2

Back-substitute 2 for z in equation (7) and solve for y.

2y − 17 · 2 = −28

2y − 34 = −28

2y = 6

y = 3

Back-substitute 3 for y and 2 for z in equation (1) andsolve for x.

Copyright © 2013 Pearson Education, Inc.

Page 274: JUDITH A. PENNA -  · PDF fileSTUDENT’S SOLUTIONS MANUAL JUDITH A. PENNA Indiana University Purdue University Indianapolis COLLEGE ALGEBRA: GRAPHS AND MODELS FIFTH EDITION

Chapter 6 Test 271

4x + 2 · 3 + 2 = 4

4x + 8 = 4

4x = −4

x = −1

The solution is (−1, 3, 2).

6. Familiarize. Let x and y represent the number of stu-dent and nonstudent tickets sold, respectively. Then thereceipts from the student tickets were 3x and the receiptsfrom the nonstudent tickets were 5y.

Translate. One equation comes from the fact that750 tickets were sold.

x + y = 750

A second equation comes from the fact that the total re-ceipts were $3066.

3x + 5y = 3066

Carry out. We solve the system of equations.

x + y = 750, (1)

3x + 5y = 3066 (2)

Multiply equation (1) by −3 and add.

−3x − 3y = −2250

3x + 5y = 30662y = 816

y = 408

Substitute 408 for y in equation (1) and solve for x.

x + 408 = 750

x = 342

Check. The number of tickets sold was 342+408, or 750.The total receipts were $3·342+$5·408 = $1026+$2040 =$3066. The solution checks.

State. 342 student tickets and 408 nonstudent tickets weresold.

7. Familiarize. Let x, y, and z represent the number oforders that can be processed per day by Latonna, Cole,and Sam, respectively.

Translate.

Latonna, Cole, and Sam can process 352 orders per day.

x + y + z = 352

Latonna and Cole together can process 224 orders per day.

x + y = 224

Latonna and Sam together can process 248 orders per day.

x + z = 248

We have a system of equations:

x + y + z = 352,

x + y = 224,

x + z = 248.

Carry out. Solving the system of equations, we get(120, 104, 128).

Check. Latonna, Cole, and Sam can process 120 + 104 +128, or 352, orders per day. Together, Latonna and Cole

can process 120 + 104, or 224, orders per day. Together,Latonna and Sam can process 120+128, or 248, orders perday. The solution checks.

State. Latonna can process 120 orders per day, Cole canprocess 104 orders per day, and Sam can process 128 ordersper day.

8. B + C =[ −5 1

−2 4

]+[

3 −4−1 0

]

=[ −5 + 3 1 + (−4)

−2 + (−1) 4 + 0

]

=[ −2 −3

−3 4

]

9. A and C do not have the same order, so it is not possibleto find A − C.

10. CB =[

3 −4−1 0

] [ −5 1−2 4

]

=[

3(−5) + (−4)(−2) 3(1) + (−4)(4)−1(−5) + 0(−2) −1(1) + 0(4)

]

=[ −7 −13

5 −1

]

11. The product AB is not defined because the number ofcolumns of A, 3, is not equal to the number of rows of B,2.

12. 2A = 2[

1 −1 3−2 5 2

]=[

2 −2 6−4 10 4

]

13. C =[

3 −4−1 0

]Write the augmented matrix.[

3 −4 1 0−1 0 0 1

]Interchange rows.[ −1 0 0 1

3 −4 1 0

]Multiply row 1 by 3 and add it to row 2.[ −1 0 0 1

0 −4 1 3

]

Multiply row 1 by −1 and row 2 by −14.

1 0 0 −1

0 1 −14

−34

C−1 =

0 −1

−14

−34

14. a) M =

0.95 0.40 0.39

1.10 0.35 0.411.05 0.39 0.36

Copyright © 2013 Pearson Education, Inc.

Page 275: JUDITH A. PENNA -  · PDF fileSTUDENT’S SOLUTIONS MANUAL JUDITH A. PENNA Indiana University Purdue University Indianapolis COLLEGE ALGEBRA: GRAPHS AND MODELS FIFTH EDITION

3x � 4y � �12

y

x

2

4

�2

�4

�2�4 42

y

2

1

3

4

x3 4 6521A D

B

C

272 Chapter 6: Systems of Equations and Matrices

b) N =[26 18 23

]c) NM =

[68.65 25.67 25.80

]d) The entries of NM represent the total cost, in dol-

lars, for each type of menu item served on the givenday.

15.

3 −4 2

2 3 11 −5 −3

x

yz

=

−8

73

16. 3x + 2y + 6z = 2,

x + y + 2z = 1,

2x + 2y + 5z = 3Write an equivalent matrix equation, AX = B. 3 2 6

1 1 22 2 5

x

yz

=

2

13

Then,

X = A−1B =

1 2 −2

−1 3 00 −2 1

2

13

=

−2

11

The solution is (−2, 1, 1).

17.∣∣∣∣ 3 −58 7

∣∣∣∣ = 3 · 7 − 8(−5) = 21 + 40 = 61

18. We will expand across the first row.∣∣∣∣∣∣2 −1 4−3 1 −25 3 −1

∣∣∣∣∣∣= 2(−1)1+1

∣∣∣∣ 1 −23 −1

∣∣∣∣+ (−1)(−1)1+2

∣∣∣∣−3 −25 −1

∣∣∣∣+4(−1)1+3

∣∣∣∣−3 15 3

∣∣∣∣= 2 · 1[1(−1) − 3(−2)] + (−1)(−1)[−3(−1) − 5(−2)]+

4 · 1[−3(3) − 5(1)]

= 2(5) + 1(13) + 4(−14)

= −33

19. 5x + 2y = −1,

7x + 6y = 1

D =∣∣∣∣ 5 27 6

∣∣∣∣ = 5(6) − 7(2) = 16

Dx =∣∣∣∣−1 2

1 6

∣∣∣∣ = −1(6) − (1)(2) = −8

Dy =∣∣∣∣ 5 −17 1

∣∣∣∣ = 5(1) − 7(−1) = 12

x =Dx

D=

−816

= −12

y =Dy

D=

1216

=34

The solution is(− 1

2,34

).

20.

21. Find the maximum value and the minimum value of

Q = 2x + 3y subject to

x + y ≥ 6,

2x− 3y ≥ −3,

x ≥ 1,

y ≥ 0.

Graph the system of inequalities and determine the ver-tices.

Vertex A:We solve the system x = 1 and y = 0. The coordi-nates of point A are (1, 0).

Vertex B:We solve the system 2x− 3y = −3 and x = 1. The

coordinates of point B are(

1,53

).

Vertex C:We solve the system x + y = 6 and 2x − 3y = −3.The coordinates of point C are (3, 3).

Vertex D:We solve the system x + y = 6 and y = 0. Thecoordinates of point D are (6, 0).

Evaluate the objective function Q at each vertex.

Vertex Q = 2x + 3y(1, 0) 2 · 1 + 3 · 0 = 2(

1,53

)2 · 1 + 3 · 5

3= 7

(3, 3) 2 · 3 + 3 · 3 = 15

(6, 0) 2 · 6 + 3 · 0 = 12

The maximum value of Q is 15 when x = 3 and y = 3.

The minimum value of Q is 2 when x = 1 and y = 0.

Copyright © 2013 Pearson Education, Inc.

Page 276: JUDITH A. PENNA -  · PDF fileSTUDENT’S SOLUTIONS MANUAL JUDITH A. PENNA Indiana University Purdue University Indianapolis COLLEGE ALGEBRA: GRAPHS AND MODELS FIFTH EDITION

x7050 6030 9080 100402010

y

70

80

90

100

50

60

40

30

20

10 (25, 15)

(85, 15)

(25, 75)

Chapter 6 Test 273

22. Let x = the number of coffee cakes prepared and y =the number of cheesecakes. Find the maximum value ofP = 8x + 17y subject to

x + y ≤ 100,

x ≥ 25,

y ≥ 15

Graph the system of inequalities, determine the vertices,and find the value of P at each vertex.

Vertex P = 8x + 17y(25, 15) 8 · 25 + 17 · 15 = 455

(25, 75) 8 · 25 + 17 · 75 = 1475

(85, 15) 8 · 85 + 17 · 15 = 935

The maximum profit of $1475 occurs when 25 coffee cakesand 75 cheesecakes are prepared.

23. 3x− 11x2 + 2x− 3

=3x− 11

(x− 1)(x + 3)

=A

x− 1+

B

x + 3

=A(x + 3) + B(x− 1)

(x− 1)(x + 3)Equate the numerators.

3x− 11 = A(x + 3) + B(x− 1)

Let x = −3 : 3(−3) − 11 = 0 + B(−3 − 1)

−20 = −4B

5 = B

Let x = 1 : 3(1) − 11 = A(1 + 3) + 0

−8 = 4A

−2 = A

The decomposition is − 2x− 1

+5

x + 3.

24. Graph the system of inequalities. We see that D is thecorrect graph.

25. Solve:A(2) −B(−2) = C(2) − 8

A(−3) −B(−1) = C(1) − 8

A(4) −B(2) = C(9) − 8or2A + 2B − 2C = −8

−3A + B − C = −8

4A− 2B − 9C = −8

The solution is (1,−3, 2), so A = 1, B = −3, and C = 2.

Copyright © 2013 Pearson Education, Inc.

Page 277: JUDITH A. PENNA -  · PDF fileSTUDENT’S SOLUTIONS MANUAL JUDITH A. PENNA Indiana University Purdue University Indianapolis COLLEGE ALGEBRA: GRAPHS AND MODELS FIFTH EDITION

Copyright © 2013 Pearson Education, Inc.

Page 278: JUDITH A. PENNA -  · PDF fileSTUDENT’S SOLUTIONS MANUAL JUDITH A. PENNA Indiana University Purdue University Indianapolis COLLEGE ALGEBRA: GRAPHS AND MODELS FIFTH EDITION

4

2

�2

�4

42�2�4 x

y

x2 � 20y

4

2

�2

�4

42�2�4 x

y

y2 � �6x

4

2

�2

�4

42�2�4 x

y

x2 � 4y � 0

21�1

�1

1

2

�2

�2 x

y

x � 2y2

Chapter 7

Conic Sections

Exercise Set 7.1

1. Graph (f) is the graph of x2 = 8y.

3. Graph (b) is the graph of (y − 2)2 = −3(x + 4).

5. Graph (d) is the graph of 13x2 − 8y − 9 = 0.

7. x2 = 20y

x2 = 4 · 5 · y Writing x2 = 4py

Vertex: (0, 0)

Focus: (0, 5) [(0, p)]

Directrix: y = −5 (y = −p)

9. y2 = −6x

y2 = 4(− 3

2

)x Writing y2 = 4px

Vertex: (0, 0)

Focus:(− 3

2, 0)

[(p, 0)]

Directrix: x = −(− 3

2

)=

32

(x = −p)

11. x2 − 4y = 0

x2 = 4y

x2 = 4 · 1 · y Writing x2 = 4py

Vertex: (0, 0)

Focus: (0, 1) [(0, p)]

Directrix: y = −1 (y = −p)

13. x = 2y2

y2 =12x

y2 = 4 · 18· x Writing y2 = 4px

Vertex: (0, 0)

Focus:(

18, 0)

Directrix: x = −18

15. The vertex is at the origin and the focus is (−3, 0), so theequation is of the form y2 = 4px where p = −3.

y2 = 4px

y2 = 4(−3)x

y2 = −12x

17. Since the directrix, x = −4, is a vertical line, the equationis of the form (y − k)2 = 4p(x − h). The focus, (4, 0),is on the x-axis so the axis of symmetry is the x-axis andp = 4. The vertex, (h, k), is the point on the x-axis midwaybetween the directrix and the focus. Thus, it is (0, 0). Wehave

(y − k)2 = 4p(x− h)

(y − 0)2 = 4 · 4(x− 0) Substituting

y2 = 16x.

19. Since the directrix, y = π, is a horizontal line, the equationis of the form (x− h)2 = 4p(y − k). The focus, (0,−π), ison the y-axis so the axis of symmetry is the y-axis and p =−π. The vertex (h, k) is the point on the y-axis midwaybetween the directrix and the focus. Thus, it is (0, 0). Wehave

(x− h)2 = 4p(y − k)

(x− 0)2 = 4(−π)(y − 0) Substituting

x2 = −4πy

Copyright © 2013 Pearson Education, Inc.

Page 279: JUDITH A. PENNA -  · PDF fileSTUDENT’S SOLUTIONS MANUAL JUDITH A. PENNA Indiana University Purdue University Indianapolis COLLEGE ALGEBRA: GRAPHS AND MODELS FIFTH EDITION

4

2

�2

�4

42�4 x

y

(x � 2)2 � �6(y � 1)

2

�2

�4

42�2�4 x

y

x2 � 2x � 2y � 7 � 0

�6

4

2

�4

42�2�4 x

y

x2 � y � 2 � 0

276 Chapter 7: Conic Sections

21. Since the directrix, x = −4, is a vertical line, the equationis of the form (y − k)2 = 4p(x − h). The focus, (3, 2), ison the horizontal line y = 2, so the axis of symmetry isy = 2. The vertex is the point on the line y = 2 thatis midway between the directrix and the focus. That is,it is the midpoint of the segment from (−4, 2) to (3, 2):(−4 + 3

2,2 + 2

2

), or

(− 1

2, 2)

. Then h = −12

and the

directrix is x = h− p, so we have

x = h− p

−4 = −12− p

−72

= −p

72

= p.

Now we find the equation of the parabola.

(y − k)2 = 4p(x− h)

(y − 2)2 = 4(

72

)[x−

(− 1

2

)]

(y − 2)2 = 14(x +

12

)

23. (x + 2)2=−6(y − 1)

[x−(−2)]2=4(− 3

2

)(y−1) [(x−h)2 = 4p(y−k)]

Vertex: (−2, 1) [(h, k)]

Focus:(− 2, 1 +

(− 3

2

)), or

(− 2,−1

2

)[(h, k + p)]

Directrix: y = 1 −(− 3

2

)=

52

(y = k − p)

25. x2 + 2x + 2y + 7 = 0

x2 + 2x = −2y − 7

(x2 + 2x + 1) = −2y − 7 + 1 = −2y − 6

(x + 1)2 = −2(y + 3)

[x− (−1)]2 = 4(− 1

2

)[y − (−3)]

[(x− h)2 = 4p(y − k)]

Vertex: (−1,−3) [(h, k)]

Focus:(− 1,−3+

(− 1

2

)), or

(− 1,−7

2

)[(h, k+p)]

Directrix: y = −3 −(− 1

2

)= −5

2(y = k − p)

27. x2 − y − 2 = 0

x2 = y + 2

(x− 0)2 = 4 · 14· [y − (−2)]

[(x− h)2 = 4p(y − k)]

Vertex: (0,−2) [(h, k)]

Focus:(

0,−2 +14

), or

(0,−7

4

)[(h, k + p)]

Directrix: y = −2 − 14

= −94

(y = k − p)

29. y = x2 + 4x + 3

y − 3 = x2 + 4x

y − 3 + 4 = x2 + 4x + 4

y + 1 = (x + 2)2

4 · 14· [y − (−1)] = [x− (−2)]2

[(x− h)2 = 4p(y − k)]

Vertex: (−2,−1) [(h, k)]

Focus:(− 2,−1 +

14

), or

(− 2,−3

4

)[(h, k + p)]

Directrix: y = −1− 14

= −54

(y = k−p)

Copyright © 2013 Pearson Education, Inc.

Page 280: JUDITH A. PENNA -  · PDF fileSTUDENT’S SOLUTIONS MANUAL JUDITH A. PENNA Indiana University Purdue University Indianapolis COLLEGE ALGEBRA: GRAPHS AND MODELS FIFTH EDITION

4

2

�2

�4

42�4 x

y

y � x2 � 4x � 3

4

2

�2

�4

8642

y

y2 � y � x � 6 � 0

x

y

x

( x, — )152

( x, – — )152

x15 ft

Exercise Set 7.1 277

31. y2 − y − x + 6 = 0

y2 − y = x− 6

y2 − y +14

= x− 6 +14(

y − 12

)2

= x− 234(

y − 12

)2

= 4 · 14

(x− 23

4

)[(y − k)2 = 4p(x− h)]

Vertex:(

234,12

)[(h, k)]

Focus:(

234

+14,12

), or

(6,

12

)[(h + p, k)]

Directrix: x =234

− 14

=224

or112

(x = h− p)

33. a) The vertex is (0, 0). The focus is (4, 0), so p = 4.The parabola has a horizontal axis of symmetry sothe equation is of the form y2 = 4px. We have

y2 = 4px

y2 = 4 · 4 · xy2 = 16x

b) We make a drawing.

The depth of the satellite dish at the vertex is x

where(x,

152

)is a point on the parabola.

y2 = 16x(152

)2

= 16x Substituting152

for y

2254

= 16x

22564

= x, or

33364

= x

The depth of the satellite dish at the vertex is 33364

ft.

35. We position a coordinate system with the origin at thevertex and the x-axis on the parabola’s axis of symmetry.The parabola is of the form y2 = 4px with p = 3.287.Because the parabola is 2.625 in. deep, a point on theparabola is (2.625, y), where y = one-half the diameter ofthe outside edge of the receiver.

y2 = 4px

y2 = 4 · 3.287 · 2.625

y2 = 34.5135

y ≈ 5.8748

The width at the opening is about 2 · 5.8748 in., or about11.75 in.

37. When we let y = 0 and solve for x, the only equation for

which x =23

is (h), so only equation (h) has x-intercept(23, 0)

.

39. Note that equation (g) is equivalent to y = 2x− 74

and

equation (h) is equivalent to y = −12x +

13. When we look

at the equations in the form y = mx+b, we see that m > 0for (a), (b), (f), and (g) so these equations have positiveslope, or slant up front left to right.

41. When we look at the equations in the form y = mx+b (See

Exercise 39.), only (b) has m =13

so only (b) has slope13.

43. Parallel lines have the same slope and different y-intercepts. When we look at the equations in the formy = mx + b (See Exercise 39.), we see that (a) and (g)represent parallel lines.

45. A parabola with a vertical axis of symmetry has an equa-tion of the type (x− h)2 = 4p(y − k).

Solve for p substituting (−1, 2) for (h, k) and (−3, 1) for(x, y).

[−3 − (−1)]2 = 4p(1 − 2)

4 = −4p

−1 = p

The equation of the parabola is

Copyright © 2013 Pearson Education, Inc.

Page 281: JUDITH A. PENNA -  · PDF fileSTUDENT’S SOLUTIONS MANUAL JUDITH A. PENNA Indiana University Purdue University Indianapolis COLLEGE ALGEBRA: GRAPHS AND MODELS FIFTH EDITION

y

x

20 40 60 80 100

(–100, 50) (100, 50)

(0, 10)

8

4

�8

8 124�4�8 x

y

x2 � y2 � 14x � 4y � 11

x2 � y2 � 6x � 2y � 6

2�4�6 �2

2

4

�4

�2

x

y

278 Chapter 7: Conic Sections

[x− (−1)]2 = 4(−1)(y − 2), or

(x + 1)2 = −4(y − 2).

47. Vertex: (0.867, 0.348)

Focus: (0.867,−0.190)

Directrix: y = 0.887

49. Position a coordinate system as shown below with the y-axis on the parabola’s axis of symmetry.

The equation of the parabola is of the form (x − h)2 =4p(y − k). Substitute 100 for x, 50 for y, 0 for h, and 10for k and solve for p.

(x− h)2 = 4p(y − k)

(100 − 0)2 = 4p(50 − 10)

10, 000 = 160p2504

= p

Then the equation is

x2 = 4(

2504

)(y − 10), or

x2 = 250(y − 10).

To find the lengths of the vertical cables, find y when x = 0,20, 40, 60, 80, and 100.

When x = 0 : 02 = 250(y − 10)

0 = y − 10

10 = y

When x = 20 : 202 = 250(y − 10)

400 = 250(y − 10)

1.6 = y − 10

11.6 = y

When x = 40 : 402 = 250(y − 10)

1600 = 250(y − 10)

6.4 = y − 10

16.4 = y

When x = 60 : 602 = 250(y − 10)

3600 = 250(y − 10)

14.4 = y − 10

24.4 = y

When x = 80 : 802 = 250(y − 10)

6400 = 250(y − 10)

25.6 = y − 10

35.6 = y

When x = 100, we know from the given information thaty = 50.

The lengths of the vertical cables are 10 ft, 11.6 ft, 16.4 ft,24.4 ft, 35.6 ft, and 50 ft.

Exercise Set 7.2

1. Graph (b) is the graph of x2 + y2 = 5.

3. Graph (d) is the graph of x2 + y2 − 6x + 2y = 6.

5. Graph (a) is the graph of x2 + y2 − 5x + 3y = 0.

7. Complete the square twice.x2 + y2 − 14x + 4y = 11

x2 − 14x + y2 + 4y = 11

x2 − 14x + 49 + y2 + 4y + 4 = 11 + 49 + 4

(x− 7)2 + (y + 2)2 = 64

(x− 7)2 + [y − (−2)]2 = 82

Center: (7,−2)

Radius: 8

9. Complete the square twice.x2 + y2 + 6x− 2y = 6

x2 + 6x + y2 − 2y = 6

x2 + 6x + 9 + y2 − 2y + 1 = 6 + 9 + 1

(x + 3)2 + (y − 1)2 = 16

[x− (−3)]2 + (y − 1)2 = 42

Center: (−3, 1)

Radius: 4

11. Complete the square twice.x2 + y2 + 4x− 6y − 12 = 0

x2 + 4x + y2 − 6y = 12

x2 + 4x + 4 + y2 − 6y + 9 = 12 + 4 + 9

(x + 2)2 + (y − 3)2 = 25

[x− (−2)]2 + (y − 3)2 = 52

Copyright © 2013 Pearson Education, Inc.

Page 282: JUDITH A. PENNA -  · PDF fileSTUDENT’S SOLUTIONS MANUAL JUDITH A. PENNA Indiana University Purdue University Indianapolis COLLEGE ALGEBRA: GRAPHS AND MODELS FIFTH EDITION

4

6

2

�4

42�2�4

y

x

x2 � y2 � 4x � 6y � 12 � 0

x2 � y2 � 6x � 8y � 16 � 0

2 4 6�2

2

4

6

�2

x

y

8

4

�4

�8

84�8 x

y

x2 � y2 � 6x � 10y � 0

8

4

�8

84�4�8 x

y

x2 � y2 � 9x � 7 � 4y

4

2

�2

�4

4�4 x

y

x2

4y2

1� � 1

Exercise Set 7.2 279

Center: (−2, 3)

Radius: 5

13. Complete the square twice.

x2 + y2 − 6x− 8y + 16 = 0

x2 − 6x + y2 − 8y = −16

x2 − 6x + 9 + y2 − 8y + 16 = −16 + 9 + 16

(x− 3)2 + (y − 4)2 = 9

(x− 3)2 + (y − 4)2 = 32

Center: (3, 4)

Radius: 3

15. Complete the square twice.

x2 + y2 + 6x− 10y = 0

x2 + 6x + y2 − 10y = 0

x2 + 6x + 9 + y2 − 10y + 25 = 0 + 9 + 25

(x + 3)2 + (y − 5)2 = 34

[x− (−3)]2 + (y − 5)2 = (√

34)2

Center: (−3, 5)

Radius:√

34

17. Complete the square twice.

x2 + y2 − 9x = 7 − 4y

x2 − 9x + y2 + 4y = 7

x2 − 9x +814

+ y2 + 4y + 4 = 7 +814

+ 4

(x− 9

2

)2

+ (y + 2)2 =1254(

x− 92

)2

+ [y − (−2)]2 =(

5√

52

)2

Center:(

92,−2

)

Radius:5√

52

19. Graph (c) is the graph of 16x2 + 4y2 = 64.

21. Graph (d) is the graph of x2 + 9y2 − 6x + 90y = −225.

23. x2

4+

y2

1= 1

x2

22+

y2

12= 1 Standard form

a = 2, b = 1

The major axis is horizontal, so the vertices are (−2, 0)and (2, 0). Since we know that c2 = a2 − b2, we havec2 = 4 − 1 = 3, so c =

√3 and the foci are (−√

3, 0) and(√

3, 0).

To graph the ellipse, plot the vertices. Note also that sinceb = 1, the y-intercepts are (0,−1) and (0, 1). Plot thesepoints as well and connect the four plotted points with asmooth curve.

Copyright © 2013 Pearson Education, Inc.

Page 283: JUDITH A. PENNA -  · PDF fileSTUDENT’S SOLUTIONS MANUAL JUDITH A. PENNA Indiana University Purdue University Indianapolis COLLEGE ALGEBRA: GRAPHS AND MODELS FIFTH EDITION

2

�2

42�2�4 x

y

16x2 � 9y2 � 144

4

2

�2

�4

42�2�4 x

y

2x2 � 3y2 � 6

1

�1

1�1 x

y

4x2 � 9y2 � 1

280 Chapter 7: Conic Sections

25. 16x2 + 9y2 = 144

x2

9+

y2

16= 1 Dividing by 144

x2

32+

y2

42= 1 Standard form

a = 4, b = 3

The major axis is vertical, so the vertices are (0,−4) and(0, 4). Since c2 = a2 − b2, we have c2 = 16 − 9 = 7, soc =

√7 and the foci are (0,−√

7) and (0,√

7).

To graph the ellipse, plot the vertices. Note also that sinceb = 3, the x-intercepts are (−3, 0) and (3, 0). Plot thesepoints as well and connect the four plotted points with asmooth curve.

27. 2x2 + 3y2 = 6

x2

3+

y2

2= 1

x2

(√

3)2+

y2

(√

2)2= 1

a =√

3, b =√

2

The major axis is horizontal, so the vertices are (−√3, 0)

and (√

3, 0). Since c2 = a2 − b2, we have c2 = 3 − 2 = 1,so c = 1 and the foci are (−1, 0) and (1, 0).

To graph the ellipse, plot the vertices. Note also that sinceb =

√2, the y-intercepts are (0,−√

2) and (0,√

2). Plotthese points as well and connect the four plotted pointswith a smooth curve.

29. 4x2 + 9y2 = 1

x2

14

+y2

19

= 1

x2(12

)2 +y2(13

)2 = 1

a =12, b =

13

The major axis is horizontal, so the vertices are(− 1

2, 0)

and(

12, 0)

. Since c2 = a2−b2, we have c2 =14− 1

9=

536

,

so c =√

56

and the foci are(−

√5

6, 0)

and(√

56

, 0)

.

To graph the ellipse, plot the vertices. Note also that since

b =13, the y-intercepts are

(0,−1

3

)and

(0,

13

). Plot

these points as well and connect the four plotted pointswith a smooth curve.

31. The vertices are on the x-axis, so the major axis is hori-zontal. We have a = 7 and c = 3, so we can find b2:

c2 = a2 − b2

32 = 72 − b2

b2 = 49 − 9 = 40Write the equation:

x2

a2+

y2

b2= 1

x2

49+

y2

40= 1

33. The vertices, (0,−8) and (0, 8), are on the y-axis, so themajor axis is vertical and a = 8. Since the vertices areequidistant from the origin, the center of the ellipse is atthe origin. The length of the minor axis is 10, so b = 10/2,or 5.

Write the equation:x2

b2+

y2

a2= 1

x2

52+

y2

82= 1

x2

25+

y2

64= 1

35. The foci, (−2, 0) and (2, 0) are on the x-axis, so the majoraxis is horizontal and c = 2. Since the foci are equidistantfrom the origin, the center of the ellipse is at the origin.The length of the major axis is 6, so a = 6/2, or 3. Nowwe find b2:

c2 = a2 − b2

22 = 32 − b2

4 = 9 − b2

b2 = 5

Copyright © 2013 Pearson Education, Inc.

Page 284: JUDITH A. PENNA -  · PDF fileSTUDENT’S SOLUTIONS MANUAL JUDITH A. PENNA Indiana University Purdue University Indianapolis COLLEGE ALGEBRA: GRAPHS AND MODELS FIFTH EDITION

2

6

�2

42�2�4 x

y

(x � 1)2

9(y � 2)2

4� � 1

8

4

�4

�8

84�8 x

y

(x � 3)2

25(y � 5)2

36� � 1

4

�4

�8

84�4�8 x

y

3(x � 2)2 � 4(y � 1)2 � 192

Exercise Set 7.2 281

Write the equation:x2

a2+

y2

b2= 1

x2

9+

y2

5= 1

37. (x− 1)2

9+

(y − 2)2

4= 1

(x− 1)2

32+

(y − 2)2

22= 1 Standard form

The center is (1, 2). Note that a = 3 and b = 2. The majoraxis is horizontal so the vertices are 3 units left and rightof the center:

(1 − 3, 2) and (1 + 3, 2), or (−2, 2) and (4, 2).

We know that c2 = a2 − b2, so c2 = 9− 4 = 5 and c =√

5.Then the foci are

√5 units left and right of the center:

(1 −√5, 2) and (1 +

√5, 2).

To graph the ellipse, plot the vertices. Since b = 2, twoother points on the graph are 2 units below and above thecenter:

(1, 2 − 2) and (1, 2 + 2) or (1, 0) and (1, 4)

Plot these points also and connect the four plotted pointswith a smooth curve.

39. (x + 3)2

25+

(y − 5)2

36= 1

[x− (−3)]2

52+

(y − 5)2

62= 1 Standard form

The center is (−3, 5). Note that a = 6 and b = 5. Themajor axis is vertical so the vertices are 6 units below andabove the center:

(−3, 5 − 6) and (−3, 5 + 6), or (−3,−1) and (−3, 11).

We know that c2 = a2 − b2, so c2 = 36 − 25 = 11 andc =

√11. Then the foci are

√11 units below and above

the vertex:

(−3, 5 −√11) and (−3, 5 +

√11).

To graph the ellipse, plot the vertices. Since b = 5, twoother points on the graph are 5 units left and right of thecenter:

(−3 − 5, 5) and (−3 + 5, 5), or (−8, 5) and (2, 5)

Plot these points also and connect the four plotted pointswith a smooth curve.

41. 3(x + 2)2 + 4(y − 1)2 = 192

(x + 2)2

64+

(y − 1)2

48= 1 Dividing by 192

[x− (−2)]2

82+

(y − 1)2

(√

48)2= 1 Standard form

The center is (−2, 1). Note that a = 8 and b =√

48, or4√

3. The major axis is horizontal so the vertices are 8units left and right of the center:

(−2 − 8, 1) and (−2 + 8, 1), or (−10, 1) and (6, 1).

We know that c2 = a2−b2, so c2 = 64−48 = 16 and c = 4.Then the foci are 4 units left and right of the center:

(−2 − 4, 1) and (−2 + 4, 1) or (−6, 1) and (2, 1).

To graph the ellipse, plot the vertices. Since b = 4√

3 ≈6.928, two other points on the graph are about 6.928 unitsbelow and above the center:

(−2, 1 − 6.928) and (−2, 1 + 6.928), or

(−2,−5.928) and (−2, 7.928).

Plot these points also and connect the four plotted pointswith a smooth curve.

43. Begin by completing the square twice.4x2 + 9y2 − 16x + 18y − 11 = 0

4x2 − 16x + 9y2 + 18y = 11

4(x2 − 4x) + 9(y2 + 2y) = 11

4(x2 − 4x + 4) + 9(y2 + 2y + 1) = 11 + 4 · 4 + 9 · 14(x− 2)2 + 9(y + 1)2 = 36

(x− 2)2

9+

(y + 1)2

4= 1

(x− 2)2

32+

[y − (−1)]2

22= 1

The center is (2,−1). Note that a = 3 and b = 2. Themajor axis is horizontal so the vertices are 3 units left andright of the center:

Copyright © 2013 Pearson Education, Inc.

Page 285: JUDITH A. PENNA -  · PDF fileSTUDENT’S SOLUTIONS MANUAL JUDITH A. PENNA Indiana University Purdue University Indianapolis COLLEGE ALGEBRA: GRAPHS AND MODELS FIFTH EDITION

4

2

�4

42�2�4 x

y

4x2 � 9y2 � 16x � 18y � 11 � 0

4

2

�2

�4

42�2�4 x

y

4x2 � y2 � 8x � 2y � 1 � 0

y

x

VFocus Sun

9.3 9.1

282 Chapter 7: Conic Sections

(2 − 3,−1) and (2 + 3,−1), or (−1,−1) and (5,−1).

We know that c2 = a2 − b2, so c2 = 9− 4 = 5 and c =√

5.Then the foci are

√5 units left and right of the center:

(2 −√5,−1) and (2 +

√5,−1).

To graph the ellipse, plot the vertices. Since b = 2, twoother points on the graph are 2 units below and above thecenter:

(2,−1 − 2) and (2,−1 + 2), or (2,−3) and (2, 1).

Plot these points also and connect the four plotted pointswith a smooth curve.

45. Begin by completing the square twice.

4x2 + y2 − 8x− 2y + 1 = 0

4x2 − 8x + y2 − 2y = −1

4(x2 − 2x) + y2 − 2y = −1

4(x2 − 2x + 1) + y2 − 2y + 1 = −1 + 4 · 1 + 1

4(x− 1)2 + (y − 1)2 = 4

(x− 1)2

1+

(y − 1)2

4= 1

(x− 1)2

12+

(y − 1)2

22= 1

The center is (1, 1). Note that a = 2 and b = 1. The majoraxis is vertical so the vertices are 2 units below and abovethe center:

(1, 1 − 2) and (1, 1 + 2), or (1,−1) and (1, 3).

We know that c2 = a2 − b2, so c2 = 4− 1 = 3 and c =√

3.Then the foci are

√3 units below and above the center:

(1, 1 −√3) and (1, 1 +

√3).

To graph the ellipse, plot the vertices. Since b = 1, twoother points on the graph are 1 unit left and right of thecenter:

(1 − 1, 1) and (1 + 1, 1) or (0, 1) and (2, 1).

Plot these points also and connect the four plotted pointswith a smooth curve.

47. The ellipse in Example 4 is flatter than the one in Example2, so the ellipse in Example 2 has the smaller eccentricity.

We compute the eccentricities: In Example 2, c = 3 anda = 5, so e = c/a = 3/5 = 0.6. In Example 4, c =2√

3 and a = 4, so e = c/a = 2√

3/4 ≈ 0.866. Thesecomputations confirm that the ellipse in Example 2 hasthe smaller eccentricity.

49. Since the vertices, (0,−4) and (0, 4) are on the y-axis andare equidistant from the origin, we know that the majoraxis of the ellipse is vertical, its center is at the origin, anda = 4. Use the information that e = 1/4 to find c:

e =c

a

14

=c

4Substituting

c = 1

Now c2 = a2 − b2, so we can find b2:12 = 42 − b2

1 = 16 − b2

b2 = 15

Write the equation of the ellipse:x2

b2+

y2

a2= 1

x2

15+

y2

16= 1

51. From the figure in the text we see that the center of theellipse is (0, 0), the major axis is horizontal, the vertices are(−50, 0) and (50, 0), and one y-intercept is (0, 12). Thena = 50 and b = 12. The equation is

x2

a2+

y2

b2= 1

x2

502+

y

122= 1

x2

2500+

y2

144= 1.

53. Position a coordinate system as shown below where1 unit = 107 mi.

The length of the major axis is 9.3 + 9.1, or 18.4. Thenthe distance from the center of the ellipse (the origin) toV is 18.4/2, or 9.2. Since the distance from the sun to Vis 9.1, the distance from the sun to the center is 9.2− 9.1,or 0.1. Then the distance from the sun to the other focusis twice this distance:

2(0.1 × 107 mi) = 0.2 × 107 mi

= 2 × 106 mi

55. midpoint

Copyright © 2013 Pearson Education, Inc.

Page 286: JUDITH A. PENNA -  · PDF fileSTUDENT’S SOLUTIONS MANUAL JUDITH A. PENNA Indiana University Purdue University Indianapolis COLLEGE ALGEBRA: GRAPHS AND MODELS FIFTH EDITION

y

x

(0, 14)

(–25, 0) (25, 0)

2 4

2

4

�2�4

�2

�4y2 � 12x

x

y

Chapter 7 Mid-Chapter Mixed Review 283

57. y-intercept

59. remainder

61. parabola

63. The center of the ellipse is the midpoint of the segmentconnecting the vertices:(

3 + 32

,−4 + 6

2

), or (3, 1).

Now a is the distance from the origin to a vertex. We usethe vertex (3, 6).

a =√

(3 − 3)2 + (6 − 1)2 = 5

Also b is one-half the length of the minor axis.

b =

√(5 − 1)2 + (1 − 1)2

2=

42

= 2

The vertices lie on the vertical line x = 3, so the majoraxis is vertical. We write the equation of the ellipse.

(x− h)2

b2+

(y − k)2

a2= 1

(x− 3)2

4+

(y − 1)2

25= 1

65. The center is the midpoint of the segment connecting thevertices:(−3 + 3

2,0 + 0

2

), or (0, 0).

Then a = 3 and since the vertices are on the x-axis, themajor axis is horizontal. The equation is of the formx2

a2+

y2

b2= 1.

Substitute 3 for a, 2 for x, and223

for y and solve for b2.

49

+

4849b2

= 1

49

+4849b2

= 1

4b2 + 484 = 9b2

484 = 5b2

4845

= b2

Then the equation isx2

9+

y2

484/5= 1.

67. Center: (2.003,−1.005)

Vertices: (−1.017,−1.005), (5.023,−1.005)

69. Position a coordinate system as shown.

The equation of the ellipse isx2

252+

y2

142= 1

x2

625+

y2

196= 1.

A point 6 ft from the riverbank corresponds to (25− 6, 0),or (19, 0) or to (−25 + 6, 0), or (−19, 0). Substitute either19 or −19 for x and solve for y, the clearance.

192

625+

y2

196= 1

y2

196= 1 − 361

625

y2 = 196(

1 − 361625

)y ≈ 9.1

The clearance 6 ft from the riverbank is about 9.1 ft.

Chapter 7 Mid-Chapter Mixed Review

1. The equation (x+3)2 = 8(y−2) is equivalent to the equa-tion [x − (−3)]2 = 4 · 2(y − 2), so the given statement istrue. See page 581 in the text.

3. The equation (x − 4)2 + (y + 1)2 = 9 is equivalent to theequation (x− 4)2 + [y − (−1)]2 = 32. This is the equationof a circle with center (4,−1) and radius 3, so the givenstatement is false.

5. Graph (c) is the graph of x2 = −4y.

7. Graph (d) is the graph of 16x2 + 9y2 = 144.

9. Graph (b) is the graph of (x− 1)2 = 2(y + 3).

11. Graph (g) is the graph of (x− 2)2 + (y + 3)2 = 4.

13. y2 = 12x

y2 = 4 · 3 · xVertex: (0, 0)

Focus: (3, 0)

Directrix: x = −3

15. Since the directrix, y = 1, is a horizontal line, the equationis of the form (x − h)2 = 4p(y − k). The focus, (0, 3), ison the y-axis so the axis of symmetry is the y-axis. Thevertex is the point on the y-axis that is midway betweenthe directrix and the focus. That is, it is the midpoint of

Copyright © 2013 Pearson Education, Inc.

Page 287: JUDITH A. PENNA -  · PDF fileSTUDENT’S SOLUTIONS MANUAL JUDITH A. PENNA Indiana University Purdue University Indianapolis COLLEGE ALGEBRA: GRAPHS AND MODELS FIFTH EDITION

2

2

4

6

8

�2�6

x2 � y2 � 4x � 8y � 5

x

y

2 4

2

4

�2�4

�2

�4

x

y

x2

1y2

9� � 1

2 4 6

2

4

�2

�4

�2

x

y

(x � 2)2

16(y � 1)2

� � 14

284 Chapter 7: Conic Sections

the segment from (0, 1) to (0, 3):(

0 + 02

,1 + 3

2

), or (0, 2).

Then k = 2 and the directrix is y = k − p, so we have

y = k − p

1 = 2 − p

p = 1.

Now we find the equation of the parabola.

(x− h)2 = 4p(y − k)

(x− 0)2 = 4 · 1 · (y − 2)

x2 = 4(y − 2)

17. x2 + y2 + 4x− 8y = 5

x2 + 4x + y2 − 8y = 5

x2 + 4x + 4 + y2 − 8y + 16 = 5 + 4 + 16

(x + 2)2 + (y − 4)2 = 25

[x− (−2)]2 + (y − 4)2 = 52

Center: (−2, 4); radius: 5

19. x2

1+

y2

9= 1

x2

12+

y2

32= 1

a = 3, b = 1

The major axis is vertical, so the vertices are (0,−3) and(0, 3). Since c2 = a2 − b2 we have c2 = 9 − 1 = 8, soc =

√8, or 2

√2, and the foci are (0,−2

√2) and (0, 2

√2).

21. (x− 2)2

16+

(y + 1)2

4= 1

(x− 2)2

42+

[y − (−1)]2

22= 1

The center is (2,−1). Note that a = 4 and b = 2. Themajor axis is horizontal, so the vertices are 4 units to theleft and right of the center:

(2 − 4,−1) and (2 + 4,−1), or (−2,−1) and (6,−1).

We know that c2 = a2 − b2 so c2 = 16 − 4 = 12 andc =

√12, or 2

√3. Then the foci are 2

√3 units to the left

and right of the center:

(2 − 2√

3,−1) and (2 + 2√

3,−1).

23. The vertices, (−5, 0) and (5, 0) are on the x-axis, so themajor axis is horizontal and a = 5. Since the vertices areequidistant from the origin, the center of the ellipse is theorigin. The foci are (−2, 0) and (2, 0), so we know thatc = 2. Then we have

c2 = a2 − b2

4 = 25 − b2

b2 = 21.

Now we write the equation of the ellipse.x2

25+

y2

21= 1

25. The foci, (−3, 0) and (3, 0) are on the x-axis, so the majoraxis is horizontal. The foci are equidistant from the origin,so the center of the ellipse is the origin. The length of themajor axis is 8, so a = 8/2, or 4. Now we find b2.

c2 = a2 − b2

9 = 16 − b2

b2 = 7

We write the equation of the ellipse.x2

16+

y2

7= 1

27. See page 579 of the text.

29. No, the center of an ellipse is not part of the graph of theellipse. Its coordinates do not satisfy the equation of theellipse.

Exercise Set 7.3

1. Graph (b) is the graph ofx2

25− y2

9= 1.

3. Graph (c) is the graph of(y − 1)2

16− (x + 3)2

1= 1.

5. Graph (a) is the graph of 25x2 − 16y2 = 400.

Copyright © 2013 Pearson Education, Inc.

Page 288: JUDITH A. PENNA -  · PDF fileSTUDENT’S SOLUTIONS MANUAL JUDITH A. PENNA Indiana University Purdue University Indianapolis COLLEGE ALGEBRA: GRAPHS AND MODELS FIFTH EDITION

4

2

�2

�4

4�4 x

y

x2

4y2

4� � 1

44 6

2

�2

42�2�4 x

y

(x � 2)2

9(y � 5)2

1� � 1

Exercise Set 7.3 285

7. The vertices are equidistant from the origin and are on they-axis, so the center is at the origin and the transverse axisis vertical. Since c2 = a2 + b2, we have 52 = 32 + b2 sob2 = 16.

The equation is of the formy2

a2− x2

b2= 1, so we have

y2

9− x2

16= 1.

9. The asymptotes pass through the origin, so the center isthe origin. The given vertex is on the x-axis, so the trans-

verse axis is horizontal. Sinceb

ax =

32x and a = 2, we

have b = 3. The equation is of the formx2

a2− y2

b2= 1, so

we havex2

22− y2

32= 1, or

x2

4− y2

9= 1.

11. x2

4− y2

4= 1

x2

22− y2

22= 1 Standard form

The center is (0, 0); a = 2 and b = 2. The transverse axisis horizontal so the vertices are (−2, 0) and (2, 0). Sincec2 = a2 + b2, we have c2 = 4 + 4 = 8 and c =

√8, or 2

√2.

Then the foci are (−2√

2, 0) and (2√

2, 0).

Find the asymptotes:

y =b

ax and y = − b

ax

y =22x and y = −2

2x

y = x and y = −xTo draw the graph sketch the asymptotes, plot the vertices,and draw the branches of the hyperbola outward from thevertices toward the asymptotes.

13. (x− 2)2

9− (y + 5)2

1= 1

(x− 2)2

32− [y − (−5)]2

12= 1 Standard form

The center is (2,−5); a = 3 and b = 1. The transverseaxis is horizontal, so the vertices are 3 units left and rightof the center:

(2 − 3,−5) and (2 + 3,−5), or (−1,−5) and (5,−5).

Since c2 = a2 + b2, we have c2 = 9 + 1 = 10 and c =√

10.Then the foci are

√10 units left and right of the center:

(2 −√10,−5) and (2 +

√10,−5).

Find the asymptotes:

y − k=b

a(x− h) and y − k=− b

a(x− h)

y − (−5)=13(x− 2) and y − (−5)=−1

3(x− 2)

y + 5=13(x− 2) and y + 5=−1

3(x− 2), or

y=13x− 17

3and y=−1

3x− 13

3Sketch the asymptotes, plot the vertices, and draw thegraph.

15. (y + 3)2

4− (x+ 1)2

16= 1

[y − (−3)]2

22− [x− (−1)]2

42= 1 Standard form

The center is (−1,−3); a = 2 and b = 4. The transverseaxis is vertical, so the vertices are 2 units below and abovethe center:

(−1,−3 − 2) and (1,−3 + 2), or (−1,−5) and (−1,−1).

Since c2 = a2 + b2, we have c2 = 4+16 = 20 and c =√

20,or 2

√5. Then the foci are 2

√5 units below and above of

the center:

(−1,−3 − 2√

5) and (−1,−3 + 2√

5).

Find the asymptotes:

y − k=a

b(x− h) and y − k=−a

b(x− h)

y−(−3)=24(x−(−1)) and y−(−3)=−2

4(x−(−1))

y+3=12(x+1) and y+3=−1

2(x+1), or

y=12x− 5

2and y=−1

2x− 7

2Sketch the asymptotes, plot the vertices, and draw thegraph.

Copyright © 2013 Pearson Education, Inc.

Page 289: JUDITH A. PENNA -  · PDF fileSTUDENT’S SOLUTIONS MANUAL JUDITH A. PENNA Indiana University Purdue University Indianapolis COLLEGE ALGEBRA: GRAPHS AND MODELS FIFTH EDITION

2

�2

�6

�4

6 x

y

(y � 3)2

4(x � 1)2

16� � 1

4

2

�2

�4

4�4 x

y

x2 � 4y2 � 4

4

�4

�8

�12

�16

8

12

16

8 12 16�8�16 x

y

9y2 � x2 � 81

4

2

�2

�4

42�2�4 x

y

x2 � y2 � 2

286 Chapter 7: Conic Sections

17. x2 − 4y2 = 4

x2

4− y2

1= 1

x2

22− y2

12= 1 Standard form

The center is (0, 0); a = 2 and b = 1. The transverse axisis horizontal, so the vertices are (−2, 0) and (2, 0). Sincec2 = a2 + b2, we have c2 = 4 + 1 = 5 and c =

√5. Then

the foci are (−√5, 0) and (

√5, 0).

Find the asymptotes:

y =b

ax and y = − b

ax

y =12x and y = −1

2x

Sketch the asymptotes, plot the vertices, and draw thegraph.

19. 9y2 − x2 = 81

y2

9− x2

81= 1

y2

32− x2

92= 1 Standard form

The center is (0, 0); a = 3 and b = 9. The transverseaxis is vertical, so the vertices are (0,−3) and (0, 3). Sincec2 = a2 + b2, we have c2 = 9 + 81 = 90 and c =

√90, or

3√

10. Then the foci are (0,−3√

10) and (0, 3√

10).

Find the asymptotes:

y =a

bx and y = −a

bx

y =39x and y = −3

9x

y =13x and y = −1

3x

Sketch the asymptotes, plot the vertices, and draw thegraph.

21. x2 − y2 = 2

x2

2− y2

2= 1

x2

(√

2)2− y2

(√

2)2= 1 Standard form

The center is (0, 0); a =√

2 and b =√

2. The transverseaxis is horizontal, so the vertices are (−√

2, 0) and (√

2, 0).Since c2 = a2 + b2, we have c2 = 2 + 2 = 4 and c = 2.Then the foci are (−2, 0) and (2, 0).

Find the asymptotes:

y =b

ax and y = − b

ax

y =√

2√2x and y = −

√2√2x

y = x and y = −xSketch the asymptotes, plot the vertices, and draw thegraph.

23. y2 − x2 =14

y2

1/4− x2

1/4= 1

y2

(1/2)2− x2

(1/2)2= 1 Standard form

The center is (0, 0); a =12

and b =12. The transverse axis

is vertical, so the vertices are(

0,−12

)and

(0,

12

). Since

c2 = a2 + b2, we have c2 =14

+14

=12

and c =

√12, or

√2

2. Then the foci are

(0,−

√2

2

)and

(0,

√2

2

).

Copyright © 2013 Pearson Education, Inc.

Page 290: JUDITH A. PENNA -  · PDF fileSTUDENT’S SOLUTIONS MANUAL JUDITH A. PENNA Indiana University Purdue University Indianapolis COLLEGE ALGEBRA: GRAPHS AND MODELS FIFTH EDITION

1

�1

1�1 x

y

y2 � x2 � 1–4

2

�6

4 6�4 �2 x

y

x2 � y2 � 2x � 4y � 4 � 0

84�4�8 x

y

36x2 � y2 � 24x � 6y � 41 � 0

Exercise Set 7.3 287

Find the asymptotes:

y =a

bx and y = −a

bx

y =1/21/2

x and y = −1/21/2

x

y = x and y = −xSketch the asymptotes, plot the vertices, and draw thegraph.

25. Begin by completing the square twice.

x2 − y2 − 2x− 4y − 4 = 0

(x2 − 2x) − (y2 + 4y) = 4

(x2 − 2x+ 1) − (y2 + 4y + 4) = 4 + 1 − 1 · 4(x− 1)2 − (y + 2)2 = 1

(x− 1)2

12− [y − (−2)]2

12= 1 Standard

form

The center is (1,−2); a = 1 and b = 1. The transverseaxis is horizontal, so the vertices are 1 unit left and rightof the center:

(1 − 1,−2) and (1 + 1,−2) or (0,−2) and (2,−2)

Since c2 = a2 + b2, we have c2 = 1 + 1 = 2 and c =√

2.Then the foci are

√2 units left and right of the center:

(1 −√2,−2) and (1 +

√2,−2).

Find the asymptotes:

y − k =b

a(x− h) and y − k = − b

a(x− h)

y − (−2) =11(x− 1) and y − (−2) = −1

1(x− 1)

y + 2 = x− 1 and y + 2 = −(x− 1), or

y = x− 3 and y = −x− 1

Sketch the asymptotes, plot the vertices, and draw thegraph.

27. Begin by completing the square twice.

36x2 − y2 − 24x+ 6y − 41 = 0

(36x2 − 24x) − (y2 − 6y) = 41

36(x2 − 2

3x

)− (y2 − 6y) = 41

36(x2− 2

3x+

19

)−(y2−6y+9) = 41+36· 1

9−1 · 9

36(x− 1

3

)2

− (y − 3)2 = 36

(x− 1

3

)2

1− (y − 3)2

36= 1

(x− 1

3

)2

12− (y − 3)2

62= 1 Standard

form

The center is(

13, 3)

; a = 1 and b = 6. The transverse

axis is horizontal, so the vertices are 1 unit left and rightof the center:(

13− 1, 3

)and

(13

+ 1, 3)

or(− 2

3, 3)

and(43, 3)

.

Since c2 = a2 + b2, we have c2 = 1+36 = 37 and c =√

37.Then the foci are

√37 units left and right of the center:(

13−

√37, 3

)and

(13

+√

37, 3)

.

Find the asymptotes:

y − k =b

a(x− h) and y − k = − b

a(x− h)

y − 3 =61

(x− 1

3

)and y − 3 = −6

1

(x− 1

3

)

y − 3 = 6(x− 1

3

)and y − 3 = −6

(x− 1

3

), or

y = 6x+ 1 and y = −6x+ 5

Sketch the asymptotes, plot the vertices, and draw thegraph.

Copyright © 2013 Pearson Education, Inc.

Page 291: JUDITH A. PENNA -  · PDF fileSTUDENT’S SOLUTIONS MANUAL JUDITH A. PENNA Indiana University Purdue University Indianapolis COLLEGE ALGEBRA: GRAPHS AND MODELS FIFTH EDITION

6

2

�4

8642�2 x

y

9y2 � 4x2 � 18y � 24x � 63 � 0

4

2

42�2�4 x

y

x2 � y2 � 2x � 4y � 4

288 Chapter 7: Conic Sections

29. Begin by completing the square twice.

9y2 − 4x2 − 18y + 24x− 63 = 0

9(y2 − 2y) − 4(x2 − 6x) = 63

9(y2 − 2y + 1) − 4(x2 − 6x+ 9) = 63+9·1−4·99(y − 1)2 − 4(x− 3)2 = 36

(y − 1)2

4− (x− 3)2

9= 1

(y − 1)2

22− (x− 3)2

32= 1 Standard

form

The center is (3, 1); a = 2 and b = 3. The transverse axisis vertical, so the vertices are 2 units below and above thecenter:

(3, 1 − 2) and (3, 1 + 2), or (3,−1) and (3, 3).

Since c2 = a2 + b2, we have c2 = 4 + 9 = 13 and c =√

13.Then the foci are

√13 units below and above the center:

(3, 1 −√13) and (3, 1 +

√13).

Find the asymptotes:

y − k =a

b(x− h) and y − k = −a

b(x− h)

y − 1 =23(x− 3) and y − 1 = −2

3(x− 3), or

y =23x− 1 and y = −2

3x+ 3

Sketch the asymptotes, plot the vertices, and draw thegraph.

31. Begin by completing the square twice.

x2 − y2 − 2x− 4y = 4

(x2 − 2x+ 1) − (y2 + 4y + 4) = 4 + 1 − 4

(x− 1)2 − (y + 2)2 = 1

(x− 1)2

12− [y − (−2)]2

12= 1 Standard

form

The center is (1,−2); a = 1 and b = 1. The transverseaxis is horizontal, so the vertices are 1 unit left and rightof the center:

(1 − 1,−2) and (1 + 1,−2), or (0,−2) and (2,−2).

Since c2 = a2 + b2, we have c2 = 1 + 1 = 2 and c =√

2.Then the foci are

√2 units left and right of the center:

(1 −√2,−2) and (1 +

√2,−2).

Find the asymptotes:

y − k =b

a(x− h) and y − k = − b

a(x− h)

y − (−2) =11(x− 1) and y − (−2) = −1

1(x− 1)

y + 2 = x− 1 and y + 2 = −(x− 1), or

y = x− 3 and y = −x− 1

Sketch the asymptotes, plot the vertices, and draw thegraph.

33. Begin by completing the square twice.

y2 − x2 − 6x− 8y − 29 = 0

(y2 − 8y + 16) − (x2 + 6x+ 9) = 29 + 16 − 9

(y − 4)2 − (x+ 3)2 = 36

(y − 4)2

36− (x+ 3)2

36= 1

(y − 4)2

62− [x− (−3)]2

62= 1 Standard

form

The center is (−3, 4); a = 6 and b = 6. The transverseaxis is vertical, so the vertices are 6 units below and abovethe center:

(−3, 4 − 6) and (−3, 4 + 6), or (−3,−2) and (−3, 10).

Since c2 = a2+b2, we have c2 = 36+36 = 72 and c =√

72,or 6

√2. Then the foci are 6

√2 units below and above the

center:

(−3, 4 − 6√

2) and (−3, 4 + 6√

2).

Find the asymptotes:

y − k =a

b(x− h) and y − k = −a

b(x− h)

y − 4 =66(x− (−3)) and y − 4 = −6

6(x− (−3))

y − 4 = x+ 3 and y − 4 = −(x+ 3), or

y = x+ 7 and y = −x+ 1

Sketch the asymptotes, plot the vertices, and draw thegraph.

Copyright © 2013 Pearson Education, Inc.

Page 292: JUDITH A. PENNA -  · PDF fileSTUDENT’S SOLUTIONS MANUAL JUDITH A. PENNA Indiana University Purdue University Indianapolis COLLEGE ALGEBRA: GRAPHS AND MODELS FIFTH EDITION

8

12

4

�4

�8

84�4�12 x

y

y2 � x2 � 6x � 8y � 29 � 0

y

x

(0, 6)

(0, –6)

(0, –8)

(0, 5)Hyperbola

Parabola

F1

F2

x

y

1–1–3 32 54–2–4–5–1

–2

–3

–4

1

2

3

4

5

–5

f (x ) = 2x — 3

x

y

1–1–3 32 54–2–4–5–1

–2

–3

–4

1

2

3

4

5

–5

f (x ) = 5

x 1

Exercise Set 7.3 289

35. The hyperbola in Example 3 is wider than the one in Ex-ample 2, so the hyperbola in Example 3 has the largereccentricity.

Compute the eccentricities: In Example 2, c = 5 and a = 4,so e = 5/4, or 1.25. In Example 3, c =

√5 and a = 1, so

e =√

5/1 ≈ 2.24. These computations confirm that thehyperbola in Example 3 has the larger eccentricity.

37. The center is the midpoint of the segment connecting thevertices:(

3 − 32

,7 + 7

2

), or (0, 7).

The vertices are on the horizontal line y = 7, so the trans-verse axis is horizontal. Since the vertices are 3 units leftand right of the center, a = 3.

Find c:

e =c

a=

53

c

3=

53

Substituting 3 for a

c = 5

Now find b2:c2 = a2 + b2

52 = 32 + b2

16 = b2

Write the equation:(x− h)2

a2− (y − k)2

b2= 1

x2

9− (y − 7)2

16= 1

39.

One focus is 6 units above the center of the hyperbola, soc = 6. One vertex is 5 units above the center, so a = 5.Find b2:

c2 = a2 + b2

62 = 52 + b2

11 = b2

Write the equation:y2

a2− x2

b2= 1

y2

25− x2

11= 1

41. a) The graph of f(x) = 2x− 3 is shown below.

Since there is no horizontal line that crosses thegraph more than once, the function is one-to-one.

b) Replace f(x) with y: y = 2x− 3

Interchange x and y: x = 2y − 3

Solve for y: x+ 3 = 2yx+ 3

2= y

Replace y with f−1(x): f−1(x) =x+ 3

2

43. a) The graph of f(x) =5

x− 1is shown below.

Since there is no horizontal line that crosses thegraph more than once, the function is one-to-one.

b) Replace f(x) with y: y =5

x− 1

Interchange x and y: x =5

y − 1

Copyright © 2013 Pearson Education, Inc.

Page 293: JUDITH A. PENNA -  · PDF fileSTUDENT’S SOLUTIONS MANUAL JUDITH A. PENNA Indiana University Purdue University Indianapolis COLLEGE ALGEBRA: GRAPHS AND MODELS FIFTH EDITION

290 Chapter 7: Conic Sections

Solve for y: x(y − 1) = 5

y − 1 =5x

y =5x

+ 1

Replace y with f−1(x): f−1 =5x

+ 1, or5 + x

x

45. x + y = 5, (1)x − y = 7 (2)

2x = 12 Addingx = 6

Back-substitute in either equation (1) or (2) and solve fory. We use equation (1).

6 + y = 5

y = −1

The solution is (6,−1).

47. 2x − 3y = 7, (1)

3x + 5y = 1 (2)

Multiply equation (1) by 5 and equation (2) by 3 and addto eliminate y.10x − 15y = 359x + 15y = 3

19x = 38x = 2

Back-substitute and solve for y.3 · 2 + 5y = 1 Using equation (2)

5y = −5

y = −1

The solution is (2,−1).

49. The center is the midpoint of the segment connecting(3,−8) and (3,−2):(

3 + 32

,−8 − 2

2

), or (3,−5).

The vertices are on the vertical line x = 3 and are 3 unitsabove and below the center so the transverse axis is verticaland a = 3. Use the equation of an asymptote to find b:

y − k =a

b(x− h)

y + 5 =3b(x− 3)

y =3bx− 9

b− 5

This equation corresponds to the asymptote y = 3x− 14,

so3b

= 3 and b = 1.

Write the equation of the hyperbola:(y − k)2

a2− (x− h)2

b2= 1

(y + 5)2

9− (x− 3)2

1= 1

51. Center: (−1.460,−0.957)

Vertices: (−2.360,−0.957), (−0.560,−0.957)

Asymptotes: y = −1.20x− 2.70, y = 1.20x+ 0.79

53. S and T are the foci of the hyperbola, so c = 300/2 = 150.

200 microseconds · 0.186 mi1 microsecond

= 37.2 mi, thedifference of the ships’ distances from the foci. That is,2a = 37.2, so a = 18.6.

Find b2:c2 = a2 + b2

1502 = 18.62 + b2

22, 154.04 = b2

Then the equation of the hyperbola isx2

18.62− y2

22, 154.04= 1, or

x2

345.96− y2

22, 154.04= 1.

Exercise Set 7.4

1. The correct graph is (e).

3. The correct graph is (c).

5. The correct graph is (b).

7. x2 + y2 = 25, (1)

y − x = 1 (2)

First solve equation (2) for y.

y = x+ 1 (3)

Then substitute x + 1 for y in equation (1) and solve forx.

x2 + y2 = 25

x2 + (x+ 1)2 = 25

x2 + x2 + 2x+ 1 = 25

2x2 + 2x− 24 = 0

x2 + x− 12 = 0 Multiplying by12

(x+ 4)(x− 3) = 0 Factoring

x+ 4 = 0 or x− 3 = 0 Principle of zeroproducts

x = −4 or x = 3

Now substitute these numbers into equation (3) and solvefor y.

y = −4 + 1 = −3

y = 3 + 1 = 4

The pairs (−4,−3) and (3, 4) check, so they are the solu-tions.

9. 4x2 + 9y2 = 36, (1)

3y + 2x = 6 (2)

First solve equation (2) for y.

3y = −2x+ 6

y = −23x+ 2 (3)

Then substitute −23x+ 2 for y in equation (1) and solve

for x.

Copyright © 2013 Pearson Education, Inc.

Page 294: JUDITH A. PENNA -  · PDF fileSTUDENT’S SOLUTIONS MANUAL JUDITH A. PENNA Indiana University Purdue University Indianapolis COLLEGE ALGEBRA: GRAPHS AND MODELS FIFTH EDITION

Exercise Set 7.4 291

4x2 + 9y2 = 36

4x2 + 9(− 2

3x+ 2

)2

= 36

4x2 + 9(4

9x2 − 8

3x+ 4

)= 36

4x2 + 4x2 − 24x+ 36 = 36

8x2 − 24x = 0

x2 − 3x = 0

x(x− 3) = 0x = 0 or x = 3

Now substitute these numbers in equation (3) and solvefor y.

y = −23· 0 + 2 = 2

y = −23· 3 + 2 = 0

The pairs (0, 2) and (3, 0) check, so they are the solutions.

11. x2 + y2 = 25, (1)

y2 = x+ 5 (2)

We substitute x+ 5 for y2 in equation (1) and solve for x.

x2 + y2 = 25

x2 + (x+ 5) = 25

x2 + x− 20 = 0

(x+ 5)(x− 4) = 0

x+ 5 = 0 or x− 4 = 0

x = −5 or x = 4

We substitute these numbers for x in either equation (1)or equation (2) and solve for y. Here we use equation (2).

y2 = −5 + 5 = 0 and y = 0.

y2 = 4 + 5 = 9 and y = ±3.

The pairs (−5, 0), (4, 3) and (4,−3) check. They are thesolutions.

13. x2 + y2 = 9, (1)

x2 − y2 = 9 (2)

Here we use the elimination method.x2 + y2 = 9 (1)

x2 − y2 = 9 (2)

2x2 = 18 Adding

x2 = 9

x = ±3

If x = 3, x2 = 9, and if x = −3, x2 = 9, so substituting 3or −3 in equation (1) gives us

x2 + y2 = 9

9 + y2 = 9

y2 = 0

y = 0.

The pairs (3, 0) and (−3, 0) check. They are the solutions.

15. y2 − x2 = 9 (1)

2x− 3 = y (2)

Substitute 2x− 3 for y in equation (1) and solve for x.

y2 − x2 = 9

(2x− 3)2 − x2 = 9

4x2 − 12x+ 9 − x2 = 9

3x2 − 12x = 0

x2 − 4x = 0

x(x− 4) = 0x = 0 or x = 4

Now substitute these numbers into equation (2) and solvefor y.

If x = 0, y = 2 · 0 − 3 = −3.

If x = 4, y = 2 · 4 − 3 = 5.

The pairs (0,−3) and (4, 5) check. They are the solutions.

17. y2 = x+ 3, (1)

2y = x+ 4 (2)

First solve equation (2) for x.

2y − 4 = x (3)

Then substitute 2y − 4 for x in equation (1) and solve fory.

y2 = x+ 3

y2 = (2y − 4) + 3

y2 = 2y − 1

y2 − 2y + 1 = 0

(y − 1)(y − 1) = 0

y − 1 = 0 or y − 1 = 0y = 1 or y = 1

Now substitute 1 for y in equation (3) and solve for x.

2 · 1 − 4 = x

−2 = x

The pair (−2, 1) checks. It is the solution.

19. x2 + y2 = 25, (1)

xy = 12 (2)

First we solve equation (2) for y.

xy = 12

y =12x

Then we substitute12x

for y in equation (1) and solve forx.

Copyright © 2013 Pearson Education, Inc.

Page 295: JUDITH A. PENNA -  · PDF fileSTUDENT’S SOLUTIONS MANUAL JUDITH A. PENNA Indiana University Purdue University Indianapolis COLLEGE ALGEBRA: GRAPHS AND MODELS FIFTH EDITION

292 Chapter 7: Conic Sections

x2 + y2 = 25

x2 +(12x

)2

= 25

x2 +144x2

= 25

x4 + 144 = 25x2 Multiplying by x2

x4 − 25x2 + 144 = 0

u2 − 25u+ 144 = 0 Letting u = x2

(u− 9)(u− 16) = 0u = 9 or u = 16

We now substitute x2 for u and solve for x.x2 = 9 or x2 = 16

x = ±3 or x = ±4

Since y = 12/x, if x = 3, y = 4; if x = −3, y = −4;if x = 4, y = 3; and if x = −4, y = −3. The pairs(3, 4), (−3,−4), (4, 3), and (−4,−3) check. They are thesolutions.

21. x2 + y2 = 4, (1)

16x2 + 9y2 = 144 (2)

−9x2 − 9y2 = −36 Multiplying (1) by −916x2 + 9y2 = 1447x2 = 108 Adding

x2 =1087

x = ±√

1087

= ±6

√37

x = ±6√

217

Rationalizing the de-nominator

Substituting6√

217

or −6√

217

for x in equation (1) givesus36 · 21

49+ y2 = 4

y2 = 4 − 1087

y2 = −807

y = ±√

−807

= ±4i

√57

y = ±4i√

357

. Rationalizing thedenominator

The pairs(6

√21

7,4i√

357

),

(6√

217

,−4i√

357

),(− 6

√21

7,4i√

357

), and

(− 6

√21

7,−4i

√35

7

)check. They are the solutions.

23. x2 + 4y2 = 25, (1)

x+ 2y = 7 (2)

First solve equation (2) for x.

x = −2y + 7 (3)

Then substitute −2y + 7 for x in equation (1) and solvefor y.

x2 + 4y2 = 25

(−2y + 7)2 + 4y2 = 25

4y2 − 28y + 49 + 4y2 = 25

8y2 − 28y + 24 = 0

2y2 − 7y + 6 = 0

(2y − 3)(y − 2) = 0

y =32

or y = 2

Now substitute these numbers in equation (3) and solvefor x.

x = −2 · 32

+ 7 = 4

x = −2 · 2 + 7 = 3

The pairs(4,

32

)and (3, 2) check, so they are the solutions.

25. x2 − xy + 3y2 = 27, (1)

x− y = 2 (2)

First solve equation (2) for y.

x− 2 = y (3)

Then substitute x − 2 for y in equation (1) and solve forx.

x2 − xy + 3y2 = 27

x2 − x(x− 2) + 3(x− 2)2 = 27

x2 − x2 + 2x+ 3x2 − 12x+ 12 = 27

3x2 − 10x− 15 = 0

x =−(−10) ±√(−10)2 − 4(3)(−15)

2 · 3x =

10 ±√100 + 1806

=10 ±√

2806

x =10 ± 2

√70

6=

5 ±√70

3Now substitute these numbers in equation (3) and solvefor y.

y =5 +

√70

3− 2 =

−1 +√

703

y =5 −√

703

− 2 =−1 −√

703

The pairs(5 +

√70

3,−1 +

√70

3

)and

(5 −√70

3,−1 −√

703

)check, so they are the solutions.

27. x2 + y2 = 16, x2 + y2 = 16, (1)or

y2 − 2x2 = 10 −2x2 + y2 = 10 (2)

Here we use the elimination method.

2x2 + 2y2 = 32 Multiplying (1) by 2−2x2 + y2 = 10

3y2 = 42 Addingy2 = 14y = ±√

14

Copyright © 2013 Pearson Education, Inc.

Page 296: JUDITH A. PENNA -  · PDF fileSTUDENT’S SOLUTIONS MANUAL JUDITH A. PENNA Indiana University Purdue University Indianapolis COLLEGE ALGEBRA: GRAPHS AND MODELS FIFTH EDITION

Exercise Set 7.4 293

Substituting√

14 or −√14 for y in equation (1) gives us

x2 + 14 = 16

x2 = 2

x = ±√2

The pairs (−√2,−√

14), (−√2,√

14), (√

2,−√14), and

(√

2,√

14) check. They are the solutions.

29. x2 + y2 = 5, (1)xy = 2 (2)

First we solve equation (2) for y.

xy = 2

y =2x

Then we substitute2x

for y in equation (1) and solve forx.

x2 + y2 = 5

x2 +( 2x

)2

= 5

x2 +4x2

= 5

x4 + 4 = 5x2 Multiplying by x2

x4 − 5x2 + 4 = 0

u2 − 5u+ 4 = 0 Letting u = x2

(u− 4)(u− 1) = 0u = 4 or u = 1

We now substitute x2 for u and solve for x.x2 = 4 or x2 = 1

x = ±2 x = ±1

Since y = 2/x, if x = 2, y = 1; if x = −2, y = −1; if x = 1,y = 2; and if x = −1, y = −2. The pairs (2, 1), (−2,−1),(1, 2), and (−1,−2) check. They are the solutions.

31. 3x+ y = 7 (1)4x2 + 5y = 56 (2)

First solve equation (1) for y.

3x+ y = 7

y = 7 − 3x (3)

Next substitute 7 − 3x for y in equation (2) and solve forx.

4x2 + 5y = 56

4x2 + 5(7 − 3x) = 56

4x2 + 35 − 15x = 56

4x2 − 15x− 21 = 0

Using the quadratic formula, we find that

x =15 −√

5618

or x =15 +

√561

8.

Now substitute these numbers into equation (3) and solvefor y.

If x =15 −√

5618

, y = 7 − 3(

15 −√561

8

), or

11 + 3√

5618

.

If x =15 +

√561

8, y = 7 − 3

(15 +

√561

8

), or

11 − 3√

5618

.

The pairs(

15 −√561

8,11 + 3

√561

8

)and

(15 +

√561

8,11 − 3

√561

8

)check and are the solutions.

33. a+ b = 7, (1)

ab = 4 (2)

First solve equation (1) for a.

a = −b+ 7 (3)

Then substitute −b+ 7 for a in equation (2) and solve forb.(−b+ 7)b = 4

−b2 + 7b = 4

0 = b2 − 7b+ 4

b =−(−7) ±√(−7)2 − 4 · 1 · 4

2 · 1b =

7 ±√33

2Now substitute these numbers in equation (3) and solvefor a.

a = −(7 +

√33

2

)+ 7 =

7 −√33

2

a = −(7 −√

332

)+ 7 =

7 +√

332

The pairs(7 −√

332

,7 +

√33

2

)and

(7 +√

332

,7 −√

332

)check, so they are the

solutions.

35. x2 + y2 = 13, (1)

xy = 6 (2)

First we solve equation (2) for y.

xy = 6

y =6x

Then we substitute6x

for y in equation (1) and solve forx.

x2 + y2 = 13

x2 +( 6x

)2

= 13

x2 +36x2

= 13

x4 + 36 = 13x2 Multiplying by x2

x4 − 13x2 + 36 = 0

u2 − 13u+ 36 = 0 Letting u = x2

(u− 9)(u− 4) = 0

u = 9 or u = 4

Copyright © 2013 Pearson Education, Inc.

Page 297: JUDITH A. PENNA -  · PDF fileSTUDENT’S SOLUTIONS MANUAL JUDITH A. PENNA Indiana University Purdue University Indianapolis COLLEGE ALGEBRA: GRAPHS AND MODELS FIFTH EDITION

294 Chapter 7: Conic Sections

We now substitute x2 for u and solve for x.x2 = 9 or x2 = 4

x = ±3 or x = ±2

Since y = 6/x, if x = 3, y = 2; if x = −3, y = −2;if x = 2, y = 3; and if x = −2, y = −3. The pairs(3, 2), (−3,−2), (2, 3), and (−2,−3) check. They are thesolutions.

37. x2 + y2 + 6y + 5 = 0 (1)

x2 + y2 − 2x− 8 = 0 (2)

Using the elimination method, multiply equation (2) by−1 and add the result to equation (1).

x2 + y2 + 6y + 5 = 0 (1)−x2 − y2 + 2x + 8 = 0 (2)

2x + 6y + 13 = 0 (3)

Solve equation (3) for x.

2x+ 6y + 13 = 0

2x = −6y − 13

x =−6y − 13

2

Substitute−6y − 13

2for x in equation (1) and solve for y.

x2 + y2 + 6y + 5 = 0(−6y − 132

)2

+ y2 + 6y + 5 = 0

36y2 + 156y + 1694

+ y2 + 6y + 5 = 0

36y2 + 156y + 169 + 4y2 + 24y + 20 = 0

40y2 + 180y + 189 = 0

Using the quadratic formula, we find that

y =−45 ± 3

√15

20. Substitute

−45 ± 3√

1520

for y in

x =−6y − 13

2and solve for x.

If y =−45 + 3

√15

20, then

x =−6(−45 + 3

√15

20

)− 13

2=

5 − 9√

1520

.

If y =−45 − 3

√15

20, then

x =−6(−45 − 3

√15

20

)− 13

2=

5 + 9√

1520

.

The pairs(

5 + 9√

1520

,−45 − 3

√15

20

)and

(5 − 9

√15

20,−45 + 3

√15

20

)check and are the solutions.

39. 2a+ b = 1, (1)

b = 4 − a2 (2)

Equation (2) is already solved for b. Substitute 4 − a2 forb in equation (1) and solve for a.

2a+ 4 − a2 = 1

0 = a2 − 2a− 3

0 = (a− 3)(a+ 1)

a = 3 or a = −1

Substitute these numbers in equation (2) and solve for b.

b = 4 − 32 = −5

b = 4 − (−1)2 = 3

The pairs (3,−5) and (−1, 3) check. They are the solu-tions.

41. a2 + b2 = 89, (1)

a− b = 3 (2)

First solve equation (2) for a.

a = b+ 3 (3)

Then substitute b+ 3 for a in equation (1) and solve for b.

(b+ 3)2 + b2 = 89

b2 + 6b+ 9 + b2 = 89

2b2 + 6b− 80 = 0

b2 + 3b− 40 = 0

(b+ 8)(b− 5) = 0

b = −8 or b = 5

Substitute these numbers in equation (3) and solve for a.

a = −8 + 3 = −5

a = 5 + 3 = 8

The pairs (−5,−8) and (8, 5) check. They are the solu-tions.

43. xy − y2 = 2, (1)2xy − 3y2 = 0 (2)

−2xy + 2y2 = −4 Multiplying (1) by −22xy − 3y2 = 0

−y2 = −4 Addingy2 = 4y = ±2

We substitute for y in equation (1) and solve for x.

When y = 2 : x · 2 − 22 = 2

2x− 4 = 2

2x = 6

x = 3

When y = −2 : x(−2) − (−2)2 = 2

−2x− 4 = 2

−2x = 6

x = −3

The pairs (3, 2) and (−3,−2) check. They are the solu-tions.

45. m2 − 3mn + n2 + 1 = 0, (1)

3m2 − mn + 3n2 = 13 (2)

m2 − 3mn + n2 = −1 (3) Rewriting (1)

3m2 − mn + 3n2 = 13 (2)

Copyright © 2013 Pearson Education, Inc.

Page 298: JUDITH A. PENNA -  · PDF fileSTUDENT’S SOLUTIONS MANUAL JUDITH A. PENNA Indiana University Purdue University Indianapolis COLLEGE ALGEBRA: GRAPHS AND MODELS FIFTH EDITION

Exercise Set 7.4 295

−3m2+9mn− 3n2 = 3 Multiplying (3) by −3

3m2− mn+ 3n2 = 138mn = 16mn = 2

n =2m

(4)

Substitute2m

for n in equation (1) and solve for m.

m2 − 3m(

2m

)+(

2m

)2

+ 1 = 0

m2 − 6 +4m2

+ 1 = 0

m2 − 5 +4m2

= 0

m4 − 5m2 + 4 = 0 Multiplyingby m2

Substitute u for m2.u2 − 5u+ 4 = 0

(u− 4)(u− 1) = 0

u = 4 or u = 1

m2 = 4 or m2 = 1

m = ±2 or m = ±1

Substitute for m in equation (4) and solve for n.

When m = 2, n =22

= 1.

When m = −2, n =2−2

= −1.

When m = 1, n =21

= 2.

When m = −1, n =2−1

= −2.

The pairs (2, 1), (−2,−1), (1, 2), and (−1,−2) check. Theyare the solutions.

47. x2 + y2 = 5, (1)

x− y = 8 (2)

First solve equation (2) for x.

x = y + 8 (3)

Then substitute y + 8 for x in equation (1) and solve fory.

(y + 8)2 + y2 = 5

y2 + 16y + 64 + y2 = 5

2y2 + 16y + 59 = 0

y =−16 ±√(16)2 − 4(2)(59)

2 · 2

y =−16 ±√−216

4

y =−16 ± 6i

√6

4

y = −4 ± 32i√

6

Now substitute these numbers in equation (3) and solvefor x.

x = −4 +32i√

6 + 8 = 4 +32i√

6

x = −4 − 32i√

6 + 8 = 4 − 32i√

6

The pairs(

4 +32i√

6,−4 +32i√

6)

and(4 − 3

2i√

6,−4 − 32i√

6)

check. They are the solutions.

49. a2 + b2 = 14, (1)

ab = 3√

5 (2)

Solve equation (2) for b.

b =3√

5a

Substitute3√

5a

for b in equation (1) and solve for a.

a2 +(

3√

5a

)2

= 14

a2 +45a2

= 14

a4 + 45 = 14a2

a4 − 14a2 + 45 = 0

u2 − 14u+ 45 = 0 Letting u = a2

(u− 9)(u− 5) = 0

u = 9 or u = 5

a2 = 9 or a2 = 5

a = ±3 or a = ±√5

Since b = 3√

5/a, if a = 3, b =√

5; if a = −3, b = −√5;

if a =√

5, b = 3; and if a = −√5, b = −3. The pairs

(3,√

5), (−3,−√5), (

√5, 3), (−√

5,−3) check. They arethe solutions.

51. x2 + y2 = 25, (1)

9x2 + 4y2 = 36 (2)

−4x2 − 4y2 = −100 Multiplying (1) by −4

9x2 + 4y2 = 365x2 = −64

x2 = −645

x = ±√

−645

= ± 8i√5

x = ±8i√

55

Rationalizing thedenominator

Substituting8i√

55

or −8i√

55

for x in equation (1) andsolving for y gives us

Copyright © 2013 Pearson Education, Inc.

Page 299: JUDITH A. PENNA -  · PDF fileSTUDENT’S SOLUTIONS MANUAL JUDITH A. PENNA Indiana University Purdue University Indianapolis COLLEGE ALGEBRA: GRAPHS AND MODELS FIFTH EDITION

296 Chapter 7: Conic Sections

−645

+ y2 = 25

y2 =1895

y = ±√

1895

= ±3

√215

y = ±3√

1055

. Rationalizing thedenominator

The pairs(

8i√

55

,3√

1055

),(− 8i

√5

5,3√

1055

),

(8i√

55

,−3√

1055

), and

(− 8i

√5

5,−3

√1055

)check.

They are the solutions.

53. 5y2 − x2 = 1, (1)

xy = 2 (2)

Solve equation (2) for x.

x =2y

Substitute2y

for x in equation (1) and solve for y.

5y2 −(

2y

)2

= 1

5y2 − 4y2

= 1

5y4 − 4 = y2

5y4 − y2 − 4 = 0

5u2 − u− 4 = 0 Letting u = y2

(5u+ 4)(u− 1) = 0

5u+ 4 = 0 or u− 1 = 0

u = −45

or u = 1

y2 = −45

or y2 = 1

y = ± 2i√5

or y = ±1

y = ±2i√

55

or y = ±1

Since x = 2/y, if y =2i√

55

, x =2

2i√

55

=5

i√

5=

5i√

5· −i

√5

−i√5= −i√5; if y = −2i

√5

5,

x =2

−2i√

55

= i√

5;

if y = 1, x = 2/1 = 2; if y = −1, x = 2/− 1 = −2.

The pairs(− i

√5,

2i√

55

),(i√

5,−2i√

55

), (2, 1) and

(−2,−1) check. They are the solutions.

55. The statement is true. See Example 4, for instance.

57. The statement is true because a line and a circle can in-tersect in at most two points.

59. Familiarize. We first make a drawing. We let l and wrepresent the length and width, respectively.

✚✚

✚✚

✚✚

✚✚

10

l

w

Translate. The perimeter is 28 cm.

2l + 2w = 28, or l + w = 14

Using the Pythagorean theorem we have another equation.

l2 + w2 = 102, or l2 + w2 = 100

Carry out. We solve the system:

l + w = 14, (1)

l2 + w2 = 100 (2)

First solve equation (1) for w.

w = 14 − l (3)

Then substitute 14 − l for w in equation (2) and solve forl.

l2 + w2 = 100

l2 + (14 − l)2 = 100

l2 + 196 − 28l + l2 = 100

2l2 − 28l + 96 = 0

l2 − 14l + 48 = 0

(l − 8)(l − 6) = 0

l = 8 or l = 6

If l = 8, then w = 14 − 8, or 6. If l = 6, then w = 14 − 6,or 8. Since the length is usually considered to be longerthan the width, we have the solution l = 8 and w = 6, or(8, 6).

Check. If l = 8 and w = 6, then the perimeter is 2·8+2·6,or 28. The length of a diagonal is

√82 + 62, or

√100, or

10. The numbers check.

State. The length is 8 cm, and the width is 6 cm.

61. Familiarize. We first make a drawing. Let l = the lengthand w = the width of the brochure.

l

w

Translate.Area: lw = 20

Perimeter: 2l + 2w = 18, or l + w = 9

Carry out. We solve the system:

Copyright © 2013 Pearson Education, Inc.

Page 300: JUDITH A. PENNA -  · PDF fileSTUDENT’S SOLUTIONS MANUAL JUDITH A. PENNA Indiana University Purdue University Indianapolis COLLEGE ALGEBRA: GRAPHS AND MODELS FIFTH EDITION

Exercise Set 7.4 297

Solve the second equation for l: l = 9 − w

Substitute 9−w for l in the first equation and solve for w.(9 − w)w = 20

9w − w2 = 20

0 = w2 − 9w + 20

0 = (w − 5)(w − 4)w = 5 or w = 4

If w = 5, then l = 9 − w, or 4. If w = 4, then l = 9 − 4,or 5. Since length is usually considered to be longer thanwidth, we have the solution l = 5 and w = 4, or (5, 4).

Check. If l = 5 and w = 4, the area is 5 · 4, or 20. Theperimeter is 2 · 5 + 2 · 4, or 18. The numbers check.

State. The length of the brochure is 5 in. and the widthis 4 in.

63. Familiarize. We make a drawing of the dog run. Let l =the length and w = the width.

l

w

Since it takes 210 yd of fencing to enclose the run, we knowthat the perimeter is 210 yd.

Translate.Perimeter: 2l + 2w = 210, or l + w = 105

Area: lw = 2250Carry out. We solve the system:

Solve the first equation for l: l = 105 − w

Substitute 105 − w for l in the second equation and solvefor w.(105 − w)w = 2250

105w − w2 = 2250

0 = w2 − 105w + 2250

0 = (w − 30)(w − 75)w = 30 or w = 75

If w = 30, then l = 105 − 30, or 75. If w = 75, thenl = 105 − 75, or 30. Since length is usually consideredto be longer than width, we have the solution l = 75 andw = 30, or (75, 30).

Check. If l = 75 and w = 30, the perimeter is 2·75+2·30,or 210. The area is 75(30), or 2250. The numbers check.

State. The length is 75 yd and the width is 30 yd.

65. Familiarize. We first make a drawing. Let l = the lengthand w = the width.

✚✚

✚✚

✚✚

✚✚

2

l

w

Translate.

Area: lw =√

3 (1)

From the Pythagorean theorem: l2 + w2 = 22 (2)

Carry out. We solve the system of equations.

We first solve equation (1) for w.

lw =√

3

w =√

3l

Then we substitute√

3l

for w in equation 2 and solve forl.

l2 +(√3

l

)2

= 4

l2 +3l2

= 4

l4 + 3 = 4l2

l4 − 4l2 + 3 = 0

u2 − 4u+ 3 = 0 Letting u = l2

(u− 3)(u− 1) = 0u = 3 or u = 1

We now substitute l2 for u and solve for l.l2 = 3 or l2 = 1

l = ±√3 or l = ±1

Measurements cannot be negative, so we only need to con-sider l =

√3 and l = 1. Since w =

√3/l, if l =

√3, w = 1

and if l = 1, w =√

3. Length is usually considered to belonger than width, so we have the solution l =

√3 and

w = 1, or (√

3, 1).

Check. If l =√

3 and w = 1, the area is√

3 · 1 =√

3.Also (

√3)2 + 12 = 3 + 1 = 4 = 22. The numbers check.

State. The length is√

3 m, and the width is 1 m.

67. Familiarize. We let x = the length of a side of one testplot and y = the length of a side of the other plot. Makea drawing.

x

x

Area: x2

y

y

Area: y2

Translate.

The sum of the areas is 832 ft2.︸ ︷︷ ︸ ︸︷︷︸ ︸ ︷︷ ︸↓ ↓ ↓

x2 + y2 = 832

The difference of the areas is 320 ft2.︸ ︷︷ ︸ ︸︷︷︸ ︸ ︷︷ ︸↓ ↓ ↓

x2 − y2 = 320

Carry out. We solve the system of equations.

Copyright © 2013 Pearson Education, Inc.

Page 301: JUDITH A. PENNA -  · PDF fileSTUDENT’S SOLUTIONS MANUAL JUDITH A. PENNA Indiana University Purdue University Indianapolis COLLEGE ALGEBRA: GRAPHS AND MODELS FIFTH EDITION

�2 21

2

�2

�5 �3 �1�1

�3

�5

1

3

5

3 5

y

x

(�2�2, �2�2)

(2�2, 2�2)

�2 21 4

2

�2

�4

�4�5 �3 �1�1

�3

�5

1

345

3 5

y

x

(1, 1)

(�2, 4)

�2 21 4

2

�2

�4

�4�3 �1�1

�3

1

34

3

y

x

(5, 0)

(0, �5)

�2 21 4

2

�2

�4�5 �3 �1

1

34567

3 5

y

x

(3, 6)

(�1, �2)

298 Chapter 7: Conic Sections

x2 + y2 = 832

x2 − y2 = 320

2x2 = 1152 Adding

x2 = 576

x = ±24

Since measurements cannot be negative, we consider onlyx = 24. Substitute 24 for x in the first equation and solvefor y.

242 + y2 = 832

576 + y2 = 832

y2 = 256

y = ±16

Again, we consider only the positive value, 16. The possi-ble solution is (24, 16).

Check. The areas of the test plots are 242, or 576, and162, or 256. The sum of the areas is 576 + 256, or 832.The difference of the areas is 576−256, or 320. The valuescheck.

State. The lengths of the test plots are 24 ft and 16 ft.

69. The correct graph is (b).

71. The correct graph is (d).

73. The correct graph is (a).

75. Graph: x2 + y2 ≤ 16,

y < x

The solution set of x2 + y2 ≤ 16 is the circle x2 + y2 = 16and the region inside it. The solution set of y < x isthe half-plane below the line y = x. We shade the regioncommon to the two solution sets.

To find the points of intersection of the graphs we solvethe system of equations

x2 + y2 = 16,

y = x.

The points of intersection are (−2√

2,−2√

2) and(2√

2, 2√

2).

77. Graph: x2 ≤ y,

x+ y ≥ 2

The solution set of x2 ≤ y is the parabola x2 = y and theregion inside it. The solution set of x + y ≥ 2 is the linex+ y = 2 and the half-plane above the line. We shade theregion common to the two solution sets.

To find the points of intersection of the graphs we solvethe system of equations

x2 = y,

x+ y = 2.

The points of intersection are (−2, 4) and (1, 1).

79. Graph: x2 + y2 ≤ 25,

x− y > 5

The solution set of x2 + y2 ≤ 25 is the circle x2 + y2 = 25and the region inside it. The solution set of x − y > 5is the half-plane below the line x − y = 5. We shade theregion common to the two solution sets.

To find the points of intersection of the graphs we solvethe system of equations

x2 + y2 = 25,

x− y = 5.

The points of intersection are (0,−5) and (5, 0).

81. Graph: y ≥ x2 − 3,

y ≤ 2x

The solution set of y ≥ x2 − 3 is the parabola y = x2 − 3and the region inside it. The solution set of y ≤ 2x is theline y = 2x and the half-plane below it. We shade theregion common to the two solution sets.

To find the points of intersection of the graphs we solvethe system of equations

y = x2 − 3,

y = 2x.

The points of intersection are (−1,−2) and (3, 6).

Copyright © 2013 Pearson Education, Inc.

Page 302: JUDITH A. PENNA -  · PDF fileSTUDENT’S SOLUTIONS MANUAL JUDITH A. PENNA Indiana University Purdue University Indianapolis COLLEGE ALGEBRA: GRAPHS AND MODELS FIFTH EDITION

�2 21 4

2

�2

�4

�4�5 �3 �1�1

�3

�5

1

345

3 5

y

x

(2, 4)

(�1, 1)

Exercise Set 7.4 299

83. Graph: y ≥ x2,

y < x+ 2

The solution set of y ≥ x2 is the parabola y = x2 andthe region inside it. The solution set of y < x + 2 is thehalf-plane below the line y = x + 2. We shade the regioncommon to the two solution sets.

To find the points of intersection of the graphs we solvethe system of equations

y = x2,

y = x+ 2.

The points of intersection are (−1, 1) and (2, 4).

85. 23x = 64

23x = 26

3x = 6

x = 2

The solution is 2.

87. log3 x = 4

x = 34

x = 81

The solution is 81.

89. (x− h)2 + (y − k)2 = r2

If (2, 4) is a point on the circle, then

(2 − h)2 + (4 − k)2 = r2.

If (3, 3) is a point on the circle, then

(3 − h)2 + (3 − k)2 = r2.

Thus(2 − h)2 + (4 − k)2 = (3 − h)2 + (3 − k)2

4 − 4h+ h2 + 16 − 8k + k2 =

9 − 6h+ h2 + 9 − 6k + k2

−4h− 8k + 20 = −6h− 6k + 18

2h− 2k = −2

h− k = −1

If the center (h, k) is on the line 3x−y = 3, then 3h−k = 3.

Solving the system

h− k = −1,

3h− k = 3

we find that (h, k) = (2, 3).

Find r2, substituting (2, 3) for (h, k) and (2, 4) for (x, y).We could also use (3, 3) for (x, y).

(x− h)2 + (y − k)2 = r2

(2 − 2)2 + (4 − 3)2 = r2

0 + 1 = r2

1 = r2

The equation of the circle is (x− 2)2 + (y − 3)2 = 1.

91. The equation of the ellipse is of the formx2

a2+y2

b2= 1. Substitute

(1,

√3

2

)and

(√3,

12

)for (x, y)

to get two equations.

12

a2+

(√3

2

)2

b2= 1, or

1a2

+3

4b2= 1

(√

3)2

a2+

(12

)2

b2= 1, or

3a2

+1

4b2= 1

Substitute u for1a2

and v for1b2

.

u+34v = 1, 4u+ 3v = 4,

or

3u+14v = 1 12u+ v = 4

Solving for u and v, we get u =14, v = 1. Then

u =1a2

=14, so a2 = 4; v =

1b2

= 1, so b2 = 1.

Then the equation of the ellipse isx2

4+y2

1= 1, or

x2

4+ y2 = 1.

93. See the answer section in the text.

95. See the answer section in the text.

97. x3 + y3 = 72, (1)

x+ y = 6 (2)

Solve equation (2) for y: y = 6 − x

Substitute for y in equation (1) and solve for x.

x3 + (6 − x)3 = 72

x3 + 216 − 108x+ 18x2 − x3 = 72

18x2 − 108x+ 144 = 0

x2 − 6x+ 8 = 0

Multiplying by118

(x− 4)(x− 2) = 0x = 4 or x = 2

If x = 4, then y = 6 − 4 = 2.

If x = 2, then y = 6 − 2 = 4.

The pairs (4, 2) and (2, 4) check.

99. p2 + q2 = 13, (1)1pq

= −16

(2)

Solve equation (2) for p.

Copyright © 2013 Pearson Education, Inc.

Page 303: JUDITH A. PENNA -  · PDF fileSTUDENT’S SOLUTIONS MANUAL JUDITH A. PENNA Indiana University Purdue University Indianapolis COLLEGE ALGEBRA: GRAPHS AND MODELS FIFTH EDITION

300 Chapter 7: Conic Sections

1q

= −p

6

−6q

= p

Substitute −6/q for p in equation (1) and solve for q.(− 6q

)2

+ q2 = 13

36q2

+ q2 = 13

36 + q4 = 13q2

q4 − 13q2 + 36 = 0

u2 − 13u+ 36 = 0 Letting u = q2

(u− 9)(u− 4) = 0

u = 9 or u = 4

x2 = 9 or x2 = 4

x = ±3 or x = ±2

Since p = −6/q, if q = 3, p = −2; if q = −3, p = 2;if q = 2, p = −3; and if q = −2, p = 3. The pairs(−2, 3), (2,−3), (−3, 2), and (3,−2) check. They are thesolutions.

101. Find the points of intersection of y1 = lnx+2 and y2 = x2.They are (1.564, 2.448) and (0.138, 0.019).

103. Find the points of intersection of y1 = ex and y2 = x + 2.They are (1.146, 3.146) and (−1.841, 0.159).

105. Graph y1 =

√14.5x2 − 64.5

13.5, y2 = −

√14.5x2 − 64.5

13.5, and

y3 = (5.5x− 12.3)/6.3 and find the points of intersection.They are (2.112,−0.109) and (−13.041,−13.337).

107. Graph y1 =

√56, 548 − 0.319x2

2688.7,

y2 = −√

56, 548 − 0.319x2

2688.7, y3 =

√0.306x2 − 43, 452

2688.7,

and y4 = −√

0.306x2 − 43, 4522688.7

and find the points of in-

tersection. They are (400, 1.431), (−400, 1.431),(400,−1.431), and (−400,−1.431).

Chapter 7 Review Exercises

1. x+ y2 = 1

y2 = x− 1

(y − 0)2 = 4 · 14(x− 1)

This parabola has a horizontal axis of symmetry, the focus

is(

54, 0)

, and the directrix is x =34. Thus it opens to the

left and the statement is true.

3. The statement is true. See page 597 in the text.

5. The statement is false. See Example 4 on page 610 in thetext.

7. Graph (a) is the graph of y2 = 9 − x2.

9. Graph (g) is the graph of 9y2 − 4x2 = 36.

11. Graph (f) is the graph of 4x2 + y2 − 16x− 6y = 15.

13. Graph (c) is the graph of(x+ 3)2

16− (y − 1)2

25= 1.

15. y2 = −12x

y2 = 4(−3)x

F : (−3, 0), V : (0, 0), D: x = 3

17. Begin by completing the square twice.16x2 + 25y2 − 64x+ 50y − 311 = 0

16(x2 − 4x) + 25(y2 + 2y) = 311

16(x2 − 4x+ 4) + 25(y2 + 2y + 1) = 311 + 16 · 4 + 25 · 116(x− 2)2 + 25(y + 1)2 = 400

(x− 2)2

25+

[y − (−1)]2

16= 1

The center is (2,−1). Note that a = 5 and b = 4. Themajor axis is horizontal so the vertices are 5 units left andright of the center: (2−5,−1) and (2+5,−1), or (−3,−1)and (7,−1). We know that c2 = a2 − b2 = 25 − 16 = 9and c =

√9 = 3. Then the foci are 3 units left and right

of the center: (2− 3,−1) and (2 + 3,−1), or (−1,−1) and(5,−1).

19. Begin by completing the square twice.

x2 − 2y2 + 4x+ y − 18

= 0

(x2 + 4x) − 2(y2 − 1

2y

)=

18

(x2 + 4x+ 4) − 2(y2 − 1

2y +

116

)=

18

+ 4 − 2 · 116

(x+ 2)2 − 2(y − 1

4

)2

= 4

[x− (−2)]2

4−

(y − 1

4

)2

2= 1

The center is(− 2,

14

). The transverse axis is horizon-

tal, so the vertices are 2 units left and right of the cen-

ter:(− 2 − 2,

14

)and

(− 2 + 2,

14

), or

(− 4,

14

)and(

0,14

). Since c2 = a2 + b2, we have c2 = 4 + 2 = 6 and

c =√

6. Then the foci are√

6 units left and right of the

center:(− 2 −

√6,

14

)and

(− 2 +

√6,

14

).

Copyright © 2013 Pearson Education, Inc.

Page 304: JUDITH A. PENNA -  · PDF fileSTUDENT’S SOLUTIONS MANUAL JUDITH A. PENNA Indiana University Purdue University Indianapolis COLLEGE ALGEBRA: GRAPHS AND MODELS FIFTH EDITION

Chapter 7 Review Exercises 301

Find the asymptotes:

y − k =b

a(x− h) and y − k = − b

a(x− h)

y − 14

=√

22

(x+ 2) and y − 14

= −√

22

(x+ 2)

21. x2 − 16y = 0, (1)

x2 − y2 = 64 (2)

From equation (1) we have x2 = 16y. Substitute in equa-tion (2).

16y − y2 = 64

0 = y2 − 16y + 64

0 = (y − 8)2

0 = y − 8

8 = y

x2 − (8)2 = 64 Substituting in equation (2)

x2 = 128

x = ±√128 = ±8

√2

The pairs (−8√

2, 8) and (8√

2, 8) check.

23. x2 − y2 = 33, (1)

x+ y = 11 (2)

y = −x+ 11

x2 − (−x+ 11)2 = 33 Substituting in (1)

x2 − (x2 − 22x+ 121) = 33

x2 − x2 + 22x− 121 = 33

22x = 154

x = 7y = −7 + 11 = 4

The pair (7, 4) checks.

25. x2 − y = 3, (1)

2x− y = 3 (2)

From equation (1) we have y = x2 − 3. Substitute inequation (2).

2x− (x2 − 3) = 3

2x− x2 + 3 = 3

0 = x2 − 2x

0 = x(x− 2)x = 0 or x = 2

y = 02 − 3 = −3

y = 22 − 3 = 1

The pairs (0,−3) and (2, 1) check.

27. x2 − y2 = 3, (1)

y = x2 − 3 (2)

From equation (2) we have x2 = y + 3. Substitute inequation (1).

y + 3 − y2 = 3

0 = y2 − y

0 = y(y − 1)

y = 0 or y = 1

x2 = 0 + 3

x2 = 3

x = ±√3

x2 = 1 + 3

x2 = 4

x = ±2

The pairs (√

3, 0), (−√3, 0), (2, 1), and (−2, 1) check.

29. x2 + y2 = 100, (1)

2x2 − 3y2 = −120 (2)

3x2 + 3y2 = 300 Multiplying (1) by 3

2x2 − 3y2 = −1205x2 = 180 Adding

x2 = 36x = ±6

(±6)2 + y2 = 100

y2 = 64

y = ±8

The pairs (6, 8), (−6, 8), (6,−8), and (−6,−8) check.

31. Familiarize. Let x and y represent the numbers.

Translate. The sum of the numbers is 11.

x+ y = 11

The sum of the squares of the numbers is 65.

x2 + y2 = 65

Carry out. We solve the system of equations.

x+ y = 11, (1)

x2 + y2 = 65 (2)

First we solve equation (1) for y.

y = 11 − x

Then substitute 11 − x for y in equation (2) and solve forx.

x2 + (11 − x)2 = 65

x2 + 121 − 22x+ x2 = 65

2x2 − 22x+ 121 = 65

2x2 − 22x+ 56 = 0

x2 − 11x+ 28 = 0 Dividing by 2

(x− 4)(x− 7) = 0

x− 4 = 0 or x− 7 = 0

x = 4 or x = 7

If x = 4, then y = 11 − 4 = 7.

If x = 7, then y = 11 − 7 = 4.

In either case, the possible numbers are 4 and 7.

Check. 4 + 7 = 11 and 42 + 72 = 16 + 49 = 65. Theanswer checks.

State. The numbers are 4 and 7.

Copyright © 2013 Pearson Education, Inc.

Page 305: JUDITH A. PENNA -  · PDF fileSTUDENT’S SOLUTIONS MANUAL JUDITH A. PENNA Indiana University Purdue University Indianapolis COLLEGE ALGEBRA: GRAPHS AND MODELS FIFTH EDITION

�2 21

2

�2

�5 �3 �1�1

�3

�5

1

3

5

3 5

y

x(4, 0)

(0, 4)

�2 21 4

2

�2

�4

�4�5 �1�1

�5

1

45

5

y

x

(�1, 2�2)

(�1, �2�2)

302 Chapter 7: Conic Sections

33. Familiarize. Let x and y represent the positive integers.

Translate. The sum of the numbers is 12.

x+ y = 12

The sum of the reciprocals is38.

1x

+1y

=38

Carry out. We solve the system of equations.

x+ y = 12, (1)1x

+1y

=38

(2)

First solve equation (1) for y.

y = 12 − x

Then substitute 12 − x for y in equation (2) and solve forx.

1x

+1

12 − x=

38, LCD is 8x(12− x)

8x(12 − x)(

1x

+1

12 − x

)= 8x(12 − x) · 3

8

8(12 − x) + 8x = x(12 − x) · 396 − 8x+ 8x = 36x− 3x2

96 = 36x− 3x2

3x2 − 36x+ 96 = 0

x2 − 12x+ 32 = 0 Dividing by 3

(x− 4)(x− 8) = 0

x− 4 = 0 or x− 8 = 0

x = 4 or x = 8

If x = 4, y = 12 − 4 = 8.

If x = 8, y = 12 − 8 = 4.

In either case, the possible numbers are 4 and 8.

Check. 4 + 8 = 12;14

+18

=28

+18

=38. The answer

checks.

State. The numbers are 4 and 8.

35. Familiarize. Let x = the radius of the larger circle andlet y = the radius of the smaller circle. We will use theformula for the area of a circle, A = πr2.

Translate. The sum of the areas is 130π ft2.

πx2 + πy2 = 130π

The difference of the areas is 112π ft2.

πx2 − πy2 = 112π

We have a system of equations.

πx2 + πy2 = 130π, (1)

πx2 − πy2 = 112π (2)

Carry out. We add.

πx2 + πy2 = 130π

πx2 − πy2 = 112π2πx2 = 242π

x2 = 121 Dividing by 2πx = ±11

Since the length of a radius cannot be negative, we consideronly x = 11. Substitute 11 for x in equation (1) and solvefor y.

π · 112 + πy2 = 130π

121π + πy2 = 130π

πy2 = 9π

y2 = 9

y = ±3Again, we consider only the positive solution.

Check. If the radii are 11 ft and 3 ft, the sum of the areasis π · 112 + π · 32 = 121π + 9π = 130π ft2. The differenceof the areas is 121π − 9π = 112π ft2. The answer checks.

State. The radius of the larger circle is 11 ft, and theradius of the smaller circle is 3 ft.

37. Graph: x2 + y2 ≤ 16,

x+ y < 4

The solution set of x2 + y2 ≤ 16 is the circle x2 + y2 = 16and the region inside it. The solution set of x + y < 4is the half-plane below the line x + y = 4. We shade theregion common to the two solution sets.

39. Graph: x2 + y2 ≤ 9,

x ≤ −1

The solution set of x2 + y2 ≤ 9 is the circle x2 + y2 = 9and the region inside it. The solution set of x ≤ −1 is theline x = −1 and the half-plane to the left of it. We shadethe region common to the two solution sets.

41. A straight line can intersect an ellipse at 0 points, 1 point,or 2 points but not at 4 points, so answer D is correct.

43. Familiarize. Let x and y represent the numbers.

Translate. The product of the numbers is 4.

xy = 4

The sum of the reciprocals is6556

.

1x

+1y

=6556

Copyright © 2013 Pearson Education, Inc.

Page 306: JUDITH A. PENNA -  · PDF fileSTUDENT’S SOLUTIONS MANUAL JUDITH A. PENNA Indiana University Purdue University Indianapolis COLLEGE ALGEBRA: GRAPHS AND MODELS FIFTH EDITION

x2 � 12y

2 4�4 �2

2

4

�4

�2

x

y

y2 � 2y � 8x � 7 � 0

2 4�4 �2

2

4

�4

�2

x

y

Chapter 7 Test 303

Carry out. We solve the system of equations.xy = 4, (1)1x

+1y

=6556

(2)

First solve equation (1) for y.

y =4x

Then substitute4x

for y in equation (2) and solve for x.

1x

+1

4/x=

6556

1x

+x

4=

6556

56x(

1x

+x

4

)= 56x · 65

56

56 + 14x2 = 65x

14x2 − 65x+ 56 = 0

(2x− 7)(7x− 8) = 0

2x− 7 = 0 or 7x− 8 = 0

2x = 7 or 7x = 8

x =72

or x =87

If x =72, y =

47/2

= 4 · 27

=87.

If x =87, y =

48/7

= 4 · 78

=72.

In either case the possible numbers are72

and87.

Check.72· 87

= 4;1

7/2+

18/7

=27

+78

=1656

+4956

=6556

.

The answer checks.

State. The numbers are72

and87.

45. The vertices are (0,−3) and (0, 3), so the center is (0, 0),and the major axis is vertical.

x2

a2+y2

32= 1

Substitute(− 1

2,3√

32

)and solve for a.

(− 1

2

)2

a2+

(3√

32

)2

32= 1

14a2

+34

= 1

1 + 3a2 = 4a2 Multiplying by 4a2

1 = a2

1 = a

The equation of the ellipse is x2 +y2

9= 1.

47. The equation of a circle can be written as(x− h)2

a2+

(y − k)2

b2= 1

where a = b = r, the radius of the circle. In an ellipse,a > b, so a circle is not a special type of ellipse.

49. No, the asymptotes of a hyperbola are not part of thegraph of the hyperbola. The coordinates of points on theasymptotes do not satisfy the equation of the hyperbola.

Chapter 7 Test

1. Graph (c) is the graph of 4x2 − y2 = 4.

2. Graph (b) is the graph of x2 − 2x− 3y = 5.

3. Graph (a) is the graph of x2 + 4x+ y2 − 2y − 4 = 0.

4. Graph (d) is the graph of 9x2 + 4y2 = 36.

5. x2 = 12y

x2 = 4 · 3yV : (0, 0), F : (0, 3), D : y = −3

6. y2 + 2y − 8x− 7 = 0

y2 + 2y = 8x+ 7

y2 + 2y + 1 = 8x+ 7 + 1

(y + 1)2 = 8x+ 8

[y − (−1)]2 = 4(2)[x− (−1)]

V : (−1,−1)

F : (−1 + 2,−1) or (1,−1)

D : x = −1 − 2 = −3

7. (x− h)2 = 4p(y − k)

(x− 0)2 = 4 · 2(y − 0)

x2 = 8y

8. Begin by completing the square twice.x2 + y2 + 2x− 6y − 15 = 0

x2 + 2x+ y2 − 6y = 15

(x2 + 2x+ 1) + (y2 − 6y + 9) = 15 + 1 + 9

(x+ 1)2 + (y − 3)2 = 25

[x− (−1)]2 + (y − 3)2 = 52

Copyright © 2013 Pearson Education, Inc.

Page 307: JUDITH A. PENNA -  · PDF fileSTUDENT’S SOLUTIONS MANUAL JUDITH A. PENNA Indiana University Purdue University Indianapolis COLLEGE ALGEBRA: GRAPHS AND MODELS FIFTH EDITION

4x2 � y2 � 4

2 4�4 �2

2

4

�4

�2

x

y

304 Chapter 7: Conic Sections

Center: (−1, 3), radius: 5

9. 9x2 + 16y2 = 144

x2

16+y2

9= 1

x2

42+y2

32= 1

a = 4, b = 3

The center is (0, 0). The major axis is horizontal, so thevertices are (−4, 0) and (4, 0). Since c2 = a2 − b2, we havec2 = 16 − 9 = 7, so c =

√7 and the foci are (−√

7, 0) and(√

7, 0).

10. (x+ 1)2

4+

(y − 2)2

9= 1

[x− (−1)]2

22+

(y − 2)2

32= 1

The center is (−1, 2). Note that a = 3 and b = 2. Themajor axis is vertical, so the vertices are 3 units below andabove the center:

(−1, 2 − 3) and (−1, 2 + 3) or (−1,−1) and (−1, 5).

We know that c2 = a2 − b2, so c2 = 9− 4 = 5 and c =√

5.Then the foci are

√5 units below and above the center:

(−1, 2 −√5) and (−1, 2 +

√5).

11. The vertices (0,−5) and (0, 5) are on the y-axis, so themajor axis is vertical and a = 5. Since the vertices areequidistant from the center, the center of the ellipse is at

the origin. The length of the minor axis is 4, so b =42

= 2.

The equation isx2

4+y2

25= 1.

12. 4x2 − y2 = 4

x2

1− y2

4= 1

x2

12− y2

22= 1

The center is (0, 0); a = 1 and b = 2.

The transverse axis is horizontal, so the vertices are (−1, 0)and (1, 0). Since c2 = a2 + b2, we have c2 = 1 + 4 = 5 andc =

√5. Then the foci are (−√

5, 0) and (√

5, 0).

Find the asymptotes:

y =b

ax and y = − b

ax

y =21x and y = −2

1x

y = 2x and y = −2x

13. (y − 2)2

4− (x+ 1)2

9= 1

(y − 2)2

22− [x− (−1)]2

32= 1

The center is (−1, 2); a = 2 and b = 3.

The transverse axis is vertical, so the vertices are 2 unitsbelow and above the center:

(−1, 2 − 2) and (−1, 2 + 2) or (−1, 0) and (−1, 4).

Since c2 = a2 + b2, we have c2 = 4 + 9 = 13 and c =√

13.Then the foci are

√13 units below and above the center:

(−1, 2 −√13) and (−1, 2 +

√13).

Find the asymptotes:

y − k =a

b(x− h) and y − k = −a

b(x− h)

y − 2 =23(x− (−1)) and y − 2 = −2

3(x− (−1))

y − 2 =23(x+ 1) and y − 2 = −2

3(x+ 1)

y =23x+

83

and y = −23x+

43

Copyright © 2013 Pearson Education, Inc.

Page 308: JUDITH A. PENNA -  · PDF fileSTUDENT’S SOLUTIONS MANUAL JUDITH A. PENNA Indiana University Purdue University Indianapolis COLLEGE ALGEBRA: GRAPHS AND MODELS FIFTH EDITION

���� � � ���� � � 1 (y � 2)2

4(x � 1)2

9

2 4�4 �2

2

4

�4

�2

y

x

Chapter 7 Test 305

14. 2y2 − x2 = 18

y2

9− x2

18= 1

y2

32− x2

(3√

2)2= 1

h = 0, k = 0, a = 3, b = 3√

2

y − k =a

b(x− h) and y − k = −a

b(x− h)

y − 0 =3

3√

2(x− 0) and y − 0 = − 3

3√

2(x− 0)

y =√

22

x and y = −√

22

x

15. The parabola is of the form y2 = 4px. A point on the

parabola is(

6,182

), or (6, 9).

y2 = 4px

92 = 4 · p · 681 = 24p278

= p

Since the focus is at (p, 0) =(

278, 0)

, the focus is278

in.

from the vertex.

16. 2x2 − 3y2 = −10, (1)

x2 + 2y2 = 9 (2)

2x2 − 3y2 = −10

−2x2 − 4y2 = −18 Multiplying (2) by −2− 7y2 = −28 Adding

y2 = 4

y = ±2

x2 + 2(±2)2 = 9 Substituting into (2)

x2 + 8 = 9

x2 = 1

x = ±1

The pairs (1, 2), (1,−2), (−1, 2) and (−1,−2) check.

17. x2 + y2 = 13, (1)

x + y = 1 (2)

First solve equation (2) for y.

y = 1 − x

Then substitute 1 − x for y in equation (1) and solve forx.

x2 + (1 − x)2 = 13

x2 + 1 − 2x + x2 = 13

2x2 − 2x− 12 = 0

2(x2 − x− 6) = 0

2(x− 3)(x + 2) = 0x = 3 or x = −2

If x = 3, y = 1 − 3 = −2. If x = −2, y = 1 − (−2) = 3.

The pairs (3,−2) and (−2, 3) check.

18. x + y = 5, (1)

xy = 6 (2)

First solve equation (1) for y.

y = −x + 5

Then substitute −x+ 5 for y in equation (2) and solve forx.

x(−x + 5) = 6

−x2 + 5x− 6 = 0

−1(x2 − 5x + 6) = 0

−1(x− 2)(x− 3) = 0x = 2 or x = 3

If x = 2, y = −2 + 5 = 3. If x = 3, y = −3 + 5 = 2.

The pairs (2, 3) and (3, 2) check.

19. Familiarize. Let l and w represent the length and widthof the rectangle, in feet, respectively.

Translate. The perimeter is 18 ft.

2l + 2w = 18 (1)

From the Pythagorean theorem, we have

l2 + w2 = (√

41)2 (2)

Carry out. We solve the system of equations. We firstsolve equation (1) for w.

2l + 2w = 18

2w = 18 − 2l

w = 9 − l

Then substitute 9− l for w in equation (2) and solve for l.

l2 + (9 − l)2 = (√

41)2

l2 + 81 − 18l + l2 = 41

2l2 − 18l + 40 = 0

2(l2 − 9l + 20) = 0

2(l − 4)(l − 5) = 0

l = 4 or l = 5

If l = 4, then w = 9 − 4 = 5. If l = 5, then w = 9 − 5 = 4.Since length is usually considered to be longer than width,we have l = 5 and w = 4.

Check. The perimeter is 2 · 5 + 2 · 4, or 18 ft. The lengthof a diagonal is

√52 + 42, or

√41 ft. The solution checks.

State. The dimensions of the garden are 5 ft by 4 ft.

Copyright © 2013 Pearson Education, Inc.

Page 309: JUDITH A. PENNA -  · PDF fileSTUDENT’S SOLUTIONS MANUAL JUDITH A. PENNA Indiana University Purdue University Indianapolis COLLEGE ALGEBRA: GRAPHS AND MODELS FIFTH EDITION

1 4

2

�4�5 �3 �1�1

�3

�5

1

345

3 5

y

x

(3, 5)

(�1, �3)

306 Chapter 7: Conic Sections

20. Familiarize. Let l and w represent the length and widthof the playground, in feet, respectively.

Translate.

Perimeter: 2l + 2w = 210 (1)

Area: lw = 2700 (2)

Carry out. We solve the system of equations. First solveequation (2) for w.

w =2700l

Then substitute2700l

for w in equation (1) and solve forl.

2l + 2 · 2700l

= 210

2l +5400l

= 210

2l2 + 5400 = 210l Multiplying by l

2l2 − 210l + 5400 = 0

2(l2 − 105l + 2700) = 0

2(l − 45)(l − 60) = 0

l = 45 or l = 60

If l = 45, then w =270045

= 60. If l = 60, then w =270060

=45. Since length is usually considered to be longer thanwidth, we have l = 60 and w = 45.

Check. Perimeter: 2 · 60 + 2 · 45 = 210 ft

Area: 60 · 45 = 2700 ft2

The solution checks.

State. The dimensions of the playground are 60 ft by45 ft.

21. Graph: y ≥ x2 − 4,

y < 2x− 1

The solution set of y ≥ x2 − 4 is the parabola y = x2 − 4and the region inside it. The solution set of y < 2x − 1is the half-plane below the line y = 2x− 1. We shade theregion common to the two solution sets.

To find the points of intersection of the graphs of the re-lated equations we solve the system of equations

y = x2 − 4,

y = 2x− 1.

The points of intersection are (−1,−3) and (3, 5).

22. (y−1)2 = 4(x+1) represents a parabola with vertex (−1, 1)that opens to the right. Thus the correct answer is A.

23. Use the midpoint formula to find the center.

(h, k) =(

1 + 52

,1 + (−3)

2

)= (3,−1)

Use the distance formula to find the radius.

r =12

√(1 − 5)2 + (1−(−3))2 =

12

√(−4)2 + (4)2 = 2

√2

Write the equation of the circle.

(x− h)2 + (y − k)2 = r2

(x− 3)2 + [y − (−1)]2 = (2√

2)2

(x− 3)2 + (y + 1)2 = 8

Copyright © 2013 Pearson Education, Inc.

Page 310: JUDITH A. PENNA -  · PDF fileSTUDENT’S SOLUTIONS MANUAL JUDITH A. PENNA Indiana University Purdue University Indianapolis COLLEGE ALGEBRA: GRAPHS AND MODELS FIFTH EDITION

0 100

3

Chapter 8

Sequences, Series, and Combinatorics

Exercise Set 8.1

1. an = 4n− 1

a1 = 4 · 1 − 1 = 3,

a2 = 4 · 2 − 1 = 7,

a3 = 4 · 3 − 1 = 11,

a4 = 4 · 4 − 1 = 15;

a10 = 4 · 10 − 1 = 39;

a15 = 4 · 15 − 1 = 59

3. an =n

n− 1, n ≥ 2

The first 4 terms are a2, a3, a4, and a5:

a2 =2

2 − 1= 2,

a3 =3

3 − 1=

32,

a4 =4

4 − 1=

43,

a5 =5

5 − 1=

54;

a10 =10

10 − 1=

109

;

a15 =15

15 − 1=

1514

5. an =n2 − 1n2 + 1

,

a1 =12 − 112 + 1

= 0,

a2 =22 − 122 + 1

=35,

a3 =32 − 132 + 1

=810

=45,

a4 =42 − 142 + 1

=1517

;

a10 =102 − 1102 + 1

=99101

;

a15 =152 − 1152 + 1

=224226

=112113

7. an = (−1)nn2

a1 = (−1)112 = −1,

a2 = (−1)222 = 4,

a3 = (−1)332 = −9,

a4 = (−1)442 = 16;

a10 = (−1)10102 = 100;

a15 = (−1)15152 = −225

9. an = 5 +(−2)n+1

2n

a1 = 5 +(−2)1+1

21= 5 +

42

= 7,

a2 = 5 +(−2)2+1

22= 5 +

−84

= 3,

a3 = 5 +(−2)3+1

23= 5 +

168

= 7,

a4 = 5 +(−2)4+1

24= 5 +

−3216

= 3;

a10 = 5 +(−2)10+1

210= 5 +

−1 · 211

210= 3;

a15 = 5 +(−2)15+1

215= 5 +

216

215= 7

11. an = 5n− 6

a8 = 5 · 8 − 6 = 40 − 6 = 34

13. an = (2n+ 3)2

a6 = (2 · 6 + 3)2 = 225

15. an = 5n2(4n− 100)

a11 = 5(11)2(4 · 11 − 100) = 5(121)(−56) =

−33, 880

17. an = ln en

a67 = ln e67 = 67

19. n Un

1 2

2 2.25

3 2.3704

4 2.4414

5 2.4883

6 2.5216

7 2.5465

8 2.5658

9 2.5812

10 2.5937

Copyright © 2013 Pearson Education, Inc.

Page 311: JUDITH A. PENNA -  · PDF fileSTUDENT’S SOLUTIONS MANUAL JUDITH A. PENNA Indiana University Purdue University Indianapolis COLLEGE ALGEBRA: GRAPHS AND MODELS FIFTH EDITION

0 100

3

308 Chapter 8: Sequences, Series, and Combinatorics

21.n Un

1 2

2 1.5538

3 1.4998

4 1.4914

5 1.4904

6 1.4902

7 1.4902

8 1.4902

9 1.4902

10 1.4902

23. 2, 4, 6, 8, 10,. . .

These are the even integers, so the general term might be2n.

25. −2, 6, −18, 54, . . .

We can see a pattern if we write the sequence as

−1 · 2 · 1, 1 · 2 · 3, −1 · 2 · 9, 1 · 2 · 27, . . .

The general term might be (−1)n2(3)n−1.

27.23,

34,

45,

56,

67, . . .

These are fractions in which the denominator is 1 greaterthan the numerator. Also, each numerator is 1 greaterthan the preceding numerator. The general term might ben+ 1n+ 2

.

29. 1 · 2, 2 · 3, 3 · 4, 4 · 5, . . .

These are the products of pairs of consecutive natural num-bers. The general term might be n(n+ 1).

31. 0, log 10, log 100, log 1000, . . .

We can see a pattern if we write the sequence as

log 1, log 10, log 100, log 1000, . . .

The general term might be log 10n−1. This is equivalentto n− 1.

33. 1, 2, 3, 4, 5, 6, 7, . . .

S3 = 1 + 2 + 3 = 6

S7 = 1 + 2 + 3 + 4 + 5 + 6 + 7 = 28

35. 2, 4, 6, 8, . . .

S4 = 2 + 4 + 6 + 8 = 20

S5 = 2 + 4 + 6 + 8 + 10 = 30

37.5∑

k=1

12k

=1

2 · 1 +1

2 · 2 +1

2 · 3 +1

2 · 4 +1

2 · 5=

12

+14

+16

+18

+110

=60120

+30120

+20120

+15120

+12120

=137120

39.6∑

i=0

2i = 20 + 21 + 22 + 23 + 24 + 25 + 26

= 1 + 2 + 4 + 8 + 16 + 32 + 64

= 127

41.10∑k=7

ln k = ln 7 + ln 8 + ln 9 + ln 10 =

ln(7 · 8 · 9 · 10) = ln 5040 ≈ 8.5252

43.8∑

k=1

k

k + 1=

11 + 1

+2

2 + 1+

33 + 1

+4

4 + 1+

55 + 1

+6

6 + 1+

77 + 1

+8

8 + 1

=12

+23

+34

+45

+56

+67

+78

+89

=15, 5512520

45.5∑

i=1

(−1)i

= (−1)1 + (−1)2 + (−1)3 + (−1)4 + (−1)5

= −1 + 1 − 1 + 1 − 1

= −1

47.8∑

k=1

(−1)k+13k

= (−1)23 · 1 + (−1)33 · 2 + (−1)43 · 3+

(−1)53 · 4 + (−1)63 · 5 + (−1)73 · 6+

(−1)83 · 7 + (−1)93 · 8= 3 − 6 + 9 − 12 + 15 − 18 + 21 − 24

= −12

49.6∑

k=0

2k2+1

=2

02 + 1+

212 + 1

+2

22 + 1+

232 + 1

+

242 + 1

+2

52 + 1+

262 + 1

= 2 + 1 +25

+210

+217

+226

+237

= 2 + 1 +25

+15

+217

+113

+237

=157, 35140, 885

51.5∑

k=0

(k2 − 2k + 3)

= (02 − 2 · 0 + 3) + (12 − 2 · 1 + 3)+

(22 − 2 · 2 + 3) + (32 − 2 · 3 + 3)+

(42 − 2 · 4 + 3) + (52 − 2 · 5 + 3)= 3 + 2 + 3 + 6 + 11 + 18

= 43

Copyright © 2013 Pearson Education, Inc.

Page 312: JUDITH A. PENNA -  · PDF fileSTUDENT’S SOLUTIONS MANUAL JUDITH A. PENNA Indiana University Purdue University Indianapolis COLLEGE ALGEBRA: GRAPHS AND MODELS FIFTH EDITION

Exercise Set 8.1 309

53.10∑i=0

2i2i + 1

=20

20+1+

21

21+1+

22

22+1+

23

23+1+

24

24+1+

25

25+1+

26

26 + 1+

27

27 + 1+

28

28 + 1+

29

29 + 1+

210

210 + 1

=12

+23

+45

+89

+1617

+3233

+6465

+128129

+

256257

+512513

+10241025

≈ 9.736

55. 5 + 10 + 15 + 20 + 25+ . . .

This is a sum of multiples of 5, and it is an infinite series.Sigma notation is

∞∑k=1

5k.

57. 2 − 4 + 8 − 16 + 32 − 64

This is a sum of powers of 2 with alternating signs. Sigmanotation is

6∑k=1

(−1)k+12k, or6∑

k=1

(−1)k−12k

59. −12

+23− 3

4+

45− 5

6+

67

This is a sum of fractions in which the denominator isone greater than the numerator. Also, each numeratoris 1 greater than the preceding numerator and the signsalternate. Sigma notation is

6∑k=1

(−1)kk

k + 1.

61. 4 − 9 + 16 − 25 + . . . + (−1)nn2

This is a sum of terms of the form (−1)kk2, beginning withk = 2 and continuing through k = n. Sigma notation is

n∑k=2

(−1)kk2.

63.1

1 · 2 +1

2 · 3 +1

3 · 4 +1

4 · 5 + . . .

This is a sum of fractions in which the numerator is 1 andthe denominator is a product of two consecutive integers.The larger integer in each product is the smaller integerin the succeeding product. It is an infinite series. Sigmanotation is

∞∑k=1

1k(k + 1)

.

65. a1 = 4, ak+1 = 1 +1ak

a2 = 1 +14

= 114, or

54

a3 = 1 +154

= 1 +45

= 145, or

95

a4 = 1 +195

= 1 +59

= 159, or

149

67. a1 = 6561, ak+1 = (−1)k√ak

a2 = (−1)1√

6561 = −81

a3 = (−1)2√−81 = 9i

a4 = (−1)3√

9i = −3√i

69. a1 = 2, ak+1 = ak + ak−1

a2 = 3

a3 = 3 + 2 = 5

a4 = 5 + 3 = 8

71. a) a1 = $1000(1.062)1 = $1062

a2 = $1000(1.062)2 ≈ $1127.84

a3 = $1000(1.062)3 ≈ $1197.77

a4 = $1000(1.062)4 ≈ $1272.03

a5 = $1000(1.062)5 ≈ $1350.90

a6 = $1000(1.062)6 ≈ $1434.65

a7 = $1000(1.062)7 ≈ $1523.60

a8 = $1000(1.062)8 ≈ $1618.07

a9 = $1000(1.062)9 ≈ $1718.39

a10 = $1000(1.062)10 ≈ $1824.93

b) a20 = $1000(1.062)20 ≈ $3330.35

73. Find each term by adding $1.10 to the preceding term:

$9.80, $10.90, $12, $13.10, $14.20, $15.30, $16.40, $17.50,$18.60, $19.70

75. a1 = 1 (Given)

a2 = 1 (Given)

a3 = a2 + a1 = 1 + 1 = 2

a4 = a3 + a2 = 2 + 1 = 3

a5 = a4 + a3 = 3 + 2 = 5

a6 = a5 + a4 = 5 + 3 = 8

a7 = a6 + a5 = 8 + 5 = 13

77. a) an = −659.8950216n2 + 6297.77684n+ 54, 737.29091

b) In 2010, n = 2010 − 2001 = 9.

a9 ≈ 57, 966 thousand, 57,966,000 vehicles

In 2011, n = 2011 − 2001 = 10.

a10 ≈ 51, 726 thousand, or 51,726,000 vehicles

In 2012, n = 2012 − 2001 = 11.

a11 ≈ 44, 166 thousand, or 44,166,000 vehicles

79. Familiarize. Let x, y and z represent the numberof patent applications in the United States, Japan, andChina, respectively.

Copyright © 2013 Pearson Education, Inc.

Page 313: JUDITH A. PENNA -  · PDF fileSTUDENT’S SOLUTIONS MANUAL JUDITH A. PENNA Indiana University Purdue University Indianapolis COLLEGE ALGEBRA: GRAPHS AND MODELS FIFTH EDITION

310 Chapter 8: Sequences, Series, and Combinatorics

Translate. The total number of applications was 89,348,so we have one equation.

x+ y + z = 89, 348

The number of applications in China was 3741 less thanone-half the number of applications in Japan, so we havea second equation.

z =12y − 3741.

The number of applications in the United States was32,518 more than the number of applications in China,so we have a third equation.

x = z + 32, 518

We have a system of equations.x+ y + z = 89, 348,

z =12y − 3741,

x = z + 32, 518.

Carry out. Solving the system of equations, we get(44, 855, 32, 156, 12, 337).

Check. The total number of applications was44, 855 + 32, 156 + 12, 337 = 89, 348. The number ofapplications in Japan was 32,156. We see that12· 32, 156 − 3741 = 16, 078 − 3741 = 12, 337, which was

the number of applications in China. We also see that32,518 more than this number is 12, 337+32, 518, or 44,855,which was the number of applications in the United States.The answer checks.

State. The number of patent applications in the UnitedStates, Japan, and China was 44,855, 32,156, and 12,337,respectively.

81. We complete the square twice.x2 + y2 + 5x− 8y = 2

x2 + 5x+ y2 − 8y = 2

x2 + 5x+254

+ y2 − 8y + 16 = 2 +254

+ 16(x+

52

)2

+ (y − 4)2 =974[

x−(− 5

2

)]2+ (y − 4)2 =

(√972

)2

The center is(− 5

2, 4)

and the radius is√

972

.

83. an = in

a1 = i

a2 = i2 = −1

a3 = i3 = −ia4 = i4 = 1

a5 = i5 = i4 · i = i;

S5 = i− 1 − i+ 1 + i = i

85. Sn = ln 1 + ln 2 + ln 3 + · · · + lnn

= ln(1 · 2 · 3 · · ·n)

Exercise Set 8.2

1. 3, 8, 13, 18, . . .a1 = 3

d = 5 (8 − 3 = 5, 13 − 8 = 5, 18 − 13 = 5)

3. 9, 5, 1, −3, . . .a1 = 9

d = −4 (5−9 = −4, 1−5 = −4, −3−1 = −4)

5.32,

94, 3,

154

, . . .

a1 =32

d =34

(94− 3

2=

34, 3 − 9

4=

34

)

7. a1 = $316

d = −$3 ($313 − $316 = −$3,$310 − $313 = −$3, $307 − $310 = −$3)

9. 2, 6, 10, . . .a1 = 2, d = 4, and n = 12

an = a1 + (n− 1)d

a12 = 2 + (12 − 1)4 = 2 + 11 · 4 = 2 + 44 = 46

11. 3,73,

53, . . .

a1 = 3, d = −23, and n = 14

an = a1 + (n− 1)d

a14 = 3 + (14 − 1)(− 2

3

)= 3 − 26

3= −17

3

13. $2345.78, $2967.54, $3589.30, . . .a1 = $2345.78, d = $621.76, and n = 10

an = a1 + (n− 1)d

a10 = $2345.78 + (10 − 1)($621.76) = $7941.62

15. 106 = 2 + (n− 1)(4)

106 = 2 + 4n− 4

108 = 4n

27 = n

The 27th term is 106.

17. a1 = 3, d = −23

an = a1 + (n− 1)d

Let an = −27, and solve for n.

−27 = 3 + (n− 1)(− 2

3

)−81 = 9 + (n− 1)(−2)

−81 = 9 − 2n+ 2

−92 = −2n

46 = n

The 46th term is −27.

Copyright © 2013 Pearson Education, Inc.

Page 314: JUDITH A. PENNA -  · PDF fileSTUDENT’S SOLUTIONS MANUAL JUDITH A. PENNA Indiana University Purdue University Indianapolis COLLEGE ALGEBRA: GRAPHS AND MODELS FIFTH EDITION

Exercise Set 8.2 311

19. an = a1 + (n− 1)d

33 = a1 + (8 − 1)4 Substituting 33 for a8,8 for n, and 4 for d

33 = a1 + 28

5 = a1

(Note that this procedure is equivalent to subtracting dfrom a8 seven times to get a1: 33 − 7(4) = 33 − 28 = 5)

21. an = a1 + (n− 1)d

−507 = 25 + (n− 1)(−14)

−507 = 25 − 14n+ 14

−546 = −14n

39 = n

23. 253

+ 15d =956

15d =456

d =12

a1 =253

− 16(1

2

)=

253

− 8 =13

The first five terms of the sequence are13,

56,

43,

116

,73.

25. 5 + 8 + 11 + 14 + . . .

Note that a1 = 5, d = 3, and n = 20. First we find a20:

an = a1 + (n− 1)d

a20 = 5 + (20 − 1)3

= 5 + 19 · 3 = 62

Then

Sn =n

2(a1 + an)

S20 =202

(5 + 62)

= 10(67) = 670.

27. The sum is 2+4+6+ . . . +798+800. This is the sum ofthe arithmetic sequence for which a1 = 2, an = 800, andn = 400.

Sn =n

2(a1 + an)

S400 =4002

(2 + 800) = 200(802) = 160, 400

29. The sum is 7 + 14 + 21 + . . . + 91 + 98. This is the sumof the arithmetic sequence for which a1 = 7, an = 98, andn = 14.

Sn =n

2(a1 + an)

S14 =142

(7 + 98) = 7(105) = 735

31. First we find a20:an = a1 + (n− 1)d

a20 = 2 + (20 − 1)5

= 2 + 19 · 5 = 97

Then

Sn =n

2(a1 + an)

S20 =202

(2 + 97)

= 10(99) = 990.

33.40∑k=1

(2k + 3)

Write a few terms of the sum:

5 + 7 + 9+ . . . +83

This is a series coming from an arithmetic sequence witha1 = 5, n = 40, and a40 = 83. Then

Sn =n

2(a1 + an)

S40 =402

(5 + 83)

= 20(88) = 1760

35.19∑k=0

k − 34

Write a few terms of the sum:

−34− 1

2− 1

4+ 0 +

14+ . . . +4

Since k goes from 0 through 19, there are 20 terms. Thus,this is equivalent to a series coming from an arithmetic

sequence with a1 = −34, n = 20, and a20 = 4. Then

Sn =n

2(a1 + an)

S20 =202

(− 3

4+ 4)

= 10 · 134

=652.

37.57∑

k=12

7 − 4k13

Write a few terms of the sum:

−4113

− 4513

− 4913

− . . . −22113

Since k goes from 12 through 57, there are 46 terms. Thus,this is equivalent to a series coming from an arithmetic

sequence with a1 = −4113

, n = 46, and a46 = −22113

. Then

Sn =n

2(a1 + an)

S46 =462

(− 41

13− 221

13

)

= 23(− 262

13

)= −6026

13.

39. Familiarize. We have a sequence $5000, $6125, $7250, . . ..It is an arithmetic sequence with a1 = 5000, d = 1125, andn = 25.

Translate. We want to find Sn =n

2(a1 + an) where

an = a1 + (n− 1)d, a1 = 5000, d = 1125, and n = 25.

Copyright © 2013 Pearson Education, Inc.

Page 315: JUDITH A. PENNA -  · PDF fileSTUDENT’S SOLUTIONS MANUAL JUDITH A. PENNA Indiana University Purdue University Indianapolis COLLEGE ALGEBRA: GRAPHS AND MODELS FIFTH EDITION

312 Chapter 8: Sequences, Series, and Combinatorics

Carry out. First we find a25.

a25 = 5000 + (25 − 1)1125 = 5000 + 27, 000 = 32, 000

Then S25 =252

(5000 + 32, 000) =252

· 37, 000 = 462, 500.

Check. We can do the calculations again, or we can dothe entire addition:

5000 + 6125 + 7250 + . . .+ 32, 000.

The answer checks.

State. The total amount received from the investmentwas $462,500.

41. Familiarize. We have arithmetic sequence with a1 = 28,d = 4, and n = 20.

Translate. We want to find Sn =n

2(a1 + an) where an =

a1 + (n− 1)d, a1 = 28, d = 4, and n = 20.

Carry out. First we find a20.

a20 = 28 + (20 − 1)4 = 104

Then S20 =202

(28 + 104) = 10 · 132 = 1320.

Check. We can do the calculations again, or we can dothe entire addition:

28 + 32 + 36+ . . . 104.

The answer checks.

State. There are 1320 seats in the first balcony.

43. Yes; d = 48 − 16 = 80 − 48 = 112 − 80 = 144 − 112 = 32.

a10 = 16 + (10 − 1)32 = 304

S10 =102

(16 + 304) = 1600 ft

45. Familiarize. We have a sequence 10, 12, 14, . . .. It is anarithmetic sequence with a1 = 10, d = 2, and n = 8.

Translate. We want to find Sn =n

2(a1 + an), where an =

a1 + (n− 1)d, a1 = 10, d = 2, and n = 8.

Carry out. First we find a8.

a8 = 10 + (8 − 1)(2) = 10 + 7 · 2 = 24

Then S8 =82(10 + 24) = 4 · 34 = 136

Check. We can do the calculations again, or we can dothe entire addition:

10 + 12 + 14 + . . .+ 24.

The answer checks.

State. There are 24 marchers in the last row, and thereare 136 marchers altogether.

47. Yes; d = 6 − 3 = 9 − 6 = 3n− 3(n− 1) = 3.

49. 2x + y + 3z = 12

x − 3y − 2z = −1

5x + 2y − 4z = −4

We will use Gauss-Jordan elimination with matrices. Firstwe write the augmented matrix.

2 1 3 12

1 −3 −2 −1

5 2 −4 −4

Next we interchange the first two rows.

1 −3 −2 −1

2 1 3 12

5 2 −4 −4

Now multiply the first row by −2 and add it to the secondrow. Also multiply the first row by −5 and add it to thethird row.

1 −3 −2 −1

0 7 7 14

0 17 6 1

Multiply the second row by17.

1 −3 −2 −1

0 1 1 2

0 17 6 1

Multiply the second row by 3 and add it to the first row.Also multiply the second row by −17 and add it to thethird row.

1 0 1 5

0 1 1 2

0 0 −11 −33

Multiply the third row by − 111

.

1 0 1 5

0 1 1 2

0 0 1 3

Multiply the third row by −1 and add it to the first rowand also to the second row.

1 0 0 2

0 1 0 −1

0 0 1 3

Now we can read the solution from the matrix. It is(2,−1, 3).

51. The vertices are on the y-axis, so the transverse axis isvertical and a = 5. The length of the minor axis is 4, sob = 4/2 = 2. The equation is

x2

4+y2

25= 1.

53. Sn =n

2(1 + 2n− 1) = n2

Copyright © 2013 Pearson Education, Inc.

Page 316: JUDITH A. PENNA -  · PDF fileSTUDENT’S SOLUTIONS MANUAL JUDITH A. PENNA Indiana University Purdue University Indianapolis COLLEGE ALGEBRA: GRAPHS AND MODELS FIFTH EDITION

Exercise Set 8.3 313

55. Let d = the common difference. Then a4 = a2 + 2d, or

10p+ q = 40 − 3q + 2d

10p+ 4q − 40 = 2d

5p+ 2q − 20 = d.

Also, a1 = a2 − d, so we have

a1 = 40 − 3q − (5p+ 2q − 20)

= 40 − 3q − 5p− 2q + 20

= 60 − 5p− 5q.

57. 4, m1, m2, m3, m4, 13

We look for m1, m2, m3, and m4 such that 4, m1, m2,m3, m4, 13 is an arithmetic sequence. In this case a1 = 4,n = 6, and a6 = 13. First we find d.

an = a1 + (n− 1)d

13 = 4 + (6 − 1)d

9 = 5d

145

= d

Then we have

m1 = a1 + d = 4 + 145

= 545,

m2 = m1 + d = 545

+ 145

= 685

= 735,

m3 = m2 + d = 735

+ 145

= 875

= 925,

m4 = m3 + d = 925

+ 145

= 1065

= 1115.

Exercise Set 8.3

1. 2, 4, 8, 16, . . .42

= 2,84

= 2,168

= 2

r = 2

3. 1, −1, 1, −1, . . .−11

= −1,1−1

= −1,−11

= −1

r = −1

5.23, −4

3,

83, −16

3, . . .

−43

23

= −2,

83

−43

= −2,−16

383

= −2

r = −2

7.0.62756.275

= 0.1,0.062750.6275

= 0.1

r = 0.1

9.

5a25

=a

2,

5a2

45a2

=a

2,

5a3

85a4

=a

2

r =a

2

11. 2, 4, 8, 16, . . .

a1 = 2, n = 7, and r =42, or 2.

We use the formula an = a1rn−1.

a7 = 2(2)7−1 = 2 · 26 = 2 · 64 = 128

13. 2, 2√

3, 6, . . .

a1 = 2, n = 9, and r =2√

32, or

√3

an = a1rn−1

a9 = 2(√

3)9−1 = 2(√

3)8 = 2 · 81 = 162

15.7

625, − 7

25, . . .

a1 =7

625, n = 23, and r =

− 7257

625

= −25.

an = a1rn−1

a23 =7

625(−25)23−1 =

7625

(−25)22

=7

252· 252 · 2520 = 7(25)20, or 7(5)40

17. 1, 3, 9, . . .

a1 = 1 and r =31, or 3

an = a1rn−1

an = 1(3)n−1 = 3n−1

19. 1, −1, 1, −1, . . .

a1 = 1 and r =−11

= −1

an = a1rn−1

an = 1(−1)n−1 = (−1)n−1

21.1x

,1x2

,1x3

, . . .

a1 =1x

and r =

1x2

1x

=1x

an = a1rn−1

an =1x

( 1x

)n−1

=1x· 1xn−1

=1

x1+n−1=

1xn

Copyright © 2013 Pearson Education, Inc.

Page 317: JUDITH A. PENNA -  · PDF fileSTUDENT’S SOLUTIONS MANUAL JUDITH A. PENNA Indiana University Purdue University Indianapolis COLLEGE ALGEBRA: GRAPHS AND MODELS FIFTH EDITION

314 Chapter 8: Sequences, Series, and Combinatorics

23. 6 + 12 + 24+ . . .

a1 = 6, n = 7, and r =126, or 2

Sn =a1(1 − rn)

1 − rS7 =

6(1 − 27)1 − 2

=6(1 − 128)

−1=

6(−127)−1

= 762

25.118

− 16

+12− . . .

a1 =118, n = 9, and r =

−16

118

= −16· 18

1= −3

Sn =a1(1 − rn)

1 − r

S9 =

118

[1−(−3)9

]1 − (−3)

=

118

(1 + 19, 683)

4118

(19, 684)

4=

118

(19, 684)(1

4

)=

492118

27. Multiplying each term of the sequence by −√2 produces

the next term, so it is true that the sequence is geometric.

29. Since2n+1

2n= 2, the sequence has a common ratio so it is

true that the sequence is geometric.

31. Since | − 0.75| < 1, it is true that the series has a sum.

33. 4 + 2 + 1 + . . .

|r| =∣∣∣24

∣∣∣ = ∣∣∣12

∣∣∣ = 12, and since |r| < 1, the series

does have a sum.

S∞ =a1

1 − r =4

1 − 12

=412

= 4 · 21

= 8

35. 25 + 20 + 16 + . . .

|r| =∣∣∣2025

∣∣∣ = ∣∣∣45

∣∣∣ = 45, and since |r| < 1, the

series does have a sum.

S∞ =a1

1 − r =25

1 − 45

=2515

= 25 · 51

= 125

37. 8 + 40 + 200 + . . .

|r| =∣∣∣40

8

∣∣∣ = |5| = 5, and since |r| > 1 the series does nothave a sum.

39. 0.6 + 0.06 + 0.006+ . . .

|r| =∣∣∣∣0.06

0.6

∣∣∣∣ = |0.1| = 0.1, and since |r| < 1, the series does

have a sum.

S∞ =a1

1 − r =0.6

1 − 0.1=

0.60.9

=69

=23

41.11∑k=1

15(

23

)k

a1 = 15 · 23

or 10; |r| =∣∣23

∣∣∣∣ = 23, n = 11

S11 =10[1 −

(23

)11]

1 − 23

=10[1 − 2048

177, 147

]13

= 10 · 175, 099177, 147

· 3

=1, 750, 990

59, 049, or 29

38, 56959, 049

43.∞∑k=1

(12

)k−1

a1 = 1, |r| =∣∣∣∣12∣∣∣∣ = 1

2

S∞ =a1

1 − r =1

1 − 12

=112

= 2

45.∞∑k=1

12.5k

Since |r| = 12.5 > 1, the sum does not exist.

47.∞∑k=1

$500(1.11)−k

a1 = $500(1.11)−1, or$5001.11

; |r| = |1.11−1| =1

1.11

S∞ =a1

1 − r =

$5001.11

1 − 11.11

=

$5001.110.111.11

≈ $4545.45

49.∞∑k=1

16(0.1)k−1

a1 = 16, |r| = |0.1| = 0.1

S∞ =a1

1 − r =16

1 − 0.1=

160.9

=1609

51. 0.131313 . . . = 0.13 + 0.0013 + 0.000013+ . . .

This is an infinite geometric series with a1 = 0.13.

|r| =∣∣∣∣0.0013

0.13

∣∣∣∣ = |0.01| = 0.01 < 1, so the series has a limit.

S∞ =a1

1 − r =0.13

1 − 0.01=

0.130.99

=1399

53. We will find fraction notation for 0.9999 and then add 8.

0.9999 = 0.9 + 0.09 + 0.009 + 0.0009+. . .

This is an infinite geometric series with a1 = 0.9.

|r| =∣∣∣∣0.09

0.9

∣∣∣∣ = |0.1| = 0.1 < 1, so the series has a limit.

S∞ =a1

1 − r =0.9

1 − 0.1=

0.90.9

= 1

Then 8.9999 = 8 + 1 = 9.

Copyright © 2013 Pearson Education, Inc.

Page 318: JUDITH A. PENNA -  · PDF fileSTUDENT’S SOLUTIONS MANUAL JUDITH A. PENNA Indiana University Purdue University Indianapolis COLLEGE ALGEBRA: GRAPHS AND MODELS FIFTH EDITION

Exercise Set 8.3 315

55. 3.4125125 = 3.4 + 0.0125125

We will find fraction notation for 0.0125125 and then add

3.4, or3410

, or175

.

0.0125125 = 0.0125 + 0.0000125+ . . .

This is an infinite geometric series with a1 = 0.0125.

|r| =∣∣∣∣0.0000125

0.0125

∣∣∣∣ = |0.001| = 0.001 < 1, so the series has

a limit.

S∞ =a1

1 − r =0.0125

1 − 0.001=

0.01250.999

=1259990

Then175

+1259990

=33, 9669990

+1259990

=34, 0919990

57. Familiarize. The total earnings are represented by thegeometric series

$0.01 + $0.01(2) + $0.01(2)2 + . . .+ $0.01(2)27, wherea1 = $0.01, r = 2, and n = 28.

Translate. Using the formula

Sn =a1(1 − rn)

1 − rwe have

S28 =$0.01(1 − 228)

1 − 2.

Carry out. We carry out the computation and get$2,684,354.55.

Check. Repeat the calculation.

State. You would earn $2,684,354.55.

59. a) Familiarize. The rebound distances form a geo-metric sequence:

0.6 × 200, (0.6)2 × 200, (0.6)3 × 200, . . . ,

or 120, 0.6 × 120, (0.6)2 × 120, . . .

The total rebound distance after 9 rebounds is thesum of the first 9 terms of this sequence.

Translate. We will use the formula

Sn =a1(1 − rn)

1 − r with a1 = 120, r = 0.6, and n = 9.

Carry out.

S9 =120[1 − (0.6)9]

1 − 0.6≈ 297

Check. We repeat the calculation.

State. The bungee jumper has traveled about 297 ftupward after 9 rebounds.

b) S∞ =a1

1 − r =120

1 − 0.6= 300 ft

61. Familiarize. The amount of the annuity is the geometricseries

$3200 + $3200(1.046) + $3200(1.046)2 + . . . +$3200(1.046)9, where a1 = $3200, r = 1.046, and n = 10.

Translate. Using the formula

Sn =a1(1 − rn)

1 − rwe have

S10 =$3200[1 − (1.046)10]

1 − 1.046.

Carry out. We carry out the computation and get S10 ≈$39, 505.71.

Check. Repeat the calculations.

State. The amount of the annuity is $39,505.71.

63. Familiarize. We have a sequence 0.01, 2(0.01), 22(0.01),23(0.01), . . . . The thickness after 20 folds is given by the21st term of the sequence.

Translate. Using the formula

an = a1rn−1,

where a1 = 0.01, r = 2, and n = 21, we have

a21 = 0.01(2)21−1.

Carry out. We carry out the computation and get10,485.76.

Check. Repeat the calculations.

State. The result is 10,485.76 in. thick.

65. We use the formula

V =P

[(1 +

i

n

)nN

− 1]

i/n

with P = 300, i = 5.1%, or 0.051, n = 4, and N = 12.

V =300[(

1 +0.051

4

)4·12− 1]

0.051/4V ≈ $19, 694.01

The amount of the annuity is $19,694.01.

67. We first use the formula

S∞ =a1

1 − r ,where a1 = 30% of 5,000,000 or 0.3(5,000,000), and r =30%, or 0.3.

S∞ =0.3(5, 000, 000)

1 − 0.3≈ 2, 142, 857

In all, the advertising campaign will reach about 2,142,857people.

Since2, 142, 8575, 000, 000

≈ 0.429, or 42.9%, we see that about

42.9% of the population will buy the product.

69. f(x) = x2, g(x) = 4x+ 5

(f ◦ g)(x) = f(g(x)) = f(4x+ 5) = (4x+ 5)2 =

16x2 + 40x+ 25

(g ◦ f)(x) = g(f(x)) = g(x2) = 4x2 + 5

71. 5x = 35

ln 5x = ln 35

x ln 5 = ln 35

x =ln 35ln 5

x ≈ 2.209

73. See the answer section in the text.

Copyright © 2013 Pearson Education, Inc.

Page 319: JUDITH A. PENNA -  · PDF fileSTUDENT’S SOLUTIONS MANUAL JUDITH A. PENNA Indiana University Purdue University Indianapolis COLLEGE ALGEBRA: GRAPHS AND MODELS FIFTH EDITION

316 Chapter 8: Sequences, Series, and Combinatorics

75. a) If the sequence is arithmetic, then a2−a1 = a3−a2.

x+ 7 − (x+ 3) = 4x− 2 − (x+ 7)

x =133

The three given terms are133

+ 3 =223

,133

+ 7 =343

, and 4 · 133

− 2 =463

.

Then d =123

, or 4, so the fourth term is

463

+123

=583

.

b) If the sequence is geometric, then a2/a1 = a3/a2.x+ 7x+ 3

=4x− 2x+ 7

x = −113

or x = 5

For x = −113

: The three given terms are

−113

+ 3 = −23, −11

3+ 7 =

103

, and

4(− 11

3

)− 2 = −50

3.

Then r = −5, so the fourth term is

−503

(−5) =2503

.

For x = 5: The three given terms are 5 + 3 = 8,

5 + 7 = 12, and 4 · 5 − 2 = 18. Then r =32, so the

fourth term is 18 · 32

= 27.

77. x2 − x3 + x4 − x5+ . . .

This is a geometric series with a1 = x2 and r = −x.Sn =

a1(1 − rn)1 − r =

x2(1 − (−x)n)1 − (−x) =

x2(1 − (−x)n)1 + x

Exercise Set 8.4

1. n2 < n3

12 < 13, 22 < 23, 32 < 33, 42 < 43, 52 < 53

The first statement is false, and the others are true.

3. A polygon of n sides hasn(n− 3)

2diagonals.

A polygon of 3 sides has3(3 − 3)

2diagonals.

A polygon of 4 sides has4(4 − 3)

2diagonals.

A polygon of 5 sides has5(5 − 3)

2diagonals.

A polygon of 6 sides has6(6 − 3)

2diagonals.

A polygon of 7 sides has7(7 − 3)

2diagonals.

Each of these statements is true.

5. - 23. See the answer section in the text.

25. Familiarize. Let x, y, and z represent the amounts in-vested at 1.5%, 2%, and 3%, respectively.

Translate. We know that simple interest for one year was$104. This gives us one equation:

0.015x+ 0.02y + 0.03z = 104

The amount invested at 2% is twice the amount investedat 1.5%:

y = 2x, or −2x+ y = 0

There is $400 more invested at 3% than at 2%:

z = y + 400, or −y + z = 400

We have a system of equations:0.015x + 0.02y + 0.03z = 104,

−2x + y = 0

− y + z = 400Carry out. Solving the system of equations, we get(800, 1600, 2000).

Check. Simple interest for one year would be0.015($800)+0.02($1600)+0.03($2000), or $12+$32+$60,or $104. The amount invested at 2%, $1600, is twice $800,the amount invested at 1.5%. The amount invested at 3%,$2000, is $400 more than $1600, the amount invested at2%. The answer checks.

State. Martin invested $800 at 1.5%, $1600 at 2%, and$2000 at 3%.

27. See the answer section in the text.

29. See the answer section in the text.

Chapter 8 Mid-Chapter Mixed Review

1. All of the terms of a sequence with general term an = nare positive. Since the given sequence has negative terms,the given statement is false.

3. a2/a1 = 7/3; a3/a2 = 3/ − 1 = −3; since 7/3 = −3, thesequence is not geometric. The given statement is false.

5. an = 3n+ 5

a1 = 3 · 1 + 5 = 8,

a2 = 3 · 2 + 5 = 11,

a3 = 3 · 3 + 5 = 14,

a4 = 3 · 4 + 5 = 17;

a9 = 3 · 9 + 5 = 32;

a14 = 3 · 14 + 5 = 47

7. 3, 6, 9, 12, 15, . . .

These are multiples of 3, so the general term could be 3n.

9. S4 = 1 +12

+14

+18

= 178, or

158

11. −4 + 8 − 12 + 16 − 20 + . . .

This is an infinite sum of multiples of 4 with alternatingsigns. Sigma notation is

∞∑k=1

(−1)k4k.

Copyright © 2013 Pearson Education, Inc.

Page 320: JUDITH A. PENNA -  · PDF fileSTUDENT’S SOLUTIONS MANUAL JUDITH A. PENNA Indiana University Purdue University Indianapolis COLLEGE ALGEBRA: GRAPHS AND MODELS FIFTH EDITION

Exercise Set 8.5 317

13. 7 − 12 = −5; 2 − 7 = −5; −3 − 2 = −5

The common difference is −5.

15. In Exercise 14 we found that d = 2.an = a1 + (n− 1)d

44 = 4 + (n− 1)2

44 = 4 + 2n− 2

44 = 2 + 2n

42 = 2n

21 = n

The 21st term is 44.

17.−8−16

= −12;

4−8

= −12;−24

= −12

The common ratio is −12.

19. |r| =∣∣∣∣ 4−8

∣∣∣∣ =∣∣∣∣− 1

2

∣∣∣∣ = 12< 1, so the series has a sum.

S∞ =a1

1 − r =−8

1 −(− 1

2

) =−832

= −8 · 23

= −163

21. Familiarize. The number of plants is represented by thearithmetic series 36 + 30 + 24 + . . . with a1 = 36,d = 30 − 36 = −6, and n = 6.

Translate. We want to find Sn =n

2(a1 + an) where

an = a1 + (n− 1)d.

Carry out.

a6 = 36 + (6 − 1)(−6) = 36 + 5(−6) = 36 − 30 = 6

S6 =62(36 + 6) = 3 · 42 = 126

Check. We can do the calculations again or we can dothe entire addition 36 + 30 + 24 + 18 + 12 + 6. The answerchecks.

State. In all, there will be 126 plants.

23. See the answer section in the text.

25. 1 + 2 + 3 + . . .+ 100

= (1 + 100) + (2 + 99) + (3 + 98) + . . .+

(50 + 51)

= 101 + 101 + 101 + . . .+ 101︸ ︷︷ ︸50 addends of 101

= 50 · 101

= 5050

A formula for the first n natural numbers isn

2(1 + n).

27. We can prove an infinite sequence of statements Sn byshowing that a basis statement S1 is true and then thatfor all natural numbers k, if Sk is true, then Sk+1 is true.

Exercise Set 8.5

1. 6P6 = 6! = 6 · 5 · 4 · 3 · 2 · 1 = 720

3. Using formula (1), we have

10P7 = 10 · 9 · 8 · 7 · 6 · 5 · 4 = 604, 800.

Using formula (2), we have

10P7 =10!

(10 − 7)!=

10!3!

=10 · 9 · 8 · 7 · 6 · 5 · 4 · 3!

3!=

604, 800.

5. 5! = 5 · 4 · 3 · 2 · 1 = 120

7. 0! is defined to be 1.

9.9!5!

=9 · 8 · 7 · 6 · 5!

5!= 9 · 8 · 7 · 6 = 3024

11. (8 − 3)! = 5! = 5 · 4 · 3 · 2 · 1 = 120

13.10!7!3!

=10 · 9 · 8 · 7!7!3 · 2 · 1 =

10 · 3 · 3 · 4 · 23 · 2 · 1 =

10 · 3 · 4 = 120

15. Using formula (2), we have

8P0 =8!

(8 − 0)!=

8!8!

= 1.

17. Using a calculator, we find52P4 = 6, 497, 400

19. Using formula (1), we have nP3 = n(n− 1)(n− 2).

Using formula (2), we have

nP3 =n!

(n− 3)!=n(n− 1)(n− 2)(n− 3)!

(n− 3)!=

n(n− 1)(n− 2).

21. Using formula (1), we have nP1 = n.

Using formula (2), we have

nP1 =n!

(n− 1)!=n(n− 1)!(n− 1)!

= n.

23. 6P6 = 6! = 720

25. 9P9 = 9! = 362, 880

27. 9P4 = 9 · 8 · 7 · 6 = 3024

29. Without repetition: 5P5 = 5! = 120

With repetition: 55 = 3125

31. There are 5P5 choices for the order of the rock numbersand 4P4 choices for the order of the speeches, so we have5P5 ·4 P4 = 5!4! = 2880.

33. The first number can be any of the eight digits other than0 and 1. The remaining 6 numbers can each be any of theten digits 0 through 9. We have

8 · 106 = 8, 000, 000

Accordingly, there can be 8,000,000 telephone numberswithin a given area code before the area needs to be splitwith a new area code.

Copyright © 2013 Pearson Education, Inc.

Page 321: JUDITH A. PENNA -  · PDF fileSTUDENT’S SOLUTIONS MANUAL JUDITH A. PENNA Indiana University Purdue University Indianapolis COLLEGE ALGEBRA: GRAPHS AND MODELS FIFTH EDITION

318 Chapter 8: Sequences, Series, and Combinatorics

35. a2b3c4 = a · a · b · b · b · c · c · c · cThere are 2 a’s, 3 b’s, and 4 c’s, for a total of 9. We have

9!2! · 3! · 4!

=9 · 8 · 7 · 6 · 5 · 4!2 · 1 · 3 · 2 · 1 · 4!

=9 · 8 · 7 · 6 · 5

2 · 3 · 2 = 1260.

37. a) 6P5 = 6 · 5 · 4 · 3 · 2 = 720

b) 65 = 7776

c) The first letter can only beD. The other four lettersare chosen from A, B, C, E, F without repetition.We have

1 ·5 P4 = 1 · 5 · 4 · 3 · 2 = 120.

d) The first letter can only be D. The second lettercan only be E. The other three letters are chosenfrom A, B, C, F without repetition. We have

1 · 1 ·4 P3 = 1 · 1 · 4 · 3 · 2 = 24.

39. a) Since repetition is allowed, each of the 5 digits canbe chosen in 10 ways. The number of zip-codes pos-sible is 10 · 10 · 10 · 10 · 10, or 100,000.

b) Since there are 100,000 possible zip-codes, therecould be 100,000 post offices.

41. a) Since repetition is allowed, each digit can be chosenin 10 ways. There can be10 · 10 · 10 · 10 · 10 · 10 · 10 · 10 · 10, or1,000,000,000 social security numbers.

b) Since more than 303 million social security numbersare possible, each person can have a social securitynumber.

43. x2 + x− 6 = 0

(x+ 3)(x− 2) = 0

x+ 3 = 0 or x− 2 = 0

x = −3 or x = 2

The solutions are −3 and 2.

45. f(x) = x3 − 4x2 − 7x+ 10

We use synthetic division to find one factor of the polyno-mial. We try x− 1.

1∣∣ 1 −4 −7 10

1 −3 −101 −3 −10 0

x3 − 4x2 − 7x+ 10 = 0

(x− 1)(x2 − 3x− 10) = 0

(x− 1)(x− 5)(x+ 2) = 0

x− 1 = 0 or x− 5 = 0 or x+ 2 = 0

x = 1 or x = 5 or x = −2

The solutions are −2, 1, and 5.

47. nP4 = 8 ·n−1 P3

n!(n− 4)!

= 8 · (n− 1)!(n− 1 − 3)!

n!(n− 4)!

= 8 · (n− 1)!(n− 4)!

n! = 8 · (n− 1)! Multiplying by (n−4)!

n(n− 1)! = 8 · (n− 1)!

n = 8 Dividing by (n− 1)!

49. nP4 = 8 ·n P3

n!(n− 4)!

= 8 · n!(n− 3)!

(n− 3)! = 8(n− 4)! Multiplying by(n− 4)!(n− 3)!

n!(n− 3)(n− 4)! = 8(n− 4)!

n− 3 = 8 Dividing by (n− 4)!

n = 11

51. There is one losing team per game. In order to leave onetournament winner there must be n− 1 losers produced inn− 1 games.

Exercise Set 8.6

1. 13C2 =13!

2!(13 − 2)!

=13!

2!11!=

13 · 12 · 11!2 · 1 · 11!

=13 · 122 · 1 =

13 · 6 · 22 · 1

= 78

3.( 13

11

)=

13!11!(13 − 11)!

=13!

11!2!= 78 (See Exercise 1.)

5.( 7

1

)=

7!1!(7 − 1)!

=7!

1!6!=

7 · 6!1 · 6!

= 7

7. 5P3

3!=

5 · 4 · 33!

=5 · 4 · 33 · 2 · 1 =

5 · 2 · 2 · 33 · 2 · 1

= 5 · 2 = 10

9.( 6

0

)=

6!0!(6 − 0)!

=6!

0!6!=

6!6! · 1

= 1

Copyright © 2013 Pearson Education, Inc.

Page 322: JUDITH A. PENNA -  · PDF fileSTUDENT’S SOLUTIONS MANUAL JUDITH A. PENNA Indiana University Purdue University Indianapolis COLLEGE ALGEBRA: GRAPHS AND MODELS FIFTH EDITION

Exercise Set 8.6 319

11.( 6

2

)=

6 · 52 · 1 = 15

13.(nr

)=(

nn− r

), so

( 70

)+( 7

1

)+( 7

2

)+( 7

3

)+( 7

4

)+

( 75

)+( 7

6

)+( 7

7

)

= 2[( 7

0

)+( 7

1

)+( 7

2

)+( 7

3

)]

= 2[

7!7!0!

+7!

6!1!+

7!5!2!

+7!

4!3!

]= 2(1 + 7 + 21 + 35) = 2 · 64 = 128

15. We will use form (1).

52C4 =52!

4!(52 − 4)!

=52 · 51 · 50 · 49 · 48!

4 · 3 · 2 · 1 · 48!

=52 · 51 · 50 · 49

4 · 3 · 2 · 1= 270, 725

17. We will use form (2).( 2711

)

=27 · 26 · 25 · 24 · 23 · 22 · 21 · 20 · 19 · 18 · 17

11 · 10 · 9 · 8 · 7 · 6 · 5 · 4 · 3 · 2 · 1= 13, 037, 895

19.( n

1

)=

n!1!(n− 1)!

=n(n− 1)!1!(n− 1)!

= n

21.( mm

)=

m!m!(m−m)!

=m!m!0!

= 1

23. 23C4 =23!

4!(23 − 4)!

=23!

4!19!=

23 · 22 · 21 · 20 · 19!4 · 3 · 2 · 1 · 19!

=23 · 22 · 21 · 20

4 · 3 · 2 · 1 =23 · 2 · 11 · 3 · 7 · 4 · 5

4 · 3 · 2 · 1= 8855

25. 13C10 =13!

10!(13 − 10)!

=13!

10!3!=

13 · 12 · 11 · 10!10! · 3 · 2 · 1

=13 · 12 · 11

3 · 2 · 1 =13 · 3 · 2 · 2 · 11

3 · 2 · 1= 286

27. 10C7 ·5 C3 =(

107

)·(

53

)Using the fundamen-tal counting principle

=10!

7!(10 − 7)!· 5!3!(5 − 3)!

=10 · 9 · 8 · 7!

7! · 3!· 5 · 4 · 3!

3! · 2!

=10 · 9 · 83 · 2 · 1 · 5 · 4

2 · 1 = 120 · 10 = 1200

29. 52C5 = 2, 598, 960

31. a) 31P2 = 930

b) 31 · 31 = 961

c) 31C2 = 465

33. 2x2 − x = 3

2x2 − x− 3 = 0

(2x− 3)(x+ 1) = 0

2x− 3 = 0 or x+ 1 = 0

2x = 3 or x = −1

x =32or x = −1

The solutions are32

and −1.

35. x3 + 3x2 − 10x = 24

x3 + 3x2 − 10x− 24 = 0

We use synthetic division to find one factor of the polyno-mial on the left side of the equation. We try x− 3.

3∣∣ 1 3 −10 −24

3 18 241 6 8 0

Now we have:(x− 3)(x2 + 6x+ 8) = 0

(x− 3)(x+ 2)(x+ 4) = 0

x− 3 = 0 or x+ 2 = 0 or x+ 4 = 0

x = 3 or x = −2 or x = −4

The solutions are −4, −2, and 3.

37. There are 13 diamonds, and we choose 5. We have 13C5 =1287.

39. Playing once: nC2

Playing twice: 2 ·n C2

Copyright © 2013 Pearson Education, Inc.

Page 323: JUDITH A. PENNA -  · PDF fileSTUDENT’S SOLUTIONS MANUAL JUDITH A. PENNA Indiana University Purdue University Indianapolis COLLEGE ALGEBRA: GRAPHS AND MODELS FIFTH EDITION

320 Chapter 8: Sequences, Series, and Combinatorics

41.(

nn− 2

)= 6

n!(n− (n− 2))!(n− 2)!

= 6

n!2!(n− 2)!

= 6

n(n− 1)(n− 2)!2 · 1 · (n− 2)!

= 6

n(n− 1)2

= 6

n(n− 1) = 12

n2 − n = 12

n2 − n− 12 = 0

(n− 4)(n+ 3) = 0n = 4 or n = −3

Only 4 checks. The solution is 4.

Exercise Set 8.7

1. Expand: (x+ 5)4.

We have a = x, b = 5, and n = 4.

Pascal’s triangle method: Use the fifth row of Pascal’striangle.

1 4 6 4 1

(x+ 5)4

= 1 · x4 + 4 · x3 · 5 + 6 · x2 · 52+

4 · x · 53 + 1 · 54

= x4 + 20x3 + 150x2 + 500x+ 625

Combination notation method:(x+ 5)4

=( 4

0

)x4 +

( 41

)x3 · 5 +

( 42

)x2 · 52+

( 43

)x · 53 +

( 44

)54

=4!

0!4!x4 +

4!1!3!

x3 · 5 +4!

2!2!x2 · 52+

4!3!1!

x · 53 +4!

4!0!54

= x4 + 20x3 + 150x2 + 500x+ 625

3. Expand: (x− 3)5.

We have a = x, b = −3, and n = 5.

Pascal’s triangle method: Use the sixth row of Pascal’striangle.

1 5 10 10 5 1

(x− 3)5

= 1 · x5 + 5x4(−3) + 10x3(−3)2 + 10x2(−3)3+

5x(−3)4 + 1 · (−3)5

= x5 − 15x4 + 90x3 − 270x2 + 405x− 243

Combination notation method:(x− 3)5

=( 5

0

)x5 +

( 51

)x4(−3) +

( 52

)x3(−3)2+

( 53

)x2(−3)3 +

( 54

)x(−3)4 +

( 55

)(−3)5

=5!

0!5!x5 +

5!1!4!

x4(−3) +5!

2!3!x3(9)+

5!3!2!

x2(−27) +5!

4!1!x(81) +

5!5!0!

(−243)

= x5 − 15x4 + 90x3 − 270x2 + 405x− 243

5. Expand: (x− y)5.We have a = x, b = −y, and n = 5.

Pascal’s triangle method: We use the sixth row of Pascal’striangle.

1 5 10 10 5 1(x− y)5

= 1 · x5 + 5x4(−y) + 10x3(−y)2 + 10x2(−y)3+5x(−y)4 + 1 · (−y)5

= x5 − 5x4y + 10x3y2 − 10x2y3 + 5xy4 − y5

Combination notation method:(x− y)5

=(

50

)x5 +

(51

)x4(−y) +

(52

)x3(−y)2+(

53

)x2(−y)3 +

(54

)x(−y)4 +

(55

)(−y)5

=5!

0!5!x5 +

5!1!4!

x4(−y) +5!

2!3!x3(y2)+

5!3!2!

x2(−y3) +5!

4!1!x(y4) +

5!5!0!

(−y5)

= x5 − 5x4y + 10x3y2 − 10x2y3 + 5xy4 − y5

7. Expand: (5x+ 4y)6.

We have a = 5x, b = 4y, and n = 6.

Pascal’s triangle method: Use the seventh row of Pascal’striangle.

1 6 15 20 15 6 1

(5x+ 4y)6

= 1 · (5x)6 + 6 · (5x)5(4y) + 15(5x)4(4y)2+

20(5x)3(4y)3 + 15(5x)2(4y)4 + 6(5x)(4y)5+

1 · (4y)6= 15, 625x6 + 75, 000x5y + 150, 000x4y2+

160, 000x3y3+96, 000x2y4+30, 720xy5+4096y6

Combination notation method:(5x+ 4y)6

=( 6

0

)(5x)6 +

( 61

)(5x)5(4y)+

( 62

)(5x)4(4y)2 +

( 63

)(5x)3(4y)3+

( 64

)(5x)2(4y)4+

( 65

)(5x)(4y)5+

( 66

)(4y)6

Copyright © 2013 Pearson Education, Inc.

Page 324: JUDITH A. PENNA -  · PDF fileSTUDENT’S SOLUTIONS MANUAL JUDITH A. PENNA Indiana University Purdue University Indianapolis COLLEGE ALGEBRA: GRAPHS AND MODELS FIFTH EDITION

Exercise Set 8.7 321

=6!

0!6!(15, 625x6) +

6!1!5!

(3125x5)(4y)+

6!2!4!

(625x4)(16y2) +6!

3!3!(125x3)(64y3)+

6!4!2!

(25x2)(256y4) +6!

5!1!(5x)(1024y5)+

6!6!0!

(4096y6)

= 15, 625x6 + 75, 000x5y + 150, 000x4y2+

160, 000x3y3 + 96, 000x2y4 + 30, 720xy5+

4096y6

9. Expand:(

2t+1t

)7

.

We have a = 2t, b =1t, and n = 7.

Pascal’s triangle method: Use the eighth row of Pascal’striangle.

1 7 21 35 35 21 7 1(2t+

1t

)7

= 1 · (2t)7 + 7(2t)6(

1t

)+ 21(2t)5

(1t

)2

+

35(2t)4(

1t

)3

+ 35(2t)3(

1t

)4

+ 21(2t)2(

1t

)5

+

7(2t)(

1t

)6

+ 1 ·(

1t

)7

= 128t7 + 7 · 64t6 · 1t

+ 21 · 32t5 · 1t2

+

35 · 16t4 · 1t3

+ 35 · 8t3 · 1t4

+ 21 · 4t2 · 1t5

+

7 · 2t · 1t6

+1t7

= 128t7 + 448t5 + 672t3 + 560t+ 280t−1+

84t−3 + 14t−5 + t−7

Combination notation method:(2t+

1t

)7

=( 7

0

)(2t)7 +

( 71

)(2t)6

(1t

)+

( 72

)(2t)5

(1t

)2

+( 7

3

)(2t)4

(1t

)3

+

( 74

)(2t)3

(1t

)4

+( 7

5

)(2t)2

(1t

)5

+

( 76

)(2t)

(1t

)6

+( 7

7

)(1t

)7

=7!

0!7!(128t7)+

7!1!6!

(64t6)(

1t

)+

7!2!5!

(32t5)(

1t2

)+

7!3!4!

(16t4)(

1t3

)+

7!4!3!

(8t3)(

1t4

)+

7!5!2!

(4t2)(

1t5

)+

7!6!1!

(2t)(

1t6

)+

7!7!0!

(1t7

)= 128t7 + 448t5 + 672t3 + 560t+ 280t−1+

84t−3 + 14t−5 + t−7

11. Expand: (x2 − 1)5.

We have a = x2, b = −1, and n = 5.

Pascal’s triangle method: Use the sixth row of Pascal’striangle.

1 5 10 10 5 1

(x2 − 1)5

= 1 · (x2)5 + 5(x2)4(−1) + 10(x2)3(−1)2+

10(x2)2(−1)3 + 5(x2)(−1)4 + 1 · (−1)5

= x10 − 5x8 + 10x6 − 10x4 + 5x2 − 1

Combination notation method:(x2 − 1)5

=( 5

0

)(x2)5 +

( 51

)(x2)4(−1)+

( 52

)(x2)3(−1)2 +

( 53

)(x2)2(−1)3+

( 54

)(x2)(−1)4 +

( 55

)(−1)5

=5!

0!5!(x10) +

5!1!4!

(x8)(−1) +5!

2!3!(x6)(1)+

5!3!2!

(x4)(−1) +5!

4!1!(x2)(1) +

5!5!0!

(−1)

= x10 − 5x8 + 10x6 − 10x4 + 5x2 − 1

13. Expand: (√

5 + t)6.

We have a =√

5, b = t, and n = 6.

Pascal’s triangle method: We use the seventh row of Pas-cal’s triangle:

1 6 15 20 15 6 1(√

5 + t)6 = 1 · (√5)6 + 6(√

5)5(t)+

15(√

5)4(t2) + 20(√

5)3(t3)+

15(√

5)2(t4) + 6√

5t5 + 1 · t6= 125 + 150

√5 t+ 375t2 + 100

√5 t3+

75t4 + 6√

5 t5 + t6

Combination notation method:

(√

5 + t)6 =(

60

)(√

5)6 +(

61

)(√

5)5(t)+(62

)(√

5)4(t2) +(

63

)(√

5)3(t3)+(64

)(√

5)2(t4) +(

65

)(√

5)(t5)+

Copyright © 2013 Pearson Education, Inc.

Page 325: JUDITH A. PENNA -  · PDF fileSTUDENT’S SOLUTIONS MANUAL JUDITH A. PENNA Indiana University Purdue University Indianapolis COLLEGE ALGEBRA: GRAPHS AND MODELS FIFTH EDITION

322 Chapter 8: Sequences, Series, and Combinatorics

(66

)(t6)

=6!

0!6!(125) +

6!1!5!

(25√

5)t+6!

2!4!(25)(t2)+

6!3!3!

(5√

5)(t3) +6!

4!2!(5)(t4)+

6!5!1!

(√

5)(t5) +6!

6!0!(t6)

= 125 + 150√

5 t+ 375t2 + 100√

5 t3+

75t4 + 6√

5 t5 + t6

15. Expand:(a− 2

a

)9

.

We have a = a, b = −2a, and n = 9.

Pascal’s triangle method: Use the tenth row of Pascal’striangle.

1 9 36 84 126 126 84 36 9 1(a− 2

a

)9

= 1 · a9 + 9a8(− 2a

)+ 36a7

(− 2a

)2

+

84a6(− 2a

)3

+ 126a5(− 2a

)4

+

126a4(− 2a

)5

+ 84a3(− 2a

)6

+

36a2(− 2a

)7

+9a(− 2a

)8

+1 ·(− 2a

)9

= a9 − 18a7 + 144a5 − 672a3 + 2016a−4032a−1 + 5376a−3 − 4608a−5+

2304a−7 − 512a−9

Combination notation method:(a− 2

a

)9

=(

90

)a9 +

(91

)a8(− 2a

)+(

92

)a7(− 2a

)2

+(93

)a6(− 2a

)3

+(

94

)a5(− 2a

)4

+(95

)a4(− 2a

)5

+(

96

)a3(− 2a

)6

+(97

)a2(− 2a

)7

+(

98

)a(− 2a

)8

+(99

)(− 2a

)9

=9!

9!0!a9 +

9!8!1!

a8(− 2a

)+

9!7!2!

a7( 4a2

)+

9!6!3!

a6(− 8a3

)+

9!5!4!

a5(16a4

)+

9!4!5!

a4(− 32a5

)+

9!3!6!

a3(64a6

)+

9!2!7!

a2(− 128a7

)+

9!1!8!

a(256a8

)+

9!0!9!

(− 512a9

)

= a9 − 9(2a7) + 36(4a5) − 84(8a3) + 126(16a)−126(32a−1) + 84(64a−3) − 36(128a−5)+

9(256a−7) − 512a−9

= a9 − 18a7 + 144a5 − 672a3 + 2016a− 4032a−1+

5376a−3 − 4608a−5 + 2304a−7 − 512a−9

17. (√

2 + 1)6 − (√

2 − 1)6

First, expand (√

2 + 1)6.

(√

2+1)6 =( 6

0

)(√

2)6 +( 6

1

)(√

2)5(1)+

( 62

)(√

2)4(1)2 +( 6

3

)(√

2)3(1)3+

( 64

)(√

2)2(1)4 +( 6

5

)(√

2)(1)5+

( 66

)(1)6

=6!

6!0!· 8 +

6!5!1!

· 4√

2 +6!

4!2!· 4+

6!3!3!

· 2√

2 +6!

2!4!· 2 +

6!1!5!

·√

2+6!

0!6!

= 8 + 24√

2 + 60 + 40√

2 + 30 + 6√

2 + 1

= 99 + 70√

2

Next, expand (√

2 − 1)6.

(√

2 − 1)6

=( 6

0

)(√

2)6 +( 6

1

)(√

2)5(−1)+

( 62

)(√

2)4(−1)2 +( 6

3

)(√

2)3(−1)3+

( 64

)(√

2)2(−1)4 +( 6

5

)(√

2)(−1)5+

( 66

)(−1)6

=6!

6!0!· 8 − 6!

5!1!· 4

√2 +

6!4!2!

· 4 − 6!3!3!

· 2√

2+

6!2!4!

· 2 − 6!1!5!

·√

2+6!

0!6!

= 8 − 24√

2 + 60 − 40√

2 + 30 − 6√

2 + 1

= 99 − 70√

2

(√

2 + 1)6 − (√

2 − 1)6

= (99 + 70√

2) − (99 − 70√

2)

= 99 + 70√

2 − 99 + 70√

2

= 140√

2

19. Expand: (x−2 + x2)4.

We have a = x−2, b = x2, and n = 4.

Pascal’s triangle method: Use the fifth row of Pascal’striangle.

1 4 6 4 1.

Copyright © 2013 Pearson Education, Inc.

Page 326: JUDITH A. PENNA -  · PDF fileSTUDENT’S SOLUTIONS MANUAL JUDITH A. PENNA Indiana University Purdue University Indianapolis COLLEGE ALGEBRA: GRAPHS AND MODELS FIFTH EDITION

Exercise Set 8.7 323

(x−2 + x2)4

= 1 · (x−2)4 + 4(x−2)3(x2) + 6(x−2)2(x2)2+

4(x−2)(x2)3 + 1 · (x2)4

= x−8 + 4x−4 + 6 + 4x4 + x8

Combination notation method:(x−2 + x2)4

=(

40

)(x−2)4 +

(41

)(x−2)3(x2)+(

42

)(x−2)2(x2)2 +

(43

)(x−2)(x2)3+(

44

)(x2)4

=4!

4!0!(x−8) +

4!3!1!

(x−6)(x2) +4!

2!2!(x−4)(x4)+

4!1!3!

(x−2)(x6) +4!

0!4!(x8)

= x−8 + 4x−4 + 6 + 4x4 + x8

21. Find the 3rd term of (a+ b)7.

First, we note that 3 = 2 + 1, a = a, b = b, and n = 7.Then the 3rd term of the expansion of (a+ b)7 is(

72

)a7−2b2, or

7!2!5!

a5b2, or 21a5b2.

23. Find the 6th term of (x− y)10.First, we note that 6 = 5 + 1, a = x, b = −y, and n = 10.Then the 6th term of the expansion of (x− y)10 is( 10

5

)x5(−y)5, or −252x5y5.

25. Find the 12th term of (a− 2)14.

First, we note that 12 = 11+1, a = a, b = −2, and n = 14.Then the 12th term of the expansion of (a− 2)14 is(

1411

)a14−11 · (−2)11 =

14!3!11!

a3(−2048)

= 364a3(−2048)

= −745, 472a3

27. Find the 5th term of (2x3 −√y)8.

First, we note that 5 = 4 + 1, a = 2x3, b = −√y, and

n = 8. Then the 5th term of the expansion of (2x3 −√y)8

is (84

)(2x3)8−4(−√

y)4

=8!

4!4!(2x3)4(−√

y)4

= 70(16x12)(y2)

= 1120x12y2

29. The expansion of (2u − 3v2)10 has 11 terms so the 6thterm is the middle term. Note that 6 = 5 + 1, a = 2u,b = −3v2, and n = 10. Then the 6th term of the expansionof (2u− 3v2)10 is(

105

)(2u)10−5(−3v2)5

=10!5!5!

(2u)5(−3v2)5

= 252(32u5)(−243v10)

= −1, 959, 552u5v10

31. The number of subsets is 27, or 128

33. The number of subsets is 224, or 16,777,216.

35. The term of highest degree of (x5 + 3)4 is the first term,or( 4

0

)(x5)4−030 =

4!4!0!

x20 = x20.

Therefore, the degree of (x5 + 3)4 is 20.

37. We use combination notation. Note that

a = 3, b = i, and n = 5.

(3 + i)5

=( 5

0

)(35) +

( 51

)(34)(i) +

( 52

)(33)(i2)+

( 53

)(32)(i3) +

( 54

)(3)(i4) +

( 55

)(i5)

=5!

0!5!(243) +

5!1!4!

(81)(i) +5!

2!3!(27)(−1)+

5!3!2!

(9)(−i) +5!

4!1!(3)(1) +

5!5!0!

(i)

= 243 + 405i− 270 − 90i+ 15 + i

= −12 + 316i

39. We use combination notation. Note that

a =√

2, b = −i, and n = 4.

(√

2−i)4 =( 4

0

)(√

2)4 +( 4

1

)(√

2)3(−i)+( 4

2

)(√

2)2(−i)2 +( 4

3

)(√

2)(−i)3+( 4

4

)(−i)4

=4!

0!4!(4) +

4!1!3!

(2√

2)(−i)+4!

2!2!(2)(−1)+

4!3!1!

(√

2)(i)+

4!4!0!

(1)

= 4 − 8√

2i− 12 + 4√

2i+ 1

= −7 − 4√

2i

Copyright © 2013 Pearson Education, Inc.

Page 327: JUDITH A. PENNA -  · PDF fileSTUDENT’S SOLUTIONS MANUAL JUDITH A. PENNA Indiana University Purdue University Indianapolis COLLEGE ALGEBRA: GRAPHS AND MODELS FIFTH EDITION

324 Chapter 8: Sequences, Series, and Combinatorics

41. (a− b)n =(n0

)an(−b)0 +

(n1

)an−1(−b)1+

(n2

)an−2(−b)2+· · ·+

(nn−1

)a1(−b)n−1 +

(nn

)a0(−b)n

=(n0

)(−1)0anb0+

(n1

)(−1)1an−1b1+

(n2

)(−1)2an−2b2+· · ·+

(nn−1

)(−1)n−1a1bn−1+

(nn

)(−1)na0bn

=n∑

k=0

( nk

)(−1)kan−kbk

43. (x+ h)n − xnh

=

( n0

)xn+

( n1

)xn−1h+· · ·+

( nn

)hn−xn

h

=( n

1

)xn−1 +

( n2

)xn−2h+ · · · +

( nn

)hn−1

=n∑

k=1

( nk

)xn−khk−1

45. (fg)(x) = f(x)g(x) = (x2 + 1)(2x− 3) =

2x3 − 3x2 + 2x− 3

47. (g ◦ f)(x) = g(f(x)) = g(x2 + 1) = 2(x2 + 1) − 3 =

2x2 + 2 − 3 = 2x2 − 1

49.4∑

k=0

( 4k

)(−1)kx4−k6k =

4∑k=0

( 4k

)x4−k(−6)k, so

the left side of the equation is sigma notation for (x− 6)4.We have:(x− 6)4 = 81

x− 6 = ±3 Taking the 4th root on bothsides

x− 6 = 3 or x− 6 = −3

x = 9 or x = 3

The solutions are 9 and 3.

If we also observe that (3i)4 = 81, we also find the imagi-nary solutions 6 ± 3i.

51. The (k + 1)st term of(

3√x− 1√

x

)7

is(7k

)( 3√x)7−k

(− 1√

x

)k. The term containing

1x1/6

is the

term in which the sum of the exponents is −1/6. That is,(13

)(7 − k) +

(− 1

2

)(k) = −1

673− k

3− k

2= −1

6

−5k6

= −156

k = 3

Find the (3 + 1)st, or 4th term.(73

)( 3√x)4(− 1√

x

)3

=7!

4!3!(x4/3)(−x−3/2) =

−35x−1/6, or − 35x1/6

.

53. 100C0 +100C1 + · · ·+100C100 is the total number of subsetsof a set with 100 members, or 2100.

55.23∑k=0

(23k

)(loga x)

23−k(loga t)k =

(loga x+ loga t)23 = [loga(xt)]23

57. See the answer section in the text.

Exercise Set 8.8

1. a) We use Principle P .

For 1: P =18100

, or 0.18

For 2: P =24100

, or 0.24

For 3: P =23100

, or 0.23

For 4: P =23100

, or 0.23

For 5: P =12100

, or 0.12

b) Opinions may vary, but it seems that people tendnot to select the first or last numbers.

3. The company can expect 78% of the 15,000 pieces of ad-vertising to be opened and read. We have:

78%(15, 000) = 0.78(15, 000) = 11, 700 pieces.

5. a) Since there are 14 equally likely ways of selecting amarble from a bag containing 4 red marbles and 10green marbles, we have, by Principle P ,

P (selecting a red marble) =414

=27.

b) Since there are 14 equally likely ways of selecting amarble from a bag containing 4 red marbles and 10green marbles, we have, by Principle P ,

P (selecting a green marble) =1014

=57.

c) Since there are 14 equally likely ways of selecting amarble from a bag containing 4 red marbles and 10green marbles, we have, by Principle P ,

P (selecting a purple marble) =014

= 0.

Copyright © 2013 Pearson Education, Inc.

Page 328: JUDITH A. PENNA -  · PDF fileSTUDENT’S SOLUTIONS MANUAL JUDITH A. PENNA Indiana University Purdue University Indianapolis COLLEGE ALGEBRA: GRAPHS AND MODELS FIFTH EDITION

Exercise Set 8.8 325

d) Since there are 14 equally likely ways of selecting amarble from a bag containing 4 red marbles and 10green marbles, we have, by Principle P ,

P (selecting a red or a green marble) =4 + 10

14= 1.

7. There are 6 possible outcomes. There are 3 numbers less

than 4, so the probability is36, or

12.

9. a) There are 4 queens, so the probability is452

, or113

.

b) There are 4 aces and 4 tens, so the probability is4 + 452

, or852

, or213

.

c) There are 13 hearts, so the probability is1352

, or14.

d) There are two black 6’s, so the probability is252

, or126

.

11. The number of ways of drawing 3 cards from a deck of 52is 52C3. The number of ways of drawing 3 aces is 4C3. Theprobability is

4C3

52C3=

422, 100

=1

5525.

13. The total number of people on the sales force is 10+10, or20. The number of ways to choose 4 people from a groupof 20 is 20C4. The number of ways of selecting 2 peoplefrom a group of 10 is 10C2. This is done for both the menand the women.

P (choosing 2 men and 2 women) = 10C2 ·10 C2

20C4=

45 · 454845

=135323

15. The number of ways of selecting 5 cards from a deck of 52cards is 52C5. Three sevens can be selected in 4C3 waysand 2 kings in 4C2 ways.

P (drawing 3 sevens and 2 kings) = 4C3 ·4 C2

52C5, or

1108, 290

.

17. The number of ways of selecting 5 cards from a deck of52 cards is 52C5. Since 13 of the cards are spades, then 5spades can be drawn in 13C5 ways

P (drawing 5 spades) = 13C5

52C5=

12872, 598, 960

=

3366, 640

19. a) HHH, HHT, HTH, HTT, THH, THT, TTH, TTT

b) Three of the 8 outcomes have exactly one head.

Thus, P (exactly one head) =38.

c) Seven of the 8 outcomes have exactly 0, 1, or 2

heads. Thus, P (at most two heads) =78.

d) Seven of the 8 outcomes have 1, 2, or 3 heads. Thus,

P (at least one head) =78.

e) Three of the 8 outcomes have exactly two tails.

Thus, P (exactly two tails) =38.

21. The roulette wheel contains 38 equally likely slots. Eigh-teen of the 38 slots are colored black. Thus, by PrincipleP ,

P (the ball falls in a black slot) =1838

=919.

23. The roulette wheel contains 38 equally likely slots. Only 1slot is numbered 0. Then, by Principle P ,

P (the ball falls in the 0 slot) =138.

25. The roulette wheel contains 38 equally likely slots. Thirty-six of the slots are colored red or black. Then, by PrincipleP ,

P (the ball falls in a red or a black slot) =3638

=1819.

27. The roulette wheel contains 38 equally likely slots. Eigh-teen of the slots are odd-numbered. Then, by PrincipleP ,

P (the ball falls in a an odd-numbered slot) =1838

=919.

29. zero

31. function; domain; range; domain; range

33. combination

35. factor

37. a) There are(

132

)ways to select 2 denominations

from the 13 denominations. Then in each denomina-

tion there are(

42

)ways to choose 2 of the 4 cards.

Finally there are(

441

)ways to choose the fifth card

from the 11 remaining denominations (4 · 11, or 44cards). Thus the number of two pairs hands is(

132

)·(

42

)·(

42

)·(

441

), or 123,552.

b)123, 552

52C5=

123, 5522, 598, 960

≈ 0.0475

39. a) There are 13 ways to select a denomination and then(43

)ways to choose 3 of the 4 cards in that de-

nomination. Now there are(

482

)ways to choose 2

cards from the 12 remaining denominations (4 · 12,or 48 cards). But these combinations include the3744 hands in a full house like Q-Q-Q-4-4 (Exercise38), so these must be subtracted. Thus the number

of three of a kind hands is 13 ·(

43

)·(

482

)− 3744,

or 54,912.

Copyright © 2013 Pearson Education, Inc.

Page 329: JUDITH A. PENNA -  · PDF fileSTUDENT’S SOLUTIONS MANUAL JUDITH A. PENNA Indiana University Purdue University Indianapolis COLLEGE ALGEBRA: GRAPHS AND MODELS FIFTH EDITION

326 Chapter 8: Sequences, Series, and Combinatorics

b)54, 91252C5

=54, 912

2, 598, 960≈ 0.0211

Chapter 8 Review Exercises

1. The statement is true. See page 628 in the text.

3. The statement is true. See page 663 in the text.

5. an = (−1)n(

n2

n4 + 1

)

a1 = (−1)1(

12

14 + 1

)= −1

2

a2 = (−1)2(

22

24 + 1

)=

417

a3 = (−1)3(

32

34 + 1

)= − 9

82

a4 = (−1)4(

42

44 + 1

)=

16257

a11 = (−1)11(

112

114 + 1

)= − 121

14, 642

a23 = (−1)23(

232

234 + 1

)= − 529

279, 842

7.4∑

k=1

(−1)k+13k

3k − 1

=(−1)1+131

31 − 1+

(−1)2+132

32 − 1+

(−1)3+133

33 − 1+

(−1)4+134

34 − 1

=32− 9

8+

2726

− 8180

=4171040

9.7∑

k=1

(k2 − 1)

11. an = a1 + (n− 1)d

a1 = a− b, d = a− (a− b) = b

a6 = a− b+ (6 − 1)b = a− b+ 5b = a+ 4b

13. 1 + 2 + 3+ . . . +199 + 200

Sn =n

2(a1 + an)

S200 =2002

(1 + 200) = 20, 100

15. d = 3, a10 = 23

23 = a1 + (10 − 1)3

23 = a1 + 27

−4 = a1

17. r =12, n = 5, Sn =

312

Sn =a1(1 − rn)

1 − r

312

=a1

(1 −

(12

)5)

1 − 12

312

=a1

(1 − 1

32

)12

312

=

3132a1

12

312

=3132a1 · 2

1312

=3116a1

8 = a1

an = a1rn−1

a5 = 8(

12

)5−1

= 8(

12

)4

= 8 · 116

=12

19. Since |r| =∣∣∣∣0.0027

0.27

∣∣∣∣ = ∣∣0.01∣∣ = 0.01 < 1, the series has a

sum.

S∞ =0.27

1 − 0.01=

0.270.99

=2799

=311

21. 2.43 = 2 + 0.43 + 0.0043 + 0.000043+ . . . .

We will find fraction notation for 0.43 and then add 2.

|r| =∣∣∣∣0.0043

0.43

∣∣∣∣ = |0.01| = 0.01 < 1, so the series has a limit.

S∞ =0.43

1 − 0.01=

0.430.99

=4399

Then 2 +4399

=19899

+4399

=24199

23. Familiarize. The distances the ball drops are given by ageometric sequence:

30,34· 30,

(34

)2

· 30,(

34

)3

· 30,(

34

)4

· 30,(

34

)5

· 30.

Then the total distance the ball drops is the sum of these6 terms. The total rebound distance is this sum less 30 ft,the distance the ball drops initially.

Translate. To find the total distance the ball drops wewill use the formula

Sn =a1(1 − rn)

1 − r with a1 = 30, r =34, and n = 6.

Carry out.

S6 =30(

1 −(

34

)6)

1 − 34

=50, 505

512

Then the total rebound distance is50, 505

12− 30, or

35, 145512

, and the total distance the ball has traveled is

50, 505512

+34, 145

512, or

42, 825256

, or about 167.3 ft.

Copyright © 2013 Pearson Education, Inc.

Page 330: JUDITH A. PENNA -  · PDF fileSTUDENT’S SOLUTIONS MANUAL JUDITH A. PENNA Indiana University Purdue University Indianapolis COLLEGE ALGEBRA: GRAPHS AND MODELS FIFTH EDITION

Chapter 8 Review Exercises 327

Check. We can perform the calculations again.

State. The ball will have traveled about 167.3 ft when ithits the pavement for the sixth time.

25. a), b) Familiarize. We have an arithmetic sequence witha1 = 10, d = 2, and n = 365.

Translate. We want to find an = a1 + (n − 1)d andSn =

n

2(a1 + an) where a1 = 10, d = 2, and n = 365.

Carry out. First we find a365.

a365 = 10 + (365 − 1)(2) = 738/c, or $7.38

Then S365 =3652

(10 + 738) = 136, 510/c or $1365.10.

Check. We can repeat the calculations.

State. a) You will receive $7.38 on the 365th day.

b) The sum of all the gifts is $1365.10.

27. Sn : 1 + 4 + 7 + . . .+ (3n− 2) =n(3n− 1)

2

S1 : 1 =1(3 − 1)

2

Sk : 1 + 4 + 7 + . . .+ (3k − 2) =k(3k − 1)

2Sk+1 : 1 + 4 + 7 + . . .+ (3k − 2) + [3(k + 1) − 2]

= 1 + 4 + 7 + . . .+ (3k − 2) + (3k + 1)

=(k + 1)(3k + 2)

2

1. Basis step:1(3 − 1)

2=

22

= 1 is true.

2. Induction step: Assume Sk. Add 3k + 1 to both sides.

1 + 4 + 7 + . . .+ (3k − 2) + (3k + 1)

=k(3k − 1)

2+ (3k + 1)

=k(3k − 1)

2+

2(3k + 1)2

=3k2 − k + 6k + 2

2

=3k2 + 5k + 2

2

=(k + 1)(3k + 2)

2

29. Sn :(

1 − 12

)(1 − 1

3

). . .

(1 − 1

n

)=

1n

S2 :(

1 − 12

)=

12

Sk :(

1 − 12

)(1 − 1

3

). . .

(1 − 1

k

)=

1k

Sk+1 :(

1− 12

)(1− 1

3

). . .

(1− 1

k

)(1− 1

k+1

)=

1k+1

1. Basis step: S2 is true by substitution.

2. Induction step: Assume Sk. Deduce Sk+1. Starting withthe left side of Sk+1, we have

(1 − 1

2

)(1 − 1

3

). . .

(1 − 1

k

)︸ ︷︷ ︸

(1 − 1

k + 1

)

=1k

·(

1 − 1k + 1

)By Sk

=1k·(k + 1 − 1k + 1

)

=1k· k

k + 1

=1

k + 1. Simplifying

31. 9 · 8 · 7 · 6 = 3024

33. 24 · 23 · 22 = 12, 144

35. 3 · 4 · 3 = 36

37. 28 = 256

39. Expand: (x−√2)5

Pascal’s triangle method: Use the 6th row.1 5 10 10 5 1(x−√

2)5 = x5 + 5x4(−√2) + 10x3(−√

2)2+

10x2(−√2)3 + 5x(−√

2)4 + (−√2)5

= x5 − 5√

2x4 + 20x3 − 20√

2x2 + 20x− 4√

2Combination notation method:

(x−√2)5 =

(50

)x5+

(51

)x4(−√

2)+(

52

)x3(−√

2)2+(53

)x2(−√

2)3+(

54

)x(−√

2)4+(

55

)(−√

2)5

=x5 − 5√

2x4 + 20x3 − 20√

2x2 + 20x− 4√

2

41. Expand:(a+

1a

)8

Pascal’s triangle method: Use the 9th row.1 8 28 56 70 56 28 8 1(a+

1a

)8

= a8 + 8a7

(1a

)+ 28a6

(1a

)2

+ 56a5

(1a

)3

+

70a4

(1a

)4

+ 56a3

(1a

)5

+ 28a2

(1a

)6

+

8a(

1a

)7

+(

1a

)8

= a8 + 8a6 + 28a4 + 56a2 + 70+

56a−2 + 28a−4 + 8a−6 + a−8

Combination notation method:(a+

1a

)8

=(

80

)a8+

(81

)a7

(1a

)+(

82

)a6

(1a

)2

+

(83

)a5

(1a

)3

+(

84

)a4

(1a

)4

+

(85

)a3

(1a

)5

+(

86

)a2

(1a

)6

+

(87

)a

(1a

)7

+(

88

)(1a

)8

= a8 + 8a6 + 28a4 + 56a2 + 70+

56a−2 + 28a−4 + 8a−6 + a−8

Copyright © 2013 Pearson Education, Inc.

Page 331: JUDITH A. PENNA -  · PDF fileSTUDENT’S SOLUTIONS MANUAL JUDITH A. PENNA Indiana University Purdue University Indianapolis COLLEGE ALGEBRA: GRAPHS AND MODELS FIFTH EDITION

0 100

2

328 Chapter 8: Sequences, Series, and Combinatorics

43. Find 4th term of (a+ x)12.(123

)a9x3 = 220a9x3

45. Of 36 possible combinations, we can get a 10 with (4, 6),(6, 4) or (5, 5).

Probability =336

=112

Since we cannot get a 10 on one die, the probability ofgetting a 10 on one die is 0.

47. 4C2 ·4 C1

52C3=

6 · 422, 100

=6

5525

49. a) an = 0.1285179017n+ 2.870691091

b) In 2010, n = 2010 − 1930 = 80.

a80 ≈ 13.2 lb of American cheese

51. There are 3 pairs that total 4: 1 and 3, 2 and 2, 3 and 1.There are 6 · 6, or 36, possible outcomes. Thus, we have336

, or112

. Answer A is correct.

53.ak+1

ak= r1,

bk+1

bk= r2, so

ak+1bk+1

akbk= r1r2, a constant.

55. f(x) = x4 − 4x3 − 4x2 + 16x = x(x3 − 4x2 − 4x+ 16)

We know that 0 is a zero.

Consider x3 − 4x2 − 4x+ 16.

Possibilities for p/q: ±1,±2,±4,±8,±16

−2∣∣ 1 −4 −4 16

−2 12 −161 −6 8 0

f(x) = x(x+ 2)(x2 − 6x+ 8)

= x(x+ 2)(x− 2)(x− 4)The zeros are −2, 0, 2, and 4.

57.10∑k=0

(−1)k(

10k

)(log x)10−k(log y)k

= (log x− log y)10

=(

logx

y

)10

59.(

nn− 1

)= 36

n!(n− 1)![n− (n− 1)]!

= 36

n(n− 1)!(n− 1)!1!

= 36

n = 36

61. Put the following in the form of a paragraph.

First find the number of seconds in a year (365 days):

365 days✦ · 24 hr✧1 day✦ · 60 min✦

1 hr✧· 60 sec

1 min✦ =

31,536,000 sec.

The number of arrangements possible is 15!.

The time is15!

31, 536, 000≈ 41, 466 yr.

63. Choosing k objects from a set of n objects is equivalent tonot choosing the other n− k objects.

65. In expanding (a+ b)n, it would probably be better to usePascal’s triangle when n is relatively small. When n islarge, and many rows of Pascal’s triangle must be com-puted to get to the (n + 1)st row, it would probably bebetter to use combination notation. In addition, combina-tion notation allows us to write a particular term of theexpansion more efficiently than Pascal’s triangle.

Chapter 8 Test

1. an = (−1)n(2n+ 1)

a21 = (−1)21[2(21) + 1]

= −43

2. an =n+ 1n+ 2

a1 =1 + 11 + 2

=23

a2 =2 + 12 + 2

=34

a3 =3 + 13 + 2

=45

a4 =4 + 14 + 2

=56

a5 =5 + 15 + 2

=67

3.4∑

k=1

(k2 + 1) = (12 + 1) + (22 + 1) + (32 + 1) + (42 + 1)

= 2 + 5 + 10 + 7

= 34

4.n Un

1 .66667

2 .75

3 .8

4 .83333

5 .85714

6 .875

7 .88889

8 .9

9 .90909

10 .91667

5.6∑

k=1

4k

6.∞∑k=1

2k

Copyright © 2013 Pearson Education, Inc.

Page 332: JUDITH A. PENNA -  · PDF fileSTUDENT’S SOLUTIONS MANUAL JUDITH A. PENNA Indiana University Purdue University Indianapolis COLLEGE ALGEBRA: GRAPHS AND MODELS FIFTH EDITION

Chapter 8 Test 329

7. an+1 = 2 +1an

a1 = 2 +11

= 2 + 1 = 3

a2 = 2 +13

= 213

a3 = 2 +173

= 2 +37

= 237

a4 = 2 +1177

= 2 +717

= 2717

8. d = 5 − 2 = 3

an = a1 + (n− 1)d

a15 = 2 + (15 − 1)3 = 44

9. a1 = 8, a21 = 108, n = 21an = a1 + (n− 1)d

108 = 8 + (21 − 1)d

100 = 20d

5 = d

Use an = a1 + (n− 1)d again to find a7.a7 = 8 + (7 − 1)(5) = 8 + 30 = 38

10. a1 = 17, d = 13 − 17 = −4, n = 20First find a20:

an = a1 + (n− 1)d

a20 = 17 + (20 − 1)(−4) = 17 − 76 = −59Now find S20:

Sn =n

2(a1 + an)

S20 =202

(17 − 59) = 10(−42) = −420

11.25∑k=1

(2k + 1)

a1 = 2 · 1 + 1 = 3

a25 = 2 · 25 + 1 = 51

Sn =n

2(a1 + an)

S25 =252

(3 + 51) =252

· 54 = 675

12. a1 = 10, r =−510

= −12

an = a1rn−1

a11 = 10(− 1

2

)11−1

=5

512

13. r = 0.2, S4 = 1248

Sn =a1(1 − rn)

1 − r

1248 =a1(1 − 0.24)

1 − 0.2

1248 =0.9984a1

0.8a1 = 1000

14.8∑

k=1

2k

a1 = 21 = 2, r = 2, n = 8

S8 =2(1 − 28)

1 − 2= 510

15. a1 = 18, r =618

=13

Since |r| =13< 1, the series has a sum.

S∞ =18

1 − 13

=1823

= 18 · 32

= 27

16. 0.56 = 0.56 + 0.0056 + 0.000056 + . . .

|r| =∣∣∣∣0.0056

0.56

∣∣∣∣ = |0.01| = 0.01 < 1, so the series has a sum.

S∞ =0.56

1 − 0.01=

0.560.99

=5699

17. a1 = $10, 000

a2 = $10, 000 · 0.80 = $8000

a3 = $8000 · 0.80 = $6400

a4 = $6400 · 0.80 = $5120

a5 = $5120 · 0.80 = $4096

a6 = $4096 · 0.80 = $3276.80

18. We have an arithmetic sequence $8.50, $8.75, $9.00, $9.25,and so on with d = $0.25. Each year there are 12/3, or4 raises, so after 4 years the sequence will have the origi-nal hourly wage plus the 4 · 4, or 16, raises for a total of17 terms. We use the formula an = a1 + (n − 1)d witha1 = $8.50, d = $0.25, and n = 17.

a17 = $8.50+(17−1)($0.25) = $8.50+16($0.25) = $8.50+$4.00 = $12.50

At the end of 4 years Tamika’s hourly wage will be $12.50.

19. We use the formula Sn =a1(1 − rn)

1 − r with a1 = $2500,

r = 1.056, and n = 18.

S18 =2500[1 − (1.056)18]

1 − 1.056= $74, 399.77

20.Sn : 2 + 5 + 8 + . . .+ (3n− 1) =

n(3n+ 1)2

S1 : 2 =1(3 · 1 + 1)

2

Sk : 2 + 5 + 8 + . . .+ (3k − 1) =k(3k + 1)

2Sk+1 : 2 + 5 + 8 + . . .+ (3k − 1) + [3(k + 1) − 1] =

(k + 1)[3(k + 1) + 1]2

1) Basis step:1(3 · 1 + 1)

2=

1 · 42

= 2, so S1 is true.

Copyright © 2013 Pearson Education, Inc.

Page 333: JUDITH A. PENNA -  · PDF fileSTUDENT’S SOLUTIONS MANUAL JUDITH A. PENNA Indiana University Purdue University Indianapolis COLLEGE ALGEBRA: GRAPHS AND MODELS FIFTH EDITION

330 Chapter 8: Sequences, Series, and Combinatorics

2) Induction step:

2 + 5 + 8 + . . .+ (3k − 1)︸ ︷︷ ︸+[3(k + 1) − 1]

=k(3k + 1)

2+ [3k + 3 − 1] By Sk

=3k2

2+k

2+ 3k + 2

=3k2

2+

7k2

+ 2

=3k2 + 7k + 4

2

=(k + 1)(3k + 4)

2

=(k + 1)[3(k + 1) + 1]

2

21. 15P6 =15!

(15 − 6)!= 3, 603, 600

22. 21C10 =21!

10!(21 − 10)!= 352, 716

23.(n4

)=

n!4!(n− 4)!

=n(n− 1)(n− 2)(n− 3)(n− 4)!

4!(n− 4)!

=n(n− 1)(n− 2)(n− 3)

24

24. 6P4 =6!

(6 − 4)!= 360

25. a) 64 = 1296

b) 5P3 =5!

(5 − 3)!= 60

26. 28C4 =28!

4!(28 − 4)!= 20, 475

27. 12C8 ·8 C4 =12!

8!(12 − 8)!· 8!4!(8 − 4)!

= 34, 650

28. Expand: (x+ 1)5.

Pascal’s triangle method: Use the 6th row.

1 5 10 10 5 1(x+1)5 = x5+5x4 ·1+10x3 ·12+10x2 ·13+5x·14+15

= x5 + 5x4 + 10x3 + 10x2 + 5x+ 1

Combination notation method:

(x+ 1)5 =(

50

)x5 +

(51

)x4 · 1 +

(52

)x3 · 12+(

53

)x2 · 13 +

(54

)x · 14 +

(55

)15

= x5 + 5x4 + 10x3 + 10x2 + 5x+ 1

29. Find 5th term of (x− y)7.(74

)x3(−y)4 = 35x3y4

30. 29 = 512

31.8

6 + 8=

814

=47

32. 6C1 ·5 C2 ·4 C5

15C6=

6 · 10 · 45005

=48

1001

33. an = 2n − 2

Only integers n ≥ 1 are inputs.

a1 = 2 · 1 − 2 = 0, a2 = 2 · 2 − 2 = 2, a3 = 2 · 3 − 2 = 4,a4 = 2 · 4 − 2 = 6

Some points on the graph are (1, 0), (2, 2), (3, 4), and (4, 6).Thus the correct answer is B.

34. nP7 = 9 ·n P6

n!(n− 7)!

= 9 · n!(n− 6)!

n!(n− 7)!

· (n− 6)!n!

= 9 · n!(n− 6)!

· (n− 6)!n!

(n− 6)(n− 7)!(n− 7)!

= 9

n− 6 = 9

n = 15

Copyright © 2013 Pearson Education, Inc.


Recommended